You are on page 1of 1135

dermatologyinreview.

com

Prepared by
Dr.Maysaa.m.mahdi
MBChB, FICMS

1
Basic science and s ucture of e skin

1- This wound is in the first phase of healing. What is the best term to describe this initial phase?

A. Proliferative
B. Regenerative
C. Inflammatory
D. Vascularization
E. Remodeling

Correct choice: C. Inflammatory

Explanation: Wound healing can be described with 3 phases: inflammatory, proliferative, and
remodeling. The other options represent different phases or aspects that are critical to wound
healing, but inflammation is the initial stage.

2- Eccrine glands are found on the:

A. Labia majora
B. Labia minora
C. Glans
D. Prepuce
E. Vermilion

Correct choice: A. Labia majora


Explanation: Eccrine glands are present all over the body except on the vermilion of lips, glans,
labia minora, nail beds, and inner prepuce.

2
tr
th
3- This patient has metastatic colon cancer and biopsy was done to rule out cutaneous metastases.
Pathology showed abundant granulation tissue. What is the best description of the cellular
composition of this healing tissue?

A. macrophages, fibroblasts, endothelial cells


B. neutrophils, macrophages, eosinophils
C. keratinocytes, fibroblasts
D. eosinophils, macrophages
E. mast cells, endothelial cells

Correct choice: A. macrophages, fibroblasts, endothelial cells

Explanation: Granulation tissue is composed most correctly of macrophages, fibroblasts, endothelial


cells. Eosinophils, keratinocytes, and mast cells are less prominent in the makeup of granulation
tissue.

4- Which part of the matrix gives rise to the dorsal nail plate?

A. Distal matrix
B. Proximal matrix
C. Ventral matrix
D. Dorsal matrix
E. Lateral matrix

Correct choice: B. Proximal matrix

Explanation: The proximal matrix gives rise to the dorsal nail plate. The distal matrix gives rise to
the ventral nail plate.

3
5- What is the major determinant of sodium chloride concentration of eccrine sweat that is secreted
onto the surface of the skin?
A. Temperature
B. Acetylcholine concentration in nerve endings
C. Sweat rate
D. Dietary salt intake
E. Ph

Correct choice: C. Sweat rate

Explanation: The composition of human eccrine sweat includes inorganic ions (sodium chloride,
bicarbonate, potassium), urea, lactate, ammonia, amino acids, proteases, and other proteins. The
sweat rate is the major determinant of sodium chloride concentration of eccrine sweat that is
secreted onto the surface of the skin. Eccrine sweat is secreted from eccrine glands via merocrine
secretion. The substance secreted is initially isotonic but becomes hypotonic as NaCl reabsorption
occurs in the eccrine duct. The sweat rate determines how much reabsorption occurs, and thus
determines the final concentration of eccrine sweat that reaches the skin surface. Eccrine glands
play an important role in the regulation of body temperature. They are distributed over the entire
body skin, but are highest in density on the palms and soles. Eccrine glands secrete a salty solution,
which reaches the skin pores via an eccrine duct. Eccrine sweat is a sterile electrolyte solution
primarily containing sodium chloride, potassium and bicarbonate, with smaller quantities of various
other components such as glucose and antimicrobial peptides. Sweat rate differs depending on site
on the body, the degree of thermal or physical stress, and between individuals. Under maximal
stimulation, the body can produce 3 liters of eccrine sweat per hour.

6- A mother presents to your pediatric dermatology clinic for further evaluation of her daughter,
who has red gums, episodic hair loss, and rough perifollicular papules on the arms and thighs. She
reports a family history of similar findings. After a thorough assessment of this patient, you
conclude the patient has:

A. Pili annulati
B. Pili trianguli et canaliculi
C. Hereditary mucoepithelial dysplasia
D. Wooly hair
E. None of the above

4
Correct choice: C. Hereditary mucoepithelial dysplasia

Explanation: Defects of the hair shaft include hereditary mucoepithelial dysplasia which is
characterized by red gums, episodic hair loss, and keratosis pilaris. Pili annulati is known as banded
hair. Uncombable hair syndrome results in triangle-shaped hair. Naxos syndrome is characterized by
wooly hair, hyperkeratosis of the palms & soles, and cardiac arrhythmias.

7- Occlusive moisturizers prevent evaporative water loss to the environment by placing an oily
substance on the skin surface through which water cannot penetrate. This replenishes the stratum
corneum by water movement from the viable epidermal and dermal layers. There are many different
classes of chemicals that can function as occlusive moisturizers. What is the most effective
occlusive moisturizer?

A. Glycerin
B. Petrolatum
C. Vegetable oil
D. Dimethicone
E. Ceramide

Correct choice: B. Petrolatum

Explanation: The most effective occlusive moisturizer is petrolatum, since it reduce transepidermal
water loss by 99%. Petrolatum allows barrier repair while permeating throughout the interstices of
the stratum corneum.
a- Glycerin: Humectants are substances that attract moisture; they include: glycerin, honey, sodium
lactate, urea, propylene glycol, sorbitol, pyrrolidone carboxylic acid, gelatin, hyaluronic acid, and
some vitamins and proteins. Some astringents designed for dry skin contain a humectant liquid
moisturizer, such as propylene glycol or glycerin, and skin soothing agents, such as allantoin,
guaiazulene or quaternium-19. Glycerin is not the most effective occlusive moisturizer.

c- Vegetable oil: There are many different classes of chemicals that can function as occlusive
moisturizers, such as vegetable oils (castor oil, corn oil, grape seed oil, soybean oil). Vegetable oil
and wax is a moisturizer type that composes castor oil, corn oil, ozokerite, beeswax, paraffin and
carnauba wax. There are many different classes of chemicals that can function as occlusive
moisturizers such as vegetable oil. Vegetable oil is not the most effective occlusive moisturizer.

5
d - Dimethicone: There are many different classes of chemicals that can function as occlusive
moisturizers such as silicones (dimethicone, cyclomethicone). Dimethicone and ceramides are
moisturizer types that compose water, petrolatum, dimethicone, and ceramics. Dimethicone is not
the most effective occlusive moisturizer.
e - Ceramide: There are many different classes of chemicals that can function as occlusive
moisturizers such as sterols (cholesterol and ceramides). Dimethicone and ceramides are
moisturizer types that compose water, petrolatum,dimethicone, and ceramics. Ceramides are not the
most effective occlusive moisturizers.

8- What structure is indicated by the arrow in this Mohs frozen section?

A. Erector pili muscle


B. Nerve
C. Hair shaft
D. Sebaceous gland
E. Vein

correct choice: A. Erector pili muscle

Explanation: A collection of elongated smooth muscle cells at a 45 degree angle in associated with a
hair follicle is consistent erector pili muscle. Nerves are a bundle of spindled cells and are typically
not seen in the upper dermis, and are not at a 45 degree angle, nor in connection with a hair follicle.
Veins consistent of endothelial cells that would be lining a lumen; the lumen may appear collapsed.
A hair shaft should be surrounded by various layers and sheaths, and is not made up of spindle cells.
Sebaceous glands a a round collection of lobulated cells, not spindled cells, and are seen in
association with a hair follicle.

6
9- The desmosomal connections of the epidermis are dependent on which of the following ions?

A. Iron
B. Zinc
C. Selenium
D. Calcium
E. Sodium

correct choice: D. Calcium

Explanation: The desmosomal connections in the epidermis are calcium dependent. The other
options are not required for these connections. Desmogleins and desmocollins are cadherins
(calcium-dependent adherence molecules)

10- A patient presents with a chronic pruritic papular eruption in both axillae. You suspect she may
have Fox-Fordyce disease, due to an occlusion of apocrine ducts. Which of the following is also
true of apocrine glands?

A. Parasympathetic innervation
B. Not present at birth
C. Stains with S100 and keratin
D. Denervation abolishes response to emotive stimuli
E. Distributed over the entire cutaneous surface

Correct choice: C. Stains with S100 and keratin

Explanation: Apocrine glands are innervated by sympathetic fibers. They are present at birth but do
not enlarge until hormonal stimulation from puberty. Apocrine glands stain positive for S100,
keratin, CEA, and lysozyme. Secretion is in response to emotive stimuli through action of
epinephrine/norepinephrine. Denervation does not abolish this response although apocrine sweating
requires intact nerve supply. Apocrine sweat glands are confined to certain anatomic locations
(axillae, anogenital region, periumbilical region, areolae, nipples, vermilion border of the lip).

7
11- As the shown wound heals and matures, which cell type is most important for wound
contraction?

A. Macrophages
B. Neutrophils
C. Fibroblasts
D. Myofibroblasts
E.Endothelialcells

correct choice: D. Myofibroblasts

Explanation: Myofibroblasts are modified fibroblast with smooth muscle like features.
Myofibroblasts play the important role in the contraction of the wound that occurs during the
proliferation phase. The contraction is considered one of the important events in wound healing
because it results in the closure of the wound. The correct balance between too little contraction,
which leads to non-healing wounds, and too much contraction, which leads to contractures, is
important for optimal healing. The other cell types do not play a large role in wound contraction.

12- Which type of collagen is the first to be deposited in a healing wound?

A. Type I
B. Type II
C. Type III
D. Type IV
E. Type V

Correct choice: C. Type III

Explanation: During the proliferative phase of wound healing, granulation tissue is formed, which
relies on presence of fibronectin. Fibronectin is then replaced by collagen type III, and ultimately by

8
collagen type I. Wound contraction then ensues during the second week of wound healing, a process
that is mediated by myofibroblasts. The remaining answer choices are incorrect.

13- Which of the options below, per high concentrations noted in recent studies, is the most
important factor in the pathogenesis of SJS/TEN?

A. Granzyme B
B. TNF-β
C. IL-8
D. Caspase
E. IL-3

Correct choice: A. Granzyme B

Explanation: While the pathogenesis of SJS and TEN is not exactly understood, Granzyme B has
been shown to be a key factor due to high concentrations present in affected skin. TNF-alpha,
perforin and FasL have also been shown to be key to the development of the adverse drug
dermatosis. The other cytokines listed are not as critical in the development of SJS/TEN.

14- This patient presents with chronic swelling of the lower extremities, and circumferential lesions
on both lower legs as shown in this image. What is the most likely diagnosis?

A. Chronic venous stasis


B. Elephantiasis nostras verrucosa
C. Deep fungal infection
D. Peripheral arterial disease
E. Pigmented purpura dermatosis (Schamberg’s disease)

9
Correct choice: B. Elephantiasis nostras verrucosa

Explanation: Elephantiasis nostras verrucosa is complication from chronic ymphedema. It


commonly affects distal feet and lower extremities leading to verrucous changes with
papillomatosis and hyperkeratosis. The other options would not lead to circumferential verrucous
lesions on both lower extremities.

15- A child with a stahylococcal infection has an exfoliative dermatitis. The pathogenesis of this
eruption is similar to what disease?

A. Paraneoplastic pemphigus
B. Epidermolysis bulls simplex
C. Scarlet fever
D. Pemphigus foliaceous
E. Toxic epidermal necrolysis

Correct choice: D. Pemphigus foliaceous

Explanation: Staphylococcal scalded skin syndrome (SSSS) is descibed, a staphylococcal infection


that has exfoliative properties, and the pathogenesis involves serine proteases binding desmoglein 1.
This explains the clinical similarities between pemphigus foliaceous and SSSS. The other answer
choices are not similar in pathogenesis to that of staphylococcal scalded skin syndrome.

16- Botulinum toxin acts by reducing the amount of acetylcholine release from nerve endings and
can be targeted for use on sweat glands for hyperhidrosis. Which of the following sites has the
greatest density of these sweat glands?

A. Nipple
B. Clitoris
C. Labia minora
D. External auditory canal
E. Vermilion lips

correct choice: A. Nipple

10
Explanation: A is correct. The question refers to eccrine glands, which are innervated by
postganglionic sympathetic fibers and use acetylcholine as their neurotransmitter. They are targeted
by botulinum toxin for hyperhidrosis (in contrast to apocrine glands; innervation of these glands is
less clear). All of the following sites except for the nipple (A) lack eccrine glands.

17- An 80-year-old man with Parkinson’s disease developed a new pruritic eruption with these
cutaneous findings. What is the antigenic target of the autoantibodies generated in this condition?

A. NC16A domain of BPAg1


B. NC16A domain of BPAg2
C. C-terminal of BPAg1
D. C-terminal of BPAg2
E. alpha-6-beta-4 integrin

Correct choice: B. NC16A domain of BPAg2

Explanation: Bullous pemphigoid is the most common immunobullous disorder and presents with
tense vesicles and bullae and pruritus. There has been as association of neurologic disorders, such
as Parkinson's disease and dementia, with bullous pemphigoid. Autoantibodies are most commonly
directed against the NC16A domain of BPAg2, also known as BP180 or collagen XVII.

18- Which of the following absorbs ultraviolet radiation?


A. Filaggrin
B. Loricrin
C. Involucrin
D. Urocanic acid
E. Laminin 332

11
Correct choice: D. Urocanic acid

Explanation: Urocanic acid is a chromophore that absorbs ultraviolet radiation (UVR) in the stratum
corneum.This question assesses the examinee’s basic science knowledge of the function of skin
proteins and their breakdown products. In the stratum corneum, urocanic acid (choice 4) functions
as a chromophore that absorbs UVR and also mediates UV-induced immunosuppression. None of
the remaining answer choices absorbs UVR. Urocanic acid is a degradation product of filaggrin
(choice 1). Filaggrin cements the keratin filaments composing the stratum corneum; it is mutated in
atopic dermatitis and ichthyosis vulgaris. Loricrin (choice 2) is the most plentiful component of the
cornified envelope. Involucrin (choice 3), another component of the cornified envelope, is cross-
linked to loricrin by transglutaminase 1. Laminin 332 (choice 5), found within the lamina lucida and
densa, connects anchoring filaments to collagen VII.

19-Which of the following is NOT a degranulating stimulus for the largest cells seen in the image?

A. Ibuprofen
B. Fentanyl
C. Captopril
D. Vancomycin
E. Polymyxin B

Correct choice: C. Captopril

Explanation: The largest cells in the image are mast cells, which can be identified based on their
size and presence of many intracellular granules. Captopril, and ACE-inhibitor, is not a
degranulating stimulus for mast cells. The remaining answer choices are all mast cell degranulating
stimuli, thus patients with mastocytosis should be counseled to avoid these medications if possible.

12
20- Which of the following medications is not concentrated in the eccrine glands?

A. Cyclophosphamide
B. Doxycycline
C. Cephalexin
D. Ciprofloxacin
E. Cytarabine

Correct choice: B. Doxycycline

Explanation: The above listed drugs, as well as beta-lactam antibiotics, antifungals such as
ketoconazole and griseofulvin, are known to be secreted into eccrine sweat ducts. This feature may
explain the development of neutrophilic eccrine hidradenitis and eccrine squamous
syringometaplasia in the context of chemotherapy. Doxycycline does not accumulate in eccrine
glands.

21- Which of the following is a conservative and effective treatment for chondrodermatitis
nodularis helicis?

A. Mohs micrographic surgery


B. Aggressive cryodestruction with liquid nitrogen
C. TCA peel
D. Cushion pillow support on either side of the lesion to reduce focal pressure
E. Cantharidin application

Correct choice: D. Cushion pillow support on either side of the lesion to reduce focal pressure

Explanation: Chondrodermatitis nodularis helices presents as a tender nodule usually on the helix or
antihelix of the ear. It is thought to be a result of pressure to the local, affected area. Multiple
treatment modalities have been noted with conservative, effective treatment seen with cushion
pillow application to reduce the focal pressure. The other options listed are either too invasive
(Mohs, aggressive liquid nitrogen) or are less effective (TCA peels and cantharidin).

22- At any one time, the approximate proportion of hair follicles in anagen phase is:

13
A. 9%
B. 90%
C. 1%
D. 60%
E. 95%

Correct choice: B. 90%

Explanation: Most (~90%) hair follicles are in anagen (growth) phase. The longer a hair follicle is
in anagen phase, the longer the hair can grow. Hairs of the scalp grow approximately 0.4 mm per
day, and thus the date of one's next hair cut can be accurately calculated. Approximately 1% of hairs
are in catagen (transitional) phase, whereas ~9% of hairs are in telogen (resting) phase. The
remaining answer choices (60% and 95%) are distractors.

23- All of the following are increased in psoriasis EXCEPT:

A. Ornithine decarboxylase
B. Keratin 6
C. Loricrin
D. Involucrin
E. Keratin 16

Correct choice: C. Loricrin

Explanation: Loricrin is the major protein component of the cell envelope. It is decreased in
psoriasis. The remaining answer choices are increased in psoriasis.

24- Detection of immunoglobulins, complement, or both at the basement membrane zone in lesional
skin by direct immunofluorescence occurs in approximately half of cases of which dermatologic
condition?

A. Neonatal lupus erythematosus


B. SLE
C. Erythema multiforme

14
D. Mixed connective tissue disease
E. Lichen planus

Correct choice: A. Neonatal lupus erythematosus

Explanation: The key diagnostic sign of neonatal lupus erythematosus (NLE) is the erythematous
and annular clinical appearance of lesions. The histopathologic findings may be subtle in NLE
compared with discoid or subacute cutaneous lupus erythematosus. Direct immunofluorescence
testing may be useful in supplementing the histopathologic evaluation. Detection of
immunoglobulins, complement, or both at the basement membrane zone in lesional skin by
immunofluorescence occurs in approximately half of cases; a negative study does not preclude the
diagnosis of NLE syndrome. Clinical and serologic evaluation of both the infant and the mother are
important in establishing a diagnosis of NLE syndrome, particularly when histopathologic findings
are subtle.
DIF (direct immunofluorescence) of lichen planus is positive in the vast majority, with granular
DEJ (dermal-epidermal junction) deposition and IgM and fibrinogen staining within cytoid bodies
in the superficial dermis. In MCTD DIF shows IgG deposits within epidermal cell nuclei, and rarely
along the DEJ. In SLE (systemic lupus erythematosus), while serology is more reliable, DIF can
show DEJ deposition in the lupus band test. DIF of erythema multiforme shows immunoglobulin
within superficial vessel walls, DEJ, and cytoid bodies.

25- A 16-year-old female presents with severe scarring, cystic acne. Which of the following
immunologic factors are involved in the pathogenesis of acne?

A. TRPV channels
B. TLR-2
C. MMPs
D. IL-2
E. KLK5

Correct choice: B. TLR-2

Explanation: TLR-2 is a key aspect in the pathogenesis of acne. The other options listed are not key
parts of the pathogenesis of acne, some are specifically involved in the pathogenesis of rosacea
(KLK5, MMPs and TRPV channels).

15
26- All of the following locations have apocrine glands except:

A. Internal nares
B. External ear canal
C. Areola
D. Anogenital region
E. Periumbilical region

Correct choice: A. Internal nares

Explanation: Apocrine glands are found in the axillae, anogenital region, external ear canal, eyelids,
breast, periumbilical region. Apocrine glands produce an oily fluid rich in triglycerides and fatty
acids – subsequent colonization by anaerobic bacteria results in body odor.
Apocrine sweat glands are androgen-dependent for their development and have an unclear function
in humans; primary locations are the axillae, anogenital region, periumbilical region and nipples
Apocrine glands, whose apical portion (acrosyringium) drains into terminal hair follicles,
continuously secrete a sterile odorless viscous fluid that is rich in precursors of odoriferous
substances.

27- Subcorneal pustular dermatosis type IgA pemphigus is caused by a defect in what antigen?

A. Desmoglein 3
B. Desmoglein 1
C. BPAg1
D. Desmocollin 1
E. BPAg2

Correct choice: D. Desmocollin 1


Explanation: IgA pemphigus has two types subcorneal pustular dermatosis type which is due to ab
to desmocollin 1 and intraepidermal n eutrophilic IgA dermatosis which is due to ab to desmoglein
3 and 1. Sneddon- Wilkinson disease, also known as sub-corneal pustular dermatosis has an
unknown etiology. Desmoglein 1 defect is seen in pemphigus foliaceous and staph scalded skin
syndrome. Desmoglein 3 defect is seen in pemphigus vulgaris and intraepidermal neutrophilic IgA
dermatosis. BPAg1 is seen in paraneoplastic pemphigus and bullous pemphigoid. BPAg2 is seen in
bullous pemphigoid, cicatricial mucous membrane pemphigoid and linear IgA disease.

16
IgA pemphigus (or immunoglobulin A pemphigus) is an autoimmune blistering disorder. It is also
called intercellular IgA dermatosis among other names.
IgA pemphigus has two major subtypes:
1-Subcorneal pustular dermatosis (SPD) type
2- Intraepidermal neutrophilic (IEN) type.
The clinical features of IgA pemphigus include blisters, pustules, erythema, erosions and vegetating
lesions. The first signs are flaccid vesicles and pustules. The vesicles and pustules rupture to form
erosions and crusted plaques. The eruption tends to follow an annular pattern.
IgA pemphigus appears to favor the trunk, upper and lower extremities, axillae, and
groin. Mucosal involvement is infrequent. The exact cause of IgA pemphigus is unclear.
IgA autoantibodies bind to desmogleins or desmocollins, the cells responsible for adhesion between
cells. In SPD-type IgA pemphigus, the autoantigen is described as desmocollin 1, one of the
desmosomal cadherins, the glycoproteins that maintain the shape of a cell. In at least some cases of
IEN-type IgA pemphigus, the autoantigens are desmoglein 1, desmoglein 3, and an unspecified
transmembrane protein.

28- What is the most abundant amino acid in collagen?

A. Leucine
B. Proline
C. Glycine
D. Hydroxyproline
E. Hydroxylysine

Correct choice: C. Glycine

Explanation: Collagen is composed of 3 chains of amino acids combined into a triple helix
configuration. It contains Gly-x-y repeats (glycine is always the 3rd residue). Thus, glycine is the
most abundant amino acid in collagen. The remaining answer choices are not the most abundant
amino acids in collagen.

29.Which of the following medications is concentrated in the eccrine glands?

A. Cyclophosphamide
B. Cytarabine

17
C. Ciprofloxacin
D. Cephalexin
E. All of the answers are correct

Correct choice: E. All of the answers are correct

The above listed drugs, as well as beta-lactam antibiotics, antifungals such as ketoconazole and
griseofulvin, are known to be secreted into eccrine sweat ducts. This feature may explain the
development of neutrophilic eccrine hidradenitis and eccrine squamous syringometaplasia in the
context of chemotherapy.

30.Which element is necessary for function of matrix metalloproteinases?

A. Iron
B. Nitrogen
C. Manganese
D. Magnesium
E. Zinc

Correct choice: E. Zinc

Matrix metalloproteinases are required for normal tissue architecture and normal turnover of the
extracellular matrix. All of them have zinc at the active site and require octahedral binding of
calcium ions to maintain structural integrity.

31.What is the most important cell for wound healing?

A. Fibroblasts
B. Neutrophils
C. Macrophages
D. Lymphocytes
E. Mast cells

18
Correct choice: C. Macrophages

Macrophages are the most important cell for wound healing. They secrete TGFs, cytokines, IL-1,
TNF, and PDGF. Neutropenic or lymphopenic patients do not have impaired wound healing,
whereas macrophage-deficient (quantity or function) patients heal poorly. Neutrophils however are
the first cell type to flood the wound during phase I of inflammation (within first 6-8 hours); this
movement is facilitated by TGF-beta. Fibroblasts migrate into the wound by stage II (granulation)
where they produced glycosaminoglycans and fibronectin.

32.Which of the following is true in or associated with Apert Syndrome?

A. Secondary to a mutation in FGFR3


B. AR
C. Acne localized to buttocks and thighs
D. Cutaneous/ocular depigmentation
E. Synostoses

Correct choice: E. Synostoses

Apert Syndrome is an AD syndrome secondary to a mutation in FGFR2. Also known as


acrocephlosyndactyly, this condition includes synostoses of the hands, feet, back and skull as well
as generalized acne. Mosaicism of this gene causes nevus comedonicus. There is associated
cutaneous and ocular hypopigmentation.

33.What is the most abundant collagen found on fetal skin?

A. Type I Collagen
B. Type II Collagen
C. Type III Collagen
D. Type IV Collagen
E. Type VII Collagen

Correct choice: C. Type III Collagen

19
Type III collagen is found in the fetal skin. It is also present in the gastrointestinal tract, blood
vessels, and the basement membrane. A defect in this collagen results in the Ehlers-Danlos, vascular
type.

34.Which component of hair is positive for citrulline?

A. Outer root sheath


B. Inner root sheath
C. Cortex
D. Glassy vitreous layer
E. Medulla

Correct choice: B. Inner root sheath

The inner root sheath stains red because it contains citrulline. The cortex and medulla along with
the cuticle make up the hair shaft. The outer root sheath is the most peripheral cellular structure.
The glassy vitreous layer is the basement zone equivalent of hair and is the outermost layer.

35.The major protein component of the cornified envelope is:


A. Envoplakin
B. Desmoplakin
C. Plectin
D. Loricrin
E. Transglutaminase

Correct choice: D. Loricrin

Loricrin is the major component of the cornified envelope (CE). The proteins of the CE are
synthesized in the spinous and granular layers. The CE is primarily a protein/lipid polymer formed
within the differentiating layer of keratinocytes. The CE eventually exists outside of the cornified
cells after the granular cell undergoes a programmed destruction (apoptosis). Self-destructing
granular cells are called transition cells.

20
36.The most common type of pityriasis rubra pilaris in childhood is type:

A. I
B. II
C. III
D. IV
E. V

Correct choice: D. IV

Type IV, or circumscribed juvenile, accounts for 25% of total PRP cases. The most common type is
Type I, classical adult, which accounts for 55% of cases. Types II, III, and V account for less than
10% each.

37.Elastic fibers contain the specific amino acids:


A. Lysine and proline
B. Leucine and isoleucine
C. Alanine and phenylalanine
D. Desmosine and isodesmosine
E. Glycine and proline

Correct choice: D. Desmosine and isodesmosine

Desmosine and isodesmosine are the typical amino acids of elastic fibers. Elastic fibers are
comprised of elastin that is wrapped by fibrillin microfibrils. Elastic fibers form a complex
meshwork extending from the lamina densa of the dermo-epidermo junction through the dermis.
Elastic fibers return the skin to a normal shape after being stretched.

38.Anchoring fibrils are primarily composed of:

A. Type I collagen
B. Type III collagen
C. Type IV collagen
D. Type VII collagen
E. Type II collagen

21
correct choice: D. Type VII collagen.

Anchoring fibrils are found in the sublamina densa and are made up of collagen type VII. This
collagen type is mutated in dystrophic epidermolysis bullosa, and targeted in epidermolysis bullosa
acquisita and bullous lupus erythematosus.

39.A child presents with a 1 cm yellow-red nodule on the face. Pathology shows Touton giant cells.
What is the most frequent site of extracutaneous involvement in this disease?

A. Eye
B. Lung
C. Bone
D. CNS
E. Visceral

Correct choice: A. Eye

The eye is the most frequent site of extracutaneous juvenile xamthogranuloma. The second most
common site of extracutaneous disease is the lungs. Ocular involvement is typically unilateral.

40.Keratinocytes are derived from which of the following:

A. Endoderm
B. Mesoderm
C. Ectoderm
D. Neural Crest
E. Bone marrow precursors

Correct choice: C. Ectoderm .

As implied by the root 'ecto', a prefix meaning "outer", the keratinocytes of the epidermis are
derived from the ectoderm. The other layers do contribute cell populations that are present in the
skin.

22
41.Which epidermal layer do pilar cysts generally not have a:

A. Stratum corneum
B. Stratum granulosum
C. Stratum spinosum
D. Stratum basale
E. Stratum lucidum

Correct choice: B. Stratum granulosum

Pilar cysts do not have a granular layer, they do have all the other layers.

42.Fragmentation and/or loss of elastic fibers in not seen in:

A. Cutis laxa
B. Marfan's syndrome
C. Anetoderma
D. Psuedoxanthoma elasticum
E. Buschke-Ollendorf syndrome

Correct choice: E. Buschke-Ollendorf syndrome

Cutis laxa results from decreased desmosine and lysyl oxidase and demonstrates fragmentation and
loss of elastic fibers. Marfan's sydnrome results from decreased fibrillin I and demonstrates
fragmentation of elastic fibers. Pseudoxanthoma elasticum demonstrates increased
glycosaminoglycans on elastic fibers and the accumulation of fragmented and calcified elastic
fibers. Anetoderma has decreased desmosine and demonstrates loss and fragmentation of elastic
fibers. Buschke- Ollendorf syndrome exhibits increased desmosine and an increased amount of
thickened elastic fibers.

43.Which of the following protein plays a major role in wound healing?

A. Uncein
B. Fibronectin

23
C. Nidogen
D. Entactin
E. Band-6 protein

Correct choice: B. Fibronectin

Fibronectin is a key player in wound healing, initially secreted by myofibroblasts. The bed/matrix
of fibronectin provides an adherent base for migration into the wound, provides scaffolding for
collagen fibrils and mediates wound contraction. Band-6 protein is a constituent of the desmosomes
intracellular plaque. Entactin and nidogen are synonyms and are found in the dermal-epidermal
junction, binding to laminin 1s alpha chain. Uncein is associated with anchoring filaments.

44.Keratinocytes have been shown to secrete all of the following cytokines except:

A. IL-1
B. IL-6
C. IL-8
D. TNF-alpha
E. IL-2

Correct choice: E. IL-2

Keratinocytes have been shown to secrete all of the above cytokines, except IL-2, IL- 4, and IFN-
gamma.

45.The microflora of pilosebaceous unit consist of which of the following:

A. Pityrosporum ovale
B. Staphylococcus aureus
C. Escherichia coli
D. Pseudomonas aeruginosa
E. Corynebacterium diphtheriae

24
Correct choice: A. Pityrosporum ovale

All the above bacteria and fungi are found within sebaceous glands; the Malasssezia spp. and P.
ovale are found within the acroinfundibulum, S. epidermidis is found within the midinfundibulum,
and Propionibacterium spp. deep within the follicle.

46.Regarding dermal-epidermal junction, which of the following statements is true:

A. There are no anchoring filaments in lamina lucida


B. Lamina fibroreticularis lies above lamina densa
C. Lamina fibroreticularis comprises of anchoring fibrils and the elastic microfibrils
D. Blood vessels cross the dermal-epidermal junction to reach the epidermis
E. Lamina lucida is an electron-dense layer

Correct choice: C. Lamina fibroreticularis comprises of anchoring fibrils and the elastic microfibrils

Lamina fibroreticularis comprises of anchoring fibrils and the elastic microfibrils. Lamina
fibroreticularis lies below lamina densa. Blood vessels don't cross the dermal-epidermal junction to
reach the epidermis. Basement membrane contain both: 1)Lamina basale which include a.lamina
lucida and b.lamina densa ( which contain collagene IV & other glycoproteins like
laminins,fibronectin etc.) 2)Lamina fibroreticularis contain collagen type III and anchoring fibrils
(collagen type VII).

47.Direct immunofluorescence staining of intercellular spaces and the basement membrane zone, in
combination, is seen in:

A. Paraneoplastic pemphigus
B. Anti-epiligrin pemphigoid
C. Pemphigus vegetans
D. Pemphigus foliaceous
E. IgA pemphigus

Correct choice: A. Paraneoplastic pemphigus

25
Paraneoplastic pemphigus and drug-induced pemphigus demonstrate direct immunofluorescence
staining of the intercellular space and the BMZ, in combination. Anti-epiligrin pemphigoid
demonstrates BMZ staining (dermal staining on salt-split skin). Pemphigus vegetans, pemphigus
foliaceous, and IgA pemphigus all show intercellular space deposition without staining of the BMZ.

48.Keratohyalin granules contain:

A. Desmoplakin
B. Envoplakin and Keratin 6
C. Profilaggrin and loricrin
D. Numerous Golgi apparati
E. Involucrin

Correct choice: C. Profilaggrin and loricrin

Keratohyalin granules are found in the stratum granulosum (the granular layer), and contain the
proteins profilaggrin and loricrin. Profilaggrin is
converted to filaggrin during the transformation of the granular layer to the cornified layer. This is a
calcium-dependent process. Loricrin comprises 75% of the cornified envelope�s mass.

49.Which eponym describes vestigial lines of pigmentary demarcation?

A. Futcher lines
B. Wallace's lines
C. Langer's lines
D. Lines of Blaschko
E. Dermatome

Correct choice: A. Futcher lines

Futcher lines are vestigial lines in which the dorsal surface has more melanocytes than ventral
surface. Wallace's lines are the well-demarcated lines around the margin of the foot and hand.

26
50.Sebaceous glands are located in each of the following locations except:

A. Nipple
B. Labia minora
C. Palms
D. Eyelids
E. Buccal mucosa

Correct choice: C. Palms

Sebaceous glands secrete their contents in a holocrine fashion and are primarily under the influence
of androgens. They secrete triglycerides, phospholipids, esterified cholesterol, waxes but not free
cholesterol. They may be found in association with hair follicles or in some areas of modified skin
such as the nipple/areola, labia minora, prepuce, vermilion border, and eyelids.

51.All of the following pertain to Odland bodies EXCEPT:

A. Contain squalene
B. Are found intracellularly in upper level keratinocytes
C. Discharge their contents into the extracellular space at the junction of the granular and
cornified layers
D. Establish a barrier to water loss
E. Mediate stratum corneum adhesion in conjunction with filaggrin.

Correct choice: A. Contain squalene

Odland bodies mediate stratum corneum adhesion in conjunction with FILAGGRIN. They
discharge their contents into the extracellular space at the junction of the granular and horny layers
establish a barrier to water loss. They are first found intracellularly in upper level keratinocytes and
contain ceramides not squalene.

52.Red or blonde hair pigmentation primarily results from:

A. The presence of eumelanin

27
B. The absence of melanin
C. The presence of pheomelanin
D. The reduced activity of tyrosinase
E. The reduced activity of DOPA dehydroxylase

Correct choice:C. The presence of pheomelanin

Hair color is determined by melanocytes. The melanocytic activity of follicular melanocytes is


coupled to anagen � hair is only pigmented when it is growing. Pigment is produced in the matrix
area of follicle, above the follicular papilla. Eumelanin is the pigment of brown/black hairs, and
pheomelanin is the pigment of red/blonde hairs. Intensity of color is proportional to the amount of
pigment. The absence of pigment produces white hair, and markedly reduced pigment produces
gray hair.

53.Epidermolysis bullosa simplex (EBS), Weber Cockayne type, is caused by what defect?

A. Collagen VII
B. Alpha-6-beta-4 integrin
C. Keratins 1 & 10
D. Keratins 5
E. Plectin

Correct choice: D. Keratins 5

All subtypes of EBS are caused by a defect in keratins 5 & 14 except EBS with muscular dystrophy
which is caused by a defect in plectin. An alpha-6-beta-4 integrin defect is seen in junctional
epidermolysis bullosa (JEB) with pyloric atresia. CollagenVII defect is seen in EB aquisita (EBA),
Bart's syndrome and dominant EB (DEB). Keratin 1 & 10 defects are not seen in any of the EB
subtypes.

54.Which of the following statements about elastic fibers is true?

A. Elastic fibers form 35% of the dry weight of the skin


B. Elastic fibers are 90% elastin wrapped in fibrillin

28
C. Collagen 1 is mutated in Marfan syndrome
D. Oxytalan fibers run parallel within the superficial papillary dermis
E. Elaunin fibers run perpendicular in thin bands within the reticular dermis

Correct choice: B. Elastic fibers are 90% elastin wrapped in fibrillin

Elastic fibers are responsible for much of the elasticity of the dermis. They are essentially 90%
elastin wrapped in fibrillin. They form 4% of the dry weight of the skin. Fibrillin 1 is mutated in
Marfan syndrome. Oxytalan fibers run PERPENDICULAR from the DEJ within the superficial
papillary dermis. Elaunin fibers run parallell in thin bands within the reticular dermis.

55.Eccrine glands are found in all the following areas of the body except:

A. Axillae
B. Palms
C. Labia minora
D. Scalp
E. Cutaneous lip

Correct choice: C. Labia minora

Eccrine glands are sweat glands enervated by cholinergic sympathetic nerves mediated by
acetylcholine. They absent on modified skin which lacks appendages like the vermillion border, nail
beds, glans penis, inner aspect of the prepuce and the labia minora.

56.When evaluating a foreign body, which substance would be PAS negative and have no
bifringence on polarizing microscopy?

A. Silica
B. Talc
C. Zinc
D. Aluminium
E. Wood splinters

29
Correct choice: D. Aluminium

Starch, cactus spines, and wood splinters are PAS positive. Silica, talc, zinc, and wood splinters are
positive for bifringence on microscopy.

57.Which of the following statements about the direct immunofluorescence pattern in lichen planus
is correct?

A. The DIF is negative in the vast majority of cases


B. Deposition of IgG is within cytoid bodies in the superficial dermis
C. The DEJ deposition is granular
D. Deposition of fibrinogen is within cytoid bodies in the deep dermis
E. There is prominent deposition of IgM within the spinous layer of the epidermis.

Correct choice: C. The DEJ deposition is granular

The DEJ deposition is granular. The DIF is positive in the vast majority of cases. Deposition of IgM
and fibrinogen is within cytoid bodies in the superficial dermis. There is no deposition of IgM
within the spinous layer of the epidermis.

58.Human sebum is distinguished from lipids of internal organs by the presence of:

A. Cholestrol
B. Cholestrol esters
C. Squalene
D. Wax esters
E. Glycerides

Correct choice: D. Wax esters

As human sebum exits the sebaceous gland, its major constituents are squalene, cholesterol,
cholesterol esters, triglycerides, and wax esters. With passage through the hair follicle,
triyglycerides in the sebum become hydrolyzed by bacterial enzymes, so that by the time the sebum
reaches the skin surface, it contains free fatty acids, mono- and diglycerides in addition to the

30
original components. Human sebum is distinguished by the presence of wax esters and squalene.
The lipids of humaninternal organs contain no wax esters and little squalene. The squalene that is
produced in internal organs is quickly converted to lanosterol and then to cholesterol,
so it does not remain in its original form. Human sebaceous glands do not convert squalene to
sterols.

59.Melanocytes are derived from:

A. Bone marrow
B. Neural crest
C. Mesodermal precursors
D. Endodermal precursors
E. Yolk sac derived

Correct choice: B. Neural crest

Melanocytes are derived from neural crest precursors and migrate to the epidermis, hair matrix,
retinal pigment epithelium, ear (stria vascularis), leptomeninges, and mucous membranes. The other
options listed are incorrect and are not involved with melanocytes.

60.Apocrine glands are found in all of the following areas of the body except:

A. Axillae
B. Breasts
C. Eyelid
D. Palms
E. Perineum

Correct choice: D. Palms

Apocrine glands operate by decapitation secretion and are activated by epinephrine and
norepinephrine. They are located in a few distinct areas of the body, which include axillae,
anogenital region, Moll’s glands of the eyelids, mammary glands of the breast and the ceruminous
glands of the external auditory canal.

31
61.Anagen effluvium is best described as:

A. An abrupt transition from anagen to catagen in rapidly dividing hair matrix cells
B. A cessation of mitotic activity in rapidly dividing hair matrix cells
C. An abrupt transition of telogen to anagen in resting hair matrix cells
D. A cessation of mitotic activity in resting hair matrix cells
E. A scarring alopecia affecting only anagen stage follicles

Correct choice: B. A cessation of mitotic activity in rapidly dividing hair matrix cells

Anagen effluvium results from an outside stimulus – most often an antimetabolite,


chemotherapeutic drug – inducing an abrupt cessation of hair matrix cell mitotic activity. This
process occurs within days to weeks of the stimulus, and is reversible with cessation of the drug
therapy.

62.Which of the following statements is true about eccrine glands?

A. Postganglionic sympathetic fibers with acetylcholine as the principal neurotransmitter


B. Postganglionic sympathetic fibers with norepinephrine as the principal
neurotransmitter
C. Postganglionic parasympathetic fibers with acetylcholine as the principal
neurotransmitter
D. Postganglionic parasympathetic fibers with norepinephrine as the principal
neurotransmitter
E. Postganglionic sympathetic fibers with both norepinephrine and acetylcholine as the
principal neurotransmitters

Correct choice: A. Postganglionic sympathetic fibers with acetylcholine as the principal


neurotransmitter.

Eccrine glands are innervated by postganglionic sympathetic fibers with acetylcholine as the
principal neurotransmitter. This explains why medications associated with anticholinergic side
effects may be associated with hypohidrosis.

32
63.The strength of a scar:

A. Is 5% at 1 week
B. Is 20% at 3 weeks
C. Is 70% at 1 year
D. All of these options are correct
E. None of these options are correct

Correct choice: D. All of these options are correct

A scar has 5% strength at 1 week, 20% at 3 weeks and 70% at 1 year. It will never recover strength
to the level of pre-injury.

64.The target for GABEB and bullous pemphigoid is:

A. Type XVII collagen


B. Type VII collagen
C. Laminin 5
D. Integrin subunit B4
E. Type XII collagen

Correct choice: A. Type XVII collagen

The target for GABEB and bullous pemphigoid is type XVII collagen or BPAG2. It is also the
target protein in pemphigoid gestationis, CP and linear IgA.

65.All mononuclear phagocytic cells in the dermis express:

A. CD3
B. CD6
C. CD34
d. CD68
E. CD20

33
Correct choice: B. CD6

CD6 and CD11c are expressed on all mononuclear phagocytic cells in the dermis. CD3 is a T-cell
marker and CD20 is a B-cell marker. CD34 is expressed on mast cells and CD68 on macrophages.

66.How soon does epithelialization begin after a skin wound occurs?

A. Minutes
B. Hours
C. 2 days
D. 4 days
E. 6 days

Correct choice: B. Hours

Re-epithelialization begins hours after an injury occurs. Keratinocytes from residual epithelial
structures leapfrog each other. One to two days after injury, cells at the wound margin proliferate
and begin to migrate into the
wound.

67.Regarding sebaceous glands:

A. These glands are present at birth at their adult size


B. Size of the gland is proportional to the size of the associated hair follicle
C. Are always associated with a hair follicle
D. Are found everywhere on the skin except palms and soles
E. Are unilobular glands

Correct choice: D. Are found everywhere on the skin except palms and soles

Sebaceous glands are found everywhere on the skin except on the palms and soles. They are
multilobular, emptying into the sebaceous duct. Most sebaceous glands are associated with hair
follicles, but free glands exist, especially on the lip (Fordyce spots), on the nipple/
areola(Montgomery�s tubercles), Meibomian glands of the eyelids and Tyson�s glands on the

34
genitalia. The size of the sebaceous glands are not related to the size of the associated follicle. The
sebaceous gland enlarges at puberty in response to increased androgens.

68.What is the major function of urocanic acid?

A. Bacteriocidal acid produced by stratum corneum


B. Primarily a UVB filter
C. Primarily a UVA filter
D. Helps degrade free fatty acids
E. Aids in protecting the skin from dermatophytes

Correct choice: C. Primarily a UVA filter

Urocanic acid a by product of filaggrin degradation and has a peak absorbtion of 345 nm. It serves
as a major UVA filter.

69.The main collagen component of the basement membrane is:

A. Collagen IV
B. Collagen III
C. Collagen I
D. Tenascin-X
E. Collagen VII

Correct choice: A. Collagen IV

Collagen IV is the main collagen component of basement membranes. Collagen I is the main
collagen of mature dermis, bone and tendon. Collagen III is found in fetal skin, blood vessels and
intestines. Tenascin-X is mutated in some forms of Ehlers- Danlos syndrome and is not associated
with the basement membrane. Collagen VII makes up anchoring fibrils and amnion.

70.The main permeability barrier in the lamina densa is:

35
A. heparan sulfate proteoglycan
B. collagen IV
C. laminin 5
D. nidogen
E. alpha-6-beta-4 integrin

Correct choice: A. heparan sulfate proteoglycan

All of the listed proteins are present in the lamina densa except alpha-6-beta-4 integrin, which
connects the hemidesmosome to laminin 5 in the lamina lucida. The heparin sulfate proteoglycans
(perlecan) are negatively charged, thus serve as a permeability barrier. The other listed proteins do
not serve this function in the lamina densa. Collagen IV is the main basic basement membrane
scaffold. Defects in Collagen IV have been linked to Alport�s and Goodpasture�s syndrome.
Nidogen has a �dumbbell� shape and binds both laminins and collagen IV in the lamina densa.

71.The desmosomal connections of the epidermis are dependent on which of the following ions?

A. Iron
B. Zinc
C. Selenium
D. Calcium
E. Sodium

Correct choice: D. Calcium

The desmosomal connections in the epidermis are calcium dependent. The other options are not
required for these connections.

72.Eulanin fibers:

A. Run parallel in bands within the superficial papillary dermis


B. Run perpendicular from the dermo-epidermal junction within the superficial papillary
dermis
C. Run parallel in bands within the reticular dermis

36
D. Run perpendicular in bands within the deep dermis
E. Run perpendicular in bands within the reticular dermis

Correct choice: C. Run parallel in bands within the reticular dermis

Eulanin fibers are elastic fibers that have less elastin and more fibrillin and run parallel in thin
bands within the reticular dermis. Oxytalin fibers contain no elastin and run perpendicular from the
dermo-epidermal junction withinthe superficial papillary dermis. Elastic fibers turn over slowly in
the skin, and are damaged by ultraviolet radiation.

73.Meibomian glands are:

A. Eccrine glands localized to the vermillion border of the lips


B. Sebaceous glands found on the areola of the breast
C. Sebaceous glands found on the eyelids
D. Apocrine glands found in the anogenital regions
E. Apocrine glands found on the eylelids

Correct choice: C. Sebaceous glands found on the eyelids

Sebaceous glands enlarge at puberty in response to increased levels of androgens. They are
holocrine glands. Meibomian glands are modified sebaceous glands foundin the eyelids. Free
sebaceous glands not associated with hairs are found in the nipple and areola and are called
Montgomery�s tubercles. Fordyce�s condition involves free sebaceous glands on the
vermillion border of the lips and on the buccal mucosa. Sebaceous glands are found everywhere on
the skin except the palms and
soles.

74.Which of the following is true about melanosomes?

A. Spheroid melanosomes have concentric lamellae


B. Spheroid melanosomes synthesize brown-black eumelanin
C. Elliptical melanosomes have microvesicular structure
D. Elliptical melanosomes synthesize yellow or red pheomelanin.

37
E. The melanosomes are positioned after the Golgi apparatus in the secretory pathway.

Correct choice: E. The melanosomes are positioned after the Golgi apparatus in the secretory
pathway.

Melanosomes are organelles related to lysosomes and are positioned after the Golgi apparatus in the
secretory pathway. Elliptical melanosomes have concentric lamellae and synthesize brown-black
eumelanin. Spheroid melanosomes have microvesicular structure and synthesize yellow or red
pheomelanin.

75.Which sebaceous gland is located on the eyelids in association with eyelashes?

A. Montgomery's tubercles
B. Tyson's glands
C. Zeis glands
D. Meibomian glands
E. Fordyce's spots

Correct choice: C. Zeis glands

Zeis glands are sebaceous glands associated with eyelashes. Montgomery's


tubercles are present on the areola, Tyson's glands on the labia minora and glans, and Fordyce's
spots are located on the buccal mucosa.

76.Which part of the nose is not innervated by V2 branch of Cranial nerve V?

A. nasal columella
B. nasal ala
C. nasal tip
D. nasal dorsum
E. all sensory of the nose is innervated by V2

Correct choice: C. nasal tip

38
This is innervated by the anterior ethmoidal branch of V1. The infraorbital nerve of V2 innervates
the nasal ala. The nasopalantine branch of V2 innervates the columella.

77.Tyrosinase is the enzyme that catalyzes the conversion of tyrosine to DOPA and DOPA to
DOPAquinone. The enzyme contains which of the following ions?

A. Zinc
B. Copper
C. Selenium
D. Iron
E. Magnesium

Correct choice: B. Copper

Tyrosinase is a copper containing enzyme that is responsible for the conversion to tyrosine to DOPA
and DOPA to DOPAquinone.

78.In epidermolysis bullosa simplex, where on the blister does the signal localize on a salt split skin
test?

A. Roof
B. Floor
C. Middle
D. Diffuse
E. No localization

Correct choice: A. Roof

In dystrophic EB the signal localizes to the roof of the blister. In junctional EB, the roof has BPAG2
while the floor has type IV collagen.

79.The mechanism of action of ipilimumab can be described as

39
A. Inhibitor of CTLA-4
B. Antibody to CD27
C. Fusion protein binding B7
D. Antibody to CD 8 T cell
E. Inhibitor of CD27

Correct choice: A. Inhibitor of CTLA-4

Ipilimumab is an antibody to CTLA-4, this interaction causes inhibition of CTLA-4 which normally
inhibits T cell co-activation by binding to B7 (which normally binds to CD28).

80.Desmosine and isodesmosine are typical amino acids found in:

A. Collagen fibers
B. Anchoring fibril
C. Elastic fibers
D. Heparan sulfate
E. Anchoring plaques

Correct choice: C. Elastic fibers

Desmosine and isodesmosine are typical amino acids found in elastic fibers. They crosslink
fibrillin. Anchoring fibrils are composed of collagen VII and collagen fibers and have the most
typical amino acids of proline and hydroxyproline. Heparan sulfate do not typically contain these
amino acids.

81.A salt split skin DIF is performed on a biopsy taken adjacent to the skin lesions shown. Where
would you expect staining to be seen?

A. Epidermal side
B. Dermal side
C. Epidermal and Dermal sides equally
D. In the lamina densa

40
E. In the anchoring

Correct choice: A. Epidermal side

The image shown is bullous pemphigoid. On salt split skin DIF exams, deposits are seen on the
epidermal side of the split. If dermal deposits are seen, epidermolysis bullosa acquisita or anti-
epiligrin pemphigoid are potential diagnoses.

82. Homocystinuria is an autosomal recessive condition with findings including a marfanoid


habitus, downward dislocation of the lens, cardiovascular disease and mental retardation. It is
caused by a mutation in cystathionine beta-synthetase. What does this mutation in cystathionine
beta-synthetase cause other than an accumulation of homocystine?

A. Abnormal crosslinking of collagen


B. Abnormal development of elastin fibers
C. Melanocyte death
D. Pigmentation of cartilage
E. Black urine

Correct choice: A. Abnormal crosslinking of collagen

Excess homocystine leads to abnormal crosslinking of collagen, with only ~1/3 of crosslinking
activity compared to normal controls . The other listed findings are not features of homocystinuria,
though hypopigmentation can be a feature.

83.The following is a target in both junctional EB with pyloric atresia and ocular CP:

A. Integrin subunit B4
B. Laminin 5
C. Type VII Collagen
D. Type XVII collagen
E. BPAG2

41
Correct choice: A. Integrin subunit B4

Integrin subunit B4 is the target for both junctional EB with pyloric atresia and ocular CP.

84.Direct immunofluorescence is of no value in the diagnosis of:

A. Neonatal LE
B. Lichen planus
C. Mixed connective tissue disease
D. SLE
E. Erythema multiforme

Correct choice: A. Neonatal LE

DIF is of no value in the diagnosis of scleroderma, morphea, and neonatal LE. DIF of lichen planus
is positive in the vast majority, with granular DEJ deposition and IgM and fibrinogen staining
within cytoid bodies in the superficial dermis. In MCTD DIF shows IgG deposits within epidermal
cell nuclei, and rarely along the DEJ. In SLE, while serology is more reliable, DIF can show DEJ
deposition in the lupus band test. DIF of erythema multiforme shows immunoglobulin within
superficial vessel walls, DEJ, and cytoid bodies.

85.Type 1 collagen is the most prevalent collagen in skin, accounting for 80% or more of the total
collagen in the adult dermis. The next most predominant collagen in adult human dermis is:

A. Type II collagen
B. Type III collagen
C. Type IV collagen
D. Type VII collagen
E. Type XVII collagen

Correct choice: B. Type III collagen

Type 1 collagen is the most prevalent collagen in skin, accounting for 80% or more of the total
collagen in the adult dermis. Type III collagen is the next most predominant collagen in human

42
dermis, accounting for approximately 10%. Type IV collagen is found in basement membranes.
Type VII collagen is found in human dermis as anchoring fibrils. Type XVII collagen, also known
as BPAG2, is an important transmembrane pro
tein in the basement membrane zone.

86.The formation of granulation tissue depends on the presence of:

A. Neutrophils
b. Fibronectin
C. Collagen type I
D. Platelets
E. Collagen type IV

Correct choice: B. Fibronectin

Granulation tissue forms approximately four days after injury. It is composed of new capillaries,
macrophages, fibroblasts, and blood vessels. The formation is dependent on the presence of
fibronectin. There tends to be an ordered sequence of matrix deposition. Fibronectin is deposited
first followed by collagen 3 and then collagen 1. Granulation tissue primarily contains type 3
collagen.

87.Sebaceous glands:

A. Respond to chemical stimuli such as hormones


B. Respond to cholinergic neural activity, exclusively
C. Respond to adrenergic neural activity, exclusively
D. Respond to both adrenergic and cholinergic stimuli
E. Respond to the local release of cytokines from inflammatory cells

Correct choice: A. Respond to chemical stimuli such as hormones

Sebaceous glands are androgen-responsive holocrine glands that enlarge at puberty. Meibomian
glands of the eyelids are modified sebaceous glands. Sebaceous glandsare found everywhere on the
skin except the palms and soles. Fordyce�s condition involves free sebaceous glands on the

43
vermillion border of the lips and on the buccal mucosa. Eccrine glands are thermoregulatory
structures that respond to cholinergic stimulation.

88.The function of glycosaminoglycans/proteoglycans in the dermis is:

A. Regulate water-binding capacity


B. Interact with dermal dendrocytes
C. Facilitate COLVII binding to the anchoring plaques
D. Facilitate mast cell degranulation
E. Regulate lymphocyte trafficking

Correct choice: A. Regulate water-binding capacity

The function of glycosaminoglycans/proteoglycans in the dermis is to regulate water-binding


capacity. The other listed options are not functions of glycosaminoglycans / proteoglycans.

89.Which one of the following is responsible for maintaining a barrier to water loss in the stratum
corneum?

A. Involucrin
B. Filaggrin
C. Loricrin
D. Transglutaminase
E. Odland bodies

Correct choice: E. Odland bodies

Odland bodies, also known as lamellar granules, keratinosomes, and membrane-coating granules,
are small organelles that are discharged from granular cells into the intracellular space of the
granular layer of the epidermis. These bodies have two known functions: they mediate stratum
corneum cell cohesion and they form a barrier to water loss. Odland bodies are round to oval,
measure approximately 300 to 500 nm in diameter, and possess a trilaminar membrane and a
laminated interior. They contain neutral sugars linked to lipids and/or proteins, hydrolytic enzymes,
and free sterols. Filaggrin is a breakdown product of filaggrin precursor, acomponent of

44
keratohyaline granules, which aggregates with keratin filaments and acts as a "glue" for keratin
filaments. Involucrin is a cysteine-rich protein synthesized in the cytoplasm of spinous cells. The
enzyme, transglutaminase, cross-links involucrin in the granular layer forming an insoluble cell
boundary that is resistant to denaturing and reducing chemicals. Loricrin, is a highly insoluble
sulfur- and glycine/serine-rich protein, which is the major protein comprising the cornified cell
envelope.

90.During hair follicle development, the WNT signaling pathway is one of the earliest molecular
pathways involved in hair follicle initiation. What is the downstream mediator of WNT signaling?

A. Smoothened
B. Beta-catenin
C. Keratin 16
D. p53
E. HLA-B27

Correct choice: B. Beta-catenin

Beta-catenin is the downstream mediator of WNT signaling. Through a series of signals, WNT
proteins inhibit the degradation of beta-catenin in the cytoplasm. After being translocated to the
nucleus, beta-catenin then forms a complex with LEF/TCF transcription factors, which ultimately
results in expression of downstream genes. Activation of this pathway is necessary for epithelium to
have the potential to develop a hair follicle.

91.Which sebaceous gland is located on the buccal mucosa and vermi


lion border of the lips?

A. Montgomery's tubercles
B. Tyson's glands
C. Zeis glands
D. Meibomian glands
E. Fordyce's spots

Correct choice: E. Fordyce's spots

45
Fordyce's spots are located on the buccal mucosa and vermilion border of the lips. Montgomery's
tubercles are present on the areola, Tyson's glands on the labia minora and glans, meibomian and
Zeis glands are present on the eyelids.

92.What structure delineates the anatomic region between the nail


bed and the distal groove, where the nail plate detaches of the distal portion of the digit?

A. Nail matrix
B. Proximal nail fold
C. Lunula
D. Eponychium
E. Hyponychium

Correct choice: E. Hyponychium

The hyponychium is the structure that delineates the anatomic region between the nail bed and the
distal groove, where the nail plate detaches of the distal portion of the digit. The corneal layer of the
hyponychium accumulates in part under the free margin of the nail plate. In most cases, the
hyponychium is covered by the distal nail plate, however, it may become visible in the case of nail
biters.

93.Which of the following stains would you expect to be positive in a normal eccrine unit?

A. S-100
B. Prussian blue
C. Giemsa
D. Verhoeff von Gieson
E. Steiner

Correct choice: A. S-100

Eccrine glands stain S-100 and CEA positive. The remaining stains would not be expected to stain
normal eccrine sweat glands. Prussian blue (Perl’s) is an iron stain which stains iron or hemosiderin
bright blue. Giemsa stains mast cell granules purple (heparin in the granules) and can also be useful

46
in staining in Leishmaniasis. Verhoeff von Gieson is a stain for elastic tissue that stains blue-black.
Steiner stain is a silver stin for spirochetes similar to a Warthin Starry or Dieterle stain.

94.The hair follicle is the site of production or conversion of which of the following hormones?

A. All the options are correct


B. dihydrotestosterone (DHT)
C. prolactin
D. adrenocorticotropin hormone (ACTH)
E. alpha-melanocyte stimulating hormone (MSH)Question 66 Explanation

Correct choice: A. All the options are correct

The hair follicle converts gonadal or aderenal testosterone to dihydrotestosterone locally via 5-alpha
reductase. It also cleaves proopiomelanocortin to ACTH, alpha- MSH, and B-endorphin via
prohormone convertase. It also secretes corticotropin releasing hormone (CRH) as well as prolactin.

95.Which of the following statements about glomus cells is correct?

A. Tumors of glomus cells are most commonly found on the tongue


B. Are of neural origin
C. Allow rapid shunting of blood from the arterioles to venules, bypassing capillaries
D. Tumors composed of glomus cells are asymptomatic
E. Tumors composed of glomus cells are often malignant

Correct choice: C. Allow rapid shunting of blood from the arterioles to venules, bypassing
capillaries.

Tumors composed of glomus cells are usually PAINFUL, not asymptomatic. They are derived from
Suquet-Hoyer canals and allow rapid shunting of blood from the arterioles to venules. Glomus cells
are part of glomus tumors which are most often solitary, purple dermal nodules on the extremities.
Most often, they are seen on the fingers and toes. They are usually painful and rarely malignant. As
opposed to a glomus tumor, glomangiomas are usually painless. They also most often occur on
extremities, but can also occur on the trunk.

47
96.Where are glomus cells derived from?

A. Mesenchyme
B. Bone marrow
C. Susquet-Hoyer canal
D. Skeletal muscle
E. Dermis

Correct choice: C. Susquet-Hoyer canal

Glomus cells are vascular smooth muscle cells derived from the arterial portion of the glomus body,
or the Sucquet-Hoyer canal, which is an arteriovenous shunt in the dermis that contributes to
temperature regulation. Glomus are found primarily on the palms and soles and function to allow
the rapid shunting of blood from the arteiolesto venules. Disease processes involving glomus cells
include glomus tumor and glomangioma.

97.Which of the following cytokines is secreted by adipocytes?

A. IL-2
B. IL-4
C. IL-6
D. IL-10
E. IL-12

Correct choice: C. IL-6

Adipocytes secrete tumor necrosis factor (TNF)α, interleukin-6 (IL-6), leptin, adiponectin,
angiotensinogen, and resistin. Specifically it has been shown that IL-6 mRNA levels in human
subjects are significantly greater in adipose tissue than in other tissues known to express IL-6. It is
thought that circulating IL-6 levels may be higher in obese patients, contributing to the development
and progression of vascular disease in this patient population.

98.Dimple sign refers to:

48
A. When an intact epidermis shears away from the underlying dermis, leaving a moist
surface
B. Spreading bulla phenomenon with pressure on an intact bulla
C. Swollen, itchy and or red after stroking the skin
D. Central depression within a lesion when squeezed along its margins
E. Disappearance of color when the lesion is pressed

Correct choice: D. Central depression within a lesion when squeezed along its margins

Central depression within a lesion when squeezed is referred to as the dimpling sign and is seen in
dermatofibromas. Nikolsky sign can be seen when an intact epidermis shears away from the
underlying dermis, leaving a moist surface (seen in pemphigus vulgaris, staphylococcus scalded
skin syndrome (SSSS), and toxic epidermal necrosis). Spreading bulla phenomenon with pressure
on an intact bulla is referred to as Asboe-Hansen sign, commonly seen with pemphigus vulgaris.
Swollen, itchy and or red after stroking the skin is referred to Darier’s sign and can be
seen in systemic mastocytosis or urticaria pigmentosa. Disappearance of color or blanching when
the lesion is pressed is commonly found on vascular lesions.

99.Acid keratins are expressed on which of the following chromosomes?

A. 12
B. 17
C. 9
D. 16
E. 3

Correct choice: B. 17

Acidic keratins (K9-20) are expressed from chromosome 17. Basic keratins (K1-8) are on
chromosome 12. The other listed chromosomes are not involved in keratins.

100.Which hormone is homologous to alpha-MSH (melanocyte stimulating hormone)?

A. Insulin

49
B. Human growth factor
C. Prolactin
D. Thyroid stimulating hormone
E. Adrenocorticotropic hormone

Correct choice: E. Adrenocorticotropic hormone

Both alpha-MSH and ACTH are cleavage products of proopiomelanocortin (POMC).

101.What is the average duration of the telogen cycle in terminal scalp


hair?

A. 2-6 years
B. 2-3 weeks
C. 3 months
D. 6 months
E. 9 months

Correct choice: C. 3 months

The average duration of the telogen phase of the hair cycle is 3 months; this feature explains why
telogen effluvium is typically observed 3 months following a traumatic event or serious illness. The
average duration of the anagen phase of the hair cycle is 2-6 years, whereas that of the catagen
cycle is 2-3 weeks.

102.Which sebaceous gland is located on the areola?

A. Montgomery'€™s tubercles
B. Tyson’s glands
C. Zeis glands
D. Meibomian glands
E. Fordyce's spots

50
Correct choice: A. Montgomery'€™s tubercles

Montgomery'€™s tubercles are sebaceous glands not associated with hair follicles present on
the areola. Tyson'€™s glands on the labia minora and glans, meibomina and Zeis glands are
present on the eyelids , and Fordyce'€™s spots are located on the buccal mucosa.

103.Embryologically, epidermal stratification occurs at approximately what estimated gestational


age?

A. 4 weeks
B. 8 weeks
C. 12 weeks
D. 16 weeks
E. 20 weeks

Correct choice: B. 8 weeks

Epidermal stratification begins at about 8 weeks estimated gestational age and is completed by the
2nd trimester. Epideral stratification occurs when an 'intermediate layer' develops between the
epidermal basal cells and the overlying layer of periderm cells. The periderm is an embryonic
structure that covers the primitive epidermis until keratinization begins, at which point the periderm
sloughs off and contributes to the protective covering of the newborn, the vernix caseosa. This
intermediate layer is highly proliferative, such that by 24-25 weeks estimated gestaional age, the
epidermis consists of 4 or 5 layers, in addition to the degenerating eriderm.

104.Cutaneous warts in renal transplant recipients show increased expression of which keratin?

A. K2
B. K5
C. K9
D. K13
E. K16

Correct choice: D. K13

51
Keratins are divided into acidic (type I) and basic (type II) subtypes. Keratin 13 is a type I keratin
which is expressed suprabasally in adult epithelia and is associated with terminal differentiation.
Renal transplant recipients have an increased risk of developing skin cancers and multiple warts.
Factors contributing to this increased risk include human papillomavirus infection,
immunosuppressive therapy, and exposure to ultraviolet radiation. Increased expression of K13 has
been demonstrated in high-risk papillomas in this patient population.

105.The finding on DIF that reflects binding of the Ro and La antigens in subacute cutaneous lupus
erythematosus is:

A. Granular fluorescence throughout the cytoplasm and nucleus of basal keratinocytes


B. Cytoid bodies
C. Immune deposits along the DE junction
D. Granular deposits along the basement membrane
E. A "chicken-wire" pattern within the epidermis

Correct choice: A. Granular fluorescence throughout the cytoplasm and nucleus of basal
keratinocytes.

Granular fluorescence throughout the cytoplasm and nucleus of basal keratinocytes reflect the
binding of Ro and La antigens and is unique to SCLE. Cytoid bodies and Immune deposits along
dermal-epidermal junction are seen in both DLE and SCLE.
Granular deposits along the basement membrane are seen in dermatitis
herpetiformis and a chicken-wire pattern of staining is seen in pemphigus vulgaris.

106.At any one time, the approximate proportion of hair follicles in anagen is:

A. 40%
B. 60%
C. 85%
D. 95%
E. 15%

Correct choice: C. 85%

52
Most hair follicles are in anagen, and thus most hair follicles involve growing hair. The longer a
hair follicle is anagen, the longer the hair can grow in length. Hairs of the scalp grow approximately
0.4 mm per day, and thus the date of your next hair cut can be accurately calculated.

107.Retinoids upregulate transcription of which types of collagen?

A. 1 and 3
B. 1 and 4
C. 1 and 7
D. 3 and 7
E. 4 and 7

Correct choice: A. 1 and 3

Retinoids upregulate the transcription of collagens one and thereby strengthening the dermis.

108.People with darker skin show:

A. Smaller, more concentrated melanosomes


B. A more rapid degradation of melanosomes
C. A lessened production of melanosomes within melanocytes
D. A higher degree of dispersion of melanosomes in keratinocytes
E. A grouping of melanosomes with a low degree of melanization

Correct choice: D. A higher degree of dispersion of melanosomes in keratinocytes

More darkly pigmented races show a greater production of melanosomes in the melanocyte,
melanosomes with a higher degree of melanization, larger melanosomes, a higher degree of
dispersion of melanosomes in the keratinocytes, and a slower rate of melanosome degradation.

109.What is the location of the unbound corticosteroid receptor?

53
A. Cytoplasm
B. Nucleus
C. Mitochondria
D. Plasma membrane
E. golgi apparatus

Correct choice: A. Cytoplasm

Both androgen and corticosteroid receptors localize to the cytoplasm. Estrogen receptors are found
in the nucleus. Progesterone receptors are distributed in both the nucleus and the cytoplasm.

110.Which of the following diseases has decreased or absent lamellar granules?

A. Flegel's
B. Psoriasis
C. Lamellar ichthyosis
D. Epidermolytic hyperkeratosis
E. Pemphigus vulgaris

Correct choice: A. Flegel's

Flegel's disease has decreased or absent lamellar granules. AKA hyperkeratosis lenticularis perstans,
this is a rare, possibly AD disorder with multiple disc-like keratotic papules predominenty on the
distal extremities and feet of older individuals (Bolognia, p 1714).

111.Elastic fibers are present in the dermis and are responsible for providing tissue resiliency. They
are comprised of elastin as well as microfibrillar proteins including:

A. Fibrillins and fibulins


B. Hyaluronic acid
C. Collagen II
D. Laminin 5 and BPAG2
E. Desmoplakin and plakoglobin

54
Correct choice: A. Fibrillins and fibulins

Elastic fibers are comprised of elastin as well as microfibrillar proteins including fibrillins and
fibulins. Hyaluronic acid is a glycosaminoglycan found in the extracellular matrix of the dermis.
Collagen II is the predominant collagen found in cartilage. Laminin 5 and BPAG2 are important
anchoring filaments in the basement membrane zone. Desmoplakin and plakoglobulin are proteins
found in the anchoring plaques of desmosomes in the epidermis.

112.Which of the following amino acids are typically found in elastic fibers?

A. Desmosine
B. Isoleucine
C. Proline
D. Hydroxyproline
E. Glycine

Correct choice: A. Desmosine

Desmosine and isodesmosine are typical amino acids found in elastic fibers. Glycine, proline and
hydroxyproline are found as components of collagen. Isoleucine is not specific for elastin fibers.

113.At what estimated gestational age are all layers of the keratinized epidermis identifiable?

A. 8 weeks
B. 12 weeks
C. 16 weeks
D. 20 weeks
E. 24 weeks

Correct choice: E. 24 weeks

At 24 weeks, all the layers of the mature epidermis can be identified, and the epidermis is
keratinized.

55
114.Anchoring filaments originate at the hemidesmosomes and insert into the:

A. Desmosome
B. Sub basal dense plate
C. Lamina lucida
D. BPAG 180
E. Lamina densa

Correct choice: E. Lamina densa

Anchoring filaments (smaller than anchoring fibrils) stretch from the plasma
membrane through the subbasal dense plaque and the lamina lucida to the lamina densa.

115.Which of the following make up the major protein of the cornified cell envelope?

A. Loricrin
B. Involucrin
C. Envoplakin
D. Filaggrin
E. Laminin V

Correct choice: A. Loricrin

Loricrin is the major protein component of the cornified cell envelope (CE). Involucrin is cross-
linked by transglutaminase in the granular layer to form an insoluble cell boundary. Envoplakin
may link the CE to desmosomes and to keratin filaments. Filaggrin is thought to promote
aggregation and disulfide bonding of keratin filaments in CE. It is degraded into urocanic acid and
pyrrolidone carboxylic acid. Both of which hydrate the stratum corneum and block UV radiation.
Laminin V is found in the basement membrane and is not involved in the formation of the cornified
cell envelope.

116.Glomus cells are primarily found:

A. on hands/feet

56
B. on the trunk
C. on the lateral thighs
D. on the genital skin
E. on the faceQuestion 88 Explanation

Correct choice: A. on hands/feet

Glomus cells are derived from Susquet-Hoyer canals which function to shunt blood from the
arterioles to venules and are primarily found on hands and feet. There are two types of glomus
tumors, solitary and multiple. The solitary types tend to have paroxysmal pain which can be
extreme. Multiple glomus tumors can be a autosomally dominant trait with incomplete penetrance
and are less likely to be painful. Both have a predominance on the distal extremities, especially sub-
ungual. Two tests are helpful in diagnosing glomus tumors: Hildreth sign - disappearance of pain
following application of a tourniquet proximally. Love test - eliciting pain by applying pressure to a
precise area with the tip of a pencil.

117.Which cadherin is responsible for adhesion of Langerhan cells to the epidermis?

A. E-Cadherin
B. P-Cadherin
C. N-Cadherin
D. Desmoglein
E. Desmocollin

Correct choice: A. E-cadherin

E-cadherin is responsible for the adhesion of Langerhan cells to the epidermis. There are two major
subclasses of cadherins which mediate cell adhesion and play a fundamental role in normal
development, classic (E-,P-,N-cadherin) and desmosomal (desmoglein and desmocollin). They
depend on calcium for their function.

118.A 78 year-old man is diagnosed with cicatricial pemphigoid. Which of the following antigens
being implicated in his disease should trigger a work-up for malignancy?

A. Bpag2

57
B. Laminin 5
C. Laminin 6
D. Beta-4 Integrin
E. Type VII collagen

Correct choice: B. Laminin 5

Involvement of Laminin 5 would raise the suspicion for malignancy when found. ~30- 40% of cases
may have associated malignancy. Patients with antiepiligrin (laminin 5)CP indicated an increased
risk of malignancy that approximates that for adults with dermatomyositis. The risk is particularly
high in the first year of disease (E-medicine, www.emedicine.com/derm/topic79.htm). Reports of
both lung and gastric carcinomas are seen in the literature with some feeling that gastric malignancy
is most common.

119.What component is the major barrier in the stratum corneum?

A. Sebum
B. Squalene
C. Collagen
D. Ceramide
E. Triglycerides

Correct choice: D. Ceramide

The major component of lamellar granules of the keratinocytes is ceramide. These play a major role
in the barrier function of the skin.

120.Apocrine chromhidrosis results from which of the following contents of apocrine sweat?

A. Lipofuschin
B. Squalene
C. Cholesterol
D. Fatty Acids
E. Ammonia

58
Correct choice: A. Lipofuschin

Chromhidrosis refers to the secretion of pigmented sweat, most commonly yellow, green or black. It
reflects the rich lipofuschin content of apocrine sweat. Extrinsic apocrine chromhidrosis results
from staining of sweat and garments by chromogenic bacteria, such as Corynebacterium spp.

121.What is the significance of the critical line of Auber?

A. It is the location of the insertion of the erector pili muscle


B. The bulk of the mitotic activity in the hair occurs above this line
C. The inner root sheath is formed above this line
D. It is the widest diameter of the hair bulb
E. It is where keratinization first occurs in the hair

Correct choice: D. It is the widest diameter of the hair bulb

The critical line of Auber is at the widest diameter fo the hair bulb. Below this line, the bulk of
mitotic activity that gives rise to the hair and the inner root sheath occurs. The erector pili muscle
inserts in the isthmus region of the follicle. Keratinization first occurs above this line.

122.Asboe-Hansen Sign refers to:

A. When an intact epidermis shears away from the underlying dermis, leaving a moist
surface
B. Spreading bulla phenomenon with pressure on an intact bulla
C. Swollen, itchy and or red after stroking the skin
D. Central depression within a lesion when squeezed along its margins
E. Disappearance of color when the lesion is pressed

Correct choice: B. Spreading bulla phenomenon with pressure on an intact bulla

Spreading bulla phenomenon with pressure on an intact bulla is referred to as Asboe-Hansen sign,
commonly seen with pemphigus vulgaris. Nikolsky sign can be seen when an intact epidermis
shears away from the underlying dermis, leaving a moist surface (seen in pemphigus vulgaris,

59
staphylococcus scalded skin syndrome (SSSS), and toxic epidermal necrosis). Swollen, itchy and or
red after stroking the skin is referred to Darier’s sign and can be seen in systemic
mastocytosis or urticaria pigmentosa. Central depression within a lesion when squeezed is referred
to as the dimpling sign and is seen in dermatofibromas. Disappearance of color or blanching when
the lesion is pressed is commonly found on vascular lesions

123.Anagen effluvium is

A. Rarely seen following administration of cancer chemotherapeutic agents


B. Stimulus induces the abrupt cessation of mitotic activity in the rapidly dividing hair
matrix cells
C. Occurs within 24 to 48 hours of exposure
D. Entirely irreversible
E. Patients being treated with nitrosurea agents are usually spared

Correct choice: B. Stimulus induces the abrupt cessation of mitotic activity in the rapidly dividing
hair matrix cells

It is frequently seen following administration of cancer chemotherapeutic agents. Stimulus induces


the abrupt cessation of mitotic activity in the rapidly dividing hair matrix cells. Occurs within days
to weeks following exposure to the inciting agent and is entirely reversible. Frequent causes include
antimetabolites, alkylating agents, miotic inhibitors, thallium, boron. Frequent causes include
doxorubicin, nitrosureas and cyclophosphamide.

124.Sebaceous glands secrete sebum via:

A. Holocrine mechanism
B. Exocrine mechanism
C. Endocrine mechanism
D. Exostosis
E. Mecrocrine mechanism

Correct choice: A. Holocrine mechanism

60
The sebaceous lobules have basal germinative cells and central sebocytes, which gradually become
more distended with lipid vacuoles until they are shed into the lumen (holocrine secretion).

125.Homocystinuria has abnormal crosslinking of collagen because of a mutation in:

A. Cystathione synthase
B. Type i collagen n-peptidase gene
C. Lysyl hydroxylase
D. Tenascin x
E. Lysyl oxidase

Correct choice: A. Cystathione synthase

Homocystinuria is caused by a mutation in cystathione synthase. The main skin findings are a malar
flush, livedo reticularis and leg ulcerations. A characteristic eye finding is the downward
displacement of the lens. The other options are involved in abnormalities associated with Ehlers-
Danlos syndrome. Lysyl hydroxylase is deficient in Kyphoscoliosis type of EDS. Tenascin X is
involves in ~3% of Classical type EDScases. Dermatosparaxis type EDS has recessive mutations in
the type I collagen N-peptidase gene.

126.During a salt split skin test, if the location of the deposition if found
on the roof€™ (lamina lucida), which of the following could be a diagnosis?

A. Bullous pemphigoid
B. Anti-epiligrin cicatricial pemphigoid
C. Epidermolysis bullosa acquisita
D. Bullous eruption of lupus erythematosus
E. Bullous eruption of tinea

Correct choice: A. Bullous pemphigoid

Bullous pemphigoid is one of the many bullous diseases that may be found on the
‘roof’ of the blister, the others include pemphigoid gestationis, linear IgA bullous
dermatosis, cicatricial pemphigoid. The other answer (except for bullous tinea, in which a salt split

61
would not be performed) are those found on the ‘floor’ of the blister (lamina
densa).

127.Which of the following polypeptides is found in the lam


ina lucida?

A. Plakoglobin
B. Desmoplakin
C. Keratocalmin
D. Demoyokin
E. Laminin 5

Correct choice: E. Laminin 5

Laminin-5 is a basement membrane extracellular matrix protein that mediates attachment substrate
for both adhesion and migration in a wide variety of cell types, including epithelial cells,
fibroblasts, neurons and leukocytes and is a preferred adhesion substrate for epithelial cells
(Koshikawa et al., 2001). The remaining listed items are part of the desmosomal plaque in the
epidermis.

128.When do melanocytes begin to synthesize melanin?

A. 2nd month of gestation


B. 3rd month of gestation
C. 4th month of gestation
D. 5th month of gestation
E. 6th month of gestation

Correct choice: B. 3rd month of gestation


Melanocytes begin to synthesize melanin in the 3rd month of gestation.

62
129.For the treatment of drug induced linear IgA bullous dermatosis, after stopping offensive
medication, what other treatments may be used?

A. Interferon alpha
B. Interferon gamma
C. Psoralen plus uva
D. Interleukin 2
E. Dapsone

Correct choice: E. Dapsone

Dapsone may be used to treat patients with drug induced linear IgA bullous dermatosis which
persist after the offending medication has been discontinued. The other medications listed are rare
causes of drug induced linear IgA bullous dermatosis and so should not be used for treatment.

130.Granulation tissue primarily contains:

A. Collagen I
B. Collagen III
C. Collagen IV
D. Fibrin
E. Collagen VII

Correct choice: B. Collagen III

Granulation tissue begins to form four days after injury and is composed of new capillaries,
macrophages, fibroblasts, blood vessels and primarily collagen type III. Formulation of granulation
tissue is dependent on the presence of fibronectin and progresses through an orderly sequence of
matrix deposition: fibronectin to collagen III then finally collagen I.

131.The antibody target in ocular cicatricial pemphigoid is also mutated in:

a. Junctional epidermolysis bullosa, Herlitz type


B. Recessive dystrophic epidermolysis bullosa

63
C. Junctional epidermolysis bullosa with myotonic dystrophy
D. Dominant dystrophic epidermolysis bullosa
E. Junctional epidermolysis bullosa with pyloric atresia

Correct choice: E. Junctional epidermolysis bullosa with pyloric atresia

JEB with pyloric atresia and ocular cicatricial pemphigoid both have mutations in Beta4integrin.
Herlitz type JEB is due to laminin 5. Recessive and dominant dystrophic EB is due to COL17A1
(type VII collagen). JEB with myotonic dystrophy is due to plectin.

132.Which of the following is true regarding the development of hair follicles?

A. The first primordial hair follicles form at 15 weeks gestation


B. The first hair follicles form on the scalp and eyelashes
C. Follicles develop in a cephalad to caudal direction
D. New follicles develop during the first 3 months postpartum
E. The eyebrows develop late in gestation

Correct choice: C. Follicles develop in a cephalad to caudal direction

The first primordial hair follicles form at 9 weeks gestation on the eyebrows, upper lip, and chin.
The remaining follicles develop at 4-5 months in a cephalad to caudal direction. New follicles
cannot develop in adult skin.

133.What does a western blot identify?

A. Protein
B. RNA
C. DNA
D. Genes
E. Cell membranes

Correct choice: A. Protein

64
A northern blot identifies RNA and a southern blot identifies DNA.

134.Sneddon Wilkinson disease is caused by a defect in what antigen?

A. Desmocollin 1
B. Desmoglein 1
C. Desmogelin 3
D. Bpag1
E. Bpag2

Correct choice: A. Desmocollin 1

Sneddon- Wilkinson disease, also known as sub-corneal pustular dermatosis is due to a defect in
desmocollin 1. Desmoglein 1 defect is seen in pemphigus foliaceous and staph scalded skin
syndrome. Desmoglein 3 defect is seen in pemphigus vulgaris and intraepidermal neutrophilic IgA
dermatosis. BPAg1 is seen in paraneoplastic pemphigus and bullous pemphigoid. BPAg2 is seen in
bullous pemphigoid, cicatricial mucous membrane pemphigoid and linear IgA disease.

135.Apocrine glands:

A. Demonstrate holocrine secretion


B. Demonstrate decapitation secretion
C. Are fully functional at birth
D. Are diffusely distributed on the body
E. Are thermoregulatory

Correct choice: B. Demonstrate decapitation secretion

Apocrine glands show decapitation secretion. Like eccrine glands, apocrine glands are composed of
three segments, the intraepidermal duct, the intradermal duct, and the secretory portion. The duct of
the apocrine gland usually leads to a pilosebaceous follicle above the entrance of the sebaceous
duct. Apocrine glands are found in the axillae, anogenital region, external ear canal (ceruminous
glands), in the eyelids (Moll’s glands), and in the breast (mammary glands). Apocrine glans are
functional only at puberty. Their initial secretion is odorless.

65
136.Which protein is the largest component of the cornified cell envelope?

A. Keratin
B. Involucrin
C. Profilaggrin
D. Loricrin
E. Ceramide

Correct choice: D. Loricrin

The cornified cell envelope is a durable, protein-lipid polymer that eventually acts as a mechanical
and chemical barrier on the exterior of cornified cells. In the upper spinous layer, keratohyaline
granules release profilagrin and loricrin. Profilaggrin is cleaved to filaggrin and subsequently
aggregates keratin filaments. Loricrin is the major protein component of the cornified cell envelope
and is bound to the cell membrane by transglutaminases (in addition to other structural proteins like
involucrin, keratins, elafin, cystatin A and desmosomal peptides). This forms the
highly insoluble proteinaceous component of the cornified cell envelope.

137.Acid keratins (K10-20) are expressed on which of the following chromosomes?

A. 17
B. 18
C. 19
D. 10
E. 12

Correct choice: A. 17

Acid keratins are expressed on chromosome 17 and basic keratins on chromosome 12. The other
listed options do not have active keratin functions.

138.Defects in what kind of structural protein lead to pyloric atresia


associated with junctional epidermolysis bullosa:

66
A. Collagen
B. Elastin
C. Loricrin
D. Integrin
E. Plectin

Correct choice: D. Integrin

Junctional epidermolysis bullosa with pyloric atresia involves a defect in the b4 subunit of the a6b4
integrin. The expression of this protein is limited to the basal layer of the epidermis. This integrin is
a transmembrane protein that coordinates a link between the intermediate filaments (keratins) and
the extracellular matrix of the basement membrane. The b4 domain mediates an interaction with
both plectin and BP180; its absence prevents hemidesmosomal assembly.

139.Which of the following cells are required for wound healing?

A. Neutrophil
B. Macrophage
C. Eosinophil
D. Langerhans cell
E. Lymphocyte

Correct choice: B. Macrophage

The macrophage is required for wound healing. The macrophages debride tissue, secrete
collagenase and stimulate expression of FGF, IL-1, TGF-beta, PDGF and TGF-alpha thus
facilitating transition from inflammation to repair.

140.Which of the following options is characteristic of integrins?

A. Defects in the alpha-6 domain of integrin result in epidermolysis bullosa simplex with
muscular dystrophy

67
B. Alpha-6-beta-4 integrin is found at sites where desmogleins attach
C. Expression is seen in all layers of the epidermis
D. The extracellular alpha6 domain binds collagen 7
E. These proteins coordinate linkage between intermediate filaments and extracellular
matrix of the basement membrane

Correct choice: E. These proteins coordinate linkage between intermediate filaments and
extracellular matrix of the basement membrane

Defects in the BETA-4 (not alpha-6) domain of integrin result in junctional epidermolysis bullosa
with pyloric atresia. Its expression is seen in the basal cell layer and binds to laminins.

141.Desmoglein 1 is the antigen in which of the following autoimmune diseases of the skin:

A. Pemphigus foliaceus
B. Bullous impetigo
C. Dermatitis herpetiformis
D. Bullous pemphigoid
E. Pemphigoid gestationis

Correct choice: A. Pemphigus foliaceus

The antigen implicated in pemphigus foliaceus is Desmoglein 1. Desmoglein 1 is targeted in


bullous impetigo, but this is an infectious condition, not an autoimmune disease. The antigen in
dermatitis herpetiformis is transglutaminase 3. The antigens for both bullous pemphigoid and
pemphigoid gestationis are BPAG1 and BPAG2.

142.Which of the following keratins would most likely be expressed in the nail bed?

A. K6a/16
B. K6b/17
C. K1/9
D. K2e/10
E. K4/13

68
Correct choice: B. K6b/17

Keratins 6b & 17 are expressed in the nail bed. K6a/16 is expressed in the outer root sheath and in
hyperproliferative keratinocytes, 1/9 in palmoplantar suprabasalar keratinocytes, 2e/10 in the upper
spinous and granular cell layers, and 4/13 are expressed in mucosal epithelium.

143.Which of the following abnormalities would be present in a patient with Harlequin ichthyosis?

A. Lamellar granules are uniformly absent


B. Keratohyaline granules are normal in all types of harlequin fetus
C. Demoyokin mutation
D. Band 6 protein is absent
E. Plakoglobin is abnormal

Correct choice: A. Lamellar granules are uniformly absent

In Harlequin ichthyosis, the lamellar granules are uniformly abnormal or absent. On electron
microscopy, there is no evidence of the lipid lamellae which form due to lamellar granule discharge
into the region between the granular and cornified cells (Fitz v6, p493). This is suggestive of a
primary defect in lipid synthesis and protein dephosphorylation resulting in faulty lamellar body
formation and secretion (Bolognia, ch. 57). Harlequin fetus/ichthyosis can be divided into three
types based on the microscopic appearance of the keratohyaline granules. Type I - normal, Type II -
too small to be seen by light microscopy, Type III - absent keratohyaline granules. Demoyokin,
band-6 protein (a plaque component) and plakoglobin are desmosomal proteins and not involved in
Harlequin ichthyosis. Recently, the gene defect was localized a defect in the ABCA12 protein.

144.Nikolsky sign can be seen when:

A. When an intact epidermis shears away from the underlying dermis, leaving a moist
surface
B. Spreading bulla phenomenon with pressure on an intact bulla
C. Swollen, itchy and or red after stroking the skin
D. Central depression within a lesion when squeezed along its margins
E. Disappearance of color when the lesion is pressed

69
Correct choice: A. When an intact epidermis shears away from the underlying dermis, leaving a
moist surface

Nikolsky sign can be seen when an intact epidermis shears away from the underlying dermis,
leaving a moist surface (seen in pemphigus vulgaris, staphylococcus scalded skin syndrome (SSSS),
and toxic epidermal necrosis). Spreading bulla phenomenon with pressure on an intact bulla is
referred to as Asboe-Hansen sign, commonly seen with pemphigus vulgaris. Swollen, itchy and or
red after stroking the skin is referred to Darier’s sign and can be seen in systemic
mastocytosis or urticaria pigmentosa. Central depression within a lesion when squeezed is referred
to as the dimpling sign and is seen in dermatofibromas. Disappearance of color or blanching when
the lesion is pressed is commonly found on vascular lesions

145.Acral melanomas are particularly concerning because they:

A. Are diagnosed at a later stage


B. Invade perineurally
C. Cannot be resected
D. Metastasize frequently even at shallow breslow depth
E. Do not respond to ipilimumab

Correct choice: A. Are diagnosed at a later stage

Acral melanoma is problematic because it is diagnosed at a later stage. It accounts for 5-10% of all
melanomas.

146.Keratinocytes in the basal layer of the epidermis attach to the basement membrane zone at
hemidesmosomes via what intermediate filament molecules?

A. Keratins 1 and 10
B. Keratins 5 and 14
C. Desmogleins 1 and 3
D. Desmoplakin and desmoglobin
E. Bpag1 and bpag2

70
Correct choice: B. Keratins 5 and 14

Keratins 5 and 14 are intermediate filaments found in the basal layer of the epidermis that play a
role in attaching basal layer keratinocytes to the basement membrane zone. Keratins 1 and 10 are
found in more superficial layers of the epidermis. Desmogleins, desmoplakin, and desmoglobin are
all components of desmosomes which attach keratinocytes to other surrounding keratinocytes.
BPAG1 and BPAG2 are basement membrane proteins.

147.The proteins that make up the cornified cell envelope are synthesized in the:

A. Stratum corneum
B. Stratum basale
C. Melanocytes
D. Stratum granulosum
E. Langerhans cells

Correct choice: D. Stratum granulosum

The proteins that make up the cornified cell envelope are synthesized in the spinousand granular
cell layer. The stratum basale is not involved in production of these proteins and there is no protein
synthesis in the stratum corneum. Melanocytes and Langerhans cells are components of the
epidermis but are not involved in this process.

148.Which keratins are expressed in the stratum germinativum and are present but not made de
novo in the stratum spinosum?

A. K4, 13
B. K1,10
C. K2e, 10
D. K3,12
E. K5,14

Correct choice: E. K5,14

71
K5 and K14 are expressed in the stratum germinativum (basal layer) and are
defective in epidermolysis bullosa simplex. They are still present in the stratum spinosum but are
not made de novo in this layer. K4 and 13 are found in mucosal epithelium. K1 and 10 are
expressed in suprabasal keratinocytes, K2e and 10 in the upper spinous and granular layers, and K3
and K12 in the cornea.

149.Fibrofolliculomas are associated with:

A. Birt-hogg-dube syndrome
B. Cowdens
C. Cutis laxa
D. Marfans
E. Brook spiegler

Correct choice: A. Birt-Hogg-Dube syndrome

Fibrofolliculomas are associated with Birt-Hogg-Dube Syndrome and not in the other answer
choices.

150.Which of the following skin conditions involves elastin?

A. Rothmund-thompson
B. Epidermolytic hyperkeratosis
C. Pseudoxanthoma elasticum
D. Progeria
E. Classic type ehlers-danlos syndrome

Correct choice: C. Pseudoxanthoma elasticum

There are many genodermatoses that involve elastin. Pseudoxanthoma elasticum shows increased
glycosaminoglycans on elastic fibers, calcium deposition and accumulation of fragmented and
calcified elastic fibers. Cutis laxa shows decreased desmosine and lysyl oxidase. In Marfan sydrome
there is decreased fibrillin I and fragmentation of elastic fibers. Buschke-Ollendorf syndroms shows

72
increased desmosine and increased amount of thickend elastic fibers. Other elastin disease are
congenital contractural arachnodactyly from a mutation in fibrillin II and anetoderma
which shows decreased desmosine and loss/fragmentation of elastic fibers. Classic type Ehlers-
Danlos syndrome is caused by a defect in collagen V and not in elastin. Progeria is caused by a
defect in lamin A. EHK is caused by a defect in keratin 1/10. Rothmund Thompson is caused by a
defect in RecQL4, a helicase gene.

151.Which mechanoreceptor found in hair bearing areas sense deep touch and vibration?

A. Merkel cell
B. Meissner corpuscle
C. Vater-pacini corpuscle
D. Krase end-bulb
E. Free nerve ending

Correct choice: C. Vater-pacini corpuscle

Vater-Pacini corpuscles sense deep touch and vibration. Merkel cells are slow adapting type I
mechanoreceptors found among basal keratinocytes. Meissner corpuscles are found in the dermal
papilla, especially in the palms and soles. Krase end-bulbs are mucocutaneous end-organs found on
the glans penis, prepuce, clitoris, labia minora, and vermillion border of the lip.

152.A patient presents with painless firm nodules. Pathology reveals no epidermal involvement.
The patient is ultimately diagnosed with sarcoid. What is this type of involvement called?

A. Darier-roussy disease
B. Lofgren’s syndrome
C. Heerfordt’s syndrome
D. Lupus pernio
E. Kveim-siltzbach disease

Correct choice: A. Darier-roussy disease

73
Darier Roussy disease refers to subcutaneous sarcoid. Lofgren’s syndrome is sarcoid presenting
as erythema nodosum, hilar lymphadenopathy, fever, migrating polyarthritis, and acute iritis.
Heerfordt’s is parotid gland enlargement, uveitis, fever and cranial nerve palsies. Kveim-
Siltzbach antigen is used rarely in the US but can aid in the diagnosis of sarcoid.

153.Which keratins are upregulated in hyperproliferative disease such as psoriasis?

A. Keratins 1 and 10
B. Keratins 2e and 10
C. Keratins 5 and 14
D. Keratins 6 and 16
E. Keratins 8 and 18

Correct choice: D. Keratins 6 and 16

Keratins 6 and 16 are upregulated in hyperproliferative keratinocytes, outer root sheath, and oral
epithelium. A defect in these keratins may result in pachyonychia congenita.

154.Which of the following is a member of the armadillo family of linking proteins?

A. E-cadherin
B. Periplakin
C. Envoplakin
D. Desmocollin
E. Plakoglobin

Correct choice: E. Plakoglobin

Plakoglobin is an example of an armadillo protein, which links the cytoskeleton associated linking
proteins (such as plakins in the case of intermediate filaments and alpha-catenin in the case of actin)
to the transmembrane adhesion molecules, termed cadherins. E-cadherin and desmocollin are
examples of cadherins. Periplakin and envoplakin are examples of plakin linking proteins.

74
155 -A specific marker of Merkel cells is:

A. Cytokeratin 10
B. Cytokeratin 15
C. Cytokeratin 20
D. Loricrin
E. Envoplakin

Correct choice: C. Cytokeratin 20

Cytokeratin 20 is a specific marker for the Merkel cell. Merkel cells are
mechanoreceptors located at body sites requiring high tactile sensitivity. Keratinocyte deformation
results in a secretion of chemokines by Merkel cells, which make synaptic connection with neurons.

156.In the epidermis, the cell most responsible for antigen detection and processing is the:

A. Keratinocytes
B. Merkel cell
C. Melanocyte
D. Langerhans cell
E. Cd4+ t cell

Correct choice: D. Langerhans cell

The Langerhans cell is a bone narrow-derived, antigen-presenting cell found in all layers of the
epidermis, oral mucosa, esophagus, and vagina. Langerhans cells ingest and process antigens,
mature, migrate to a local lymph node, and then present the antigen to a na�ve (or resting) T cell,
activating that T cell. The Langerhans cell is central to the pathogenesis of atopic dermatitis,
psoriasis, allergic contact dermatitis, and certain infections, such as Leishmaniasis.

157.In which of the following locations would you be least likely to identify
melanocytes?

A. Stria vascularis of the ear

75
B. Iris
C. Leptomeninges
D. Retina
E. Pericardium

Correct choice:E. Pericardium

Melanocytes are found in the stria vascularis of the ear, iris, leptomeninges and retina. There are no
normal populations of melanocytes found in the pericardium.

158.Darier's sign is described as:

A. When an intact epidermis shears away from the underlying dermis, leaving a moist
surface
B. Spreading bulla phenomenon with pressure on an intact bulla
C. Swollen, itchy and or red after stroking the skin
D. Central depression within a lesion when squeezed along its margins
E. Disappearance of color when the lesion is pressed

Correct choice: C. Swollen, itchy and or red after stroking the skin

Swollen, itchy and or red after stroking the skin is referred to Darier's sign and can be seen in
systemic mastocytosis or urticaria pigmentosa. Nikolsky sign can be seen when an intact epidermis
shears away from the underlying dermis, leaving a moist surface (seen in pemphigus vulgaris,
staphylococcus scalded skin syndrome (SSSS), and toxic epidermal necrosis). Spreading bulla
phenomenon with pressure on an intact bulla is referred to as Asboe-Hansen sign, commonly seen
with pemphigus vulgaris. Central depression within a lesion when squeezed is referred to as the
dimpling sign and is seen in dermatofibromas. Disappearance of color or blanching when the lesion
is pressed is commonly found on vascular lesions

159.The embryonic periderm becomes part of the :

A. Vernix caseosa
B. Stratum corneum

76
C. Stratum basale
D. Dermis
E. Hair follicle

Correct choice: A. vernix caseosa

During the second trimester of fetal development, the periderm is sloughed frommost of the skin
surface revealing the underlaying epidermis. The periderm becomes part of the protective coating,
vernix caseosa, together with the shed lanugo, sebum and other amniotic fluid materials. Bolognia
p.38

160.Which of the following statements about Laminins is correct?

A. Laminins span from the plasma membrane of basal keratinocytes to the lamina lucida
B. Laminins provide little structural support in the basement membrane
C. Laminins provide signaling molecules that interact with other proteins to transmit
morphogenetic information to the cell's interior
D. Laminin 5 is also called plectin
E. Laminin 5 is the only laminin found in the basement membrane

Correct choice: C. Laminins provide signaling molecules that interact with other proteins to
transmit morphogenetic information to the cell's interior

Laminins do transmit signals to the cell's interior. They span from the plasma membrane of basal
keratinocytes to the lamina DENSA, not lucida. Laminin 5 (also known as epiligrin) is part of a
structural network in the basement membrane that also includes laminin 6.

161.Which of the following elements is necessary for melanin production?

A. Copper
B. Selenium
C. Iron
D. Zinc
E. Calcium

77
Correct choice: A. Copper

Melanin is synthesized from tyrosine via the action of the enzyme tyrosinase (tyrosine to DOPA to
DOPAquinone to melanin). Tyrosinase is a copper containing enzyme. The other elements are not
specifically required for melanin synthesis.

162.Once a keratinocyte leaves the basal cell layer, the normal transit time to stratum corneum is at
least:

A. 7 days
B. 14 days
C. 21 days
D. 28 days
E. 35 days

Correct choice: B. 14 days

Once a basal cell leaves the basal layer in humans, normal transit time
to stratum corneum is at least 14 days. Transit time through the stratum corneum to desquamation
requires 14 more days.

163.Regarding the stratum germinativum (basale):

A. Intermediate filaments in basal cells insert into only hemidesmosomes


B. Keratins 1 and 10 are expressed
C. Not all basal cells have the potential to divide
D. Microfilaments assist in downward movement of cells
E. Plectins regulate adhesion and initiation of differentiation.

Correct choice: C. Not all basal cells have the potential to divide

Not all basal cells have the potential to divide. Stem cells give rise to transient amplifying cells
which give rise to the epidermal keratinocytes. Intermediate filaments in basal cells insert into

78
desmosomes and hemidesmosomes. Keratins 5/14 are predominantly expressed. Microfilaments
assist in upward movement of cells. Integrins regulate adhesion and initiation of differentiation.

164.Each of the following is true about the basement membrane zone except:

A. Anchoring filaments attach the basal cell membrane to the lamin lucida
B. Can be visualized on light microscopy with pas staining
C. Lamina densa is composed of type iv collagen
D. Contains laminin 1 and laminin 5
E. Anchoring fibrils are composed of type vii collagen

Correct choice: A. Anchoring filaments attach the basal cell membrane to the lamin lucida

The basement membrane zone is seen on staining with PAS stain. It appears as a homogenous band
approximately 1 micron thick at the dermo-epidermal junction. The hemidesmosomal complex and
basement membrane zone play an integral role in maintaining cellular adhesion. Anchoring
filaments (primarily composed of laminin5 and BPAG2) attach the basal cell membrane to the
lamina densa NOT lamina lucida.

165.Regarding the stratum spinosum, which of the following is correct?

A. No keratin 1/10 is present


B. New synthesis of k5/14 occurs in this layer
C. The "spines" seen on pathology are due to desmosomal connections between
keratinocytes
D. This layer contains melanocytes
E. This layer contains the cornified cell membrane

Correct choice: C. The "spines" seen on pathology are due to desmosomal connections between
keratinocytes .

The "spines" seen on pathology are due to desmosomal connections between keratinocytes, NOT
hemidesmosomes. The hemidesmosomes are present in the cells at the base of the basal layer and

79
are part of the connection between the epidermis and basement membrane. Keratin expression
continues in the spinous layer. Keratins 1/10 are synthesized, not keratins 5/14. Keratin 5/14,
however, is still present. Keratohyaline granules are typically seen in the granular layer.

166.Choose the correct answer regarding melanin and skin color:

A. In black and brown skin the melanosomes are smaller in diameter and length
B. Facultative skin color is the amount of cutaneous melanin pigment generated according
to cellular genetics
C. In white skin the melanosomes form groups within the secondary lysosomes
D. Eumelanin produces a yellow chromophore
E. The number of melanocytes increases with one exposure to UVA/visible light

Correct choice: C. In white skin the melanosomes form groups within the secondary lysosomes

Melanocytes of dark skin synthesize melanosomes larger than those produced in light skin. The
number of melanocytes in the epidermis is the same, regardless of the person's race or color: it is the
number and size of the melanosomes or pigment granules, continuously synthesized by these
melanocytes, that determine differences in skin color. The size of the melanosome is the principle
factor in determining how the melanosomes will be distributed within the keratinocytes. The larger
the melanosomes of dark skin are individually dispersed within the cytoplasm of keratinocytes:
smaller melanosomes of light skin are packaged in membrane-Bound complexes within
keratinocytes. Eumelanin is in dark oval melanosomes found in black hair

167.As you move upward through the epidermis toward the stratum corneum, which of the
following is true?

A. Calcium increases, phospholipids decrease, sphingolipids decrease


B. Calcium increases, phospholipids increase sphingolipids increase
C. Calcium increases, phospholipids decrease, sphingolipids increase
D. Calcium decreases, phospholipids decrease, sphingolipids decrease
E. Calcium decreases, phospholipids increase sphingolipids decrease

Correct choice: C. calcium increases, phospholipids decrease, sphingolipids increase

80
Calcium increases, phospholipids decrease, sphingolipids increase. Calcium is needed for
desmosome formation and enzyme activation. Phospholipid content of epidermis decreases with
differentiation but neutral lipids and sphingolipids (ceramide) increase with differentiation.

168.Pick the correctly paired keratin with its structure:

A. K1/k10 - basal cells


B. K3/k12 - esophagus
C. K4/k13 - cornea
D. K5/k14 - suprabasal cells
E. K1/k9 - Palms and soles

Correct choice: E. K1/K9 - palms and soles

Keratin 1 and 10 are found in the stratum spinosum. K5 and K14 are found in the basal layer. K3
and K12 are found in the suprabasilar cells of the cornea. K4 and K13 are found in the non-
cornifying cells of stratified mucosa. K1 and K9 are found in the palms and soles.

169.Which of the following groups of adhesion proteins are found in both the hemidesomsome-
anchoring filament complexes and lamina densa?

A. Plectin
B. Heparin sulfate proteoglycan
C. Laminin 5
D. Nidogen
E. Type vii collagen

Correct choice: C. Laminin 5

Laminin 5 is found both in the hemidesomsome-anchoring filament complexes and lamina densa.
Plectin is found in the hemidesomsome-
anchoring filament complexes.
Heparin sulfate proteoglycan is found only in the lamina densa, collagen VII in the sublamina
densa, and nidogen in the lamina densa.

81
170.All of the following are consider intermediate filament except:

A. Microtubules
B. Keratins
C. Vimentin
D. Desmin
E. Peripherin

Correct choice: A. Microtubules

Intermediate filaments are composed of keratins, vimentin, desmin, peripherin, neurofilaments,


nuclear laminins, and nestin. These are part of cytoskeletal elements.

171.Mutations in which of the following genes will produce red hair?

A. MC1-R
B. Agouti
C. Hairless
D. C-kit
E. Tyrosinase

Correct choice: A. MC1-R

MC1-R is the receptor that binds alpha-MSH, and ACTH>alpha-MSH. Defects in this receptor will
produce a phenotype including red hair. Agouti is not involved in this process. Agouti describes the
banding of hairs seen in some mammals such as dogs, foxes, and mice, which is due to alternating
production of eumelanin and pheomelanin occurs. This has not been described in humans
(Bolognia, p942). Thehairless gene is associated with alopecia totalis. C-kit mutations are
associated with piebaldism and urticaria pigmentosa. Tyrosinase defects are associated with Type I
Oculocutaneous albinism.

172.Surgery, Parturition, Fever, Kwashiokor and Hypervitaminosis A are all causes of:

A. Telogen effluvium

82
B. Anagen effluvium
C. Alopecia areata
D. Androgenetic alopecia
E. Both telogen and anagen effluvium

Correct choice: A. Telogen effluvium

All of the listed stressors can induce telogen effluvium. Other causes include traction and some drug
exposures. Usually will only involve up to 50% of scalp hairs and will resolve within 2-3 months.

173.Which of the following glands is not under neural control?

A. Sebaceous glands
B. Apocrine glands
C. Eccrine glands
D. Salivary glands
E. Molls glands

Correct choice: A. Sebaceous glands

Sebum is secreted continuously on the skin under hormonal glands. It contains squalene,
cholesterol, cholesterol esters, wax, and triglycerides.

174.Mutations in which of the following proteins results in epidermolysis bullosa simplex


associated with muscular dystrophy:

A. Uncein
B. Plectin
C. A6b4 integrin
D. Laminin 5
E. Collagen type iv

Correct choice: B. Plectin

83
EBS with muscular dystrophy is due to a plectin mutation. There are many types of EBS. Anchoring
filaments exist within the lamina lucida. They are primarily comprised of laminin 5 and BP180.
Laminin 5 is a cross-shaped assembly of 3 classes of polypeptides, a, b, g. The anchoring filaments
function as a structural network to which other proteins attach, and they function as signaling
molecules that transmit morphogenetic information to transmembrane proteins of the basal cell
layer (such as the integrins).
Laminin 5 is also called epiligrin and binds to the a6b4 integrin at the hemidesmosome.

175.Sebaceous glands secrete sebum through which of the following secretory mechanisms?

A. Holocrine
B. Merocrine
C. Apocrine
D. Holocrine and merocrine
E. Holocrine and apocrine

Correct choice: A. Holocrine

Sebaceous glands exhibit holocrine secretion, whereby the sebocytes disintegrate in transit to the
gland center, releasing their sebum contents. Merocrine secretion refers to the formation of
intracellular secretory vesicles that translocate to the apical cell surface for secretion. Apocrine
secretion refers to the process whereby secretory contents are packaged using the apical cell
membrane, and 'pinched off' to achieve secretion.

176.Type VII collagen in found in anchoring fibrils and also in:

A. Fetal skin
B. Bone
C. Amnion
D. Aorta
E. Blood vessels

Correct choice: C. Amnion

84
Type VII collagen is present in anchoring fibrils and amnion. Fetal skin and blood vessels contain
type III collagen. The aorta contains type VI collagen. Bone contains type I collagen.

177.Keratin filaments in basal cells insert into:

A. Desmosomes
B. Adherens junctions
C. Connexins
D. Lamellar granules
E. Odland bodies

Correct choice: A. Desmosomes

Keratin filaments insert into desmosomes and hemidesmosomes in the basal cell layer and into
desmosomes in the layers above. Connexins do not bind keratin filaments. Odland bodies and
lamellar granules are synonyms. These are a "membrane bounded organelle, specialized for the
storage and secretion various substances (surfactant phospholipids, glycoproteins and acid
phosphates) which are arranged in the form of tightly packed, concentric, membrane sheets or
lamellae. Has some similar properties to, but is distinct from, a lysosome." Desmosomes are multi-
protein complexes that function as cell–cell adhesion structures (junctions) in epidermal cells. They
also provide attachment sites for the keratin intermediate filament cytoskeleton of keratinocytes.
Consequently, these junctions are critical components of a supracellular filament network that
traverses the interfollicular epidermis and the epithelia of skin appendages, such as hair follicles.

178.Moving from internally to externally choose the correct description of the hair follicle:

A. Inner root sheath cuticle - huxley's layer - henle's layer - medulla - cortex- hair shaft
cuticle
B. Hair shaft cuticle - cortex - medulla - henle's layer - huxley's layer _ inner root sheath
cuticle
C. Henle's layer - huxley's layer - inner root sheath cuticle - hair shaft cuticle - cortex -
medulla
D. Medulla _ cortex _ hair shaft cuticle _ inner root sheath cuticle _ huxley's layer _ henle's
layer

85
E. Inner root sheath cuticle _ outer root sheath cuticle _ hair shaft cuticle _ cortex _
huxley's layer _ henle's layer

Correct choice: D. Medulla _ cortex _ hair shaft cuticle _ inner root sheath cuticle _ Huxley's layer
_ Henle's layer (As described in correct choice)

179- upon presentation of an antigen in the skin surface, a hapten forms. The first cell to take up the
hapten is:

A. B cells
B. Langerhans cells
C. Keratinocytes
D. T cells
E. Mast cell

Correct choice: B. Langerhans cells

Most of the contact allergens are low-molecular weight chemicals, which after penetrating into the
skin, have to couple with host proteins to be able to act as full antigens. These are called haptens.
Upon epicutaneous application to a naive host, Langerhans cells take up the hapten, Process it and
migrate towards the regional lymph nodes, where the anitgen is presented to the naive T cells.

180.Merkel cells are mechanoreceptors found in areas of high-tactile sensitivity. This


immunohistochemical marker is restricted to Merkel cells in
the skin and is thus a reliable marker for these cells:

A. Keratin 7
B. Keratin 20
C. S-100
D. Factor xiiia
E. Lyve-1

86
Correct choice: B. Keratin 20

Keratin 20 is reliable immunohistochemical markers for Merkel cells as it is restricted to these cells
in the skin. Keratin 7 can be used as a marker for Paget's Disease. S- 100 is frequently used to stain
neural cells and melanocytes. Factor XIIIa can be used to differentiate a dermatofibroma from
dermatofibroma sarcoma pertuberans (positive in DF; negative in DFSP). LYVE-1 is a marker for
lymphatics.

181.Which cell type is required for wound healing?

A. Macrophage
B. Lymphocyte
C. Mast cell
D. Dermal dendrocyte
E. Neutrophil

Correct choice: A. Macrophage

The macrophage is required for wound healing. Without macrophages, there is no healing.
Macrophages debride tissue through phagocytosis and digestion of organisms, tissue debris and
effete PMN's. They secrete
collagenase and secrete growth factors that facilitate transition from inflammation to repair.

182.Odland bodies:

A. Contain keratins
B. Are found intercellularly in the basal cell layer
C. Are exclusively intracellular
D. Crosslink with keratins 5 and 14
E. Are secretory granules with features of lysosomes

Correct choice: E. Are secretory granules with features of lysosomes

87
Odland bodies (AKA lamellar granules) are lamellated bodies containing ceramide, are found
intercellularly in upper level keratinocytes; they discharge contents into the extracellular space at
the junction of the granular and horny layers, establish a barrier to water loss, and with filaggrin
mediate stratum corneum adhesion. Recent studies suggest that they are a type of secretory granule
with features of lysosomes. (They contain lamellar-body-derived enzymes that are important in
desquamation.)

183.Which signaling molecule mediates the transition of hair cycling from telogen to anagen phase?

A. Foxn1
B. Fgf5
C. Sonic hedgehog
D. Dihydotestosterone
E. 5a-reductase

Correct choice: C. Sonic hedgehog

Sonic hedgehog (Shh), a signaling molecule secreted by ectodermal cells of the developing hair
follicle, appears to be critical in mediating the transition from telogen to anagen during postnatal
hair cycling

184.Which of the following is true regarding BPAg1?

A. It is pathogenic in cicatricial pemphigoid


B. It is a member of the plakin family
C. It is pathogenic in pemphigoid gestationis
D. It is not pathogenic in paraneoplastic pemphigus
E. It coprecipitates with plakoglobin

Correct choice: B. It is a member of the plakin family

BPAg1 is a member of the plakin family, which includes envoplakin, periplakin, desmoplakin,
plectin, and BPAg1. BPAg2 is pathogenic in cicatricial pemphigoid and pemphigoid gestationis.

88
BPAg1 is pathogenic in paraneoplastic pemphigus. Desmoglein 3 is pathogenic in pemphigus
vulgaris and coprecipitates with
plakoglobin.

185.The first primordial hair follicles form on the eyebrows, upper lip and chin at which gestational
age?

A. 7 weeks
B. 9 weeks
C. 12 weeks
D. 16 weeks
E. 20 weeks

Correct choice: B. 9 weeks

The first primordial hair follicles form at 9 weeks gestation on the eyebrows, upper lip and chin.
The remaining follicles develop at 4-5 months in a cephalad to caudal direction. New follicles
cannot develop in adult skin.

186.Apocrine glands:

A. Are coiled glands


B. Have a two segment ducts that empties onto the skin
C. Are present everywhere on the skin except on the palms and soles
D. Function from birth
E. Secretions are initially odorless

Correct choice: E. Secretions are initially odorless

Apocrine glands are tubular glands that demonstrate decapitation secretion. Like eccrine glands, the
ducts are composed of three segments: intraepidermal duct, intradermal duct and secrectory portion.
The duct usually leads to a pilosebaceous follicle above the entrance of the sebaceous duct. They
are found in the axillae, anogenital region, external ear canal (ceruminous glands), in the eyelids
(Moll�s glands) and in the breast (mammary glands). These glands are functional starting at

89
puberty. The initial secretions are odorless with the odor being derived from C6 � C11
acids. The most abundant being 3-methyl-2-hexenoic acid.

187.Epidermolysis bullosa simplex with muscular dystrophy is due to which defective molecule
A. Plectin
B. Keratin 5 and 14
C. Laminin 5
D. Collagen vii
E. Integrin alpha 6 or beta 4

Correct choice: A. Plectin

EBS with muscular dystrophy is due to a mutation in plectin, plectin is also found in skeletal
muscle. Keratin 5 and 14 are defective in EBS, EBS herpetiformis, EBS weber cockayne, EBS
koebner. Laminin 5 is defective in Herlitz Junctional EB. Collagen VII is mutated in dominant
dystrophic EB, Barts syndrome and EB acquisita. Integrin alpha 6/beta 4 is mutated in junctional
epidermolysis bullosa with pyloric atresia. Epidermolysis bullosa (EB) is a group of inherited
diseases that are characterised by blistering lesions on the skin and mucous membranes. These may
occur anywhere on the body but most commonly appear at sites of friction and minor trauma such
as the feet and hands. In some subtypes, blisters may also occur on internal organs, such as the
oesophagus, stomach and respiratory tract, without any apparent friction.

EB should be distinguished from common friction blisters, and from epidermolysis bullosa
acquisita (EBA), which is a blistering autoimmune disease that is not inherited and often doesn't
develop until adult life. The EB conditions result from genetic defects of molecules in the skin
concerned with adhesion. Loss of adhesion results in blister formation. There are four major types
of EB based on different sites of blister formation within the skin structure:

EB type The site of blister formation within skin


Epidermis or uppermost layer of skin cells (keratinocytes)
(EBS) Epidermolysis bullosa simplex
(JEB)Junctional epidermolysis bullosa
Lamina lucida within the basement membrane zone
(DEB)Dystrophic epidermolysisLamina
bulls densa and upper dermis
Kindler syndrome Mixed pattern or multiple levels within and beneath the basement membrane zo

Within each of these types of EB, there are various subtypes. Varying degrees of severity that range
from mild to severe are found with each EB type. International consensus on the diagnosis and

90
classification of EB has resulted in updated recommendations (2014), based on newer clinical
and molecular data.

188 -Sebaceous carcinoma stains positive for:

A. EMA
B. CEA
C. S100
D. CKIT
E. CD1A

Correct choice: A. EMA

Sebaceous carcinomas stain positive for EMA. The other answer choices are
incorrect as they do not stain positive in sebaceous carcinoma.

189.The triple helix of the collagen molecule is largely maintained due to its amino acid
composition. The polypeptide chains of collagen are repeating triplets of Glycine-X-Y.
The X and Y positions can be occupied by multiple amino acids, but are most often:

A. Alanine and asparagine


B. Tyrosine and threonine
C. Histidine and ornithine
D. Leucine and isoleucine
E. Proline and hydroxyproline

Correct choice: E. Proline and hydroxyproline

Collagen fibers provide tensile strength to the skin and allow it to serve as a
protective organ against the external environment. Collagen makes up 80% of the dry weight of the
dermis. The structure of collagen is comprised of three alpha chains arranged into a triple helix. The
repeating amino acid triplets of the triple helix of collagen are most often Glycine-Proline-
Hydroxyproline.

91
190.What percentage of the dry weight of skin in elastin?

A. 2
B. 4
C. 6
D. 8
E. 10

Correct choice: B. 4

Elastin fibers make up 4% of dry weight of the skin, forming a complex meshwork extending from
lamina densa of the dermoepidermal junction through the dermis and into the hypodermis. Elastins
help return the skin to the normal configuration after being stretched. Elastic fibers are 90% elastin
wrapped by fibrillin microfibrils. Fibrillin is mutated in Marfan syndrome. The typical amino acids
found in elastic fibers are desmosine and isodesmosine. Elastic fibers turn over slowly in the skin
and are damaged by ultraviolet radiation.

191.Numerous neuromediators are involved in cutaneous neurobiology and many play a role in the
development of inflammation in the skin. One such mediator can be induced by application of
capsaicin to the skin. Which of the following is the correct neuromediator?

A. Noradernaline
B. Substance p
C. Neurokinin a
D. Acetylcholine
E. Pro-opiomelanocortin

Correct choice: B. Substance p

Substance P is a neuromediator that binds the tachykinin receptor. It is released upon stimulation of
sensory nerve fibers. It has numerous cutaneous functions, including development of skin edema,
erythema, and pruritus, upregulation of adhesion molecule expression, release of proinflammatory
mediators, etc. Capsaicin causes release of substance P. This has been utilized pharmacologically in
conditions such as zoster, in which consistent application of capsaicin leads to depletion of
substance P, which in turn can help to reduce/eliminate post-herpetic neuralgia. The other answer

92
choices are also neuromediators that are involved in cutaneous inflammation, but their release is not
induced by capsaicin.

192.Mast cells are derived from bone marrow ____+ cells?


A. CD3
B. CD6
C. CD20
D. CD34
E. CD68

Correct choice: D. CD34

CD34+ cells in the bone marrow are the precursors of mast cells. CD3 and CD20 are T and B cell
markers respectively. CD6 is found on mononuclear phagocytic cells in the dermis, CD68 is a
macrophage marker. Mast cells are normal cells in the body, usually found in the skin and other
tissues. Mast cells have a role in the early steps of the body’s coordination of healing responses to
an injury. Granules within the mast cells contain histamine and other chemicals.
• histamine
• leukotriene C4
• prostaglandin D2
• carboxypeptidase
• heparin
• cathespin G-like protease
• tryptase
• tumour necrosis factor-A
• chymase
• interleukin-8
• others

When a mast cell is activated, these chemicals are released into the surrounding skin. Mast
cell chemicals are mediators of inflammation, and cause the blood vessels to leak, resulting
in localised itching, swelling, redness and sometimes blistering. This is what happens normally in
insect bites and is thought to be a protective mechanism. For example, a mosquito injects saliva
when it bites. The saliva triggers mast cellactivation to a varying degree, depending on the
individual's hypersensitivity to the saliva. The unpleasant itch soon persuades the person to try to
avoid getting bitten again.

93
Rubbing an area of skin affected by mastocytosis may also activate the mast cells. The rubbed skin
becomes reddened, swollen and itchy within a few minutes (Darier sign). In young children, the
rubbed area may later blister.

193.Dermal dendrocytes:

A. Are responsible for immediate-type hypersensitivity reactions.


B. Actively synthesize and release IgG.
C. Present antigen to naïve t cells in the lymph node.
D. Are the primary cell found in a glomangioma.
E. Are the primary cell found in an angiosarcoma.

Correct choice: C. Present antigen to naïve t cells in the lymph node.

A dermal dendrocyte is a mononuclear phagocytic cell that is a type of antigen presenting cell. It is
derived from the bone marrow and found both in the papillary and upper reticular dermis. This cell
is highly phagocytic and synonymous with the melanophage that has ingested pigment. Dermal
dendrocytes are likely very important to the afferent limb of the immune response.

194.Which hair condition is associated with abnormal regulation of the peroxisome proliferator-
activated receptor-à ŽÂ³ (PPARγ):

A. Alopecia areata
B. Lichen planopilaris
C. Telogen effluvium
D. Seborrheic dermatitis
E. Androgenic alopecia

Correct choice: B. Lichen planopilaris

Lichen planopilaris is a chronic, scarring condition that results in permanent loss of hair and
destruction of the hair follicles. Defect in lipid metabolism and peroxisome biogenesis may
contribute to LPP pathogenesis. Medications that act as PPAR agonists, such as pioglitazone, may

94
have a role in controlling the progression of this disease. (Karnik P, et al. (2009) Hair follicle stem
cell-specific PPAR deletion causes scarring alopecia. J Invest Dermatol 129(5):1243-57)

195.Telogen effluvium:

A. Involves a diffuse alopecia affecting more than 50% of the scalp


B. Often results from antimetabolites used during cancer chemotherapy
C. Is an inflammatory alopecia devoid of scarring
D. Is a patchy alopecia affecting less than 50% of the scalp
E. Results from sudden illness or surgery

Correct choice: E. Results from sudden illness or surgery

Telogen effluvium is an excessive loss of club hairs from the normal resting follicles of the scalp.
The follicle is pushed from anagen to catagen to telogen. There is no inflammatory process
involved. Causes of telogen effluvium are illness, surgery, parturition, fever, drugs, traction,
starvation, and hypervitaminosis A. Usually, the hair loss only involves less than 50% of the scalp.
There is no specific therapy, and most cases are self-resolving within months.

196.The first cell type to migrate into a new wound in great numbers is the:

A. Neutrophil
B. Monocyte
C. Macrophage
D. Lymphocyte
E. Mast cell

Correct choice: A. Neutrophil

Neutrophils migrate with monocytes concurrently, but arrive first in great numbers because of their
abundance in circulation.
Chemoattractants for the PMNs are fibrinogen, fibrin split products, C5a and leukotrienes. If wound
contamination is controlled, PMN migration ceases within a few days and they become entrapped
within the wound clot, undergo apoptosis or are phagocytosed by macrophages.

95
197.A 25-year-old man presents with a new diagnosis of plaque-type psoriasis. keratotic follicular
papules involving the bilateral dorsal arms, thighs, and cheeks. A biopsy of an unaffected region of
skin would likely exhibit which of the following findings:

A. Increased filaggrin
B. Increased loricrin
C. Increased involucrin
D. Diminished lamellar bodies
E. Decreased transglutaminase I activity

Correct choice: C. Increased involucrin

In psoriasis, involucrin levels are increased. In contrast, filaggrin and loricrin levels are diminished.
Decreased lamellar bodies are implicated in Flegel’s disease andHarlequin icthyosis. Tissue
transglutaminase I is faulty in lamellar icthyosis and non-bullous congenital icthyosiform
erythroderma.

198.Itch is most commonly transmitted by:

A. C-polymodal nociceptor class nerves


B. A-delta class nerves
C. A-beta class nerves
D. Parasympathetic postganglionic fibers
E. A-beta and a-delta fibers

Correct choice: A. C-polymodal nociceptor class nerves

Itch is transmitted primarily by C-polymodal nociceptor class nerves. These are small diameter
unmyelinated nerves that carry pain, thermal, mechanical and pruritic stimuli. A-delta fibers carry
pain, thermal, mechanical and in some cases pruritic stimuli. A-beta fibers carry light touch and
motion stimuli. Parasympathetic post-ganglionic fibers do not contribute to cutaneous pruritus

199.Which of the following enzymes does not require copper for functioning?

96
A. Lysyl oxidase
B. Atp7a
C. Tyrosinase
D. Cystathione beta-synthase
E. Ferrochelatase

Correct choice: E. Ferrochelatase

All of the listed enzymes are copper containing or dependent except ferrochelatase. Lysyl oxidase
facilitates crosslinking of fibrillin in elastic fibers. ATP7a is deficient in Menkes Kinky Hair
Syndrome. Cystathione beta-Synthase is defective in homocystinuria. Tyrosinase catalyzes the first
2 steps, and at least 1 subsequent step, in the conversion of tyrosine to melanin. Ferrochelatase
mutation leads to excess protoporphyrin production and photosensitivity.

200.Which of the following statements about plectin is correct?

A. Links fillagrin to the plasma membrane


B. Crosslinks proteins in adherens junction
C. Mutations result in junctional epidermolysis bullosa with pyloric atresia
D. Plectin is a member of the plakin family
E. Plectin is a constituent protein of the desmosomal plaque

Correct choice: D. Plectin is a member of the plakin family

Plectin is a hemidesmosomal protein and is not present in the desmosome. It links intermediate
filaments to the plasma membrane, crosslinks hemidesmosomal proteins, mutations result in
epidermolysis bullosa simplex with muscular dystrophy and is a member of the plakin family.

201.Which of the following is not a specialized type of sebaceous gland?

A. Moll's gland
B. Meibomian gland
C. Gland of zeis
D. Montgomery's tubercle

97
E. Fordyce spot

Correct choice: A. Moll's gland

There are several types of specialized sebaceous glands that are not associated with a hair follicle.
They include Montgomery's areolar tubercle, Fordyce spots of the lip, Glands of Zeis of the
cutaneous eyelid, and Meibomian glands of the eyelid. Moll's gland of the eyelid are a modified
apocrine gland.

202.Glomus cells are composed of what?

A. Modified fibroblasts
B. Modified skeletal muscle cells
C. Vascular smooth muscle cells
D. Monocytes
E. Neuronal cells

Correct choice: C. Vascular smooth muscle cells

Glomus cells are derived from Suquet-Hoyer canals. They are vascular smooth muscle cells that
allow the rapid shunting of blood from the arterioral to venularsystem, bypassing the capillaries.
This process occurs primarily on the palms and soles.

203.Each of the following is true about melanosomes except:

A. Most characteristic organelle of the melanocyte


B. Tyrosinase activity decreases as melanosomoes mature
C. Are transferred to keratinocytes via phagocytosis
D. Are singly dispersed in the basal layer in white skin
E. Are larger in size in black skin compared to white skin

Correct choice: D. Are singly dispersed in the basal layer in white skin

98
Several differences exist that may explain the heterogeneity of skin color. The number of
melanocytes are the same; however, there are several differences in the melanosomes. Differences
in skin color can be attributed to five factors. In racially heavily pigmented skin, (1) there is greater
production of melanosomes in melanocytes, (2) individual melanosomes show a higher degree of
melanization, (3) melanosomes are larger, (4) the melanosomes are dispersed to a greater degree in
the keratinocytes, and (5) there is a slower rate of degradation.

204.Which of the following cytokines are secreted from keratinocytes?

A. Il-2
B. Il-3
C. Il-17
D. Il-22
E. Il-23

Correct choice: E. Il-23

IL-2 is a cytokine that primarily produced by T lymphocytes and acts on


Other T lymphocytes as a growth factor for both survival anddifferentiation. IL-3 is also produced
by T lymphocytes and acts as a growth factor for cells of the myeloid lineage. IL-17 is an effector
cytokine that is implicated in the pathogenesis of psoriasis and produced by Th17 cells. IL-22 is
also an effector cytokine produced by T lymphocytes that contributes to epidermal acanthosis in
psoriasis. All of these cytokines are not secreted from epidermal keratinocytes. IL-23, which
promotes Th17 effector function, is secreted by epidermal keratinocytes and upregulated in
psoriasis. The p40 subunit of IL-23 is the target of drug ustekinimab used to treat psoriasis

204.The greatest density of mast cells is found in the:

A. Stratum spinosum
B. Stratum basale
C. Papillary dermis
D. Reticular dermis
E. Subcutaneous fat

99
Correct choice: C. Papillary dermis

Mast cells are found in the greatest density in the papillary dermis, in sheaths of the appendages,
and around blood vessels and nerves of the subpapillary plexus. These cells are derived from
CD34+ stem cells residing in the bone marrow. They produce and store in secretory granules many
inflammatory mediators such as histamine, heparin, tryptase, chymase, carboxypeptidase,
neutrophil chemotactic factor and eosinophilic chemotactic factor of anaphylaxis. They also release,
without storing, growth factors, cytokines and leukotrienes. Giemsa or Leder stains are used to
identify mast cells in biopsy specimens. Giemsa stains mast cell granules purple. Leder stains
granules red.

205.Tissue contraction begins:

A. At 3rd day of wound healing


B. During the 2nd week of wound healing
C. After the first month of wound healing
D. After the 3rd month of wound healing
E. After the 9th month of wound healing

Correct choice: B. During the 2nd week of wound healing

Wound healing tends to be a predictable process that begins initially with the inflammatory stage.
During this stage, clot formation occurs. This is the initial step in wound healing. Platelets,
neutrophils and macrophages all migrate to the wound and secrete many mediators of wound
healing. Epithelialization then begins hours after injury. This is followed by granulation tissue
formation (four days after injury) and angiogenesis (first week of repair). Wound contraction ensues
during the second week of healing.

206.The cutaneous immunofluorescence pattern in patients with Senear-Usher syndrome is:

A. Intercellular IgG and C3


B. Linear IgG and C3 along the basement membrane zone
C. Intercellular IgG on guinea pig esophagus
D. Intercellular IgG and C3 and linear IgG and C3 along the basement membrane zone

100
E. Linear IgG and C3 along the basement membrane zone and intercellular IgG on guinea
pig esophagus

Correct choice: D. Intercellular IgG and C3 and linear IgG and C3 along the basement membrane
zone

Senear-Usher syndrome, or pemphigus erythematosus, is a variant of pemphigus foliaceus


characterized by crusted papules and plaques with a seborrheic distribution (malar region, scalp,
upper chest and back). They demonstrate in-vivo IgG and C3 deposition on keratinocyte cell
membranes and the basement membrane zone, in addition to circulating anti-nuclear antibodies.

207.S100 staining would be negative in?

A. Langerhans cells
B. Eccrine cells
C. Schwann cells
D. Adipocytes
E. Keratinocytes

Correct choice: E. Keratinocytes

S100 is sensitive for melanocytes and melanoma, but not specific as it also stains langerhans cells,
eccrine cells, schwann cell, adipocytes, and chondrocytes. It is negative in keratinocytes and can
help discriminate melanoma from pagetoid bowens or spindle cell squamous cell carcinoma.

208.Platelets release which of the following factors to promote new tissue growth?

A. Neutrophil chemotactic factor


B. IL-1
C. ADP
D. TGF-alpha
E. FGF

101
Correct choice: D. TGF-alpha

Platelets release PDGF, TGF-alpha and TGF-beta which promote new tissue growth. FGF and IL-1
are released from monocytes as growth factors (monocytes also release PDGF, TGF-alpha and
TGF-beta). ADP is released from platelets, but is not functioning as a growth factor. Neutrophil
chemotactic factor is released from mast cells and is an inflammatory mediator.

209.Which of the following domains is targeted by the autoantibodies in bullous pemphigoid?

A. NC16A of BP180
B. Laminin 5
C. Plectin
D. alpha-6 integrin
E. NC16A of BP230

Correct choice: A. NC16A of BP180

The NC16A domain of BP180 is the target of autoantibodies in bullous pemphigoid. It interacts
with alpha-6 integrin extracellularly.

210.LEMD3 is mutated in which of the following syndromes?

A. Buschke Ollendorf
B. Albright Hereditary Osteodystrophy
C. Goltz
D. Mccune-Albright
E. Menkes

Correct choice: A. Buschke Ollendorf

LEMD3 or MAN1 is mutated in Buschke-Ollendorf syndrome. Common skin findings include


dermatofibrofibrosis lenticularis disseminata. The radiographic finding commonly seen is
osteopoikilosis. The gene defect was recently described as MAN1/LEMD3, a gene encoding for a
nuclear membrane protein. Albright Hereditary Osteodystrophy is caused by inactivation of

102
GNAS1, Goltz is an x-linked dominant condition with unknown pathogenesis, McCune-Albright
syndrome is caused by an activation of GNAS1. Menke's Syndrome is caused by a defect in
ATP7A/MKN on Xq13.

211.BP230 is a member of of which of the following families?

A. Plakin
B. Integrin
C. Laminin
D. Collagen
E. Elastin

Correct choice: A. Plakin

BP230 is a member of the plakin family and is homologous to desmoplakin and is intracellular. It
attaches intermediate filaments to hemidesmosomal plaque. The remaining options are not part of
the plakin family.

212.Which of the following dermal cells always express CD11c and CD6?

A. Mononuclear phagocytic cells


B. Fibroblasts
C. Mast cells
D. Erythrocytes
E. Glomus cells

Correct choice: A. Mononuclear phagocytic cells


The mononuclear phagocytic cell includes monocytes, macrophages and dermal dendrocytes. All
phagocytic skin macrophages express CD11c and CD6. Fibroblasts do not produce a CD marker.
Mast cells are derived from bone marrow-residing CD34+ stem cells. They do not produce CD11c
or CD6.

103
213.Hair grows at:

A. 0.004 mm/day
B. 0.04 mm/day
C. 0.4 mm/day
D. 4 mm/day
E. 10 mm/day

Correct choice: C. 0.4 mm/day

Hair grows at an average of 0.4 mm/day. The other options are incorrect.

214. Which cytokeratin would you expect to be preferentially expressed in the keratinocytes of a
psoriatic plaque:

A. K1
B. K5
C. K12
D. K15
E. K16

Correct choice: E. K16

K16 and K6 act as markers for hyperproliferative keratinocytes. They are found in skin disease such
as psoriasis, warts, actinic keratoses, and SCC's.

215.The following protein is the target for both dytrophic EB and Bullous SLE:

A. Type VII collagen


B. Laminin 5
C. Integrin subunit b4
D. Type XVII collagen
E. BPAG2

104
Correct choice: A. Type VII collagen

Type VII collagen is the target for both dystrophic EB and bullous SLE. It is also the target for EB
acquisita.

216.Dystrophic epidermolysis bullosa results from mutations in:

A. Collagen type I
B. Collagen type II
C. Collagen type III
D. Collagen type V
E. Collagen type VII

Correct choice: E. Collagen type VII

Dystrophic epidermolysis bullosa results from mutations in Type 7 collagen. Type I collagen
mutations are associated with osteogenesis imperfecta. Antibodies to Type II collagen are associated
with relapsing polychondritis. Mutations in type V collagen result in the classic form of Ehlers
Danlos while mutations in type III collagen result in the vascular form of Ehlers Danlos.

217.Krause end bulbs are:

A. Adapting mechanoreceptors found on weight-bearing sites that respond to vibrational


stimul
B. Found on the vermillion border of the lips
C. Located in the dermal papillae of digital skin
D. Located in the deep dermis and within the subcutis in weight-bearing sites of the body
E. Found at the orifice of the hair follicle and particularly sensitive to cold

Correct choice: B. Found on the vermillion border of the lips

Krause end bulbs are mucocutaneous receptors found on the glans penis, clitoris, labia minora,
perianal area, and vermillion border of the lips. Papillary nerve endings are free nerve endings
found at the orifice of the hair follicle and particularly sensitive to cold. Meissner's corpuscles are

105
ovoid, elongated mechanoreceptors located in the dermal papillae of digital skin that detect touch
and light pressure. Pacinian (Vater- Pacini) corpuscles exist in the deep dermis and within the
subcutis in weight-bearing sites and function as adapting mechanoreceptors that respond to
vibrational stimuli.

218.Hair follicle development in the human embryo begins during:

A. 1st trimester
B. 2nd trimester
C. 3rd trimester
D. At the blastocoele stage
E. Within 2 weeks of fertilization

Correct choice: A. 1st trimester

Hair follicles begin in the third month of fetal life as a downgrowth of cells from the epidermis (3rd
month=12th week=1st trimester).

219.Meibomian glands are:

A. Modified sebaceous glands


B. Found everywhere except on the palms and soles
C. Sebaceous lobules that feed into a lactiferous duct
D. Modified ceruminous glands
E. Modified apocrine glands

Correct choice: A. Modified sebaceous glands

Meibomian glands of the eyelids are modified sebaceous glands. Sebaceous glands are found
everywhere on the skin except the palms and soles. Montgomery's areolar tubercles consist of
several sebaceous lobules feeding into a lactiferous duct. Ceruminous glands are apocrine glands of
the external ear canal. Apocrine glands in the eyelids are Moll's glands.

106
220.During embryogenesis, periderm cells of the fetus contain which of the following substances?

A. Ceramide
B. Glycogen
C. Free fatty acids
D. Porphyrins
E. Sebum

Correct choice: B. Glycogen

In week 7 of embryogenesis the surface ectoderm produces two layers. The external layer is the
periderm which contains glycogen and gives rise to the stratum corneum by week 21. The other
layer is the stratum germinativum.

221.1 melanocyte has contact with:

A. 6 keratinocytes
B. 12 keratinocytes
C. 18 keratinocytes
D. 30 keratinocytes
E. 36 keratinocytes

Correct choice: E. 36 keratinocytes

One melanocyte and 36 keratinocytes make up one epidermal melanin unit. This melanocytes
transfers pigment to these keratinocytes.

222.Which of the following statements about darkly pigmented races versus lighter pigmented races
is correct?

A. The number of melanosomes in melanocytes are the same


B. The individual melanosomes have the same degree of melanization
C. The melanosomes are equal in size melanosomes
D. There are equal numbers of melanocytes

107
E. There is a faster rate of melanosome degradation

Correct choice: D. There are equal numbers of melanocytes

Individuals with darker pigmentation have an equal ratio of melanoyctes to


keratinocytes. Darker pigmentation is related to an increased number of melanosomes, increased
melanization, greater size of melanosomes, and slower degradation.

223.All of the following are true regarding the formation of hair except:

A. The inner root sheath keratinizes by means of trichohyalin granules


B. Henle’s layer is outside of Huxley’s layer
C. The outer root sheath is a downward extension of the epidermis
D. Huxley’s layer contains melanin
E. The hair matrix becomes the hair and the inner root sheath

Correct choice: D. Huxley’s layer contains melanin

The inner root sheath is composed of three layers, which are the inner
root sheath cuticle, Huxley layer, Henle layer. None of these layers contain melanin. All three layer
keratinize by trichohyaline granules and disintegrate when they reach the isthmus of the hair
follicle.

224.The classical types of Ehlers-Danlos Syndromes (Type 1 and 2) lead to varying degrees of
hyperextensible skin, easy bruising, wide, atrophic scars, and hypermobile joints. The underlying
defect in this disorder is a mutation in:

A. Elastin
B. Fibrillin 1
C. ABCC6
D. Collagen V
E. Cystathionine beta-synthase

108
Correct choice: D. Collagen V

Classical types of Ehrlers-Danlos are manifested by varying degrees of


hyperextensible skin, easy bruising, wide, atrophic scars, and hypermobile joints. The genetic defect
is in Collagen V. Elastin is mutated in Cutis Laxa. ABCC6 is defective in Pseudoxanthoma
elasticum. Cystathionine beta-synthase is mutated in homocystinuria.

225.Which type of collagen in mutated in osteogenesis imperfecta?

A. Collagen I
B. Collagen II
C. Collagen III
D. Collagen IV
E. Collagen VII

Correct choice: A. Collagen I

Collagen type I is mutated in osteogenesis imperfecta. The COL1A2 gene is located on 7q22 and
the COL1A1 is on 17q22. The other listed collagens are not involved in osteogenesis imperfecta.
Collagen II is in cartilage and vitreous, Collagen III is in fetal skin, blood vessels and intestines,
Collagen IV is in basement membranes and Collagen VII in anchoring fibrils and amnion.

226- on electron microscopy, which cell demonstrates cytoplasmic projections and secretory
granules?

A. Langerhans cell
B. Keratinocyte
C. Mast cell
D. Melanocyte
E. Macrophage

Correct choice: C. Mast cell

109
Electron microscopy of mast cells demonstrates large long villi at the periphery and round or oval
secretory granules. Langerhans cells show a folded nucleus and rarely phagocytized melanosomes
on electron microscopy. Their characteristic feature is the presence of Birbeck granules whose disk
shape with one or two vesicles at either end represents a tennis raquet. Examination of melanocytes
shows an absence of tonofilaments or desmosomes with characteristic melanosomes in various
stages of formation. Macrophages often contain phagocytized material within phagosomes.

227.Which of the following is the most common genetic alteration seen in mucosal melanomas?

A. GNAQ
B. Cyclin Dependant Kinase 4/6
C. BRAF
D. KIT
E. MDM2

Correct choice: D. KIT

Mucosal melanomas tend to have an activating mutation in KIT. GNAQ mutations are seen in both
uveal melanomas and blue nevi. Cyclin dependant kinase 4/6 binds cyclin D and together
phosphorylate Retinoblastoma. MDM2 targets p53 for ubiquitination.

228.What is the major component of the anchoring fibril?

A. Type I Collagen
B. Type III Collagen
C. Type IV Collagen
D. Type VII Collagen
E. Laminin 5

Correct choice: D. Type VII Collagen

Anchoring fibrils are made of Type VII collagen. Anchoring plaques are made of Type IV collagen,
and interact with a network of Type I And Type III collagen fibers in the dermis.

110
229.Which of the following markers are specific and reliable for Merkel cells?

A. CD20
B. HMB-45
C. CD34
D. CD3
E. CK20

Correct choice: E.Cytokeratin (CK) 20 is a reliable marker for Merkel cells.

CD20 is a marker for B-cells and CD3 is a T-cell marker. HMB-45 is used in staining for immature
melanosomes and is reactive in melanoma. CD34 is used to stain dermatofibrosarcoma protuberans.

230.The epidermis is comprised of what type of cells?

A. Keratinocytes, Melanocytes, Merkel cells, Langerhan cells


B. Keratinocytes, Endothelial cells, Merkel cells, Langerhan cells
C. Keratinocytes, Melanocytes, Neutrophils, Langerhan cells
D. Keratinocytes, Melanocytes, Merkel cells, Goblet cells
E. Keratinocytes, Endothelial cells, Merkel cells, Goblet cells

Correct choice: A. Keratinocytes, Melanocytes, Merkel cells, Langerhan cells

The adult epidermis is composed of three basic cell types: Keratinocytes,


melanocytes, and Langerhans cells. An additional cell, the Merkel cell, can be found in the basal
layer of the palms and soles, the oral and genital mucosa, the nail bed, and the follicular
infundibula.

111
112
113
Immunoderma logy

1- Treatment of this condition with IL-17 blockade has been shown to:

A. Increase the risk of Reversible Posterior Leukoencephalopathy syndrome


B. Worsen pre-existing multiple sclerosis
C. Increase risk for mucosal candidal infections
D. Cause hepatic fibrosis with long term use
E. Improve pre-existing inflammatory bowel disease

Correct choice: C. Increase risk for mucosal candidal infections

Explanation: Choice 1 is ustekinumab; choice 2 is TNF blockers; choice 4 is methotrexate

2- Blocking which cytokine would would be most beneficial in this condition?

A. IL-1
B. IL-10
C. IL-4
D. IL-5
E. IL-17

114
to
Correct choice: E. IL-17

Explanation: Nail psoriasis, characterized here by irregular nail pitting and distal onycholysis,
responds to treatment against IL-17 including secukinumab, ixekizumab, and brodalumab

3-A medical student with a family history of psoriasis and severe plaque psoriasis would like to
know if there are any hereditary disease associations with the major histocompatibility complex.
You recall that psoriasis has the following associations with HLA types:

A. HLA-Cw6/B13/B17/B27
B. HLA-DQ2/B8
C. HLA-B51
D. HLA-DR4/DQ8
E. HLA-DR3/DR4

Correct choice: A. HLA-Cw6/B13/B17/B27

Explanation: Psorsiasis is associated with a number of histcompatibility antigens. HLACw*0602–


positive patients usually have a younger onset age, more severe psoriasis course, guttate or eruptive
plaque psoriasis phenotypes, more frequent streptococcal throat carriage or infections, and
streptococcal-associated psoriasis exacerbation. HLA-B27 is common in psoriatic arthritis,
inflammatory bowel disease, and reactive arthritis. HLA-DQ2 is most frequently associated with
dermatitis herpitformis. HLA-B51 most commonly with Behcet's. HLA-DR4 and HLA-DQ8 have
been linked to chronic urticaria. Pemphigus gestationis has been assocated with HLA-DR3/DR4.

4- Mononuclear phagocytes residing in tissues:

A. Are called macrophages


B. Phagocytose antibodies and degrade them into peptides
C. Present antigen to B-cells
D. Do not produce cytokines
E. Possess Fc receptor for IgE

115
Correct choice: A. Are called macrophages

Explanation: Mononuclear phagocytes are components of the innate immune system. Once in
tissues they are called macrophages; when they are in the bloodstream they are referred to as
monocytes. Macrophages phagocytose foreign antigens (NOT antibodies) and degrade them into
peptides, present antigen to T-cells (NOT B-cells), produce cytokines (which recruit other
inflammatory cells), and possess Fc receptor for IgG (NOT IgE).

5- Contact urticaria is a:

A. Type I hypersensitivity
B. Type II hypersensitivity
C. Type III hypersensitivity
D. Type IV hypersensitivity
E. Type V hypersensitivity

Correct choice: A. Type I hypersensitivity

Explanation: Contact urticaria is an example of a type I hypersensitivity reaction. The classic


example is a latex-induced contact urticaria. Type I reactions (immediate hypersensitivity reactions)
involve immunoglobulin E (IgE)–mediated release of histamine and other mediators from mast cells
and basophils. Contact urticaria is not a Type II, III, or IV hypersensitivity reaction. There is no
Type V hypersensitivity reaction.

6- Which of the following cell types induce susceptibility to tumor growth?

A. Suppressor T-cells
B. NK cells
C. Helper T-cells
D. Mast cells
E. Langerhans cells

Correct choice: A. Suppressor T-cells

116
Explanation: The suppressor T-cells induce susceptibility to tumor. These cells appear to arise in
UV-irradiated hosts prior to tumor developing, and play a role in carcinogenesis.

7- A 14-year-old male presents with a severe eczematous dermatitis. His mother states that his skin
has been a problem since he was an infant. Acute atopic dermatitis is associated with which of the
following cytokine profiles?

A. Th1: IFN-γ, IL-2, IL-3


B. Th1: IL-1, TNF-β
C. Th2: TNF-β, IL-4
D. Th2: IL-4, IL-5, IL-13
E. Th2: IL-2, TNF-β

Correct choice: D. Th2: IL-4, IL-5, IL-13

Explanation: The acute atopic dermatitis immune profile is Th2-cytokine-based, with IL4, IL-5 and
IL-13 as prominent aspects. In chronic atopic dermatitis has Th1 cytokines (IFN-γ, IL-12)
dominate. Other listed cytokines are not prominent in acute atopic dermatitis.

8- A child develops lesions see below. How can you differentiate the genetic form from the drug-
induced form?

A. Porphyrins
B. Direct immunofluorescence
C. Indirect immunofluorescence
D. Histologic pattern
E. ELISA

117
Correct choice: A. Porphyrins

Explanation: Porphyria cutanea tarda (PCT) can be due to a genetic mutation in UROD. Pseudo-
PCT is most commonly secondary to drugs such as naproxen, NSAIDs, tetracycline, furosemide,
and hemodialysis. The main differentiating factor is that urine, serum, and fecal porphyrins are
normal in pseudo-PCT. Pseudoporphyria can look histologically exactly like PCT with DIF with
IgG, IgM, IgA, C3 at DEJ and around vessels.

9- In a patient with IPEX (immunodysregulation, polyendocrinopathy, enteropathy, X-linked)


syndrome, which of the following T-cell lines are affected?

A. Th17
B. Th1
C. Th2
D. Cytotoxic T-cells
E. Regulatory T-cells

Correct choice: E. Regulatory T-cells

Explanation: FOXP3 is the most often mutated gene in IPEX syndrome. FOXP3 is critical in the
function of regulatory T-cells. The other T cell lines are not directly affected in IPEX syndrome.

10- A 32 year-old woman presents with meningitis and palpable purpura. A diagnosis of
meningococcemia is confirmed via culture of cerebrospinal fluid. Which complement component is
most likely to be deficient in this patient?

A. C1
B. C2
C. C3
D. C4
E. C5

Correct choice: E. C5

118
Explanation: Patients with deficiencies of late complement components (C5-9) are at increased risk
of meningococcemia due to Neisseria. Early complement component (C1-4) deficiencies are not
associated with increased risk of meningococcemia.

11- IL-23 plays a critical role in the pathogenesis of psoriasis. Which of the following cytokines is
critical for IL-23-mediated epidermal hyperplasia in psoriasis?

A. IL-2
B. IL-12
C. IL-17A
D. TNF-alpha
E. IFN-gamma

Correct choice: C. IL-17A

Explanation: IL-23 and Th17 cells producing IL-17A and IL-22 are found in excess in skin affected
by psoriasis. IL-6, IL-22, and IL-17A have all been shown to be critical in mediating epidermal
hyperplasia in psoriasis in response to IL-23.
Psoriasis is a chronic systemic inflammatory disease causing erythematous and scaly skin plaques;
up to 30% of patients with psoriasis develop Psoriatic Arthritis (PsA), which is characterized by
inflammation and progressive damage of the peripheral joints and/or the spine and/or the entheses.
The pathogenic mechanisms driving the skin disorder in psoriasis and the joint disease in PsA are
sustained by the activation of inflammatory pathways that can be overlapping, but also, at least
partially, distinct. Cytokines members of the IL-23/IL-17 family, critical in the development of
autoimmunity, are abundantly expressed within the cutaneous lesions but also seem to be involved
in chronic inflammation and damage of the synovium though, not in all patients. IL-2, IL-12, TNF-
alpha, and IFN-gamma are not critical in playing an interactive role with IL-23 in psoriasis.

12- Patients with the above condition should be evaluated for what immunodeficiency?

A. C5-9
B. IgM
C. C3
D. B2-integrin
E. Immunoglobulin

119
Correct choice: A. C5-9

Explanation: This is a photo of disseminated gonococcemia, which can be associated with a


deficiency of C5-9. This deficiency increases susceptibility to Neisseria.
C3 and C4 are decreased in active systemic lupus erythematosus, especially when the kidneys are
involved. They can also be decreased in hypocomplementemic urticarial vasculitis. Decreased IgM
is associated with Wiskott–Aldrich. Leukocyte adhesion 1 deficiency is associated with a B2-
integrin deficiency. Bruton’s agammaglobulinemia is associated with a decrease in immunoglobulin
due to decreased mature B cells.

13 -The immunoglobulin most commonly found in mucous secretions is:

A. IgA
B. IgD
C. IgE
D. IgG
E. IgM

►A

IgA is found in mucous membrane secretions and is able to agglutinate antigens and activate the
alternate but not the classic complement pathway. IgG is the antibody that can cross the placenta
and the most common antibody found in circulation. IgD is not found in circulation other than in
hyper-IgD syndrome, an autosomal recessive disorder caused by mutations in the mevalonate
kinase gene. A significant elevation of serum IgD is seen in 95% of these patients. IgE is an
anaphylactic antibody that is involved in nearly all immediate allergic and anaphylactic type
reactions. IgM is the antibody produced in the early stages of antibody responses. It is a pentamer
which can agglutinate antigen and active the classic complement pathway.

14- A 24 year old female patient is referred for management of chronic idiopathic urticaria. Many
cases of this disease are associated with autoantibodies against what?

A. IgE receptor
B. Tryptase

120
C. Histamine receptor
D. TNF-alpha receptor
E. IL-6 receptor

►A

Many cases of idiopathic urticaria have circulating autoantibodies directed against the chain of high
affinity IgE receptor on the mast cell surface.

15- The pharmacologic activity of tacrolimus includes:

A. Phosphorylation of NFAT (nuclear factor of transcription)


B. Binding and inhibition of NF kappa B
C. Inhibition of interleukin-1 gene transcription
D. Activation of calcineurin
E. Inhibition of interleukin-2 gene transcription

►E

Tacrolimus is a macrolide similar to cyclosporine and pimecrolimus (Elidel). Calcineurin is a


phosphatase that dephosphorylates subunit on NFAT (nuclear factor of activated T cells).
Dephosphorylated NFAT is active and causes the increase in IL2. Tacrolimus binds
macrophillin1 which blocks calcineurin so NFAT is not dephosphorylated ie it remains
phosphorylated and is inactive therefore preventing transcription of IL 2.

16 -This patient says the rash is spreading and not controlled with topical therapy. You give him a
course of oral treatment that lasts:

A. 1 week
B. 2 weeks
C. 3 weeks
D. 4 weeks
E. 5 weeks

121
►C

Generally, for poison ivy dermatitis, if patients are given a course of oral steroids, the course
should be at least 3 weeks long, as if the duration is shorter, patients may develop a rapid
rebound.

17 -Anti Jo-1 antibodies are directed against which of the following?

A. Topoisomerase
B. Lysyl oxidase
C. Gyrase
D. Histidyl transfer RNA synthetase
E. Telomerase

►D

Anti Jo-1 antibody is typical of autoimmune diseases that involve muscle, including
dermatomyositis.

18 -Anti-epiligrin (laminin 5) antibodies may be seen in:

A. Pemphigoid gestationis
B. Pemphigus vegetans
C. Fogo selvagem
D. Cicatricial pemphigoid
E. Paraneoplastic pemphigus

►D

122
Patients with cicatricial pemphigoid have been reported to have anti-epiligrin antibodies.

19- Subacute cutaneous lupus erythematosus has been associated with the ingestion of which of the
following drugs?

A. Phenytoin
B. Allopurinol
C. Terbinafine
D. Trimethoprim/sulfamethoxazole
E. Auranofin

►C

SCLE has been reported to be associated with terbinafine. This condition is often associated with
anti-Ro (SS-A) and anti-La (SS-B) antibodies.

20- This patient has a lichenified plaque in the lower mid abdomen as well as these two
exczematous plaques. This patient needs:

A. Patch testing
B. A steroid
C. An antiviral
D. A KOH scraping
E. An antifungal

►A

This patient is likely allergic to the nickel in her belt buckle and possibly to metals that she places in
her pocket. She needs patch testing. While a topical steroid might help resolve pruritus, the patient
ultimately needs to be diagnosed with her allergy and avoid nickel. Dimethylglyoxime can be used
as an indicator to detect nickel in metals.

123
21- A previously healthy child presents with palpable purpura, arthritis, and vomiting. You suspect a
hypersensitivity vasculitis characterized by:

A. Perivascular IgA
B. P-ANCA autoantibodies
C. Granulomas and eosinophilia
D. Infiltration of destruction of vessels by atypical lymphocytoid and plasmacytoid cells
E. Nectrotizing granulomatous vasculitis

►A

The child has findings of Henoch-Schonlein purpura, a hypersensitivity vasculitits that presents
clinically with a triad of purpura, joint pain, and GI complaints. Histologically it is characterized by
perivascular deposition of IgA.

22- Which immunoglobulin is most efficient at fixing complement?

A. IgM
B. IgE
C. IgD
D. IgA
E. IgG

►A

IgM is the first antibody produced by B cells. It is secreted by plasma cells as a pentamer. Due to its
size, IgM doesn't enter tissue well. It is the most efficient immunoglobin at fixing complement.

23 -Gene rearrangement analysis is useful for determining:

A. Lymphocyte clonality in mycosis fungoides


B. Lymphocyte activity
C. Gene Function
D. Gene Mutations

124
E. T cell receptor status

►A

Gene rearrangement studies are useful to detect clonality in antigen specific cell types (B cells, T
cells).

24- Major histocompatibility complex class I molecules bind to:

A. Peptides derived from proteins synthesized and degraded in the cytosol


B. Peptides derived from proteins degraded in endocytic vesicles
C. Peptides external to the cell membrane
D. Immunoglobulin E
E. None of these answers are correct

►A

Major histocompatibility complex (MHC) class I molecules bind to peptides derived from proteins
synthesized and degraded in the cytosol. They present these processed peptides to CD8+ T-cells.
MHC class II molecules bind stably to peptides derived from proteins degraded in endocytic
vesicles. CD4+ T-cells recognize the MHC class II molecules. Immune activation against the
foreign antigens or pathogens taken up by the cell is the result of these interactions. Peptides
external to the cell are not recognized by MHC molecules. IgE does have a receptor on the cell
surface, especially basophils and mast cells, but is not recognized by the MHC complex.

25- The most definitive HLA association with psoriasis is:

A. HLA-Cw6
B. HLA-B27
C. HLA-B13
D. HLA-B17
E. HLA-B37

125
►A

HLA-Cw6 is associated with a 9-15x greater risk for developing psoriasis. All of the other HLA
antigens listed are associated with various types of psoriasis, but at with lesser strength of
association.

26- Immunocytomas are:

A. Low grade B-cell lymphomas


B. Aggressive B-cell lymphomas
C. Low grade T-cell lymphomas
D. Aggressive T-cell lymphomas
E. NK cell lymphomas

►A

These indolent tumors present as solitary or multiple nodules usually on the extremities. The cells
have been reported to have CD-20 (B cell marker) and have been reported to be bcl-2 positive.

27 -Anti-Ro (SS-A) antibodies are most commonly found in:

A. Mixed connective tissue disease


B. Eosinophilic fasciitis
C. Drug-induced systemic lupus erythematosus
D. Homozygous C2 deficiency
E. Neonatal lupus erythematosus

►E
Ro 60 kDa autoantigen is a major target for patients with SLE and Sjogrenǁs syndrome. Neonatal
lupus occurs in newborns of mothers with anti-Ro antibodies and leads to children with
photosensitive skin lesions and a cardiac conduction defect, a third degree heart block.

126
28- Which of the following cytokines shifts the immune response towards TH2?

A. IL-4
B. IL-5
C. IL-10
D. TNF
E. IFN-gamma

►A

IL-4 is the cytokine that polarizes the immune response towards Th2. IL-5 is an eosinophil growth
factor. IL-10 is a general down-regulator of immunity. TNF and IFN-gamma are TH1 cytokines, not
TH2 cytokines. IFN-gamma is secreted by TH1 cells, and is the main macrophage-activating
cytokine.

29- Which of the follwing is a chemotactic factor for eosinophils?

A. TNF
B. IL2
C. C5a
D. Plasminogen activator
E. IL8

►C

Eosinophil chemotactic factors include all of the following: Histamine, soluble immune complexes,
C5a, and HETE

30- Herpes simplex virus-related erythema multiforme has been associated with an increased
frequency of:

A. HLA-B7
B. HLA-B8
C. HLA-B13

127
D. HLA-B15
E. HLA-B27

►D

Erythema multiforme associated with herpes simplex has been reported to have an increase of
HLA-B15.

31- A homeless patient presents with a scaling, pustular periorifical eruption around the mouth and
genitalia. What lab abnormality is associated with this condition?

A. Increased zinc level


B. Increased copper level
C. Decreased copper level
D. Decrease alkaline phosphatase level
E. Increased alkaline phosphatase level

►D

In a homeless patient with poor nutrition, acrodermatitis enteropathica due to zinc deficiency can
presents with a scaling eruption of the periorificial regions. In addition to a low zinc level, levels of
alkaline phosphatase, a zinc dependent enzyme, are decreased.

32- Which of the following immunoglobulins cannot activate the complement pathway?

A. IgM
B. IgG1
C. IgG2
D. IgG3
E. IgG4

►E

128
Immunoglobulins (Ig) differ in their ability to activate complement. IgM is the largest Ig, is the
major Ig in the primary immune response, and consists of a pentamer that activates the classic
complement pathway. IgG is the most abundant Ig and the major Ig in the secondary immune
response. Four subclasses of IgG exist based on the amino acid residue sequences of their constant
region, IgG1 through IgG4. IgG1 and IgG3 are potent activators of the classic complement
pathway, IgG2 is less effective and IgG4 is unable to do so.

33- Which of the following is an example of a delayed hypersensitivity reaction?

A. Allergic contact dermatitis


B. Anaphylaxis
C. Latex allergy
D. Transfusion reaction
E. Serum sickness

►A

There are 4 types of Hypersensitivities: ("ACID") Type I: Anaphylactic and Atopic: Exampl es
Urticaria, Asthma, and Allergic Rhinitis Type II: cytotoxic: examples - Transfusion reactions, ABO
incompatibility, Rh disease (erythroblastosis fetalis), Autoimmune reactions, Hemolytic disease of
newborn, Goodpasture's syndrome Type III: Immune complex, Serum sickness, and arthrus
reactions: examples -PAN, glomerulonephritis, SLE, Rheumatoid arthritis, and serum sickness Type
IV: Delayed (cell-mediated)typesL examples -TB skin test, transplant rejection, contact dermatitis,
interactions and skin repsonsiveness to bacteria, fungi, viruses, and protozoa, photo-allergies, insect
bites, etc

34- This woman should have a workup for:

129
A. Lymphoma
B. Nephrolithiasis
C. Pancreatic cancer
D. Hemochromatosis
E. Thalassemia

►D

Porphyria cutanea tarda has been shown to be associated with hemochromatosis. Patients with
hemochromatosis have mutations in the HFE gene, and early detection of mutations can improve
life expectancy for these patients.

35- Langerhans cell adhesion to keratinocytes is mediated by what adhesion molecule?

A. E-cadherin
B. N-cadherin
C. Desmoglein 3
D. P-cadherin
E. B7

►A

Langerhans cells are found in the epidermis and on mucosal surfaces. After antigen exposure and
activation, they migrate to regional lymph nodes where they mature into antigen presenting cell s.
Adhesion to keratinocytes in the epidermis is mediated by e-cadherin.

36- When attempting to identify Langerhans cells in a specimen, which marker is most helpful?

A. CD1
B. CD4
C. CD7
D. CD8
E. CD20

130
►A

CD1 is a surface antigen specific for epidermal Langerhans cells. It is not expressed in other
epidermal structures. The other characteristic ultrastructural feature of Langerhans cells is the
Birbeck granule. CD4 is found on T-helper cells and occasionally on Langerhans cells. CD7 is a T-
cell marker that is often lost in cutaneous T-cell lymphoma. CD8 is found on cytotoxic T-cells.
CD20 is a B-cell marker.

37- Elaboration of which of the following cytokines is characteristic of TH2 response?

A. Interferon-8 (gamma)
B. Interleukin-1
C. Interleukin-2
D. Interleukin-4
E. Interleukin-12

►D

Interleukin-4 (IL-4) is a cytokine involved in B-cell proliferation. Along with IL-5 and IL-13, IL4 is
classified as a TH2 cytokine.

38- Which cytokine is most important in recruiting neutrophils?

A. Interleukin-1
B. Interleukin-2
C. Interleukin-4
D. Interleukin-8
E. Interleukin-10

►D

Chemotaxis is the process of cells moving through a gradient of towards increasingly higher
concentrations. IL-8 has activating and chemoattractant properties on neutrophils.

131
39- Which monoclonal gammopathy is most commonly associated with erythema elevatum
diutinum?

A. IgA
B. IgD
C. IgE
D. IgG
E. IgM

►A

A report of 13 patients indicated IgA is most commonly associated with EED.

40- Which cytokine is not upregulated in atopic dermatitis patients?

A. IL-13
B. IL-4
C. IL-5
D. IL-10
E. IFN-gamma

►E

Interferon gamma is Th1 cytokine which downregulates Th2 responses. The remaining are Th2
cytokines active in atopic dermatitis. IL-4 is a B-cell growth factor and active in signaling isotope
switching from IgM to IgE. IL-5 is an eosinophil growth factor. IL-10 downregulates Th1 immunity
and IL-13 signals isotope switching along with IL-4.

41- Dermatitis Herpetiformis is most commonly associated with which HLA?

A. HLA-DR3
B. HLA-B27
C. HLA-B8
D. HLA-Bw35

132
E. HLA-DQ(A1*0501, B1*02)

►E

HLA-DQ (A1*0501, B1*02) genes are present on 90% of all patients with Dermatitis
Herpetiformis. The remainder of patients have the HLA-DQ8 gene. Other associations include:
HLA-B8, HLA-DR3 and DR5/DR7.

42- The most common autoimmune disorder in patients with chronic hepatitis C infection is:

A. Autoimmune thyroiditis
B. Idiopathic thrombocytopenic purpura
C. Rheumatoid arthritis
D. Sjӧgrenǁs syndrome
E. Systemic lupus erythematosus

►A

Autoimmune thrombocytopenia has been reported with hepatitis C infection but not as commonly
as autoimmune thyroiditis.

43- A 27 year old gentleman with a known history of chronic plaque psoriasis complains of pain
and stiffness of his joints, particularly in his hands. Which HLA subtype is associated with psoriatic
arthritis?

A. B27
B. Cw6
C. B17
D. B51
E. B8

►A

133
Psoriatic arthritis, as well as generalized psoriasis,is associated with HLA-B27. HLA-Cw6 is
associated with psoriasis, Bw35 with cutaneous lichen planus, B8 with oral lichen planus, and B51
with Behcet's disease.

44- Topical tacrolimus and pimecrolimus are used to treat atopic dermatitis and other inflammatory
skin conditions. On which of the following ions is the inflammatory pathway blocked by these
medications dependent?

A. Sodium
B. Potassium
C. Calcium
D. Selenium
E. Zinc

►C

Calcium. Both pimecrolimus and tacrolimus penetrate cutaneous T-cells, forming a complex with
calcineurin, which blocks the activation of NF-AT, thus blocking the transcription of a variety of
genes with a resultant decrease in T-cell mediated inflammation. This pathway is calcium
dependent. The other ions are not involved in this process.

45- Natural killer (NK) cells eliminate infected cells in all of the following ways except:

A. NK cells adhere to and kill target cells coated with IgG


B. NK cells secrete perforin
C. NK cells secrete granzyme
D. NK cells secrete myeloperoxidase
E. NK cells do not target cells expressing major histocompatibility (MHC) class I
molecules

►D

NK cells focus on the destruction of infected or malignant cells. They achieve this via recognition
of IgG on target cells, the so-called 'antibody-dependent cellular toxicity.' NK cells also eliminate

134
cells by secreting perforin, which makes holes in the cell membrane, through which granzyme is
injected. Granzyme induces apoptosis by activating the caspase cascade. In addition, NK cells do
not target cells expressing MHC class I molecules on their surface; some virus downregulate MHC
class I molecules to evade recognition by cytotoxic T cells, which may make them susceptible to
NK cell attack. Neutrophils kill ingested organims using myeloperoxidase.

46- An elderly gentleman with metastatic colon cancer is prescribed erlotinib, an epidermal growth
factor receptor monoclonal antibody. What is the most common cutaneous finding seen as a result
of this class of medication?

A. Acneiform eruption
B. Acral erythema
C. Morbilliform eruption
D. Hypotrichosis
E. Urticaria

►A

Erlotinib, along with cetuximab and gefitinib, are epidermal growth factor receptor inhibitors that
are used for the treatment of multiple visceral malignancies, including metastatic colon cancer.
They have all been associated with an acneiform eruption.

47- Tacrolimus is a non-steroidal anti-inflammatory medication that works by inhibiting calcineurin


activity through complexing with what binding protein?

A. FK506
B. TGF-beta
C. NF-kappa-B
D. SRE
E. IL-23

►A

135
Tacrolimus and pimecrolimus are non-steroidal calcineurin inhibitors that act as antiinflammatory
medications. In dermatology, they are most commonly used in topical preparations. These
medications form complexes with FK506 binding protein, which inhibits calcineurin activity. A key
regulatory step in the activation of T cells is the activation of calcineurin via calmodulin.

48- Keratinocytes express what class of major histocompatibility complex under normal conditions?

A. MHC Class I
B. MHC Class II
C. MHC Class III
D. MHC Class IV
E. MHC Class V

►A

Keratinocytes express MHC Class I molecules and therefore can be attacked by CD8+ Tc cells, in
particular after viral infection. Although keratinocytes do not express MHC Class II molecules
under normal conditions, they can be induced to do so in the setting of inflammatory conditions.

49- Which component of the classical pathway of complement acts as an opsonin?

A. C3b
B. C3a
C. C5a
D. C5b
E. C1 INH

►A

The classical pathway of complement is activated by antigen-antibody complexes. C3a is a


neutrophil chemoattractant. C3b is an opsonin. C5a is an anaphylatoxin. C5b forms a part of the
membrane attack complex.

136
50- Which of the following immune-mediated events has been demonstrated in psoriasis vulgaris?

A. Clonal expansion of CD8+ T cells


B. Decrease dermal Langerhans cells
C. Downregulation of keratin 16
D. Increase Th2 CD4+ T cells
E. Decreased production of interferon-gamma

►A

The involvement of T cells in the pathophysiology of psoriasis vulgaris is well-recognized.


Availability of monoclonal antibodies has allowed for extensive characterization of T cell subsets
and other mediators increased in psoriasis lesions. CD8+ T cells are highly concentrated in psoriatic
epidermis and studies have demonstrated increased IL-2R and HLA-DR surface molecules
indicative of persistent activation. Clonal expansion of CD8+ T cells has been observed suggesting
that this subset is the major antigen-reactive population.

51- Which cytokine is responsible for fever in patients with sunburn?

A. IL-1
B. IL-5
C. IL-10
D. IL-11
E. TNF-beta

►A

IL-1 is a pyrogenic cytokine responsible for the fever in sunburn. It also is causes B cell maturation
and proliferation and NK cell activation.

52- Which of the following substances is located in the core of an eosinophil?

A. Eosinophilic cationic protein


B. Eosinophil-derived neurotoxin

137
C. Eosinophil peroxidase
D. Major basic protein
E. Chymase

►D

Major basic protein is the only protein located in the core of an eosinophil. Eosinophil cationic
protein, eosinophil-derived neurotoxin, and eosinophil peroxidase are all located in the matrix.
Chymase is a mediator stored in granules of a mast cell.

53 -Which paraprotein is found most commonly in patients with pyoderma gangrenosum?

A. IgG
B. IgA
C. IgM
D. IgE
E. IgD

►B

IgA paraproteinemia has been reported in over 10-18% of pyoderma gangrenosum.

54- Which of the following TH2 cytokines is a B cell growth factor?

A. IL-4
B. IL-5
C. IL-10
D. IL-13
E. IFN-gamma

►A

138
IL-4 is a B cell growth factor. IL-5 is an eosinophil growth factor. IL-10 is a general downregulator
of TH1 immunity. IL-13 (along with IL-4) promotes an isotype switch from IgM to IgE. IFN-
gamma is not a TH2 cytokine. It is secreted by TH1 cells, and is the main macrophage activating
cytokine.

55- Which is not a feature of mast cells?

A. Expresses c-kit
B. Expresses CD-3
C. Produces IL-8
D. Produces prostaglandin D2
E. Stains with napththol chloro-acetate esterase

►B

Mast cells are an integral portion of immediate type hypersensitivity. CD3 is a T cell marker.

56 -Which of the following is associated with hepatitis C infection?

A. Essential mixed cryoglobulinemia


B. Rheumatoid arthritis
C. Relapsing polychondritis
D. Wegener‘s granulomatosis
E. Dermatomyositis

►A

Hepatitis C infections can present as urticaria or papable purpura and cryoglobulinemia.

57- All of the following stains can be reactive in this condition except
A. CDXIIIa
B. CD31

139
C. CD34
D. Ulex europaeus
E. Factor VIII-related antigen

►A

The picture shows Kaposi's scarcoma(KS). It is controversial whether KS represents neoplasia or


hyperplasia; all clinical variants are viewed as a virally induced disease - human herpesvirus 8
(HHV-8) is the suspected agent. Cutaneous lesions present as variably distributed pink patches,
blue-violet to black nodules or plaques, and polyps, depending on clinical variant and stage.
Variable staining can occur with CD31, CD34, Ulex europaeus and factor VIII-related antigen. CD
XIIIa is positive in dermatofibroma.

58- Angiocentric NK/T-cell lymphoma in children may present as:

A. Papular acrodermatitis of childhood


B. Acropustulosis of infancy
C. Childhood dermatomyositis
D. Hydroa vacciniforme
E. En coup de sabre

►D

Hydroa vacciniforme a photodermatitis that typically occurs with sun exposure in the spring has
been reported with NK/T cell lymphomas in childhood.

140
59- Histamine is a biologic amine produced by which of the following cells?

A. Monocytes
B. Eosinophils
C. Basophils
D. Platelets
E. Basophils and Platelets

►E

In the skin, histamine is mainly contained within the granules of dermal mast cells. Histamine is
present in mast cells, basophils, and platelets.

60- The elicitation of nickel contact dermatitis requires signaling by which of the following?

A. TLR2
B. TLR4
C. LFA-1
D. TNF-alpha
E. IL-4

►B

Allergies to nickel (Ni(2+)) are the most frequent cause of contact hypersensitivity (CHS) in
industrialized countries. The efficient development of CHS requires both a T lymphocytespecific
signal and a proinflammatory signal. Ni(2+) triggers an inflammatory response by directly
activating human Toll-like receptor 4 (TLR4). Studies with mutant TLR4 proteins revealed that the
non-conserved histidines 456 and 458 of human TLR4 are required for activation by Ni(2+) but not
by the natural ligand lipopolysaccharide.

61- Which of the following is the target antigen in pemphigus vulgaris?

A. Desmoglein 3
B. Desmoplakin

141
C. loricrin
D. Type XVII collagen
E. Desmoglein 1

►A

Desmoglein 3 is the dominant target auto-antigen in pemphigus vulgaris and is in the cadherin
family.

62- Which cytokine is primarily responsible for stimulation of neutrophils?

A. IL-1
B. IL-4
C. IL-5
D. IL-6
E. IL-8

►E

IL-8 is primarily responsible for the stimulation of neutrophils. IL-5 stimulates eosinophils. IL-4
stimulates mast cells and IgE isotype switching

63- Which virus is most closely associated with Kaposiǁs sarcoma in HIV-infected patients?

A. Human herpes virus 2


B. Cytomegalovirus
C. Human herpes virus 6
D. Human herpes virus 8
E. Epstein-Barr virus

►D

HHV-8 has been repeatedly associated with all forms of Kaposiǁs sarcoma.

142
64- Which component of the T cell receptor is associated with superantigen recognition?

A. D-beta
B. J-alpha
C. J-beta
D. V-alpha
E. V-beta

►E

Superantigens are able to bypass many elements of the normal immune response. They are not
processed by antigen presenting cells. Instead, they bind directly to the MHCII complex and interact
with T cells in a relatively non-specific fashion. Whereas conventional antigens require recognition
in all 5 elements of the T-cell receptor (V-alpha, J-alpha, V-beta, D-beta, J-beta), superantigens are
recognized by V-beta alone.

65- Which of the following cytokines is primarily involved in activating eosinophils?

A. IL-5
B. TNF-alpha
C. IL-10
D. IL-2
E. Interferon-gamma

►A

Eosinophils provide many functions of the immune system, including protection against helminths.
They are activated by IL-5.

66 -Relapsing polychondritis is an autoimmune disease associated with immunity to which type of


collagen?

A. I
B. II

143
C. III
D. IV
E. VII

►B

Relapsing polychondritis has been reported to have autoantibodies to type II collage.

67- A deficiency of this complement component may result clinically in susceptibility to pyogenic
infections, glomerulonephritis, and partial lipodystrophy:

A. C1 Esterase Inhibitor
B. C3
C. C4
D. C50
E. Properidin

►B

C3 is the central component of the complement cascade. It plays a key role in the opsonization of
bacteria. An autosomal recessive deficiency of C3 may result in susceptibility to pyogenic
infections, glomerulonephritis, and partial lipodystrophy.

68- Which of the following cytokines, together with IL-4, promotes isotype switching from IgM to
IgE?
A. IL-5
B. IL-10
C. IL-13
D. TNF
E. IFN-gamma

►C

144
IL-5 is an eosinophil growth factor. IL-10 is a general down-regulator of TH1 immunity. IL-13
(along with IL-4) promotes an isotype switch from IgM to IgE. TNF and IFN -gamma are TH1
cytokines. IFN-gamma is secreted by TH1 cells, and is the main macrophage-activating cytokine.

69- A 26 year-old man presents with a history of recurrent episodes of targetoid, erythematous,
edematous macules, patches, and plaques on the arms, legs, palms, and soles. The most likely
etiologic agent is:

A. Parvovirus B19
B. Herpes simplex virus
C. Coxsackievirus
D. Cytomegalovirus
E. Human immunodeficiency virus

►B

This description of the skin disorder is compatible with either erythema multiforme or erythema
elevatum diutinum, both of which have been reported in association with HSV.

70- Efalizumab is an antibody which is directed against LFA1 on the T -cell, blocking this
molecule's interaction with:

A. CD40
B. B7
C. LFA3
D. ICAM-1
E. P-selectin

►D
ICAM-1 on the antigen presenting cell interacts with LFA1 on the T-cell. By interrupting this
interaction, T-cell activation is blocked by preventing the pairing of LFA1 with ICAM -1. This
prevents costimulatory signals from being given to the T-cell. T-cell ability to traffic into the skin is
also inhibited in the arrest stage of trafficking.

145
71 -This skin disease has been shown to be associated with reduced Beta 2 defensin. The diagnosis
is:

A. Atopic dermatitis
B. Psoriasis
C. Lepromatous leprosy
D. Tuberculoid leprosy
E. Subacute cutaneous lupus erythematosus

►A

The answer is atopic dermatitis. Cathelicidin 37 (LL-37) and Human Beta 2 Defensins (HBD-2)
have been reported reduced in atopic dermatitis and normal or elevated in psoriasis. This could be
one factor that predisposes atopics, but not psoriatics to bacterial and viral infections.

72- Which common contact allergen is detected via the dimethylglyoxime test?

A. Benzocaine
B. Chromates
C. Formaldehyde
D. Nickel
E. Rhus

►D

Nickel is found in virtually all metals of common use. It is the most common sensitizer in women,
and is detected by the dimethylglyoxime test.

146
73- Which component of the classical pathway of complement acts as a neutrophil chemoattractant?

A. C3a
B. C3b
C. C5a
D. C5b
E. C1 INH

►A

The classical pathway of complement is activated by antigen-antibody complexes. C3a is a


neutrophil chemoattractant. C3b is an opsonin. C5a is an anaphylatoxin. C5b forms a part of the
membrane attack complex.

74 -Which of the following features of IgG is true?

A. IgG is not an opsonizing antibody


B. IgG is the only class of immunoglobulin that can pass through the placenta
C. IgG cannot activate the complement cascade
D. IgG represents 15% of the total protein in serum
E. IgG is the second immunoglobulin synthesized by the fetus

►B

IgG opsonizes bacteria, fixes complement, neutralizes bacterial toxins and viruses, crosses the
placenta. It has the highest serum concentration of all immunoglobulins.

75- A 5 year old patient is seen at a tertiary referral center for evaluation of a complex syndrome.
Ultimately, he is diagnosed with IPEX syndrome (immune dysregulation, polyendocrinopathy,
enteropathy, X-linked syndrome.) What gene is mutated in IPEX syndrome?

A. FOXP3
B. LFA-3
C. TGF-beta

147
D. B7
E. Mac-1

►A

IPEX syndrome is a rare x-linked disorder. Mutation of FOXP3 results in absence of regulatory T
cells and is the cause of this syndrome, which includes immune dysregulation, polyendocrinopathy,
and enteropathy.

76- Which cytokine is the main macrophage-activating cytokine?

A. TNF
B. IFN-gamma
C. IL-4
D. IL-10
E. lymphotoxin

►B

IFN-gamma is the main macrophage-activating cytokine, and is secreted by TH1 cells.

77- Which of the following is not true about the effects of ultraviolet radiation on the immune
system?

A. UV radiation causes an increase in number of Langerhans cells in the epidermis


B. UV radiation causes nuclear DNA damage
C. Effects can be demonstrated by the example of reactivation of latent herpes simplex
infection after sun exposure
D. UV radiation acts to suppress the immune system both locally and systematically
E. Effects can be demonstrated by the ability of an antigen to induce an allergic
hypersensitivity reaction when applied to skin which has been exposed to low doses of UV
radiation

148
►A

Effects of UV radiation on the immune system: UV radiation inhibits function of Langerhans cells
(A-False). It causes DNA damage, can reactivate HSV, suppresses immune system, and can induce
allergic hypersensitivity reaction

78- What is the mechanism of action of Ipilimumab?

A. BRAF inhibitor
B. anti-CTLA 4 receptor antibody
C. blocks TNF alpha
D. inhibition of hedgehog signaling pathway
E. inhibits IL2

►B

Ipilimumab is an anti-CTLA 4 receptor antibody. Side effects include rash and coitis. Vemurafenib
is a BRAF inhibitor specifically for patients with V600E mutation. Vismodegib inhibits the
hedgehog signaling pathway.

79- Which of the following dietary supplements may inhibit platelet function?

A. Vitamin A
B. Vitamin C
C. Vitamin D
D. Vitamin E
E. Vitamin K
►D

Supplemental vitamin E can inhibit platelet function and predispose to hemorrhagic stroke. It can be
particularly hazardous in patients with beta-thalassaemia mutations.

80- Serum IgA antibodies to tissue transglutaminase occur in:

149
A. Bullous pemphigoid
B. Linear IgA disease
C. Pemphigus foliaceus
D. Bullous lupus erythematosus
E. Dermatitis herpetiformis

►E

Autoantibodies to tissue transglutaminase are an area of active investigation in both celiac disease
and dermatitis herpetiformis.

81- The endothelial ligand for cutaneous lymphocyte antigen (CLA) is:

A. Intercellular adhesion molecule 1 (ICAM-1)


B. L-selectin
C. E-selectin
D. Vascular cell adhesion molecule (VCAM-1)
E. Leukocyte functional antigen (LFA 3)

►C

Cutaneous lymphocyte antigen (CLA) allow memory T cells to home to the skin, where it binds to
its ligant E-selectin on cutaneous microvessels. Transmigration of memory T cells into the dermis,
however, further requires interaction between leukocyte functional antigen 1 (LFA-1) and ICAM-1,
and B-integrin very late antigen 4 (VLA-4) and VCAM-1. L-selectin is expressed on post-capillary
venules in the lymph nodes and serves as the attachment points for naive T cells.

82- Which of the following causes a photoallergic contact dermatitis that is exacerbated by UVA
radiation?

A. Ascorbic acid
B. Titanium dioxide
C. Oxybenzone
D. Zinc oxide

150
E. Dihydroxyacetone

►C

Oxybenzone is the most common sunscreen agent causing photoallergic contact dermatitis. Patients
sensitive should be instructed to avoid sunscreens containing oxybenzone.

83- For this patient, you request that the lab perform indirect immunofluorescence using what
substrate?

A. Monkey esophagus
B. Rat bladder
C. Guinea pig esophagus
D. Mouse epithelium
E. Hep-2 cells

►C

This patient has pemphigus folicaceus, and indirect immunofluorescence works best on guinea pig
esophagus.

84- Rituximab works by targeting the CD20 antigen which is predominantly expressed on which of
the following cells?

A. Plasma cells
B. B cells
C. B cells and plasma cells
D. T cells
E. Dendritic cells

►B

151
Rituximab targets the CD20 antigen which is predominantly expressed on mature B cells but not
plasma cells, T cells, or dendritic cells. Therefore, even when used in antibody-mediated diseases
(such as pemphigus vulgaris), the response to therapy requires a long duration of time as the drug
targets B cells which must then differentiate into plasma cells that ultimately produce the
pathogenic antibodies in their secreted form.

85- Which antibody can bind the FcER1 portion of mast cells, basophils, Langerhans cells, dermal
dendritic cells?

A. IgA
B. IgD
C. IgE
D. IgG
E. IgM

►C

IgE is an anaphylactic antibody that is involved in nearly all immediate allergic and anaphylactic
type reactions and commonly seen in elevated levels in patients with atopic dermatitis. Mast cells,
basophils, Langerhans cells, dermal dendritic cells as well as monocytes from atopic individuals all
express high-affinity FcERI receptor which can bind IgE. More recently, it became clear that can
bind monomeric IgE via the high-affinity FcRI IgG is the antibody that can cross the placenta and
the most common antibody found in circulation. IgA is found in mucous membrane secretions and
is able to agglutinate antigens and activate the alternate but not the classic complement pathway.
IgD is not found in circulation other than in hyper-IgD syndrome, an autosomal recessive disorder
caused by mutations in the mevalonate kinase gene. A significant elevation of serum IgD is seen in
95% of these patients. IgM is the antibody produced in the early stages of antibody responses. It is a
pentamer which can agglutinate antigen and active the classic complement pathway.

86- What is the best screening test for hereditary angioedema?

A. C1 esterase
B. C4
C. CH50
D. C2

152
E. C3

►B

C4 is the best screening test for hereditary angioedema (Quinke's edema). There are two types of
hereditary angioedema. In type I, there are low antigenic and functional levels of a NORMAL C1
esterase inhibitor protein. In type II, there is a normal or elevated level of a DYSFUNCTIONAL C1
esterase inhibitor. The low C4 level is a result of continuous activation and consumption of
complement components.

87- With which HLA type is psoriasis most definitively linked?

A. HLA-B51
B. HLA-B8
C. HLA-DR4
D. HLA-DR3
E. HLA-Cw6

►E

Psoriasis is linked with HLA-Cw6. Patients with this HLA type have a relative risk of having
psoriasis that is 9-15 times normal.

88- The major histocompatibility complex (MHC)consists of a linked set of genes encoding for
MHC Class I, Class II, Class III, and Class IB. Which of the following is/are true:

A. Class II molecules are present on all nucleated cells


B. Class I molecules are present on erythrocytes
C. Class I molecules are expressed on B8 cells, monocytes and dendritic cells
D. The level of Class I and II expression can be modulated by cytokines
E. The MHC region is located on chromosome 17 in humans

►D

153
The major hisotcompatibility complex (MHC) consists of a linked set of genes encoding for MHC
Class I, Class II, Class III, and Class IB. The level of Class I and II expression can be modulated by
cytokines.

89- All of the following statements regarding major histocompatibility complex molecules are true
EXCEPT:

A. MHC class II molecules bind stably to peptides derived from proteins sythesized and
degraded in the cytosol
B. MHC class I molecules bearing viral peptides are recognized by cytotoxic Tcells that
subsequently kill the infected cell
C. Class I molecules such as HLA-A, B, and C, are present on all nucleated cells
D. The MHC is located on chromosome 6 in humans
E. MHC Class II molecules bearing peptides are recognized by TH1 or TH2 cells.

►A

MHC class I molecules bind stably to peptides derived from proteins sythesized and degraded in the
cytosol, while MHC class II molecules bind stably to peptides derived from proteins degraded in
endocytic vesicles. MHC class I molecules bearing viral peptides are recognized by cytotoxic T-
cells that subsequently kill the infected cell. MHC Class II molecules bearing peptides trigger
helper T-cells and are recognized by TH1 or TH2 cells. Class I molecules such as HLA-A, B, and C,
are present on all nucleated cells, whereas Class II molecules are on B cells, monocytes, dendritic
cells, and are inducible on keratinocytes and endothelial cells. The MHC is located on chromosome
6 in humans, its polygenic and polymorphic nature contributes to the ability of the immune system
to respond to a multitude of different and rapidly evolving pathogens.

90- Eight complement receptors have been described. CR1 is the main receptor for which
component of complement?

A. C3b
B. Factor B
C. Properdin
D. C1

154
E. C5b

►A

CR1 (also known as CD35) is the main receptor for C3b. It plays an important role in mediating
clearance of immune complexes, phagocytosis, and immune adherence of antibody-coated bacteria
to erythrocytes.

91- The most likely target for exfoliative toxin A in bullous impetigo is:

A. Desmoglein 3
B. Laminin 5
C. Desmoglein 1
D. Collagen VII
E. Cesmocollin

►C

Desmoglein 1, the dominant target antigen of the autoantibody involved in pemphigus foliaceus, is
also the target of the exfoliative toxin of group II staphylococcus aureus that is usually responsible
for bullous impetigo.

92- Linear IgA disease is most closely associated with which of the following medications?

A. Erythromycin
B. Vancomycin
C. Streptomycin
D. Azithromycin
E. Clarithromycin

►B

155
Vancomycin is an antibiotic frequently used in subjects allergic to penicillin. It has been reported to
produce subepidermal blistering disease with linear IgA deposits.

93- Which of the following is a criterion for the diagnosis of Behcet'sdisease?

A. Inflammatory bowel disease


B. Uveitis
C. Conjunctivitis
D. Nasal septal perforation
E. Lobular panniculitis

►B

Behcet's is a triad that includes oral ulcerations, genital ulcerations and uveitis. Behcet's also
displays arthritis and gastrointestinal disease. Blindness is the most-feared outcome, and relates to
the uveitis.

94- A patient with Stage 3 mycosis fungoides is referred to a tertiary referral center for further
management. Denileukin diftitox is offered as a potential therapy. This medication binds to what
receptor on the T cell?

A. IL-2
B. MHC II
C. CD20
D. TNF-alpha
E. CD19

►A

Denileukin diftitox is a fusion of a fragment of diphtheria toxin and IL-2. It binds to high affinity
IL-2 (CD25) receptors on T cells. As the medication is internalized, the toxin leads to cell death.
This medication has been approved for cutaneous T cell lymphoma.

156
95- The antibody produced in the early stages of antibody responses is:

A. IgA
B. IgD
C. IgE
D. IgG
E. IgM

►E

IgM is the antibody produced in the early stages of antibody responses. It is a pentamer which can
agglutinate antigen and active the classic complement pathway. IgG is the antibody that can cross
the placenta and the most common antibody found in circulation. IgA is found in mucous membrane
secretions and is able to agglutinate antigens and activate the alternate but not the classic
complement pathway. IgD is not found in circulation other than in hyper-IgD syndrome, an
autosomal recessive disorder caused by mutations in the mevalonate kinase gene. A
significant elevation of serum IgD is seen in 95% of these patients. IgE is an anaphylactic antibody
that is involved in nearly all immediate allergic and anaphylactic type reactions.

96- Eosinophils are typically seen in the cutaneous infiltrate of:

A. Krabbeǁs disease
B. Kaposiform hemangioendothelioma
C. Kawasakiǁs disease
D. Kimuraǁs disease
E. Ki-1 lymphoma

►D

Eotaxin, which attracts eosinophils, is produced by T cells in Kimuraǁs disease.

97- Allergic contact dermatitis is caused by T-cell response to topical exposure to compounds that
form complexes with host proteins (haptens). Which cell is responsible for initial sensitization of
the T-cells?

157
A. Melanocyte
B. Langerhans Cell
C. B-cell
D. Mast Cell
E. Macrophage

►B

The Langerhans cell phagocytizes the haptens in the epidermis or dermis, begins the process of
maturation and migrates to the regional lymph node, where naǁve T-cells are educated to expand to
a clone of CLA positive memory T-cells. These cells then can expand and extravasate with
subsequent exposure to the same allergen. Melanocytes, B-cells, Mast cells and macrophages are
not involved in the sensitization process of allergic contact dermatitis.

98- Which of the following diseases does NOT respond with a Th1-type responses?

A. Leishmaniasis which self-resolves


B. Lepromatous Leprosy
C. Tuberculoid Leprosy
D. Allergic contact dermatitis
E. Psoriasis

►B

Lepromatous leprosy is Th2 biased. The remaining conditions are Th1 predominant responses.
Leishmaniasis which show strong cell-mediated immunity to the parasite and with self-resolving
lesions are Th1 responses. In indolent/progressive leishmaniasis, a Th2 pattern is predominant.

99- A very common pentadecacatechol sensitizer is found in all of the following plants/pl ant
components except:

A. Gingko fruit pulp


B. Poison sumac
C. Mango fruit pulp

158
D. Cashew oil
E. Indian marking nut oleoresin

►C

The sensitizer in rhus dermatitis is a pentadecacatechol found in the oleoresin of anacardaciae


plants, including poison ivy/oak/sumac, mango rinds (but not pulp), cashew nutshells and oil,
Indian marking nut oleoresin, gingko fruit pulp, and Japanese lacquer tree oleoresin.

100- A male infant presents with thrombocytopenia, eczema, and recurrent infections. You suspect
which of the following immunodeficiency disorders?

A. Ataxia telangiectasia
B. Di-George anomaly
C. Hyper-IgM syndrome
D. Leiner s disease
E. Wiskott-Aldrich syndrome

►E

Wiskott-Aldrich syndrome is an X-linked recessive disorder that presents with thrombocytopenia,


small defective platelets, eczema, autoimmune disease, infections, and lymphoreticular malignancy.

101- Which of the following diseases occur with an increased frequency in persons deficient in C2?

A. Psoriasis
B. Dermatitis Herpetiformis
C. Androgenetic Alopecia
D. DLE
E. Leiner's disease

►D

159
Among the complement deficiencies, C2 deficiency is most frequently seen. Most of these patients
are healthy. Diseases that occur with increased frequency in patients with C2 deficiency are SLE,
SLE-like syndrome, frequent infections, anaphylactoid purpura, lethal dermatomyositis, vasculitis,
disseminated cutaneous lupus erythematosus, and cold urticaria.

102- Herpes gestationis is most commonly associated with which HLA?

A. HLA-DR3
B. HLA-B27
C. HLA-B51
D. HLA-DR9
E. HLA-DQ8

►A

HLA-DR3 is the most commonly found HLA association in herpes gestationis. HLA-DR4 is also
found in addition to HLA-DR3 in about 50% of patients. There is nearly 100% incidence of
antiHLA antibodies patients affected by herpes gestationis.

103- All of the following statements regarding Toll receptors are true EXCEPT:

A. Toll receptors are present on macrophages and dendritic cells


B. Toll 2 receptors are typically activated by lipopolysaccharide
C. Toll 4 receptors are typically activated by gram negative bacteria
D. Nuclear factor kappa B (NFKB) is the final common pathway of toll receptors
E. None of the above (all are true)

►B
Bacteria can induce inflammation through activation of Toll receptors, which are present on a
variety of cutaneous cells including macrophages, dendritic cells, keratinocytes, and mast cells.
Typically, Toll 2 receptors are activated by peptidoglycans and lipoproteins on the surface of gram
positive bacteria and yeast. Toll 4 receptors are activated by lipopolysaccharides on the surface of
gram negative bacteria. Nuclear factor kappa B is a final common pathway of toll receptors and
other immune receptors involved in initiating a variety of proinflammatory cytokines.

160
104- Which of the following cytokines has been shown to be critical for epidermal acanthosis in
psoriasis?

A. TNF-alpha
B. TGF-beta
C. IL-2
D. FGF
E. IL-22

►E

IL-22 has recently been shown to be critical in causing epidermal acanthosis. Interleukin (IL)-23, a
cytokine involved in the development of IL-17-producing T helper cells (Th17 cells), was found to
have a potential function in the pathogenesis of psoriasis. IL-22 is preferentially produced by Th17
cells and mediates the acanthosis induced by IL-23. IL-23 can directly induce the production of
IL-22 in human naive T cells. Furthermore, IL-22 mediates IL-23-induced acanthosis and dermal
inflammation through the activation of Stat3.

105- Imiquimod induces which of the following cytokines?

A. Interleukin-10
B. Interferon-alpha
C. Interleukin-2
D. Interleukin-4
E. Interleukin-5

►B

Imiquimod is a topical immunomodulator that increases a variety of cytokines including interferon-


alpha.

106- Which systemic anti-inflammatory agent targets CD2+ activated T cells for apoptosis?

A. Etanercept

161
B. Infliximab
C. Efalizumab
D. Alefacept
E. None of the above

►D

Alefacept is a soluble form of LFA-3 that blocks the immunologic synapse between CD2 on the T
cell and LFA-3 on the antigen presenting cell. Furthermore, alefacept targets CD2+ activated T cells
for apoptosis.

107- The target antigen of chronic bullous disease of childhood is:

A. BPAG 1 9230 kd BPAG)


B. 97 kd LAD-1 (a component of BPAG2)
C. Collagen type VII
D. Alpha 6 beta 4 integrin
E. Plectin

►B

This rare chronic bullous disease of childhood is a subepidermal blistering disease with a
homogeneous IgA deposits at the epidermal basement membrane. This occurs in children usually
less than 5 years of age.

108- Psoriatic arthritis is most commonly associated with which HLA?

A. HLA-B27
B. HLA-Cw6
C. HLA-Aw19
D. HLA-Bw35
E. None of these options are correct

162
►A

HLA-B27 is associated with an increase in psoriatic arthritis as well as pustular psoriasis and
acrodermatitis continua of Hallopeau.

109- Dermatitis herpetiformis is associated with which HLA type(s)?

A. HLA-B8
B. HLA-DR3
C. HLA-DQw2
D. All of the above
E. None of the above

►D

Dermatitis herpetiformis is associated with HLA-B8, -DR3, and -DQw2.

110- This patient developed an acute vesicular rash after eating a mango. She has returned for a
routine follow-up. She needs to be careful of exposure to:

A. Ginkgo fruit
B. Croton
C. Ragweed
D. Tea tree oil
E. All of these answers are correct

►A

163
Patients allergic to the peel of a mango can also be allergic to other plants/products of the
Anacardiaceae family. Cross-reactions can occur with exposure to any plants of the genus
Toxicodendron, to the oil from the cashew nut shell, to the Brazilian pepper tree, to lacquer from the
Japanese lacquer tree, to ink from the Indian marking nut, and to the fruit pulp of the ginkgo tree,
and others.

111- Psoriasis affects over 2% of the worldǁs population and has a strong association with which
HLA class I haplotype?

A. HLA-DR4
B. HLA-DR1
C. HLA-CW6
D. HLA-B27
E. HLA-DQ6

►C

HLA-CW6 has been seen in up to 90% of patients with early onset psoriasis, 50% with late onset
psoriasis and only 7.4% of the general population. HLA-DR1 and DR4 are both related to
Rheumatoid Arthritis and have a 7x increased relative risk (RR) for developing disease. HLAB27 is
linked to ankylosing spondylitis, postinfection arthridites and Reiterǁs disease, with increase relative
risks of 100x, 10-20x, and 35x respectively.

112- Which cytokine is up-regulated in this geometric, eczematous dermatitis?

A. IL-2
B. IL-4
C. IL-1
D. TNF-Alpha
E. IFN-Gamma

►B

164
The geometric pattern of erythema suggests "outside job". Allergic contact dermatitis is a delayed
type hypersensitivity reaction mediated by Th2. The cytokines up-regulated in this process are IL-4,
5, 10.

113 -Major histocompatibility complex (MHC) Class I molecules:

A. Are inducible on keratinocytes


B. Complexed with antigen trigger cytotoxic T cells
C. Are recognized by receptors on CD4+ T cells
D. Bear peptides derived from pathogens taken up into vesicles
E. All of the above

►B

MHC Class I molecules are present on all nucleated cells. They are recognized by receptors on
surfaces of CD8+ T cells, and, when complexed with antigen, trigger cytotoxic T cells. The other
statements apply to MHC Class II molecules.

114- MHC Class II molecules are present on which of the following cell types:

A. B cells
B. T cells
C. NK cells
D. Mast cells
E. All of the above

►A

MHC Class II molecules are on B cells, monocytes, dendritic cells, and are inducible on
keratinocytes and endothelial cells.

115- The gene for NEMO (NF-kappa bets essential modulator) is mutated in:

165
A. Papillon-Lefevre syndrome
B. Pachyonychia congenital type II
C. Dyskeratosis congenital
D. Noonan's syndrome
E. Incontinentia pigmenti

►E

NEMO/IKK gamma is an essential component of the nuclear factor kappa B pathway, which is a
common signaling pathway for many cytokines. Mutation in this pathway have been found to cause
incontinentia pigmenti.

116- All of the following statements regarding Natural Killer (NK) cells are true EXCEPT:

A. NK cells have properties of innate and acquired immunity


B. NK cells express CD 3 molecules
C. NK cells are large granular lymphocytes
D. NK cells mediate tumor lysis
E. NK cells mediate lysis of viral-infected cells

►B

NK cells do not express CD 3 molecules. The other statements are true. Nk cells express CD2.

117 -Which cytokine is upregulated in lesions of tuberculoid leprosy?

A. IL 2
B. IL 4
C. IL 5
D. IL 10
E. None of the answers are correct

►A

166
Tuberculoid leprosy is characterized by a type I immune related response. Tuberculoid leprosy
has increased amounts of TH1 cytokines including IL-2, interferon gamma, and IL-12. These
lead to a t-cell and macrophage mediated cytotoxic response. IL-4, IL-5, and Il-10 are typically
downregulated in tuberculoid leprosy.

118 -Which antibody is most commonly found in circulation of patients with atopic dermatitis?

A. IgA
B. IgD
C. IgE
D. IgG
E. IgM

►C

IgE is an anaphylactic antibody that is involved in nearly all immediate allergic and anaphylact ic
type reactions and commonly seen in elevated levels in patients with atopic dermatitis. Mast
cells, basophils, langerhans cells, dermal dendritic cells as well as monocytes from atopic
individuals all express high-affinity FcRI receptor which can bind IgE. More recently, it became
clear that can bind monomeric IgE via the high-affinity FcRI IgG is the antibody that can cross
the placenta and the most common antibody found in circulation. IgA is found in mucous
membrane secretions and is able to agglutinate antigens and activate the alternate but not the
classic complement pathway. IgD is not found in circulation other than in hyper-IgD syndrome,
an autosomal recessive disorder caused by mutations in the mevalonate kinase gene. A significant
elevation of serum IgD is seen in 95% of these patients. IgM is the antibody produced in the early
stages of antibody responses. It is a pentamer which can agglutinate antigen and active the classic
complement pathway.

119- All of the following statements regarding mast cells are true EXCEPT:

A. Mast cells reside near small blood vessels


B. Mast cells protect mucosal surfaces against pathogens
C. Mast cells release substances that affect vascular permeability
D. Mast cells have receptors for certain fragments of complement on their surface
E. None of the above (all are true)

167
►E

Mast cells are the central cell in immediate-type hypersensitivity, and are very important in
cutaneous inflammation. They reside near small blood vessels, and when activated release
substances that affect vascular permeability. They orchestrate allergic responses and protect
mucosal surfaces against pathogens. They have receptors for IgE and certain fragments of
complement on their cell surface.

120- In adult patients with Henoch-Schönlein purpura with IgA vasculitis, which of the following
complications is most likely to occur?

A. Pulmonary hemorrhage
B. Hemorrhagic cystitis
C. Peripheral neuropathy
D. Mesangial nephropathy
E. Facial edema

►D

A urinalysis should be ordered in patients with palpable purpura. Red blood cells and proteinuria are
often seen in Henoch-Schönlein syndrome.

121- Langerhans cells are characterized by the racket-shaped Birbeck granules seen on electron
microscopy. What role do Birbeck granules perform in cells?

A. Endosomal recycling
B. Peptide packaging
C. Cellular trafficking
D. Initiation of apoptosis
E. Scaffolding for RNA synthesis

►A

168
Langerhans cells are important dendritic antigen-presenting cells in the epidermis and mucosa. The
characteristic Birbeck granules are part of the endosomal recycling compartment.

122- The most useful pair of immunohistochemical stains to distinguish between Dermatofibroma
protuberans (DFSP) and a fibrous histiocytoma would be:
A. Synaptophysin, chromogranin
B. CD34, factor XIIIa
C. CD34, CD31
D. CD31, CD3
E. Vimentin, synaptophysin

►B

Dermatofibroma, the more common benign histiocytoma, is usually CD34 negative and factor XIIIa
positive. DFSP is usually CD34 positive and factor XIIIa negative.

123- A patient with chronic atopic dermatitis is found to have elevated IgE levels. Which cytokine
promotes isotype switching to IgE?

A. IL-4
B. IL-6
C. IL-5
D. Interferon-gamma
E. IL-23

►A

In a typical humoral response, isotype switching occurs subsequent to exposure to antigen.


Switching is regulated by T cell derived cytokines. IgG is promoted by IL-4, IL-6, IL-2 and
IFNgamma. IgA is promoted by IL-5 and TGF-beta. IgE is promoted by IL-4.

124- Which of the following best describes the mechanism of action for infliximab?

169
A. Inhibition of calcineurin-mediated dephosphorylation of transcription factors
B. Inhibition of retinoic acid 4-hydroxylase activity
C. Inhibition of tumor necrosis factor alpha activity
D. Selective elimination of activated T cells via binding to high affinity IL-2 receptor
E. Selective T-cell up-regulation of IL-4 and IL-5 production

►C

Infliximab is a monoclonal antibody given intravenously that binds to membrane-bound and soluble
TNF-alpha.

125-The best screen for classical pathway complement deficiency or dysfunction is:

A. CD4/CD8 ratio
B. CH50
C. C2 esterase levels
D. Total C4 levels
E. Serum electophoresis

►B

The appropriate screening test for a complement deficiency is the CH50. The CH50 will detect
deficiencies in the classical pathway and membrane attack pathways.

126-Which of the following immunoglobulin crosses the placenta?

A. IgA
B. IgD
C. IgE
D. IgG
E. IgM

►D

170
IgG is the antibody that can cross the placenta and the most common antibody found in circulation.
IgA is found in mucous membrane secretions and is able to agglutinate antigens and activate the
alternate but not the classic complement pathway. IgD is not found in circulation other than in
hyper-IgD syndrome, an autosomal recessive disorder caused by mutations in the mevalonate
kinase gene. A significant elevation of serum IgD is seen in 95% of these patients. IgE is an
anaphylactic antibody that is involved in nearly all immediate allergic and anaphylactic type
reactions. IgM is the antibody produced in the early stages of antibody responses. It is a pentamer
which can agglutinate antigen and active the classic complement pathway.

127- Imiquimod has been used for many dermatologic conditions. It works by binding to toll-like
receptor 7. What is the native ligand of TLR-7?

A. Single-stranded RNA
B. TNF-alpha
C. IL-10
D. IL-23
E. Interferon-alph

►A

Toll-like receptors are pattern recognition receptors involved in innate immunity. They are most
highly expressed on monocytes, dendritic cells, and B cells. Imiquimod works through binding to
toll-like receptor 7. The native ligand for TLR7 is single-stranded RNA.

128- Which of the following HLA alleles is most strongly associated with Behcet‘s disease?

A. HLA-B27
B. HLA-B51
C. HLA-DQw2
D. HLA-DR1
E. HLA-DR4

►B

171
HLA-B51 has been reported with increased relative risks in European, Asian and Middle Eastern
populations.

129- All of the following statements regarding neutrophils are true EXCEPT:
A. Neutrophils' major function is antigen presentation
B. Neutrophils have receptors for IgG and complement
C. Neutrophils are granulocytes
D. Neutrophils are the most abundant leukocytes
E. Neutrophils play an important role in wound healing

►A

Neutrophils' major function is phagocytosis. The other statements are correct.

130- Of the following complement components, the most powerful neutrophil chemoattractant is:

A. C3
B. C5a
C. C3a
D. C5b, C6, C7, C8, C9
E. C4a

►B

The complement system plays an important role in innate immunity. Of this group, C5a is the most
powerful neutrophil chemoattractant. C3 represents the endpoint for the classic, alternative and
lectin pathways, and results in the generation of immunologically active substances. C3a and C4a
cause mast cell degranulation. Assembly of C5b, C6, C7, C8, C9 components results in the
membrane attack complex (MAC), which perforates cell membranes causing death by osmotic
lysis.

172
131- Patients with a type I reaction to latex may have cross-reactions with which of the following
foods?

A. Avocado
B. Horseradish
C. Cashews
D. Parsnips
E. Garlic

►A

Latex allergy is a TH2 form of contact sensitivity that is IGE mediated. Using the old Gel and
Coombs classification, it was classified as type I hypersensitivity.

132 -All of the following statements are true regarding cells of the innate immune system EXCEPT:

A. Neutrophils have receptors for IgG


B. Basophils are a type of granulocyte, as are neutrophils
C. IL-5 downregulates the functions of eosinophils
D. Langerhans cells are poorly phagocytic
E. Langerhans cells express CD1 on their surface

►C

Neutrophils, eosinophils, and basophils are collectively known as granulocytes. Neutrophils have
receptors for IgG and complement. IL-5 enhances all functions of eosinophils. Langerhans cells are
dendritic cells found in high concentration in epithelial surfaces and some areas of lymph nodes and
spleen. They have a high density of class II MHC molecules and express CD1 on their surface.
They are poorly phagocytic.

133- This lesion is hypoesthetic and is associated with which of the following:

A. Gamma interferon
B. Interleukin 2

173
C. Interleukin 12
D. All of these answers are correct
E. None of these answers are correct

►D

The lesion is high immune or tuberculoid leprosy, which is associated with TH1 cytokines including
IFN-gamma, IL2, IL12, IL15, IL18 and IL23.

134- A patient with severe chronic psoriasis has failed multiple systemic therapies. Alefacept is
initiated for a three month trial. This medication induces apoptosis of T cells by binding to what T
cell receptor molecule?

A. CD2
B. CD45RO
C. LFA-3
D. CTLA-4
E. p40

►A

Alefacept is a human fusion protein of LFA-3 with Fc protion of IgG1. It binds CD2 on CD45RO
memory effector T cells, thus blocking the 2nd signal of T cell activation and leading to apoptosis.
A provider must monitor the CD4 T cell count while a patient is on this medication.

135- Purpuric contact dermatitis is most likely to be associated with:

A. Nickel
B. Formaldehyde
C. P-phenylenediamine
D. Propylene glycol
E. Sorbic acid

174
►B

Textile dermatitis has been reported to be associated with purpuric contact dermatitis.

136- Which of the following is true of Langerhan cells?


A. They are the primary antigen presenting cell in the epidermis
B. They are primarily involved in the innate immune response
C. They do not express the CD1a marker
D. They contain intranuclear birbeck granules
E. They are increased on the palms, soles, genitalia, and buccal mucosa

►A

Langerhans' cell function primarily in the afferent limb of the immune response by providing for the
recognition, uptake, processing, and presentation of antigens to sensitized T lymphocytes. They are
normally found scattered among keratinocytes of the stratum spinosum, or prickle cell layer. of the
epidermis. They can be stained with CD1a or S-100. They originate in the bone marrow. They
contain intracytoplasmic Birbeck granules.

137- Which cytokine is chemotactic for neutrophils?

A. IL-2
B. IL-3
C. IL-5
D. IL-6
E. IL-8

►E

IL-8 is chemotactic for neutrophils. The other cytokines elicit other types of immune cells, IL-2 (T-
ells), IL-3 (mast cells), IL-5 (eosinophils), IL-6 (plasma cells).

175
138- This patient had antibodies to 180 kd antigen. The likely diagnosis is:

A. Bullous pemphigoid
B. Pemphigus vulgaris
C. Pemphigus foliaceous
D. Erythema multiforme
E. Bullous lichen planus
►A

The answer is bullous pemphigoid, which shows a characteristic antibasement membrane antibody
to Bp180.

139- The main cytokine secreted by Th1 CD8+ effector T-cells is:

A. IL-1
B. IL-2
C. IFN-gamma
D. IL-4
E. IL-5

►C

Interferon-gamma is the main cytokine secreted by Th1 CD8+ effector T -cells and is active in
blocking viral replication. IL-2 is also released in Th1 reactions and causes T cell proliferation and
differentiation, macrophage, NK, & Tc cell activation. It also acts as a Th1 autocrine growth factor.
IL-1 is a pro-inflammatory cytokine and a pyrogen. IL-4 is a B-cell growth factor and active in
signaling isotope switching from IgM to IgE. IL-5 is an eosinophil growth factor. Both IL-4 and -5
are secreted by Th2 T-cells.

140- A patient had antibodies to desmoglein 3, but no antibodies to desmoglein 1 or desmplakin.


The likely diagnosis is:

A. Pemphigus vulgaris
B. Pemphigus foliaceous
C. Bullous pemphigoid

176
D. Dermatitis herpetiformis
E. Erythema multiforme

►A

The answer is pemphigus vulgaris, which typically has autoantibodies to desmoglein 3. While
mixed forms of pemphigus can occur, the histology in this case confirmed pemphigus vulgaris.

141 -Which systemic anti-inflammatory agent specifically blocks the ability of T cells to leave the
vasculature and enter the skin?

A. Etanercept
B. Infliximab
C. Efalizumab
D. Alefacept
E. None of the above

►C

Efalizumab is a humanized monoclonal antibody to CD11a that blocks the immunologic synapse
between LFA-1 on the T-cell and ICAM-1 on the antigen presenting cell. By targeting LFA-1
efalizumab specifically blocks the ability of T cells to leave the vasculature and enter the skin.

142 -All of the following statements regarding Langerhans cells are true EXCEPT:

A. They are highly phagocytic


B. They express CD1 on their surface
C. They are found in some areas of lymph nodes and spleen
D. They have a high density of Class II molecules on their surface
E. None of these answers are correct

►A

177
Langerhans cells are dendritic cells found in high concentrations in epithelial surfaces and some
areas of lymph nodes and spleen. They express CD1 on their surface and have a high density of
Class II MHC molecules. They are poorly phagocytic.

143- An 8 month-old baby with diffuse purpura is admitted to the hospital for her third episode of
bacterial meningitis. Which component of her immune system is impaired?

A. CD4 + T cells
B. Natural killer cell activation
C. CD8 + T cells
D. Complement activation
E. Antibody production

►D

Predisposition to sporadic and occasionally recurrent meningococcal disease occurs in patients with
congenital or acquired complement deficiencies, particularly late acting components C5-9

144- Which of the following is the most abundant leukocyte?

A. Neutrophil
B. Eosinophil
C. Mast cell
D. Lymphocyte
E. Monocyte

►A

Neutrophils are the most abundant leukocyte. The major function of neutrophils is phagocytosis.

145 -This disease is most associated with:

178
A. Gamma interferon
B. IL-4
C. IL-5
D. IL-13
E. None of these answers are correct

►A

The answer is gamma interferon, which is the TH1 cytokine associated with a hyperproliferative
epidermis and with micaceous scale as seen in psoriasis.

146- Which of the following suture materials induces the least inflammation?

A. Surgical gut
B. Polyglycolic acid (Dexon)
C. Polyglycan 910 (Vicryl)
D. Polypropylene (Prolene)
E. Silk

►D

Prolene is a nonabsorbable suture material that evokes only minimal inflammation.

147- Anti-centromeric antibodies are associated with which rheumatologic disease?

A. CREST
B. Mixed connective tissue disease
C. SLE
D. Dermatomyositis/polymyositis
E. Progressive systemic sclerosis

►A

179
Anti-centromeric antibodies are associated with CREST syndrome. Anti-RNP antibodies are
associated with MCTD, anti-dsDNA, ssDNA, and Sm associated with SLE, anti-Jo-1 associated
with polymositis, and anti-Scl 70 associated with PSS.

148- Which cytokine is predominantly responsible for the Jarish-Herxheimer reaction?

A. TNF-alpha
B. IL-2
C. IL-5
D. IL-6
E. IL-12

►A

The Jarish-Herxheimer reaction may occur after the initiation of treatment of many systemic
infections including syphillis. It is an indirect drug induced effect caused by bacterial endotoxins
or microbial antigens liberated by the destruction of micro-organisms. Clinical manifestations may
include fever, lymphadenopathy, arthralgias, and exacerbation of pre-existing cutaneous lesions.
Tumor necrosis factor-alpha is the primary cytokine that mediates this reaction.

149- IL-23 plays a critical role in the pathogenesis of psoriasis. Which of the following cytokines is
critical for IL-23-mediated epidermal hyperplasia in psoriasis?

A. IL-2
B. IL-6
C. IL-12
D. TNF-alpha
E. IFN-gamma

►B

IL-23 facilitates the differentiation and induces complete maturation of Th17 cells. Lesional
psoriatic skin has increased levels of IL-23. IL-6 is essential for development of the IL-23elicited
responses and is required for the development of epidermlal hyerplasia.

180
150- Calcipotriene-induced improvement in psoriasis is associated with increased lesional levels of
which cytokine?

A. Interleukin-2
B. Interleukin-8
C. Tumor necrosis factor
D. Interleukin-10
E. Interluekin-12

►D

Psoriasis is generally described as a TH1 autoimmune disease where IL-12/IFN-gamma pathway is


dominant. IL-10 is the prototype of TH2 and calcipotriene application results in increased levels of
IL-10, thus decreasing TH1 disease.

151- Which T-cell subset is commonly found in Sezary syndrome?

A. CD4+/CD7+
B. CD8+/CD7-
C. CD4-/CD7+
D. CD4+/CD7-
E. CD8+/CD7+

►D

Sezary syndrome is the leukemia phase of mycosis fungoides (cutaneous T-cell lymphoma) and is
usually a TH2-biased CD4+ leukemia.

152- T-cell anergy occurs if:

A. Stimulation by a MHC Class III molecule is involved


B. MHC/TCR engagement occurs without costimulatory molecules
C. FasL is bound on the T-cell
D. MHC Class I or II is bound in the presence of IL-2
E. A HLA-DM facilitator is not involved with the binding

181
►B

MHC/TCR engagement occurs without costimulatory molecules. MHC Class III molecules are not
involved in this process. The MHC III region encodes for soluble proteins of the complement
cascade and the tumor necrosis family. Fas-FasL interactions promote apoptosis on the target cell,
not stimulation. IL-2 is a stimulatory molecule, produced by Th1 T-cells. It does not produce
anergy. HLA-DM facilitates loading of proteins processed by antigen presenting cells into the MHC
class II molecule before being brought to the cell surface.

153 -Which of the following B cell receptors is involved in immunoglobulin isotype switching?

A. CD40
B. CD19
C. CD20
D. CD154
E. CD22

►A

CD40 activation on B cells by CD40-ligand (CD154) on T cells induces isotype switching from an
IgM to IgG response. Defects in the expression of CD40-ligand result in an immunodeficiency state
(hyper-IgM syndrome) characterized by low levels of IgG, IgA and IgE, but elevated IgM. CD19,
CD20, CD22 are pan-B cell markers.

154- Toll-like receptors (TLRs) have been found to play an important role in innate immunity. This
has been utilized in the development of medications frequently used in dermatology. The
mechanism of what medication involves activation of TLR7.

A. Clobetasol
B. Tacrolimus
C. Cyclosporine
D. 5-Fluorouracil (5-FU)
E. Imiquimod

182
►E

TLRs recognize pathogen-associated molecular patterns (PAMPs) present in a variety of pathogens


and activate signaling pathways involved in innate immunity, as well as augmenting adaptive
immunity. The mechanism of action of imiquimod involves activation of these pathways via TLR7.
None of the other medications listed above involve TLRs as their main mechanism of action.

155- Chronic idiopathic urticaria is associated with which HLA type(s)?

A. HLA-DR4
B. HLA-DRB4
C. HLA-DQ8
D. all of the above
E. none of the above

►D

Chronic idiopathic urticaria is associated with HLA -DR4, -DRB4, and -DQ8.

156 -This patient recently developed this rash. You decide to patch test her, but in the meanwhile
you tell her to avoid:

A. Chamomile
B. Primin
C. Abietic acid
D. Benzocaine
E. Cinnamon

►E

This patient likely has a fragrance allergy. Patients allergic to fragrances also need to avoid certain
spices like cinnamon.

183
157- IL-23 plays a critical role in the pathogenesis of psoriasis. Which of the following cytokines is
critical for IL-23-mediated epidermal hyperplasia in psoriasis?

A. IL-2
B. IL-12
C. IL-17A
D. TNF-alpha
E. IFN-gamma

►C

IL-23 and Th17 cells producing IL-17A and IL-22 are found in excess in skin affected by psoriasis.
IL-6, IL-22, and IL-17A have all been shown to be critical in mediating epidermal hyperplasia in
psoriasis in response to IL-23.

158- Sensation is intact in this lesion, but a Fite stain is positive. This lesions is associated with
which of the following:

A. IL-4
B. IL-5
C. IL-10
D. All of these answers are correct
E. None of these answers are correct

►D

The lesion is low immune or lepromatous leprosy, which is associated with TH2 cytokines
including IL-4, IL-5, IL-10, and IL-13.

159- The putative mechanism of action of topical macrolide immunomodulators is inhibition of:

A. Lymphokine production
B. Prostaglandin secretion
C. Antigen presentation

184
D. Neutrophil migration
E. Lymphocyte migration

►A

Tacrolimus and pimecrolimus are topical macrolide immunomodulators that inhibit lymphokine or
cytokine production via binding to macrophilin. This complex inhibits calcineurin, a phosphatase
involved in the activation of NF-AT. This suppresses the production of IL-2 and IFN-gamma (TH1
cytokines) as well as IL-4, 5 and 13 (TH2 cytokines). In addition, they decrease the expression of
IgE receptors on Langerhans cells and reduce mast cell degranulation.

160- Which Th2 cytokine is involved in the proliferation and differentiation of B cells?

A. IL-4
B. IL-5
C. IL-6
D. IL-10
E. IL-13

►C

The Th2 response is important in promoting humoral immunity. IL-4 promoted Th2 differentiation,
isotype switching and inhibition of macrophages. IL-5 activates eosinophils. IL-6 assists in the
proliferation and differentiation of B cells. IL-10 decreases expression of MHC II. IL-13 is related
to IL-4, and is implicated in allergic inflammation.

161- The Epstein-Barr virus utilizes which cell surface receptor to gain access to cells?

A. CR2
B. B7
C. T-cell receptor
D. TLR7
E. MHC II

185
►A

EBV utilizes the CR2 receptor for cell entry and infection. the CR2 receptor (also known as CD21)
presents antigen to B cells and is a co-receptor for B cell signalling.

162- Which cell surface component of T cells is part of the \"signal 2\" which in addition to T cell
receptor binding leads to T cell activation?

A. CD28
B. B7
C. CD80
D. TNF-alpha receptor
E. CD1a

►A

Activation of T cells requires two signals. The first is T cell receptor binding to MHC molecule on
the antigen presenting cell. THe second signal involves CD28 on T cell interacting with B7
molecules on the antigen presenting cells. CD80 is expressed on Langerhans cells after activation.
CD1a is a Langerhans cell surface marker.

163 -What cytokine is most critical for the development and maturation of eosinophils.

A. Interleukin-2
B. Interleukin-4
C. Interleukin-5
D. Interferon-alpha
E. Interferon-gamma

►C

IL-5 is critical in the development and maturation of eosinophils. IL-2 is an autocrine factor for
activated T cells. IL-4 is involved in B-cell class switching and Th2 differentiation. IFN-alpha is a

186
major part of the antiviral response. IFN-gamma is involved in macrophage activation and specific
isotype switching.

164- Mononuclear phagocytes residing in tissues:

A. Are called macrophages


B. Phagocytose foreign antigens and degrade them into peptides
C. Present antigen to T-cells
D. Produce cytokines, which recruit other inflammatory cells
E. All of the above

►E

Mononuclear phagocytes are components of the innate immune system. Once in tissues they are
called macrophages. All of the above are correct.

165- The classical complement pathway:

A. Can be activated in the absence of antibody


B. Can be activated by IgG4
C. Can be activated by IgM
D. Includes C3 and factor B
E. Does not cause membrane damage

►C

The classical complement pathway is activated by antigen-antibody complexes, while the alternate
complement pathway can be activated in the absence of antibody. IgM and IgG are capable of
activating the classical pathway, but IgG4 does not activate complement. The proteins of the
classical pathway are C1, C2, C3, and C4. Factor B, Factor D, properdin, and C3 are proteins in the
alternate pathway. Biologic activities of complement include opsonization, chemotaxis,
anaphylaxis, immune complex solubilization, membrage damage, and B cell activation.

187
166- Which cytokine is responsible for activating natural killer cells?

A. Interleukin 4
B. Interleukin 2
C. Interleukin 5
D. Interferon-gamma
E. Tumor necrosis factor-alpha

►B

Activation of natural killer cells occurs via interleukin 2 (IL-2). IL-2 is a key component of the Th1
(cell-mediated) immunity. It acts to promote growth, proliferation and activation of T cells, B cells
and natural killer cells.

167- Proteins in the alternate complement pathway include:

A. Factor B
B. properdin
C. C3
D. Factor B and properdin
E. Factor B, properdin and C3

►E

Proteins in the alternate complement pathway are Factor B, Factor D, properdin, and C3. The
alternate complement pathway is active against pathogenic microorganisms, virus-infected cells,
and neoplastic cells. The proteins of the classical pathway are C1, C2, C3, and C4.

168- All the following conditions exhibit a T-helper cell 1 (Th1) cytokine secretion profile except:

A. Psoriasis
B. Systemic lupus erythematosus
C. Granulomatous leprosy
D. Rheumatoid arthritis

188
E. Multiple sclerosis

►B

Systemic lupus erythematous is associated with a Th2 cytokine profile, whereas the other conditions
are associated with a Th1 cytokine profile. Th2 cells are typified by secretion of IL-4, IL-5, IL-6,
IL-9, IL-10 and IL-13, whereas Th1 cells are typified by secretion of IFN-gamma, TNF-beta, and
IL-2.

169-A 44 year old female complains of intra-oral burning lesions. Examination reveals
erythematous plaques with white striae on the buccal mucosa consistent with oral lichen planus. No
cutaneous lesions are seen. Which of the following HLA types is associated with oral lichen planus?

A. B8
B. Cw6
C. Bw35
D. B27
E. B51

►A

Oral lichen planus is a relatively common condition that can cause significant morbidit y. Oral
lichen planus is associated with HLA-B8. HLA-Cw6 is associated with psoriasis, Bw35 with
cutaneous lichen planus, B27 with psoriatic arthritis, and B51 with Behcet\'s disease.

170 -Naive T cells express which of the following surface molecules:

A. CD19
B. CD20
C. CD79
D. CD45RO
E. CD45RA

189
►E

Naive T cells express CD45RA whereas memory T cells express CD45RO. CD19, 20, and 79 are
B cell surface markers.

171 -The T cell surface receptor CD2 binds to what partner on endothelial cells to assist with
adhesion?

A. LFA-3
B. B7
C. E-cadherin
D. CD40
E. ICAM-1

►A

CD2 molecules are T cell surface receptors that bind to LFA-3 on endothelial cells and antigen
presenting cells to participate in adhesion and activation.

172 -Pemphigus is associated with which HLA type(s)?

A. HLA-DRw6
B. HLA-B8
C. HLA-B51
D. All of these answers are correct
E. None of these answers are correct

►A

Pemphigus is associated with HLA-DR4 or DRw6.

190
173 -The human major histocompatibility complex (MHC) is located on chromosome:

A. 2
B. 6
C. 9
D. 11
E. 17

►B

The MHC is highly polygenic and polymorphic, and this contributes to the ability of of the
immune system to respond to a multitude of different and rapidly evolving pathogens. The MHC
is located on chromosome 6 in humans.

191
192
Genoderma ses

1. The most common enzyme abnormality in congenital adrenal hyperplasia is:

A. 3-beta-hydroxysteroid dehydrogenase isomerase


B. 11-beta-hydroxylase
C. 21-hydroxylase
D. 15-hydroxylase
E. 17-hydroxylase

C. 21-hydroxylase

Explanation: 21-hydroxylase deficiency is present in 95% of cases of congenital adrenal


hyperplasia. This defect in adrenal steroidogenesis can occur at any point in life, but affected girls
will generally present around puberty with hirsutism and mentsrual irregularity/primary
amenorrhea. The key feature is excess androgen production.
The other listed enzymes are not the most common mutations in congenital adrenal hyperplasia

2-This 8-year-old patient is brought in by his father for the lesions pictured on the lips. The father
tells you that the patient is healthy aside from an episode of intestinal intussuusception several
months ago. Which of the following is the most likely diagnosis?

A. LEOPARD syndrome
B. Carney complex
C. Peutz-Jeghers syndrome
D. Bannayan-Riley-Ruvalcaba syndrome
E. Laugier-Hunziker syndrome

193
to
C. Peutz-Jeghers syndrome

Explanation: Peutz-Jeghers syndrome results from an autosomal-dominant mutation in the STK11


gene (serine threonine kinase). Clinical manifestations may include: mucocutaneous (oral/acral)
lentigines, intestinal polyposis +/- intussusception, and various malignancies.
The remaining answer choices may produce multiple lentigines, but are not typically associated
with intussusception.

3-Which of the following conditions is NOT found in Von-Hippel Lindau syndrome?

A. Connective tissue nevi


B. Bilateral retinal hemangioblastomas
C. Cerebellar/CNS hemangioblastomas
D. Renal cell carcinoma
E. Pheochromocytoma

A. Connective tissue nevi

Explanation: Connective tissue nevi are not a feature of Von Hippel-Lindau syndrome.
Von Hippel-Lindau syndrome is characterized by all the options listed except connective tissue
nevi. Other findings include pancreatic cysts/carcinoma and cutaneous capillary malformations of
the head and neck and polycythemia.

4-Patients with Cornelia de Lange syndrome have a low set hair line, trichomegaly, hirsutism, and
synophrys. The skin finding is:

A. Cutis marmorata
B. Vitiligo
C. Poikiloderma
D. Scleroderma
E. Hyperpigmentation

A. Cutis marmorata

194
Explanation: Patients with Cornelia de Lange syndrome have cutis marmorata with low set hair
line, trichomegaly, synophrys, hirsutism, and heart defects. The associated gene is NIPBL and
SMC1L1.Poikiloderma occurs in Rothmund-Thomson and Werner syndrome.
Scleroderma occurs in Werner syndrome.

5-Retention of primary teeth is a prominent dental finding of which of the following conditions?

A. Hypomelanosis of Ito
B. Letterer-Siwe disease
C. Tuberous sclerosis
D. Jackson Sertoli syndrome
E. Hyper-IgE syndrome

E. Hyper-IgE syndrome

Explanation: Hyper-Immunoglobulin E syndrome is an autosomal dominant condition with


impaired regulation of IgE function and deficient neutrophil chemotaxis. There is increased
susceptibilty to infections and increased IgE serum levels. Retained primary teeth and lack of
development of secondary teeth are characteristic findings.

6-Mutations in c-kit are associated with which of the following conditions?

A. Waardenburg syndrome
B. Piebaldism
C. Tuberous sclerosis
D. Vogt-koyanagi-harada
E. Cornelia de Lange Syndrome

B. Piebaldism

Explanation: Piebaldism is caused by mutations in c-kit.


Waardenburg syndrome mutations include PAX3, MITF, EDNRB. Tuberous sclerosis mutations are
in TSC1 (which encodes hamartin) and TSC2 (which encodes tuberin). Vogt-koyanagi-harada is an
autoimmune condition. Cornelia de Lange Syndrome mutation is NIPBL.

195
7-Underlying defect for the disease shown in picture is

A. ATP2A2
B. ATP2C1
C. BPAG1
D. BPAG2
E. Collagen type 17

Correct choice: B. ATP2C1

Explanation: The disease shown in image is Hailey-Hailey disease (Familial Benign Pemphigus)
which is an autosomal dominant genodermatosis, caused by mutation in ATP2C1, encoding a
calcium pump protein related to SERCA2. It is characterized by recurrent vesicles and erosions,
which most commonly appear on the sides and back of the neck, in the axillae, in the groin, and in
the perianal regions. The disorder is not seen before puberty and usually has its onset in the late
teens or early 20s. In the intertriginous area lesions tend to form erythematous plaques with dry
crusting and soft, flat, and moist granular vegetations. Burning or pruritus is common, and,
particularly in the intertriginous areas, lesions tend to become irritating, painful, and exceedingly
uncomfortable. ATP2A2 is underlying defect in Darier's disease, other choices are defects seen in
pemphigoid and epidermolysis bullosa.

8 -A 16 year-old girl presents with a family history of Gardner syndrome. Her mother is very
concerned that her daughter may have the syndrome as it runs in her family and she has many skin
complaints. Gardner syndrome has been linked to defects in beta-catenin mediated transcription.
Which of the following ocular finding is diagnostic for Gardner syndrome?

A. Lisch nodules
B. Lester iris
C. Congenital Hypertrophy of the Retinal Pigment Epithelium
D. Angioid streaks

196
E. Retinal detachment

C. Congenital Hypertrophy of the Retinal Pigment Epithelium

Explanation: CHRPE (Congenital Hypertrophy of the Retinal Pigment Epithelium) is the


characteristic eye finding for Gardner syndrome. Lisch nodules are seen in Neurofibromatosis I,
Lester iris in Nail-Patella syndrome and angioid streaks are present in Pseudoxanthoma elasticum.
Retinal detachment is not part of Gardner syndrome.

🔆 Clinical features of Gardner syndrome can be divided into two types, cutaneous and non-
cutaneous. The most noticeable cutaneous feature of Gardner syndrome is the appearance of
epidermoid cysts. These cysts can be differentiated from ordinary epidermoid cysts by the following
factors:
• Epidermoid inclusion cysts of Gardner syndrome (50–65%) occur at an earlier age (around
puberty) than ordinary cysts
• Epidermoid cysts occur in less common locations such as the face, scalp and extremities
compared to ordinary cysts
• Cysts tend to be multiple in over half of the patients with Gardner syndrome
• As with ordinary epidermoid cysts, cysts in Gardner syndrome are
usually asymptomatic (without symptoms), however in some cases they may
be pruritic (itchy) and/or inflamed, and they may rupture.
• Sometimes the cysts have hybrid features with pilomatricoma-like histopathology

Other cutaneous features include desmoid cysts, fibromas, lipomas, leiomyomas, neurofibromas
and pigmented skin lesions.

Non-cutaneous features include:


• Gastrointestinal polyps that nearly always transform into colonic adenocarcinomas
(colon cancer).
• Osteomas – these benign bone tumours are essential in making the diagnosis of
Gardner syndrome. They occur most commonly in the mandible (jawbone) but may also grow
in the skull and long bones.
• Dental abnormalities – as well as osteomas in the jaw there may be other dental
abnormalities such as unerupted extra teeth and caries
• Multifocal pigmented lesions of the fundus in the eye – seen in 80% of patients. These
lesions may be present shortly after birth and can be the first marker of the disease.

197
9-An infant presents with poikiloderma on his face, buttocks, arms and legs. He is also noted to
have a hypoplastic thumb and no radius. Yearly ophthalmologic examination is indicated because of
the infant is at risk for developing:

A. Glaucoma
B. Cataracts
C. Subcapsular lens displacement
D. Copper deposition
E. Macular degeneration

B. Cataracts

Explanation: The patient described has Rothmund-Thomson syndrome (or poikiloderma


congentiale), an autosomal recessive disease localized to chromosome 8 and believed to be due to a
DNA helicase mutation (RECQL4). 40-50% of patients will develop juvenile cataracts before
puberty. Other clinical features include alopecia, dystrophic nails, short stature, hypogonadism and
dental dysplasia.

🔆 Affected children may be identified early in life by their small size, their tendency to sunburn
easily, and from the appearance of their skin, teeth and bones. Rothmund–Thomson syndrome is
slightly more common in females than males.
Skin
• Photosensitivity: sunburn-like redness, swelling and blisters on cheeks and face; may extend
to involve buttocks and extremities. Noted during the first year of life in 90%
• Poikiloderma: variegated pigmentation, telangiectasia (prominent tiny blood vessels) and
skin thinning; usually evident on cheeks, hands and buttocks by 3-5 years of age
• Thin eyebrows and sparse scalp hair
• Abnormal, brittle nails
Eyes
• Cataracts: lens opacities occur in 50% of children aged 3-7 years and are often bilateral.
• Corneal lesions are less common.
Bones
• Bony defects affect over 50% of children, who are often of short stature.
• These include dysplasia (abnormal growth), sclerosis (thickening and hardening)
and cystic abnormalities of the long bones.
• Small hands and feet; absent or malformed radii and thumbs
• Osteoporosis and bone hypoplasia (bone thinning) are common with ageing. Pathological
fractures may occur with minimal trauma.

198
• Other changes include widened long bone epiphyses (part of the bone where growth occurs),
iliac bone hyperplasia (excessive growth), trabeculated metaphyses (middle part of the long
bone).
Dentition
• Agenesia (absent formation of teeth)
• Microdontia (small teeth)
• Delayed and ectopic eruption of teeth
• Supernumerary teeth
Reproductive system
• Hypogonadism in 25%
• Juvenile-appearing genital organs
• Amenorrhoea (lack of menstruation)
• Sterility
Endocrine system
• Parathyroid adenoma
• Disturbed thyroid function
Gastrointestinal system
• Chronic nausea and vomiting
• Diarrhoea
Intellect
• Intellectual impairment in up to 30%

10-A patient is diagnosed with Rothman Thomson syndrome and has juvenile cataracts,
hypogonadism, and dystrophic nails. The hair findings is/are:

A. Sparse scalp, eyebrow, and eyelash hair


B. Unruly hair
C. Uncombable hair
D. Woolly hair
E. Pili trianguli et canaliculi

A. Sparse scalp, eyebrow, and eyelash hair

Explanation: Rothmund-Thompson syndrome is an autosomal recessive disorder with a


heterozygous mutation in the RECQL4 gene, which encodes DNA helicase. They have sparse scalp,
eyebrow, and eyelash hair. Poikiloderma is also seen in sun-exposed areas, dystrophic nails, and

199
juvenile cataracts. They have increased malignancies, such as basal cell carcinoma, squamous cell
carcinoma, and osteogenic sarcoma. The other findings are not seen in Rothmund-Thompson
syndrome.

11-A woman with hypopigmented in lines of Blaschko and scarring alopecia likely suffers which of
the following conditions?

A. Chondrodysplasis punctata
B. Anhidrotic Ectodermal Dysplasia
C. Focal Dermal Hypoplasia
D. Rothmund-Thomson Syndrome
E. Bloch-Sulzberger Syndrome

E. Bloch-Sulzberger Syndrome

Explanation: Bloch-Sulzberger Syndrome (incontinentia pigmenti) is an X-linked dominant


syndrome which typically presents this clinical picture.
The other listed syndromes do not typically present with these two findings.

12-Which syndrome is least exacerbated by UV radiation:

A. Bloom syndrome
B. Hartnup’s disease
C. Refsum syndrome
D. Cockayne syndrome
E. Rothmund-Thomson syndrome

C. Refsum syndrome

Explanation: Refsum’s syndrome is an autosomal recessive disorder caused by mutations in


phytanoyl-CoA hydroxylase. Clinically, patients have mild icthyosis, cerebellar ataxia,
polyneuropathy, salt and pepper retinitis pigmentosa, sensorineural deafness, and arrhythmias with
heart block. They are not overly sensitive to UV radiation. Photosensitivity is a prominent feature of
the other listed conditions.

200
13-A child is brought to your clinic for evaluation. Upon examination of his nails, you notice
triangular lunulae. Which of the following do you also expect to find on exam?

A. Pili torti
B. Pear-shaped nose
C. Anterior iliac horns
D. Absent or hypoplastic patellae
E. Hypopigmented papillary margin of iris

D. Absent or hypoplastic patellae

Explanation: The finding of triangular lunulae should alert one to the diagnosis of Nail-Patella
syndrome, an autosomal dominant condition due to a LMX1B mutation. Absent or hypoplastic
patellae is characteristic of this condition. Pili torti is not found in Nail-Patella syndrome. A pear-
shaped nose is often found in those with trichorhinophalangeal syndrome. Patients with Nail-Patella
syndrome usually also display posterior iliac horns, a hyperpigmented papillary margin of the iris
(termed Lester iris), thickened scapulae, radial head subluxation, and glomerulonephritis.

14-A patient is referred to you for multiple red-brown painful papules on his trunk. You perform a
biopsy, which results as a pilar leiomyoma. After discussing the biopsy results, you refer the patient
to genetics as you suspect he may have a heterozygous germline mutation in which enzyme?

A. steroid sulfatase
B. malate dehydrogenase
C. fumarate hydratase
D. alpha-galactosidase
E. phytanoyl-CoA hydroxylase

C. fumarate hydratase

Explanation: Hereditary leiomyomatosis and renal cell cancer (HLRCC) syndrome, also known as
Reed’s syndrome) is a rare autosomal dominant disorder that predisposes individuals to multiple
cutaneous pilar leiomyomas and uterine leiomyomas (in women). It is caused by a heterozygous
germline mutation in the fumarate hydratase gene which codes for the fumarase enzyme, which
catalyzes the hydration of fumarate to malate in the citric acid/Krebs cycle. Screening
recommendations for renal cell carcinomas vary, but should begin at 10 years of age with annual

201
MRI scans. Steroid sulfatase deficiency is seen in X-linked recessive icthyosis. A defect of
phytanoyl-CoA hydroxylase, a peroxisomal enzyme, is the cause of Refsum disease. A defect in
alpha-galactosidase causes Fabry’s disease. Malate dehydrogenase is an enzyme in the Kreb’s cycle.

15-This syndrome is characterized by reticulated hyperpigmentation and scaly erythematous


lichenoid papules favoring the flexural areas on the trunk, with the histopathologic finding of
prominent acantholysis. It is due to an autosomal dominant mutation in keratin 5. This syndrome is
called:

A. Dowling-Degos disease
B. Galli-Galli disease
C. Haim-Munk Syndrome
D. Gardner Syndrome
E. Degos disease

B. Galli-Galli disease

Explanation: Galli-Galli disease (GGD) is an eponym introduced in 1982 to describe a skin disease
in two affected brothers with reticulated hyperpigmentation and erythematous lichenoid papules
resembling Dowling-Degos disease (DDD), but with the histopathologic finding of prominent
acantholysis. GGD is considered an acantholytic variant of DDD as both are due to an autosomal
mutation in keratin 5.

Dowling-Degos disease (DDD) is a rare autosomal dominant genodermatosis with variable


penetrance. It is characterized by reticular hyperpigmentation and pruritic follicular hyperkeratotic
papules in the axillae, groin, and other body folds. Comedo-like lesions on the back or neck, pitted
perioral scars, and hypopigmented macules are also described. Histopathologically, DDD shows
acanthotic elongation of rete ridges in conjunction with basal hyperpigmentation, and NO
acantholysis. Haim-Munk syndrome is due to an autosomal recessive mutation in cathepsin C, and
clinical findings include erythematous palmo-plantar keratoderma, onychogryphosis, periodontitis,
early loss of teeth, and acro-osteolysis. Gardner syndrome is due to an autosomal dominant
mutation in APC, with clinical findings including GI polyps, increased risk of colon cancer, jaw
osteomas, supernumerary teeth, epidermoid cysts, and congenital hypertrophy of the retinal pigment
epithelium (CHRPE). Degos disease (aka malignant atrophic papulosis) should not be confused
with Dowling-Degos disease. Degos disease is a vaso-occlusive disorder typified by crops of small
erythematous papules that resolve with porcelain white scars (similar to atrophie blanche), and is
associated with GI and CNS symptoms.

202
16-A 11 year-old female patient with hypoparathyroidism is referred to your clinic secondary to
chronic mucocutanous candidiasis which is refractory to standard treatments. The patient also has
malabsorption and severe chronic diarrhea. You determine that she has autoimmune
polyendocrinopathy-candiasis-ectodermal dystrophy syndrome(APECED). Since only two of three
major criteria are needed to make this diagnosis, this patient having demonstrated
hypoparathyroidism and chronic mucocutanous candidiasis is diagnosed with APECED. What is the
third major criteria that would qualify a patient to meet the diagnosis of APECED?

A. Ectodermal dysplasia
B. Insulin dependent diabetes mellitus
C. Chronic autoimmune hepatitis
D. Addison's disease
E. Hypothyroidism

Correct choice: D. Addison's disease

Explanation: This patient has APECED, also know as autoimmune polyendocrinopathy syndrome
type 1(APS type 1). Two of three major criteria are needed to make this diagnosis - chronic
mucocutaneous candidiasis, hypoparathyroidism and Addison's disease. They usually present within
the first 5 years of life with chronic mucocutaneous candidiasis, then before the age of ten with
hypoparathyroidism, and finally in early adulthood with Addison's disease. They often present with
chronic diarrhea and malabsorption. Ectodermal dysplasia usually does not present until the fifth
decade. Autoimmune skin disease, such as vitiligo and alopecia areata are not uncommon.
Autoimmune polyglandular syndrome type 1 (APS1) is an autoimmune condition hat results in
insufficiencies of multiple endocrine glands. It is also known
as autoimmune polyendocrine syndrome type 1, polyendocrinopathy-candidiasis–
ectodermal dystrophy (APECED), Whitaker syndrome, and candidiasis-hypoparathyroidism–
Addison disease syndrome, among its many other names.

APS1 is based on three main clinical features:


• Mucocutaneous candidiasis affecting the skin and mucous membranes
• Hypoparathyroidism, resulting in numbness and tingling in the face and limbs, muscle
cramps and aches, weakness and fatigue due to low levels of circulating calcium
• Addison disease, an insufficiency of the adrenal glands, presenting with changes in
skin pigmentation, loss of appetite and weight loss, fatigue, low blood pressure and fatigue.

While less common, other possible features of this syndrome can include:
• Hypogonadotropic hypogonadism
• Pernicious anaemia

203
• Chronic active hepatitis
• Asplenia
• Keratoconjunctivitis
• Interstitial nephritis
• Diabetes mellitus type 1
• Cholelithiasis
• Alopecia areata
• Malabsorption
• Vitiligo

17-A patient diagnosed with keratitis, ichthyosis, and deafness has a mutation in:

A. Connexin 26
B. NEMO
C. DNA helicase
D. RECQL4
E. Plakoglobin

A. Connexin 26

Explanation: A patient with KID syndrome has an autosomal dominant mutation in connexin 26.
They have keratitis, ichthyosis, congenital sensiorneural hearing loss, and alopecia. They have
increased SCCs and skin infections. Mutations in NEMO occur in Incontinentia Pigmenti (Bloch-
Sulzberger Syndrome) and Hypohidrotic Ectodermal Dysplasia with Immune Deficiency.
Mutations in DNA helicase occur in Bloom syndrome, Rothmund-Thompson syndrome
(poikiloderma congenitale), and Werner syndrome. Mutations in RECQL4 occur in Rothmund-
Thompson syndrome (poikiloderma congenitale). Mutations in Plakoglobin occur in Naxos
Syndrome.

18-Which of the following statements regarding Basal Cell Nevus Syndrome are true?

A. Associated tumors include pancreatic cancer


B. It is inherited in an autosomal recessive manner
C. The mutation is in the p53 gene
D. The affected gene is mutated in 30-40% of sporadic basal cell carcinomas

204
E. Muscular defects are common

D. The affected gene is mutated in 30-40% of sporadic basal cell carcinomas

Explanation: Basal cell nevus syndrome (Gorlin Syndrome) is caused by a mutation in the PTCH
gene, located on chromosome 9q22. This gene is found to be mutated in 30-40% of sporadic basal
cell carcinomas. Gorlin Syndrome is inherited in an autosomal dominant pattern. Tumors associated
with this sydrome include medulloblastoma and meningioma. It is characterized by the appearance
of multiple BCCs during childhood, odontogenic keratocysts of the jaw, and skeletal defects
(including frontoparietal bossing and hypertelorism, among others). Basal cell nevus syndrome
(Gorlin Syndrome) is caused by a mutation in the PTCH gene, located on chromosome 9q22. This
gene is found to be mutated in 30-40% of sporadic basal cell carcinomas.

19-A child presents to clinic with cataracts, scarring alopecia and a flat nasal bridge. The parents
say that at birth she was erythrodermic but resolved by 6 months. Imaging reveals punctate
calcification of epiphyses. What is the error in this condition?

A. Impaired cholesterol synthesis


B. Peroxisomal biogenesis disorder
C. Absence of enzyme 3b-hydroxysteroid-dehydrogenase
D. Absent granular layer
E. Defect in serine protease inhibitor

A. Impaired cholesterol synthesis

Explanation: The patient has Conradi-Hunermann-Happle syndrome. The defect is from a EBP gene
mutation (emopamil-binding protein) resulting in impaired cholesterol synthesis. CHH is also called
X-linked dominant chondrodysplasia punctata type II. After resolution of the erythroderma and
scaling by 6 mohts, follicular atrophoderma will follow Blaschko's lines. CHILD syndrome can also
have similar punctate stippled epiphyses.

Refsum's is associated with a peroxisomal biogenesis disorder. CHILD syndrome is associated with
a mutation in 3b-hydroxysteroid-dehydrogenase. Absent granular layer is present in ichthyosis
vulgaris. Netherton syndrome is associated with absent serine protease inhibitor.

205
20-A patient diagnosed with Naxos syndrome has woolly hair, diffuse palmoplantar keratoderma,
and right ventricular arrhythmogenic cardiomyopathy. The mutation is in:

A. Plakoglobin
B. DNA helicase
C. Ectodysplasin A
D. Connexin 30
E. Connexin 26

A. Plakoglobin

Explanation: Patients with Naxos syndrome have an autosomal recessive mutation in plakoglobin. It
is associated with woolly hair, diffuse palmoplantar keratoderma, and right ventricular
arrhythmogenic cardiomyopathy.
DNA helicase is mutated in Rothmund-Thomson Syndrome, Werner Syndrome, and Bloom
syndrome.
Ectodysplasin A is mutated in Anhidrotic Ectodermal Dysplasia.
Connexin 30 is mutated in Hidrotic Ectodermal Dysplasia (Clouston Syndrome).
Connexin 26 is mutated in KID Syndrome (Keratitis-Ichthyosis-Deafness) and Vohwinkel
Syndrome.

21- An infant presents with numerous congenital hemangomas in a generalized distribution. What is
the most serious associated condition?

A. Congestive Heart Failure


B. Obstructive jaundice
C. Portal hypertension
D. Medulloblastoma
E. Pulmonary fibrosis

A. Congestive Heart Failure

Explanation: High output congestive heart failure can lead to death in children with numerous
congenital hemangomas in a generalized distribution.

206
Obstructive jaundice and portal hypertension both occur, but are less likely to cause death. The
hemangiomas will undergo spontaneous regression. Medulloblastoma and pulmonary fibrosis are
not associated.

22-What mutation is responsible for this skin finding seen on the back of this patient's neck?

A. ABCC6
B. PORCN
C. LMNA
D. FBLN5
E. LEMD

A. ABCC6

Explanation: The image depicts pseudoxanthoma elasticum (PXE), which commonly appears as
"plucked chicken" skin on flexures. This condition is due either an autosomal recessive (most
common) or autosomal dominant mutation in the ABCC6 gene. Other findings include: angioid
streaks with retinal hemorrhage, gastric artery hemorrhage, mitral valve prolapse, hypertension, and
myocardial infarction.
The other genes listed are not associated with PXE:
PORCN- Focal dermal hypoplasia (Goltz syndrome)
LMNA- Familial partial lipodystrophy
FBLN5- Cutis laxa
LEMD3- Buschke-Ollendorf syndrome

23- Which form of EB simplex has a defect in actin assembly, and not in keratin formation?

207
A. Dowling Meara
B. Weber-Cockayne
C. Koebner
D. Kindler
E. Ogna

Correct choice: D. Kindler

Explanation: Kindler syndrome has been reclassified as a subtype of EB simplex due to congenital
blistering being the first symptom. These patients do not demonstrate defects in keratin proteins.
Instead, the KIND1 gene defect leads to disruption of actin assembly.
The other listed forms of EB have a defect in keratin.

24-An infant is brought into your clinic by her mother due to scaly eczematous plaques on the scalp,
peri-oral region, hands, feet, and peri-anal region. Lab evaluation reveals a decreased serum zinc
level. This condition's inherited form has which type of inheritance pattern?

A. Autosomal dominant
B. Autosomal recessive
C. X-linked dominant
D. X-linked recessive
E. Sporadic

B. Autosomal recessive

Explanation: This infant is presenting with acrodermatitis enteropathica. Acrodermatitis


enteropathica (AE) is a rare congenital disorder owing to an abnormality with intestinal absorption
and/or transportation of zinc. Lab work-up in these patients reveals decreased levels of zinc and
alkaline phosphatase (a zinc-dependent enzyme). AE may be inherited or acquired. The inherited
form of AE has an autosomal recessive pattern.
The remaining answer choices do not describe the inheritance pattern of AE.

25- A patient presents to your clinic for evaluation of "eczema." In addition to widespread ill-
defined erythematous finely scaly patches, you notice coarse facies, cold subcutaneous fluctuant

208
nodules, and retained primary teeth. Lab testing reveals an elevated IgE level and peripheral
eosinophilia. Which of the following is most likely mutated in this patient?

A. WASP
B. SPINK5
C. AIRE
D. LYST
E. STAT3

E. STAT3

Explanation: The stem describes many findings characteristic of Hyper-IgE syndrome (Job
syndrome). Two types of inheritance patterns exist: Autosomal dominant mutation in STAT3, and
autosomal recessive mutation in DOCK8. WASP is mutated in Wiskott-Aldrich syndrome. SPINK5
is mutated in Netherton syndrome. AIRE is mutated in autoimmune polyendocrinopathy-
candidiasis-ectodermal dystrophy (APECED). Lastly, LYST is mutated in Chediak-Higashi
syndrome.

26-Nevoid basal cell syndrome is an inherited condition with basal cell carcinomas. Patients with
this syndrome have dental findings of:

A. Odontogenic keratocysts
B. Destructive periodontitis
C. Hypodontia
D. Gingival fibromas
E. Retention of primary teeth

A. Odontogenic keratocysts

Explanation: Nevoid basal cell syndrome (Gorlin syndrome) is an inherited condition with a defect
in the PTCH gene. The dental findings are odontogenic keratocysts.
Destructive periodontitis can occur in Langerhans cell histiocytosis and Papillon-Lefevre
syndrome.
Hypodontia can occur in X-linked hypohidrotic ectodermal dysplasia, Hypomelanosis of Ito,
Incontientia pigmentosa, and Anhidrotic Ectodermal Dysplasia. Gingival fibromas can occur in

209
Tuberous sclerosis. Retention of primary teeth can occur in Hyperimmunoglobulin E syndrome (Job
syndrome).

27- Spastic diplegia is associated with which of the following disorders?

A. Sjogren-Larsson syndrome
B. X-linked ichthyosis
C. Lamellar ichthyosis
D. KID syndrome
E. Refsum syndrome

A. Sjogren-Larsson syndrome

Explanation: Sjogren-Larsson syndrome is an autosomal recessive disorder caused by mutations in


the ALDH3A2 gene that encodes the enzyme fatty aldehyde dehydrogenase. This disorder is
characterized by ichthyosis with pruritus, spastic diplegia or quadriplegia, mental retardation,
epilepsy, glistening dot retinal pigmentation, and dental enamel dysplasia. The other listed
syndromes do not include spastic diplegia.

28-All of the following are true regarding Incontinentia Pigmenti EXCEPT:

A. It is caused by a mutation in the NEMO gene


B. It is an X-linked recessive disorder
C. Inflammation and blistering may be followed by hyperkeratotic, verrucous lesions
D. It is associated with cerebellar ataxia
E. It is associated with coloboma and retinal detachment

B. It is an X-linked recessive disorder

Explanation: Incontinentia Pigmenti is an X-linked dominant disorder (NOT X-linked recessive).


Incontinentia Pigmenti is caused by a mutation in the NEMO gene. Females only present at birth
with linear lesions of inflammation and blistering (stage 1), followed by hyperkeratotic verrucous
areas (stage 2), hyperpigmentation (stage 3), and then atrophy and hypopigmentation (stage 4).

210
Systemic findings include psychomotor retardation, microcephaly, seizures, cerebellar ataxia,
coloboma, and retinal detachment.

29-Which of the following is true regarding ataxia telangiectasia?

A. Condition is associated with a significantly increased risk of ovarian cancer in carriers


B. Telangiectasias present in late adulthood
C. Carriers do not have an increased risk of malignancy
D. Condition is inherited in XLR (X-linked recessive) pattern
E. First clinical sign is neurologic

E. First clinical sign is neurologic

Explanation: First sign is ataxia. Telangiectasias occur later in puberty. Carriers show an increased
risk of breast cancer. Telangiectasias occur later in puberty. Carriers show an increased risk of breast
cancer. The condition is autosomal recessive.

30-patient presents with this lesion shown in the image. Which of the following are true?

A. The patient has lymphangioma circumscriptum


B. The patient likley will develop renal and cardiac failure
C. The patient should be refered for genetic counceling for accumulation of glycosphingolipids
D. An excisional biopsy should be taken
E. They are commonly related to trauma and require no further work up

E. They are commonly related to trauma and require no further work up

211
Explanation: Angiokeratomas are dilated vesels in the dermis. They are related to injury and are
benign. Options B and C refer to Fabry disease, which present with angiokeratoma corporis
diffusum in a bathing suit distribution. There are 5 different types of angiokeratomas: solitary
angiokeratoma as in this case; angiokeratoma of mibelli seen on dorsal fingers and toes in
adolescence, angiokeratoma corporis diffusim which are predominantly seen on lower torso and
thighs and is seen in Fabry's disease; angiokeratoma of fordyce on the scrotum or vulva, and
angiokeratoma circumscriptum which develop on the lower limb and can be assocaited with Cobb
syndrome and Klippel-Trenaunay-Weber Syndrome.

31-Which type of neurofibroma is known to be pathognomonic for neurofibromatosis type I and is


composed of neurofibroma with hypertrophied nerves in a plexiform patter?

A. Plexiform variant
B. Diffuse neurofibroma
C. Wagner Meissner variant
D. Myxoid variant
E. Unencapsulated

A. Plexiform variant

Explanation: The plexiform variant is composed of neurofibroma with hypertrophied nerves in a


plexiform pattern. This is pathognomonic of neurofibromatosis type I.
Diffuse neurofibroma is a variant seen in a proportion of neurofibromatosis type I patients. It
extends into the subcutaneous tissue with honey combing this variant has typical Wagner-Meissner
bodies that resemble Meissner's corpuscles.

32-The x-linked recessive type of dyskeratosis congenita displays a mutation in:

A. Dyskerin
B. TERC
C. CDKN2A
D. PTEN
E. Menin

212
A. Dyskerin

Explanation: The dyskerin gene, whose product is involved in ribosomal RNA synthesis, is mutated
in X-linked recessive dyskeratosis congenita. TERC is linked with autosomal domininant
transmission of the syndrome. CDKN2A is involved in familial dysplastic nevi/melanoma
syndrome, PTEN in Cowden syndrome and Menin in MEN type I.

33-Which of the following statements is true regarding oculocutaneous albinism?

A. The number of melanocytes in the patient's skin is unchanged.


B. All subtypes have a mutation in tyrosinase.
C. OCA1a is rarely associated with Prader-Willi Syndrome.
D. About 25% of cases are sporadic.
E. Patients slowly lose pigmentation over their lifetime.

A. The number of melanocytes in the patient's skin is unchanged.

Explanation: Oculocutaneous albinism is a rare, inheritied disorder in melanin synthesis. Therefore


the number of melanocytes is normal and unchanged in these patients. This is an inheritied
condition, not sporadic. Patients typically present with hypo or depigmentation from birth, not
progressively throughout their lifetime. Subtypes OCA1a (absence of tyrosinase) and OCA1b
(reduced tyrosinase) have mutations in tyrosinase. OCA2 has a mutation in the P gene and 1% of
these cases also present with Prader-Willi or Angelman's syndromes. OCA3 results from a mutation
in Tyrosinase-Related Protein 1 (TYRP1). OCA4 results from a mutation in Membrane-Associated
Transport Protein (MATP).

34-Patients with Werner’s syndrome typically experience which of the following types of cardiac
disease?

A. Hypertrophic cardiomyopathy
B. Aortic aneurysms
C. Premature atherosclerosis
D. Cardiomegaly
E. Mitral valve prolapse

213
C. Premature atherosclerosis

Explanation: Werner syndrome or adult progeria is caused by autosomal recessive mutations in


WRN (Recql2) gene, which encodes DNA helicase. This defect leads to defects in DNA repair and
replication. Patients prematurely age and essentially experience many diseases of aging early in
childhood and teen years. They exhibit tight atrophic skin, relatively large heads for body size, leg
ulcers and cannities. Patients experience early, accelerated atherosclerosis leading to death by
myocardial infarction. In addition, type II diabetes, cataracts, osteoarthritis, osteoporosis and
hypogonadism are features. The alternative answers are not features of Werner syndrome regarding
heart problems with the condition.

35-. A patient with sparse hair, a pear-shaped broad nose and cone-shaped epiphyses likely suffers
which of the following conditions?

A. Focal Dermal Hypoplasia (Goltz syndrome)


B. Tricho-rhino-phalangeal Syndrome
C. Hay-Wells Syndrome
D. Ectrodactyly-Ectodermal dysplasia-Clefting (EEC)
E. Trichodentoosseous Syndrome

B. Tricho-rhino-phalangeal Syndrome

Explanation: Tricho-rhino-phalangeal Syndrome is characterized by sparse hair, a pear-shaped


broad nose and cone-shaped epiphyses.The other listed syndromes do not fit this clinical scenario.

36- Ankyloblepharon filiforme adnatum is seen with a defect in:

A. Plakophilin
B. Plakoglobin
C. Desmoglein
D. C-kit
E. p63

E. p63

214
Explanation: The ectodermal-clefting syndromes are caused by a defect in the p63 gene.
Specifically, AEC or Hay-Wells syndrome is comprised of ankyloblepharon filiforme adnatum
(fusion of the eyelids), ectodermal dysplasia and clefting.
The other listed gene mutations are not commonly associated with ankyloblepharon filiforme
adnatum.

37- . A patient diagnosed with Hay Wells syndrome has wiry, sparse hair and ankyloblepharon. The
patient has partial anhidrosis, cleft lip/palate, and dystrophic nails. The mutation is in:

A. p63
B. TP73L
C. DLX3
D. PORCN
E. Arylsulfatase

A. p63

Explanation: Patients with Hay-wells syndrome have mutations in P63. They have wiry, sparse hair,
ankyloblepharon, PPK, partial anhidrosis, cleft lip/palate, and absent/dystrophic nails.
TP73L mutation occurs in Rapp-Hodgkin Syndrome.
DLX3 mutation occurs in Trichodentoosseous Syndrome.
PORCN mutation occurs in Focal Dermal Hypoplasia (Goltz syndrome).
Arylsulfatase mutation occurs in Chondrodysplasis Punctata and X-linked ichthyosis.

38-Which of the following is caused by mutations in gap junction proteins?

A. Rothmund-Thompson syndrome
B. Hidrotic ectodermal dysplasia
C. Anhidrotic ectodermal dysplasia
D. Netherton syndrome
E. Naxos syndrome

B. Hidrotic ectodermal dysplasia

215
Explanation: Hidrotic ectodermal dysplasia (Clouston syndrome) is caused by mutations in
connexin 30, which is a gap junction protein.
Rothmund-Thompson syndrome is caused by a mutation in the RECQL4 gene, which encodes a
helicase. Anhidrotic (hypohidrotic) ectodermal dysplasia can be caused by mutations in the EDA,
EDAR, and EDARADD genes. Netherton syndrome is caused by mutations in the SPINK5
gene, encoding a serine proteinase inhibitor. Naxos syndrome is caused by a mutation in
plakoglobin, which is component of desmosomes and adherens junctions structures.

39- Mutations in the STK11 gene, which encodes a serine threonine kinase, are seen in:

A. Muir-Torre Syndrome
B. Bannayan-Riley-Ruvalcaba Syndrome
C. Birt-Hogg-Dubbe Syndrome
D. Cronkhite-Canada Syndrome
E. Peutz-Jeghers Syndrome

E. Peutz-Jeghers Syndrome

Explanation: Peutz-Jeghers Syndrome is an autosomal dominant syndrome. 50% of patients have


mutations in the STK11 gene, which encodes a serine threonine kinase. It presents with periorificial
and mucosal lentigines beginning in infancy, as well as hamartomatous intestinal polyps with
intussusception and bleeding; there is an increased risk of gallbladder, pancreatic, breast, ovarian,
and testicular cancers.
Muir-Torre Syndrome results from mutations in MLH1, MSH2, or MSH6 (DNA mismatch repair
genes). Bannayan-Riley-Ruvalcaba Syndrome occurs due to a mutation in PTEN. Birt-Hogg-Dubbe
Syndrome results from a FLCN (folliculin) mutation. Lastly, Cronkhite-Canada Syndrome is rare,
acquired, nonhereditary syndrome; no specific gene mutation has yet been found to cause this
syndrome.

40- What is the most common cutaneous finding in POEMS syndrome?

A. hypertrichosis
B. glomeruloid hemangiomas
C. sclerodermoid changes
D. skin thickening

216
E. hyperpigmentation

E. hyperpigmentation

Explanation: Hyperpigmentation. POEMS syndrome, also known as Crow-Fukase syndrome, is an


acronym for polyneuropathy, organomegaly, endocrinopathy (most common being hypogonadism),
M-protein (IgG and IgA light chains), and skin changes. Of the cutaneous signs, hyperpigmentation
is the most common, occurring in up to 90% of affected patients. The remaining cutaneous findings
are findings in POEMS syndrome however are less frequent in occurrence.

41-patient with curly hair that straightened after puberty, enamel hypoplasia, dental pits, and
increased bone density likely has a mutation in which of the following genes?

A. Distal-less homeobox-3 gene (DLX3)


B. Vascular-endothelial growth factor receptor 3
C. Bone morphogenetic protein type 2
D. SLURP 1
E. SPINK 5

A. Distal-less homeobox-3 gene (DLX3)

Explanation: Distal-less homeobox-3 gene (DLX3) mutations cause trichodentoosseous syndrome,


which is characterized by curly hair that straightens in the 2nd-3rd decades, enamel hypoplasia,
dental pits, and increased bone density. Vascular-endothelial growth factor receptor 3 is mutated in
Milroy disease. SLURP 1 is mutated in Mal de Maleda. SPINK 5 is mutated in Netherton syndrome

42- A 5-year-old male is diagnosed with neurofibromatosis type 1 (NF1). NF1 is due to a mutation
in the gene which encodes neurofibromin. Which of the following best describes the function of
neurofibromin?

A. Turns Ras to its inactive form


B. Activates B-Raf
C. Causes phosphorylation of merlin protein
D. Stimulates K-Ras

217
E. Indirectly activates PI3 kinase

A. Turns Ras to its inactive form

Explanation: Neurofibromin is a GTPase-activating protein that negatively regulates RAS/MAPK


pathway activity by accelerating the hydrolysis of Ras-bound GTP. Thus neurofibromin acts as a
tumor suppressor by reducing Ras activity. In NF1, decreased neurofibromin results in unrestricted
Ras activity in the RAS/MAPK and leads to the tumor growth seen in this disease.The other answer
choices do not accurately describe the function of neurofibromin.

43- A 30 year-old man presents to your office for a full body skin examination. On exam, you note
generalized lentigines, which the patient states have been present since shortly after birth. He has a
history of cardiac arrhythmia and wide set eyes. What other associated abnormalities would you
expect?

A. Infertility
B. Deafness
C. Mental retardation
D. Emphysema
E. Renal abnormalities

B. Deafness

Explanation: This patient likely has LEOPARD syndrome, an autosomal dominant disease caused
by a gene mutation in PTPN11, which encodes Shp2 (tyrosine phosphatase). Other findings in
LEOPARD syndrome include multiple lentigines, EKG abnormalities, ocular hypertelorism,
pulmonic stenosis, abnormal genitalia, retarded growth, and deafness. LEOPARD syndrome is
allelic to Noonan syndrome.The remaining answer choices are not seen in LEOPARD syndrome.

44- The combination of ankyloblepharon, ectodermal dysplasia, and cleft palate with wiry sparse
hair is characteristic of which of the following syndromes?

A. Focal Dermal Hypoplasia (Goltz syndrome)


B. Tricho-rhino-phalangeal Syndrome

218
C. Hay-Wells Syndrome
D. Ectrodactyly-Ectodermal dysplasia-Clefting (EEC)
E. Trichodentoosseous Syndrome

C. Hay-Wells Syndrome

Explanation: Hay-Wells Syndrome, also known as AEC (Ankyloblepharon-Ectodermal dysplasia-


Clefting) is characterized by wiry, sparse hair or alopecia, ankyloblepharon, PPK, partial anhidrosis,
cleft lip, palate, absent, and dystrophic nails.The other listed syndromes do not fit this clinical
presentation.

45- Which subtype of this disease is most common in the United States?

A. A
B. E
C. C
D. B
E. D

Correct choice: C. C

Explanation: This is xeroderma pigmentosum. XPC is the most common subtype in the US. In this
subtype there are no neurologic symptoms. In XPA, which is the most common subtype in Japan,
there are often severe neurologic symptoms.
Xeroderma pigmentosum (XP) is a very rare skin disorder where a person is highly sensitive to
sunlight, has premature skin aging and is prone to developing skin cancers.
Xeroderma pigmentosum is caused by cellular hypersensitivity to ultraviolet (UV) radiation, as a
result of a defect in the DNA repair system. Xeroderma pigmentosum has also been called
DeSanctis-Cacchione syndrome.

219
At least seven different gene abnormalities or complementation groups have been described in
different families (XPA to XPG) resulting in varying disease severity.
XPA and XPC are relatively common
XPE is fairly rare
XPG is severe
XPF is mild.

46- Which of the following is not a major diagnostic criteria for nevoid basal cell carcinoma
syndrome?

A. Calcification of the falx


B. Palmar or plantar pits (3 or more)
C. Development of >2 BCCs or 1 BCC before the age of 20
D. Odontogenic keratocysts of jaw
E. All of the above are features of nevoid BCC syndrome

Correct choice: E. All of the above are features of nevoid BCC syndrome

Explanation: The 5 major diagnostic criteria for nevoid BCC syndrome include: 1. Development of
>2 BCCs or 1 BCC before the age of 20 2. Odontogenic keratocysts of jaw 3. Palmar or plantar pits
(3 or more) 4. Calcification of the falx cerebri 5. 1st degree relative with nevoid BCC syndrome .All
the above features are major diagnostic criteria for nevoid BCC syndrome.

47- This is due to a mutation in what gene?

220
A. Transglutaminase 1
B. ABCA12
C. Keratin 1 and 10
D. GJB3
E. Keratin 2e

Correct choice: A. Transglutaminase 1

Explanation: This is lamellar ichthyosis which is in flexures and has thick scales on the palms and
soles. It is most commonly due to a mutation in transglutaminase 1. BCIE is due to K1 and K10
mutation. Ichthyosis bullosa of Siemens is due to a keratin 2e mutation. Harlequin fetus is due to an
ABCA12 mutation. Erythrokeratoderma variabilis is due to a GJB3 and GJB4 mutation.

Autosomal recessive congenital ichthyosis


Harlequin ichthyosis ABCA12
Lamellar ichthyosis TGM1 (transglutaminase 1 is an enzyme) and others
Congenital ichthyosiform erythroderma ALOXE3 and others

Autosomal recessive congenital ichthyosis-1(ARCI1):The baby is encased by collodion


membrane at birth, which cracks and is then shed; 10% are self-healing
Harlequin ichthyosis: severe collodion membrane, ectropion (drooping lower
eyelids), eclabium (out-turned lips) and contractures
Lamellar ichthyosis: plate-like dark scaling over the whole body, including creases of arms and legs
Bathing suit variant: localised to scalp and trunk (warmer sites of the body)
Nonbullous congenital ichthyosiform erythroderma (NCIE): erythroderma and fine white,
superficial, semiadherent scales
Palmoplantar keratoderma varies with affected gene

Biopsy reveals:
Thick, hyperkeratotic stratum corneum, Reduced or absent granular cell layer and filaggrin in
ichthyosis vulgaris, Lamellar body ‘ghosts’ in lamellar ichthyosis, Vacuolar degeneration of
suprabasal keratinocytes and coarse keratohyalin granules in a thickened granular layer in
keratinopathic ichthyosis.

221
48- What is the most common association with this dermatologic finding?

A. Lymphoma
B. Melanoma
C. Thyroid cancer
D. Lung cancer
E. Colon cancer

Correct choice: E. Colon cancer

Explanation: This pathologic image identifies a sebaceous adenoma (well circumscribed lobules of
mature sebocytes). Sebaceous adenomas are frequently seen in Muir-Torre syndrome. Patients with
Muir-Torre syndrome can present with gastrointestinal, genitourinary, breast cancers, and
keratoacanthomas. Melanoma, thyroid cancer, lymphoma, and lung cancers are not the most
common malignances seen in Muir Torre syndrome.

49- Which of the following findings is most likely to be associated with below

A. Trichodiscomas
B. Trichoepitheliomas
C. Hutchinson’s teeth
D. Pilomatricomas
E. Natal teeth

222
Correct choice: E. Natal teeth

Explanation: This photo has a histologic image of steatocystoma – It is lined by a characteristic


eosinophilic crenulated cuticle (shark tooth lining), sebum typically falls out during processing, and
sebaceous glands attach to cyst wall on the outside. Steatocystoma multiplex can occur in Type II
Pachyonychia congenita (Jackson-Lawler syndrome), which involves mutations in KRT17. It will
have findings of natal teeth and less severe PPK than type I pachyonychia congenita.
Trichodiscomas occur in Birt-Hogg-Dube syndrome which has fibrofolliculomas, trichodiscomas,
achrochordons, lipomas, oral fibromas, renal cell carcinoma, medullary thyroid carcinoma, and
colon cancer. Pilomatricomas occur in Gardner syndrome and Rubinstein-Taybi syndrome.
Trichoepitheliomas occur in Rasmussen syndrome, Rombo syndrome, and Brooke-Spiegler
syndrome. Hutchinson’s teeth are widely spaced, peg-shaped upper incisors that occur in congenital
syphilis.

50- A 35 y/o female is found to have mismatch repair protein mutations and this finding. Which of
the following is the next best step?

A. Positron emission tomography


B. Urinary cytology
C. CT Head
D. Check BUN/Cr
E. Check TSH/T4

Correct choice: B. Urinary cytology

Explanation: The question is describing a patient with Muir-Torre Syndrome (photo is depicting a
keratoacanthoma). These patients have autosomal dominant mutations in MSH2, MLH1, MSH6
(DNA mismatch repair genes). Dermatologically, they are at risk for sebaceous adenomas,
epitheliomas, and carcinomas;keratoacanthomas (esp. w/sebaceous differentiation). These patients
are also at high risk for internal malignancies, including GI, genitourinary, and breast cancers.

223
These patients need workup with upper and lower endoscopy, genitourinary surveillance, chest
xray, CBC, and liver function tests. Kidney abnormalities are not a common feature of Muir Torre.
Thyroid abnormalities are not a common feature of Muir Torre. PET Scan would be appropriate if
initial screening shows suspicion for visceral malignancy. Brain abnormalities are not a typical
feature of Muir Torre.

51- The target of the autoantibodies causing this condition are mutated in which of the following?

A. Birt-Hogg-Dube syndrome
B. Cowden syndrome
C. Lipoid proteinosis
D. Familial cylindromatosis
E. Piebaldism

Correct choice: C. Lipoid proteinosis

Explanation: This photo shows lichen sclerosis, which in 80% of cases has been associated with
IgG1 autoantibodies against ECM-1. It presents with white, polygonal, and flat-topped papules or
plaques surrounded by erythema. ECM-1 has also been found to be mutated in lipoid proteinosis.
Piebaldism has been associated with mutations in the c-kit proto-oncogene.
Cowden syndrome is associated with a PTEN mutation.
Birt-Hogg-Dube syndrome is associated with a FLCN (folliculin) mutation.
Familial cylindromatosis is associated with a CYLD mutation.

52- This lesion occurs in which of the following syndromes?

224
A. Muir-Torre syndrome
B. Lipoid proteinosis
C. Cowden syndrome
D. Schöpf-Schulz-Passarge Syndrome
E. Birt-Hogg-Dube syndrome

Correct choice: D. Schöpf-Schulz-Passarge Syndrome

Explanation: This kodachrome depicts a hidrocystoma, which can occur in Schopf-Schulz-Passarge


syndrome. It presents with a cyst lining composed of a double layer of cuboidal epithelial cells. It
can occur with apocrine type decapitation secretion.
Schopf-Schulz-Passarge syndrome is an autosomal recessive mutation of the WNT gene. It occurs
with PPK, hidrocystoma, BCC, SCC, hypodontia, and eccrine tumors (eccrine
syringofibradenoma). Cowden syndrome is an autosomal dominant mutation of the PTEN gene,
which presents with trichilemmonas, oral papillomas, and sclerotic fibromas.
Muir-Torre syndrome is an autosomal dominant mutation in the DNA mismatch repair genes,
presenting with sebaceous adenomas, carcinomas, and keratoacanthomas.
Birt-Hogg-Dube syndrome is an autosomal dominant syndrome presenting with fibrofolliculomas,
trichodiscomas, lipomas, oral fibromas, and achrocordons. Lipoid proteinosis is an autosomal
recessive mutation of the ECM1 gene, which presents with hyaline papules along the eyelid
margins “string of pearls”.

53- A patient presents with the following condition. The gene associated with this disorder has also
been implicated in which of the following?

225
A. Cowden syndrome
B. Piebaldism
C. Birt-Hogg-Dube syndrome
D. Lipoid proteinosis
E. Familial cylindromatosis

Correct choice: B. Piebaldism

Explanation: This photo depicts urticaria pigmentosa, which occurs due to a mutation in c-kit
(CD117). This mutation has also been found in piebaldism.Cowden syndrome is associated with a
PTEN mutation. Birt-Hogg-Dube syndrome is associated with a FLCN (folliculin) mutation.
Familial cylindromatosis is associated with a CYLD mutation. Lipoid proteinosis is associated with
an ECM-1 gene mutation.

54-Patients with Darier s disease are at increased risk for:

A. Kaposiǁs varicelliform eruption


B. Melanoma
C. Decreased life span
D. Basal cell carcinoma
E. Lipid abnormalities

►A

Kaposiǁs varicelliform eruption is the condition in which viral infection occurs in a patient with a
pre-existing chronic dermatitis. Darierǁs disease is an autosomal dominant genodermatosis caused
by a mutation in ATP2A2 which encodes SERCA2. Cutaneous manifestations of warty,
hyperkeratotic papules in a seborrheic dermatitis, which may be infected with HSV or bacteria.

55- A patient with coarse facies, broad nasal bridge, and extensive eczema might be expected to
have which abnormal laboratory value?

A. Hypertriglyceridemia
B. High Copper levels
C. Low Zinc levels

226
D. High IgE levels
E. Anemia

►D

Patients with Hyper IgE Syndrome (Job syndrome) have impaired regulation of the IgE function
and increased susceptibility to infections. In addition to recurrent cutaneous infections including
cold abscesses, patients have widespread eczematous dermatitis, recurrent sinopulmonary infections
and typically have coarse facies with broad nasal bridge and a prominent nose.

56- Refsum syndrome is due to a deficiency in phytanyl coenzyme A hydroxylase. Treatment for
this condition is:

A. Diet low in green vegetables, dairy and ruminant fats


B. Diet high in green vegetables, dairy and ruminant fats
C. Enzyme replacement
D. No treatment is available at this time
E. Avoid phenylalanine

►A

Treatment is with a diet low in green vegetables, dairy and ruminant fats is the treatment of choice
for Refsum syndrome. Avoidance of specific amino acids is not helpful.

57 -Which of the following eye findings is caused by the rupture of Bruch's membrane?

A. Angioid streaks
B. Blue sclerae
C. Retinal detachment
D. Ruptured globe
E. Keratoconus

►A

227
The rupture of Bruch's membrane causes angioid streaks in pseudoxanthoma elasticum. Bruch's
membrane is the innermost layer of choroid with a central layer of elastic fibers. The other findings
are found in Ehlers-Danlos syndrome and are not related to Bruch's membrane.

58- Which genetic defect could explain cutaneous findings in addition to abnormal immunoglobulin
levels, recurrent respiratory infections, hypogonadism, and an increased risk of leukemia and
lymphoma?

A. RecQL3
B. ERCC6
C. WAS gene
D. NADPH oxidase
E. Adenosine deaminase

►A

Bloom's syndrome is an autosomal recessive disorder caused by mutations in the RecQL3 gene
encoding a DNA helicase. Clinically, individuals with Bloom's syndrome have a
photodistributed erythema with telangectasia on the malar eminences. The may also have
decreased IgM and IgA levels, hypogonadism, and an increased risk for leukemia and
lymphoma.

59-Which of the following diseases is seen only in females and mosaic males?

A. Griscelli syndrome
B. Piebaldism
C. Hypomelanosis of Ito
D. Incontinentia pigmenti
E. Carney complex

►D

Incontinentia pigmenti (Bloch-Sulzberger syndrome) is an X-linked dominant disease that is


lethal in males unless they are a mosaic. There are four stages: vesicular, verrucous,

228
hyperpigmented and hypopigmented. Also seen are peg and conical teeth, eye abnormalities,
CNS defects, and alopecia. This condition is caused by mutations in the NEMO gene.

60-What is the most likely gene mutation in this individual who has migratory patches and fixed
plaques as depicted in this picture?

A. Connexin 26
B. Connexin 30.3 and 31
C. Calcium ATPase 2C1
D. Loricrin
E. Keratins 1 and 10

►B

Erythrokeratodermis variabilis is an autosomal dominant disease characterized by transient


patches of geographic erythema and fixed hyperkeratotic plaques. The disease is due to gene
defects in connexin 30.3 and 31.

61-A 27 year old presents with some mildly itchy spots on his posterior neck. Biopsy reveals
elastosis perforans serpiginosa. Other notable exam findings include tall stature, joint laxity,
pectus excavatum, and a high-arched palate. Which gene is most likely mutated in this patient?

A. Fibrillin 1
B. Fibronectin
C. Collagen 5
D. ABCC6
E. Fibulin 4

229
►A

Marfan syndrome is an autosomal dominant disordercharacterized by tall stature, arachnodactyly,


pectus excavatum, high-arched palate, joint laxity, ectopia lentis, aortic dilatation, striae, and
elastosis perforans serpiginosa. The syndrome is caused by a defect in fibrillin 1 or 2. Fibronectin is
defected in Type 10 Ehlers-Danlos syndrome. Collagen 5 defects can cause Ehlers-Danlos types 1
or 2. ABCC6 is mutated in pseudoxanthoma elasticum. Cutis Laxa is caused by defects in fibulin 4

62-A patient has lusterless hair, diffuse hypopigmentation, doughy skin with severe psychomotor
retardation has a defect in the:

A. ATP7A gene
B. GJB2 gene
C. Connexin 26
D. c-kit gene
E. PAX1

►A

This patient has Menkes Disease that is an X-linked recessive disorder caused by a mutation of
the ATP7A gene. This gene encodes copper-transporting ATPase. The disease is lethal in males.
Girls develop variably depigmented lusterless hair. Polaroscopy reveals monilethrix, pili torti,
trichorrhexis nodosa. In addition, there is diffuse cutaneous hypopigmentation, doughy skin and
an exaggerated cupid bow.

63-Findings of eyelid papules (string of pearls) and a hoarse cry in infants is characteristic of which
of the following syndromes?

A. Lipoid proteinosis
B. Amyloidosis
C. Pseudoxanthoma elasticum
D. Disseminated xanthomas
E. None of these answers are correct

230
►A

Findings of the eyelid string of pearls and a hoarse cry during the first years of life (due to vocal
cord infiltration) is characteristic of Lipoid Proteinosis (Urbach-Wiethe disease or Hyalinosis
cutis et mucosae). It is an autosomal recessive condition with mutations in the extracellular
matrix protein 1 gene. Other findings include calcifications of the temporal lobe and hippocampus,
hairloss, atrophic scars and waxy papules on the face, verrucous nodules and a thick tongue. The
other conditions could be considered on the differential for Lipoid Proteinosis, but do not have the
findings described above.

64 -What phenotype results from a low activity of double stranded RNA adenosine deaminase?

A. Waardenberg's syndrome type 2


B. Piebaldism
C. Tietz syndrome
D. dyschromatosis symmetrica hereditaria
E. oculocutaneous albinism type 4

►D

Dyschromatosis symmetrica hereditaria (or acropigmentation symmetrica of Dohi) is an


autosomal dominant disease with hypo and hyper pigmented macules and patches on the dorsal
hands and feet associated with a low activity of double stranded RNA adenosine deaminase.

65-What is the mode of transmission for lamellar ichthyosis?

A. Autosomal dominant
B. Autosomal recessive
C. X-linked dominant
D. X-linked recessive
E. Sporadic

►B

231
Lamellar ichthyosis which is characterized by collodian membrane in newborns and platelike
scale in children and adults is an autosomal recessive syndrome. The gene defect is
transglutaminase 1 (TGM1).

66 -A triangular-shaped lunula is a characteristic finding in which disease?

A. Darier's disease
B. Dyskeratosis congenita
C. Epidermal nevus syndrome
D. Incontinentia pigmenti
E. Nail-patella syndrome

►E

Nail-patella syndrome, also called hereditary osteo-onychodysplasia is a rare autosomal dominant


condition caused by a defect in the LMX1B gene. It is characterized by triangular lunulae,
palmoplantar hyperhidrosis, renal dysplasia, glomerulonephritis, and hyperpigmentation of the
papillary margin of the iris, an ophthalmologic finding also known as Lester iris. Other nail findings
include micronychia with hemionychia, anonychia, and longitudinal fissures. Bony findings include
absent or hypoplastic patella, posterior iliac horns, radial head subluxation, thickened scapulae, and
scoliosis. Nail findings in Darier's disease include red and white longitudinal bands, subungual
hyperkeratosis and V-shaped nicking of the distal nail plate. Dystrophic nails with longitudinal
ridges, pterygium, and atrophic or absent nails can be found in dyskeratosis congenita. Dystrophic
changes of the nails can be seen in approximately 5-10% of patients with incontinentia pigmenti.

67-Adenosine deaminase deficiency is associated with which of the following disorders?

A. Wiskott Aldrich syndrome


B. Gout
C. Job syndrome
D. Xeroderma pigmentosum
E. Severe combined immunodeficiency

►E

232
Adenosine deaminase deficiency is associated with severe combined immunodeficiency. The most
common inheritance is x-linked recessive. It is a mixed group of disorders all sharing defects in
cell-mediated and humoral immunity. Skin findings include: candidal infections, Mucocutaneous,
bacterial pyodermas, seborrheic-like dermatitis/lichen planus-like Sclerodermatous changes,
aplastic thymus and pneumonias. The other listed syndromes are not associated with adenosine
deaminase deficiency.

68-A teenage female presents with the complaint of "nail fungus". On exam, she has triangular
lunulae, palmoplantar hyperhidrosis, micronychia and an absent patella. Which of the following
gene defects is most likely in this patient?

A. LMX1B
B. COL5A1
C. EEC1
D. LKB1
E. NEMO

►A

This patient has nail-patella syndrome. The gene defect is an autosomally transmitted defect in
LMX1B. This gene is involved in dorsal/ventral limb patterning and is in close proximit to the
COL5A1 gene. EEC1 is defective in ectrodactyly-ectodermal dysplasia-cleft lip/palate syndrome
and LKB1 in Peutz Jeghers syndrome.

69- Pseudoxanthoma elasticum is caused by mutations in which of the following genes?

A. Fibrillin 1
B. Elastin gene
C. Lysyl oxidase
D. ABCC6 gene
E. Collagen 5

►D

233
Pseudoxanthoma elasticum is caused by mutations in the ABCC6 gene, which is an ATP -using
cell transporter. Elastin and lysyl oxidase mutations cause cutis laxa, fibrillin 1 mutations cause
Marfan syndrome, and collagen 5 mutations cause Ehlers-Danlos syndrome.

70 -Which of the following is NOT a characteristic skin finding in patients with Down Syndrome?

A. Syringomas
B. Elastosis perforans serpiginosa
C. Single palmar crease
D. Flat nipples
E. Small tongue

►E

Down syndrome is caused by nondisjunction and results in trisomy 21. Clinical features include
single palmar crease, flat nipples, increased nuchal skin folds, syringomas, elastosis perforans
serpiginosa, xerosis, epicanthic folds of eyes, protruding scrotal tongue and fissured thickened
lips.

71- Which eye findings would be expected in an individual with this disorder associated with
atherosclerosis?

A. Dendritic corneal ulcerations


B. Salt and pepper retinitis pigmentosa
C. Ectopia lentis
D. Angioid streaks
E. Keratoconus

►D

Pseudoxanthoma elasticum is an autosomal rescessive or autosomal domminant disease caused


by a mutation in ABCC6 (adenosine triphosphate-binding cassette subfamily C member 6).
Associated findings include gastric artery hemorrhage, angiod streaks, retinal hemorrhage,
atherosclerotic disease, and a possible increased risk of first trimester miscarriage.

234
72 -Defects in Fibrillin 2 are linked with:

A. Congenital contractural arachnodactyly


B. Cutis Laxa
C. Arthrochalasis multiplex congenita
D. Occipital horn syndrome
E. Lipoid proteinosis

►A

Fibrillin 2 defects arelinked primarily with congenital contractural arachnodactyly. This syndrome
is associated with long limbs, arachnodactyly, scoliosis and crumpled ears. Occasionally, fibrillin 2
can be associated with Marfan syndrome also. The other conditions are not linked to fibrillin
mutations.

73-A 32 year-old woman is 5 weeks pregnant and is diagnosed with hyperthyroidism. Her doctor
gives her a prescription for on methimazole 10 mg PO tid. Which of the following fetal
abnormalities could be caused by this exposure?

A. Aplasia cutis congenita


B. Meningocele
C. Encephalocele
D. Spina bifida
E. Dermoid cyst

►A

In-utero methimazole exposures has been linked to aplasia cutis congenita and should not be
used in pregnant women. The FDA pregnancy class is D. The other listed options are not linked
with maternal methimazole usage.

74 -Spontaneous mutations causing tuberous sclerosis are more likely to occur in which gene?

A. TSC2

235
B. TSC1
C. TSC 3
D. TSC 4
E. TSC 5

►A

Spontaneous mutations are four times more likely to occur in TSC2. In familial TS half
mutations are in TSC1 and half in TSC2.

75 -This syndrome is characterized by lentigines, ECG conduction defects, ocular hypertelorism,


pulmonic stenosis, abnormal genitalia, retardation of growth and deafness:

A. Moynahan syndrome
B. Nail patella syndrome
C. Bloch-sulzberger syndrome
D. Incontinentia pigmenti achromians
E. Gardner syndrome

►A

Moyahan syndrome is also known as Leopard syndrome and can have all the findings of
lentigines, ECG conduction defects, ocular hypertelorism and pulmonic stenosis. They also get
abnormal genitalia, retardation of growth and deafness.

76 -Patients that are diagnosed with Peutz-Jeghers syndrome must have a colonoscopy for
hamartomatous polyps transforming to carcinoma every:
A. 10 years
B. 8 years
C. 5 years
D. 2 years
E. 1 year

236
►D

Patients that are diagnosed with Peutz-Jeghers syndrome need to have a colonscopy every 2
years to assure that the hamartomatous polyps do not progress to carcinomas. It is an autosomal
dominant disorder with hyperpigmented macules in the oral mucosa.

77 -A patient has Conradi-Hunermann-Happle syndrome with congenital ichthyosiform


Erythroderma, ventricular septal defect, and asymmetric limb shortening. The bone finding for the
disease is:

A. Chondrodysplasia punctata
B. Bowing of the knees
C. Invisible bone syndrome
D. Thickened epiphyses
E. Distal calcification

►A

Patients with Conradi-Hunermann-Happle syndrome has chondrodysplasia punctata that is X-


linked dominant. They have ventricular septal defects and asymmetric limb shortening.

78 -Premature aging seen in Cockayneǁs syndrome is caused by a mutation in which gene?

A. Transglutaminase
B. ATM gene
C. Patched gene
D. NEMO gene
E. DNA helicase gene

►E

Cockayneǁs syndrome is caused by a mutation in a DNA helicase gene, CSA or ERCC8. The
condition is defined by growth deficiency, premature aging, and pigmentary retinal degeneration.
75% of patient have photosensitive eruptions and severe cataracts before the age of 3 are

237
associated with poor prognosis.

79 -Keratins 1 and 10 are important intermediate filaments in suprabasal keratinocytes. Genetic


mutation of these keratins can lead to which of the following disorders?

A. Netherton syndrome
B. Sjogren-Larsson syndrome
C. Pachyonychia congenita type 1
D. Epidermolytic hyperkeratosis / Bullous ichthyosiform erythroderma
E. White sponge nevus

►D

Epidermolytic hyperkeratosis is caused by a genetic mutation in keratins 1 and 10. White sponge
nevus is caused by a genetic mutation in keratins 4 and 13. Pachyonychia congenita is caused by
a genetic mutation in keratins 6a and 16. Netherton syndrome and Sjogren-Larsson syndrome are
not disorders of keratins.

80 -What is the inheritance pattern of a dermatosis with acantholytic dyskeratosis?

A. Autosomal dominant
B. Autosomal recessive
C. X-linked dominant
D. X-linked recessive
E. Sporadic

►A

Hailey-Hailey, or Familial Benign Pemphigus, is an autosomal dominant genodermatosis which


is caused by a mutation in ATP2C1. Vesicles and erythematous plaques develop in the skin folds
such as axillae and groin area. Darier's disease is an autosomal dominant genodermatosis due to
a mutation in ATP2A2 and is often in a seborrheic distribution.

238
81 -Which PPK is associated with a right-sided cardiomyopathy?

A. Haim-Munk
B. Papillon-Lefevre
C. Naxos
D. Olmstead
E. Huriez

►C

Naxos is associated with right sided cardiomyopathy. It is an AR, non epidermolytic PPK in
which patients have congenital wooly hair and fragile desmosomes. These patients are at risk for
heart failure and death.

82 -Yellow-brown depositions in Descemet's membrane of the corneas is diagnostic for:

A. Hemochromatosis
B. Neurofibromatosis
C. Hepatolenticular degeneration
D. Pseudoxanthoma elasticum
E. Diabetes mellitus

►C

Hepatolenticular degeneration or Wilson Disease is caused by a defect in biliary excretion of


copper leading to accumulation in the liver, brain, cornea, pretibial hyperpigmentation,
hepatomegaly and cirrhosis. The Kayser-Fleischer ring is the yellow-brown copper deposition in
Descemet's membrane of the cornea. Other findings include ataxia, dysarthria and dementia.
Hemochromatosis does not have an eye finding. Neurofibromatosis has Lisch nodules and
Pseudoxanthoma elasticum has angioid streaks of the retina.

83 -What cutaneous manifestation is associated with familial cerebral cavernomas?

A. Verrucous hemangioms

239
B. Glomeruloid hemangiomas
C. hyperkeratotic cutaneous capillary-venous malformations(HCCVM)
D. segmental facial hemangiomas
E. Tufted angiomas

►C

Familial cerebral cavernomas are due to a defect in the CCM gene which encodes the KRIT-1
protein. These patients often times have hyperkeratotic cutaneous capillary-venous malformations.

84- Anodontia is a bone finding seen in which of the following conditions:

A. Hypomelanosis of Ito
B. Letterer-Siwe disease
C. Tuberous sclerosis
D. Jackson Sertoli syndrome
E. Hyper-IgE syndrome

►A

Hypomelanosis of Ito, or Incontinentia pigmenti achromians is a condition characterized by


marble-cake hypopigmentation, epilepsy, alopecia, scoliosis and mental/motor retardation. The
characteristic dental abnormality is anodontia. The remaining syndromes are not associated with
anodontia.

85 -A 4-year old boy presents with generalized white scale. The mother reports that her son was
born with a tight membrane enveloping his body. Peripheral blood smear is within normal limits.
What is the most likely diagnosis?

A. Congenital ichthyosiform erythroderma


B. Neutral lipid storage disease
C. Lamellar ichthyosis
D. Netherton syndrome
E. Ichthyosis vulgaris

240
►A

The most likely diagnosis is Congenital ichthyosiform erythroderma. In neutral lipid storage
disease, the peripheral blood smear would demonstrate lipid vacuoles in leukocytes and
monocytes. Lamellar ichthyosis is characterized by plate-like scale in children/adults. Netherton
syndrome is characterized by ichthyosis linearis circumflexa. Ichthyosis vulgaris does not
typically present with collodian baby.

86 -Cutaneous osteomas are seen in which syndrome?

A. Waardenburg syndrome
B. LEOPARD syndrome
C. Carney complex
D. Albright hereditary osteodystrophy
E. Gaucherǁs syndrome

►D

Albright hereditary osteodystrophy is caused by mutations in the Gs subunit of adenylate


cyclase. There is calcification and ossification due to pseudohypoparathyroidism, absent 4th
knuckle, and hypogonadism.

87-A patient is diagnosed with Leopard syndrome with cafe au lait macules, melanoma, local
hypopigmentation, triangular face, frontal bossing and low ears. The cardiac findings are:

A. AV block
B. Mitral valve prolapse
C. Tricuspid valve prolapse
D. Atrial fibrillation
E. Ventricular fibrillation

►A

Patients that have Leopard syndrome have AV blocks, arrhythmias, bundle branch block,

241
ventricular hypertrophy. The patient has ocular hypertelorism, pulmonic stenosis, abnormal
genitals, and deafness.

88 -A patient has cutaneous amyloidosis, pancreatic tumors, parathyroid tumors,


pheochromocytoma and medullary carcinoma. The patient also had notalgia paresthetica as a child.
The child most likely has:

A. MEN Type IIA


B. MEN Type I
C. MEN Type IIB
D. MEN Type IIC
E. MEN Type III

►A

This patient has MEN type IIA also known as "Sipple's Syndrome". It is an autosomal dominant
disorder caused by the RET gene. Work up should include calcitonin, calcium, PTH and urine
catecholamines.

89 -A patient with renal cell carcinoma caused by mutations in fumarate hydratase deficiency likely
suffers which of the following conditions?

A. Von-Hippel-Lindau syndrome
B. Cowden syndrome
C. Birt-Hogg-Dube syndrome
D. Familial multiple cutaneous leiomyomatosis
E. Multiple endocrine neoplasia

►D

Familial multiple cutaneous leiomyomatosis is an autosomal dominant condition caused by


mutations in the fumarate hydratase gene. Clinically, there are multiple cutaneous leiomyomas,
uterine leiomyomas and leiomyosarcomas, as well as renal cell carcinomas.

242
90 -A 45 year old woman with history of uterine fibroids presents with painful firm papules of the
left upper arm. Biopsy of one of these lesions reveals smooth muscle fascicles. She notes a
history of renal cell cancer in her family. The gene implicated in this syndrome has which of the
following functions?

A. Telomere maintenance
B. Involved in the Kreb's cycle
C. Inhibits mTOR signalling
D. Impairs folliculin function
E. Encodes a deubiquitinating enzyme

►B

The patient scenario describes Reed's syndrome (multiple cutaneous and uterine leiomyomatosis)
characterized by uterine and cutaneous leiomyomas, in addition to papillary renal cell cancer.
This autosomal dominant disorder is due to mutations within the fumarate hydratase gene, which
catalyses the conversion of fumarate to malate in the Kreb\'s cycle. It is also thought to act as a
tumor suppressor gene. With regards to the other answer choices, telomere maintenance is disrupted
in dyskeratosis congenita. The tuberous sclerosis genes hamartin (TSC1) and tuberin (TSC2) inhibit
mTOR signalling. Mutations in the FLCN gene, with encodes folliculin (thought to act as a tumor
suppressor), cause Birt Hogg Dube syndrome. The CYLD gene, a deubiquitinating enzyme, has
been implicated in Brooke-Spiegler syndrome.

91 -Epidermolysis bullosa with muscular dystrophy is caused by mutations in which of the


following?

A. Keratins 5 and 14
B. Plectin
C. Loricrin
D. Collagen 7
E. Collagen 17

►B

Epidermolysis bullosa with muscular dystrophy is caused by mutations in plectin.

243
92 -Retinal hemangioblastomas are found in which syndrome:

A. Osler-Weber-Rendu disease
B. Von-Hippel Lindau disease
C. Kasabach-Merritt syndrome
D. Klippel-Trenaunay Weber syndrome
E. Sturge-Weber syndrome

►B

Von Hippel-Lindau syndrome is an autosomal dominant condition caused by a defect in the VHL
tumor suppressor gene. This disease is characterized by retinal hemangioblastomas, often
resulting in visual impairment and blindness if left untreated. In addition, many tumors are seen
including pheochromocytoma, renal cell carcinoma, and hemangioblastomas of the cerebellum,
medulla, and spinal cord. Pancreatic and renal cysts are also a feature of this condition. Finally,
polycythemia can occur as a result of erythropoietin production by renal cell carcinoma. Von
Hippel-Lindau syndrome is a progressive, universally fatal condition which presents most often
in the fourth decade of life.

93 -Patients with hemochromatosis are at increased risk for which of the following?

A. Vibrio vulnificus infections


B. Yersenia infections
C. Polyarthritis
D. Generalized metallic-grey hyperpigmentation
E. All of these options are correct

►E

Patients with hemochromatosis have increased intestinal iron absorption leading to systemic iron
overload. Signs include a generalized metallic-grey hyperpigmentation, koilonychia, alopecia
(especially pubic/axillary hair) cardiac failure/arrhythmias/heart block, hepatomegaly with
cirrhosis, diabetes (bronze diabetes), polyarthritis with chondrocalcinosis and are susceptible to
Vibrio vulnificus and Yersinia infections.

244
94 -What is the gene mutation that most commonly causes herlitz junctional epidermolysis bullosa?

A. LAMB3
B. COL7A1
C. NEMO
D. ATP2A2
E. ATP 2C1

►A

LAMB3 is the most common mutation, it is usually a nonsense mutation causing a premature
stop codon resulting in the absence of laminin 332. Non herlitz type is due to a missense or
splice site mutation resulting in rudimentary/decreased laminin 332. COL7A1 is mutated in
dystrophic epidermolysis bullosa. NEMO gene is mutated in incontinnentia pigmenti. ATP2A2
gene is mutated in Dariers. ATP2C1 is mutated in Hailey Hailey.

95 -Which of the following syndromes demonstrate atrophoderma vermiculatum?

A. Tuzun Syndrome
B. ROMBO Syndrome
C. Nicolau-Balus Syndrome
D. Braun-Falco-Marghescu Syndrome
E. All of these answers are correct

►E

All of the listed syndromes include atrophoderma vermiculatum as part of their constellation of
symptoms. Atrophoderma vermiculatum is characterized by honeycomb pattern of atrophic scars
on the face. Tuzun Syndrome also has scrotal tongue. ROMBO has BCCs, milia, peripheral
vasodilation, trichoepitheliomas. Nicolau-Balus has eruptive syringomas and milia. Braun-Falco-
Marghescu has keratosis pilaris and palmoplantar hyperkeratosis.

96 -A patient presents with focal symmetric palmoplantar keratoderma, thickened, hyperkeratotic


fingernails and toenails with a "pincer" appearance and frequent staph and candida paronychial

245
infections, follicular hyperkeratosis of the knees and elbows and oral leukokeratosis. The patients
mother and grandfather both have similar skin findings. What is the defect in PC type I?

A. Keratin 6a/16
B. Keratin 6b/17
C. Keratin 1/10
D. Keratin 2e/10
E. None of these options are correct

►A

Keratin 6a/16 are defective in PC type I. K6b17 is defective in PC type II, K1/10 in Unna-Thost
PPK and epidermolytic hyperkeratosis, K2e/10 in Ichthyosis bullosa of Siemens.

97 -A 2 year old female patient has linear atrophy in a Blasckoid distribution, alopecia, nail
dystrophy, abnormal teeth, and colobomas. What is the inheritance pattern of this condition?

A. X-linked dominant
B. X-linked recessive
C. Autosomal dominant
D. Autosomal recessive
E. Mitochondrial

►A

Focal dermal hypoplasia, or Goltz syndrome, is an x-linked dominant condition with a defect in

246
the PORCN gene. It is characterized by linear atrophy in a Blasckoid distribution, alopecia, nail
dystrophy, abnormal teeth, and colobomas.

98 -Cutaneous meningiomas have been associated with what syndrome?

A. Neurofibromatisis
B. Cowden
C. Neurocutaneous melanosis
D. Gorlin
E. Glomangiomatosis

►A

Cutaneous meningiomas are sperated into 3 types. Type 1 are thought to be misplaced meningeal
cells, or rudimentary mengioceles. Type II develop along the course of cranial nerves. Type III
lesions represent cutaneous metastasis or an underlying primary meningioma. Cutaneous
meningiomas have been associated with cranial developmental anomalies and in neurofibromatosis.

99-Which of the following is caused by a defect in a gap junction protein?

A. Epidermolysis bullosa simplex


B. Hailey-hailey
C. Erythrokeratoderma variabilis
D. Dyskeratosis congenita
E. Bullous ichthyosis of siemens

►C

Erythrokeratoderma variabilis is also known as Mendes da Costa Syndrome. It is caused by a


defect in connexin 31, a gap junction protein. EB simplex is caused by a mutation in keratins 5 &
14, Hailey-Hailey is caused by a mutation in calcium transporters, dyskeratosis congenita is caused
by a defect in rRNA synthesis, and bullous icthyosis of Siemens is caused by mutations in keratin
2e.

247
100- A patient has a bird head facies, "Mickey Mouse" ears, photosentivity, deafness and basal
ganglia calcifications has:

A. Cockayne syndrome
B. Xeroderma pigmentosum
C. Tay's syndrome
D. Louis Bar syndrome
E. Bloch-sulzberger disease

►A

The patient has Cockayne syndrome. It is autosomal recessive associated with a defect in
nucleoside excision repair with a mutation in the XPG gene. It is similar to XP with
photosensitivity, neurologic degeneration, deafness, retinal atrophy, basal ganglia calcifications and
peripheral neuropathy.

101- A 16 month-old girl presents with patchy alopecia, whorled erythematous scaly eruption, and
asymmetric limb shortening. What laboratory or radiologic test may aid in diagnosis?

A. Brain MRI
B. Alkaline phosphatase
C. Chest radiograph
D. Bone films
E. Complete blood count

►D

The patient has Conradi-Hunermann Syndrome. This is a X-linked dominant disorder characterized
by ichthyosiform erythroderma in Blaschko's lines in infancy which resolves with follicular
atrophoderma, patchy alopecia, short stature, cataracts, scoliosis, assymetric limb shortening. Bone
films will demonstrate stippled epiphyses. Ichthyosis and stippled epiphyses resolve after infancy.

102- Which of the following is correct about nevoid basal cell carcinoma syndrome?

248
A. known as Gardner syndrome
B. autosomal recessive
C. multiple keratoacanthomas
D. calicification of the falx
E. odontogenic steatomas

►D

Nevoid basal cell carcinoma syndrome, also known as Gorlin syndrome, is an autosomal dominant
disease, with abnormalities in the PTCH gene. It presents with multiple basal cell cancers early in
life, frontal bossing, hypertelorism, odontogenic keratocysts, and bifid ribs. The presence of
intracranial calcification of the falx cerebri is characteristic.

103 -A 7 year old boy presents to your office with short and sparse hair. He is also short for his age.
A hair mount reveals alternating light and dark bands under polarized light. Which of the following
statements is correct regarding this condition?

A. Eyebrows and eyelashes are not affected


B. There is no increased risk of skin cancer
C. Amino acid analysis of hair reveals high sulfur content
D. There is no increased hair fragility.
E. Dark bands represent air

►B

The patient described has trichothiodystrophy, with the hair finding of trichoschisis. These patients
have photosensitivity but no increased risk of skin cancer. They also have intellectual impairment,
decreased fertility, short stature, and progeria-like facies. Low sulfur content of the hair is found.
Eyebrows and eyelashes are also affected. Dark bands in pili annulati, not trichoschisis, are air
cavities.

104 -Which of the following metals is deficient in the serum of patients with Menkes kinky hair
syndrome?

249
A. Copper
B. Iron
C. Selenium
D. Zinc
E. Biotin

►A

Menkes kinky hair syndrome is transmitted in an X-linked recessive manner and is caused by a
mutation in ATP7A, an ATP-dependent copper tranporter. This defect results in low serum
levels of copper. These individuals will have hair abnormalities such as sparse, hypopigmented
brittle hair, eyelashes and eyebrows, lax skin, a "cupid's bow" upper lip, CNS progressive
deterioration, seizures, skeletal abnormalities and tortuous arteries. The other listed items are not
associated with Menkes syndrome.

105 -A child has ichthyosis and is found to have leukocytes with vacuoles filled with lipids on
peripheral smear. She most likely has:

A. Chanarin-Dorfman syndrome
B. Ichthyosis hystrix
C. Naxos disease
D. Ichthyosis bullosa of Siemens
E. Ichthyosis vulgaris

►A

This child has Chanarin-Dorfman syndrome, also called neutral lipid storage disease. This is an
autosomal recessive disorder characterized by accumulation of triglycerides in the cytoplasm of
leukocytes, muscle, liver, fibroblasts, and other tissues. Patients generally have normal blood
lipid levels and a finely scaling ichthyosis. They can also have extracutaneous involvement such
as cataracts, decreased hearing, myopathy, and neurologic abnormalities.

106 -Which type of porphyria is associated with hyponatremia?

250
A. Acute intermittent porphyria
B. Porphyria cutanea tarda
C. Variegate porphyria
D. Hereditary coproporphyria
E. Erythropoietic protoporphyria

►A

Acute intermittent porphyria can cause hyponatremia due to the syndrome of inappropriate
antidiuretic hormone secretion.

107 -A patient has macular telangiectasias on the oral mucosa, face and acral surfaces. These
patients also have epistaxis, melena related to angiomas in the GI tract and AV malformation in
other organs. The gene mutation is:

A. ALK1, ACVRL1
B. PTEN
C. TCS1, TCS2
D. Endokinase
E. Antigen kinase-1

►A

Patients with hereditary hemorrhagic telangiectasia or Osler-Weber Rendu have a mutation in


Endoglin or Activin receptor like kinase -1 (ALK1, ACVRL1) gene that result in HHT1 and
HHT2. Both genes play a role in the vascular system.

108 -What medication may exacerbate this autosomally dominant, acnatholytic disorder?

A. Phenytoin
B. Lithium
C. Oral contraceptives
D. Anti-malarials
E. Corticosteroids

251
►B

Darier's disease is autosomal dominant condition characterized by hyperkeratotic papules


coalescing into warty plaques and cobblestoned papules on mucosal surfaces. The cutaneous
manifestations may be exacerbated by lithium.

109 -In alkaptonuria there is a blue-black discoloration in the sclera near the insertion of the rectus
muscles. This sign is called the:

A. Osler sign
B. Hutchinson sign
C. Blue sign
D. Amyloid sign
E. Ochronosis

►A

Alkaptonuria is am autosomal recessive disease with a deficiency in homogentistic acid oxidase.


The osler sign is a blue black discoloration in the sclera near the insertion of the rectus muscles, oil
droplet opacities in the cornea, pigmented pingucela, granules in episclera.

110 -Ivory-colored papules between the angles of the scapulae are characteristic of which
syndrome:

A. Hurler
B. Scheie
C. Morquio
D. Hunter
E. Sanfilippo

►D

252
These syndromes are all mucopolysaccharidoses. These papules are characteristic of Hunter
syndrome which is caused by a deficiency in iduronate sulfatase.

111- A child presents with macroglossia, exopthalmos and gigantism. He has a history of
omphalocele repair and has circular depressions on the rim of the posterior helices. Although this
syndrome is most often transmitted in a sporadic manner, 15% of cases have defects in which gene?

A. KIP2
B. ATM
C. VHL
D. FLT4
E. FOXC2

►A

KIP2 can be mutated in 15% of cases of Beckwith-Wiedemann Syndrome. KIP2 is a


cyclindependent kinase inhibitor gene which acts as a negative regulator of cell proliferation. These
children have an increase risk of Wilms' tumor and organomegaly. In addition to the circular ear
depression, they can also have a linear earlobe crease. The remaining genes are mutated in other
syndromes with vascular disorders as a component: ATM in ataxia telangectasia, VHL in Von
Hippel-Lindau, FLT4 in Hereditary lymphedema and FOXC2 in Lymphedema-distichiasis
syndrome.

112- Hyper IgE syndromes are associated with genetic deficiency of which of the following?

A. WASp
B. IL-17 receptor
C. AIRE
D. FOXP3
E. Dock8

►E

To date, three genetic etiologies of hyper IgE syndromes have been identified: STAT3, DOCK8,

253
and Tyk2. All of these hyper IgE syndromes are characterized by eczema, sinopulmonary infections,
and greatly elevated serum IgE. However, each has distinct clinical manifestations. Mutations in
STAT3 cause autosomal dominant HIES (Job\'s syndrome), which is unique in its diversity of
connective tissue, skeletal, and vascular abnormalities. DOCK8 deficiency is characterized by
severe cutaneous viral infections such as warts, and a predisposition to malignancies at a young age.
Hyper IgE syndrome associated with Tyk2 deficiency is characterized by nontuberculous
mycobacterial infections.

113 -In one variant of epidermolysis bullosa simplex, those affected have muscular dystrophy in
addition to the skin findings. Which protein mutation has been linked to this finding?

A. Plakoglobin
B. Plakophilin
C. Plectin
D. Desmoglein
E. Desmocollin

►C

A plectin mutation is linked with this subtype of epidermolysis bullosa simplex.

114 -A patient has premature graying and a mutation in laminin A. This patient also has a bird like
face, early wrinkling, dyspigmentation, atrophy of skin, decreased subcutaneous fat and
atherosclerosis. This patient has:

A. Progeria
B. Hypotrichosis simplex
C. Costello syndrome
D. Marie-Unna
E. Cartilage hair hypoplasia

►A

Patients with premature aging with thin hair, bird like face, early wrinkling, and atherosclerosis

254
has progeria. THey also have dyspigmentation, atophy of the skin and decreased subcutaneous
fat.

115-You receive a hospital consult from the gastroenterology service for a 42 -year old woman with
esophageal cancer. They would like your opinion on the yellow, thickened areas on her palms
and soles in areas of pressure. When you speak with her, she says that her father had similar
problems and it runs in her family. Which of the following is defective?

A. TOC gene
B. Desmoplakin
C. Plakoglobin
D. Connexin 30.3
E. Connexin 31

►A

This case describes Howell-Evans syndrome. This AD syndrome characteristically has a PPK in
areas of pressure, oral leukoplakia and esophageal carcinomas. Desmoplakin is defective in
Carvajal syndrome and plakoglobin in Naxos syndrome. Connexin 31 and 30.3 are linked to
erythrokeratoderma variabilis, which includes a PPK, but not esophageal carcinoma.

116 -A 6-year-old boy presents with brachyonychia and three firm subcutaneous nodules with a
bluish hue about the trunk. Biopsy of a representative lesion is consistent with a pilomatricoma.
What is the most likely associated syndrome?

A. Noonan syndrome
B. Carney Complex
C. Nail-Patella syndrome
D. Werner syndrome
E. Rubinstein-Taybi syndrome

►E

Rubenstein-Taybe syndrome is caused by a amutation in CREB-Binding protein and presents

255
with brachyonychia, eruptive keloids, and multiple pilomatricomas. Nail-Patella syndrome is
associated with anonychia, along with hypoplastic patellae, triangular lunulae, radial subluxation,
Lester iris, iliac horns, and glomerulonephritis. The other disorders are not associated with
brachyonychia.

117 -What is the underlying gene defect for this transgrediens form of palmoplanter keratoderma

A. SLURP-1
B. TOC gene
C. Plakoglobin
D. Keratin type 1
E. Keratin type 9

►A

Attached picture is Mal de Meleda (keratosis palmoplantaris transgrediens) which is an autosomal


recessive form of diffuse PPK, associated with keratotic plaques that extend to the dorsal aspects of
the hands and feet ("transgrediens") and may overlie joints . Hyperhidrosis, superinfection, and
occasionally perioral erythema, brachydactyly, and nail abnormalities are associated. Mal de
Meleda is due to mutations in ARSB, which encodes SLURP-1. The other choices represent gene
defects for "non-transgrediens" forms of PPK (Plakoglobin in Naxos syndrome, TOC gene in
Howel-Evans syndrome, K1 in non-epidermolytic PPK "Unna-Thost", and K9 in epidermolytic
PPK "Vorner"

118- Patients with x-linked icthyosis are more prone to getting which two malignancies?

256
A. Pancreatic and acute lymphocytic leukemia (ALL)
B. Pancreatic and acute myelogenous leukemia (AML)
C. Testicular and AML
D. Testicular and ALL
E. Renal cell and ALL

►D

Patients with x-linked icthyosis have a 20% chance of having cryptorchidism and are more prone to
both testicular cancer and acute lymphocytic leukemia.

119- Dyshcromatosis symmetrica hereditaria (DSH) is a disorder characterized by asymptomatic


hypo- and hyperpigmented macules on the face and dorsal extremities. Mutations in which of the
following genes is associated with this condition?

A. DKC
B. PTEN
C. ADAR1
D. ERCC4
E. STK11

►C

Dyshcromatosis symmetrica hereditaria (DSH) is a rare autosomal dominant pigmentary disease


characterized by asymptomatic hypopigmented and hyperpigmented macules distributed on the face
and dorsal extremities. It is most common in patients of Asian descent but has also been reported in
patients of European and South American descent. The condition is typically limited
to the skin but has been associated in rare instances with neurological deterioration and brain
calcificiations. The lesions tend to appear in infancy and childhood and stabilize during
adolescence. Mutations in the double-stranded RNA-specific adenosine deaminase gene
(ADAR1 or DSRAD) underlie DSH.
The ADAR1 protein catalyzes the deamination of adenosine to inosine in double-stranded RNA
which is important for alternative splicing or alternations of codons for protein translation.
However, the precise mechanism by which these mutations lead to actual disease remain unknown.
There are two diseases that are phenoptypically similar to DSH and previously thought to be
related.

257
The first is dyschromatosis universalis hereditaria (DUH) which is characterized by a mixture of
hyperpigmented and hypopigmented macules that are widespread. DSH can be distinguished from
DUH by its localization to the face and distal acral sites.

The second disease is acropigmentatio retiticularis (AR, also known as acropigmentation of


Kitamura). AR is also characterized by hyperpigmentated macules on the dorsal hands and feet as
well as palmar pits. However, in contrast to DSH, it is notably devoid of hypopigmented macules.
Suzuki et al. sequenced 20 cases of patients presenting with DSH, DUH, and AR. All patients with
DSH demonstrated mutations in ADAR1, however, none of the patients with DUH or AR
demonstrated mutations. Based on these studies, they proposed that these diseases are in fact
distinct and separate from DSH. There are no effective therapies for DSH. However, it is important
to distinguish DSH from other more serious disorders that can present with pigment alterations such
as dyskeratosis congenita and xeroderma pigmentosum. Both of these conditions present with
photosensitivity and squamous cell carcinomas which are not present in DSH.

120 -Which of the following is caused by a defect in keratins 4 & 13?

A. White sponge nevus


B. Epidermolysis bullosa simplex
C. Epidermolysis bullosa simplex with myotonic dystrophy
D. Junctional EB with pyloric atresia
E. Cloustonǁs syndrome

►A

White sponge nevus is caused by a defect in keratins 4 & 13. The remaining entities have the

corresponding defects:
EB simplex→►►►keratins 5 & 14
EB simplex with myotonic dystrophy→►►►plectin
Junctional EB with pyloric atresia→►►►Integrin ɑ-6, β4
Cloustonǁs syndrome (hidrotic ectodermal dysplasia)→►►►connexin 30

121 -A patient with multiple lentigines and blue nevi may also have:

258
A. Deafness
B. Pulmonary valve stenosis
C. Atrial myxoma
D. Mental retardation
E. GI malignancy

►C

This patient may have a constellation of features associated with NAME syndrome, otherwise
known as Carney complex or LAMB syndrome. This condition is inherited in an autosomal
dominant pattern and is due to a defect in the PRKAR1A gene. This condition is characterized
by the following features: blue Nevi, Atrial myxomas, cutaneous Myxomas, and Ephelides. In
addition, testicular tumors are seen as well as sexual precocity. Finally, patients may have
endocrine abnormalities including pigmented nodular adrenocorticoal disease and Cushing
syndrome as well as pituitary adenomas. Deafness, pulmonary stenosis, GI malignancies, and
mental retardation are not features of this condition.

122 -AKT1 mutation is seen in which of the following condition?

A. Gardner syndrome
B. Proteus syndrome
C. Incontinentia pigmenti
D. Noonan syndrome
E. Beckwith-Wiederman syndrome

►B

Proteus syndrome is a sporadic condition due to mutation in AKT1. Clinical features include
subcutaneous lymphovenous malformations, capillary malformations, lipomas, connective tissue
nevi of palms/soles, hemihypertrophy, frontal bossing, hyperostoses of epiphyses & skull
(especially external auditory canal), scoliosis, bilateral ovarian cystadenomas, and parotid
monomorphic adenomas.

123 -Which of the following is a potentially serious complication of the blue rubber bleb nevus
syndrome?

259
A. Development of chondrosarcomas
B. Development of angiosarcomas
C. Gastrointestinal hemorrhage
D. Development of lymphedema
E. Development of fluid retention

►C

Blue rubber bleb nevus syndrome is characterized by multiple tender venous malformations of
skin and gastrointestinal tract, which can lead to gastrointestinal bleeding.

124 -What finding is seen on brain imaging of patients with Papillon-Lefevre Syndrome?

A. Tram track calcifications


B. Calcification of the falx cerebri
C. Calcification of the hippocampus
D. Calcification of the dura
E. Agenesis of the corpus callosum

►D

Pappilon Lefevre Syndrome is an autosomal recessive syndrome characterized by transgredient


PPK and periodontitis. There is a defect in cathepsin C. One sees dural calcification at the
tentorium and choroid plexus. Tram track calcifications are seen in
STurge-WEber. CAlcification of the falx cerebri and agenesis of the corpus callosum is seen in
basal cell nevus syndrome. Hippocampal calcification is seen in lipoid proteinosis.

125 -A patient is diagnosed with tuberous sclerosis with seizures, mental retardation and
hypopigmented macules has a mutation in the

A. TSC1 and TSC2 gene


B. PTEN gene
C. p53 gene
D. WEC gene

260
E. KI gene

►A

Patients with tuberous sclerosis have a mutation in the TSC1 and TSC2 gene. The encode for
tumor suppressor proteins hamartin (TSC1) and tuberin (TSC2). Other skin findings include
hypopigmented macules, facial angiofibromas, collagenomas, shagreen patch, periungual
fibromas, Koenen's tumors and forehead plaques.

126 -Angioid streaks on retinal exam are characteristic of which of the following syndromes?

A. Pseudoxanthoma elasticum
B. Choroid malformations
C. Eyelid papillomas
D. Lester iris
E. Salt & pepper retinitis pigmentosa

►A

Angioid streaks are characteristic of pseudoxanthoma elasticum. They are caused by rupture of
Bruch's membrane of the choroid. Choroid malformations are found in Sturge-Weber syndrome,
eyelid papillomas in xeroderma pigmentosum, Lester iris in Nail-patella syndrome and salt &
pepper retinitis pigmentosa in Refsum syndrome.

127- Which of the following is NOT a complication of Kasabach-Merritt Syndrome:

A. Thrombocytopenia
B. CHF
C. Ataxia
D. Disseminated intravascular coagulation
E. Gastrointestinal bleeding

►C

261
Kasabach-Merritt Syndrome results from platelet trapping. It occurs with tufted angiomas and
kaposiform hemangioendotheliomas. Hematologic complications include thrombocytopenia,
microangiopathic hemolytic anemia, DIC, and acute hemorrhage. The presence of large angiomas
can lead to high output failure (CHF) and they can also compress on surrounding structures.

128- A patient with this autosomal recessive disorder caused by a defect in helicase is an increased
risk for which malignancy?

A. Acute leukemia
B. Renal cell carcinoma
C. Medullary thyroid carcinoma
D. Squamous cell carcinoma of the lung
E. Prostate carcinoma

►A

Bloom's syndrome is an autosomal recessive disorder caused by a mutation in DNA helicase. It is


characterized by photodistributed erythema in a butterfly distribution, malar hypoplasia with a
prominent nose, high pitched voice, and an increased risk for malignancy (acute leukemia,
lymphoma, and GI adenocarcinoma.)

129- Papillon-Lefevre and Haim-Munk syndromes have which of the following symptoms?

A. Right-ventricular cardiomyopathy
B. Pseudoainhum
C. Esophageal cancer
D. Eccrine syringofibradenoma
E. Periodontitis with tooth loss

►E

Right-ventricular cardiomyopathy is associated with Naxos syndrome, pseudoainhum is


associated with Vohwinkel syndrome, esophageal cancer is associated with Howel-Evans
syndrome, and eccrine syringofibradenomas are associated with Schopf-Schulz-Passarge

262
syndrome. Periodontitis with tooth loss is associated with Papillon-Lefevre and Haim-Munk
syndromes, which are caused by mutations in Cathepsin C.

130-Most common malignancy to develop in a patient with tricholemmomas, acral verrucous


papules and cobble-stoning of buccal and gingival mucosa?
A. Thyroid cancer
B. Breast cancer
C. Colon cancer
D. Lymphoma
E. Melanoma

►B

Cowden's syndrome is an autosomally dominant inherited defect of PTEN. Patient may present
with multiple trichilemmoma, hamartomatous tumors of the breast, thyroid and endometrium,
acral keratoses and papillomatous papules. Breast cancer is the most common cancer to arise
while thyroid carcinoma is the second most common.

131 -A patient has multiple basal cell carcinomas and palmoplantar pits with bifid ribs. This patient
also has jaw cysts in the first decade of life. The gene mutation in this patient is:

A. PATCH
B. PTEN
C. MCS1, MCS2
D. p53
E. UBO

►A

This patient has basal cell nevus syndrome or Gorlin Syndrome. The mutation is in the PATCH gene
that encodes PTC protein involved in the sonic hedge hog pathway. It leads to multiple basal cell
carcinomas in the adulthood and also palmoplantar pits in nearly all patients.

263
132- A seven month old infant diagnosed with eczema on her face returns for a diaper-rash follow-
up. A one-month trial of topical antifungals has failed to improve the infant's systems. The part of
the physical exam that might prove most useful include:

A. Stool samples
B. Fontanelle examination
C. Examination of palms and soles
D. Cardiac ausculatation
E. Hearing test

►C

Acrodermatitis enteropathica presents itself in infancy once breastfeeding has stopped. It can
clinically mimic atopic dermatitis, seborrheic dermatitis and candidiasis. Clinical features include
scaly red rash around mouth, eyes, and palms, diarrhea, stomatitis, glossitis, alopecia, and failure to
thrive.

133- Giant lysosomal granules are seen in which disease?

A. Chediak-Higashi syndrome
B. Griscelli syndrome
C. Piebaldism
D. Incontinentia pigmenti
E. Carney complex

►A

Chediak-Higashi syndrome is caused by an autosomal recessive mutation in a lysosomal transport


gene (LYST, CHS1). This disorder is characterized by oculocutaneous albinism, ataxia, muscle
weakness, and giant lysosomal granules. There is an accelerated phase characterized by
lymphohistiocytic infiltration of reticuloendothelial system, pancytopenia and death.

134 -What is the inheritance pattern of chronic granulomatous disease?

264
A. Autosomal recessive
B. Autosomal dominant
C. X-linked recessive
D. X-linked dominant
E. Autosomal recessive and X linked recessive

►E

Chronic granulomatous disease is inherited in an autosomal recessive and x-linked recessive


manner. There are mutations 5 mutations total. If the mutation is present in CYBA (a cytochrome
subunit), NCF1 & 2 (neutrophil cytosol factors 1 & 2 & 4) it is inherited in an autosomal recessive
manner. The mutation in CYBB is X linked recessive.

135 -The presence of natal teeth and pincer nails suggests which disease entity?

A. Congenital syphillis
B. Thalidomide exposure in utero
C. Incontinentia pigmenti
D. Pachyonychia congenita
E. Anhidrotic ectodermal dysplasia

►D

Pachyonychia congentia is an autosomal dominant condition characterized by a constellation of


findings affecting ectodermal structures. These include the presence of natal teeth, steatocystoma
multiplex, follicular hyperkeratosis of the knees, elbows and extensor extremities, eruptive vellus
hair cysts, and oral leukokeratosis which is not pre-malignant. In addition, nail findings include
twenty-nail dystrophy, subungual hyperkeratosis with increase transverse curvature ("pincer nails")
and candidal paronychia. There are two forms of pachyonychia congenital: Type 1(Jadassohn-
Lewandowsky syndrome) caused by defects in keratin 6a and 16, and Type 2 (Jackson-Lawler type)
caused by defects in keratins 6b and 17. Anhidrotic ectodermal dysplasia is associated with peg-
shaped teeth, hypoanodontia, and a non-specific nail dystrophy. Likewise, incontientia pigmenti
also is characterized by anodontia and peg-shaped teeth and dystrophic changes of the nail. Finally
congenital syphilis is a well-recognized cause of pegged teeth. Limb deformities are the most
serious sequelae of thalidomide exposure in utero.

265
136 -The combination of gastrointestinal polyposis, nail atrophy, alopecia, generalized pigmentation
of skin, and melanotic macules of the fingers is characteristic of which of the following syndromes?

A. Nicolau-Balus syndrome
B. Peutz-Jeghers syndrome
C. Cronkhite-Canada syndrome
D. Cowden syndrome
E. Bannayan-Riley-Ruvalcaba syndrome

►C

Cronkhite-Canada syndrome is a sporadic gastrointestinal polyposis syndrome associated with


nail atrophy, alopecia, generalized pigmentation of the skin, and melanotic macules on the
fingers.

137-Non-bullous icthyosiform erythroderma is caused by which of the following mutations:

A. Transglutaminase-1 gene (TGM1)


B. 12R-lipoxygenase gene (ALOX12B)
C. Lipoxygenase-3 gene (ALOXE3)
D. Both 12R-lipoxygenase gene (ALOX12B) and lipoxygenase-3 gene (ALOXE3)
are correct
E. All of these answers are correct

►E

Non-bullous congenital erythroderma (NCIE)is an autosomal recessive disorder characterized by


a collodion baby presentation at birth, and generalized erythroderma with fine white scale,
palmoplantar keratoderma, and heat intolerance. NCIE may be caused by mutations in
transglutaminase-1. gene (TGM1), the 12R-lipoxygenase gene (ALOX12B), and the lipoxygenase-3
gene (ALOXE3). Mutations in the keratinocyte TGM1 gene interferes with normal cross-linking of
structural proteins and the lipid envelope, leading to defective cornification and desquamation.
ALOXE3 functions as an epoxy alcohol synthase using the product of ALOX12B as the preferred
substrate; either gene can be the site of mutations causing NCIE.

266
138 -A 2 year old girl presents with sunken eyes, large ears, microcephaly and a photodistributed
eruption on her face. Eye exam reveals ―salt and pepperǁ retina. The gene responsible for this
syndrome codes for a:

A. Transcription factor
B. Surface glycoprotein
C. Lysosomal protease
D. DNA helicase
E. Mismatch repair gene

►D

The patient described has Cockayne syndrome, an autosomal recessive disorder believed to be
due to a mutation in either DNA helicase ERCC6 or defective ERCC8 which does nucleotide
excision repair. UV irradiated cells have decreased DNA and RNA synthesis and increased
chromosomal breaks. Clinical features include cachectic dwarfism with microcephaly, thin nose,
large ears, photosensitive eruption, cataracts, salt & pepper retina, and diffuse demyelination.

139 -In biopsies from blisters in patients with junctional epidermolysis bullosa, the split is found in
the:

A. Basal cell layer of the epidermis


B. Lamina lucida
C. Lamina densa
D. Squamous cell layer of the epidermis
E. None of the answers are correct

►B

The split seen in junctional epidermolysis bullosa is in the lamina lucida. The other locations can
be involved in blistering disease, but not junctional epidermolysis bullosa.

140 -The following enzyme defect is most commonly seen in CHILD Syndrome.

267
A. 3-beta-hydroxysteroid dehydrogenase
B. 3-beta-hydroxysteroid isomerase
C. Aryl sulfatase E
D. NAD oxido reductase
E. DNA helicase

►A

CHILD Syndrome is a X-linked dominant disorder characterized by unilateral ichthyosiform


erythroderma, ipsilateral limb deformity, and ipsilateral organ hypoplasia. The most commom
gene defect is NSDHL which encodes 3-beta hydroxysteroid dehydrogenase. EBP gene defects
which encode 3-beta-hydroxysteroid isomerase have been described, however this is the usual
defect in Conradi-Hunermann Syndrome. Aryl sulfatase E is mutated in X-linked recessive
chondrodysplasia punctata.

141 -Medulloblastoma is seen in which syndrome?

A. Gardnerǁs syndrome
B. Multiple endocrine neoplasia 2b
C. Muir-Torre syndrome
D. Basal cell nevus syndrome
E. Neurofibromatosis Type 1

►D

Basal cell nevus syndrome is an autosomal dominant syndrome caused by a mutation in PTC gene,
which acts in the Sonic hedgehog pathway. Cutaneous manifestations of this genodermatosis
include basal cell carcinomas, palmoplantar pits, epidermoids cysts. Other findings include
odotogenic cysts, frontal bossing, bifid ribs, calcification of the falx cerebri and medulloblastomas.

142- Birt-Hogg-Dube syndrome is strongly associated with which of the following most
malignancies?

A. Basal cell carcinoma

268
B. Medulloblastoma
C. Renal cell carcinoma
D. Trichoepithelial carcinoma
E. Eccrine syringofibroadenoma

►C

Birt-Hogg-Dube syndrome is characterized by multiple fibrofolliculomas, trichodiscomas,


acrocollagenomas, lipomas, and oral fibromas. Patients develop renal cell carcinoma, colon cancer,
and medullary thyroid carcinoma.

143- Familial macular and lichen amyloidosis is a feature of which of the following conditions?

A. Sipple syndrome
B. Peutz-Jeghers syndrome
C. Marfan syndrome
D. Dyskeratosis congenita
E. Birt-Hogg-Dube syndrome

►A

Sipple syndrome (MEN 2a) is caused by autosomal dominant mutations in the ret protooncogene.
Patients develop parathyroid cancers, pheochromocytomas, and medullary cancer of the thyroid
gland. Familial macular and lichen amyloidosis is also a feature of this syndrome.

144- A 17 y/o man presents with facial acne that he would like treated. You notice that he has fine
brown scale on his neck and do a complete skin exam. This scale is present on the remainder of his
body, sparing his palms, soles and flexural areas. He informs you that his uncles on his mother‘s
side have similar skin findings. He is not concerned about the skin and would like to proceed with
acne treatment only. What other clinical exam should you perform to screen for a potential
malignancy to which this patient is at higher risk for acquiring?

A. Testicular
B. Abdominal
C. Lymph node

269
D. Lung
E. Rectal

►A

Men with x-linked ichthyosis are at increased risk of testicular cancer and cryptorchidism. A
testicular exam is simple to perform and a good screening exam for detecting testicular
abnormalities. The remaining exams are not useful as these patients are not at higher risk for other
types of cancer.

145 -Nail patella syndrome is inherited in an autosomal dominant fashion. The eye finding for this
disease with hyperpigmentation of the pupillary margin of the iris seen in 45% of patients is also
called:

A. Lester iris
B. Heterochromia of the iris
C. Microcornea
D. Glaucoma
E. Cataracts

►A

Nail patella syndrome is also known as hereditary osteonychodysplasia (HOOD) and is inherited
in an autosomal dominant fashion. The defect is in the gene LMX1B. The ocular abnormality is
Lester iris seen in 45% of patients.

146 -Ichthyosis hystrix is characterized by the following gene defects?

A. Keratins 1 and 9
B. Keratins 1 and 10
C. Keratins 5 and 14
D. Keratins 6 and 16
E. None of these answers are correct

270
►B

Ichthyosis hystrix or extensive epidermal nevi occurs secondary to a somatic mosaicism for
keratins 1 and 10. If the mosaicism occurs on gonadal cells, offspring may have full blown
epidermolytic hyperkeratosis (EHK).

147 -A double row of eyelashes is associated with:

A. Lymphedema-distichiasis syndrome
B. Cornelia de Lange syndrome
C. Rubinstein-Taybi syndrome
D. Russell-Silver syndrome
E. Hunters syndrome

►A

A double row of eyelashes is defined as distichiasis and is associated with the Lymphedema-
distichiasis syndrome. This syndrome is transmitted in an autosomal dominant fashion and is
related to a mutation in FOXC2. Findings include late onset lymphedema, distichiasis, corneal
irritation, ectropion, webbed neck and congenital heart defects. The remaining syndromes do not
include distichiasis as a feature.

148 -Which of the following is NOT part of the Carney complex?

A. Peg or conical teeth


B. Cardiac, cutaneous or mammary myxomas
C. Pigmented skin lesions
D. Endocrine abnormalities
E. Primary pigmented nodular adrenocortical disease

►A

Peg/conical teeth are not part of the Carney complex. This is found in incontinentia pigmenti and
anhidrotic ectodermal dysplasia. The remaining skin findings are part of this complex sometimes

271
known as NAME syndrome. It consists of multiple, diffuse mucocutaneous lentigines, cardiac
and subcutaneous myxomas and endocrine abnormalities may be present. Other findings include:
testicular tumors, thyroid disease, primary pigmented nodular adrenocortical disease,
psammomatous melanotic schwannomas and hormone-secreting pituitary adenomas.

149 -A patient has soft compressible blue tumors that are seen in the trunk and arms. There is also
nocturnal pain. This condition is also caused by a mutation in the VMCM1 gene:

A. Blue Rubber Bleb Nevus Syndrome


B. Kaposi's Sarcoma
C. Henoch-Schonlein Purpura
D. Hereditary Hemorrhagic Telangiectasia
E. Olser-Weber Rendu

►A

This patient has all the clinical signs of blue rubber bleb nevus syndrome. These patient can also
have melena that occurs with gastrointestinal hemangioma rupture. Other involvement are in the
lung, eye and CNS.

150 -The arylsulfatase E gene is mutated in which disease?

A. X-linked ichthyosis
B. Refsum syndrome
C. Haim-Munk syndrome
D. Naxos syndrome
E. Griscelli syndrome

►A

Arylsulfatase E is also known as steroid sulfatase and is mutated in X-linked ichthyosis. This
condition is inherited in a X-linked recessive pattern. Clinical findings include: brown scale sparing
palms, soles and flexures, comma-shaped corneal opacities, failure of labor progression and
cryptorchidism. It is also mutated in X-linked recessive type chondrodysplasia punctata.

272
151 -The syndrome characterized by generalized mild hyperkeratosis, erythematous keratotic
plaques, palmoplantar keratoderma, non-progressive sensorineural deafness, progressive bilateral
keratitis with secondary blindness is:

A. KID syndrome
B. Vohwinkel syndrome
C. Erythrokeratoderma variabilis
D. CHILD syndrome
E. Refsum syndrome

►A

KID syndrome is described above. It is an autosomal dominant mutation in connexin 26.


Vohwinkel syndrome is also a connexin 26 mutation, but is characterized by diffuse honeycombed
palmoplantar keratoderma, pseudoainhum, starfish-shaped keratotic plaques over joints and
deafness. Erythrokeratoderma variabilis is an autosomal dominant mutation in connexin 31 and
30.3 characterized by erythematous migratory patches, fixed hyperkeratotic plaques and a
palmoplantar keratoderma. CHILD syndrome is an X-linked dominant mutation condition due to a
mutation in NAD(P)H Steroid dehydrogenas-like protein, lethal in males. Unilateral ichthyosiform
erythroderma, limb/visceral hypoplasias are characteristic. Refsum syndrome is an autosomal
recessive condition with a mutation in phytanoyl coenzyme A hydroxylase characterized by mild
ichthyosis, cerebellar ataxia, peripheral neuropathy, retinitis pigmentosa (salt & pepper) and
deafness.

152 -The most common cardiovascular defect in patients with Noonan syndrome is:

A. Atrial septal defect


B. Ventricular septal defect
C. Enlarged aorta
D. Pulmonic valve stenosis
E. Aortic stenosis

►D

Noonans syndrome is similar to cardiofaciocutaneous syndrome as they both have mutations in


PTPN11. Noonan patients have short stature, ptosis, hypertelorism, low-set ears, thick lips and

273
curly hair. Pulmonic valve stenosis is the most common cardiovascular defect, with atrial septal
defects also seen.

153 -Which of the following syndromes is associated with hematologic abnormalities?

A. Sturge Weber disease


B. Klippel-Trenaunay-Parks-Weber
C. Kasabach-Merritt syndrome
D. Blue rubber bleb nevus syndrome
E. Zollinger-Ellison syndrome

►C

Kasabach-Merritt syndrome is associated with hematologic abnormalities, such as


thrombocytopenia, microangiopathic hemolytic anemia, disseminated intravascular coagulation.
The condition develops from platelet-trapping within a large hemangioma, most commonly a
kaposiform hemangioendothelioma in the retroperitoneal location.

154 -Patients that have been diagnosed with hypohidrotic ectodermal dysplasia have the hair
findings of:

A. Longitudinal groove on electron microscopy


B. Trichorrhexis nodosum
C. Monilethrix
D. Beaded hair
E. Pili trianguli et canaliculi

►A

Patients with hypohidrotic ectodermal dysplasia have longitudinal groove on electonmicroscopy.


They also have peg teeth with problems with sweating.

274
155 -The most common ocular association with cutis marmorata telangiectatica congenital is:

A. Cataracts
B. Glaucoma
C. Retinoblastoma
D. Corneal opacity
E. Angioid streaks

►B

Glaucoma is the most common associated eye finding in CMTC patients. Glaucoma is also seen
in patients with neurofibromatosis type 1 and Sturge Weber patients.

156 -A child presents with the hair finding seen in the image in addition to brittle nails, keratosis
pilaris, abnormal teeth and cataracts. Which of the following abnormalities is the most likely
mutated?

A. Keratin 1/10
B. Keratin hHb1/hHb6
C. Keratin 6/16
D. Keratin 6/17
E. Keratin 2e

►B

Keratin hHb1/hHb6 is defective in monilethrix, which is described above. Keratin 1/10 defects
are found in epidermolytic hyperkeratosis, K6/16 in inflamed skin and pachyonychia congenita
type I, K6/17 in pachyonychia congenita type II and K2e in Ichythosis bullosa of Siemens.

157 -A patient with Bloom Syndrome is most likely to have which laboratory abnormalities:

A. Decreased immunoglobulins
B. Macrocytic anemia
C. Elevated IgE

275
D. Thrombocytopenia
E. Positive ANA

►A

Bloom syndrome is an autosomal recessive disorder due to a mutation in the BLM gene which
codes for a DNA helicase. Patients have impaired DNA repair after UV exposure and increased
photosensitivity. Clinical features include photodistributed erythema, cheilitis, high -pitched
voice, hypogonadism, and increased risk for leukemia, lymphoma and GI adenocarcinoma.
Laboratory evaluation reveals decreased IgA, IgM and IgG leading to increased risk of respiratory
infections

158- Which of the following laboratory test might prove useful in the diagnosis of Fabry disease:

A. Complete blood count with differential


B. Fasting lipids
C. Urinary sediment exam with polarizing light microscopy
D. Stool guaiac
E. Bleeding time

►C

Patients with Fabry disease have a defect in the alpha-galactosidase A enzyme, leading to an
accumulation of glycosphingolipids in all tissues. Although patients are at increased risk for
myocardial infrctions and strokes, the serum lipid levels are normal. Ischemic events occur as a
result of glycosphingolipid accumulation in endothelial cells leading to swelling. In the brain,
strokes occur from direct vessel occlusion or stretching and distention of branches of dolichoectatic
parent vessels. Deposits in the kidneys leads to progressive renal failure with urine exam exhibiting
proteinuria and birefringent lipid globules (―maltese crossesǁ) seen with polarizing light
microscopy.

159- Osteopathia striata is seen in which of the following disorders?

A. Gaucher‘s disease

276
B. Albright‘s syndrome
C. Aplasia cutis congenita
D. Focal dermal hypoplasia
E. Scleroderma

►D

Focal dermal hypoplasia (Goltz syndrome) is an X-linked dominant disorder that is lethal in males.
There is linear atrophy following Blaschko‘s lines with areas of fat herniation with underlying
osteopathia striata, which is radiologically characterized by linear bony hyperdensity. Other features
include mucocutaneous papillomas and pits, alopecia, nail dystrophy, tooth abnormalities, and
colobomas.

160- Christ-Siemens-Touraine Syndrome is most commonly linked with defects in which of the
following genes?

A. NEMO
B. Ectodysplasin (EDA)
C. ERCC2
D. ATP7A
E. None of these options are correct

►B

Ectodysplasin (EDA) on Xq12-q13 is transmitted in an X-linked recessive fashion and is the most
common cause of anhidrotic ectodermal dysplasia (Christ-Siemens-Touraine syndrome or
hypohidrotic ectodermal dysplasia). NEMO can be linked to this syndrome and is associated with
immunodeficiency, but this is a rare association. ERCC2 is associated with trichothiodystrophy and
ATP7A with Menkes kinky hair syndrome. They are not associated with anhidrotic ectodermal
dysplasia.

161 -A patient with a port wine stain covering one enlarged leg likely has which of the following
associated symptoms?

277
A. Lymphatic and deep venous insufficiency
B. Visceromegaly with omphalocele
C. Bilateral retinal hemangioblastomas
D. Enchondromas
E. Distichiasis

►A

Klippel-Trenaunay-Weber syndrome is a sporadic condition characterized by port-wine stains


typically covering one lower extremity that is enlarged with underlying lymphatic and d eep venous
insufficiency.

162- The most common neoplasm seen in Maffucci Syndrome is:

A. Enchondromas
B. Angiosarcomas
C. Osteosarcomas
D. Lymphangiosarcomas
E. chondrosarcoma

►A

Maffucci syndrome comprises of superficial and deep venous malformations, enchondromas, and
short stature. Enchondromas are the most common neoplasm, while chondrosarcomas are the most
common malignancies.

163 -Which ocular finding may be seen in a patient with this skin condition?

A. Comma-shaped corneal opacities


B. Retinitis pigmentosa
C. Congenital hypertrophy of the retinal pigmented epithelium
D. Angioid streaks
E. Pingueculae

278
►D

Pseudoxanthoma elasticum is caused by a defect in connective tissue. Angioid streaks develop when
a rupture occurs in Bruch's membrane.

164- The gene defect in LEOPARD syndrome is:

A. PTPN11
B. PRKAR1A
C. KIP2
D. ATM
E. Neurofibromin

►A

PTPN11 gene is mutated in LEOPARD syndrome. This syndrome consists of the complex of
Lentigines, EKG abnormalities, Ocular hypertelorism, Pulmonary stenosis, Abnormal genitalia,
Retardation of growth and Deafness. PRKAR1A is associated with the Carney complex of diseases.
KIP2 is found in 15% of cases of Beckwith-Wiedermann syndrome. ATM is mutated in ataxia
telangectasia syndrome and Neurofibromin in Neurofibromatosis type I.

165- Ectopia lentis (downward displacement of the lens) is characteristic of:

A. Marfan syndrome
B. Homocystinuria
C. Phenylketonuria
D. Multiple Carboxylase deficiency
E. Cutis laxa

►B

Ectopia lentis (downward displacement) is seen in homocystinuria. Upward displacement is seen in


Marfan syndrome. There are no changes in the lens in phenylketonuria or multiple carboxylase
deficiency.

279
166 -Which of the following is defective in Ehlers-Danlos syndrome (EDS) with congenital adrenal
hyperplasia?

A. Tenascin-X
B. Lysyl oxidase
C. Lysyl hydroxylase
D. None of these answers are correct
E. All of these answers are correct

►A

Tenascin-X defects are associated with EDS and with congenital adrenal hyperplasia. The
phenotype is that of typical EDS with hyperextensible skin, hypermobile joints, and tissue fragility.
Lysyl oxidase is defective in X-linked EDS (type V) and Occipital horn syndrome (type IX). Lysyl
hydroxylase is defective in ocular-scoliotic (type VI) EDS.

167 -A child with phenylketonuria likely presents with which cutaneous problems?

A. Blue-gray generalized hyperpigmentation


B. Alopecia universalis
C. Generalized hypopigmentation
D. Generalized hyperpigmentation
E. Leg ulcers

►C

Phenylketonuria is an autsomal recessive disorder caused by a mutation on the long arm of


chromosome 12. A deficiency of phenylalanine hydroxylase or its cofactor tetrahydrobiopterin
leads to accumulation of phenylalanine. Clinical features include generalized hypopigmentation,
eczematous dermatitis, sclerodermoid changes, seizures, psychomotor delay, urine with ―mousyǁ
odor, mental retardation.

168 -A 7 year old girl with abnormally short hair has progressive seizures, lethargy, ataxia, and
mental retardation. A blood test reveals elevated levels of ammonia. What is the affected gene?

280
A. Argininosuccinase
B. ATP7A
C. Ectodysplasin A
D. Connexin 30
E. p63

►A

Argininosuccinic aciduria is an autosomal recessive disorder caused by mutations in


argininosuccinase. It is characterized by trichorrhexis nodosa, hyperammonemia, hepatomegaly and
vomiting, seizures, lethargy, coma, ataxia, and mental retardation.

169 -A 20-year-old woman presents with hypodontia, sparse hair, palmoplantar hyperkeratosis,
and nail dystrophy. Examination of her eyelids reveal multiple, translucent-appearing papules. The
most likely gene defect is:

A. WNT10A
B. PTCH
C. CYLD
D. BRAF
E. PTEN

►A

Schopf-Schulz-Passarge syndrome is characterized by multiple apocrine hydrocystomas (most


commonly appearing on the eyelids) and syringofibroadenomas, in addition to hypodontia,
hypotrichosis, onychodystrophy, and palmoplantar keratoderma. It is inherited as an autosomal
recessive condition and is caused by mutations in WNT10A, which encodes a cutaneous
signaling molecule involved in ectodermal appendageal development.

170 -A teenage female presents with the complaint of "nail fungus". On exam, she has triangular
lunulae, palmoplantar hyperhidrosis, micronychia and an absent patella. Which of the following
screening tests should you order first?

281
A. Urinalysis
B. CBC
C. Fasting lipids
D. Renal ultrasound
E. X-ray of the knees, elbows and pelvis

►A

Patients with nail-patella syndrome can have glomerulonephritis and renal dysplasia leading to renal
failure. Screening with a urinalysis is a reasonable first test. If this is abnormal, referral for a renal
ultrasound could be useful. Fasting lipids and a CBC are not indicated.

171- The porphyrias are a group of diseases related by abnormal heme synthesis. Which is the only
porphyria that is inherited in an autosomal recessive manner?

A. Congenital erythropoietic porphyria


B. Variegate porphyria
C. Porphyria cutanea tarda
D. Acute intermittent porphyria
E. Erythropoietic protoporphyria

►A

The porphyrias are generally inherited in an autosomal dominant manner. Congenital erythropoietic
porphyria is unique in being inherited in an autosomal recessive manner. Helpful mnemonic: all
porphyrias are autosomal dominant exCEPt one.

172- A 4 year old girl suffers multiple fractures. She also has thin skin, easy bruising, and blue
sclera. An echocardiogram reveals mitral valve prolapse. What type of osteogenesis imperfecta does
she most likely have?

A. Type I
B. Type II
C. Type III

282
D. Type IV
E. Type V

►A

Osteogenesis imperfect is a defect in collagen I which leads to thin skin, easy bruising, blue sclera,
and multiple fractures. Mitral valve prolapse is seen especially in Type I.

173- Which one of the following is the most common oncogenic virus in patients with
epidermodysplasia verruciformis?

A. HPV-5
B. HPV-8
C. HPV-13
D. HPV-16
E. HPV-33

►A

Epidermodysplasia verruciformis is a rare autosomal recessive disorder in which an impaired


cellular immunity allows widespread infection with certain subtypes of the human papilloma virus
(HPV). Some of these lesions have a tendency for malignant transformation, most commonly those
verruca caused by HPV type 5.

174- Collagen III is mutated in which type(s) of Ehlers-Danlos syndrome (EDS)?

A. All of the answers are correct


B. None of the answers are correct
C. Benign Hypermobile (type III)
D. Vascular (type IV)
E. Periodontitis (type VIII)

►A

283
Collagen III is mutated in all three types of EDS listed. Benign hypermobile type EDS is associated
with hypermobile joints and is autosomal dominant (AD) in transmission. Vascular type EDS is
associated with arterial and visceral rupture leading to early death, and visible venous patterns. It is
transmitted autosomal recessive (AR) or AD. Periodontitis type EDS is associated with mild EDS
symptoms and periodontitis.

175- Painful crises and 'whorled' corneal opacities are seen with which of the following enzyme
abnormalities?

A. Homogentisic acid oxidase


B. Alpha-galactosidase A
C. Glucocerebrosidase
D. Iduronate sulfatase
E. Glucoronidase

►B

Painful crises and whorled corneal opacities are found in Fabry disease which is caused by a defect
in alpha-galactosidase A.

176 -Acropigmentation of Dohi is characterized by:

A. Reticulated pigmentation of the axillae, neck, and groin


B. Linear palmar pits and pigmented macules on volar and dorsal hands and feet
C. Pigmented and depigmented macules on the distal dorsal extremities and face
D. Hyperpigmented macules on the lips and oral mucosa
E. Flaccid, superficial pustules that burst and leave pigmented macules

►C

Patients with Acropigmentation of Dohi (dyschromatosis symmetrica hereditaria) are usually


from Europe, India, or the Carribean. They develop pigmented and depigmented macules on
dorsal distal extremities and face. This disorder is due to mutation in DSRAD gene. Reticulated
pigmentation of the axillae, neck, and groin is seen in Dowling-Degos' disease. Linea palmar pits

284
and pigmented macules on volar and dorsal hands an feet is seen in Reticulate acropigmentation
of Kitamura. Hyperpigmented macules on oral mucosa and lips can be seen in Peutz-Jeghers,
Cronkite-Canada, and Laugier-Hunziker syndromes. Flaccid, superficial pustules that burst and
leave pigmented macules is seen in transient neonatal pustulosis.

177 -Beckwith-Wiedemann syndrome is characterized by which of the following triads?

A. Hemangioblastomas, renal cysts and renal cell carcinoma


B. Epistaxis, telangictases, and gastrointestinal tract bleeding
C. Enlarged limb, port wine stain, and deep venous thrombosis
D. Omphalocele, venous malformations, and ataxia
E. Exomphalos, macroglossia, and gigantism

►E

Beckwith-Wiedemann syndrome is also known as EMG syndrome as it includes exomphalos,


macroglossia, and gigantism. It is usually a sporadic condition but is sometimes caused by
autosomal dominant mutations in p57. Clinical features include facial capillary malformations,
macroglossia, visceromegaly with omphalocele, and hemihypertrophy associated with tumors
(especially Wilm‘s tumors).

178 -A patient with multiple sebaceous adenomas should be screened with which of the following
examinations?

A. Retinal examination
B. Laryngoscopy
C. Colonoscopy
D. MRI of the spine
E. Renal ultrasound

►C

Muir-Torre syndrome is an autosomal dominant disorder caused by the HMSH2 and MLH1
DNA mismatch repair genes. Clinically, there are numerous sebaceous adenomas, epitheliomas

285
and carcinomas and multiple keratoacanthomas associated with indolent colon and other visceral
adenocarcinomas. Patients and first-degree relatives should be screened by colonoscopy as
colonic adenocarcinomas may precede the development of cutaneous tumors.

179 -Hereditary Hemorrhagic Telangiectasia syndrome is transmitted in an autosomal dominant


fashion and can have two variants. Type I is linked to defects in HHT1, the endoglin gene. Type
II is linked to defects in HHT2, the ALK1 gene. What feature that differentiates type I from type
II clinically?

A. Type I families have an increase incidence of pulmonary arteriovenous fistulas


B. Type II families have an increased incidence of pulmonary arteriovenous fistulas
C. Type I families have an increased incidence of hepatic arteriovenous malformations
D. Type II families have a decreased incidence of hepatic arteriovenous malformations
E. None of the answers are correct
►A

There is an increased incidence of pulmonary arteriovenous fistulas in HHT type I. Type II has
an increased incidence of hepatic arteriovenous malformations. Note ALK1 ACVRL1.

180 - Which of the following elastic tissue diseases demonstrates calcified elastic fibers?

A. Cutis laxa
B. Marfan syndrome
C. Anetoderma
D. Pseudoxanthoma elasticum
E. Buschke-ollendorf syndrome

►D

Pseudoxanthoma elasticum is usually an autosomally recessive inherited condition due to a


defective transport protein, ABCC6. The clinical manifestations of the disease arise from
fragmented and calcified fibers of the skin, eyes and arteries. Patients may have yellow papules,
loose redundant skin, angioid streaks and hemorrhage. Histologically, the hallmark of
pseudoxanthoma elasticum is calcified elastic fibers.

286
181- A patient presents with starfish keratoses, pseudoainhum, honeycombed PPK, and generalized
ichthyosis. What is the most likely genetic defect?

A. Connexin 31
B. Connexin 26
C. Connexin 30
D. Loricrin
E. Connexin 33

►D

The patient has Vohwinkel syndrome. This is an autosomal dominant syndrome with 2 clinical
variants. The variant described above with generalized ichthyosis is due to a loricrin mutation. In
the classic form with nonprogressive hearing loss connexin 26 is mutated.

182 -Patients with progeria typically die of which of the following conditions?

A. Infection
B. Metastatic carcinoma
C. Atherosclerotic heart disease
D. Nail atrophy
E. Progressive systemic sclerosis

►C

Progeria (Hutchinson-Gilford syndrome) is a sporadic condition characterized by lipoatrophy,


sclerodermoid skin, alopecia, nail atrophy, craniomegaly with small face, muscle/bone wasting, and
severe premature atherosclerosis resulting in early death.

183- Mutations affecting the VEGF receptor-3 cause which of the following disorders?

A. Hereditary lymphedema (Nonne-Milroy disease)


B. Lymphedema-distichiasis syndrome
C. Lymphedema and ptosis

287
D. Noonan syndrome
E. Hereditary hemorrhagic telangiectasias

►A

Hereditary lymphedema (Nonne-Milroy disease) is an autosomal dominant condition caused by


mutations in the FLT4 gene which encodes for VEGF receptor-3. There is congenital lymphedema
and chylous ascites, scrotal swelling, intestinal tract protein loss, persistent bilateral pleural
effusion, and hypoproteinemia.

184 -Which of the following is a feature of Neurofibromatosis type II?

A. Congenital hypertrophy of the retinal pigment epithelium


B. Lisch nodules
C. Juvenile posterior subcapsular lenticular opacities
D. Lester iris
E. Optic gliomas

►C

Neurofibromatosis type II is an autosomal dominant disorder caused by mutations in schwannomin/


merlin. Clinical features include cutaneous schwannomas and neurofibromas, bilateral vestibular
schwannomas, and juvenile posterior subcapsular lenticular opacities. Optic gliomas are usually
seen in NF1.

185- Which syndrome is due to a defective secreted mammilian Ly6/uPAR-related protein-1?

A. Netherton's syndrome
B. Refsum's syndrome
C. Sjogren-Larsson syndrome
D. Mal de Meleda syndrome
E. Haim-Munk syndrome

288
►D

Mal de Meleda, also known as keratoderma palmoplantaris transgrediens, is due to a defect in


secreted mammilian Ly6/uPAR-related protein or SLURP-1.

186- You are examining a child with mild albinism, immunodeficiency and silver grey highlights in
his hair. You diagnose the child with Chediak-Higashi syndrome. Why are you confident that this
isn‘t Griscelli syndrome?

A. Giant lysosomal granules are present in neutrophils in the blood smear


B. Griscelli syndrome does not have albinism as a feature
C. Griscelli syndrome has no changes in hair color
D. All of these answers are correct
E. None of these answers are correct

►A

Chediak-Higashi syndrome and Griscelli syndrome have similar features including silver-grey
highlights of hair, immunodeficiency, mild albinism and an accelerated phase of disease. Examining
a peripheral blood smear is helpful in distinguishing between these two syndromes. Patients with
the LYST defect (a lysosomal storage transport gene) have Chediak-Higashi syndrome and will
have giant lysosomal granules visible in white blood cells on a blood smear.

187 -Marfan's syndrome is characterized by striae distensae, abdominal wall hernia, elastosis
perforans serpiginosa, and tall stature. The genetic defect is:

A. Fibrillin 1
B. Intermediate fibers 1
C. Actin 5
D. Req12
E. PTPN11

►A

289
Marfan's syndrome is caused by a mutation in the fibrillin 1, it is a major component of
intermediate fibrils found in the skeletal, ocular and cardiovascular system. The cardiac
irregularities are aortic root aneurysms, rupture and dissection.

188- A 9 year old boy has sparse, short, brittle hair. A blood test reveals low serum copper. What
is the inheritance pattern of this disease?

A. X-linked recessive
B. X-linked dominant
C. Autosomal dominant
D. Autosomal recessive
E. Mitochondrial

►A

Menkes kinky hair syndrome is an x-linked recessive disorder caused by defects in ATP7A, an ATP-
dependent copper transporter. It is characterized by pili torti, trichorrhexis nodosa, short, britle hair,
lax skin, CNS deterioration, seizures, and tortuous arteries.

189- Primary pigmented nodular adrenocortical disease and psammomatous melanotic


schwannomas are characteristic of which of the following syndromes?

A. Hypomelanosis of Ito
B. Carney complex
C. McCune-Albright syndrome
D. Gaucher‘s syndrome
E. Tuberous sclerosis

►B

Carney complex is an autosomal dominant disorder caused by mutations in PRKAR1A (protein


kinase A regulatory subunit 1-alpha). Key features include cardiac, cutaneous and mammary
myxomas, pigmented skin lesions, endocrine abnormalities (pituitary, testicular, thyroid, etc),
primary pigmented nodular adrenocortical disease, and psammomatous melanotic schwannomas.

290
190 -Junctional epidermolysis bullosa with pyloric atresia is associated with mutations in:

A. The alpha-6 subunit of integrin


B. The beta-4 subunit of integrin
C. Both subunits of integrin can have mutations causing this type of junctional
epidermolysis bullosa
D. Plectin
E. Laminin 5

►C

Both subunits of integrin can have mutations causing this type of junctional epidermolysis bullosa.
Plectin is associated with epidermolysis bullosa simplex with muscular dystrophy. Laminin 5 is
mutated in Herlitz and non-Herlitz types of junctional epidermolysis bullosa.

191- A 2 year old patient is small for his age. He has thin, pale, translucent skin, with prominent
veins. His father died from aortic rupture when he was in his twenties. Vascular Ehlers -Danlos is
caused by mutations in what gene?

A. Type III Collagen


B. Tenascin-X
C. Type V Collagen
D. Lysyl hydroxylase
E. Type I Collagen

►A

Vascular Ehlers-Danlos (Type 4) is characterized by the skin findings discussed above, as well as
the onset of severe complications of fragile blood vessels and organs in early adulthood. The
disease is inherited in an autosomal dominant manner. Defects are seen in type III collagen.
Tenascin-X mutations are seen in hypermobile Ehlers-Danlos (Type 3), type V collagen in classical
Ehlers-Danlos (Type 1 and 2), lysyl hydroxylase in kyphoscoliosis Ehlers-Danlos (Type 6), and type
I collagen in Ehlers-Danlos types 1, 2, and 7.

291
192 -You are consulted on a patient with possible Nethertons Syndrome. Which location of the body
would most likely have hairs demonstrating trichorrhexis invaginata?

A. Scalp
B. Eyebrow
C. Eyelash
D. All of these answers are correct
E. None of these answers are correct

►B

Eyebrow hair is most common site with hairs demonstrating trichorrhexis invaginata.

193 -A 3 year old boy has white hair of the central frontal scalp and depigmented symmetrical
patches on the knees since birth. No ocular abnormalities or deafness are noted. What is true of this
disorder?

A. It is an autosomal recessive disorder of melanocyte development


B. There is typically no progression of depigmented patches
C. Regression of the white forelock has been noted
D. One form of this condition is associated with iris pigmentary abnormalities
E. Results from a mutation in a tumor suppressor gene

►B

The patient in this scenario has piebaldism, an autosomal dominant disorder of pigmentation
caused most commonly by dominant negative missense mutations in the KIT proto-oncogene.
Piebaldism almost always has no progression of the depigmented patches, and is thought to be a
static disoder. One exception was noted in a family with a novel Val620Ala (1859T>C) mutation
in the KIT gene where development of new depigmented patches did occur. Regression of the
white forelock has been noted in select cases. Piebaldism is thought to be unresponsive to
systemic treatments or phototherapy. If the patient presented with either eye abnormalities or
deafness, Waardenburg syndrome would be considered in the diagnosis.

292
194 -Mutation in lamin A (nuclear envelope protein) has been found in:

A. Peutz-Jeghers syndrome
B. Buschek-Ollendorf syndrom
C. Progeria (Hutchinson-Gilford)
D. Albrightǁs syndrome
E. Marfan syndrome

►C

Progeria (Hutchinson-Gilford syndrome) is a sporadic condition characterized by lipoatrophy,


sclerodermoid skin, alopecia, nail atrophy, craniomegaly with small face, muscle/bone wasting,
and severe premature atherosclerosis resulting in early death. Recent studies have shown that
mutations in nuclear envelope protein lamin A is associated with progeria.

195 -Which of the following immunoglobulins is commonly decreased in


Wiskott-Aldrich syndrome?

A. IgA
B. IgD
C. IgE
D. IgM
E. IgG

►D

IgM is decreased in WAS. IgA, IgD and IgE levels are all elevated. IgG is not abnormal in WAS.

196 -A 16 year-old girl presents with a family history of Gardner syndrome. Her mother is very
concerned that her daughter may have the syndrome as it runs in her family and she has many
skin complaints. Gardner syndrome has been linked to defects in beta-catenin mediated
transcription. Which of the following genes dysfunction is responsible?

A. APC

293
B. STK11
C. CYLD
D. PTCH
E. ABCC6

►A

APC gene has been associated with defects in beta-catenin mediated transcription. The
remaining genes are not associated with Gardner syndrome.

197 -Which of the following are cutaneous features of Marfan syndrome?

A. Loose skin and crumpled ears


B. Dermatofibrosis lenticularis and striae
C. Fat herniation and cutaneous atrophy
D. Sclerodermoid changes and dyspigmentation
E. Striae and elastosis perforans serpiginosa

►E

Marfan syndrome is an autosomal dominant disorder caused by mutations in fibrillin 1 and 2.


Patients have tall stature, arachnodactyly, pectus excavatum, high-arched palate, joint laxity, ectopia
lentis with upward dislocation, dilated aorta with rupture, mitral valve prolapse, striae, and elastosis
perforans serpiginosa.

198 -A patient that has patchy leukoderma containing smaller spots of hyperpigmentation and a
white forelock with CNS abnormalities with impaired motor coordination has a defect in the:

A. c-KIT gene
B. PAX 3 gene
C. MITF gene
D. SOX10 gene
E. EDN3 gene

294
►A

This patient has piebaldism, which is autosomal dominant caused by a defect in the porto oncogene
c-KIT. The gene encodes tyrosine kinase receptor on melanocytes. It is associated with CNS
abnormalities with motor coordination, cerebellar ataxia, mental retardation, and deafness.

199- A patient has multiple erythematous papules with central keratotic plug with many annular or
serpiginous patters on the neck. This condition is associated with all the following except for:

A. Marfan's syndrome
B. Down's syndrome
C. Osteogenesis imperfecta
D. Ehlers-Danlos syndrome
E. Pseudoxanthoma elasticum

►E

The patient has elastosis perforans serpiginosa and is associated with medications and certain
systemic conditions. It is associated with the medicine penicillamine, Marfan's syndrome, Down's,
osteogenesis imperfecta, and Ehler's Danlos syndrome. It is often self limiting.

200 - A patient you notice has unruly hair that looks like spun-glass hair. She is found to have
uncombable hair syndrome, which is a condition with the associated hair findings of:

A. Pili trianguli et canaliculi


B. Beaded hair
C. Trichorrhexis nodosa
D. Longitudinal groove on electron microscopy
E. Monilethrix

►A

Uncombable hair syndrome has the characteristic findings of spun-glass hair or cheveux
incoiffables. It is associated with pili trianguli et canaliculi and can be inherited in an autosomal
dominant or autosomal recessive fashion.

295
201- All of the following disorders are exacerbated by UV radiation except:

A. Bloom syndrome
B. Hartnup‘s disease
C. Refsum syndrome
D. Cockayne syndrome
E. Rothmund-Thomopson syndrome

►C

Refsum‘s syndrome is an autosomal recessive disorder caused by mutations in phytanoyl -CoA


hydroxylase. Clinically, patients have mild icthyosis, cerebellar ataxia, polyneuropathy, salt and
pepper retinitis pigmentosa, sensorineural deafness, and arrhythmias with heart block. They are not
overly sensitive to UV radiation.

202 -A patient with Klinefelter Syndrome may be expected to experience which of the following:

A. Recurrent pulmonary infections


B. Recurrent leg ulcers
C. Pulmonary valve stenosis
D. Gastroesophageal reflux
E. Scarring alopecia

►B

Klinefelter syndrome results from nondisjunction during meiosis, leading to the XXY genotype.
Patients are characteristically tall (long lower extremities) with scant body and pubic hair.
Klinefelter patients have numerous varicosities predisposing them to recurrent leg ulcers.

203- What is the gene defect in this condition, which is also called Mendes da Costa syndrome?
A. Keratin 1 and 10
B. SPINK5
C. SLURP-1
D. Connexin 26

296
E. Connexin 31

►E

Mendes da COsta syndrome is also called Erythrokeratoderma Variabilis. It is an autosomal


dominantly inherited due to a mutation in connexin 31 or connexin 30.3. It is characterized by
transient geographic patches of erythema and hyperkeratotic plaques.

204- Pruritus is Sjogren Larsson syndrome is attributed to accumulation of what molecule(s)?

A. Bile salts
B. Histamine
C. Leukotriene
D. All of these answers are correct
E. None of these answers are correct

►C

Accumulation of leukotriene B4 contributes to pruritus in Sjogren Larsson syndrome. Leukotriene


inhibitors may be helpful in controlling symptoms.

205- Which of the following is true regarding tuberous sclerosis?

A. Confetti-like macules are typically present at birth


B. Facial angiofibromas are the most common cutaneous manifestation
C. Hypomelanotic macules (ash leaf spots) have a decreased number of melanocytes

297
D. Periungual fibromas are considered a major feature in the diagnosis of tuberous sclerosis
E. 6 or more hypomelanotic macules (ash leaf spots) are considered a major feature in the
diagnosis of tuberous sclerosis

►D

The earliest and most common cutaneous manifestation of tuberous sclerosis are hypomelanotic
macules (ash leaf spots), typically presenting at birth or early infancy. 3 or more of these lesions
are considered a major criteria in diagnosis. Melanocyte numbers are normal. Confetti-like macules,
on the other hand, are usually not apparent until the second decade of life. Facial angiofibromas
occur in approximately 75% of patients, and tend to become more prominent with age. Periungual
fibromas are considered a major feature for diagnosis, usually presenting around puberty to early
adulthood.

206 -Urticaria pigmentosa is linked to a defect in the c-kit protooncogene. What autosomal
dominant skin disease also has been linked to this defect?

A. Piebaldism
B. Hypomelanosis of Ito
C. Waardenburg syndrome
D. Hermansky-Pudlak syndrome
E. Incontinentia pigmenti

►A

Piebaldism is linked to a defect in the c-kit protooncogene. Hypomelanosis of Ito has whorled
hypopigmentation, occasional CNS defects, scoliosis and anodontia, a sporadic mutation.
Waardenburg syndrome has 4 types, 1-3 are AD, 4 is AR. Type 1 & 3 have defects in PAX-3, 2
in MITF, and 4 in Sox10, endothelin-3 ligand or receptor genes. Hermansky-Pudlak syndrome is
AR and most commonly linked to defects in HPS, a lysosomal transport protein and AP3B1, a
protein important in endocytic/exocytic sorting. Incontinentia pigmenti is an X-linked dominant
syndrome with a defect in the NEMO gene.

207 -Exposure to what medicine in-utero has been most closely associated with aplasia cutis

298
congenita?
A. Methimazole
B. Levothyroxine
C. Magnesium
D. Isotretinoin
E. Trimethoprim/sulfamethoxazole

►A

Aplasia cutis congenita presents as well-demarcated erosions at birth with heal with atrophic,
alopecic scars. It can be inherited in autosomal dominant, autosomal recessive, or sporadic
forms. Aplasia cutis congenia can be caused by teratogens, particular methimazole.

208 -Nevoid basal carcinoma syndrome (Gorlin syndrome) is autosomal dominant transmitted
mutation of the patched gene. Symptoms include innumberable basal cell carcinomas, painful
odontogenic jaw keratocysts, palmoplantar pits, frontal bossing, bifid ribs and what other bony
abnormality?

A. Polyostotic fibrous dysplasia


B. Stippled epiphyses
C. Calcification of falx cerebri
D. Osteopoikilosis
E. Sphenoid wing dysplasia

►C

Calcification of falx cerebri is seen in Gorlin's syndrome. CHILD syndrome and chondrodysplasia
punctata both can exhibit stippled epiphyses. Polyostotic fibrous dysplasia is found in McCune-
Albright syndrome, osteopoikilosis in seen in Buschke-Ollendorf syndrome. Sphenoid wing
dysplasia is seen in neurofibromatosis type I.

209 -A neonate has a papulopustular rash that progresses into a impetiginized eczematous dermatitis
and abnormally high IgE level. Patients with this sydnrome can have an IgE that is as:

299
A. 10x the normal value
B. 5x the normal value
C. 2x the normal value
D. 3x the normal value
E. 15x the normal value

►A

This patient has Jobs syndrome. It is characterized with papulopustular rash that progresses to
chronic impetiginized eczematous dermatitis and IgE levels that are 10x the normal level. It is also
highly suggestive of autosomal dominant hyper IgE syndrome.

210- Retention of primary teeth a dental finding of which of the following conditions?

A. Hypomelanosis of Ito
B. Letterer-Siwe disease
C. Tuberous sclerosis
D. Jackson Sertoli syndrome
E. Hyper-IgE syndrome

►E

Hyper-Immunoglobulin E syndrome is an autosomal dominant condition with impaired regulation


of IgE function and deficient neutrophil chemotaxis. There is increased susceptibilty to infections
and increased IgE serum levels. Retained primary teeth and lack of development of secondary teeth
are characteristic findings. The remaining conditions do not have this as a prominent finding.

211- Ichthyosis bullosa of Siemens is a condition characterized by fragile blisters at birth,


hyperkeratotic plaques on elbows/knees later in life, and a gene mutation in:

A. Keratin 2e
B. Keratin 5/14
C. Keratin 6a/16
D. Keratin 1/10

300
E. None of these options are correct

►A

Keratin 2e is mutated in ichythosis bullosa of Siemens. Keratin 5 and 14 are defective in


epidermolysis bullosa simplex, 6a/16 in Pachyonychia congenital type I (JadassohnLewandowsky),
1/10 in epidermolytic hyperkeratosis and Unna-Thost PPK.

212 -A sporadic syndrome affecting transcriptional coactivator CREB-binding protein is:

A. Rubinstein-Taybi syndrome
B. Cornelia de Lange syndrome
C. Nonne-Milroy disease
D. Maffucci syndrome
E. Blue rubber bleb nevus syndrome

►A

Rubinstein-Taybi syndrome is caused by a sporadically transmitted defect in transcriptional


coactivator CREB-binding protein. This gene is responsible for encoding a nuclear protein which
acts as a co-activator of cAMP regulated gene expression. Findings of this syndrome include:
capillary malformation, short stature, broad thumbs, craniofacial abnormalities including beaked
nose, mental retardation, strabismus, congenital heart defects and cryptorchidism. The other listed
conditions are not related to this defect.

213- Which of the following is not a feature of Cockayne syndrome?

A. Dwarfism
B. Salt and pepper retina
C. Increased sister chromatid exchange
D. Increased risk of skin cancer
E. Poikiloderma

301
►D

Patients with Cockayne syndrome do not have an increased risk of skin cancer. All the other
findings are associated with Cockayne syndrome, as well as photosensitivity, deafness, cataracts,
and sunken facies. Increased chromatid exchange is also seen in Bloom syndrome, dyskeratosis
congenita, and Fanconi's anemia.

214- Xeroderma pigmentosum (XP) variant is different than classic XP in which of the following
way?

A. Defective DNA nucleotide excision repair of the global genome


B. Defective post-replication repair
C. Increased chromosomal breakage and sister chromatid exchanges
D. Defective DNA nucleotide excision repair of actively transcribing genes
E. Low IgM

►B

XP variant is DNA nucleotide excision repair proficient, but the defect is in post replication repair
of DNA. Increased chromosomal breakage and sister chromatid exchanges is found in Bloom‘s
syndrome, an autosomal recessive syndrome caused by a defect in BLM gene, whose product
functions as a helicase. Clinical findings include: Telangiectasias, short stature, malar erythema,
recurrent infection, increased frequency of leukemia and lymphoma, normal intelligence. Defective
DNA nucleotide excision repair of actively transcribing genes is a feature of Cockayne‘s syndrome,
an autosomal recessive syndrome with clinical findings including: Cachexia, short stature,
pigmentary retinal degeneration, progressive deafness and no increase in neoplasms. Xeroderma
pigmentosum has seven different complementation groups (A-G), each associated with a different
form of impairment of DNA nucleotide excision repair.

215 -A 13 year old girl has woolly hair, keratoderma of the soles, edema of the lower extremities,
and swelling of the abdomen. A cardiac evaluation reveals right-sided cardiomyopathy. What is the
most likely defective protein?

A. Plakoglobin
B. Plectin

302
C. Desmocollin
D. Desmoglein
E. Desmoplakin

►A

Naxos disease is caused by a defect in the plakoglobin protein. It is characterized by woolly hair,
keratoderma, and right-sided cardiomyopathy.

216- The development of which malignancy is most commonly associated with lymphomatoid
papulosis?

A. Non-Hodgkin's lymphoma
B. Mycosis fungoides
C. Multiple myeloma
D. Immunoblastic lymphoma
E. Waldenstrom's macroglobulinemia

►B

Lymphomatoid papulosis is a recurrant eruption of unclear etiology characterized by the appearance


of red-brown papules and nodules which spontaneously disappear in 3 to 8 weeks. It is notable for
histologic features which suggest a CD30 positive malignant lymphoma. There is controversy
regarding whether lymphomatoid papulosis (LyP) is a malignant, premalignant or benign condition.
The disease may last from months to years and in up to 20% of patients, it may be preceded by,
followed by, or associated with another type of cutaneous malignancy. Generally, this is mycosis
fungoides, a CD30-positive large T-cell lymphoma or Hodgkin's disease. Because of this potential
risk, long-term follow-up of these patients is required.

217- Which of the following syndromes is characterized by follicular atrophoderma, hypohidrosis,


hypotrichosis and multiple basal cell carcinomas?

A. Bazex-Christol-Dupre syndrome
B. Rombo syndrome

303
C. Rasmusen syndrome
D. Gorlin syndrome
E. Incontinentia Pigmenti

►A

Bazex-Christol-Dupre syndrome has the findings of follicular atrophoderma, hypohidrosis,


hypotrichosis and multiple basal cell carcinomas (BCC). Rombo syndrome is associated with BCC
and hypotrichosis, but not the other listed findings. The atrophoderma in Rombo syndrome is
vermicular, not follicular. Rasmusen syndrome is not associated with BCC.

218 -A patient has epidermal cysts, demoid tumor, fibromas, lipomas and dental cysts. The patient
is diagnosed with an autosomal dominant syndrome with the characteristic eye finding of congenital
hypertrophy of retinal epithelium (CHRPE). This person will develop adenomatous polyposis of the
colon that will transform to carcinoma by the age of 30:

A. 100% of the time


B. 90% of the time
C. 80% of the time
D. 70% of the time
E. 50% of the time

►A

This patient has Gardner's syndrome with a mutation in the tumor suppressor APC gene. These
patients with develop adenenomatous polyposis of the colon and rectum with 100% transformation
to carcinoma by the age of 30.

219- The x-linked recessive type of dyskeratosis congenita is:

A. Dyskerin
B. TERC
C. CDKN2A
D. PTEN

304
E. Menin

►A

The dyskerin gene, whose product is involved in ribosomal RNA synthesis, is mutated in Xlinked
recessive dyskeratosis congenita. TERC is linked with autosomal domininant transmission of the
syndrome. CDKN2A is involved in familial dysplastic nevi/melanoma syndrome, PTEN in Cowden
syndrome and Menin in MEN type I.

220 -The gene defect in Griscelli Syndrome is:

A. Myosin Va or Rab27a
B. LYST or CHS1
C. P gene
D. TRP1
E. None of these answers are correct

►A

Myosin Va or Rab27a are defective in Griscelli syndrome, an AR syndrome with mild albinism,
pancytopenia, immunodeficiency, neurologic symptoms and an accelerated phase similar to
Chediak-Higashi syndrome. LYST/CHS1 is defective in Chediak-Higashi syndrome. The P-gene is
mutated in oculocutaneous albinism type 2 and TRP in oculocutaneous albinism type 3.

221 -Comma shaped corneal opacities are seen in what disease?

A. Refsum Syndrome
B. Sjogren-Larson Syndrome
C. Pseudoxanthoma elasticum
D. X-linked ichthyosis
E. Proteus syndrome

►D

305
X-linked ichthyosis is a X-linked recessive disorder secondary to steroid sulfatase deficiency
characterized by brown adherent scale. Additional findings include comma-shaped corneal
opacities, cryptorchisdism, and failure to progress during labor.

222 -The NEMO gene is defective in Bloch-Sulzberger syndrome. What other syndrome has been
linked with defects in the NEMO gene?

A. Hypohidrotic ectodermal dysplasia with immune deficiency


B. Hypomelanosis of Ito
C. Tuberous sclerosis
D. Waardenburg syndrome
E. Piebaldism

►A

Hypohidrotic ectodermal dysplasia with immune deficiency, is caused by mutations in the


NEMO (IKK-gamma gene). As opposed to the X-linked dominant inheritance of Bloch- Sulzberger
syndrome (incontinentia pigmenti), this is a X-linked recessive disorder.
Hypomelanosis of Ito is sporadically inherited and is not linked with a gene defect. Tuberous
sclerosis is autosomal dominant and has been linked to defects in tuberin and hamartin tumor
suppressor genes. Waardenburg syndrome has four subtypes, linked with the PAX-3, MITF and
SOX10/endothelin-3 receptor genes. Piebaldism is linked to defects in the c-kit protooncogene.

223 -Deficiency of filaggrin is the strongest known predisposing genetic factor for the
development of atopic dermatitis. What is the mode of inheritance of mutations in filaggrin?

A. Autosomal dominant
B. Autosomal semidominant
C. Autosomal recessive
D. X-linked dominant
E. X-linked recessive

►B

306
Autosomal dominant means that a mutation in a gene on one of the two chromosomes in any
autosomal chromosome pair results in disease. Autosomal recessive means that mutations in the
same gene on both of the chromosomes in the pair must occur for the disease to occur.
Autosomal semidominant means that a mutation in one gene of the autosomal chromosome pair
results in a mild version of the disease, while mutations in the gene on both chromosomes results
in the full-blown disease. Ichthyosis vulgaris is caused by mutations in filaggrin and inherited in
a autosomal semidominant manner. This is the genetic explanation for why the phenotype of
ichthyosis vulgaris can vary significantly across individuals.

224 -Dystrophic epidermolysis bullosa is associated with mutations in collagen VII. Trauma or
friction induced blistering in these patients have a plane a splitting in the:

A. Sublamina densa
B. Stratum spinosum
C. Lamina lucida
D. Stratum basale
E. None of these answers are correct

►A

The split in dystrophic epidermolysis bullosa is found in the sublamina densa, where the collagen
VII anchors the epidermis to the anchoring plaques in the dermis. The remaining options are
incorrect.

225 - Connexin 30 (GJB6 gene) is defective in which of the following syndromes?

A. KID syndrome
B. Vohwinkel syndrome
C. Vohwinkel syndrome variant
D. Clouston syndrome
E. Erythrokeratoderma variabilis

►D

307
Clouston syndrome is associated with a defect in Connexin 30 (GJB6 gene). Findings include
palmoplantar keratoderma with transgradiens, dystrophic nails, sparse hair with absent body,
eyelash, eyebrow hair after puberty. KID syndrome and Vohwinkel syndrome are associated
with a defect in Connexin 26 (GJB2). Vohwinkel syndrome variant is associated with a loricrin
defect. Erythrokeratoderma variabilis has mutations in Connexin 31 (GJB3) and 30.3(GJB4).

226 -Adenosine deaminase deficiency is seen in which immunodeficient disease?

A. Wiskott-Aldrich syndrome
B. Chronic granulomatous disease
C. Job syndrome
D. Severe combined immunodeficiency syndrome
E. Leineri's disease

►D

Severe combined immunodeficiency is a heterogeneous group of disorders characterized by


decreased humoral and cell mediated immunity. Patients may have recurrent infections including
cutaneous ones, GVHD (due to in utero cmaternal lymphocytes), sepsis, oral candidiasis, and
diarrhea. Implicated genes include the IL-2 receptor (x-linked recessive form) and adenosine
deaminase deficiency (autosomal recessive form).

227 -A patient has recurrent infections and is found to have a disorder of phagocytic cells from an
inability of phagocytes to undergo the respiratory burst needed to kill certain types of bacter ia
and fungi. Carriers of this disease are also at risk for which condition?

A. Breast cancer
B. Vitiligo
C. Discoid lupus
D. Thyroid disease
E. Cataracts

►C

308
This patient has chronic granulomatous disease characterized by dysfunctional phagocytic cells.
The most common molecular defect in chronic granulomatous disease is a mutation in the gene
encoding the b subunit of cytochrome b. Carriers have a significant incidence of discoid lupus
erythematosus, photosensitivity, Raynaud's phenomenon, and aphthous ulcers. Carriers of ataxia
telangiectasia are at risk for breast cancer.

228- In patients with diffuse congenital hemangiomatosis, the most common site for extracutaneous
involvement is the :

A. Liver
B. Thyroid
C. Lungs
D. Colon
E. Brain

►A

Diffuse congenital hemangiomatosis is characterized by multiple hemangiomas with the liver being
the most common extracutaneous site, followed by the lungs. Liver hemangioma may be
complicated by hepatomegaly, obstructive jaundice, and portal hypertension.

229- A 5 year old boy is being seen by neurology for seizures and by ENT for a hoarse voice. He is
referred to dermatology for evaluation of verrucous nodules of the elbows and knees. Wh ile
reviewing his chart, it is noted that he has calcifications in the hippocampus seen on imaging. What
is the most likely diagnosis of this patient?

A. Lipoid proteinosis
B. Psoriasis
C. Severe combined immunodeficiency disorder
D. Poland syndrome
E. Vohwinkel syndrome

►A

309
Lipoid proteinosis is an autosomal recessive disorder caused by defects in the extracellular matrix
protein 1 gene. It is characterized by scars and yellow papules of the face and oropharynx,
yellowish papules on the eyelid margin, hoarse voice, verrucous nodules of the elbows and knees,
and calcification of the temporal lobe and hippocampus with occasional seizures.

230- A 50 year man presents with generalized metallic-grey hyperpigmentation. His past medical
history includes diabetes, hepatomegaly and arrythmias. Laboratory tests should include:

A. Copper levels
B. Lead levels
C. Iron levels
D. Arsenic levels
E. Cyanide levels

►C

Hemochromatosis is an autosomal recessive disease resulting in increased intestinal iron absorption


and iron deposition in a variety of organs. Clinical features include generalized metallic-grey
hyperpigmentation, koilonychia, sparse or absent hair, hepatomegaly, cardiac failure/arrhythmias,
insulin-dependent diabetes, hypogonadism and polyarthritis

231- A 4 year-old boy presents with 2 soft, dark-blue, compressible nodules on his extremities. His
mother has noted that these lesions have increased sweating and that they were present at birth. No
one else in the family has had similar skin lesions. What step is indicated first to help determine the
diagnosis?

A. Stool guiac
B. MRI of the abdomen
C. CBC
D. Immediate referral to a gastroenterologist
E. Biopsy of a skin lesion

►A

310
Blue rubber bleb nevus syndrome is described above. There are multiple venous malformations on
the extremities and trunk, often present at birth to early childhood. The number of these lesions
increase with age. The lesions may have increased sweating and can be combined with lymphatic-
venous malformations. Skin lesions can be a clue to gastrointestinal venous malformations which
can lead to secondary bleeding and anemia. The most reasonable screening test to determine if the
patient has GI hemorrhage is a stool guiac. An MRI or complete blood count can be helpful, but are
not the best test to start with. A skin biopsy is not indicated. If there is GI blood loss, evaluation by
a gastroenterologist is useful.

232 - Patients with neurofibroma type I have a pheochromocytoma incidence of:

A. 1%
B. 5%
C. 7%
D. 9%
E. 10%

►A

Patients that have NF1 have a 1% incidence of pheochromocytoma. Other syndromes with flushing
and pheo like symptoms are von Hippel-Lindau and MEN2.

233- What is the gene defect in harlequin fetus?

A. Transglutaminase
B. Steroid sulfatse
C. ABCA12
D. ABCC6
E. None of these answers are correct

►C

Harlequin fetus is an autosomal recessive disorder. The gene defect is ABCA12.

311
234 -A patient with a white, spongy overgrowth of the buccal mucosa that has passed in an
autosomal dominant fashion is most likely related to a mutation in which of the following?

A. Keratin 1/10
B. Keratin 6b/17
C. Keratin 4/13
D. Keratin 6a/16
E. None of these options are correct

►C

This description is most likely a white sponge nevus, an autosomal dominant defect in keratin 4/13.
Keratin 1/10 is mutated in epidermolytic hyperkeratosis and Unna-Thost PPK, Keratin 6a/16 in
pachyonychia congenita type I, and keratin 6b/17 in pachyonychia congenita type II.

235 -A middle-aged woman complains of multiple rough lesions on her trunk. Biopsy reveals
connective tissue nevi. Because other family members have had similar lesions, genetic testing is
performed, revealing a defect in the LEMD3 gene. What is the classic bone finding in patients with
a defective LEMD3 gene?

A. Osteopoikilosis
B. Osteopathia striata
C. Tibial bowing
D. Lumbar ankylosis
E. Frequent fractures of the long bones

►A

Buschke Ollendorff syndrome is a rare genetic syndrome characterized by multiple connective


tissue nevi and osteopoikilosis. Osteopoikilosis is dysplasia of bone leading to the presence of
multiple bone islands in the skeleton. The defective gene is LEMD3. Osteopathia striata is
associated with focal dermal hypoplasia. Tibial bowing is seen in rickets. Lumbar ankylosis is
found in ankylosing spondylitis. Frequent fractures of long bones is seen in osteogenesis
imperfecta.

312
236- Naxos syndrome is characterized by a right sided cardiomyopathy, wooly hair, and
keratoderma. The epidermal structure defective in Naxos syndrome is:

A. Desmoglein 1
B. Desmoglein 3
C. Plakoglobin
D. Desmoplakin
E. Keratin 1/10

►C

Plakoglobin is an intracellular desmosomal component which binds desmogleins/desmocollins on


one side and to desmoplakin on the other. Desmoplakin in turn binds to the keratin intermediate
filaments, K1/10 in most cases. Mutation of desmoplakin leads to CarvajaL syndrome, which is
associated with a striate palmoplantar keratoderma, woolly hair and Left sided cardiomyopathy. A
simple way to remember this is the L in Carvajal cooresponds to the Left sided cardiomyopathy vs.
the right sided disease in Naxos disease.

237 -Which of the following pairs of diseases are caused by mutations in DNA helicases?

A. Bloom syndrome and Rothmund-Thompson syndrome


B. Xeroderma pigmentosum and Cockayne syndrome
C. Muir-Torre syndrome and Birt-Hogg-Dube syndrome
D. Dyskeratosis congenita and Peutz-Jeghers syndrome
E. Cowden syndrome and Bannayan-Riley-Ruvalcaba syndrome

►A

Bloom Syndrome and Rothmund-Thompson syndromes are caused by autosomal recessive


mutations in DNA helicases. The common features include photodistributed poikiloderma.
Patients with Bloom syndrome also demonstrate facial dysmorphism, hypogammaglobulinemia
with recurrent respiratory and gastrointestinal tract infections, hypogonadism, leukemias and
lymphomas, gastrointestinal adenocarcinomas, and oral/esophageal SCCs. Patients with
Rothmund-Thompson syndrome develop premalignant acral keratoses, alopecia, nail dystrophy,
cataracts, hypogonadism, and occasional solid tumors.

313
238 -Fabrys disease is a defect in the alpha galactosidase. The eye finding that looks like whorled
corneal deposits is:

A. Cornea verticillata
B. Chemosis
C. Lagophthalmos
D. Corneal ulceration
E. Conjunctival shrinkage

►A

Fabrys disease is an X linked recessive disorder. Patients can also get cornea verticillata that look
like whorled corneal deposits. They can also get spoke like lens deposits, conjunctival and retinal
tortuosity, oculomotor abnormalities.

239 -A Puerto Rican woman is seen in clinic for a pruritic rash on her trunk. A punch biopsy is
performed. The biopsy site continues to bleed, with hematoma formation. The bleeding is
eventually controlled. On further exam, her skin and hair are light brown. She has a history of
granulomatous colitis. What it the most likely reason she had excess bleeding with a simple
procedure?

A. Her platelets lack dense bodies, causing excess bleeding


B. Her intrinsic factor is deficient
C. Her Factor VIII levels are low
D. She is congentially deficient in platelets
E. None of the answers are correct

►A

Platelets without dense bodies cause excess bleeding in Hermansky-Pudlak syndrome. Other
features of this condition include oculocutaneous albinism, ceroid lysosomal storage disease
resulting in pulmonary fibrosis, granulomatous colitis.

314
240 -A patient that has Darier's disease has skin findings of hyperkeratotic papules in the
seborrhei careas and has nail findings with:

A. Alternating red and white longitudinal bands


B. Long nails
C. Horizontal parallel ridges
D. Longitudinal fissures
E. Corn

►A

Patient with Darier's disease have alternating red and whit longitudinal bands with a V-shaped
distal nicking, subungual hyperkeratosis and skin findings in the seborrheic areas.

241 -Which of the following conditions is inherited in an X-linked recessive manner?

A. Epidermolysis bullosa simplex


B. Ichthyosis vulgaris
C. Sjogren-Larsson syndrome
D. Wiskott-Aldrich Syndrome
E. Netherton‘s Syndrome

►D

Wiskott-Aldrich Syndrome is inherited in an X-linked recessive manner. Epidermolysis bullosa


simplex and ichthyosis vulgaris are inherited in an autosomal dominant (AD) manner. Sjogren-
Larsson and Netherton‘s syndrome are inherited in an autosomal recessive manner.

242 - Which genetic disease is characterized by round opacities seen on radiographs of the long
bones?

A. Buschke-Olendorf syndrome
B. Chondrodysplasia punctata
C. Netherton syndrome

315
D. Osteogenesis imperfecta
E. Focal dermal hypoplasia

►A

Buschke-Ollendorf syndrome is an autosomal dominant disorder with dermatofibrosis lenticularis


disseminata (elastomas) and osteopoikilosis, or round opacities of the bones. It is caused by a defect
in LEMD 3 (MAN1), which codes an inner nuclear membrane protein. Focal dermal hypoplasia can
cause osteopathia striata; osteogenesis imperfecta can have bone fractures; and chondrodysplasia
punctata can have stippled epiphyses.

243 -Palmoplantar keratoderma with deafness is caused by a defect in which gene?

A. SLURP-1
B. Plakophilin
C. Connexin 26
D. Lysosomal papain like cysteine proteinase
E. Unknown

►C

Palmoplantar keratoderma with deafness is caused by a defect connexin 26 encoded by GJB2. A


defect in SLURP-1 causes Mal de Meleda. A defect in plakophilin causes ectodermal dysplasia with
skin fragility. A defect in cathepsin C lysososomal papain like cysteine proteinase causes Papillon
LeFevre and Haim Munk.

244- Which disease can clinically mimic pellagra but is inherited in an autosomal recessive fashion
and is due to a defect in the transport of neutral amino acids?

A. Wilsons
B. Hemochromatosis
C. Hartnup Disease
D. Fabry
E. Gaucherǁs

316
►C

The clinical manifestation of Hartnup disease is similar to that of pellagra because the resultant
defect in the transport of amino acids leads to low levels of tryptophan. Since tryptophan is required
to make nicotinic acid, pts with Hartnup disease manifest the same symptoms as niacindeficient
patients (pellagra).

245 -Patients with X-linked hypohidrotic ectodermal dysplasia have dental findings of:

A. Pegshaped/conical teeth
B. Erythrodontia
C. Hutchinson teeth
D. Enamel hypoplasia
E. Mulberry molars

►A

Patients with X-linked hypohidrotic ectodermal dysplasi have pegshaped/conical teeth. They have
problems with perspiration and thickened nails.

246 - A patient Buschke-Ollendorff syndrome has osteopoikilosis and which cutaneous finding?

A. Waxy papules along the eyelids


B. Cafe au lait macules
C. Port wine stain
D. Juvenile elastoma

317
E. Epidermal nevi

►D

Buschke-Ollendorf syndrome is an autosomal dominant syndrome associated with increased elastic


fiber in the skin. Key features include dermatofibrosis lenticularis disseminata (also called juvenile
elastomas) and osteopoikilosis.

247- The diagnostic test for chronic granulomatous disease is:

A. Dimethylglyoxime test
B. Nitroblue tetrazolium reduction assay
C. Histamine skin test
D. Bone marrow biopsy
E. Potassium hydroxide

►B

Chronic granulomatous disease is characterized by a defect in the ability to kill catalase positive
organisms within phagocytic leukocytes. This results from a neutrophilic defect in the cytochrome
found in the NADPH oxidative pathway responsible for a respiratory burst. The nitroblue
tetrazolium (NBT) reduction assay demonstrates the leukocytes ability to reduce the dye and
produce a blue color change. Patients with chronic granulomatous disease are unable to reduce the
dye.

248- Homocystinuria is caused by a defect in:

A. Phenylalanine hydroxylase
B. Biotinidase
C. Holocarboxylase synthetase
D. Cystathione beta-synthetase
E. Gp91-phox

318
►D

Cystathione beta-synthetase is defective in homocystinuria, an autosomal recessive conditions


characterized by increased homocystine and methionine levels in blood and urine. Other findings
include a malar flush, DVTs/emboli, cardiovascular disease, livedo reticularis, leg ulcers, blonde
hair/fair complexion, downward lens dislocation, glaucoma, mental retardation, seizures,
psychiatric disorders and a marfanoid body habitus. The other enzymes are not involved in this
condition.

249- A patient presents with focal symmetric palmoplantar keratoderma, thickened, hyperkeratotic
fingernails and toenails with a "pincer" appearance and frequent staph and candida paronychial
infections, follicular hyperkeratosis of the knees and elbows and oral leukokeratosis. The patients
mother and grandfather both have similar skin findings. Which syndrome is described?

A. Jadassohn-Lewandowsky syndrome
B. Jackson-Lawler syndrome
C. Schafer-Branauer syndrome
D. Pachyonychia congenita tarda
E. None of the options are correct

►A

The description above is the classic description for pachyonychia congenita (PC) type I or
Jadassohn-Lewandowsky syndrome. Jackson-Lawler syndrome is PC type II, Schafer-Branauer
syndrome is PC type III and pachyonychia congenita tarda is PC type IV.

250 -A child presents with sparse, short hair and sensorineural deafness. On microscopic
examination of the hair, pili torti is noted. Which of the following syndromes is the most likely
diagnosis?

A. Bjornstad syndrome
B. Menkes kinky hair syndrome
C. Argininosuccinic aciduria
D. Trichothiodystrophy
E. None of the options are correct

319
►A

Bjornstad syndrome is the most likely diagnosis. This rare syndrome (~25 cases) is autosomal
recessive. Findings are of deafness and pili torti. The most common hair finding in Menkes
syndrome is pili torti, but it is not associated with hearing loss. Argininosuccinic aciduria is
associated with trichorrhexis nodosa and has no associated hearing loss.

251- A patient with Crowe‘s sign and an optic glioma has which of the following disorders?

A. Neurofibromatosis I
B. Neurofibromatosis II
C. Watson syndrome
D. Tuberous sclerosis
E. Lester iris syndrome

►A

The diagnostic criteria for neurofibromatosis I include meeting 2 or more of the following 7 criteria:
(1) >5 café au lait macules (CALMs) that are >5mm in a prepubertal person or >15 mm in a
postpubertal person, (2) >1 neurofibroma or 1 plexiform neurofibroma, (3) axillary/inguinal
freckling (Crowe‘s sign), (4) optic glioma, (5) >1 Lisch nodule (iris hamartoma), (6) sphenoid
dysplasia, (7) 1st degree relative with neurofibromatosis I.

252- Which of the following features is not associated with Cornelia de Lange Syndrome?

A. Normal intelligence
B. Characteristic facies with downturned mouth, hirsutism, trichomegaly, synophrys,
anteverted nostrils, long philtrum and low set ears
C. Cryptorchidism
D. Fifth finger clinodactyly
E. Recurrent lung infections

►A

320
Children with Cornelia de Lange are usually severly retarded with an IQ <35. In addition to the
features listed above, other features include cutis marmorata, hypoplastic nipples and umbilicus,
low-pitched cry in infancy and congenital heart defects. While most cases are inherited in a sporadic
manner, those cases which are familial are thought to be autosomal dominant and associated with
the NIPBL (nipped-beta-like) gene. Prognosis is poor with premature death often secondary to
sspiration or recurrent pulmonary infection.

253 -Which of the following is caused by a mutation in a gene that leads to defective NF -KB
activation?

A. MEN IIa
B. Piebaldism
C. Chediak-Higashi
D. Vohwinkel‘s
E. Incontinentia pigmenti

►E

Incontinential pigmenti is an X-linked dominant disorder caused by a mutation in the NEMO


gene located at Xq28. NEMO is an NF-KB modulator. A genetic defect in NEMO leads to
defective activation of NF-KB (a transcription factor). MEN IIa is caused by a mutation in the
RET proto-oncogene, piebaldism is caused by the C-kit protocogene, chediak-higashi is caused
by a mutation in lysosomal tracking, and Vohwinkel‘s is caused by a defect in GJB2 which
encodes connexin 26.

254 -Crumpled ears are associated with which disorder?

A. Ehlers-Danlos Syndrome
B. Marfan syndrome
C. Congenital contractural arachnodactyly
D. Cutis laxa
E. Buschke-Ollendorf syndrome

►C

321
Congenital contractural arachnodactyly is an autosomal dominant disorder caused by mutationsin
fibrillin 2. Affected patients have long limbs, arachnodactyly, scoliosis, and crumpled ears.

255 -What gene defect would you expect to find in a child with white forelock, dystopia canthorum,
and upper limb abnormalities?

A. Pax3
B. MITF
C. SOX10
D. Endothelin-3
E. C-kit proto-oncogene

►A

Waardenburg's syndrome is characterized by depigmented patches, white forelock, and deafness.


Both type 1 and 3 are caused by mutations in Pax3. Type 3 is also associated with limb
abnormalities. MITF and SOX10 defects are responsible for types 2 and 4 respectively. C-kit proto-
oncogene mutatios are seen in piebaldism.

256 -The prenatal diagnosis of X-linked ichthyosis is detected by the triple screen during pregnancy
which can show:

A. Low to absent estradiol levels


B. Non hydrolyzed sulfated steroids
C. Low to absent levels of hydrolyzed steroids
D. Low HCG levels
E. Increased serum cholesterol sulfate levels

►A

Prenatal diagnosis of X-linked ichthyosis is by low to absent estriol during triple screen,
presence of non hydrolyzed sulfated steroids in maternal urine. Postnatal diagnosis by serum
protein electrophoresis shows increased serum cholesterol sulfate levels.

322
257 -Which of the following hereditary skin disorders is associated with the RAS-ERK-MAPK
pathway?

A. Costello syndrome
B. Rothmund-Thompson Syndrome
C. Carney complex
D. Tuberous Sclerosis
E. Griscelli syndrome

►A

Skin disorders associated with the RAS-ERK-MAPK pathway include: Cardio-facio-cutaneous


syndrome, Costello, LEOPARD, NF, and Noonan. Rothmund-Thompson is a RecQ DNA helicase
defect. Carney complex and Tuberous sclerosis are associated with the cAMP & AMP activated
protein kinase pathway. Griscelli syndrome involves defective vesicle trafficking/transport.

258 -Which of the following condition is NOT found in Von-Hippel Lindau syndrome?

A. Connective tissue nevi


B. Bilateral retinal hemangioblastomas
C. Cerebellar/CNS hemangioblastomas
D. Renal cell carcinoma
E. Pheochromocytoma

►A

Von Hippel-Lindau syndrome is characterized by all the options listed except connective tissue
nevi. Other findings include pancreatic cysts/carcinoma and cutaneous capillary malformations of
the head and neck and polycythemia.

259- The combination of painful palmoplantar keratoderma and pseudoherpetic keratitis is


characteristic of which of the following syndromes?

A. Naxos syndrome

323
B. Vohwinkel syndrome
C. Richner-Hanhart syndrome
D. Howel-Evans syndrome
E. Schopf-Schulz-Passarge syndrome

►C

Richner-Hanhart syndrome (tyrosenemia type II) is an autosomal recessive disorder caused by a


deficiency in hepatic tyrosine aminotransferase. This disease is characterized by painful PPK,
pseudoherpetic keratitis and blindness. Treatment is low-tyrosine/phenylalanine diet.

260- A child presenting with the scalp findings shown and a right arm hypoplasia would be
diagnosed with which of the following syndromes?

A. Adams-Oliver syndrome
B. Bart's syndrome
C. Progeria
D. Dunnigan syndrome
E. None of these options are correct

►A

Adams-Oliver syndrome is defined by aplasia cutis congenita (ACC) (shown in the image), usually
of the midline scalp with limb hypoplasia. Bart's syndrome also has ACC as a finding, but it is
usually present on the lower extremities and associated dominant dystrophic epidermolysis bullosa.
Progeria is a premature aging syndrome associated with a lamin -A mutation. Dunnigan syndrome
is also known as Familial partial lipodystrophy and is associated with a mutation in the laminin A/
C. Neither are associated with findings of ACC.

261- Ichthyosis linearis circumflexa is one of the findings seen in the syndrome caused by which of
the following genes?

A. SPINK5
B. SLURP1

324
C. GJB2
D. LMX1B
E. CYLD

►A

The SPINK5 gene encodes for LEKTI, a serine protease inhibitor important in the regulation of
proteolysis in epithelia formation and keratinocyte terminal differentiation, is mutated in
Netherton‘s Syndrome. Other findings include: trichorrhexis invaginata (bamboo, ball and socket
hair), atopic dermatitis, and anaphylaxis from food allergy. SLURP1 is mutated in Mal de Maleda,
GJB2 in Vohwinkel‘s syndrome, LMX1B in Nail-Patella syndrome, and CYLD in Familial
Cylindromiasis.

262- What is the characteristic radiographic finding in type I Gaucher disease?

A. Enchondromas
B. Osteopoikilosis
C. Melorheostosis
D. Ehrlenmeyer flask deformity
E. Supernumerary vertebrae with extra ribs

►D

The Ehrlenmeyer flask deformity is found in the femoral midshaft as well as aseptic necrosis of the
femoral head and widening of the distal femur. Endochondromas are seen in Maffucci syndrome,
Osteopoikilosis in Buschke-Ollendorf syndrome, Melorheostosis (linear hyperostosis under affected
skin) in linear scleroderma and supernumerary vertebrae with extra ribs in incontinentia pigmenti.

263 - A 18 yo man presents for evaluation of foot lesions. There are thick hyperkeratotic plaques
symmetrically on only the weight bearing plantar surfaces. What test(s) should this patient be
referred for?

A. Head CT
B. Thoracic CT

325
C. Hepatic ultrasound
D. Endoscopy
E. Knee films

►D

The patient likely has Howel-Evans Syndrome. These patients present with symmetric focal weight
bearing PPK in the second decade to adulthood. After the third decade, esophageal carcinoma can
occur. These patients should have periodic endoscopic evaluation.

264- Mutations in which of the following receptors underlie chronic mucocutaneous candidiasis
disease (CMCD)?

A. IL-12 receptor
B. IL-15 receptor
C. AIRE receptor
D. IL-17 receptor
E. IL-23 receptor

►D

Chronic mucocutaneous candidiasis disease (CMCD) is characterized by recurrent or persistent


infections of the skin, nails, and oral and genital mucosae caused by Candida albicans and, to a
lesser extent, Staphylococcus aureus, in patients with no other infectious or autoimmune
manifestations. Mutations in IL-17 receptor A (IL-17RA, autosomal recessive) and IL-17 receptor F
(IL-17F, autosomal dominant) have been reported. IL-17RA deficiency is complete, abolishing
cellular responses to IL-17A and IL-17F homo- and heterodimers. By contrast, IL17F deficiency is
partial, with mutant IL-17F-containing homo- and heterodimers displaying impaired, but not
abolished, activity.

265- An infant presents with multiple congenital hemangomas in an generalized distribution. What
is the most serious associated condition?

A. Congestive Heart Failure

326
B. Obstructive jaundice
C. Portal hypertension
D. All of the answers are correct
E. None of the answers are correct

►A

High output congestive heart failure can lead to death in these children. Obstructive jaundice and
portal hypertension both occur, but are less likely to cause death. The hemangiomas will undergo
spontaneous regression.

266- What is the genetic defect of this autosomal dominant disorder?

A. PTEN
B. PTPN11
C. Calcium ATPase 2A2
D. Calcium ATPase 2C1
E. SPINK5

►D

Hailey-Hailey is an autosomal dominant disorder that usually affects the intertriginous areas.
Clinically, there is erythema and linear fissures of the axilla and groin. On pathology, the
characteristic finding is acantholytic dyskeratosis in a "dilapidated brick wall" pattern. The gene
defect responsible is calcium ATPase 2C1.

267- A patient with a personal and family history of multiple fibrofolliculomas may have:

A. Birt-Hogg-Dube syndrome
B. Basal cell nevus syndrome
C. Muir-Torre syndrome
D. Gardner's syndrome
E. Tuberous sclerosis

327
►A

Fibrofolliculomas are small, benign, yellow or flesh colored papules that are usually inherited in an
autosomal dominant fashion and have a predilection for the face, neck and upper trunk. Birt Hogg-
Dube syndrome is an autosomal dominant condition characterized by multiple fibrofolliculomas,
trichodiscomas, acrochordons, collagenomas. This condition is important to recognize due to its
association with renal cell carcinoma. In addition, lung cysts and bullous emphysema are also
features of the condition with spontaneous pneumothorax being a potential complication. Muir-
Torre syndrome is an autosomal dominant condition caused by a defect in hMSH2 gene. This
condition is associated with sebaceous neoplasms including sebaceous carcinoma, sebaceous
hyperplasia, sebaceous epithelioma, sebaceous adenoma as well as keratoacanthomas. These can be
markers for underlying malignancy in this condition; notably adenocarcinoma of the colon, breast,
urinary tract, lung and endometrium. Gardner syndrome is an autosomal dominant condition caused
by a defect in the APC familial adenomatous polyposis gene in which patients have multiple
hamartomatous polyps of the colon with a high rate of malignant transformation. Cutaneous clues to
the diagnosis include multiple epidermoid cysts, fibromas, and desmoid tumors. Other
manifestations include osteomas, supernumary teeth, and congential hypertrophy of the retinal
pigment epithelium. Tuberous sclerosis, (TS),also known as Bourneville's syndrome is caused by
defects in hamartin and tuberin, found on chromosome 9 and 16, respectively. Patients with TS are
at increased risk of muliple neoplasms including retinal hamartomas, angiomyolipomas, and cardiac
rhabdomyomas. Cutaneous manifestations include ash-leaf macules, shagreen patchs, cafe-au-lait
macules, confetti macules, facial angiofibromas, and periungual fibromas. As the name implies,
basal cell nevus syndrome is associated with multiple basal cell carcinomas. In addition
palmoplantar pitting, multiple milia and epidermoid cysts are seen. It is autosomal dominantly
inherited and caused by a defect in the patched gene.

268- A patient with port wine stain on a lower extremity, hemihypertrophy of the limb and
lymphatic and deep venouse insufficiency of the affected limb would be considered to have
KlippelTrenaunay-Weber syndrome. What additional feature would need to be present to define the
patient as having Parkes-Weber syndrome?

A. Arteriovenous fistulas
B. Multiple cafe-au-lait macules
C. Macroglossia
D. Cutis marmorata
E. Distichiasis

328
►A

Parkes-Weber syndrome has the additional feature of arteriovenous fistulas. The remaining features
are not part of these syndromes.

269- A 20-year old male with a history of pheochromocytoma and medullary thyroid cancer
presents with mucosal papules. His overall body appearance is most likely to demonstrate:

A. Cushingoid features
B. Marfanoid features
C. Short stature
D. Lipodystrophy
E. Unilateral limb shortening

►B

This patient has multiple endocrine neoplasia type IIb (MEN IIb) characterized by
pheochromocytoma, thyroid cancer, and rare parathyroid carcinoma as well as mucosal neuromas
cutaneously. These patients have a marfanoid body habitus. This syndrome is caused by the RET
proto-oncogene.

270- Congenital erythropoetic porphyria has the dental findings of:

A. Erythrodontia
B. Hutchinson teeth
C. Mulberry molars
D. Hypodontia
E. Enamel hypoplasia

►A

Congenital erythropoetic porphyria has dental findings of erythrodontia. Hutchinson teeth and
mulberry molars are congenital syphilis. Hypodontia is a finding of X-linked hypohidrotic
ectodermal dysplasia.

329
271- In which of the following Genodermatoses would one find cutaneous hyperpigmentation, blue
lunulae and Kayser-Fleishcher rings:

A. Marfan's Disease
B. Hemochromatosis
C. Gaucher's Disease
D. Wilson's Disease
E. Osteogenesis Imperfecta

►D

In Wilson's disease (Hepatolenticular Degeneration) one will find a vague greenish discoloration of
the skin on the face, neck, and gentalia Hyperpigmentation), azure lunulae (sky-blue moons) of the
nails, and Kayser-Fleischer rings. This is due to the body retaining excessive amounts of copper.

272 -A child presents with pretibial hyperpigmentation, ataxia, decreased motor coordination,
cirrhosis, and decreased motor coordination. The physical exam which would reveal the most
specific finding for this disease is:

A. Hearing test
B. Slit-lamp eye exam
C. EKG
D. Colonoscopy
E. Renal ultrasound

►B

Wilson‘s disease (also known as hepatolenticular degeneration) is an autosomal recessive disorder


result in defective biliary excretion of copper, leading to copper accumulation in the liver, brain and
cornea. Clinical features include hepatomegaly, cirrhosis, ataxia, dysarthria, decreased motor
coordination, pretibial hyperpigmentation, blue lunulae, and copper deposition in the cornea—
Kayser-Fleisher ring, which can be diagnosed using a slit-lamp.

273- Which opthamologic disease is associated with this disorder? This lesion was present at birth.

330
A. Glaucoma
B. Ectopia lentis
C. Cataracts
D. Posterior subcasular lentiular opacity
E. Retinitis pigmentosa

►A

Sturge-Weber syndrome is a sporadic disease characterized by a capillary malformation in the


trigeminal distribution. Patients may have associated cerebral atrophy, vascular malformations of
the leptomeninges, and seizures. All patients with Sturge-Weber should be referred to the
opthamologist for glaucoma screening.

274- Patients with Chondrodysplasia punctata can have findings of stippled epiphyses on X-ray
examination. Which other x-linked dominant condition can have stippled epiphyses?

A. CHILD syndrome
B. Incontinentia Pigmenti
C. Focal Dermal Hypoplasia
D. Goltz syndrome
E. Bazex syndrome

►A

All of the syndromes listed have X-linked dominant inheritance. CHILD syndome also has findings
of stippled epiphyses. Incontinentia pigmenti is caused by defecdts in the NEMO gene. Findings
include peg/conical teeth, eye and CNS defects and alopecia. There are no bone abnormalities.
Focal Dermal Hypoplasia, otherwise known as Goltz syndrome has findings of linear atrophy
following Blaschko's lines with areas of fat herniation, mucocutaneous papillomas and pits,
alopecia, nail dystrophy, tooth abnormalities and osteopathia striata (striations of the long bones).
Bazex syndrome is associated with follicular atrophoderma, hypohidrosis, hypotrichosis and
multiple basal cell carcinomas. There are no bone abnormalities associated.

331
275- Electron microscopic examination of a hair shaft reveals a canal-like groove along the shaft of
a triangular-shaped hair. This patient has:

A. Nethertonǁs syndrome
B. Menkeǁs Kinky Hair syndrome
C. Spun-glass hair
D. Trichothiodystrophy
E. Bjornstad syndrome

►C

Pili trianguli et canaliculi is also known as Spun-glass hair or Uncombable Hair Syndrome.
Netherton patients have trichorexis invaginata, Menkeǁs kinky hair patients have short, brittle sparse
hairs, ―tiger tailǁ• hair is seen in trichothiodystrophy, and pili torti is seen in bjornstad syndrome.

276 -Which of the following is the first symptom of ataxia telangiectasias?

A. Conjunctival telangictases
B. Facial telangiectases
C. Hematologic malgignancy
D. Cerebellar ataxia
E. Breast cancer

►D

Ataxia Telangiectasia (Louis-Bar syndrome) is an autosomal recessive disorder usually caused by


mutations in the ATM gene, which is a chromosomal strand break repair enzyme. Cerebellar ataxia
is the first sign, followed by telangiectases of the conjunctiva and skin. Thymic hypoplasia
predisposes to increased infections. There is increased sensitivity to ionizing radiation resulting in
hematologic and solid tumors. Female carriers have increased risk of breast cancer.

277- The nucleotide excision DNA repair pathway is defective in which disease:

A. Bourneville's disease

332
B. Severe combined deficiency syndrome
C. Griscelli syndrome
D. Xeroderma pigmentosa
E. Sjogren-Larssen syndrome

►D

The pathogensis of Xeroderma Pigmentosum shows mutations i genes encoding DNA repair
enzymes, leading to defective DNA excision repair upon exposure to UV radiation. Severe
combined deficiency syndrome-major defect in cell-mediated and humoral immunity; most lack
antibody-dependent cellular cytotoxicity and natural killer cell function. The pathogensis for
Griscelli Syndrome is a mutation in gene encoding for myosin Va or RAB27 a. Sjogren -Larsson
Syndrome has mutations in the FALDH gene. Bourneville's syndrome (Tuberous Sclerosis)
shows a mutation in either TSC1 ancoding hamartin or TSC2 encoding tuberin.

278 -A patient with myotonic dystrophy and multiple skin lesions most likely has activating
mutations in which of the following?

A. Beta-catenin
B. Desmoplakin
C. Plakoglobin
D. Desmoglein
E. Alpha 6-beta 4 integrin

►A

Myotonic dystrophy with multiple pilomatricomas is described above. Activating mutations in


Beta-catenin are found in this syndrome. The other listed options are desmosomal proteins and
are not involved in this syndrome.

279 -Epidermal nevus syndromes inheritance pattern is:

A. Sporadic
B. X-linked recessive

333
C. X-linked dominant
D. Autosomal recessive
E. Autosomal dominant

►A

Epidermal nevus syndrome has many findings, including: sporadic inheritance, nevus unius lateris,
capillary malformations, cafe au lait macules, mantal retardation and seizures, deafness,
hemiparesis, hemihypertrophy of limbs, kyphoscoliosis and rare solid tumors. A biopsy is helpful to
rule out epidermolytic hyperkeratosis. If positive, the patientǁs offspring are at risk for generalized
epidermolytic hyperkeratosis.

290-A 5 month old girl presents with failure to thrive. She has had life-long atopic dermatitis treated
with topical hydrocortisone cream and has persisent hypernatremia. On your exam, she has
generalized erythema and scaling of her body and trichorrhexis invaginata on examination of hairs
from her eyebrows. Which syndrome is she most likely to have?

A. Leiner syndrome
B. Omenn Syndrome
C. Netherton Syndrome
D. Wiskott-Aldrich Syndrome
E. Severe atopic dermatitis

►C

Netherton syndrome is caused by a mutation in the SPINK5 gene, encoding LEKT1. This is a serine
protease inhibitor which is important in downregulating inflammation. Early presentation is with
failure to thrive, generalized erythema/scale, hypernatremia, and sparse hair with the characteristic
finding of trichorrhexis invaginata. Pili torti and trichorrhexis nodosa also can be seen. Eyebrow
hair is most commonly affected. Omenn syndrome is an autosomal recessive form of severe
combined immunodeficiency (SCID) with findings of failure to thrive, erythroderma, scaling,
chronic diarrhea, lymphadenopathy, and hepatosplenomegaly. Leiner syndrome can present with
failure to thrive, immunodeficiency and seborrheic dermatitis. Wiskott-Aldrich syndrome is an x-
linked recessive condition with mutations of the WAS gene. Presentation includes atopic dermatitis,
thrombocytopenia, recurrent bacterial infection, lymphoreticular malignancy with non-Hodgkin's

334
lymphoma being the most common, and increased IgA, D and E. With the characteristic hair
changes, atopic dermatitis alone is not the most likely diagnosis.

291 -Focal Dermal Hypoplasia (Goltz Syndrome) can differentiated from Incontinentia Pigmenti by

A. Type of inheritance
B. Presence of Linear lesions along the lines of Blaschko
C. Presence of blistering lesions
D. Hair and teeth abnormality
E. Eye and CNS abnormality

►C

Both Focal Dermal Hypoplasia (Goltz Syndrome) and Incontinentia Pigmenti are inherited as X
linked dominant (lethal in males). And both can have lesions along the lines of Blaschko with many
similarities in systemic involvement. However, Incontinentia Pigmenti is differentiated from Focal
Dermal Hypoplasia by presence of blistering lesions in addition to hyperkeratosis and
hyperpigmentation.

292- A patient with 20 nail dystrophy, steatocystoma multiplex and natal teeth likely has a mutation
in the genes coding for:

A. Keratins 5
B. Laminin 5
C. Plakophilin 1
D. Keratins 6b & 17
E. Keratins 6 &16

►D

Pachyonychia congenital is an autosomal dominant condition with 20 nail dystrophy. The patient
described has Type II (Jackson-Sertole) disease, which includes steatocystoma multiplex, natal
teeth, multiple cysts, and micropthalmia, and is caused by mutations in keratins 6b& 17. Type I
(Jadassohn-Lewandowsky) also includes focal symmetric PPK, follicular hyperkeratosis, oral

335
leukokeratoses and is caused by mutations in keratins 6 &16. Type III includes the clinical features
of type I + corneal leukokeratosis. Mutations in keratins 5&14 represents EB simplex, Laminin 5
mutation is seen in Junctional EB, and plakophilin 1 mutation is seen in ectodermal dysplasia with
skin fragility.

293- Which of the following is caused by a defect in lysosomal transport:


A. oculocutaneous albinism I
B. oculocutaneous albinism II
C. Piebaldism
D. Chediak-higashi
E. Bloom‘s syndrome

►D

Chediak higashi is an autosomal recessive disorder caused by a mutation in the LYST gene codes
for a lysosomal tracking protein. This protein regulates microtubule mediated lysosomal fusion. A
defect in this gene leads to giant lysosomal granules seen in neutrophils (leading to defecting
phagocytosis and decreased chemotaxis), melanocytes (pigment dilution), and neurons. OCA1 is
tyrosinase negative albinism; OCA2 is tyrosinase positive albinism with a mutation in P gene on
chromosome 15.

294- Germline KILLIN methylation is associated with which of the following syndromes?

A. Bannayan-Riley-Ruvalcaba Syndrome
B. Proteus Syndrome
C. Cowden Syndrome
D. Birt-Hogg-Dube Syndrome
E. Galli-Galli Disease

►C

Loss-of-function mutations in phosphatase and tensin homolog gene (PTEN) mutations cause 80%
of Cowden Syndrome, a rare autosomal-dominant disorder, characterized by high risks of breast,
thyroid, and other cancers. Other mechanisms of loss of function such as hypermethylation of the

336
KILLIN gene has been identified in Cowden Syndrome. Bannayan Riley-Ruvalcaba and Proteus
syndrome are associated with PTEN loss-of-function mutations. Birt-Hogg-Dube is associated with
loss-of-function mutations in FLCN gene. Galli-Galli Disease is not a malignancy-associated
disease and is the result of KRT5 mutations.

295- The main cause of death in patients with dyskeratosis congenita is which of the following?

A. Oral basal cell carcinoma


B. Leukemia
C. Renal cell carcinoma
D. Pancytopenia
E. Atherosclerotic heart disease

►D

Dyskeratosis congenita is usually inherited in an X-recessive fashion due to mutations in the


dyskerin gene, which is involved in ribosomal RNA synthesis. The less common autosomal
dominant form is caused by mutations in the telomerase gene. Clinically, there is reticulated
pigmentation of skin, poikiloderma, alopecia, nail atrophy, premalignant oral leukoplakia, and
Fanconi-type pancytopenia that can result in early death. They also get severe oral SCC.

296- Pili trianguli et canaliculi is characteristic of which of the following syndromes?

A. Uncombable hair syndrome


B. Bjornstad syndrome
C. Menkes kinky hair syndrome
D. Leiners disease
E. Netherton's syndrome

►A

Uncombable hair syndrome is characterized by pili trianguli et canaliculi. On examination of the


hair, it is triangular with a canal-like groove runs along the shaft. The clinical findings are that of

337
blond, shiny, "spun glass" hair. It is an autosomal dominant syndrome with no known gene locus.
The remaining syndromes do not have this hair finding present.

297 -A patient with neurofibromatosis 1 has axillary and inguinal freckling that is also known as:

A. Crowe's sign
B. Button hole sign
C. Hutchinson's sign
D. Plexiform sign
E. Neurofibromin sign

►A

Patient with NF 1 have an initial presenting sign of cafe-au-lait macules. Axillary or inguinal
freckling is known as Crowe's sign. This is pathogonomic for NF 1. Patients with NF 1 can have a
lit lam examination that will show Lisch nodules that will confirm the diagnosis.

298- Ichthyosiform erythroderma in lines of Blaschko, follicular atrophoderma, and stippled


epiphyses are characteristic of which of the following disorders?

A. Refsum syndrome
B. Chondrodysplasia punctata
C. Erythrokeratodermia variabilis
D. Netherton syndrome
E. Sjogren-Larsson syndrome

►B

Chondrodysplasia punctata is a peroxisomal disorder caused by mutations in several genes that is


associated with ichthyosiform erythroderma in lines of Blaschko, follicular atrophoderma, and
stippled epiphyses.

338
299- Dermatofibrosis lenticularis disseminata is seen in which of the following conditions?

A. Ehlers-Danlos syndrome
B. Marfan syndrome
C. Pseudoxanthoma elasticum
D. Focal dermal hypoplasia
E. Buschke-Ollendorf syndrome

►E

Buschke-Ollendorf syndrome is an autosomal dominant disorder characterized by dermatofibrosis


lenticularis disseminata (cutaneous elastomas distributed symmetrically over the buttocks, trunk and
proximal extremities), and osteopoikilosis (round opacities in bones).

300- Epistaxis in early childhood to teens followed by multiple cutaneous and gastrointestinal
telangectasias describes which of the following syndromes?

A. Hereditary Hemorrhagic Telangiectasia syndrome


B. Maffucci syndrome
C. CREST syndrome
D. Ataxia telangectasia
E. Fabry disease

►A

Hereditary Hemorrhagic Telangiectasia syndrome is described above. The first signs in over 50% of
cases is epistaxis in childhood to young adulthood. Telangectiasias develop in the 30's and 40's.
Other findings include gastrointestinal telangiectasia, hepatic and pulmonary arteriovenous
malformations. The other syndromes listed can have cutaneous vascular lesions and should be
considered on the differential for hereditary hemorrhagic telangiectasia syndrome.

301- What is the genetic defect of this syndrome?

A. ABCA12 gene
B. Transglutaminase 1 (TGM1).
C. GJB2 (connexin 26) gene.

339
D. Steroid sulfatase (STS) gene
E. Profilaggrin gene

►A

Mutations in the ABCA12 gene cause harlequin ichthyosis. X-linked ichthyosis (XLI) caused by a
steroid sulfatase (STS) deficiency. Keratitis-ichthyosis-deafness syndrome is caused by a mutation
in the GJB2 (connexin 26) gene. Ichthyosis vulgaris is caused by a defect in profilaggrin gene and
lamellar ichthyosis involves a mutation in the gene for transglutaminase 1 (TGM1).

302- A patient with melanoma and a malignant glioma is diagnosed with Li-Fraumeni syndrome.
Which of the following tumors occurs most frequently in this disease?

A. Rhabdomyosarcoma
B. Adrenocortical carcinoma
C. Lung carcinoma
D. Breast carcinoma
E. Leukemia

►A

Li-Fraumeni syndrome is a familial tumor syndrome caused by mutations in the tumor suppressor
gene p53. They are at risk for a wide range of malignancies with particularly high occurrences of
soft tissue sarcomas, breast cancer, brain tumors, acute leukemia, and adrenal cortical carcinoma.
Soft tissue sarcomas are among the most common reported with this disease.

303- Findings of dysplastic nevi and melanoma inherited in an autosomal dominant fashion is
linked with which of the following?

A. Breast malignancy
B. Thyroid malignancy
C. Pancreatic malignancy
D. Renal cell carcinoma
E. Colon carcinoma

340
►C

Familial dysplastic nevi/melanoma syndrome is linked with increased risk of pancreatic cancers and
astrocytomas. There are no reported increased risks for patients with familial dysplastic nevi
syndrome and the other types of cancers listed.

304 -Mucosal neuromas, pheochromocytoma and medullary thyroid carcinoma in a patient with a
marfanoid body habitus is associated with which of the following gene defects?

A. Menin
B. RET proto-oncogene
C. PTEN
D. BHD
E. STK11

►B

The RET proto-oncogene is mutated in Multiple Endocrine Neoplasia type II. Type IIb is described
above. Other findings include rare parathyroid abnormalities, megacolon, thickened lips and thick,
everted upper eyelids. Menin is associated with MEN type I, PTEN with Cowden disease, BHD
with Birt-Hogg-Dube syndrome and STK11 with Peutz-Jeghers syndrome.

305- Hyperextesible skin, gaping wounds, cigarette-paper scars, molluscoid pseudotumors, and
calcified subcutaneous nodules are characteristic of which syndrome?

341
A. Marfan syndrome
B. Congenital contractural arachnodactyly
C. Ehlers-Danlos Syndrome
D. Cutis laxa
E. Pseudoxanthoma elasticum

►C

The most common type of EDS is EDS type I (gravis). Cutaneous features include hyperextensible
skin, gaping wounds, cigarette-paper scars, molluscoid pseudotumors, calcified subcutaneous
nodules, and bruises. Systemic features include hypermobile joints with dislocation, hernias, mitral
valve prolapse, blue sclerae, Gorlinǁs sign (tongue reaches nose), and absence of the lingual
frenulum.

306- Which of the following syndromes is associated with cutis marmorata?


A. Netherton‘s
B. Papillon-Lefevre syndrome
C. Hemansky-Pudlak syndrome
D. Maffucci syndrome
E. Cornelia de Lange syndrome

►E

Cornelia de Lange is also known as Brachmann-de Lange syndrome. Cutaneous manifestations


include cutis marmorata, hirsutism, hypoplastic nipples and umbilicus. Patients also have small
hands and feet. They have characteristic facies which include hirsutism on the forehead,
trichomegaly, synophrys, anteverted nostrils, long philtrum, and low-set ears.

307- A patient is diagnosed with Hay Wells syndrome with wiry, sparse hair and ankyloblepharon.
The patient has partial anhidrosis, cleft lip, palate and dystrophic nails. The mutation is in the:

A. p63
B. TP73L
C. DLX3

342
D. PORCN
E. Arylsulfatase

►A

Patients with Hay-wells syndrome has a mutations in P63. They have wiry, sparse hair,
ankyloblepharon, PPK, partial anhidrosis, cleft lip, palate, absent, and dystrophic nails.

308 - Spastic ditetraplegia is associated with which of the following disorders?

A. Sjogren-Larsson syndrome
B. X-linked ichthyosis
C. Lamellar ichthyosis
D. KID syndrome
E. Refsum syndrome

►A

Sjogren-Larsson syndrome is an autosomal recessive disorder caused by mutations in the fatty


aldehyde oxidoreductase/alcohol dehydrogenase gene. This disorder is characterized by ichthyosis,
spastic ditetraplegia, mental retardation, epilepsy, glistening dot retinal pigmentation, and dental
enamel dysplasia.

309- Painful crises and 'whorled' corneal opacities are seen with which of the following enzyme
abnormalities?

A. Homogentisic acid oxidase


B. Alpha-galactosidase A
C. Glucocerebrosidase
D. Iduronate sulfatase
E. Glucoronidase

►B

343
Painful crises and whorled corneal opacities are found in Fabry disease which is caused by a defect
in alpha-galactosidase A. The remaining conditions do not have these findings.

310- Enchondromas and chondrosarcomas are most strongly associated with which of the following
syndromes?

A. Proteus syndrome
B. Osler-Weber-Rendu syndrome
C. Maffucci syndrome
D. Nonne-Milroy disease
E. Blue rubber bleb nevus syndrome

►C

Maffucci syndrome is a sporadic condition caused by defects in the parathyroid hormone/


parathyroid hormone related protein type 1 receptor. There are venous malformations of distal
extremities, and benign enchondromas that can degenerate into chondrosarcomas.

311 -To help diagnose trichothiodystrophy, which of the following levels are decreased in hairs of
affected individuals?

A. Arginine
B. Histidine
C. Phenylalanine
D. Glycine
E. Cysteine

►E

Cysteine and Methionine levels are decreased in hair and nails of patients with trichothiodystrophy.
Other sulfur containing amino acids include: Gluthathione, Taurine, and Homocysteine. Testing
hairs for decreased sulfur content is an indirect method of determining this. The other listed amino
acids are present in normal levels in the hair and nails of trichothiodystrophy patients.

344
312 -A mitten deformity is most characteristic of what inherited disease?

A. Hallopeau Siemens dystrophic epidermolysis bullosa


B. Non-hallopeau Siemens dystrophic epidermolysis bullosa
C. Herlitz type junctional epidermolysis bullosa
D. Non-herlitz type junctional epidermolysis bullosa
E. Epidermolysis bullosa simplex

►A

Hallopeau Siemens dystrophic EB is characterized by pseudosyndactyly, mitten deformities of


the hands and feet. Non Hallopeau Siemens is much less severe. Not present in junctional
epidermolysis bullosa or epidermolyisis bullosa simplex.

313 -Which keratins are expressed in the suprabasal palmoplantar epidermis?

A. Keratins 1 and 9
B. Keratins 1 and 10
C. Keratins 4 and 13
D. Keratins 5 and 14
E. Keratins 8 and 18

►A

Keratins are expressed in pairs of acidic and basic keratins and are tissue- and differentiation-
specific. Keratins 1 and 9 are expressed in the suprabasal palmoplantar epidermis, keratins 1 and
10 in suprabasal nonpalmoplantar epidermis, keratins 2e and 10 in the granular layer, keratins 4

345
and 13 in mucosal epithelium, keratins 5 and 14 in the basal layer, and keratins 8 and 18 in
simple epithelium.

314 - Ehlers-Danlos Syndrome with congenital adrenal hyperplasia is caused by mutations


affecting which of the following?

A . Lysyl hydroxylase
B. Collagen 5
C. Fibronectin
D.Lysyloxidase
E. Tenascin-X

►E

Ehlers-Danlos Syndrome type with congenital adrenal hyperplasia is caused by mutations in


tenascin-X.

315 -This syndrome has colobomas of the eyes, heart defects, ichthyosiform dermatosis, mental
retardation and ear defects. These are the characteristic findings of:

A. CHIME syndrome
B. Vogtg-Koyanagi-Harada syndome
C. Sturge Weber syndrome
D. Conradi-Hunermann syndrome
E. Refsum syndrome

►A

CHIME syndrome has all the findings of colobomas of the eye, heart defects, ichthyosiform
dermatosis, mental retardation and ear defects.

316 -The treatment for acrodermatitis enteropathica is:

346
A. Zinc supplementation
B. Iron supplementation
C. Vitamin B1 supplementation
D. Vitamin B12 supplementation
E. Phlebotomy

►A

Acrodermatitis enteropathica is due to a defect in zinc absorption and will respond to zinc
supplementation. Iron, Vitamin B1/12 supplementation will not result in improvement in this
condition. Findings include periorificial, scalp, and acral dermatitis, scaling, vesicles/bullae,
erosions, alopecia, diarrhea and stomatitis.

317- What condition is associated with this finding of inflammatory keratotic facial papules which
may result in scarring and atrophy?

A. Chloracne
B. Systemic lupus erythematosus
C. Keratosis pilaris
D. Reiter's syndrome
E. Ulerythema ophryogenes

►E

Ulerythema ophryogenes is a rare disorder that affects children and young adults. It is characterized
by keratosis pilaris atrophicans and loss of lateral third of eyebrow.

318- Which of the following subtypes of Ehlers-Danlos Syndrome (EDS) is associated with early
demise?

A. Type 1 (Gravis)
B. Type 4 (Vascular)
C. Type 5 (X-linked)
D. Type 7 (Arthrochalasis multiplex congenita)

347
E. Type 10 (Fibronectin)

►B

Ehlers-Danlos Syndrome type 4 (Vascular) is caused by mutations in collagen 3 in some cases.


There is a tendency to develop arterial and visceral rupture resulting in early death.

319- The characteristic dental findings in patients with tuberous sclerosis are:

A. Peg teeth
B. Anodontia
C. Enamel pits
D. Odontogenic cysts
E. Retention of primary teeth

►C

Enamel pits are the characteristic dental findings in tuberous sclerosis. Peg teeth are found in
incontinentia pigmenti and anhidrotic ectodermal dysplasia. Anodontia is found in hypomelanosis
of ito and incontinentia pigmenti. Odontogenic cysts are seen in Gorlin syndrome, and retention of
primary teeth is characteristic of Job syndrome.

320- Which of the following syndromes are linked to a AKT1 gene mutation?

A. Banayan-Zonana
B. Proteus
C. Cowden
D. All of these options are correct
E. Lhermitte-Duclos disease

►B

348
Proteus is due to an AKT1 mutation. The remaining syndromes are due to PTEN mutations. The
PTEN gene produces a phosphatase the regulates the cell-cycle and apoptosis, therefore acting as a
tumor suppressor gene. Tissues affected by this mutation are those with increased proliferation such
as epidermis, the oral and gastrointestinal mucosa, and the thyroid and breast epithelium. All of the
syndromes listed have mutations in PTEN.

321- A mentally retarded patient is found to also have a malar rash and downward lens
displacement. Another clinical feature might include:

A. Hypertrichosis
B. Short stature
C. Multiple nevi
D. Periodontitis
E. Deep venous thromboses

►E

The above described patient has homocystinuria, an autosomal recessive disorder caused by a
deficiency in cystathionine beta-synthase, leading to an accumulation of homocysteine. Clinical
features include: malar flush, DVTs, livedo reticularis, leg ulcers, downward lens displacement,
myopia, glaucoma, sparse fine hair, mental retardation, and musculoskeletal anomalies.

322- Which of the following bony defect is found in CHILD syndrome?

A. Polyostotic fibrous dysplasia


B. Stippled epiphyses
C. Calcification of falx cerebri
D. Osteopoikilosis
E. Sphenoid wing dysplasia

►B

CHILD syndrome is an X-linked dominant syndrome which is lethal in males. It is caused by a


peroxisomal biogenesis disorder. It is characterized by unilateral ichthyosiform erythroderma, limb/

349
visceral hypoplasia, and stippled epiphyses. Stippled epiphyses can also be seen in
chondrodysplasia punctata. Polyostotic fibrous dysplasia is found in McCune-Albright syndrome,
calcification of falx cerebri in Gorlin's syndrome, osteopoikilosis in seen in BuschkeOllendorf
syndrome.

323- Which of the following diseases is caused by a mutation in a gap junction protein?

A. Striated PPK
B. Schopf-Schulz-Passarge syndrome
C. Mal de Meleda
D. Vohwinkel syndrome (classic)
E. Vohwinkel syndrome (ichthyotic)

►D

Classic Vohwinkel syndrome is caused by mutations in connexin 26, a gap junction protein.
Ichthyotic Vohwinkel syndrome is caused by mutations in loricrin and has ichthyosis but not
deafness.

324- A patient that has the clinical triad of ichthyosis, di- or tretraplegia, mental retardation that is
caused by a deficiency in FALDH has:

A. Sjogren Larsson Syndrome


B. KID Syndrome
C. Vohwinkel Syndrome
D. Refsum Disease
E. Bjornstad's Syndrome

►A

This patient has Sjorgen-Larsson syndrome, which is autosomal recessive disorder caused by a
deficiency of FALDH. These patients have the clinical triad of congenital ichthyosis, tetraplegia,
and mental retardation. The ichthyosis is notable in that there is accentuation of the flexures and
lower abdomen.

350
325 - A patient develops palmoplantar keratoderma and has esophageal cancer. They also state that
their parent had the same condition. This is most characteristic of:

A. Howel-Evans
B. Cowden's
C. Fabry's Disease
D. Castleman's tumor
E. Dowling darlings syndome

►A

Patients that have Howel-Evans syndrome are subject to squamous cell carcinoma of the esophagus
and also PPK on the hands. It is inherited in an autosomal dominant fashion.

326- A patient with thyroid carcinoma and cobblestone-like changes of the oral mucosa will also
likely have:

A. Trichoepitheliomas
B. Fibrofolliculomas
C. Tricholemmomas
D. Cylindromas
E. Syringomas

►C

The patient described may have Cowden's syndrome, an autosomal dominant condition caused by a
defect in the PTEN tumor suppressor gene. Patients with Cowden's disease are at increased risk for
thyroid and breast carcinoma. In addition, they characteristically have multiple hamartomatous
polyps of the gastrointestinal tract that are typically benign. Cutaneous features of Cowden's
syndrome which may serve as clues to the diagnosis include multiple oral papillomas with a
"cobblestone" appearance on the lips, gingival, and buccal mucosa, acral keratotic papules on the
dorsal hands and wrists, palmoplantar punctate keratoses and multiple facial tricholemmomas.
Patients with this condition need careful malignancy surveillance.

351
327- Which of the following is caused by a mutation in a gene which codes for steroid sulfatase?

A. Fabry disease
B. Lesch-nyhan disease
C. X-linked ichthyosis
D. Lamellar icthyosis
E. Chediak-Higashi

►C

The mutation in X-linked icthyosis is found in the gene for aryl sulfatase C, a steroid sulfatase. The
genetic defect in Fabry disease occurs in alpha-galactosidase A (which hydrolyzes glycolipids and
glycoproteins), the defect in lamellar icthyosis codes for transglutaminase 1 and the defect in
chediak higashi occurs in a lysosomal transport protein.

328- A patient is diagnosed with Naxos syndrome with woolly hair, diffuse palmoplantar
keratoderma, and right ventricular arrhythmogenic cardiomyopathy. The mutation is in:

A. Plakoglobin
B. DNA helicase
C. Ectodysplasin A
D. Connexin 30
E. Connexin 26

►A

Patients with Naxos syndrome has a mutation in plakoglobin which is autosomal recessive. It is
associated with woolly hair and cardiomyopathy.

329 -Which is a major criterion for the diagnosis of neurofibromatosis type 1?

A. One plexiform neurofibroma


B. Scoliosis
C. Pheochromocytoma

352
D. Hypertension
E. Two cafe au lait macules

►A

The major diagnostic criteria are: 6 or more cafe au lait macules >5mm in prepubertal individuals
and >15mm in post pubertal individuals, 2 or more neurofibromas of any type or one plexiform
neurofibroma, axillary or inguinal freckling, 2 or more lisch nodules, sphenoid wing dysplasia or
thinning of long bone cortex with or without pseudoarthosis, first degree relative with NF1.

330- A patient is diagnosed with progeria and has a large bald head, prominent scalp veins, and lack
of eyebrows and eyelashes. The gene mutation is the:

A. LMNA gene
B. Recq12 gene
C. PTPN11 gene
D. PRKAR1A gene
E. PTEN gene

►A

A patient was progeria has a mutation in the LMNA gene which encodes lamin A and C. It is
distinguished from adult progeria by early onset.

331- A 15 year old male patient is seen for evaluation of a congenital nevus. As a part of the review
of systems, it is discovered that he has recurrent joint dislocations. Multiple family members have a
similar problem. What is the defective gene in Type 3 Ehlers-Danlos syndrome?

A. Collagen 3
B. Collagen 5
C. Lysyl oxidase
D. Collagen 2
E. PLOD gene

353
►A

Type 3 Ehlers-Danlos syndrome, or benign hypermobile Ehlers-Danlos syndrome, is a relatively


mild form of EDS with recurrent joint dislocations. The syndrome is a result of defective collagen
3. Defective collagen 5 is seen in EDS types 1 and 2 (gravis and mitis, respectively). Lysyl oxidase
deficiency is seen in Type 5 (x-linked). Collagen 2 mutations can lead to Type 7 (arthrochalasis
multiplex congenita). The PLOD gene is involved in type 6 (ocular-scoliotic).

332- Mucosal malignancy is a complication of:

A. Oral hairy leukoplakia


B. White sponge nevus
C. Dyskeratosis congenita
D. Chronic candidiasis
E. Focal epithelial hyperplasia

►C

Dyskeratosis congenita, also called Zinsser-Engman-Cole syndrome, is an X-linked recessive


genodermatosis caused by a mutation in DKC1. DKC1 encodes for dyskerin, which helps to
maintain telomeres through the pseudouridylation of rRNA. Features of this condition include
reticulate gray-brown hyperpigmentation, dystrophic nails, alopecia and Fanconiǁs type
pancytopenia. Patients may have premalignant leukoplakia which should be followed closely.

333- Patients with homocystinuria are deficient in:

A. Alpha galactosidase A
B. Transglutaminase
C. Lysyl oxidase
D. Cystathionine synthetase
E. Steroid sulfatase

►D

354
Homocystinuria is an autosomal recessive disease caused by a deficiency of cystathionine
synthetase. Manifestations of this condition may include mental retardation, marfanoid habitus,
malar flush, ectopic lentis, embolic phenomena and leg ulcers.

334- Which of the following is caused by a defect in cathepsin C?

A. Rubenstein-Taybi
B. Bullous Icthyosis of Siemens
C. Muir-Torre
D. Papillon-Lefevre
E. Nail-Patella syndrome

►D

Papillon-Lefevre is a palmoplantar keratoderma caused by a mutation in chromosome 11q14,


leading to a defect in Cathepsin C, a lysosomal enzyme. Clinical manifestations of papillon lefevre
include sharply demarcated palmoplantar keratoderma with extension to dorsal surface
(transgrediens), spare hair, periodontitis, and pyoderma.

335- What is this syndrome which is histologically characterized by widely dispersed granular
material amidst normal fibers?

A. Ehlers Danlos Syndrome


B. Pseudoxanthoma Elasticum
C. Buschke-Ollendorf Syndrome
D. Focal Dermal Hypoplasia
E. Lipoid Proteinosis

►B

Pseudoxanthoma elasticum is genodermatosis characterized by redundant skin, angioid streaks,


yellow papules on the mucous membranes and bleeding from gastric artery. On histology, readily
apparent denerative changes of the elastic fibers are prominent, even without special stains.

355
336- Odontogenic cysts and palmoplantar pits are seen in:

A. Gardnerǁs syndrome
B. Gorlin Syndrome
C. Bloomǁs Syndrome
D. Goltz Syndrome
E. Refsum syndrome

►B

Gorlin syndrome (Basal Cell Nevus Syndrome)is an autosomal dominant disorder due to a defect in
the PTCH gene whose function normally inhibits ―SMOOTHENEDǁ signaling. Odontogenic cysts
and palmoplantar pits are characteristic features, in addition to multiple bas al cell carcinomas.
Other features include: frontal bossing, kyphoscoliosis, calcification of falx cerebri, hypertelorism,
ovarian fibromas and rarely mental retardation.

337- Which of the following mutations has been linked to an NF-1 like presentation?

A. SLURP-1
B. Plakophilin-1
C. SPRED-1
D. ALK-1
E. TSC-1

►C

SPRED-1 acts as a suppressor of the Ras/MAPK pathway, the same pathway involved in
neurofibromatosis-1. Likewise, defects in SPRED1 are the cause of neurofibromatosis type 1 like
syndrome, also known as Legius Syndrome. Legius Syndrome has the following characteristics:
cafe au lait spots, axillary and inguinal freckling, learning disability, and macrocephaly; likewise
patients can actually meet criteria for the clinical diagnosis of NF-1. Although Legius Syndrome has
many overlapping features with NF1, neurofibromas, lisch nodules, and CNS tumors have not been
reported. (Brems H, et al. Nat Gen 2007.)

356
338- A 11 year-old female patient with hypoparathyroidism is referred to your clinic secondary to
chronic mucocutanous candidiasis which is refractory to standard treatments. The patient also has
malabsorption and severe chronic diarrhea. You determine that she has autoimmune
polyendocrinopathy-candiasis-ectodermal dystrophy syndrome(APECED). What is the gene defect
for APECED?

A. XAP101 gene
B. AIRE gene
C. CGI-58 gene
D. ALOX gene
E. PAHX gene

►B

The gene responsible for APECED is the AIRE gene, an autoimmune regulator. Th e XAP101
encodes dyskerin, a defect in which causes dyskeratosis congenita. A defect in CGI-58 causes
Chanarin-Dorfman or neutral lipid storage disease with ichthyosis. Defect in the ALOX genes cause
congenital ichthyosiform erythroderma. A defect in the PAHX gene causes Refsum syndrome.

339- An infant girl of short stature and shortened 4th and 5th metacarpals is being evaluated for
coarctation of the aorta and horseshoe kidneys. Physical examination most likely reveals:

A. Webbed neck
B. Alopecia
C. Hemangioma
D. Giant congenital melanocytic nevus
E. Arachnodactyly

►A

Turnerǁs syndrome results from nondysjunction during gametogeneiss leading to the XO genotype.
Clinical features include short stature, redundant neck folds/webbed neck, multiple pigmented nevi,
low set hairline, triangular facies, low-set ears, ptosis, wide-set nipples, shortened 4th and 5th
metacarpals, hypoplasia of lymphatics, coarctation of the aorta, and horseshoe kidneys.

357
340- The coast of Maine cafe au lait macule is a common finding in which of the following
syndromes?

A. Tuberous Sclerosis
B. Neurofibromatosis type I
C. McCune-Albright syndrome
D. Watson syndrome
E. Russell-Silver syndrome

►C

McCune-Albright syndrome is due to a sporadic somatic mutation in Gs subunit of adenylate


cyclase. The "coast of Maine" cafe au lait macule is a characteristic finding. Other findings in clude
polyostotic fibrous dysplasia and precocious puberty. Tuberous sclerosis, Neurofibromatosis type I,
Watson syndrome (pulmonic stenosis and CALM's) and Russell-Silver syndrome (CALMs, short
stature, precocious puberty, cryptorchidism and musculoskeletal/craniofacial defects) all have
CALMs as a feature, but not the jagged type seen in McCune-Albright syndrome.

341 -A patient is diagnosed with Rothmund-Thomson syndrome and has juvenile cataracts,
hypogonadism and dystrophic nails. The hair findings is/are:

A. Sparse scalp, eyebrow, and eyelash hair


B. Unruly hair
C. Uncombable hair
D. Woolly hair
E. Pili trianguli et canaliculi

►A

Rothmund-Thomson syndrome is an autosomal recessive disorder. They have a sparse scalp,


eyebrow, and eyelash hairs. Poikiloderma is seen in sun-exposed areas, dystrophic nail and juvenile
cataract.

358
342- In ataxia telangectasia, the ATM gene is mutated. The product of the ATM gene is an enzyme
which:

A. Responds to DNA damage by phosphorylating key DNA repair substrates


B. Binds transforming growth factor beta protein
C. Is the VEGF receptor 3
D. Participates in NF-kB activation
E. Is an inhibitor of G1 cyclin/Cdk complexes

►A

The ATM gene is a member of the phosphatidylinositol-3 family of proteins that respond to DNA
damage by phosphorylating key substrates involved in DNA repair according to OMIM. Defects in
endoglin (TGF beta3 binding protein) is deficient in Osler-Weber-Rendu syndrome. The VEGF
receptor 3 is defective in hereditary lymphedema. The NEMO gene is defective in Incontinentia
Pigmenti. Its product, NF-kB essential modulator (NEMO) is a key activator in the NF-kB pathway.
KIP2 is involved in AD Beckwith-Wiedemann syndrome and is an inhibitor of G1 cyclin/Cdk
complexes.

343- Which of the following conditions is worsened by ingestion of lithium?

A. Darier‘s Disease
B. Hailey-Hailey Disease
C. Haim-Munk syndrome
D. Hereditary lymphedema (Nonne-Milroy disease)
E. Epidermolytic hyperkeratosis

►A

Patients with Darier‘s disease should not be treated with lithium due to its worsening or in some
cases unmasking the disease. The mechanism for this is not known. Lithium treatment does not
worsen the other listed conditions.

344- Which of the following signs is not a criteria for the diagnosis of Neurofibromatosis type I?

359
A. Bilateral vestibular schwannomas
B. Axillary freckling
C. Optic gliomas
D. greater than 5 café-au-lait macules
E. Sphenoid dysplasia

►A

Bilateral vestibular schwannomas are related to type II neurofibromatosis. The remaining options
are diagnostic criteria for NF-1. Greater than 5 CALMS is the same as 6 or more CALMS as listed
in the diagnostic criteria.

345- A thirty-year-old woman presents with new progressively worsening headaches. Upon exam,
you notice multiple acral keratotic papules and papillomas of the tongue. What malignancy does
this patient need to be screened for?

A. colon cancer
B. ovarian cancer
C. breast cancer
D. basal cell carcinoma
E. melanoma

►C
This patient has Lhermitte-Duclos disease, which is a hamartomatous overgrowth of cerebellar
ganglion cells. Approximately half of patients have Cowden syndrome. Breast cancer is the correct
answer, which affects 25-35% of female patients. Patients can also develop thyroid and
genitourinary carcinoma (endometrial, urethral, renal cell, and transitional cell carcinoma of the
renal pelvis). Malignant degeneration of hamartomatous colon polyps is rare.

346 -Multiple lipomas may be found in all of the following except?

A. Gardner's syndrome
B. Gorlin's syndrome
C. Cowden's syndrome

360
D. Neurofibromatosis
E. Proteus syndrome

►B

Gorlin's syndrome (Basal cell naevus syndrome) is not associated with multiple lipomas. The
PTEN associated syndromes (Cowden's, Bannayan-Riley-Ruvalcaba and Proteus syndromes) are
all associated with multiple lipomas as well as Gardner's, neurofibromatosis and MEN.

347 -Which of the following syndromes is X-linked dominant?

A. Anhidrotic ectodermal dysplasia


B. Dyskeratosis congenita
C. X-linked icthyosis
D. Orofaciodigital syndrome 1
E. Menkes kinky hair syndrome

►D

Orofaciodigital sydrome 1 is an X-linked dominantly inherited disorder caused by a defect in the


CXORF5 gene. The rest of the above conditions are inherited in an x-linked recessive pattern.

348 -Ichthyosis with confetti is a severe, sporadic ichthyosis caused by mutations in which gene?

A. Keratin 1
B. Keratin 2e
C. Keratin 10
D. Keratin 14
E. Keratin 15

►C

Ichthyosis with confetti, a severe, sporadic skin disease in humans, is caused by mutations in the

361
gene encoding keratin 10 (KRT10); all result in frameshifts into the same alternative reading frame,
producing an arginine-rich C-terminal peptide that redirects keratin 10 from the cytokeratin filament
network to the nucleolus.

349 -This syndrome has a characteristic sensorineural hearing loss and also pili torti that makes the
hair lusterless. This patient most likely has:

A. Bjornstad syndrome
B. Argininosuccinic aciduria
C. Monilethrix
D. Uncombable hair syndrome
E. Hypohidrotic ectodermal dysplasia

►A

Bjornstad syndrome has a characteristic sensorineural deafness that is bilateral and also flattened
twisted hairs called pili torti that causes the hair to look lusterless. It can be autosomal dominant
and cases of autosomal recessive conditions have been reported

350 -Homocystinuria is characterized by arterial and venous thromboses and osteoporosis and
mental retardation. It is also characterized by a downward displacement of the lens. It is also due
to a deficiency of:

A. Cystathionine beta synthase


B. Pyridoxine folate
C. Homocysteine
D. Methionine
E. Elastolysis

►A

Patients with homocystinuria have elevated serum homocysteine and methionine levels due to a
deficiency of cystathionine beta synthase.

362
351 -A patient with hypohydrosis and hyperpyrexia, anodontia, and sparse hair has which
syndrome:

A. Dyskeratosis congenita
B. Pachyonychia congenita
C. Anhidrotic ectodermal dysplasia
D. Hidrotic ectodermal dysplasia
E. Papillon Lefevre

►C

Anhidrotic ectodermal dysplasia is an x-linked recessive disorder which presents with the triad
of hypohydrosis (or anhidrosis) with hyperpyrexia, anodontia (other dental findings include peg
teeth, molars with hooked cusps) and sparse hair. Patients tend to overheat.

352- A patient with multiple facial trichilemmomas is at risk of which of the following cancers?

A. Basal cell carcinoma


B. Breast carcinoma
C. Acute leukemia
D. Cylindroma
E. Oral squamous cell carcinoma

►B

Cowden syndrome (multiple hamartoma syndrome) is an autosomal dominant disorder caused by


mutations in PTEN, a phosphatase that dephosphorylates tyrosine, serine, and threonine. Clinically,
there are numerous facial trichilemmomas, oral papillomas, acral keratotic papules, sclerotic
fibromas, breast fibroadenomas and adenocarcinomas, thyroid adenomas and adenocarcinomas, and
hamartomatous polyps of the gastrointestinal tract.

353 -The most common cutaneous neoplasm associated with Muir Torre Syndrome is:

A. Sebaceous carcinoma
B. Sebaceous adenoma

363
C. Keratoacanthoma
D. Basal cell carcinoma with sebaceous differentiation
E. Squamous cell carcinomas

►B

Muir-Torre syndrome is an autosomal dominant disorder due to a DNA mismatch repair gene
(MSH2). Patients present with sebaceous tumors, with adenomas being the most common
(sebaceous carcinomas next common) and associated visceral malignancies such as colon cancer
(most common).

354- In chronic granulomatous disease, the diagnosis is made by which of the following tests?

A. Assay for fumarate hydratase


B. Nitroblue tetrazolium reduction assay
C. Assay for sphingomyelinase
D. Skin biopsy
E. Assay for glucocerebrosidase
►B

Chronic granulomatous disease is diagnosed by the nitroblue tetrazolium reduction assay. The
abnormal white blood cells cannot reduce dye due to the inability to produce the respiratory
burst. This is needed to kill catalase positive organisms after phagoc ytosis. Fumarate hydratase is
defective in familial multiple cutaneous leiomyomatosis, sphingomyelinase in Niemann-Pick
disease and glucocerebrosidase in Gaucher disease.

355- Meleda is an island off the coast of Croatia. Its inhabitants have an increased frequency of a
malodorous transgradiens palmoplantar keratoderma in a stocking-glove distribution. This condition
is autosomal recessive with a defect in:

A. SLURP1 gene
B. Keratin 1/9
C. TOC gene
D. Loricrin
E. Cathepsin C

364
►A

SLURP1 (Secreted LY6/UPAR-related protein 1) is defective in Mal de Meleda. Keratin 1/9 are
defective in Unna-Vorner/Thost palmoplantar keratoderma, an AD diffuse symmetric
nontransgradiens PPK. TOC gene is defective in Howel-Evans syndrome, an AD PPK associated
with focal, pressure-related, non-transgradiens PPK and esophageal cancer/oral leukoplakia.
Loricrin mutations are seen in Vohwinkel syndrome variant and symmetric progressive
erythrokeratodermia. Cathepsin C defects are seen in Haim-Munk syndrome
(PPK+periodontitis+acroosteolysis+onychogryphosis) and Papillon-Lefevre syndrome (sharply
demarcated transgradiens, stocking-glove PPK+periodontitis+dural calcifications and choroids
attachments).

356- Beare-Stevenson cutis gyrata syndrome is linked with mutations in:

A. Fibroblast growth factor receptor 2


B. BSCL2
C. LMNA
D. ATP7A
E. None of these answers are correct

►A

Beare-Stevenson cutis gyrata syndrome has been linked to mutations in fibroblast growth factor
receptor 2. This syndrome is characterized by: craniosynostosis, cutis gyrata, acanthosis nigricans,
anogenital anomalies, skin tags, prominent umbilical stump, furrowed palms and soles. Apert
syndrome is also linked to this mutation. BSCL2 is linked to Berardinelli-Seip congenital
lipodystrophy, LMNA to Familial partial lipodystrophy and ATP7A to Menkes kinky hair syndrome.

357 -The most common cutaneous association with monilethrix is:

A. Eczema
B. Hypopigmentation
C. Hyperpigmentation
D. Keratosis Pilaris
E. Atrophy

365
►D

Monilethrix is an autosomal dominant condition which, by definition, presents with ―be adedǁ•
hear. Clinically, patients present with short, sparse lusterless hair. Keratosis pilaris is the most
common associated feature.

358- Menkeǁs kinky hair syndrome is caused by a defect in:

A. Mitochondrial gene
B. DNA helicase
C. Gap junction protein
D. Copper Transporting ATPase
E. Proto-oncogene

►D

Menkeǁs kinky hair syndrome is an x-linked recessive disorder caused by a mutation at Xq12
leading to defective intestinal copper transport.

359 -A patient with colon cancer is diagnosed with Muir-Torre syndrome. Which of the following
cutaneous lesions might the patient have?

A. Keratoacanthomas
B. Seborrheic keratoses
C. Tricholemmomas
D. Arsenical keratoses
E. clear cell acanthomas

►A

Muir-Torre syndrome is an autosomal dominant disease caused by mutations in MSH2 and MLH1,
DNA mismatch repair genes. Clinically, patients have multiple sebaceous tumors (adenomas are
most common), keratoacanthomas, and are at risk for adenocarcinoma of the colon.

366
360- Medulloblastomas are seen in which of the following syndromes?

A. Bazex syndrome
B. Familial cylindromatosis
C. Nicolau-Balus syndrome
D. Birt-Hogg-Dube syndrome
E. Nevoid basal cell carcinomas syndrome

►E

Nevoid basal cell carcinomas syndrome (Gorlin syndrome) is an autosomal dominant condition
caused by mutations in the patched gene, which is involved in the hedgehog signaling pathway.
Patients develop innumerable BCCs, palmoplantar pits, painful odontogenic jaw keratocysts, frontal
bossing, bifid ribs, and calcification of the falx cerebri. Medulloblastomas and ovarian fibromas and
fibrosarcomas are associated with this condition.

361- Cowden's disease is associated with this kind of malignancy:

A. Follicular thyroid
B. Renal cell carcinoma
C. Squamous cell carcinoma
D. Leukemia
E. Basal cell carcinoma

►A

Cowden's disease is an autosomal dominant disorder is a mutation in the PTEN gene. It is


associated with breast cancer, follicular thyroid, and colon hamartoma.

362 -A 3 year-old boy presents with the findings seen in the image. He also has thrombocytopenia
with purpura and a history of recurrent pyogenic bacterial infections. What is the most likely
diagnosis in this child?

A. Chronic Granulomatous disease

367
B. Wiskott-Aldrich syndrome
C. Hyper-IgE syndrome
D. Severe combined immunodeficiency
E. Leiner syndrome

►B

The findings described are consistent with Wiskott-Aldrich syndrome. The characteristic triad can
be simplified to the 3 P's - Pruritus (atopic dermatitis), Purpura (thrombocytopenia leading to
purpura and other bleeding) and pyogenic infections. The remaining options are related to Wiskott-
Aldrich in that they all have immunodeficiency as a feature, but not the same spectrum of disease.

363 -A patient with pseudoherpetic keratitis and a painful PPK would improve with a diet low in:

A. Tyrosine/Phenylalanine
B. Glycine
C. Cytosine
D. Biotin
E. Zinc

►A

Pseudoherpetic keratitis and a painful PPK describes a patient with Richner-Hanhart syndrome
(tyrosenemia type II). Treatment is with a diet low in tyrosine and phenylalanine. A diet low in
glycine, cytosine, biotin or zinc would not be helpful in this syndrome.

364- Regarding the inheritance of Ehlers-Danlos syndrome, which subtype is inherited in an X-


linked recessive manner?

A. The type with the lysyl oxidase deficiency


B. The type with the lysyl hydroxylase deficiency
C. The type with a collagen 5 defect
D. The type with a collagen 3 defect
E. The type with a defect in procollagen aminopeptidase

368
►A

Lysyl oxidase deficiency is related to type V or X-linked Ehlers-Danlos syndrome (EDS). Type IX,
also has x-linked recessive inheritance with mild symptoms of EDS with occipital exostoses and
hernias. Lysyl hydroxylase deficiency is seen in type VI EDS, linked with severe kyphoscoliosis,
retinal detachment and other eye abnormalities. Collagen V deficiencies are seen in type I (Gravis)
and type II (Mitis) EDS, associated with skin elasticity, gaping wounds, hypermobile joints, Gorlins
sign, blue sclera and mitral valve prolapse.

365- Lamellar ichthyosis is caused by a defect in transglutaminase 1. It can present as a colloidion


baby at birth with subsequent large thick plates of scale especially on flexures, ectropion and
eclabium. If two unaffected carrier parents have a child, how likely is their child to have this
condition?

A. 25%
B. 75%
C. Only male offspring are affected
D. Only female offspring are affected
E. None of these answers are correct

►A

Lamellar ichthyosis is an autosomal recessive condition. If each parent is a heterozygous carrier,


there is a 25% chance that the child will be affected, 50% chance that the child will be a
heterozygous carrier and a 25% chance that the child will not be a carrier or affected with lamellar
ichthyosis.

366-A patient with multiple deeply pigmented papules has a skin biopsy which reveals an epitheloid
blue nevus. The next appropriate step is:

A. Reassure the patient and follow up as needed


B. Schedule prophylactic excision of the lesion
C. Begin a malignancy work-up
D. Refer to genetics
E. Order an echocardiogram

369
►E

Epithelioid blue nevi have been reported with and without association with cardiac myxomas as a
component of the Carney complex (NAME/LAMB syndrome). Carney complex is an autosomal
dominant disorder caused by mutations in PRKAR1A. Patients have cutaneous and atrial myxomas,
blue nevi, ephelides, adrenocortical disease, and testicular tumors.

367 -A patient has sebaceous neoplasms with multiple keratoacanthomata and is diagnosed with
MuirTorr Syndrome. The most common malignancy is:

A. Colon cancer
B. Thyroid cancer
C. Breast cancer
D. Laryngeal cancer
E. Uterine cancer

►A

Patient with Muir-Torre Syndrome have a high incidence of colon cancer. It is an autosomal
dominant disorder caused by the mutation in either MSH2 or MLH1 gene. The other malignancy
can be seen with this syndrome but colon is the most common.

368 -Which gene is most commonly mutated in melanoma on non sun-damaged skin?

A. BRAF
B. NRAS
C. C-kit
D. CTLA-4
E. CDKN2A

►A

370
Regarding the genetics of melanoma, BRAF mutations are more common than NRAS mutations on
non sun-damaged skin. C-kit is associated with mucosal melanoma. Acral melanoma are associated
with c-kit more commonly than with BRAF.

369 -What is the most common genetic defect associated with this syndrome?

A. Neurofibromin
B. Merlin
C. Tuberin
D. Hamartin
E. Folliculin

►A

Neurofibromatosis I occur due to a microdeletion at 17q11.2 involving the NF1 gene, which
encodes for neurofibromin. It is an autosomal dominant disorder characterized by numerous benign
tumors (neurofibromas) of the peripheral nervous system, cafe au lait macules, freckling in the area
of the armpit (crowǁs sign), two or more growths on the iris of the eye (known as Lisch nodules or
iris hamartomas), tumor on the optic nerve (optic glioma), abnormal development of the spine
(scoliosis), the temple (sphenoid) bone of the skull, or the tibia (one of the long bones of the shin)
and a first degree relative (parent, sibling, or child) with NF1. The other proteins in the list are
associated with other syndromes: In neurofibromatosis type 2, a NF2 gene mutation has been
identified which encodes for a protein called Merlin, in tuberous sclerosis two genetic mutations
have been identified on two separate chromosomes namely tuberin and hamartin, and a Folliculin
mutation is seen in Birt Hogg Dube syndrome.

370 -Ataxia- Telangiectasia

371
A. Is associated with significantly increased risk of ovarian cancer in carriers
B. Telangiectasias present in late adulthood
C. Carriers do not have an increased risk of malignancy
D. Is inherited in XLR (X-linked recessive)
E. First clinical sign is neurologic

►E

First sign is ataxia. Telangiectasias occur later in puberty. Carriers show an increased risk of breast
cancer.

371- Patients with Hermansky-Pudlak syndrome may experience which of the following systemic
complications as a result of their disease?

A. Arteriovenous malformations
B. Pulmomary fibrosis
C. Gastroesophageal reflux disease
D. Aortic stenosis
E. Rectal abscesses

►B

Patients with Hermansky-Pudlak syndrome are tyrosinase positive albinos. In addition to their
pigmentary dilution and increased risks for cutaneous malignancies, patients lack platelet granules
leading to impaired platelet aggregation (and therefore a bleeding diathesis). There is also a
lysosomal membrane defect which leads to the accumulation of ceroid lipofuscion in macrophages
within the lung (leading to pulmonary fibrosis), gastrointestinal tract (leading to granulomatous
colitis), and heart (cardiomyopathy).

372- Hypoplasia of the breast can be seen in which disease?

A. Anhidrotic ectodermal dysplasia


B. Maffucci syndrome
C. Congenital syphilis

372
D. Marfan syndrome
E. Osteogenesis imperfecta

►A

Anhidrotic ectodermal dysplasia is a X-linked recessive disease caused by mutations in


ectodysplasin, a member of the tumor necrosis family. Patients may have dry skin with
pigmentation periorbitally, hypohidrosis, sparse hair, hypo-anodontia, nail dystrophy, and frontal
bossing, and saddle nose deformity. In addition to abnormalities of other ectodermally derived
structures, the breast and nipple-areolar complex may be absent or hypoplastic.

373 -Which of the following disorders is associated with delayed separation of the umbilical cord?

A. Leukocyte adhesion deficiency type 1 (LAD-1)


B. Immunedysregulation, polyendocrinopathy, enteropathy, x-linked (IPEX)
C. Severe combined immunodeficiency disorder(SCID)
D. X-linked agammaglobulinemia
E. Myeloperoxidase deficiency

►A

LAD-1 manifests as a B2-integrin deficiency and often times presents as at birth with a delayed
umbilical cord separation.

374- Eyelid string of pearls are seen in which of the following conditions?

A. Focal dermal hypoplasia


B. Lipoid proteinosis
C. Hutchinson-Gilford syndrome
D. Beare-Stevenson cutis gyrata syndrome
E. Gaucher‘s disease

►B

373
Lipoid proteinosis is an autosomal recessive condition characterized by yellow papules on the face
and oropharynx, eyelid string of pearls, hoarse voice, verrucous nodules of elbows and knees, and
bean-shaped temporal and hippocampal calcification with occasional seizures. Histologically, there
are PAS+ deposits in the affected tissue.

375- Which of the following conditions is inherited in an X-linked dominant (XD) manner?

A. Chodrodysplasia punctata
B. CHILD Syndrome
C. Focal dermal hypoplasia
D. All of the answers are correct
E. None of the answers are correct

►D

All of the syndromes listed are XD. Other XD syndromes are: Incontinentia pigmenti and Bazex
syndrome.

376 -Nevoid basal cell syndrome is in inherited condition with basal cell carcinoma. Patients with
this syndrome have dental findings of:

A. Odontogenic keratocysts
B. Destructive periodontitis
C. Hypodontia
D. Gingival fibromas
E. Retention of primary teeth

►A

Nevoid basal cell syndrome is an inherited condition with a defect in the PTCH gene. The dental
findings are odontogenic keratocysts.

374
377 -Which malignancy is associated with Cowden syndrome?

A. Colon cancer
B. Rhabdomyosarcoma
C. Lung cancer
D. Ovarian cancer
E. Basal cell cancer

►A

Cowden syndrome is a autosomal dominant syndome with tricholemmomas, oral mucosal papules,
acral keratotic papules, thyroid goiter, GI polyps, and fibrocystic breast disease. Malignant
associations include breast, thyroid follicular carcinoma, and colon adenocarcinoma-malignant
transformation of the polyps. They are also at increased risk for renal cell carcinoma and
endometiral carcinoma

378 -A patient that has clinical signs of xeroderma pigmentosum with a history of sun sensitivity
and skin cancers plus mental retardation, dwarfism, and gonadal hypoplasia is known as:

A. De-Sanctis Cachione syndrome


B. Cockayne syndrome
C. Tay's syndrome Louis-Barr syndrome
D. Xeroderma pigmentosum plus syndrome
E. Bloch Sulzberger syndrome

►A

A patient that has clinical signs of xeroderma pigmentosum with mental retardation, dwarfism
and gonadal hypoplasia is known as De-Santis-Cachione syndrome. Those that have xeroderma
pigmentosum have skin cancer at a young age and is from mutations in DNA excision repair
enzymes involved in UV induced thymidine dimers.

379 -Which vascular disorder is characterized by facial vascular malformation and ipsilateral
intracranial and retinal arteriovenous malformations(AVMs)?

375
A. Sturge-Weber syndome
B. PHACES
C. Encephalotrigeminal angiomatosis
D. Bonnet Dechaune Blanc syndrome
E. Von Lohuizenǁs disease

►D

Bonnet Bechaune Blanc syndome, also know as Wyburn-Mason syndrome, is characterized by a


facial vascular malformation and ipsilateral intracranial and retinal AVMs. Encephalotrigeminal
angiomatosis is another name for Sturge-Weber. Von Lohuizen's disease is another name for
cutis marmorata telangiectatica congenita.

380 -Which of the following syndromes is associated with tricholemmomas?

A. Birt-Hogg-Dube
B. Brooke-Spiegler
C. Bannayan-Riley-Ruvacalba
D. Basex
E. Rasmussen's

►C

Tricholemmomas are seen in Bannayan-Riley-Ruvacalba syndrome. This is an autosomal dominant


condition with macrocephaly, lipomas, hemangiomas, skeletal abnormalities, lymphangioma
circupscriptum, angiokeratomas, penile lentigines, acanthosis nigricans, and achrocrodons. There is
an increased incidence of breast, thyroid, and GI cancers. Tricholemmomas are also associated in
Cowden's syndrome.

381 -In a patient suspected of having multiple endocrine neoplasia type IIb, which lab test would be
appropriate?

A. Calcitonin
B. Glucagon

376
C. Parathyroid hormone
D. Calcium
E. Cortisol

►A

Multiple endocrine neoplasia (MEN) syndrome type Iib, also called multiple mucosal neuroma
syndrome is an autosomal dominant condition due to a defect in the RET protooncogene on
chromosome 10q11.2. This rare condition is associated with mucosal neuromas on the tongue
and lips, medullary thyroid carcinoma, pheochromocytoma, and gastrointestinal
ganglioneuromatosis.

In addition, patients can also present with a marfanoid habitus and facial dysmorphism. Mucosal
neuromas can be a dermatologic clue to the underlying diagnosis as these lesions appear during
early childhood and present as pink, pedunculated nodules. As the major cause of mortality in these
patients is medullary thyroid cancer, which nearly all patients will have by early adulthood,
aggressive screening, with serial calcitonin level, and prophylactic thyroidectomy are warranted.

382 -Erythematous keratotic plaques of KID Syndrome most commonly occur in which location?

A. Face
B. Neck
C. Chest
D. Back
E. Abdomen

►A

The erythematous keratotic plaques of KID Syndome occur on face, extremities > trunk. KID
Syndrome is characterized by keratitis, icthyosis, and deafness. It is inherited in an autosomal
dominant fashion and caused by a mutation in the GJB2 gene which encodes connexin 26.

383 -Which of the following is not classically associated with pheochromoctyoma?

377
A. Multiple Endocrine Neoplasia Type IIA
B. Multiple Endocrine Neoplasia Type IIB
C. Von-Hippel-Lindau Syndrome
D. Cobb Syndrome
E. Neurofibromatosis

►D

Cobb syndrome is a sporadic disease characterized by cutaneous vascular malformations


associated with malformations of the spinal cord.

384 -A 12 year-old boy with pits on his palms and lateral fingers may have:

A. Arsenic exposure
B. A hereditary keratoderma
C. A corynebacteria infection
D. An inherited cancer syndrome
E. Secondary syphilis

►D

Basal cell nevus syndrome is an autosomal dominant disease caused by mutations in the PTCH1
gene. Clinically, patients may have numerous basal cell carcinomas, palmoplantar pits, jaw cysts,
frontal bossing, bifid ribs, calcification of falx cerebri, medulloblastoma, ovarian fibromas and
fibrosarcomas.

385- Cronkhite-Canada is associated with gastrointestinal polyposis, nail atrophy, alopecia,


generalized pigmentation of skin and melanotic macules on the fingers. Which of the following
describes its inheritance pattern?

A. Autosomal recessive
B. Autosomal dominant
C. X-linked recessive
D. X-linked dominant
E. Sporadic

378
►E

Cronkhite-Canada is associated with gastrointestinal polyposis, nail atrophy, alopecia, generalized


pigmentation of skin and melanotic macules on the fingers. It is inherited in a sporadic manner.

386 -Patients with junctional epidermolysis bullosa have been found to have mutations in:

A. Laminin 5
B. Bullous pemphigoid antigen 2
C. Collagen 17
D. BP180
E. All of the answers are correct

►E

All of the answers are correct. Laminin 5 is a protein integral in the adhesion of the dermis to the
epidermis. Also involved in junctional epidermolysis bullosa is bullous pemphigoid antigen 2,
collagen 17 and BP180, which are synonymous for the same structure.

387- Which syndrome is characterized by hyperhidrosis, lack of pain sensation, hypersalivation,


and absent fungiform papillae?

A. Turner Syndrome
B. Noonan Syndrome
C. Riley-Day
D. Rubinstein-Taybi syndrome
E. Cornelia de lange Syndrome

►C

Riley-Day syndrome is also known as Familial Dysautonomia. It is an autosomal recessive disorder


with the gene defect on the long arm of chromosome 9. Patients have unmyelinated sensory and
sympathetic neurons and autonomic dysfunction, leading to hyperhidrosis, decreased corneal

379
sensation and tear flow, hypersalivation, gastroesophageal reflux, decreased deep tendon reflexes,
and lack of pain sensation. They also exhibit abnormal histamine skin test.

388 -Low-cystine content in hair and nails may contribute to the phenotype seen in:

A. Wilson‘s disease
B. Menke‘s Kinky Hair syndrome
C. Tay Syndrome
D. Nethertons
E. Bjornstad

►C

Tay Syndrome is also known as trichothiodystrophy, or (P)IBIDS: (photosensitivity), icthyosis,


brittle hair, intellectual impairment, decreased fertility, and short stature. Hair shaft has a
characteristic ―tiger tailǁ appearance under polarized light and the low cystine content in hair and
nails is thought to be responsible for the phenotype seen.

389 -Coast of Maineǁ cafe au lait macules are characteristic of which condition

A. Hypomelanosis of Ito
B. Carney complex
C. McCune-Albright syndrome
D. Gaucherǁs syndrome
E. Tuberous sclerosis

►C

McCune-Albright syndrome is a sporadic condition caused by somatic mutations in the Gs subunit


of adenylate cyclase. Key clinical features include ―coast of Maineǁ cafe au lait macules,
polyostotic fibrous dysplasia, and precocious puberty.

380
390- The best screening test for hemochromatosis is:

A. Ferritin
B. Hematocrit
C. Complete Blood Count
D. Transferrin
E. Copper levels

►A

The ferritin is the best screening test for hemochromatosis, an autosomal recessive disease of
increased intestinal iron absorption. The other tests are not the best method for screening.

391- A patient has Cornelia de Lange syndrome have a low set hair line, trichomegaly, hirsutism
and synophrys. The skin finding is:

A. Cutis marmorata
B. Vitiligo
C. Poikiloderma
D. Scleroderma
E. Hyperpigmentation

►A

The patient has cutis marmorata with low set hair line, trichomegaly, synophrys, hirsutism, and
heart defect. The associated gene is NIPBL and SMC1L1.

392 -Which of the following syndromes is associated with markedly increased IgE levels, cold
abscesses and a characteristic coarse facies?

A. Wiskott-Aldrich syndrome
B. Chronic granulomatous disease
C. Job syndrome
D. Severe combined immunodeficiency

381
E. Leiner's disease

►C

Job syndrome or Hyper IgE syndrome is characterized by these findings. In addition, there is a
peripheral eosinophilia, eczematous dermatitis, frequent bronchitis and pneumonia, otitis media and
sinusitis. The other listed conditions are associated with immunodeficiency. Wiskott-Aldrich can
have eczematous dermatitis and all of these syndromes will have abnormal infections. They do not
have markedly increased levels of IgE like Job syndrome.

393- A 16 year-old girl presents with a family history of Gardner syndrome. Her mother is very
concerned that her daughter may have the syndrome as it runs in her family and she has many skin
complaints. What is likelihood that this girl has Gardner syndrome based on what you now about
the inheritance pattern and the fact that her father is unaffected and her mother is a heterozygote for
this condition?

A. 50%
B. 25%
C. 10%
D. 75%
E. 90%

►A

Her mother is a heterozygote for the condition and her father is homozygous as a non -carrier.
Gardner syndrome is passed in an autosomal dominant fashion. She has a 50% chance of also being
a heterozygote from her mother and a 50% chance of being completely unaffected.

394- An infant with a very hoarse cry presents to dermatology. His parents have noted that normal
scratches are slow to heal and leave broad scars. White and yellow lesions have been noted on the
lips, eyelids, and buccal mucosa. The child has had frequent upper respiratory tract infections. What
defective gene is associated with the most likely implicated disease?

A. Extracellular matrix protein-1

382
B. Collagen III
C. Elastin
D. Tenascin-X
E. Fibrillin-1

►A

The child described most likely has lipoid proteinosis, a rare inherited disease of connective tissues
that is inherited in an autosomal recessive manner. The implicated defective protein is extracellular
matrix protein-1. Collagen III and Tenascin-X defects are seen in Ehlers-Danlos syndrome. Elastin
defects are seen in cutis laxa. Fibrillin-1 defects are seen in Marfan disease.

395- Which disease is found more commonly in mothers of patients with chronic granulomatous
disease?

A. Sarcoidosis
B. Erythema nodosum
C. Churg-Straus disease
D. Wegener's disease
E. Discoid lupus erythematous

►E

Female carriers of chronic granulomatous disease have an increase incidence of discoid lupus,
infections and apthous stomatitis.

396- Menkes kinky hair syndrome is associated most commonly with which of the following hair
abnormalities?

A. Pili torti
B. Trichorrhexis nodosa
C. Trichorrhexis invaginata
D. Pili triangulati et canaliculati
E. None of these options are correct

383
►A

While pili torti is not exclusively found in Menkes kinky hair syndrome, this is the most common
hair abnormality found. Trichorrhexis nodosa can be also seen. Trichorrhexis invaginata is
commonly found in Netherton syndrome. Pili triangulati et canaliculati is the finding seen in
Uncombable hair syndrome.

397- This syndrome has glaucomas, choroid angiomas, anisometropic amblyopia with facial
capillary malformation and skeletal hypertrophy:

A. Sturge Weber syndrome


B. Osler Weber Rendu
C. Vogt-Koyanagi Harada syndrome
D. CHIME syndrome
E. Kid syndrome

►A

Sturge-weber syndrome has defects in facial capillary malformation with underlying soft tissue and
skeletal hypertorphy, ipsilateral arteriovenous malformation (AV), cerebral calcification,
hemiparesis, hemianopia, and contralateral seizures.

398- Findings of milia, cylindromas and the condition shown in the pathology image are
characteristic of which of the following syndromes?

A. Gorlin's syndrome
B. Familial cylindromatosis
C. Brook-Spiegler syndrome
D. Rasmusen syndrome
E. Rombo syndrome

►C

384
Brooke-Spiegler syndrome is an uncommon disease with a predisposition to develop cutaneous
adnexal neoplasms such as cylindromas, trichoepitheliomas, spiradenomas, trichoblastomas, basal-
cell carcinomas, follicular cysts, organoid nevi, and malignant transformation of preexisting tumors
in the affected individuals.

399 -Which malignancy is seen in approximately 15-20% of people with the disease characterized
by a defect in isocitrate dehydrogenase?

A. Osteosarcoma
B. Angiosarcoma
C. Chondrosarcoma
D. Rhabdomyosarcoma
E. Epitheliod sarcoma

►C

Approximately 15-20% of patients with Maffucci's syndrome and Oillier's will develop
chondrosarcoma. They are due to a defect in a isocitrate dehydrogenase 1 or 2.

400 -A patient with Ref sum syndrome has ocular anomalies such as nystagmus and night blindness
has a deficiency in:

A. Phytanic acid oxidase deficiency


B. Abnormal transglutaminase
C. Abnormal steroid sulfatase
D. Fatty alcohol oxidoreductas deficiency
E. Defects in endoglin

►A

Patients with Refusum syndrome have a deficiency in phytanic acid oxidase deficiency. The ocular
abnormalities seen are pigmentary retinopathy, cataracts, nystagmus, and night blindness

385
401- A patient has many reticulated, yellow patches in the skin folds and lax skin. The associated
eye abnormality with this disease is:

A. Angioid streaks
B. Papillary edema
C. Conglobata
D. Dyschromia
E. Loss of peripheral vision

►A

The patient has pseudoxanthoma elasticum which is due to a mutation in an adenosine triphosphate
binding transporter protein. The eye abnormality seen is angioid streaks on the retina, hemorrhages
and loss of vision. They also have claudicaton and gastrointestinal hemorrhage.

402- A deficiency in sialophorin, a surface glycoprotein, is thought to play a role in which genetic
disorder?

A. Wiskott-aldrich syndrome
B. Chediak-higashi syndrome
C. Werner syndrome
D. Bloom Syndrome
E. Cornelia de Lange syndrome

►A

Wiskott-Aldrich syndrome is an x-liked recessive disorder localized to Xp11.3. The gene involved
codes for WAS, and the protein product has been implicated in lymphocyte and megakaryocyte
signal transduction.

403- What is the most likely syndrome that this woman has in this photograph?

A. A. Klippel Trenaunay Weber


B. Filariasis
C. Neurofibromatosis

386
D. Proteus syndrome
E. Turner syndrome

►A

Klippel-Trenaunay Weber syndrome is characterized by a triad of port -wine stain, varicose veins,
and bony and soft tissue hypertrophy involving an extremity. Filariasis is an acquired infection that
leads to obstruction and scarring of the lymphatics. Neurofibromatosis is associated with
neurofibromas, cafe-au-lait macules and Lisch nodules, but not with lymphedema and vascular
malformations. Proteus syndrome is associated with vascular abnormalities, lipomas and
asymmetric limb hyperplasia, and plantar connective tissue nevi, but usually not lymphedema.
Lymphedema of the dorsal hands and feet can be seen in Turner syndrome, but typically not this
extensive.

404 -Reticulate pigmentation of skin, poikiloderma, alopecia, nail atrophy, premalignant oral
leukoplakia, and a Fanconi-type pancytopenia resulting in early death in addition to posterior fossa
malformations is characteristic of which of the following syndromes.

A. Dyskeratosis congenita
B. Hoyeraal-Hreidarsson syndrome
C. Bloom syndrome
D. Cockayne syndrome
E. Wiskott-Aldrich syndrome

►B

Hoyeraal-Hreidarsson syndrome is has all of the features of dyskeratosis congenita plus posterior
fossa malformations. Bloom syndrome and Cockayne syndrome both have poikiloderma as
features, but do not include posterior fossa malformations as part of the syndrome. WiskottAldrich
syndrome does not include any of these findings.

405 -A patient presents with bilateral retinal hemangioblastomas and a capillary malformation on
her neck. Which gene mutation is most likely?

387
A. VHL
B. Endoglin
C. PTH/PTHrP type I receptor
D. VEGF receptor-3
E. MFH1

►A

These findings are most characteristic of Von Hippel-Lindau syndrome. VHL is a tumor suppressor
gene which is mutated in this syndrome. Other findings include: renal and pancreatic cysts/
carcinoma, pheochromocytoma, and cerebellar/other CNS hemangioblastomas. Endoglin is
defective in Osler-Weber-Rendu, PTH/PTHrP type I receptor in Maffucci syndrome, VEGF
receptor-3 in Nonne-Milroy disease (hereditary lymphedema) and MFH1 in lymphedema and ptosis
syndrome.

406 -Sphenoid wing dysplasia is seen in:

A. Tuberous sclerosis
B. Tay Syndrome
C. Mafucci syndrome
D. NF-1
E. NF-2

►D

Sphenoid wing dysplasia is seen in neurofibromatosis type I. Patients with Tay syndrome have short
stature, patients with tuberous sclerosis have phalangeal cysts and periosteal thickening, patients
with Mafucci syndrome have enchondromas and short stature, and patients with NF-2 do not have
any characteristic musculoskeletal findings.

407- Dermatofibrosis lenticularis disseminata and osteopoikilosis are findings seen with mutations
of which of the following genes?

A. LEMD3
B. Fibrillin 2

388
C. ABCC6
D. Lysyl hydroxylase
E. Lysyl oxidase

►A

Buschke-Ollendorf syndrome is caused by a loss-of-function mutation in LEMD3.

408 -Which of the following findings is characteristic of a mutation in lamin A?

A. Lipoatrophic sclerodermoid skin


B. Alopecia
C. Craniomegaly with small face
D. Severe premature atherosclerosis with early death
E. All of the answers are correct
►E

A mutation in Lamin A causes Progeria (Hutchinson-Gilford syndrome). Other findings include nail
atrophy and muscle/bone wasting. Presentation is in the first or second year of life. An increased
urine hyaluronic acid can be helpful in diagnosis.

409- What condition is associated with a mutation in CXCR4?

A. Ataxia telangiectasia
B. Leukocyte adhesion deficiency
C. Chronic granulomatous disease
D. SCID
E. WHIM syndrome

►E

Ataxia telangiectasia is associated with mutations in ATM gene. Leukocyte adhesion deficiency is
associated with mutations in the common chain of CD18 which is a beta-2 integrin family member,
FUCT1, or KINDLIN3. Chronic granulomatous disease is due to mutations in the NADPH oxidase.

389
SCID is a heterogenous group of genetic disorders due to mutations in Jak-3, IL-7Ralpha, CD45,
CD3delta/CD3episilon, RAG1/RAG2, and Artemis (DCLREC1C). W HIM syndrome (warts,
hypogammaglobulinemia, infections, myelokathexis) is due to a gain-offunction mutation in
CXCR4 which results in the retention of neutrophils in the bone marrow. It is inherited in an
autosomal-dominant fashion.

410- What nail change is seen in patients with Mal de Meleda Syndrome?

A. Onycholysis
B. Longitudinal ridging
C. Koilonychia
D. leukonychia
E. pterygium

►C

Mal de Meleda is an autosomal recessive disease characterized by transgedient malodorous PPK,


hyperhidrosis, keratotic plaques at knees and elbows, subungual hyperkeratosis, and koilonychia.
The gene defect is SLURP 1.

411 -A patient has a white forelock, depigmented patches on the central face, mid-extremities,
deafness and heterochromia irides. The patient has:

A. Piebaldism
B. Klippel Feil
C. Cornelia de Lange syndrome
D. Noonan syndrome
E. Turner syndrome

►A

A patient has Piebaldism with a mutation in c-KIT protooncogene with a white forelock. The patient
has depigmented patches on the central faces, mid-extremities, and abdomen. The patient can have
Hirschsprung, deafness and heterochromia irides.

390
412 -On cutaneous exam, angiokeratoma corporis diffusum is characteristic of which of the
following conditions?

A. Sialodosis
B. Fucosidosis
C. Fabry disease
D. All of these options are correct
E. None of these options are correct

►D

Findings of angiokeratoma corporis diffusum are found in all three listed conditions. They cannot
by distinguished by skin exam.

413 -A BSCL2 gene mutation with the cutaneous findings of generalized lipodystrophy,
hyperlipemia, hepatomegaly, acanthosis nigricans, elevated basal metabolic rate and non-ketotic
insulin resistant diabetes mellitus are characteristic of which of the following syndromes?

A. Berardinelli-Seip congenital lipodystrophy


B. Familial partial lipodystrophy
C. Bjornstad syndrome
D. All of the answers are correct
E. None of the answers are correct

►A

Berardinelli-Seip congenital lipodystrophy is described above. Familial partial lipodystrophy is


characterized by a defect in LMNA and has symmetric lipoatrophy of trunk and limbs with sparing
of neck, shoulders, buffalo hump area and genitalia, tuboeruptive xanthomas, acanthosis nigricans
and hypertriglyceridemia. Bjornstad syndrome is characterized by pili torti and deafness.

414- What is the most likely nail findings in a patient who has this autosomal dominant disease with
these keratotic papules and cobblestoning of the oral mucosa?

391
A. Koilonychia
B. Red and white longitudinal band
C. Melanonychia
D. Half and half nails
E. Pincer nails

►B

Darier's disease is an autosomal dominant disorder characterized by greasy hyperkeratotic papules.


The papules often coalesce into a warty plaque and have a tendency for secondary viral or bacterial
infection. The mutation is in calcium ATPase 2A2. The classic nail finding is red and white
longitudinal bands with V-shaped nicking.

415 -Tyrosinase positive albinism (oculocutaneous albinism type 2) is caused by a mutation in


which of the following:

A. Tyrosinase
B. P gene
C. Tyrosinase related protein 1
D. C-kit
E. NEMO

►B

Oculocutaneous albinism (OCA) type 1 (Tyrosinase negative albinism) is caused by mutations in


the tyrosinase gene. OCA type 2 (tyrosinase positive albinism) is caused by mutations in the P gene.
OCA type 3 is caused by mutations in the tyrosinase related protein 1 gene. C -kit mutations cause
piebaldism and NEMO mutations cause incontinentia pigmenti.

416- Maffucci syndrome is has characteristic venous malformations of the distal extremities and
benign endochondromas which can compromise bone strength and lead to chondrosarcomas. The
defect causing this believed to be the IDH-1 or IDH-2 which is inherited in which manner?

A. Sporadic

392
B. Autosomal dominant
C. Autosomal recessive
D. X-linked dominant
E. X-linked recessive

►A

Maffucci syndrome is inherited in a sporadic manner, with mutations in isocitrate dehydrogenase 1


or 2 (IDH-1, IDH-2).

417 -Which of the following genodermatoses has premalignant oral leukoplakia as a feature?

A. Pachyonychia congenita
B. Dyskeratosis congenita
C. Keratosis follicularis
D. Anhidrotic ectodermal dysplasia
E. Papillon-Lefevre

►B

Dyskeratosis congenita has premalignant oral leukoplakia in addition to cutaneous poikiloderma


and nail dystrophy. Pachyonychia congenita also has leukoplakia, but it is benign in nature.
Additional features of pachyonychia congenita are thickened nails with nail bed hyperkeratosis and
palmoplantar keratoderma. Keratosis follicularis, also known as Darier\'s disease, is characterized
by benign whitish papules on the oral mucosa, cutaneous keratotic papules in seborrheic
distribution, and longitudinal erythronychia with V-nicking of the distal nail plate. Papillon-Lefevre
has gingivitis, without oral leukoplakia, and acral hyperkeratosis. Anhidrotic ectodermal dysplasia
does not have leukoplakia. It consists of anhidrosis or hypohidrosis, hypotrichosis, and anodontia.

418 -What of the following is present in desmoplastic melanoma?

A. b-raf
B. c-myc
C. c-kit

393
D. Ras
E. p53

►C

C-kit staining is present in desmoplastic melanoma. It is also seen in acral melanocytic lesions. Ras
is found in all melanocytic lesions. P53 mutations are found in actinic keratoses, SCCIS, and SCC.

419- Comma-shaped corneal opacities are characteristic of which type of ichthyosis?

A. Ichthyosis vulgaris
B. X-linked ichthyosis
C. Lamellar ichthyosis
D. Nonbullous congenital ichthyosiform erythroderma
E. Refsum syndrome

►B

X-linked ichthyosis patients have comma-shaped corneal opacities that are asymptomatic yet highly
characteristic.

420 -"Mousy" odor of urine is characteristic of which of the following syndromes?

A. Phenylketonuria
B. Hunter disease
C. Alkaptonuria
D. Maple Syrup Urine disease
E. Hurler disease

►A

394
Phenylketonuria will cause a "mousy" odor in the urine. Patients with Alkaptonuria will have black
urine. Maple syrup urine disease will have a sweet odor. Hunter/Hurler diseases are not associated
with urinary issues.

421- Dental enamel pits are seen in which of the following conditions?

A. Hypomelanosis of Ito
B. Letterer-Siwe disease
C. Tuberous sclerosis
D. Jackson Sertoli syndrome
E. Hyper-IgE syndrome

►C

Tuberous sclerosis is an autosomal dominant condition caused by mutations of the TSC1 (hamartin)
or TSC2 (tuberin) genes. These are tumor suppressor genes. Skin findings include hypopigmented
macules, connective tissue nevus, facial angiofibromas, periungual fibromas and cafe au lait
macules. Dental enamel pits and gingival fibromas are oral findings that are associated with this
condition. The remaining conditions do not have dental pits.

422- The finding of 'maltese crosses' in the urine is characteristic of which of the following
conditions?

A. Alkaptonuria
B. Fabry disease
C. Gaucher disease
D. Neimann-Pick disease
E. Hunter syndrome

►B

The 'maltese cross' finding in urine is characteristic of Fabry disease. Alkaptonuria will show dark
urine with a pH > 7.0. There are no urinary findings in Hunter syndrome, Gaucher or Neimann-Pick
disease.

395
423 -An infant with doughy, redundant skin and short sparse hairs is likely to show which features
on x-ray?

A. Metaphyseal widening in the long bones


B. Sphenoid wing dysplasia
C. Periosteal thickening
D. Osteopoikilosis
E. Stippled epiphyses

►A

The patient described has Menkes Kinky Hair syndrome, an X-linked recessive disease due to a
defect in an intestinal copper transport protein. Clinical features include pili torti, short, brittle
steel-woolǁ• hair, and spare eyelashes and sparse broken eyebrows. The skin is often hypopigmented
with a soft, ―doughyǁ• consistency and redundancy. Musculoskeletal manifestations include
metaphyseal widening with spurs in the long bones

424- Which cutaneous finding is seen in patients with phenylketonuria?

A. Angular stomatitis
B. Ichthyosis
C. Pigment dilution of hair and skin
D. Phyrnoderma
E. Erosive diaper dermatitis

►C

Phenylketonuria is an autosomal recessive condition caused by a mutation in the gene coding for
phenylalanine hydroxylase. Defect in this enzyme results in accumulation of phenylalanine and its
metabolites. Increased phenylalanine has toxic effects on the central nervous system in addition to
competitively inhibiting tyrosine in melanogenesis. Inhibition of melanogenesis results in
pigmentary dilution of the hair and skin. Other features of this condition include a predisposition to
eczema, sclerodermoid changes of the skin, urine that has a distinctive "mousy" odor, psychomotor
delay, mental retardation, seizures and hyperreflexia. A low-phenylalanine diet instituted early on
can prevent these manifestations of the disease. The morbidity of phenylketonuria has improved
since the advent of routine neonatal screening for this condition.

396
425 -Epidermolysis bullosa simplex is caused by blistering in which structure?

A. Granular layer keratinocyte


B. Spinous layer keratinocyte
C. Basal layer keratinocyte
D. Lamina densa
E. Sublamina densa

►C

Epidermolysis bullosa simplex is caused by mutations in keratin 5 and 14 resulting in bullae within
basal cell keratinocytes.

426 -A patient diagnosed with KID syndrome with keratitis-icthyosis-deafness has a mutation in:

A. Connexin 26
B. NEMO
C. DNA helicase
D. RECQL4
E. plakoglobin

►A

A patient with KID syndrome has an autosomal dominant disease and mutation in connexin 26.
They have keratitis, ichthyosis, deafness and alopecia.

427- Which type of epidermolysis bullosa simplex is associated with early death?

A. Weber-Cockayne
B. Generalized (Koebner)
C. Dowling-Maera
D. Ogna variant
E. Non-Herlitz variant

397
►C

The Dowling-Maera variant of epidermolysis bullosa simplex is associated with widespread bullae,
significant mucous membrane and laryngeal/esophageal involvement, nail dystrophy, and early
death.

428- A patient presents with several light blue cyst-like lesions on the eyelid. They consult their list
of problems and bring up plantar hyperkeratosis and dysplastic toenails. On oral exam, you note
that they have both upper and lower dentures. The patient relates that after losing their "baby teeth",
only 3 teeth grew in their place. What syndrome does this person most likely have?

A. Schopf-Schulz-Passarge
B. Gardner syndrome
C. Hypohidrotic ectodermal dysplasia
D. Cowden syndrome
E. Cronkhite-Canada

►A

Schopf-Schulz-Passarge syndrome is associated with hydrocystomas of the eyelids, hypotrichosis


(near complete loss of hair early in life), hypodontia, nail abnormalities and multiple palmoplantar
eccrine syringofibroadenomas. The other listed syndromes do not fit the description above.

429- Patients with Russell-Silver syndrome exhibit:

A. Clinodactyly of fifth finger


B. Peg teeth
C. Osteopathia striata
D. Broad thumbs
E. Shortened 4th and 5th metacarpals

►A

Characteristic features of Russell-Silver include short stature, bony asymmetry, triangular facies,

398
clinodactyly of fifth finger, and precocious sexual development with cryptochordism/hypospadias.
Broad thumbs are seen in Rubinstein-Taybi and shortened fourth and fifth metacarpals are seen in
Turner syndrome. Osteopathia striata is characteristic of focal dermal hypoplasia.

430 -Which syndrome is characterized by broad thumbs, a large beaked nose, and capillary
malformation?

A. Klinefelter
B. Proteus syndrome
C. Bloom syndrome
D. Rubinstein-Taybi
E. Ehlers-Danlos syndrome

►D

Rubinstein-Taybi syndrome has been associated with a deletion localized to the short arm of
chromosome 16. Patients are severely retarded with strabismus, crytorchidism, and congenital heart
defects. They have a characteristic beaked nose with nasal septum below alae accompanied by a
broad nasal bridge, downslanting palpebral fissures, and broad thumbs and halluces.

431 -Which finding would you expect in a child with dyskeratosis congenita?

A. Normal mucosa
B. Oral leukoplakia
C. Hirsutism
D. Normal hematologic status
E. Normal nails

►B

Mild to moderate mental retardation occurs in up to 50% of cases. A Fanconi -type pancytopenia
may occur resulting in early death. These children have alopecia, not hirsutism, along with n ail
dystrophy. There is an increased risk for development of premalignant oral leukoplakia, as well
as many cancers.

399
432- A 12 year old boy presents complaining of pain and stinging of his skin within minutes of
going out to play in the park. Examination reveals rare milia of the hands and posterior neck. A
diagnosis of porphyria is made. What is the most likely defective gene in this patient?

A. Ferrochelatase
B. Uroporphyrinogen decarboxylase
C. Porphybilinogen deaminase
D. Uroporphyrinogen III synthase
E. Protoporphyrinogen oxidase

►A

Erythropoietic protoporphyria is a relatively common form of porphyria, characterized by


photosensitivity. Over time, superficial waxy scars and milia are identified in sun-exposed areas.
The defective gene is ferrochelatase. Uroporphyrinogen decarboxylase is mutated in porphyria
cutanea tarda. Porphybilinogen deaminase is defective in acute intermittent porphyria.
Uroporphyrinogen III synthase is defective in congenital erythropoietic porphyria.
Protoporphyrinogen oxidase is mutated in variegate porphyria.

400
401
General Derma logy

1- A 32-year-old patient from Iraq presents with a new oral ulceration. He has had several oral
ulcerations before that resolved on their own. Upon further physical examination, you notice he also
has redness of both eyes and tender nodules on both shins. Cultures have not grown any organisms
to date. A biopsy shows lymphocytic vasculitis with a neutrophilic angiocentric infiltrate. Which of
the following is the most likely diagnosis?

A. Sweet's syndrome
B. Familial Mediterranean fever
C. Pyoderma gangrenosum
D. Lofgren's syndrome
E. Behcet's disease

Correct choice: E. Behcet's disease

Explanation: Behcet’s disease is a multisystem polysymptomatic disease with cutaneous findings


ranging from sterile papulopustules and palpable purpura to erythema nodosum-like lesions. The
diagnosis is based on International Study Group criteria that includes: recurrent oral ulceration,
recurrent genital ulceration, ocular abnormalities (uveitis, retinal vasculitis), and skin lesions. On a
skin biopsy, the characteristic finding is a neutrophilic angiocentric infiltrate with leukocytoclastic
(early) or lymphocytic (late) vasculitis.

1 – Sweet’s syndrome is histologically characterized by superficial dermal edema, diffuse dermal


neutrophils and traditionally no true vasculitis (no vessel wall necrosis), but nuclear dust is
common. 2 – Familial Mediterranean fever is an autosomal recessive inherited disorder
characterized by recurrent febrile episodes lasting for 1-2 days, with painful self-limiting episodes
of pleuritis, peritonitis or synovitis. Cutaneous manifestations include erysipelas-like erythema over
the joints, lower legs and dorsal feet in addition to small vessel vasculitis and nonspecific purpura. 3
– Pyoderma gangrenosum is characterized by a necrotic epidermis and ulceration, occasionally with
pustules and sometimes pseudoepitheliomatous hyperplasia at the edge of the ulcer. It also has a
diffuse infiltrate of neutrophils, lymphocytes, and histiocytes in the dermis, and occasionally has
vasculitis. 4 – Lofgren’s syndrome is a more acute yet transient form of sarcoidosis with erythema
nodosum, hilar adenopathy, fever, polyarthritis and iritis. Sarcoidosis is characterized by non-
caseating, well-demarcated granulomas in the dermis or subcutaneous tissue.

402
to
2- A 42-year-old male presents with multiple round to oval hypomelanotic patches measuring
several centimeters in size. The lesions coalesce centrally and are more prominent on the trunk.
Associated fine scale is apparent upon scratching the skin. The distribution is symmetric and favors
the chest, abdomen and back. Which of the following is not a proposed mechanism of the hypo
pigmented skin in tinea versicolor?

A. Azelaic acid inhibits tyrosinase produced by Malassezia furfur


B. Abnormal melanosome production
C. Decreased melanin synthesis
D. Partial block in melanosome transfer to keratinocytes
E. Kojic acid plays a role in decreased melanin synthesis

Correct choice: E. Kojic acid plays a role in decreased melanin synthesis

Explanation: Kojic acid does not play a role in decreased melanin synthesis. In the hypopigmented
skin of tinea versicolor, there is a decreased density of melanosomes within keratinocytes, but no
change in the melanocyte density. Abnormal melanosome production, decreased melanin synthesis,
and a partial block in melanosome transfer to keratinocytes have all been suggested as underlying
defects. Azelaic acid, a competitive inhibitor of tyrosinase produced by Malassezia furfur, plays a
role in the decreased melanin synthesis. Although the differential diagnosis may include
postinflammatory hypopigmentation (e.g. secondary to parapsoriasis), progressive macular
hypomelanosis, and early vitiligo, the diagnosis of tinea versicolor is easily confirmed by
examination of the associated scale in a potassium hydroxide preparation.

1 - Azelaic acid inhibits tyrosinase produced by Malassezia furfur: Azelaic acid is a competitive
inhibitor of tyrosinaes produced by Malassezia furfur and plays a role in decreased melanin
synthesis. 2 - Abnormal melanosome production: Abnormal melanosome production, decreased
melanin synthesis and a partial block in melanosome transfer to keratinocytes have all been
suggested as underlying defects in tinea versicolor.

403
3 - Decreased melanin synthesis: Abnormal melanosome production, decreased melanin synthesis
and a partial block in melanosome transfer to keratinocytes have all been suggested as underlying
defects in tinea versicolor.

4 - Partial block in melanosome transfer to keratinocytes: Abnormal melanosome production,


decreased melanin synthesis and a partial block in melanosome transfer to keratinocytes have all
been suggested as underlying defects in tinea versicolor.

3- Which of the following is TRUE?

A. Malignant transformation occurs in 2-13%


B. These are pathognomonic of Von Recklinghausen disease
C. They are composed of a mixture of the neuromesenchyme (Schwann cells, endoneurial
fibroblasts, perineurial cells)
D. Relative ratio of axons to Schwann cells in these tumors is always >1:1
E. These commonly display overlying hypertrichosis or hyperpigmentation

Correct choice: C. They are composed of a mixture of the neuromesenchyme (Schwann cells,
endoneurial fibroblasts, perineurial cells)

Explanation: C is correct and the photograph displays a neurofibroma. A, B, D, & E all describe
plexiform neurofibromas. For answer D, the ratio of axons to Schwann cells is always <1:1, since
axons do not replicate but Schwann cells can.

4- A 37 week primigravid female presents with pruritic rash and blisters involving the abdomen
and extremities. What is the pathognomonic cell type on H&E?

404
A. Neutrophils
B. Eosinophils
C. Lymphocytes
D. Mast cells
E. Plasma cells

Correct choice: B. Eosinophils

Explanation: Eosinophils are the predominant cells see in histology of pemphigoid gestationis.

5- A 42-year-old female with keratosis follicularis has a severe disease flare after starting a new
medication. She has a history of epilepsy, bipolar disorder and gout. What is the most likely new
treatment that induced the flare?

A. Phenytoin
B. Valproate
C. Carbamazepine
D. Allopurinol
E. Lithium

Correct choice: E. Lithium

Explanation: Keratosis follicularis (or Dariers disease) is known to worsen with initiation of certain
medications including lithium. The other medications listed are not known to worsen keratosis
follicularis.

405
6- Dihydroxyacetone is found in which of the following products?

A. Rubber accelerators
B. Shampoos
C. Artificial nails
D. Hair dyes
E. Sunless tanning preparations

Correct choice: E. Sunless tanning preparations

Explanation: Dihydroxyacetone is the active ingredient in sunless tanning preparations. Upon


oxidation it turns orange-brown and binds to the stratum corneum. Rubber accelerators contain
mercaptobenthothiazole, carba mix, thiuram mix, or mercapto mix. Shampoos, especially "tear-
free" ones, may contain cocamidopropyl betaine. Artificial nails may contain methyl methacrylate
or ethyl acrylate. Hair dyes often have paraphenylenediamine.

7- What is the diagnosis:

A. Alopecia areata
B. Trichotillomania
C. Androgenetic alopecia
D. Tinea capitis
E. Temporal triangular alopecia

Correct choice: A. Alopecia areata

Explanation: Alopecia areata presents with discrete, round or oval patches of non-scarring hair loss.
Initial sparing of non-pigmented hair may occur leading patients to report rapid whitening of the
hair. Similarly, initial regrowth of hair may include white or grey hairs.

406
Trichotillomania is a an impulse disorder characteriszed by repetitive hair pulling. As a result,
varying lengths of hair can be seen within areas of alopecia. The mainstay of treatment is
behavioral modification.

Androgenetic alopecia is an androgen-dependent form of nonscarring alopecia. It affects men more


commonly than women and frequency increases with age. The characteristic pattern in men
includes frontalparietal recession and thinning of the vertex hair. In women, central thinning is
most commonly seen.

Traction alopecia is the result of chronic mechanical trauma to the frontal and bitemporal scalp.
Though initially nonscarring, with repetitive traction, hair loss can become permanent.

Temporal triangular alopecia is a form of congenital alopecia. Presenting at birth or in early


childhood, there is unilateral or bilateral nonscarring alopecia of the temporal area. The hair loss is
permanent.

8- You are evaluating a 44-year-old man who presents with a new rash. He takes off his shirt and
raises his arms to show you. In the left axilla, you see a hyperpigmented velvety plaque. Which one
of the following medical conditions should consider?

A. Diabetes mellitus
B. Kidney stones
C. Hepatitis B virus infection
D. Hypertension
E. Benign prostatic hyperplasia

Correct choice: A. Diabetes mellitus

Explanation: The patient likely has acanthosis nigricans associated with diabetes mellitus and
insulin resistance. Acanthosis nigricans is not associated with kidney stones, hepatitis B virus
infection, hypertension, or benign prostatic hyperplasia.

9- A 15-year-old boy presents with keratotic follicular papules involving the bilateral dorsal arms,
thighs, and cheeks. Analysis of an unaffected region of skin would likely exhibit which of the
following findings:

407
A. Diminished filaggrin
B. Diminished loricrin
C. Diminished involucrin
D. Diminished lamellar bodies
E. Decreased Transglutaminase I activity

Correct choice: A. Diminished filaggrin

Explanation: Filaggrin mutations are most commonly associated with atopic dermatitis, but have
also been implicated in keratosis pilaris as described in this question. Lamellar bodies are
diminished in Flegel’s disease and absent in Harlequin icthyosis. Diminished tissue
transglutaminase I activity is associated with lamellar icthyosis and non-bullous congenital
icthyosiform erythroderma. The other findings are not classically associated with keratosis pilaris.

10- What is the most likely diagnosis?

A. Mixed Cryoglobulinemia
B. Cold Panniculitis
C. Calciphylaxis
D. Homozygous Protein C deficiency
E. Sneddon Syndrome

Correct choice: C. Calciphylaxis

Explanation: An obese patient with dusky, retiform and stellate necrotic plaques predominantly on
the fatty areas of the body is most consistent with calciphylaxis. The differential diagnosis includes
warfarin necrosis. Oxalate or cholesterol emboli can in rare cases resemble calciphylaxis, but tend
to affect distal extremities as opposed to fatty, central areas.

408
Mixed cryoglobulinemia is a vasculitis, which manifests with palpable purpura as opposed to
retiform or stellate purpura and necrosis. Cold panniculitis is a subcutaneous disorder which
manifests with painful pink-red to violaceous plaques - this condition is seen most often in
newborns, but can affect adults and is usually seen on the outer thighs of adults (equestrian
panniculitis). Homozygous deficiency or severe dysfunction of either protein C or protein S leads to
neonatal purpura fulminans within a few hours to 5 days after birth, and it is fatal unless treated.
Sneddon syndrome is a vascular disorder which manifests with widespread livedo reticularis or
racemosa with CNS problems (stroke, TIA, seizures). There is no retiform purpura or necrosis
apparent.

11- A 21 year-old Mexican female presents to clinic with these asymptomatic skin findings (see
clinical images below) that she has had since she was 2 years old. Her biopsy results are shown in
the histopathology image below. Which of the following is true regarding the disorder?

A. The disorder can be inherited in an autosomal dominant fashion, but most cases are
sporadic.
B. This condition does not undergo malignant degeneration
C. The characteristic finding on histopathology is a keratin-filled epidermal invagination
with an angulated parakeratotic tier
D. Both A and C are correct
D. All of the above are correct

409
Correct choice: D. Both A and C are correct

Explanation: This patient has linear porokeratosis.


Porokeratosis is a disorder of keratinization characterized by hyperkeratotic plaques surrounded by
a peripheral ridge-like scale that expands centrifugally. The diagnosis is typically made clinically
but biopsy can help by showing the hallmark cornoid lamella which is a thin column of tightly
packed parakeratotic cells extending from an invagination through the adjacent stratum corneum
with an attenuated or absent granular layer.1 There are several recognized variants, the most
common are porokeratosis of Mibelli and disseminated superficial actinic porokeratosis, with less
common variants including linear porokeratosis, and punctate porokeratosis. Porokeratosis can be
inherited in an autosomal dominant fashion, but most cases are sporadic.

When managing patients with porokeratosis, it is important to remember there is an increased risk
for malignant transformation. Most at risk are those with the linear variant who have up to an 11%
lifetime risk of developing skin cancer in a lesion with a latency period of 3-4 decades after initial
diagnosis.2 Risk factors include longer duration, larger plaques and acral involvement. The most
common cancers are SCC and Bowen’s disease and there may be multiple types in the same
plaque.2

12- A patient presents for evaluation of a solitary painful lesion on the shin. She feels well and
denies any systemic symptoms. A recent fasting blood glucose was within normal limits. Which of
the following is the most likely diagnosis?

A. Pretibial myxedema
B. Morphea
C. Lipodermatosclerosis
D. Necrobiotic xanthogranuloma
E. Necrobiosis lipoidica

410
Correct choice: E. Necrobiosis lipoidica

Explanation: Concomitant diabetes has been reported in 11-65% of patients with necrobiosis
lipoidica (NL).
This question asks the examinee to identify NL (choice E) by a typical yellowish-brown plaque with
prominent atrophy and telangiectasia on the shin, recognizing that a history of diabetes is not
required to make the diagnosis. Once widely called necrobiosis lipoidica diabeticorum, NL is now
the preferred term for this condition, given that 11-65% of patients with NL have diabetes;
conversely, 0.3-1.2% of diabetics have NL. Pretibial myxedema (choice 1) presents on the shins
with waxy indurated plaques or nodules with a peau d’orange quality. Plaque-type morphea (choice
2) favors the trunk and proximal extremities with erythematous-violaceous patches that progress to
indurated, ivory white to hyperpigmented plaques with lilac borders. Lipodermatosclerosis (choice
3) manifests in the acute phase with painful erythematous plaques on the lower legs above the
malleoli, and chronically with sclerotic reddish-brown plaques resembling an inverted wine bottle.
Necrobiotic xanthogranuloma (NXG, choice 4) shows indurated yellowish papules, plaques, or
nodules with variable atrophy, telangiectasia, and ulceration that can cause scarring. While it can
morphologically resemble NL, NXG most commonly develops in the periorbital region rather than
on the shins.

13- Which of the following hormonal contraceptive methods are most effective in treating acne?

A. intrauterine device (IUD)


B. norgestimate
C. levonorgestrel
D. depot injection
E. drospirenone

Correct choice: E. drospirenone

Explanation: Drospirenone. While some hormonal contraceptive methods are used to treat acne,
some have been shown to worsen acne. Combined oral contraceptives (COC) and the vaginal ring
improved acne, whereas depot injections, subdermal implants, and hormonal intrauterine devices
worsened acne. Within the COC category, drospirenone was the most effective at treating acne.

411
14- A 59 year old female presents to the dermatology office with the complaint of a red, non-scaly,
asymptomatic rash of the lower extremities. She was on a 4-hour walking tour of Boston the day
before she noticed the rash. What is the most likely diagnosis?

A. Cutaneous T cell lymphoma


B. Allergic contact dermatitis
C. Lichen planus
D. Psoriasis
E. Pigmented purpuric dermatosis

Correct choice: E. Pigmented purpuric dermatosis

Explanation: The patient has pigmented purpuric dermatosis (PPD) representing with extravasated
red blood cells with a sharp demarcation at the sock line on her leg. PPD presents in several forms
with the most common form being Schamberg disease with red-to-orange cayenne-pepper macules
on the bilateral lower extremities in the setting of dependent edema. Treatment consists of leg
elevation, compression stockings, and topical steroids if pruritic. In addition, anti-oxidants by
mouth may improve the condition as well.

Cutaneous T cell lymphoma is on the differential diagnosis of PPD but given the acuity of the onset,
the history, and the sock line demarcation, PPD is more likely. Allergic contact dermatitis would be
a consideration but is much less likely given the lack of scale and lack of pruritus. The rash does not
appear like psoriasis, which is a condition with pink plaques with micaceous, thick scale. Lichen
planus presents as purple polygonal papules with Wickham striae classically seen on the wrists,
ankles, and mucosal sites - the rash in the picture does not represent this condition.

412
15- A 51-year-old man presents with a soft mass on the scalp. Which one of the following is true
concerning the evaluation and management of lipomas?

A. They are usually painful and tender on examination


B. They are usually associated with a malignant condition
C. They can be watched without surgical intervention and can be familial
D. They usually require surgical removal to prevent further complications
E. They are associated with Addison disease

Correct choice: C. They can be watched without surgical intervention and can be familial

Explanation: Lipomas are benign lesions and can be familial. Lipomas are benign and do not
require surgical intervention. They are not usually painful and are not associated with malignant
conditions or Addison disease.

16-worker presents at your occupational dermatology clinic with the development of a white finger
in response to cold, associated transient loss of sensation, and permanent finger neuropathy and pain
in the affected limb. He operates chainsaws and pneumatic tools. Which of the following is the most
likely diagnosis for this patient’s condition?

A. Raynaud phenomenon
B. Scleroderma
C. Mixed connective tissue disease
D. Hypothenar hammer syndrome
E. Vibration white finger

Correct choice: E. Vibration white finger

Explanation: Vibration white finger is a relatively frequent disorder among operators of chainsaws,
pneumatic tools, and hand grinders who work in cold climates. It is characterized by development
of a white finger or fingers in response to cold, associated transient loss of sensation, possible
permanent finger neuropathy and pain in the affected limb. It is associated with exposures to
vibrations between 30 and 300 Hz.

Raynaud phenomenon: In contrast to the symmetric distribution seen in Raynaud disease, in


vibration white finger the blanching is asymmetric and occurs only on those digits most exposed to

413
vibration. Hypothenar hammer syndrome: The hypothenar hammer syndrome results from
occlusion of the ulnar artery as a consequence of repeated trauma to the palms and may be
misdiagnosed as Raynaud phenomenon.

17- These lesions typically erupt in what condition?

A. Morphea
B. Leprosy
C. Erythema nodosum
D. Sarcoid
E. Pregnancy

Correct choice: E. Pregnancy

Explanation: This is a pyogenic granuloma. They can erupt in pregnancy.

18- Similar lesions are present on the contralateral leg, and this patient also reports joint pain and
tarry stools. A biopsy for direct immunofluorescence is most likely to show which of the following?

414
A. IgA deposition in the dermal papillae
B. Perivascular IgA deposition
C. Perivascular IgG deposition
D. Linear IgA deposition along the basement membrane zone
E. Linear IgG deposition along the basement membrane zone

Correct choice: B. Perivascular IgA deposition

Explanation: The associated image depicts the classic "palpable purpura" of cutaneous small-vessel
vasculitis (CSVV). A specific type of CSVV now known as IgA vasculitis (formerly Henoch-
Schonlein Purpura) typicallt presents with palpable purpura on the lower extremities and buttocks,
along with arthritis, hematuria, colicky abdominal pain +/- GI bleeding and vomiting. A biopsy for
H&E reveals leukocytoclastic vasculitis, and a biopsy for DIF shows perivascular IgA, C3, and
fibrin. Treatment is mainly supportive as it is typically self-limited. Dermatitis herpetiformis - IgA
deposition in the dermal papillae. Linear IgA bullous disease/chronic bullous disease of childhood -
Linear IgA deposition along the basement membrane zone. Bullous pemphigoid - Linear IgG (and
C3) deposition along the basement membrane zone.

19- This patient had significantly elevated serum CPK. The likely diagnosis is:

A. Dermatomyositis
B. Lupus erythematosus
C. Psoriasis
D. Atopic dermatitis
E. Lichen planus

415
Correct choice: A. Dermatomyositis

Explanation: The answer is dermatomyositis with the heliotrope color and distribution of erythema
or violaceous color. The skin over the metacarpal and proximal interphalangeal joints can become
inflamed and erythematous forming Gottron’s papules.

20- A patient has new-onset shortness of breath and the rash shown. She denies weakness or any
other systemic symptoms. Which of the following is the most likely diagnosis?

A. Limited systemic sclerosis


B. Diffuse systemic sclerosis
C. Dermatomyositis
D. Systemic lupus erythematosus
E. Sjögren’s syndrome

Correct choice: C. Dermatomyositis

Explanation: Dermatomyositis (DM) can present as clinically amyopathic disease featuring


characteristic skin changes and interstitial lung disease.

This question asks the examinee to identify DM (choice 3) presenting with pathognomonic
cutaneous findings without gross muscle involvement, but with shortness of breath concerning for
interstitial lung disease (ILD). The photo depicts ragged cuticles (Samitz sign), violaceous papules
overlying joints (Gottron’s papules), and a violaceous scaly plaque on the knee (Gottron’s sign).
Anti-MDA5 (CADM-140) is an antibody associated with clinically amyopathic DM with ILD,
which may be quickly progressive. While multiple rheumatologic conditions cause dyspnea by
various mechanisms, their skin findings will differ from those of DM. Limited and diffuse systemic
sclerosis (SS, choices 1 and 2) would show cutaneous sclerosis (confined to distal extremities and

416
face in limited SS; extending to proximal extremities and trunk in diffuse SS), Raynaud’s
phenomenon, and nailfold capillary changes. On the hands, systemic lupus erythematosus (choice
4) favors the phalanges and spares the joints, opposite to the pattern of DM. Sjögren’s syndrome
(choice 5) can manifest with xerosis, purpura, vasculitis, annular erythema, Sweet syndrome,
erythema nodosum, nodular amyloidosis, and other findings distinct from those of DM.

21- A 57 year-old-female is post-op day 2 from a knee replacement. Dermatology is consulted for
the dermatitis seen here. It is pruritic, and has not responded to 1% hydrocortisone. What is the
most likely etiology of the physical findings?

A. Allergic contact dermatitis to a soap or other topical application


B. Irritant contact dermatitis due to excessive rubbing during the cleansing process
C. Allergic contact dermatitis to the metal within the joint replacement
D. New-onset psoriasis
E. Phototoxic drug eruption to doxycycline given pre-operatively

Correct choice: A. Allergic contact dermatitis to a soap or other topical application

Explanation: The sharp demarcation and linear almost “dripping” of the proximal aspect of the
dermatitis suggest an allergic contact dermatitis particularly to a topical application. Often stronger
steroids than a 1% hydrocortisone are needed. Irritant contact dermatitis is less pruritic, and less
well-demarcated. Allergic dermatitis to a metal and psoriasis both will not be as well-demarcated
and geometric. A phototoxic drug eruption could unlikely be on the hip and nowhere else, and is
unlikely to have that “dripping” seen.

Reference: Bolognia, Jean., Jorizzo, Joseph L.Schaffer, Julie V. (Eds.) (2012) "Allergic Contact
Dermatitis." Dermatology /[Philadelphia] : Elsevier Saunders.

417
22- Atopic dermatitis is associated with all except:

A. Ichthyosis hystrix
B. Central facial pallor
C. Pityriasis alba
D. Nipple eczema
E. Hyperlinear palms

Correct choice: A. Ichthyosis hystrix

Explanation: Icthyosis vulgaris (not hystrix) is associated with atopic dermatitis as one of the minor
criteria of Hanifin. The other listed answers are associated with atopic dermatitis.

23- What is the most likely diagnosis?

A. Keratoacanthoma
B. Lentigo maligna melanoma
C. Amelanotic melanoma
D. Merkel cell carcinoma
E. Basal cell carcinoma

Correct choice: E. Basal cell carcinoma

Explanation: Basal cell carcinoma is the most common skin cancer diagnosed and is the most likely
diagnosis.

418
Keratoacanthoma presents as a pink to red dome shaped papule with a central keratotic core.
Lentigo maligna melanoma presentas an irregular tan to brown patch. Amelanotic melanoma is
possible based on the picture but is much less common than basal cell carcinoma. Merkel cell
carcinoma presents as a red to purple papule.

24- What is the most likely diagnosis in this patient with a monoclonal gammopathy?

A. Sarcoidosis
B. Leprosy
C. Phymatous rosacea
D. Necrobiotic xanthogranuloma
E. Cutaneous tuberculosis

Correct choice: D. Necrobiotic xanthogranuloma

Explanation: The patient has necrobiotic xanthogranuloma (NXG) associated with a monoclonal
gammopathy, which is most often an IgG kappa or lambda monoclonal gammopathy. NXG is a rare,
chronic, progressive granulomatous disorder. It manifests as yellowish plaques and nodules most
commonly in the mid face to periocular region.

Sarcoidosis presents more commonly as pink predominant papules with a yellow tinge on the nose
and nasal columella and is not classically associated with a monoclonal gammopathy. Leprosy is an
infectious cause of leonine facies but is not associated with pink granulomatous plaques on the face
with an associated monoclonal gammopathy. Phymatous rosacea presents on the nose to medial
cheeks with sebaceous gland hyperplasia leading to lobular and sebaceous, disfiguring appearance
to the nose. Cutaneous tuberculosis is not associated with monoclonal gammopathy.

419
25- What is the most common area affected in multifocal fixed drug eruption?

A. Oral or genital mucosa


B. Central face
C. Upper arms
D. Thighs
E. Lower abdomen

Correct choice: A. Oral or genital mucosa

Explanation: Over half of FDE lesions occur in the oral or on the genital mucosa.

26- Which of the following substances is known to cause a delayed positive patch test reaction?

A. Gold
B. Nickel
C. Bacitracin
D. Fragrance mix
E. Rosin

Correct choice: A. Gold

Explanation: Gold is known to cause a delayed patch test reading, with some recommending a final
patch test reading at 3 weeks to avoid missing any positive reactions. Patients allergic to gold often
also react to nickel and cobalt. Bacitracin is a frequent contact allergen that often coexists with an
allergy to neomycin. Fragrance mix contains cinnamic alcohol and aldehyde, hydroxycitronellel,
isoeugenol, eugenol, oak moss absolute, alpha-amyl cinnamic aldehyde, geraniol. Rosin
(colophony) is found in adhesive tape, cosmetics, glossy papers, violin bows, and chewing gum.

27- In addition to gentle washing of the face two times a day, what is the most appropriate
treatment step for this patient who is prone to dry skin?

A. Start Differin 0.1 gel nightly to every other night after wash

420
B. Aggressive exfoliation daily
C. Start doxycycline 100 mg by mouth two times a day with food and water
D. Monotherapy with clindamycin lotion every morning after wash
E. Start isotretinoin after checking labs and a pregnancy test

Correct choice: A. Start Differin 0.1 gel nightly to every other night after wash

Explanation: The patient has comedonal acne. She should wash her face 2 times a day with a gentle
cleanser. She should start using Differin 0.1 gel nightly to her face after washing. If too drying, the
patient can reduce usgae of Differin to every other night. A topical retinoid treats comedones and
assists with the proper keratinization of the hair follicle.

Aggressive exfoliation can damage the skin and make acne worse. Doxycycline is a systemic
treatment option in cases of inflammatory acne that has not responded to combined topical
treatment with clindamycin/benzoyl peroxide in the AM and a topical retinoid in the PM.
Monotherapy with clindamycin is ill advised in cases of inflammatory acne as bacterial resistance
becomes a problem; the patient should simultaneously used a benzoyl peroxide product. Isotretinoin
is the not the first treatment in cases of comedonal acne.

28- A 32-year-old man presents with several annular and polycyclic lesions commencing in the
flexures. His skin condition has a very cyclic course, as pustules resolve they are replaced by
superficial scaling and then new pustules form again. He also has an associated IgA
paraproteinemia. What is the most likely diagnosis?

A. Sneddon Wilkinson disease


B. Pustular psoriasis
C. Acute generalized exanthematous pustulosis

421
D. Superficial folliculitis
E. Benign familial pemphigus

Correct choice: A. Sneddon Wilkinson disease

Explanation: Sneddon Wilkinson is characterized by annular or polycyclic lesions, usually


commencing in the flexures. Very superficial (subcorneal) sterile pustules are the hallmark of
Sneddon–Wilkinson disease. There may be a gravity-induced demarcation in some
vesiculopustules, with clear fluid superiorly and pus inferiorly. This disease has a cyclic course, i.e.
as the pustules resolve they are replaced by superficial scaling and then new pustules form again.
Some patients with Sneddon–Wilkinson disease have an associated IgA paraproteinemia. Its
response to dapsone, combined with subcorneal pustules (in the absence of spongiform pustules),
provide support for this condition being a disease entity distinct from pustular psoriasis.
Pustular psoriasis: In generalized pustular psoriasis, infiltration of neutrophils dominates the
histologic picture and the clinical picture consists of sterile pustules on a background of bright
erythema.
Acute generalized exanthematous pustulosis: This condition would present with mild spongiform
pustulation, eosinophils and often a drug history with either a new medication or a change in drug
history.
Superficial folliculitis: The clinical presentation and distribution would differ as superficial
folliculitis is clinically characterized with appearance of pustules at follicular orifices and in
infundibulum.
Benign familial pemphigus is an uncommon autosomal dominant disorder clinically characterized
by flaccid blisters and erosions seen in intertriginous areas, especially the axillae and groin; moist,
malodorous vegetations and fissures can develop.

29- A 49-year-old female presents with diffuse pustules on the hands and feet as seen here. It is
incredibly pruritic and she is very uncomfortable. She has tried over the counter hydrocortisone
with no success. She smokes a half pack of cigarettes per day and monthly uses cocaine. In addition
to prescription medications, what is the most important point to counsel your patient on today?

422
A. She is at higher risk of cancer because of these cutaneous findings
B. The dermatosis is a variant of atopic dermatitis
C. Smoking is significantly related to these findings and cessation may be helpful
D. Cocaine is related to these findings and increased use worsening cutaneous symptoms
E. The dermatosis will resolve without intervention

Correct choice: c. Smoking is significantly related to these findings and cessation may be helpful

Explanation: Palmoplantar pustulosis, seen and descibed here, had been linked to smoking and
smoking cessation can lead to improvement in some patients. The other options are not true
regarding palmoplantar pustulosis.

30- Which of the following is NOT a cause of a saddle nose deformity?

A. Relapsing polychondritis
B. Hypohidrotic ectodermal dysplasia
C. Congenital syphilis
D. Trichorhinophalangeal syndrome
E. Wegener's granulomatosis

Correct choice: D. Trichorhinophalangeal syndrome

Explanation: Trichorhinopharyngeal syndrome is the only condition from the mentioned conditions
that is not associated with saddle nose deformity.It is associated with sparse hair, a bulbous (pear-
shaped) nose and cone-shaped epiphyses. The other listed conditions can result in saddle nose
deformity.

31- A 14-year-old male with moderate inflammatory and comedonal acne reports no improvement
three months after he was prescribed a topical retinoid/benzoyl peroxide combination product.
Which of the following is the most appropriate next step in management?

A. Increase topical retinoid strength


B. Add an oral antibiotic
C. Discuss initiation of isotretinoin

423
D. Discuss medication usage patterns
E. Add a topical antibiotic

Correct choice: D. Discuss medication usage patterns

Explanation: Considering high rates of medication nonadherence in acne, it is most appropriate to


review a patient’s usage patterns and address factors limiting use before altering therapy.

Using a common clinical scenario, this question emphasizes to the examinee the need to assess
medication adherence prior to changing therapy. A wide variety of patient-, therapy-, and healthcare
system-associated factors influence treatment adherence. Therefore, it is most appropriate to first
discuss with the patient how (or if) they are using their medication (choice 4) in order to discover
any barriers, real or perceived, to optimal usage. Commonly, irritancy limits use of topical acne
products, so it would be inappropriate to increase retinoid strength (choice 1) without first
reviewing actual use. A short course of oral antibiotics (choice 2) or addition of a topical antibiotic
(choice 5) may be appropriate measures if the patient is first determined to be using the prescribed
topical product as intended. Isotretinoin (choice 3) is indicated for severe, recalcitrant nodulocystic
acne and thus would not be appropriate to initiate prior to understanding how the patient is using
their topical product and considering the full range of available treatment options.

32- A 33-year-old female presents with the shown painful lesions that appeared abruptly one week
ago. She recently had gotten over an upper respiratory infection the previous week. She is currently
febrile to 38.3C with malaise and headaches. You perform a biopsy to confirm your clinical
suspicions. Which of the following medications should result in quick clearance of her skin lesions.

A. Doxycycline
B. Isotretinoin
C. Colchicine
D. Prednisone

424
E. Triamcinolone

Correct choice: D. Prednisone

Explanation: The clinical presentation most likely represents Sweet syndrome (acute febrile
neutrophilic dermatosis). Classical Sweet syndrome is frequently associated with recent infections,
IBD and pregnancy.
Diagnosis is made with 2 major and 2 minor criteria.
Major criteria: 1. Abrupt onset of painful erythematous plaques 2. Histopathological evidence of a
dense neutrophilic infiltrate without evidence of leukocytoclastic vasculitis
Minor criteria: 1. Fever 2. Association with malignancy, inflammatory disease, pregnancy or recent
URI, GI infection, vaccination 3. Excellent response to systemic steroids 4. Abnormal lab values
(ESR, CRP, leukocytosis, >70% neutrophils)

33- A 24-year-old male presents with the findings seen here. The phenomenon seen on the patient’s
arm is seen in many diagnoses in dermatology. The linearity of the dermatosis suggests which
phenomenon?

A. Koebnerization
B. Allergic contact dermatitis
B. Pathergy
D. Malingering
E. Phototoxic eruption

Correct choice: A. Koebnerization

Explanation: Koebnerization is seen in several skin conditions including lichen nitidus, seen here. It
is also referred to as an isomorphic response. The skin condition can grow within areas of trauma,

425
as seen by the linearity and nearby scratches in this photo. The other options are not seen with
lichen nitidus.

34- A 16-year-old male presents with the findings seen here. The lesions have been present for 5
months and are asymptomatic. Mother is concerned that they are not going away. What is the most
appropriate treatment?

A. Observation
B. Potent topical steroids
C. Oral acitretin
D. Topical tretinoin
E. Topical calcineurin inhibitor

Correct choice: A. Observation

Explanation: Lichen nitidus, seen here, is a benign often self-resolving dermatosis. Treatment is not
needed, particularly if asymptomatic, and observation is appropriate. Topical steroids and
calcineurin inhibitors can be used if lichen nitidus is pruritic, otherwise no treatment is needed. Oral
acitretin and topical vitamin A medications are unlikely to be helpful.

35- Which of the following is not a formaldehyde-releasing preservative?

A. Bronopol
B. Methylchloroisothiazinolone
C. Quaternium-15
D. Imidazolidinyl urea
E. DMDM hydantoin

426
Correct choice: B. Methylchloroisothiazinolone

Explanation: Methylchloroisothiazinolone, also known as Kathon CG, is the only non-


formaldehyde releasing preservative. It is found in cosmetics, skin/hair products, "acid" permanent
waves, soaps, latex emulsions, and biocides. All the others are formaldehyde-releasing
preservatives.

36- A 74-year-old female presents with a diffuse eruption as seen here. It affects nearly her entire
body. She is most likely to have which additional systemic findings?

A. Elevated creatinine
B. Neutrophilia
C. Anemia
D. Seizures
E. Rsspiratory distress

Correct choice: B. Neutrophilia

Explanation:Acute generalized exanthematous pustulosis is often seen with fever and neutrophilia
and can also have facial edema. Renal, bone marrow, neurologic and pulmonary involvement are
rare.

37- What is the most appropriate action in this otherwise healthy patient based on the image in the
photograph?

A. Punch biopsy

427
B. Tissue culture
C. Reassurance
D. Plain film of the foot
E. Magnetic resonance imaging of the foot

Correct choice: C. Reassurance

Explanation: The image depicts piezogenic papules, a benign entity associated with fatty connective
tissue protrusion (most commonly) of the heel. Plain film of the foot will not reveal anything.
Magnetic resonance imaging of the foot may provide additional detail particularly if the foot were
under pressure - revealing areas of slight protrusion of fatty connective tissue - but imaging is not
necessary to make this diagnosis. Punch biopsy and tissue culture are unnecessary and will do more
harm than good.

38- Which of the following findings portends a favorable course in sarcoidosis?

A. Lupus pernio
B. Blau syndrome
C. Heerfordt syndrome
D. Löfgren syndrome
E. Mikulicz syndrome

Correct choice: D. Löfgren syndrome

Explanation: Löfgren syndrome is associated with a favorable prognosis in sarcoidosis.

428
This question tests the examinee’s knowledge of the types of cutaneous sarcoidosis and their
associated prognoses. Of the answer choices, Löfgren syndrome (choice 4), which includes
erythema nodosum, hilar adenopathy, arthritis, and fever, tends to run the most limited course,
resolving spontaneously within 1-2 years. In contrast, lupus pernio (choice 1) is associated with
more severe and longstanding systemic sarcoidosis requiring intensive treatment. Blau syndrome
(familial juvenile systemic granulomatosis, choice 2) is a dominantly inherited disease that is
phenotypically similar to sarcoidosis, manifesting in childhood with granulomatous inflammation of
the skin, eyes, and joints. Heerfordt syndrome (uveoparotid fever, choice 3) comprises uveitis,
parotitis, fever, and cranial nerve palsy, while Mikulicz syndrome (choice 5) describes involvement
of lacrimal, salivary, and parotid glands; neither is as favorable as Löfgren syndrome.

39- What percentage of patients with this condition will develop systemic lupus erythematosus?

A. 0%
B. 5-15%
C. 40-50%
D. 70-80%
E. 100%

Correct choice: B. 5-15%

Explanation: 5-15% of patients with discoid lupus will develop systemic lupus erythematosus.

40- Which of the following concerning Degos' Disease is true?

A. After undergoing multiple stages, it resolves without scarring

429
B. It affects women more than men
C. Gastrointestinal involvement portends a poor prognosis
D. Glucocorticoids are standard of care
E. Lab results indicate a low plasma fibrinogen level and decreased platelet aggregation

Correct choice: C. Gastrointestinal involvement portends a poor prognosis

Explanation: In Dego's Disease (aka Malignant Atrophic Papulosis), GI involvement portends a


poor prognosis as death from this condition is usually due to fulminant peritonitis caused by
multiple perforations of the intestines.
Dego's Disease (aka Malignant Atrophic Papulosis): It most frequently affects men and is a
potentially fatal obliterative arteritis. After undergoing multiple stages, the patient is left with
varicelliform scars. Later, anemic infarcts involve the intestines to produce acute abdominal
symptoms of epigastric pain, fever, and hematemesis. Lab results indicate a high plasma fibrinogen
level and increased platelet aggregation. Administration of corticosteroids has not been beneficial.

41- A 45-year-old man presents for evaluation of 3 months of intensely pruritic violaceous papules
over the wrists, ankles, and genitals. Which of the following elements of the history is most
important in establishing the diagnosis?

A. Hepatitis B immunization status


B. Sexual history
C. Medication history
D. Family history
E. Age-appropriate malignancy screening

Correct choice: C. Medication history

Explanation: For a new-onset lichenoid eruption, it is crucial to review the medication history and
consider the possibility of lichenoid drug eruption, which can be clinically and histopathologically
identical to idiopathic lichen planus (LP).

This question tests the examinee’s knowledge of pertinent history to obtain for a new-onset
lichenoid eruption. In this scenario, it is essential to review both prescription and nonprescription
medication use (choice 3) and consider the possibility of lichenoid drug eruption, which can be
clinically and histologically indistinguishable from idiopathic LP. Rather than hepatitis B (choice

430
1), hepatitis C status is important to determine, more so in specific populations and with oral LP. LP
is not related to sexual activity (choice 2). Familial LP (choice 4) is relatively uncommon and less
useful to review than medications. As LP is not a paraneoplastic phenomenon, malignancy
screening (choice 5) would not be relevant in this situation.

42- A 60-year-old male patient on dialysis develops a fixed livedo reticularis which is firm to the
touch. The areas become increasingly violaceous and purpuric, eventually bullous and necrotic. The
patient also similar lesions on his abdomen, buttocks and thighs. He complains of severe pain, and
requires analgesia for control. What is the most likely diagnosis?

A. Livedo racemosa
B. Calciphylaxis
C. Cryoglobulinemia
D. Disseminated intravascular coagulation
E. Hematoma

Correct choice: B. Calciphylaxis

Explanation: Calciphylaxis begins as fixed livedo reticularis (livedo racemosa), which is frequently
firm or hard to the touch. Areas within the livedo become increasingly violaceous and eventually
purpuric, bullous, and necrotic. Affected tissue has reduced oxygenation. Lesions affect the legs
below the knees in 90% of patients. More proximal lesions and those of the fatty areas of the thighs,
buttocks, and abdomen occur in about two-thirds. Severe pain is a cardinal feature of calciphylaxis,
often requiring narcotic analgesia for control.

1 - Livedo racemosa refers to a form of LR that has a larger, branching, and more irregular pattern
and is often more widespread, affecting both the extremities and the trunk
3 - Cryoglobulins are cold-precipitable immunoglobulins that can be divided into three subtypes, all
of which have cutaneous manifestations including palpable purpura, myalgias and arthralgia,
peripheral neuropathy and glomerulonephritis.

431
4 - Disseminated intravascular coagulation is a condition in which small blood clots develop
throughout the bloodstream, blocking small blood vessels. The increased clotting depletes the
platelets and clotting factors needed to control bleeding, causing excessive bleeding.
5 - Hematoma is a solid swelling of clotted blood in the tissues

43- Which of the following is not true of this condition?

A. Associated with twin gestation


B. Associated with primigravidas
C. Associated with premature labor
D. Associated with maternal obesity
E. Associated with third trimester of pregnancy

Correct choice: C . Associated with premature labor

Explanation: The figure illustrates polymorphic eruption of pregnancy (PUPPP) with erythematous
papules within striae. There is no associated maternal or fetal risk. All of the remaining choices are
true.

44- What test should be performed for our patient with the hair loss pictured?

432
A. Serum zinc level
B. RPR
C. ESR and CRP
D. Serum copper level
E. Free and total testosterone levels

Correct choice: B. RPR

Explanation: Patient has moth-eaten alopecia of secondary syphilis. The correct answer is RPR to
screen for syphilis. Copper and zinc levels are not necessary. The hair loss is not consistent with
androgenetic alopecia, but even if it were you do not need free testosterone and total testosterone
levels to make that diagnosis. ESR and CRP are non-specific tests to evaluate for inflammation in
the body and are not useful in this setting.

45- Which of the following is the best treatment for this condition?

A. Brimonidine gel
B. Oxymetazoline cream
C. Hydrocortisone ointment
D. Ketoconazole cream
E. Doxycycline

Correct choice: E. Doxycycline

Explanation: This is peri-oral dermatitis, which is best treated with doxycycline of these
choices. Several therapies, including oral antibiotics (e.g., tetracyclines), topical antibiotics (e.g.,
erythromycin, clindamycin), topical metronidazole, topical azelaic acid, topical sulfacetamide-

433
sulfur, and topical calcineurin inhibitors (e.g., tacrolimus, pimecrolimus), have been used with some
success to treat peri-oral dermatitis. Brimonidine gel and oxymetazoline cream are used to treat the
persistent erythema of rosacea. While many believe that peri-oral dermatitis may represent a form
of rosacea, brimonidine gel and oxymetazoline cream will not treat the papules and pustules seen in
the image here. Hydrocortisone ointment (and other topical steroids) typically worsen peri-oral
dermatitis and thus should be avoided. Ketoconazole cream is a topical anti-fungal that does not
treat peri-oral dermatitis.

47- Which of the following conditions is most likely associated with these cutaneous findings?

A. Hypertension
B. Ulcerative colitis
C. Congestive heart failure
D. Diabetes
E. Cataracts

Correct choice: D. Diabetes

Explanation: Granuloma annulare is a benign, non-infectious, granulomatous condition of the skin


that is usually asymptomatic and self-limited. It typically presents with annular erythematous
plaques that are most commonly found on the dorsal surfaces of the hands and feet. Generalized and
perforating variants are most closely associated with diabetes.

48- A 66-year-old male who lives alone presents for a dark rash on the back that his PCP was
concerned about. What is the best treatment for the findings seen on this patient’s back?

A. Metronidazole gel

434
B. Scrubbing with an alcohol swab
C. Methotrexate
D. Oral prednisone
E. Clindamycin solution

Correct choice: B. Scrubbing with an alcohol swab

Explanation: This is an example of terra firma-forme or Duncan's dirty dermatosis. It presents as


dirt-like plaques and is both diagnosed and treated with isopropol alcohol. The other options have
not been shown to help treat terra firma-forme.

49- An HIV positive male presents with psoriasis over 8% of his body (not face or scalp), he has not
tried any prescription medications. He is not using moisturizers. What is the correct initial
approach?

A. Start cyclosporine and monitor renal function and blood pressure


B. Test the patient for tuberculosis and if negative start Humira (adalimumab) subcu
injection 80 mg initial dose on day 1 followed by 40 mg subcu injection on day 8 and 40
mg subcu every two weeks thereafter
C. Begin acitretin 25 mg PO daily after checking CBC, CMP, lipid panel
D. Start mometasone 0.1 ointment BID to affected areas on body, advise face to feet
moisturizer at least BID
E. Begin Otezla (apremilast) starter pack by mouth as directed with goal to titrate up to 30
mg two times a day

435
Correct choice: D. Start mometasone 0.1 ointment BID to affected areas on body, advise face to feet
moisturizer at least BID

Explanation: The correct answer is to begin topical steroid two times a day to the rash and use
emollient at least two times a day to the body in patient with 8% BSA psoriasis who has not tried
any prescription treatments. While the other options are treatments that could be used in for
psoriasis in a patient with HIV and psoriasis, it is most appropriate to start with topical steroid and
emollient for treatment. Cyclosporine is not a chronic medication for psoriasis and is not a
medication to use for 8% BSA psoriasis.

50- A 32 year old male patient presents with the sudden eruption of multiple itchy bumps on his feet
after playing with his children in the grass. Hydrocortisone is improving the itchiness somewhat.
What is the most likely diagnosis?

A. Dermatitis herpetiformis
B. Chigger bites
C. Neurotic excoriations
D. Allergic contact dermatitis
E. Porphyria cutanea tarda

Correct choice: B. Chigger bites

Explanation: The patient has encountered chiggers, mite larvae who feed on human keratinocytes.
Their feeding process induces the development of the characteristic chigger bite, which is very
pruritic. Topical steroids can reduce the itch sensation. Dermatitis herpetiformis commonly presents
on extensor surfaces as pruritc papules and vesicles with excoriation. The onset would not be as
sudden and limited to the feet. The lesions are not excoriations (a secondary skin lesion) and thus
neurotic excoriations is incorrect. Allergic contact dermatitis would present as ill-defined
eczematous pink patches with scale, not as discrete papules. Porphyria cutanea tarda would show

436
evidence of vesicles, scars, and milia - none of which are present in this photograph. In addition, the
condition is chronic.

51- Which one of the following can develop into squamous cell skin cancer over time?

A. Actinic keratosis
B. Onychomycosis
C. Seborrheic keratosis
D. Psoriasis
E. Impetigo

Correct choice: A. Actinic keratosis

Explanation: Actinic keratosis can develop into squamous cell skin cancer, thus it is a pre-cancerous
lesion. Seborrheic keratosis is a benign thickening of the skin not associated with a pre-malignant
states. Onychomycosis is a fungal infection of the nail not associated with a pre-malignant state.
Psoriasis is an inflammatory skin condition not associated with a pre-malignant state. Impetigo is a
superficial infection of the skin by bacteria and is not associated with a pre-malignant state.

52- Autoantibodies to ECM-1 are found in which of the following conditions?

A. Lipoid proteinosis
B. Lichen sclerosus
C. Focal dermal hypoplasia
D. Incontinentia pigmenti
E. Marfan syndrome

Correct choice: B. Lichen sclerosus

Explanation: Autoantibodies to extracellular matrix protein-1 (ECM-1) are found in lichen sclerosus
(LS). This question tests the examinee’s knowledge of the pathogenesis of LS. In LS (choice 2), up
to 80% of patients have circulating autoantibodies to ECM-1, a glycoprotein that contributes to
integrity of the basement membrane zone and assembly of collagen fibrils. An important distinction
needs to be made between LS and lipoid proteinosis (choice 1), in which there is an ECM1 gene

437
mutation. The remaining answer choices are caused by distinct gene mutations: PORCN in focal
dermal hypoplasia (Goltz syndrome, choice 3); IKBKG/NEMO in incontinentia pigmenti (choice
4); and fibrillin 1 in Marfan syndrome (choice 5).

53- Patients with this disorder may develop exaggerated reactions to insect bites:

A. Chronic lymphocytic leukemia


B. Bullous pemphigoid
C. Lupus erythematosus
D. Dermatomyositis
E. Incontinentia pigmenti

Correct choice: A. Chronic lymphocytic leukemia

Explanation: Patients with chronic lymphocytic leukemia may develop exaggerated reactions to
insect bites, including bullous reactions. Patients with the other listed conditions to do exhibit
exaggerated reactions to insect bites.

54- On patch testing, which of the following is most likely to produce a relevant positive reaction?

A. Budesonide
B. Gold sodium thiosulfate
C. Thimerosal
D. Epoxy resin
E. Paraben mix

438
Correct choice: B. Gold sodium thiosulfate

Explanation: Several studies have found gold to be the most common relevant culprit in eyelid
allergic contact dermatitis (ACD).

This question assesses the examinee’s knowledge of ACD involving the eyelids. Common eyelid
contact allergens include metals (most prominently gold [choice 2], nickel, and cobalt), fragrances,
preservatives, topical antibiotics, and surfactants. None of the other options are among the most
common causes of eyelid ACD: budesonide (choice 1) is the screening agent for group 1 topical
corticosteroids; thimerosal (choice 3) is a preservative that uncommonly causes ACD (although it is
found in some ophthalmic drops); epoxy resin (choice 4) is found in adhesives; and paraben mix
(choice 5) screens for parabens, common preservatives in cosmetics and personal care products that
rarely cause ACD.

55- A 42-year-old man presents with milky white macules and patches in his periorbital area
surrounded by normal skin. The lesions have discrete margins and are irregular in shape. On
Wood’s lamp examination, the contrast between the affected areas and the surrounding skin is
striking. Which o the following statements is not true in this condition?

A. Vitiligo vulgaris is the most common clinical type of vitiligo observed in children
B. Segmental vitiligo is significantly increased in adults compared to children
C. In lesions of vitiligo, melanocytes are typically absent or present in small numbers
D. The epidermal melanocyte density can be assessed with Melan-A, MITF, and HMB-45
E. The epidermal melanocyte density can be assessed via incubation of biopsy specimens
with dihydroxyphenylalanine

Correct choice: B. Segmental vitiligo is significantly increased in adults compared to children

439
Explanation: Segmental vitiligo is significantly increased in children compared to adults. Although
vitiligo vulgaris is the most common clinical type observed in children, the frequency of segmental
vitiligo (~15–30%) is significantly increased compared to that in adults (<5–10%). The incidence of
associated endocrinopathies is less than in the adult vitiligo population. A family history
of vitiligo is associated with an earlier age of onset.

- Vitiligo vulgaris is the most common clinical type of vitiligo observed in children
- In lesions of vitiligo, melanocytes are typically absent or present in small numbers. The epidermal
melanocyte density can be assessed with melanocyte-specific immunohistochemical stains, such as
Melan-A (MART-1), MITF, and HMB45, or via incubation of biopsy specimens with
dihydroxyphenylalanine (DOPA; detects tyrosinase activity). Although ultrastructural studies may
be performed for research purposes, very few hypomelanotic disorders have specific ultrastructural
findings.

56- Morsicatio buccarum results from which of the following?

A. Metal dental amalgams


B. Nicotine exposure
C. Melkersson-Rosenthal syndrome
D. Keratin 4 and 13 mutations
E. Chronic irritation from biting

Correct choice: E. Chronic irritation from biting

Explanation: Morsicatio buccarum is the term describing a shaggy white plaque on the buccal
mucosa occurring secondary to chronic irritation from biting.

57- You are evaluating a 25-year-old woman who presents with a rash on her face. She states she
might be having a reaction to a food she ate, but she is not sure. The rash is erythematous and
patchy on both cheeks, and there is no sparing of the nasolabial folds. Which one of the following
would you order at this point?

A. ANA, C3, and C4


B. Oral prednisone
C. Oral doxycycline

440
D. Oral acyclovir
E. Rheumatoid factor, ESR, and CRP

Correct choice: C. Oral doxycycline

Explanation: The patient has rosacea, a chronic inflammatory rash classically of the mid face.
Rosacea does not spare the nasolabial folds, while cutaneous lupus does. Lupus spares the
nasolabial fold. In the case of lupus, you would consider ordering ANA, C3, and C4 and you may
start prednisone if the patient is having disease severe enough to warrant treatment with systemic
steroid. Ordering rheumatoid factor, ESR, and CRP will not help the patient and oral acyclovir is
not indicated as the patient does not have a viral disease such as herpes simplex or herpes zoster.

58- A 40-year-old man presents with rough papules on the dorsal aspect of the proximal fingers
which are also seen on the trunk and extremities. In some areas, the papules coalesced to form large
orange-red plaques. His palms and soles have an orange-red appearance, and there is erythema and
diffuse fine scale of the scalp. What is the most likely diagnosis for this patient’s condition?

A. Seborrheic dermatitis
B. Cutaneous T-cell lymphoma
C. Pityriasis rubra pilaris
D. Kawasaki disease
E. Psoriasis

Correct choice: C. Pityriasis rubra pilaris

Explanation: Pityriasis rubra pilaris characteristically presents with follicular papules with an
erythematous base, including on the proximal dorsal fingers. There is a coalescence of orange–red
plaques, but with obvious islands of sparing. An orange–red waxy keratoderma of the palms and
soles is often seen.

1 - Seborrheic dermatitis: Early PRP of the scalp can mimic seborrheic dermatitis. As additional
clinical features of PRP develop, these two entities can be distinguished. PRP has additional
distinguishing features such as follicular papues, orange-red plaques, islands of sparing, and waxy
keratoderma. Scalp dermatitis is much more responsive to conventional therapy. The relative
recalcitrant nature of scalp PRP may serve as a clue to the diagnosis.

441
2 - Cutaneous T-cell lymphoma: A PRP-like eruption can be seen in patients with dermatomyositis
(Wong type) and cutaneous T-cell lymphoma, as can erythema and scaling of the scalp however the
other clinical findings are characteristic of pityriasis rubra pilaris.

4 - Kawasaki disease: Children with acute-onset PRP may be misdiagnosed as having Kawasaki
disease, however the older age of this patient and the other associated clinical features present do
not support this diagnosis.

5 - Psoriasis: The major entity in the differential diagnosis of pityriasis ruby pillars is psoriasis. The
distinctive orange–red palmoplantar keratoderma plus the keratotic follicular papules with a nutmeg
grater appearance, the classic islands of sparing of the trunk, a finer scale, and no family history of
psoriasis help differentiate PRP from psoriasis. The presence of oil-drop changes, small pits, and
marginal onycholysis of the nails favor psoriasis.

59- Which of the following is associated with this condition?

A. Regular nail pitting


B. White dermatographism
C. Decreased risk for cutaneous infections
D. Scarring hair loss
E. Inflammatory arthritis

Correct choice: B. White dermatographism

Explanation: The photo is of Dennie-Morgan lines in a chronic atopic dermatitis patient. Regular
nail pitting is seen in psoriasis; atopic dermatitis has increased cutaneous infections; scarring hair
loss and inflammatory arthritis are not associated with atopic dermatitis.

442
60- Which of the following is TRUE regarding this condition?

A. 50% of patients will eventually develop systemic involvement


B. C3 deficiency increases susceptibility to this condition
C. Biopsy typically shows a thinned basement membrane
D. Squamous cell carcinoma is a potential sequela
E. UV exposure is helpful in treatment

Correct choice: D. Squamous cell carcinoma is a potential sequela

Explanation: This is an image of discoid lupus erythematosus (DLE). Squamous cell carcinoma
may develop from lesions of DLE. SCCs arising from lesions of DLE are rare and aggressive
tumors with greater likelihood of metastases. Cases have been reported among patients with
different clinical characteristics. Classically, 5-15% of patients with DLE will eventually develop
SLE. C2 (not C3) deficiency increases susceptibility to autoimmune conditions, particularly lupus
(DLE, SCLE, SLE). Biopsy characteristically shows a thinned epidermis with plugged follicles,
vacuolar degeneration of the basal layer, a thickened basement membrane, melanin incontinence,
perivascular/periadnexal lymphocytes, and increased mucin deposition between collagen bundles.
UV exposure exacerbates DLE, so UV protection is a critical aspect of treatment.

61- A 30 year old pregnant woman presents with a red lesion near her nail that has rapidly appeared.
She is in her third trimester, and has had a normal pregnancy course to date. Which of the following
is the most likely diagnosis?

A. Melanoma
B. Glomus tumor
C. Bacillary angiomatosis
D. Pyogenic granuloma
E. Inflamed wart

443
Correct choice: D. Pyogenic granuloma

Explanation: A pyogenic granuloma is one of several vascular lesions that can present on the skin
during pregnancy, and they often develop rapidly in a peri-ungual location. At present, the
pathogenesis of pyogenic granuloma is unknown. In addition to pregnancy, reported triggers include
antecedent trauma and certain drugs, including acitretin, HIV protease inhibitors, EGFR inhibitors,
docetaxel, capecitabine and rituximab. In the case of pyogenic granulomas arising during
pregnancy, they often regress after delivery and treatment is not usually required unless they cause
excessive bleeding. Other vascular lesions that can develop during pregnancy include palmar
erythema, spider angiomas, varicose veins, cavernous hemangiomas, glomus tumors, and
hemorrhoids. The development of vascular lesions during pregnancy is thought to be due to a
combination of increased hormones and increased intravascular pressure. Of the answer choices
listed, a pyogenic granuloma is the most likely diagnosis based on the clinical description.

62- Which of the following drug(s) are commonly implicated in drug-induced vasculitis?

A. Propylthiouracil
B. Adalimumab
C. Propranolol
D. Procainamide
E. Dapsone

Correct choice: A. Propylthiouracil

Explanation: PTU is well-reported to cause p-ANCA positive vasculitis.

63- What is the best medication choice for the condition pictured that involves 8% body surface
area (BSA) only on the legs, no prior treatment tried.

A. Total body nbUVB treatment 3 times per week (covering genitals and eyes with shields)
B. Tremfya by injection (starter dose then standard maintenance)
C. Remicade by infusion (every 8 weeks)
D. Halobetasol 0.05% ointment 2 x a day to areas with rash
E. Otezla by mouth (uptitration via starter pack then standard dose of 30 mg 2 x a day by
mouth)

444
Correct choice: D. Halobetasol 0.05% ointment 2 x a day to areas with rash

Explanation: The patient has psoriasis with BSA 8% with no treatment trial previously. The most
reasonable next step is to begin a potent topical steroid on the areas with psoriasis on the legs 2 x a
day. You can consider occlusion of the steroid at night if patient not responsive to topical steroid or
if the steroid tends to rub off at night onto the bed.

The patient does not have a BSA% greater than 10 and thus choices such as Tremfya and Remicade
are an over-treatment in this patient who has not trialed any conservative measures yet. If patient
had psoriatic arthritis, you should then entertain biologic treatment but not for psoriasis vulgaris
with BSA 8% on the legs. nbUVB to the whole body 3 x a week is over-treatment for a patient with
such limited skin disease of 8% on the legs. You could consider Excimer treatment (handheld
nbUVB targeted treatment for smaller disease areas on the skin) if you had the device in your office
but that was not one of the choices. Much like Tremfya and Remicade, Otezla would represent an
over-treatment for the patient in the vignette but you could consider using the medication if the
patient also had psoriatic arthritis.

64- A 45 year old man presents to clinic with a history of Crohn’s disease, joint stiffness in the
morning, and these skin findings. Which of the following is the best treatment choice for this
patient?

445
A. Secukinumab
B. Ixekizumab
C. Guselkumab
D. Ustekinumab
E. Apremilast

Correct choice: D. Ustekinumab

Explanation: This patient has psoriasis, psoriatic arthritis, and Crohn’s disease. Ustekinumab binds
the p40 subunit to inhibit IL-12 and IL-23. Ustekinumab is approved for the treatment of plaque
psoriasis, psoriatic arthritis, and Crohn’s disease. It is the best treatment choice. A higher dose of
ustekinumab is needed to treat Crohn’s disease than to treat psoriasis.

Secukinumab is a monoclonal antibody that inhibits IL-17A. It is FDA-approved to treat psoriasis


and psoriatic arthritis, but like other IL-17 inhibitors (e.g. Ixekizumab) should not be used in
Crohn’s disease. Guselkumab inhibits IL-23 by selectively binding the p19 subunit of IL-23. It was
FDA-approved in July 2017 to treat plaque psoriasis only. Apremilast is approved for the treatment
of psoriasis and psoriatic arthritis, but not Crohn's disease (thereby making Ustekinumab the better
choice).

65- What is the most likely diagnosis?

A. Psoriasis
B. Pityriasis rosea
C. Cutaneous T-cell lymphoma
D. Contact dermatitis
E. Lichen planus

Correct choice: D. Contact dermatitis

446
Explanation: This periumbilical eczematous eruption is classic for allergic contact dermatitis
secondary to nickel exposure. The metal snaps on the pants are the cause of the nickel exposure in
this case. The remaining answer choices are less likely, especially given that the image shows a
well-demarcated eczematous periumbilical eruption and metal snaps suggesting a contact nickel
allergy.

66- A 27-year-old woman presents with an eczematous rash on the earlobes after wearing new
earrings. She mentions that she gets the same kind of rash when she wears a metal belt. Which of
the of the following metals should she most likely avoid?

A. Stainless steel
B. White gold
C. Gold (24 ct)
D. Sterling silver
E. Platinum

Correct choice: B. White gold

Explanation: The case presented is consistent with a contact dermatitis, likely allergic contact
dermatitis to a metal. The most common allergen is nickel. White gold may contain nickel, therefore
this type of metal should be avoided. Patient's allergic to nickel, typically can tolerate gold 18 ct or
higher, stainless steel, sterling silver, and pure platinum.

67- A 37-year-old female presents with this eruptions, most prominent on the arms and legs. It is
slightly pruritic, but the appearance is most concerning to her. What is the most likely diagnosis?

447
A. Cutaneous Crohns disease
B. Necrobiosis lipoidica
C. Granuloma annulare
D. Nummular eczema
E. Pityriasis rosea

Correct choice: C. Granuloma annulare

Explanation: The dermatosis seen here is annular, with erythematous and raised borders and is
slightly pruritic. Of the choices, granuloma annulare is the most likely diagnosis. The alternative
options do not represent the clinical granulomatous disease of granuloma annulare.

68- Which of the following is the most appropriate initial treatment for the condition pictured?

A. Prednisone
B. Mycophenolate mofetil
C. Infliximab
D. Cyclosporine
E. Doxycycline

Correct choice: A. Prednisone

Explanation:Prednisone is the most appropriate initial treatment for neutrophilic dermatosis of the
dorsal hands.
The examinee is presented with the image of an eroded violaceous plaque on the dorsal hand that is
highly consistent with neutrophilic dermatosis of the dorsal hands, an entity combining clinical and
pathologic features of Sweet’s syndrome and pyoderma gangrenosum. Prednisone (choice 1) is the
most appropriate first-line treatment. While the remaining options (choices 2-5) have been used as

448
second-line and beyond treatment of Sweet’s syndrome and/or pyoderma gangrenosum, none of
them represents the most appropriate first-line therapy.

69- Hydrogen peroxide is often chosen for treatment of which of the following?

A. Cleaning a chronic wound on the lower extremity


B. Treatment of a chronic venous ulcer
C. Post-operative follow-up of a full thickness skin graft with a bolster
D. Cutaneous fungal infections
E. Acne vulgaris

Correct choice: C. Post-operative follow-up of a full thickness skin graft with a bolster

Explanation: Hydrogen peroxide is cytotoxic and therefore not a first choice treatment in cleaning
chronic wounds and ulcers. Hydrogen peroxide is use to remove crusts on healing wounds such as
bolster on a skin graft.

70- Which of the following does NOT predict mortality with the pictured eruption?

449
A. Age
B. Serum bicarbonate
C. Serum glucose
D. Serum urea level
E. Serum protein level

Correct choice: E. Serum protein level

Explanation: Mortality from Stevens Johnson Syndrome/toxic epidermal necrolysis is predicted by


the SCORTEN scoring system. Prognostic factors include age, heart rate, presence of cancer/
malignancy, BSA involved > 10% on day 1, serum urea level, serum glucose level, and serum
bicarbonate level.

71- A 32-year-old man originally from southern China presents to your clinic with rash (pictured)
and complaints of muscle weakness. Which of the following is the most likely underlying
malignancy responsible for his presentation?

A. Lung adenocarcinoma
B. Colorectal adenocarcinoma
C. Gastric adenocarcinoma
D. Nasopharyngeal carcinoma
E. Testicular germ cell tumor

Correct choice: D. Nasopharyngeal carcinoma

450

Explanation: Muscle weakness coupled with erythematous papules overlying the dorsal hand joints
(Gottron's papules) is highly suggestive of dermatomyositis, which can be a paraneoplastic
phenomenon. Nasopharyngeal carcinoma, one of the most common cancers in southern China and
Southeast Asia, was the most common associated cancer in Asian studies of dermatomyositis. The
remaining malignancies may produce dermatomyositis as a paraneoplastic phenomenon, but they
are less likely in a man originally from southern China.

72- A 65-year-old female patient presents with poorly demarcated, symmetric, very painful patches
of erythema and retiform purpura, favoring the thighs and buttocks. Bullae and a dusky gray
discoloration developed, as well as the appearance of ulcerations with black, leathery eschars. A
skin biopsy of the affected area is suggestive of calciphylaxis. Shortly after, she dies from
secondary infection and sepsis. Which of the following is NOT a known risk factor for the
development of this condition?

A. Diabetes mellitus
B. Obesity
C. End-stage renal disease
D. Hemodialysis
E. Male gender

Correct choice: E . Male gender

Explanation: Known risk factors for calciphylaxis include end-stage renal disease, hemodialysis,
obesity, diabetes mellitus, and female gender as well as warfarin and liver disease.
Hyperparathyroidism, calcium supplementation, and vitamin D intake are additional risk factors,
with the latter possibly related to the differentiation of vascular smooth muscle cells into osteoblast-
like cells. Known risk factors for calciphylaxis include end-stage renal disease, hemodialysis,
obesity, diabetes mellitus, and female gender as well as warfarin and liver disease.

451
73- Which type of calcification best characterizes the condition pictured?

A. Metastatic
B. Dystrophic
C. Iatrogenic
D. Idiopathic
E. Mixed

Correct choice: D. Idiopathic

Explanation: Scrotal calcinosis, or “idiopathic calcified nodules of the scrotum,” cannot be


attributed to any identifiable local or systemic causes.

This question assesses the examinee’s ability to recognize scrotal calcinosis and discriminate
between the five main types of cutaneous calcification (calcinosis cutis). Scrotal calcinosis is a
common form of idiopathic calcification (choice 4), meaning that it is not related to any
recognizable local or systemic etiologies. In contrast, metastatic calcification (choice 1) affects
normal tissue during states of calcium/phosphate dysregulation, most commonly renal disease.
Dystrophic calcification (choice 2) occurs when there is local tissue damage in the setting of normal
calcium metabolism, taking place in a broad range of conditions including autoimmune

connective tissue diseases (CREST syndrome, childhood dermatomyositis), lobular panniculitides


(especially pancreatic), traumas (neonatal heel sticks), genodermatoses (pseudoxanthoma
elasticum), infections (parasitic primarily), and neoplasms (pilomatricomas). Iatrogenic
calcification (choice 3) is induced by diagnostic or therapeutic interventions, most often by
extravasation of calcium-containing intravenous solutions. Mixed calcification (choice 5) combines
the features of metastatic and dystrophic calcification; calciphylaxis is a prominent example.

74- A 28-year-old patient presents with a rare, pruritic vesiculobullous eruption that developed in
the immediate postpartum period after a recent childbirth. A direct immunofluorescence shows

452
linear C3 deposition along the basement membrane zone. Her infant was born premature and small
for gestational age. Which of the following is the most likely diagnosis?

A. Prurigo of pregnancy
B. Herpes gestationis
C. Polymorphic eruption of pregnancy
D. Atopic eruption of pregnancy
E. Intrahepatic cholestasis of pregnancy

Correct choice: B. Herpes gestationis

Explanation: Herpes gestationis is also known as gestational pemphigoid or pemphigoid gestationis.


It is a rare, pruritic vesiculobullous eruption that develops during late pregnancy or the immediate
postpartum period. On direct immunofluorescence, there is linear C3 deposition along the basement
membrane zone in 100% of patients (30% of patients also have IgG deposition). IgG1
autoantibodies are directed against a transmembrane hemidesmosomal protein (BP180, BPAG2,
collagen XVII). There is an increased risk of prematurity and small-for-gestational age neonates –
the risk correlates with the severity of disease. It commonly recurs in subsequent pregnancies.

1 – Prurigo of pregnancy is a benign nonspecific pruritic papular rash that arises during pregnancy.
It has also been described with other names such as prurigo gestationis, early onset prurigo of
pregnancy, and pruritic folliculitis of pregnancy. It typically manifests as a papular dermatitis
(prurigo) and can resemble nodular prurigo. Direct immunofluorescence is negative and histology is
nonspecific. It is not associated with any maternal or fetal risks, and typically resolves after delivery
of the baby. 3 – Polymorphic eruption of pregnancy manifests as urticarial papules and plaques that
usually appear within striae distensae during the latter portion of the third trimester or immediately
postpartum. There is development of polymorphous features (vesicles, erythema, target, and
eczematous lesions) with disease progression. It is most frequent in primiparous women. Direct
immunofluorescence is negative, routine labs are normal, and histologic features are nonspecific.
There are no maternal or fetal risks. 4 – Atopic eruption of pregnancy is characterized by
eczematous or papular skin lesions in a patient with atopic diathesis in whom other specific
dermatoses have been excluded. It generally appears earlier than other pregnancy-related
dermatoses (75% before the third trimester). Histology is nonspecific, direct immunofluorescence is
negative. Up to 70% of patients have elevated serum IgE levels. There are no maternal or fetal risks.
5 – Intrahepatic cholestasis of pregnancy is characterized by pruritus without primary skin lesions
with an onset during the third trimester. Secondary changes correlate with disease duration and vary
from subtle excoriations to severe prurigo nodularis. Histology is nonspecific and direct
immunofluorescence is negative. Elevated total serum bile acid levels are diagnostic. There is an
increased risk of prematurity, intrapartum fetal distress and stillbirths.

453
75- A child develops an allergic reaction at the site of an influenza vaccine. To which of the
following substances may she be allergic to?

A. Lanolin
B. Thimerosol
C. Ethylenediamine dichloride
D. Triclosan
E. Gluteraldehyde

Correct choice: B. Thimerosol

Explanation: Thimerosol is a preservatives in vaccines such as the influenza, tetanus, and diphtheria
vaccines. It is also found in antitoxins and immunoglobulins. Thimerosol is a mercury-containing
organic compound. Lanolin is from the sebum of sheep. Ethylenediamine dichloride is a stabilizer
in topical creams, medicines, dyes, insecticides, and fungicides and was previously found in
nystatin cream. Triclosan is an antibacterial agent found in soap, shampoo and mouthwash.
Gluteraldehyde is a cold sterilizing solution used for medical and dental equipment.

76- You have been caring for an adolescent male with acne conglobata. He presents to your clinic
with osteoarticular lesions that gradually improve with anti-inflammatory medication. You suspect a
diagnosis of SAPHO syndrome. Which of the following is NOT one of the osteoarticular
manifestations of SAPHO syndrome?

A. Inflammatory synovitis
B. Arthrosteitis
C. Hyperostosis and osteitis of the anterior chest wall and spine
D. Septic osteomyelitis
E. Chronic recurrent multifocal osteomyelitis

Correct choice: D. Septic osteomyelitis

Explanation: SAPHO syndrome consists of a spectrum of aseptic neutrophilic dermatoses in


association with aseptic osteoarticular lesions that have distinctive histologic and radiographic
features. Osteoarticular manifestations include: inflammatory synovitis; arthrosteitis (inflammation
of the osseous structures of a joint); hyperostosis or osteitis, most often anterior chest wall (sternum,
clavicles, ribs) and axial skeleton (spine, pelvis). A subtype of chronic recurrent multifocal

454
osteomyelitis (CRMO) manifests as frequent involvement of metaphyses of tubular bones and is
seen primarily in children.

77- A 65-year-old woman presents with yellow-brown patches with an irregular outline and
pinpoint petechiae within patches in successive crops. The lesions are mostly present in the lower
legs and began after a long hike. Which of the following pigmented purpuric eruptions is the most
likely diagnosis in this scenario?

A. Schamberg disease
B. Lichen aureus
C. Purpura annularis telangiectoides of Majocchi
D. Pigmented purpuric lichenoid dermatitis of Gougerot and Blum
E. Eczematid-like purpura of Doucas and Kapetanakis

Correct choice: A. Schamberg disease

Explanation: Schamberg disease is the most common orm of pigmented purpura. It can occur in
children and has a peak frequency in middle-aged to older men. It is characterized by yellow-brown
patches with an oval to irregular outline. There are pinpoint peteachiae within patches - likened to
cayenne pepper. It occurs in successive crops and favors the lower legs.
2 - Lichen aureus is a rare variant of pigmented purpura which is characterized by a solitary patch
or purpuric macules, papules and patches primarily on the lower extremities.
3 - Purpura annularis telangiectoides of Majocchi is uncommon and occurs in adolescents and
young adults, especially women. It is characterized by 1-3 cm annular plaques that may slowly
expand. There are punctate telangiectasias and petechiae within the border. The location is the trunk
and proximal lower extremities.
4 - Pigmented purpuric lichenoid dermatitis of Gougerot and Blum is a rare variant of pigmented
purpura that occurs in middle-aged to older men. It is an admixture of two ypes of lesions:
Schamberg-like, purpuric red-brown lichenoid papules. It is chronic and occasionally pruritic.
5 - Eczematid-like purpura of Doucas and Kapetanakis is a rare variant of pigmented purpura
characterized by scaly petechial or purpuric macules, papules and patches. It is pruritic and mostly
on the lower extremities.

79- Which of the following is true regarding Nekam's Disease?

455
A. Is generally responsive to topical and intralesional steroids
B. Characteristically lacks scale
C. Rarely involves the buttocks
D. Presents with a reticulate pattern on the dorsal hands and feet
E. Presents with hypopigmented, atrophic lesions on the extremities

Correct choice: D. Presents with a reticulate pattern on the dorsal hands and feet

Explanation: Nekam's Disease (keratosis lichenoides chronica) presents with violaceous papules
and nodules, hyperpigmented and hyperkeratotic, covered with gray scales. There is often a linear
and reticulate pattern on the dorsal hands and feet, extremities and buttocks. This condition is
generally very refractory to treatment. The other answer choices listed are incorrect.

80- Pediatric patients that present with this deformity should be monitored for which of the
following sequela?

A. Seizures
B. Sclerodactyly
C. Raynaud's phenomenon
D. Cutaneous calcification
E. Photosensitivity

Correct choice: A. Seizures

Explanation: En coup de sabre means "stroke or blow of the sword" and is a term for linear
morphea of the scalp/forehead. Rarely this can affect underyling structures including muscle, bone,
meninges and brain and can create a focus for seizures.

456
Sclerodactyly, Raynaud's phenomenon and cutaneous calicification are seen in systemic
scleroderma or CREST syndrome, not linear morphea which is typically limited to the skin.
Photosensitivity is not a feature of linear morphea.

81- Biopsy of this lesion reveals massive papillary dermal edema, a diffuse neutrophilic infiltrate,
and a lack of vasculitis. Stains for infectious organisms are negative. What is the first line
treatment?

A. Dapsone
B. Colchicine
C. Thalidomide
D. Potassium iodide
E. Prednisone

Correct choice: E. Prednisone

Explanation: The associated image and described biopsy findings are consistent with the diagnosis
of Sweet's syndrome (aka acute febrile neutrophilic dermatosis), which typically presents with
tender, erythematous edematous papules and plaques over the face and upper extremities. A
vesiculobullous or pustular presentation is frequently associated with underlying myelogenous
leukemia. Systemic steroids (e.g. prednisone), typically for 4-6 weeks, is the first line treatment for
Sweet's syndrome barring any contraindications. Furthermore, an excellent response to systemic
steroids is one of the minor diagnostic criteria for Sweet's syndrome. Potassium iodide, dapsone,
and colchicine are considered 2nd line agents in the treatment of Sweet's syndrome. There have
been rare case reports of thalidomide being effective for recalcitrant Sweet's syndrome.

457
82- A 54-year-old female presents to clinic with a bilateral eruption of red-brown papules on her
dorsal hands and elbows. Some of the lesions are occasionally pruritic with a burning sensation. A
biopsy is done and demonstates findings consistent with erythema elevatum diutinum (EED).
Which of the following infections is EED associated with?

A. HSV
B. HIV
C. HPV
D. Leprosy
E. MRSA

Correct choice: B. HIV

Explanation: EED has been linked to a number of systemic diseases. Infections reported to occur in
association with EED have included HIV, beta-hemolytic strep, hepatitis and tuberculosis. A
literature review in 2012 identified 19 reported cases of EED in association with HIV. Nodular
lesions progressing to bulky masses appear to be more common in EED patients with HIV. The
relationship between EED and HIV is thought to involve immune complex deposition in blood
vessels triggered by the HIV infection acting as antigenic stimuli. EED also has been associated
with hematologic disorders, particularly IgA monoclonal gammopathy. Additionally, EED has been
linked to a variety of autoimmune diseases, such as RA, IBD and lupus.

83- Periorificial dermatitis is thought to be a variant of rosacea. What is a known association with
periorificial dermatitis?

A. Doxycycline intake
B. Gluten sensitivity
C. Sunscreen
D. Application of topical calcineurin inhibitors
E. Application of fluorinated topical steroids

Correct choice: E. Application of fluorinated topical steroids

Explanation: Periorificial dermatitis (or perioral dermatitis) has been associated with prior use of
topical or inhaled fluorinated steroids, therefore they should be discontinued.

458
The other choices have not been associated with periorificial dermatitis. Doxycycline is a treatment
for periorifacial dermatitis.

84- A 20-year-old female presents with prickling, tingling, burning and a stinging sensation within
30 minutes of water contact, which lasts for up to 2 hours. The symptoms begin on the lower
extremities and then generalize, with sparing of the head, palms, soles and mucosa. She notes that
the symptoms occur irrespective of the water temperature or salinity. On examination, specific skin
lesions are not seen. What is the most likely diagnosis?

A. Polycythemia vera
B. Mastocytosis
C. Hypereosinophilic syndrome
D. Aquagenic pruritus
E. Aquagenic urticaria

Correct choice: D. Aquagenic pruritus

Explanation: Aquagenic pruritus is usually secondary to a systemic disease or another skin disorder
such as urticaria or dermatographism. Primary idiopathic aquagenic pruritus is uncommon.
Aquagenic pruritus presents with prickling, tingling, burning, or stinging sensations within 30
minutes of water contact, irrespective of its temperature or salinity, and lasts for up to 2 hours.
Typically, symptoms begin on the lower extremities and then generalize, with sparing of the head,
palms, soles, and mucosae. On exam, specific skin lesions are not seen. The pathologic mechanism
is unknown although elevated dermal and epidermal level sof acetylcholine, histamine, serotonin,
and prostaglandin E2 have been described.

1 - Polycythemia vera: Aquagenic pruritus can sometimes be secondary to a systemic disease such
as polycythemia vera. However, the clinical scenario is a classic description of aquagenic pruritus,
not polycythemia vera which presents with a ruddy complexion.
2 - Mastocytosis: Mastocytosis usually presents with cutaneous lesions and a positive Darier's sign.
In patients with aquagenic pruritus, specific skin lesions are not seen.
3 - Hypereosinophilic syndrome: Cutaneous lesions are seen in > 50% of patients with
hypereosinophilic syndrome. Hypereosinophilic syndrome is on the differential diagnosis of
pruritus or prickling sensation provoked by water contact. In patients with aquagenic pruritus,
specific skin lesions are not seen.

459
5 - Aquagenic urticaria: Aquagenic pruritus can sometimes be secondary to another skin disorder
such as urticaria or dermatographism. However, the clinical scenario is a classic description of
aquagenic pruritus, not aquagenic urticaria. In aquagenic pruritus, specific skin lesions are not seen.

85- A 73-year-old man presented with a slow-healing postsurgical defect on the left cheek after
undergoing Mohs surgery two months earlier. He has noticed a large amount of malodorous
drainage from the site that has soaked through several bandages. What is the best dressing to use on
this patient’s wound?

A. Hydrocolloid dressing
B. Hydrogel dressing
C. Alginate dressing
D. Film
E. Foam

Correct choice: C. Alginate dressing

Explanation: The correct answer is alginate dressings (Choice 3) which are polysaccharides derived
from kelp and algae that are ideal for highly exudative wounds. Other advantages of alginates are
that they confer hemostatic benefits and are suitable for use in sinuses. Hydrocolloid dressings
(Choice 1), which are waterproof gel or foam within polyurethane films, are not suitable for this
patient as these dressings are good for mildly exudative wounds. Hydrogel dressings (Choice 2),
hydrophilic polymer holding significant amount of water, are not suitable for this patient as these
dressings are best for dry, necrotic wounds. Films (Choice 4), thin layers of elastic polyurethane,
provide a barrier against bacteria and are not the best choice for highly exudative wounds. Foams
(Choice 5), hydrophobic polyurethane sheets with hydrophilic surface, are ideal for mildly to
moderately exudative wounds.

86- A 17-year-old male has cystic acne vulgaris that is severe on his face, chest, and back x 3 years
that is causing scarring. He has tried topical clindamycin and tretinoin 0.05% gel QAM and QPM,
respectively, and washes with Neutrogena original. He is otherwise healthy. What is the next best
step?

A. Start doxycycline 100 mg PO BID with food and water

460
B. Start isotretinoin weight-based dosing after checking baseline labs and enrolling in
iPledge
C. Incise and drain each cyst
D. Begin a sulfur/sulfacetamide wash in place of Neutrogena wash
E. Start blue light therapy monthly for patient

Correct choice: B. Start isotretinoin weight-based dosing after checking baseline labs and enrolling
in iPledge

Explanation: With a patient who has failed topical treatment who has severe acne burden with
scarring, the correct choice is to start isotretinoin after enrolling in iPledge and checking baseline
labs. With severe acne with scarring in a male patient who has failed topical treatment, the next
most appropriate step is to initiate isotretinoin after enrolling in iPledge and checking baseline labs.

87- Which of the following carcinomas has been most associated with erythema gyratum repens?

A. Lung cancer
B. Breast cancer
C. Colon cancer
D. Prostate cancer
E. Upper GI tract cancer

Correct choice: A. Lung cancer

Explanation: 80% of cases of erythema gyratum repens have been associated with underlying
malignancy. Lung cancer is the most common neoplasm. The skin eruption preceedes the detection
of malignancy by an average of 9 months

88- Mastocytosis is a systemic disease that causes wheals upon rubbing a lesion (Darier's sign).
Mastocytosis can be measured by examination of serum:

A. Tryptase
B. Kinase
C. Bilirubin

461
D. Histamine
E. IgE

Correct choice: A. Tryptase

Explanation: The most common form of cutaneous mastocytosis is urticaria pigmentosa. It is


diagnosed by measuring serum tryptase levels. The new diagnostic method is measurement of
tryptase in bone-marrow blood, which is a new, sensitive marker of the mast cell burden in bone
marrow of patients with systemic mastocytosis. The other markers do not diagnose mastocytosis.

89- Patients with dermatitis herpetiformis should avoid which of the following?

A. Corn
B. Rice
C. Oats
D. Sulfites
E. Iodides

Correct choice: E. Iodides

Explanation: Exposure to iodides has been reported to cause flares of dermatitis herpetiformis (DH).

This question assesses the examinee’s knowledge of triggers to be avoided in DH. Oral as well as
topical exposure to iodides (choice 5) has been reported to cause flares of DH. Patients must strictly
avoid consumption of gluten, which is found in wheat, barley, and rye; safe dietary grains include
corn (choice 1), rice (choice 2), and oats (choice 3). Sulfites (choice 4) do not cause flares of DH.

90- A 42-year-old patient develops an abrupt and short-lived swelling of her tongue, mouth, and
mucous membranes. There are no other parts of the body affected. The swelling is most pronounced
around the eyes and lips. The internal lining of the upper respiratory tract is involved and the patient
is immediately intubated. You suspect a diagnosis of angioedema. Which of the following
medications the most likely cause of this reaction?

A. Lisinopril

462
B. Vancomycin
C. Ampicillin
D. Trimethoprim-sulfamethoxazole
E. Doxycycline

Correct choice: A. Lisinopril

Explanation: A number of drugs can cause angioedema without wheals. The most common are
NSAIDs and ACE inhibitors. Aspirin intolerance may present with angioedema alone, or with
urticaria or anaphylaxis; cross-reactions with other NSAIDs can occur. A causative link to ACE
inhibitors may be overlooked because angioedema can present over a year after starting the
medication. For practical purposes, ACE inhibitors should be discontinued in favor of other
antihypertensive medications if angioedema occurs without wheals. ACE inhibitors are
contraindicated in patients with HAE and acquired C1 inh deficiency.

2-Vancomycin is not the most common cause of angioedema out of the medications listed.
3-Ampicillin is not the most common cause of angioedema out of the medications listed.
4-Trimethoprim-sulfamethoxazole is not the most common cause of angioedema out of the
medications listed.
5-Doxycycline is not the most common cause of angioedema out of the medications listed.

91- An 8-year-old boy presents with subcutaneous nodules over bony prominences, migratory joint
pains, chest pain, and rapid, irregular, and aimless involuntary movements of the arms and legs,
trunk and facial muscles. On closer examination, you notice that he has an evanescent annular and
polycyclic erythematous eruption. You suspect he may have cutaneous and clinical manifestations
of acute rheumatic fever. What is the most appropriate diagnosis for his cutaneous eruption?

A. Annular erythema of infancy

463
B. Erythema migrans
C. Erythema marginatum
D. Erythema gyratum repens
E. Erythema annulare centrifugum

Correct choice: C. Erythema marginatum

Explanation: Erythema marginatum: Erythema marginatum rheumaticum is a migratory, annular


and polycylic erythematous eruption. It is a cutaneous manifestation of acute rheumatic fever.
Associated findings incude: carditis, migratory polyarthritis, Sydenham chorea, and subcutaneous
nodules. It is seen more commonly in children than in adults.

1 - Annular erythema of infancy: The differential diagnosis of erythema marginatum inclues annular
erythema of infancy however the patient is not an infant in this scenario. The typical associated
findings of acute rheumatic fever are absent in this condition.
2 - Erythema migrans: Erythema migrans is an annular erythema that develops at the site of a bite of
a Borrelia-infected tick. Several species of Ixodes ticks are infected with different genospecies of
Borrelia burgdorferi. Erythema migrans represents the initial cutanoues manifestation of Lyme
disease and is seen in 60-90% of patients diagnosed with the disease. Multiple secondary lesions
that are smaller in size can occur as a result of spirochetemia or lymphatic spread.
4 - Erythema gyratum repens: Erythema gyratum repens is a figurate erythema that is migratory and
copmosed of concentric rings with a wood-grain appearance. It represents a paraneoplastic
phenomenon and the most common underlying neoplasm is carcinoma of the lung. The lesions may
have associated pruritus and scale, and they characteristically exhibit rapid migration (up to 1 cm
per day). The cutaneous lesions resolve when the neoplasm is successfully treated.
5 - Erythema annulare centrifugum: Erythema annulare centrifugum is characterized by
erythematous annular lesions that migrate centrifugally. Superficial lesions can have the classic
trailing white scale, while the deep gyrate erythemas have a more infiltrated border. The disorder
occurs more commonly in adults, and superficial lesions favor the thighs and hips. Individual
lesions usually last for several days to a few months. Although often idiopathic in nature, it can be
associated with infecitons (i.e. tinea pedis) and anecdotally with other disorders or exposures.

92- A patient in your occupational dermatology clinic presents with mottled hyperpigmentation with
areas of hypopigmentation and hair loss. She has a history of numerous SCCis (squamous cell
carcinomas in situ). Upon further examination, she has many symmetric, yellow, punctate corn-like
papules on both palms (shown in image). The patient likely was chronically exposed to which of the
following?

464
A. Mercury
B. Lead
C. Arsenic
D. Bismuth
E. Chromium

Correct choice: C. Arsenic

Explanation: Chronic arsenic exposure can occur through contaminated drinking water or
occupational exposure. It is characterized by mottled hyperpigmentation with areas of
hypopigmentation, keratoses on the palms and soles and other sites, alopecia, and multiple non-
melanoma skin cancers, particularly Bowen’s disease (SCCis). Arsenical keratoses are
precancerous papules on the palms and soles that are symmetric, yellow and punctate. They
measure 2-10 mm in size and favor the thenar and hypothenar eminences, distal palms, lateral
fingers, dorsal aspect of the interphalangeal joints, and weight-bearing plantar surfaces. They rarely
occur on the trunk, proximal extremities, eyelids and genitalia and may coalesce into plaques.

1 – Mercury poisoning and exposure may present with acrodynia, tattoo reaction (cinnabar),
granulomas, exanthema, slate-gray discoloration, allergic and irritant contact dermatitis and baboon
syndrome. 2 – Lead poisoning may present with irritant contact dermatitis and a gingival lead line.
4 – Exposure to bismuth may present with irritant contact dermatitis and black tongue. 5 –
Chromium exposure may present with allergic and irritant contact dermatitis, skin ulceration, and
burns. Systemic absorption can lead to renal failure, hepatic failure, anemia, and coagulopathy.

93- Autoantibodies to ECM-1 may be associated with which of the following phenotypic
alterations?

A. Hoarse cry at birth, with beaded papules at the eyelid margins noted later in life
B. 1-3mm yellow papules and wrinkled skin of the lateral neck and flexures

465
C. Thickened toenails with painful plantar keratoderma
D. Tense blisters arising within urticarial plaques
E. Pruritic, atrophic scarring of the genitalia

Correct choice: E. Pruritic, atrophic scarring of the genitalia

Explanation: Antibodies to ECM-1 are associated with lichen sclerosus, choice 5. Lichen sclerosus
is characterized by pruritic, atrophic plaques of the vulva, though this condition can affect the male
genitals or extragenital skin as well. Choice 1 describes Lipoid Proteinosis, which is caused by a
mutation in ECM-1, as opposed to antibodies. Choice 2 described Pseudoxanthoma elasticum,
which is caused by a mutation in ABCC6. Choice 3 describes Pachyonychia Congenita, a family of
diseases associated with mutations in keratins 6 and 16. Choice 4 describes bullous pemphigoid (or
a BP-like presentation of EBA), which is caused by antibodies to BPAG1 or 2 (or Collagen 7 in the
case of EBA).

94- You are called to see a consult in the emergency room. The patient has erythema and scaling
involving 90% of the body surface area, peripheral edema, tachycardia, and a fever. From an
epidemiological standpoint, which of the following is the most common cause of erythroderma?

A. Idiopathic
B. Atopic dermatitis
C. Cutaneous T-cell lymphoma
D. Psoriasis
E. Drug reactions

Correct choice: A. Idiopathic

Explanation: The most common causes of erythroderma are idiopathic, dermatitis, psoriasis, drug
reactions, and cutaneous T-cell lymphoma. In a study of 746 patients, the underlying causes of
erythroderma were dermatitis (24%), psoriasis (20%), drug reactions (19%), and CTCL (8%). When
categories within the dermatitis group were examined, atopic dermatitis (9%) was the most common
type, followed by contact dermatitis (6%), seborrheic dermatitis (4%), and chronic actinic
dermatitis (3%). Despite multiple skin biopsies, in-depth clinical investigations, and detailed
medical histories, the underlying cause of erythroderma cannot be found in 25% of patients. Cases
of idiopathic erythroderma also tend to be chronic and are more likely to recur after treatment.

466
2 – According to a study of 746 patients, dermatitis represents 24% of the most common underlying
causes of erythroderma. When categories of the dermatitis group were examined, atopic dermatitis
(9%) was the most common type, followed by contact dermatitis (6%), seborrheic dermatitis (4%),
and chronic actinic dermatitis (3%). 3 – Cutaneous T-cell lymphoma represents about 8% of the
most common underlying causes of erythroderma. 4 – Psoriasis represents about 20% of the most
common underlying causes of erythroderma. 5 – Drug reactions represent approximately 19% of the
most common underlying causes of erythroderma.

95- A mother brings here 3-year-old child in for an intensely pruritic rash. What is the next step in
management?

A. Potassium hydroxide prep


B. Wound culture
C. Tzanck smear
D. Scabies prep
E. Punch biopsy

Correct choice: D. Scabies prep

Explanation: The diagnosis is scabies and therefore a scabies prep is the appropriate next step in
management. Characteristic features include burrows and crusted papules in the webbing between
digits. Other commonly involved areas include the areolae, umbilicus and in particular the penis
and scrotum where characteristic nodules may form. In infants, the head may be involved, which is
rarely seen in adult patients. A scabies prep can be performed using a #15 blade with a drop of
mineral oil followed by evaluation under the microscope. A wound culture is used to evaluate for
bacterial or candidal infection. Pustules should be unroofed and the purulent contents swabbed. A
tzanck smear is performed to evaluate herpetic infections. The scrapings from the base of an
unroofed blister reveal multinucleated giant cells. A potassium hydroxide prep is performed when
there is clinical suspicion of a fungal infection. Most often, the rash will be annular with a leading

467
scale. A punch biopsy is performed when the diagnosis is in question. A punch biopsy is selected
when the suspected pathology involves the deeper skin structures. Though a biopsy could confirm
the diagnosis of scabies, a scabies prep is an in office or bedside procedure offering rapid diagnosis.

96- Nail findings occur in 10-15% of cases of lichen planus, usually in combination with other
classic cutaneous lesions. Which of the following is not a commonly reported feature of nail lichen
planus?

A. Trachyonychia
B. Onychoschizia
C. Anonychia
D. Dorsal pterygium
E. Ventral pterygium

Correct choice: E. Ventral pterygium

Explanation: Lichen planus of the nails occurs in 10-15% of cases, usually in combination with
other LP lesions on the skin. 20-nail dystrophy (trachyonychia) can be seen, but it is not diagnostic
of LP. Thinning, longitudinal ridging, and distal splitting of the nail plate can occur (onychoschizia).
Also, onycholysis, longitudinal striation (onychorrhexis), subungual hyperkeratosis, or anonychia
can be seen. The classic finding is dorsal pterygium or forward growth of the eponychium with
adherence of the proximal nail plate. Ventral pterygium is more classicaly seen in Scleroderma.

97- A 19-year-old female presents with chronic, focal intense pruritus of the upper back, especially
over the right medial scapular border. She has had these symptoms since childhood.
On exam you note that this corresponds to the area innervated by the posterior rami of thoracic
spinal nerves 2-6. Occasionally she senses paresthesias. On exam, you notice a hyperpigmented
patch due to chronic rubbing of the affected area. Given the onset during childhood, this condition
may also represent a sign of which syndrome?

468
A. Multiple endocrine neoplasia type 1
B. Multiple endocrine neoplasia type 2A
C. Multiple endocrine neoplasia type 2B
D. Tuberous sclerosis
E. Neurofibromatosis

Correct choice: B. Multiple endocrine neoplasia type 2A

Explanation: Multiple endocrine neoplasia type 2A: Notalgia paresthetica is a common condition in
adults that is characterized by focal, intense pruritus of the upper back, especially over the medial
scapular borders; this corresponds to the area innervated by the posterior rami of thoracic spinal
nerves 2–6 (T2–T6). Occasionally, patients also report pain, paresthesias, or hyperesthesias. A
characteristic finding on physical examination is a hyperpigmented patch due to chronic rubbing of
the affected area. Dermal melanophages are evident histologically, and there is considerable overlap
with macular amyloidosis.Most evidence suggests that notalgia paresthetica represents a sensory
neuropathy. Degenerative changes of the spine corresponding to the affected dermatome(s) are
evident in ~60% of affected patients, suggesting a pathogenic role for spinal nerve impingement.
The posterior rami of T2–T6 pursue a right-angle course through the multifidus spinae muscle,
potentially predisposing them to entrapment and injury. Notalgia paresthetica may also represent a
sign of multiple endocrine neoplasia type 2A (Sipple syndrome), especially if the onset is during
childhood or adolescence.

1 - Multiple endocrine neoplasia type 1:


Notalgia paresthetica may also represent a sign of multiple endocrine neoplasia type 2A (Sipple
syndrome), especially if the onset is during childhood or adolescence. It is not associated with
multiple endrocine neoplasia type 1.

3 - Multiple endocrine neoplasia type 2B:


Notalgia paresthetica may also represent a sign of multiple endocrine neoplasia type 2A (Sipple
syndrome), especially if the onset is during childhood or adolescence. It is not associated with
multiple endrocine neoplasia type 2B.

4 - Tuberous sclerosis: Notalgia paresthetica may also represent a sign of multiple endocrine
neoplasia type 2A (Sipple syndrome), especially if the onset is during childhood or adolescence. It
is not associated with tuberous sclerosis.

469
5 - Neurofibromatosis: Notalgia paresthetica may also represent a sign of multiple endocrine
neoplasia type 2A (Sipple syndrome), especially if the onset is during childhood or adolescence. It
is not associated with neurofibromatosis.

98- A 38-year-old African American female presents with grouped follicular papules, acneiform
lesions, indurated plaques and tumors which are most pronounced in the head and neck area (see
image). The skin lesions are associated with alopecia and mucinorrhea. She has infiltrated plaques
in both eyebrows and complains of severe pruritus. She has experienced frequent secondary
bacterial infections with Staphylococcus aureus. Which of the following is the most common
phenotype of this patient’s condition?

A. CD3+, CD4+, CD8-


B. CD3+, CD4-, CD8-
C. CD3-, CD4+, CD8+
D. CD3-, CD4-, CD8+
E. CD3+, CD4-, CD8+

Correct choice: A. CD3+, CD4+, CD8-

Explanation: Folliculotropic mycosis fungoides is a distinct variant of mycosis fungoides


characterized by the presence of folliculotropic infiltrates with preferential involvement of the head
and neck region. Patients may present with grouped follicular papules, acneiform lesions, indurated
plaques and sometimes tumors. Skin lesions are often associated with alopecia and sometimes
mucinorrhea. Infiltrated plaques in the eyebrows and concurrent alopecia are common and highly
characteristic. In most cases, the neoplastic T cells have a CD3+, CD4+, CD8- phenotype as in
classic mycosis fungoides. Admixture with CD30-positive blast cells is common.

470
2 – In most cases of folliculotropic mycosis fungoides, the neoplastic T cells have a CD3+, CD4+,
CD8- phenotype as in classic mycosis fungoides. Admixture with CD30-positive blast cells is
common. 3 – In most cases of folliculotropic mycosis fungoides, the neoplastic T cells have a
CD3+, CD4+, CD8- phenotype as in classic mycosis fungoides. Admixture with CD30-positive
blast cells is common. 4 – In most cases of folliculotropic mycosis fungoides, the neoplastic T
cells have a CD3+, CD4+, CD8- phenotype as in classic mycosis fungoides. Admixture with CD30-
positive blast cells is common. 5 - In most cases of folliculotropic mycosis fungoides, the neoplastic
T cells have a CD3+, CD4+, CD8- phenotype as in classic mycosis fungoides. Admixture with
CD30-positive blast cells is common.

99- Which of the following is the most likely diagnosis?

A. Erythema multiforme
B. Annular urticaria
C. Secondary syphilis
D. Fixed drug eruption
E. Polymorphous light eruption

Correct choice: A. Erythema multiforme

Explanation:Erythema multiforme (EM) presents with target lesions favoring the extensor
extremities.
Without a history, the examinee must discover distinguishing features in the clinical photo to arrive
at the correct diagnosis. On the dorsal hand, there are variable edematous urticarial papules, some of
which feature three zones: a dusky center surrounded by two concentric rings of different colors.
Although not all of the lesions are fully developed, based on the presence of typical targets, EM
(choice 1) is the best answer. The differential diagnosis includes annular urticaria (choice 2), which
displays central erythema or normal skin, never epidermal damage as evidence by duskiness,
necrosis, or vesiculation; secondary syphilis (choice 3), which tends to favor the palms over the
dorsa and usually has scale; fixed drug eruption (choice 4), which can be morphologically similar to

471
EM but generally shows fewer lesions; and polymorphous light eruption (choice 5), which also
presents on the sun-exposed extremities and may at times be difficult to distinguish from EM, but
will not have true targets. In clinical practice, additional history and possibly biopsy will readily
differentiate between these entities.

100- A 55-year-old female presents with a one month history of the following clinical pictures. She
additionally has developed thickened nails with subungal debris and a few small hyperkeratotic red
plaques on the abdomen. What is the most likely diagnosis?

A. Mal de meleda syndrome


B. Pityriasis rubra pilaris
C. Psoriasis vulgaris
D. Lichen spinulosus
E. Cutaneous T-cell lymphoma

Correct choice: B. Pityriasis rubra pilaris

Explanation: This presentation of a cutaneous eruption of palmer keratoderma, hyperkeratotic


follicular papules ("nutmeg grater"), red-orange plaques and nail findings is most consistent with
Type I pityriasis rubra pilaris. This clinical presentation less likely represents the other answer
choices.

101- What is the most likely diagnosis based on the picture?


A. Neurotic excoriations
B. Reactive perforating collagenosis
C. Disseminated herpes simplex virus infection
D. Miliaria crystallina

472
E. Pyoderma gangrenosum

Correct choice: B. Reactive perforating collagenosis

Explanation: The patient has reactive perforating collagenosis (RPC) characterized by collagen
extruding through the skin. While it is not possible to discern which perforating disorder the patient
has without biopsy and clinicopathologic correlation, the only perforating disorder option on the list
is RPC. Neurotic excoriations is incorrect as excoriations are a secondary lesion and the patient has
primary lesions (that are not excoriated). Disseminated or localized herpes simplex virus should
present as vesicles, not papules with the appearance of central substance extrusion. Miliaria
crystallina presents with translucent, small 1-2 mm vesicles that appear wet, which is not the
appearance of the rash in the picture. Pyoderma gangrenosum is a neutrophil-rich inflammatory
condition of the skin characterized by a non-healing ulcer with a red to purple appearance with an
undermined border.

102- Which of the following is the most common nail finding in patients with alopecia areata?

A. Oil spots
B. Onycholysis
C. Splinter hemorrhages
D. Trachyonychia
E. Pitting

Correct choice: E. Pitting

473
Explanation: Nail changes are a common feature of alopecia areata (AA), with an average
prevalence of 30%, and can cause significant disfigurement and loss of function. Pitting and
trachyonychia were by far the most common manifestations of AA, with an average prevalence of
20 and 8%, respectively. Red spotted lunulae, onycholysis, and punctate leukonychia were other
reported nail findings in AA. Other etiologies, such as onychomycosis or lichen planus, may coexist
with or confound the diagnosis.

103- Which of the following conditions is associated with an IgM monoclonal gammopathy?

A. Schnitzler's syndrome
B. Sweet's syndrome
C. Scleredema
D. Scleromyxedema
E. Subcorneal pustular dermatosis

Correct choice: A. Schnitzler's syndrome

Explanation: Schnitzler’s syndrome is associated with IgM monoclonal gammopathy.


This question tests the examinee’s knowledge of monoclonal gammopathies of note in dermatology.
Of the answer choices, only Schnitzler’s syndrome (choice 1) is associated with IgM monoclonal
gammopathy, in association with chronic urticaria, bone pain, and fevers. Sweet’s syndrome (choice
2) and subcorneal pustular dermatosis (choice 5) are associated with IgA gammopathy; scleredema
(choice 3), with IgGκ; and scleromyxedema (choice 4), with IgGλ.

104- What is the appropriate and necessary medical management for a seborrheic keratosis
diagnosed by biopsy?

A. Superficial radiation therapy


B. Electrodessication and curettage
C. Excision with 5 mm margin
D. Cryotherapy
E. Reassurance

Correct choice: E. Reassurance

474
Explanation: Seborrheic keratoses are benign lesions that do not require additional treatment. You
may consider cryotherapy and/or curettage to seborrheic keratosis lesions that are bothersome for a
patient but neither is necessary. Superficial radiation therapy is not appropriate treatment for a
seborrheic keratosis. Excision with a 5 mm margin is an over-treatment for a seborrheic keratosis.

105- What is the diagnosis?

A. Granuloma annulare
B. Tinea corporis
C. Erythema annulare centrifigum
D. Subacute cutaneous lupus
E. Cutaneous larva migrans

Correct choice: A. Granuloma annulare

Explanation: Granuloma annulare is a benign condition characterized by annular dermal plaques.


They are often pink to skin-colored. High potency topical or intralesional steroids may be used,
though often observation and reassurance is the treatment of choice. Tinea corporis is a cutaneous
dermatophyte with a characteristic annular appearance with an advancing edge. Erythema annulare
centrifigum is a gyrate characterized by erythematous, annular patches and plaques with a trailing
scale. This feature can help distinguish from tinea corporis. Subacute cutaneous lupus (SCLE)
presents with annular and often scaly patches and plaques. It is most commonly seen in
photodistributed areas. There is an association between SCLE and the anti-Ro antibody. Cutaneous
larva migrans is a cutaneous hookworm infection acquired most commonly by walking barefoot on
contaminated grounds. The classic serpiginous arrangement represents movement of the hookworm
through the epidermis.

475
106- A patient presents to your clinic with Raynaud's of the hands, and purpura on the lower
extremities. Lab testing is notable for positive cryoglobulins, with the subtype revealing
monoclonal IgM only. Which of the following underlying conditions is most likely?

A. Hepatitis B
B. Hepatitis C
C. Waldenström macroglobulinemia
D. Angioimmunoblastic T cell lymphoma
E. Systemic sclerosis

Correct choice: C. Waldenström macroglobulinemia

Explanation: The stem describes a case of Type I cryoglobulinemia, which is typified by


monoclonal IgM or IgG (no rheumatoid factor activity). Type I cryoglobulins are always linked to a
B-cell lymphoproliferative disorder, that is, multiple myeloma, Waldenström macroglobulinemia,
chronic lymphocytic leukemia, B-cell non-Hodgkin lymphoma, or hairy cell leukemia. Mixed
cryoglobulinemias are characterized by monoclonal IgM (or IgG) with polyclonal IgG (Type II) or
polyclonal IgM complexed with polyclonal IgG (Type III). Mixed cryoglobulinemias (type II or III)
are associated with hepatitis C virus (HCV) infection, systemic autoimmune diseases,
lymphoproliferative disorders, and other chronic infections.

107- First line treatment for this condition includes:

A. Intralesional steroid injections


B. 2.4 million units IM benzathine penicillin
C. Methotrexate
D. Ruxolitinib

476
E. Treatment for underlying lymphoma

Correct choice: A. Intralesional steroid injections

Explanation: This image demonstrates exclamation point hairs of alopecia areata. Intralesional
injection with low potency steroids is a first line treatment option.

108- A patient with scalp psoriasis has recently started a medication which results in increased
levels of cyclic AMP which decreases production of TNF, IL-17, IL-23. You inform her that the
most common side effect upon initially starting this medication is which of the following?

A. Skin site reaction


B. Diarrhea
C. Fungal infections
D. Reactivation of tuberculosis
E. Dizziness

Correct choice: B. Diarrhea

Explanation: Apremilast is a phosphodiesterase 4 inhibitor which results in increased levels of


cyclic AMP which in turn decreases production of TNF, IL-17, IL-23. It is dosed twice daily and is
titrated up over 6 days to a final dose of 30 mg PO twice daily to minimize diarrhea. It is relatively
more effective for palmoplantar psoriasis and scalp psoriasis. The most common side effects are GI
upset/diarrhea in 20% of cases.

109- A 40-year-old male presents with a chronic pruritic eruption. Which of the following is true?

477
A. Apple green birefringence on PAS stain
B. Improves after treating underlying infection
C. Stains PAS positive and diastase resistant
D. Demonstrates amyloid deposition predominantly around blood vessels
E. Associated with a lymphoproliferative disorder

Correct choice: C. Stains PAS positive and diastase resistant

Explanation: Choice 1 is actually apple green birefringensce on CONGO-RED staining; choice 2,


there is no underlyin infection in lichen amyloid; choice 4, lichen amyloid demonstrates dermal
amyloid deposition, not perivascular; choice 5 lichen amyloid is not associated with underlying
malignancy (like primary systemic amyloid)

110- A 20-year-old female presents with an allergic contact dermatitis to a perfume containing Lily
of the valley. What is the causative allergen?

A. Cinnamic acid
B. Vanillin
C. Hydroxycitronellal
D. Atranorin
E. Evernic acid

Correct choice: C. Hydroxycitronellal

Explanation: The main allergen in Lily of the valley, which is found in perfumes, soaps, cosmetics,
eye cream and aftershaves, is hydroxycitronellal (synthetic). Cinnamic acid and vanillin are present
in Balsam of Peru, while atranorin and evernic acid are allergens in oak moss absolute.

111- This 48 year old woman tells you these lesions began as "acne bumps" on her legs. A biopsy
reveals ulceration and a mixed inflammatory infiltrate, including neutrophils. A tissue culture for
infectious organisms is negative. What is the most likely underlying systemic condition in this
patient?
A. Celiac disease
B. Ovarian cancer

478
C. Chronic pancreatitis
D. IgA monoclonal gammopathy
E. Castleman's disease

Correct choice: D. IgA monoclonal gammopathy

Explanation: The associated image and described biopsy findings are consistent with a diagnosis of
pyoderma gangrenosum (PG), which most often presents as an acneiform papule or pustule that
progresses into a rapidly expanding ulceration with an undermined border. ~50% of patients with
PG have an underlying systemic condition, most commonly inflammatory bowel disease (ulcerative
colitis more often than Crohn's disease). Other associated conditions include: arthritis (typically
asymmetric, seronegative, monoarticular arthritis of the large joints), leukemia (usually
myelogenous), multiple myeloma, IgA monoclonal gammopathy, polycythemia, Hepatitis C
infection, HIV infection, and systemic lupus erythematosus. Celiac disease is associated with
dermatitis herpetiformis. Ovarian cancer may underlie the development of dermatomyositis.
Chronic pancreatitis is associated with pancreatic panniculitis. Lastly, paraneoplastic pemphigus
may develop in patients with Castleman's disease.

112- A 19-year-old male presents with asymptomatic, multiple, yellow-white papules on the
vermilion portion of the upper lip. The papules are no more than 1-2 mm in diameter and are
present bilaterally and also on the buccal mucosa. The patient recently underwent puberty, and is
otherwise healthy. Which of the following is the most appropriate term for these ectopic sebaceous
glands in this location?

A. Montgomery tubercles
B. Fordyce spots
C. Meibomian glands

479
D. Glands of Zeis
E. Tyson glands

Correct choice: B. Fordyce spots

Explanation: Fordyce granules are sebaceous glands that are located in the vermilion zone of the
lips and the oral mucosa. Historically, these sebaceous glands have been considered ectopic;
however, because they are such a common clinical finding in the oral cavity, Fordyce granules
should be regarded as a variation of normal anatomy. “Free” sebaceous glands (see below) can also
be found on the eyelids (meibomian glands), areolae (Montgomery tubercles), labia minora, and
prepuce (Tyson glands).

1- Montgomery tubercles are ectopic sebaceous glands found around the areolae.
3- Meibomian glands are ectopic sebaceous glands found on the eyelids.
4- Glands of Zeis are unilobar sebaceous glands located on the margin of the eyelid. The glands of
Zeis service the eyelash. These glands produce an oily substance that is issued through the excretory
ducts of the sebaceous lobule into the middle portion of the hair follicle.
5- Tyson glands are ectopic sebaceous glands found on the prepuce.

113- You diagnose a 22-year-old male member of the army with tinea versicolor of the back, chest,
and arms. The rash flares in the summertime. What is the most appropriate initial recommendation?

A. Fluconazole 200 mg by mouth daily ongoing


B. Selenium sulfide 1% lotion daily to affected areas
C. Frequent moisturizers
D. BenzaClin (benzoyl peroxide/clindamycin) 5%/1% daily to affected areas
E. Hydrogen peroxide to the involved areas

Correct choice: B. Selenium sulfide 1% lotion daily to affected areas

480
Explanation: The patient has tinea versicolor and should be treated with selenium sulfide as the
starting therapy. You may consider fluconazole by mouth intermittent dosing for severe, refractory
cases of tinea versicolor but you should not use ongoing fluconazole therapy as treatment for the
condition. Benzoyl peroxide/clindamycin is used for acne. Hydrogen peroxide solutions are used to
treat seborrheic keratoses. Frequent moisturizers will not treat tinea versicolor.

114- The best diagnosis for this congenital pale patch without any extracutaneous associations is:

A. Hypomelanosis of Ito
B. Segmental vitiligo
C. Ash leaf spots
D. Nevus anemicus
E. Nevus depigmentosus

Correct choice: E. Nevus depigmentosus

Explanation: Nevus achromicus is interchangeable for nevus depigmentosus. It usually presents at


birth or appear during early infancy as normal pigmentation increases. Most individuals will have a
solitary lesion of nevus depigmentosus, but multiple lesions and segmental forms of nevus
depigmentosus have been described.

Nevus depigmentosus tends to persist lifelong, but remains unchanged after onset. The
hypopigmented white spots of tuberous sclerosis are most difficult to distinguish from nevus
depigmentosus, but lack of other cutaneous or systemic manifestaions exclude tubrous sclerosis.
Lesions of vitiligo tend to be depigmented (melanocytopenic not melanopenic as in the question),
and show a bright white coloration with Wood's lamp examination. Nevus anemicus is a distinct
vascular birthmark characterized by blanching of cutaneous blood vessels, hence presenting as a
"white" patch of skin that becomes unnoticeable when the surrounding skin is blanched with a glass
slide ("diascopy").

481
Achromic nevus is an uncommon birthmark characterized by a well-defined pale patch. This is
usually several centimeters in diameter, with an irregular but well-defined border. Shape and size
varies. Often, smaller hypopigmented macules arise around the edges, resembling a splash of paint.
Achromic nevus is also known as nevus depigmentosus and non-pigmented nevus. The name is not
quite right, as thehypomelanotic patches of an achromic nevus are not completely white, unlike the
areas of depigmentation in vitiligo, which are amelanotic, and completely lacking melanin.
Achromic nevi are usually solitary, in contrast to tuberous sclerosis, where multiple pale patches
occur and are called ash-leaf spots. Achromic nevus is usually noted at birth or early childhood,
although lesions may not be apparent until mid-childhood in those with light-colored skin.
The nevi remain stable over time. Achromic nevus most commonly arises on the trunk, but may also
arise on the limbs and elsewhere.

115- Which extracutaneous organ is typically associated with this subtype of sarcoidosis?

A. Heart
B. Lungs
C. Kidneys
D. Eyes
E. Liver

Correct choice: B. Lungs

Explanation: This is an example of lupus pernio, which is often associated with chronic sarcoidosis
of the lungs. Although the patient may have involvement of other organs, pulmonary sarcoid is most
often seen with lupus pernio.

482
116- An 87-year-old female gets a dry flaky rash on her lower legs and arms each wintertime when
the humidity decreases. The areas are itchy. She scrubs her arms and legs very hard to clean them.
What is the most appropriate treatment for this patient?

A. Topical emollients at least three times per day and gentle skin care
B. Clotrimazole 1% cream two times a day to affected areas of the skin
C. Unna wrap the arms and legs weekly
D. Clobetasol 0.05% ointment three times a day to affected areas of the skin
E. Fluconazole 200 mg by mouth weekly x 4 weeks then stop

Correct choice: A. Topical emollients at least three times per day and gentle skin care

Explanation: The correct answer is gentle skin care and the use of emollients for the patient
described who has winter's itch, otherwise known as asteatotic eczema. Topical anti-fungal and
systemic anti-fungal medications are not indicated for this condition. You may use topical steroid in
the future if not responsive to emollient and gentle skin care but you would start with a lower
strength steroid than clobetasol. Unna wraps are helpful for individuals who are picking their skin
but are not indicated for the patient in this vignette.

117- This patient presents with a papule on the abdomen that he would like you to remove. He
states he's always had it and finds it to be unattractive. Which of the following is true?

A. A similar finding is present in 20% of the general population


B. In women there is no change with changing hormones
C. On histology demonstrate smooth muscle
D. Has increased malignant potential and should be immediatly removed

483
E. Renal imaging is recommended

Correct choice: C. On histology demonstrate smooth muscle

Explanation: The patient has a supernumerary nipple. Found in 1-6% of the general population.
Histology shows increased smooth muscle, sebaceous glands that are opened directly onto skin
surface, and mammary glands. There is no increased malignant potential but examination such as
mammography or ultrasound has been recommended as with other breast tissue. Routine renal
imaging is not recommended. Can be present in Incontinentia Pigmenti patients.

118- Compared to children, adults with the condition pictured are more likely to develop which of
the following complications?

A. Abdominal pain
B. Persistent fever
C. Hepatic impairment
D. Renal impairment
E. Cerebral arteriovenous malformation

Correct choice: D. Renal impairment

Explanation: Adult patients with Henoch-Schönlein purpura (HSP) are more likely than children to
develop persistent renal impairment.
This question tests the examinee’s ability to recognize HSP presenting as palpable purpura on the
lower extremity and knowledge of how HSP complications differ in adults versus children.
Compared to children, adults with HSP are at higher risk for renal impairment (choice 4), but less

484
commonly experience abdominal pain (choice 1) or fever (choice 2). Hepatic impairment (choice 3)
and cerebral arteriovenous malformations (choice 5) are not commonly recognized sequelae of HSP.

119- A 42 year-old woman has the findings pictured, as well as erythematous patches on the upper
chest and shoulders. No Raynaud's, polyarthritis, pulmonary, or cardiac involvement is noted.
Which antibody might you expect to be present in this patient?

A. Anti-SRB
B. Anti-Jo-1
C. Anti-Mi-2
D. Anti-Ku
E. Anti-La

Correct choice: C. Anti-Mi-2

Explanation: This patient has dermatomyositis. Anti-Mi-2 antibodies in DM correlate with the
presence of a shawl sign, cuticular changes, and good prognosis. Anti-Jo-1 antibodies correlate
with pulmonary fibrosis, Raynaud's, and polyarthritis. Anti-SRP antibodies correlate with cardiac
disease and poor prognosis. Anti-Ku antibodies correlate with sclerodermatomyositis. Anti-La
antibodies correlate with Sjogren's syndrome.

120 - The syndrome that is associated with the disease shown in this image is known as:

A. SAPHO syndrome
B. Follicular occlusion tetrad syndrome
C. LEOPARD syndrome
D. NAME syndrome

485
E. PAPA syndrome

Correct choice: E. PAPA syndrome

Explanation: PAPA syndrome ( Pyogenic Arthritis, Pyoderma gangerosum and Acne) The image
shows classic ulcerative pyoderma gangrenosum with undermined violaceous gray border. Re-
epithelialization occurs from the margins and the ulcer heal usually with atrophic cribriform
pigmented scar. Although the classic morphologic clinical presentation of pyoderma gangrenosum
is an ulceration, there are several variants (bullous, pustular, and superficial granulomatous) which
differ by their clinical presentation, location, and associated diseases.

121- A patient with plaque psoriasis complains of morning stiffness of joints lasting longer than 60
minutes. On exam, you notice nail involvement and reduced range of motion of her shoulders, neck,
and lower back. You order an x-ray for further evaluation, to see if this patient has psoriatic arthritis.
Which of the following radiographic features would support a diagnosis of psoriatic arthritis?

A. Osteophyte formation
B. Tuft resorption
C. Subchondral sclerosis
D. Cyst formation
E. Bony enlargement

Correct choice: B. Tuft resorption

Explanation: Radiographic features of psoriatic arthritis include “sausage digits;” large eccentric
erosions; pencil-in-cup deformities – erosive changes of the joint – phalangeal distal tip is pencil on

486
an eroded cuplike joint space; tuft resorption – acroosteolysis; periostitis – inflammation of the
periosteum; and sacroileitis. The other changes are seen on radiograph in osteoarthritis.

122- This is a child's finger, what may be associated with this condition?

A. Osteoma cutis
B. Diabetes
C. Lupus
D. Calcinosis cutis
E. Cryoglobulins

Correct choice: E. Cryoglobulins

Explanation: In kids, pernio can be associated with cryoglobulins and cold agglutinins. It is
triggered by cold and wet which results in acral violaceous color with burning and itching.

123- Which of the following conditions can predispose the patient to warfarin-induced skin
necrosis?

A. Factor V Leiden
B. Protein C deficiency
C. Protein S deficiency
D. Antithrombin III deficiency
E. All of the above

Correct choice: E. All of the above

487
Explanation: All of these conditions are pro-coagulopathic and therefore predispose the patient to
warfarin-induced skin necrosis.

124- What is the most common drug offender in drug-induced bullous pemphigoid?

A. Penicillin
B. Naproxen
C. Propranolol
D. Furosemide
E. Spironolactone

Correct choice: D. Furosemide

Explanation: Furosemide is the most common cause of druginduced BP. Sulfasalazine is another
common offender.

125- A patient with renal amyloidosis, urticaria, fevers, limb pains, and deafness is diagnosed with
Muckle-Wells Syndrome. What is the mutation?

A. MEFV
B. MVK
C. TNF-1 Receptor
D. CIAS1
E. CD2BP1

Correct choice: D. CIAS1

Explanation: Patients with Muckle-Wells has amyloid AA fibrils. It is an autosomal dominant


condition characterized by deafness, hives, and mutation in the CIAS1 gene

126- A 48-year-old white woman presents to your office complaining of a new rash and recent-onset
difficulty walking up and down stairs. On exam you notice peri-ungual erythema of the hands,

488
ragged fingernail cuticles, and poikiloderma on her upper chest and back. Which of the following
would be most important to include in her work up for malignancy?

A. Mammogram
B. Chest X-ray
C. Thyroid ultrasound
D. Transvaginal ultrasound
E. Colonoscopy

Correct choice: D. Transvaginal ultrasound

Explanation: Dermatomyositis is an idiopathic inflammatory disease characterized by myositis and


typical cutaneous manifestations. There is an increased incidence of malignancy in these patients
which may precede, occur with, or follow the diagnosis of dermatomyositis. The most common
form of malignancy in adult white women with dermatomyositis is ovarian cancer, thus a
transvaginal ultrasound is the best answer as this is a common screening test for ovarian cancer.
Other malignancies that have been associated with dermatomyositis include testicular cancer,
gastrointestinal, lung and nasopharyngeal carcinomas. Although they are potentially reasonable
options, neither of the remaining answer choices is the best answer because ovarian cancer is the
most common form of malignancy in adult white women with dermatomyositis.

127- Which of the following is TRUE?

A. Oral cephalexin should provide significant improvement in this lesion.


B. Excision is the most definitive treatment for this.
C. This patient likely has an underlying immunodeficiency.
D. This patient likely has an underlying psychiatric disorder.
E. Histologically, numerous neutrophils are seen at the edge of the ulcer.

489
Correct choice: D. This patient likely has an underlying psychiatric disorder.

Explanation: The correct answer is D- this is a factitial ulcer, which is evidenced by the irregular,
sharp, geometric borders, lack of surrounding inflammation, and prior sites of picking (with
secondary scarring and incomplete alopecia).

128- Which feature of porphyria cutanea tarda is not commonly seen in drug-induced
pseudoporphyria?

A. Varioliform scarring
B. Bullous skin reaction
C. Photosensitivity
D. Pain
E. Hypertrichosis

Correct choice: E. Hypertrichosis

Explanation: Hypertrichosis is rarely reported in drug-induced pseudoporphyria.

129- What is the treatment of choice for delusions of parasitosis?

A. Risperidone
B. Doxepin
C. Sertaline
D. Gabapentin
E. Diphenhydramine

Correct choice: A. Risperidone

Explanation: Of all the answer choices, the second generation antipsychotic Risperidone is the best
for delusions of parasitosis. The rest of the answer choices can be used to treat itchy and neurogenic
pruritus in a number of conditions.

490
130- Which of the following is characteristic of this diagnosis?

A. This is a reaction from a tick bite.


B. This patient likely has rheumatic fever.
C. A topical preparation that inhibits 14 alpha-demethylase will likely clear this.
D. HIstologically, many plasma cells may be seen.
E. Histologically, parakeratosis with superficial and deep perivascular “cuffing” of
lymphocytes may be seen.

Correct choice: E. Histologically, parakeratosis with superficial and deep perivascular “cuffing” of
lymphocytes may be seen.

Explanation: The correct answer is E and describes erythema annulare centrifugum, with the
characteristic “trailing scale” seen clinically. Erythema migrans does not have scale (A) and this is
not the typical appearance for erythema marginatum (B), although both are annular. An azole
antifungal will not help this (C) and plasma cells are not characteristic (E).

131- What is the most frequently affected organ in graft-versus-host disease?

A. Stomach
B. Liver
C. Small intestines
D. Esophagus
E. Skin

correct choice: E. Skin

Explanation: Skin is the most affected organ in both acute and chronic GVHD.

491
132- Which of the following is true regarding this diagnosis?

A. Histologically, many nerve fibers are seen.


B. There is likely polarizable material histologically.
C. This patient was likely born with this.
D. Histologically, there is epidermal hyperplasia and elastic fibers may be decreased.
E. This is often painful.

Correct choice: D. Histologically, there is epidermal hyperplasia and elastic fibers may be
decreased.

Explanation: D is the correct answer. This is an acral fibrokeratoma. A and C describe a


supernumerary digit, which is usually on the ulnar side of the 5th finger

133- Which of the following is the most appropriate initial treatment?

A. Topical permethrin
B. Oral ivermectin
C. Topical clobetasol
D. Topical tacrolimus
E. Oral methotrexate

492
Correct choice: C. Topical clobetasol

Explanation: The correct answer is C. Topical steroids are typically first line treatment for lichen
planus especially if not widespread.

134- Triggering factors in this generalized variant of psoriasis include all of the following except:

A. pregnancy
B. rapid tapering of sytemic therapies
C. hypocalcemia
D. infections
E. topical irritants

Correct choice: E. topical irritants

Explanation: In generalized pustular psoriasis, the infiltration of neutrophils dominates the


histologic picture, while erythema and the appearance of sterile pustules dominate the clinical
picture. It is an unusual manifestation of psoriasis, and triggering factors include pregnancy, rapid
tapering of corticosteroids (or other systemic therapies), hypocalcemia, infections, and, in the case
of the localized pattern, topical irritants. Recently, biallelic mutations in the gene encoding the
IL-36 receptor antagonist (leading to increased production of IL-8 and enhanced responses to
IL-1β) have been identified in a subset of patients with generalized pustular psoriasis. Generalized
pustular psoriasis during pregnancy is also referred to as impetigo herpetiformis.

135- These areas were orange/brown macules that became more raised after examining and
manipulating them, as depicted in the photograph. Which of the following stains is LEAST likely to
help diagnose this condition histologically?

493
A. Toluidine blue
B. Giemsa
C. Tryptase
D. CD117
E. SMA

Correct choice: E. SMA

Explanation: The correct answer is E. The picture shows a form of cutaneous mastocytosis and the
remainder of the choices will help in identifying mast cells histologically. SMA does not stain mast
cells, but a smooth muscle hamartoma is in the differential of a positive Darier sign.

136- A 30-year-old female presents with hypertrophy and nodular enlargement of both lips and
pinpoint red mucosal macules. A mucosal biopsy of the lower lip shows inflammatory hyperplasia
of the labial salivary glands and dilated secretory ducts that appear inflamed. Which of the
following is the most likely diagnosis?

A. Cheilitis granulomatosa
B. Cheilitis glandularis
C. Melkersson-Rosenthal syndrome
D. Actinic cheilitis
E. Pyostomatitis vegetans

494
Correct choice: B. Cheilitis glandularis

Explanation: Cheilitis glandularis is demonstrated by inflammatory hyperplasia of the lower labial


salivary glands that may result from chronic irritation and can arise in association with actinic
cheilitis. The secretory ducts of the salivary glands become dilated and inflamed and appear as
pinpoint red mucosal macules. The clinical presentation varies from slight hypertrophy of the lower
lip to nodular enlargement resulting in eversion. 1 – Cheilitis granulomatosa is a rare, persistent,
painless idiopathic chronic swelling of the lip. It is considered a manifestation of orofacial
granulomatosis – a term describing orofacial swelling caused by non -caseating granulomatous
inflammation in the absence of systemic dise ase. 3 –Melkersson-Rosenthal syndrome is a rare
neurological disorder characterized by recurrent facial paralysis, swelling of the face and lips and a
fissured tongue. 4 – Actinic cheilitis describes the characteristic changes that occur on the lower
vermilion lip of individuals with moderate to severe photodamage. 5 – Pyostomatitis vegetans is
seen in patients with inflammatory bowel disease. It is a chronic vegetate pyoderma of the labial
and buccal mucosa. It may be associated with vegetative or ulcerat ive pyoderma gangrenosum and
is characterized by large verrucous plaques that may be studded with pustules.

137- Which of the following are potential cutaneous manifestations of voriconazole therapy?

A. Cheilitis
B. Pseudoporphyria
C. Eruptive lentigines
D. Squamous cell carcinomas
E. All of the above

Correct choice: E. All of the above

Explanation: These are all potential photosensitive reactions to voriconazole. These resolve on
discontinuation of voriconazole. Close

138- A 35-year-old man presents with several round to oval, hypomelanotic macules several
centimeters in diameter on his back. There is associated fine scale apparent upon scratching the
skin. The distribution is symmetric and is mostly on the posterior trunk and shoulders. While the
contrast between the hypomelanotic infected skin and the surrounding normal skin is striking in
darkly pigmented patients, or following a tan in lightly pigmented patients, Wood’s lamp

495
examination may be required to locate all the lesions. Which of the following mechanisms does
NOT play a role leading to decreased melanin synthesis in this condition?

A. Azelaic acid
B. Abnormal melanosome production
C. Decreased melanin synthesis
D. Partial block in melanosome transfer to keratinocytes
E. Destruction of melanocytes

Correct choice: E. Destruction of melanocytes Explanation:

In tinea versicolor, the hypopigmented skin, there is a decreased density of melanosomes within
keratinocytes, but no change in the melanocyte density. Abnormal melanosome production,
decreased melanin synthesis, and a partial block in melanosome transfer to keratinocytes have all
been suggested as underlying defects. Azelaic acid, a competitive inhibitor of tyrosinase produced
by Malassezia furfur, may play a role in the decreased melanin synthesis. Although the differential
diagnosis may include postinflammatory hypopigmentation (e.g. secondary to parapsoriasis),
progressive macular hypomelanosis, and early vitiligo, the diagnosis of tinea versicolor is easily
confirmed by examination of the associated scale in a potassium hydroxide preparation.

1- Azelaic acid, a competitive inhibitor of tyrosinase produced by Malassezia furfur, may play a role
in the decreased melanin synthesis in tinea versicolor.

2-In tinea versicolor, the hypopigmented skin, there is a decreased density of melanosomes within
keratinocytes, but no change in the melanocyte density. Abnormal melanosome production,
decreased melanin synthesis, and a partial block in melanosome transfer to keratinocytes have all
been suggested as underlying defects.

3-In tinea versicolor, the hypopigmented skin, there is a decreased density of melanosomes within
keratinocytes, but no change in the melanocyte density. Abnormal melanosome production,

496
decreased melanin synthesis, and a partial block in melanosome transfer to keratinocytes have all
been suggested as underlying defects.

4-In tinea versicolor, the hypopigmented skin, there is a decreased density of melanosomes within
keratinocytes, but no change in the melanocyte density. Abnormal melanosome production,
decreased melanin synthesis, and a partial block in melanosome transfer to keratinocytes have all
been suggested as underlying defects. 5-Vitiligo is a common depigmentation disorder resulting
from the destruction of functional melanocytes in the affected skin. In patients with vitiligo,
melanocytes are destroyed in the skin, the eyes, and possibly the ears.

139- A 20-year-old female presents with prickling, tingling, burning and a stinging sensation within
30 minutes of water contact, which lasts for up to 2 hours. The symptoms begin on the lower
extremities and then generalize, with sparing of the head, palms, soles and mucosa. She notes that
the symptoms occur irrespective of the water temperature or salinity. On examination, specific skin
lesions are not seen. What is the most likely diagnosis?

A. Polycythemia vera
B. Mastocytosis
C. Hypereosinophilic syndrome
D. Aquagenic pruritus
E. Aquagenic urticaria

Correct choice: D. Aquagenic pruritus

Explanation: Aquagenic pruritus is usually secondary to a systemic disease or another skin disorder
such as urticaria or dermatographism. Primary idiopathic aquagenic pruritus is uncommon.
Aquagenic pruritus presents with prickling, tingling, burning, or stinging sensations
within 30 minutes of water contact, irrespective of its temperature or salinity, and lasts for up to 2
hours. Typically, symptoms begin on the lower extremities and then generalize, with sparing of the
head, palms, soles, and mucosae. On exam, specific skin lesions are not seen. The pathologic
mechanism is unknown although elevated dermal and epidermal level sof acetylcholine, histamine,
serotonin, and prostaglandin E2 have been described.

1 - Polycythemia vera: Aquagenic pruritus can sometimes be secondary to a systemic disease such
as polycythemia vera. However, the clinical scenario is a classic description of aquagenic pruritus,
not polycythemia vera which presents with a ruddy complexion.

497
2 - Mastocytosis: Mastocytosis usually presents with cutaneous lesions and a positive Darier's sign.
In patients with aquagenic pruritus, specific skin lesions are not seen.

3 - Hypereosinophilic syndrome: Cutaneous lesions are seen in > 50% of patients with
hypereosinophilic syndrome. Hypereosinophilic syndrome is on the differential diagnosis of
pruritus or prickling sensation provoked by water contact. In patients with aquagenic pruritus,
specific skin lesions are not seen.

5 - Aquagenic urticaria: Aquagenic pruritus can sometimes be secondary to another skin disorder
such as urticaria or dermatographism. However, the clinical scenario is a classic description of
aquagenic pruritus, not aquagenic urticaria. In aquagenic pruritus, specific skin lesions are not seen.

140- You have a patient on both methotrexate and adalimumab and want to avoid live vaccines
while they are immunosuppressed. Which of the following vaccines is NOT a live vaccine?

A. Oral polio vaccine


B. MMR (measles, mumps, rubella)
C. BCG
D. Rabies
E. VZV

Correct choice: D. Rabies

141- What is the best treatment for this condition?

A. Dermabrasion
B. Aggressive chemical peels

498
C. Photoprotection
D. Tanning beds
E. Hydroquinone

Correct choice: C. Photoprotection

Explanation: This is melasma and the best treatment option is aggressive photoprotection with hats
and sunscreen. Hydroquinone may work if it is only epidermal melasma. Melasma is made worse
by any irritation of the skin therefore aggressive procedures and treatments are typically not
recommended.

142- Which drug is a frequent cause of drug-induced pemphigus?

A. Gold
B. Penicillamine
C. TMP-SMX
D. Penicillin
E. Vancomycin

Correct choice: B. Penicillamine

Explanation: Penicillamine is a frequent cause of drug-induced pemphigus.

143- This patient presents with the following rash. What is the most likely diagnosis?

499
A. Contact dermatitis
B. Scleroderma
C. Eczema
D. Seborrheic dermatitis
E. Dermatomyositis

Correct choice: E. Dermatomyositis

Explanation: This patient has the classic heliotrope rash with shawl sign (photosensitive rash) on the
neck/chest, which is most consistent with dermatomyositis.

There is no vesiculation, scale, erosions, lichenification or other features to indicate classic contact
dermatitis, seborrheic dermatitis or eczematous dermatitis. The rash in the photo is photo-
distributed and suspicious for a connective tissue disease. Scleroderma would have skin tightening
rather than erythematous patches.

144- Which of the following is FALSE regarding this diagnosis?

A. Most patients have a preceding history of the plaque-subtype of this diagnosis.


B. Histologically, numerous neutrophils are seen in the epidermis.
C. Acitretin is an effective therapy.
D. This may progress to erythroderma.
E. This may cause loss of the nails if it progresses to the distal digits.

Correct choice: A. Most patients have a preceding history of the plaque-subtype of this diagnosis.

Explanation: The correct answer is A (A is false); patients with pustular psoriasis uncommonly have
a preceding diagnosis of plaque type psoriasis

500
145- Cheilitis granulomatosa is the rare triad of:

A. Large puffy lips, vagus nerve palsy, lingua plicata


B. Lip swelling, beads of mucus on the surface, tenderness
C. Lip swelling, eyelid swelling, thyroid enlargement
D. Lip swelling, lacrimal gland inflammation, lingua plicata
E. Large puffy lips, facial nerve palsy, fissured tongue

Correct choice: E. Large puffy lips, facial nerve palsy, fissured tongue

Explanation: granulomatous cheilitis has been reported with facial nerve palsy and fissured tongue,
resulting in the condition known as Melkersson–Rosenthal syndrome.

146- Which of the following are considered risk factors for the development of graft-versus-host
disease ?

A. Older age at time of transplant


B. Peripheral blood source
C. HLA incompatible donor
D. Donor is not related to the recipient
E. All of the above

Correct choice: E. All of the above

Explanation: All these are factors related to increased risks of developing graft-versus-host disease.

147- What is the most common ocular finding in this disease with naked granulomas?

A. Iritis
B. Posterior uveitis
C. Cataracts
D. Acute anterior uveitis
E. Blindness

501
Correct choice: D. Acute anterior uveitis

Explanation: This is sarcoid which histologically shows naked granulomas. The most common
ocular finding is acute anterior uveitis. The skin lesions are only found in 25% of patients.

148- Which anticonvulsant and which HLA type would you screen for in Asian patients for higher
risk of this potential complication?

A. Carbamazepine; HLA-B1502
B. Carbamazepine; HLA-B51
C. Phenytoin; HLA-B17
D. Phenytoin; HLA-DR4
E. Lamotrigine; HLA-DQw2

Correct choice: A. Carbamazepine; HLA-B1502

Explanation: The correct answer is A. Han Chinese, Malaysian, and Thai populations are at a
significantly higher risk of carrying the HLA-B1502 allele and developing SJS/TEN from
carbamazepine (but not Japanese or Caucasians).

502
149- A patient presents with this rash and joint pains after starting a new medication. Which of the
following is the most likely?

A. Cefaclor
B. Cefotaxime
C. Cefepime
D. Ceftriaxone
E. Tobramycin

Correct choice: A. Cefaclor

Explanation: Serum sickness-like reactions are most likely secondary to cefaclor.

150- The first-line treatment for the disorder depicted in this image is:

A. high-potency steroids
B. mid-potency steroids
C. low-potency steroids
D. corrective surgery
E. topical calcineurin inhibitors

503
Correct choice: A. high-potency steroids

Explanation: The main aim of therapy for lichen sclerosus is to bring the disease under control as
quickly as possible with the fewest side effects. For initial treatment (after the diagnosis is
established), a potent topical corticosteroid such as clobetasol propionate 0.05% ointment is
prescribed. One possible regimen is daily application of clobetasol propionate 0.05% ointment to
the affected area for 3 months. This alleviates symptoms in the majority of patients, often within
weeks. Subsequently, the corticosteroid is tapered down over 2 weeks and the clinical remission
maintained by occasional application of a corticosteroid, using less than 30 g in any 6-month
period. In some patients there is complete resolution of signs as well as symptoms, with the
exception of the scarring, which is irreversible

151- A 55 year old male with medical history significant for type 2 diabetes mellitus, hypertension,
and 25 pack/year smoking history presents to the emergency department complaining of sudden
onset shortness of breath and hemoptysis. CT scan of the chest/abdomen/pelvis reveals large
bilateral pulmonary artery emboli as well as a portal vein thrombus. He is admitted to the ICU
where his exam is significant for the findings seen in the image. Lab evaluation reveals eosinophilia
and acute kidney injury. What is the most likely diagnosis?

A. Type II Cryoglobulinemia
B. Cholesterol Emboli
C. Warfarin necrosis
D. Henoch Schonlein Purpura
E. Levamisole vasculopathy

Correct choice: B. Cholesterol Emboli

Explanation: The image shows retiform purpura on the the distal lower extremity. Given the history
of pulmonary arterial emboli along with eosinophilia and AKI, this is most consistent with

504
cholesterol emboli (choice 2). Although the classic description of this is in a patient who underwent
a catheterization, this entity can present from fragmentation of an thrombus. The showers of emboli
result in cholesterol deposition in the distal extremities. Type II cryoglobulinemia is a vasculitis, not
a vasculopathy, and would present with palpable purpura as opposed to retiform purpura (Type I
cryoglobulinemia is a true vasculopathy). The patient has no history to suggest warfarin necrosis,
and this entity is similar to calciphylacxis in that it affects more central, fatty areas of the body.
Henoch Schonlein Purpura generally affects children (or adults with ESRD), and is a vasculitis
which manifests with palpable purpura. There is no history of cocaine use to suggest levamisole
vasculopathy.

152- A 32-year-old female presents with a firm, minimally elevated papule that measures 5 mm in
diameter and is hyper pigmented. On palpation, the lesion seems attached to subcutaneous tissue.
On dermoscopy, a central white scar-like patch surrounded by a delicate pigment network is
observed. Which of the following is not a common cause of multiple dermatofibromas?

A. HIV infection
B. Systemic lupus erythematosus
C. Arthropod bites
D. Atopic dermatitis
E. Melanocytic nevi

Correct choice: E. Melanocytic nevi

Explanation: Melanocytic nevi is not a common cause of multiple dermatofibromas. Although some
dermatofibromas are thought to arise at sites of trauma or arthropod bites, their precise etiology is
not known. Multiple eruptive dermatofibromas have been observed in patients with autoimmune
disorders such as lupus erythematosus and atopic dermatitis and in the setting of
immunosuppression (i.e. HIV infection). Clinically, dermatofibromas can be confused with cysts or

505
melanocytic nevi, especially those with fibrosis. However, melanocytic nevi are not a common
cause of multiple dermatofibromas.

1 - HIV infection: Multiple eruptive dermatofibromas have been observed in patients with
autoimmune disorders such as lupus erythematosus and atopic dermatitis and in the setting of
immunosuppression (i.e. HIV infection).

2 - Systemic lupus erythematosus: Multiple eruptive dermatofibromas have been observed in


patients with autoimmune disorders such as lupus erythematosus and atopic dermatitis and in the
setting of immunosuppression (i.e. HIV infection).

3 - Arthropod bites: Some dermatofibromas are thought to arise at sites of trauma or arthropod bites.

4 - Atopic dermatitis: Multiple eruptive dermatofibromas have been observed in patients with
autoimmune disorders such as lupus erythematosus and atopic dermatitis and in the setting of
immunosuppression (i.e. HIV infection).

1563- What would this look like clinically?

A. Violaceous polygonal papules


B. Skin-colored firm fixed dermal nodule
C. Thick hyperkeratotic plaques
D. Scattered small hyperkeratotic papules centered around hair follicles
E. Juicy papules on the fingers

Correct choice: A. Violaceous polygonal papules

506
Explanation: Clinically, these lesions are polygonal papules which are consistent with lichen planus.
On histology, as seen in the photos, lichen planus is characterized by saw-tooth rete ridges and a
lichenoid infiltrate.

154- The following eruption has been associated with which virus(es)?

A. HSV 1 and 2
B. CMV
C. HHV 6 and 7
D. EBV
E. Coxsackievirus A6

Correct choice: C. HHV 6 and 7

Explanation: Pityriasis rosea has been associated with reactivation of HHV 6 and 7.

155- A patient presents with persistent centrofacial erythema, flushing, telangiectasias and skin
sensitivity. You suspect he has erythematotelangiectatic rosacea. Which of the following skincare
recommendations is appropriate to advise this patient for his skin condition?

A. Procedures such as glycolic peels and dermabrasion


B. Use of waterproof cosmetics
C. Use of cosmetics with alcohol, menthols, camphor, witch hazel, peppermint, and
eucalyptus oil
D. Use of astringents, toners, and exfoliators
E. Moisturizers containing humectants and occlusives

507
Correct choice: E. Moisturizers containing humectants and occlusives

Explanation: Moisturizers containing humectants (e.g. glycerin) and occlusives (e.g. petrolatum)
help to repair the epidermal barrier in rosacea. Skincare recommendations for patients with rosacea
include:
o Wash with lukewarm water and use soap-free cleansers that are pH balanced
o Cleansers are applied gently with fingertips
o Use sunscreens with both UVA and UVB protection and an SPF ≥30
o Sunscreens containing the inorganic filters titanium dioxide and/or zinc oxide are usually well
tolerated
o Use cosmetics and sunscreens that contain protective silicones
o Water-soluble facial powder containing inert green pigment helps to neutralize the perception of
erythema
o Moisturizers containing humectants (e.g. glycerin) and occlusives (e.g. petrolatum) help to repair
the epidermal barrier
o Avoid astringents, toners, and abrasive exfoliators
o Avoid cosmetics that contain alcohol, menthols, camphor, witch hazel, fragrance, peppermint, and
eucalyptus oil
o Avoid waterproof cosmetics and heavy foundations that are difficult to remove without irritating
solvents or physical scrubbing
o Avoid procedures such as glycolic peels or dermabrasion

156- Additional manifestations of which disorder range from poikiloderma of the scalp to central
flagellate erythema to erosions and ulcerations:

A. systemic lupus erythematosus


B. scleroderma
C. sarcoidosis
D. dermatomyositis
E. Behcet's disease

508
Correct choice: D. dermatomyositis

Explanation: Additional manifestations of dermatomyositis range from poikiloderma of the scalp to


centripetal flagellate erythema to erosions and ulcerations. It is also important to look for
dermatologic signs of other connective tissue diseases in patients with dermatomyositis, because of
the frequency of overlap syndromes. The most common cutaneous findings would be signs of
overlap with systemic sclerosis, especially limited disease with CREST features (calcinosis,
Raynaud’s phenomenon, esophageal dysmotility, sclerodactyly, telangiectasias), annular lesions
suggestive of subacute cutaneous lupus erythematosus, or nodules suggestive of rheumatoid
arthritis. Cutaneous small vessel vasculitis can occur as an associated finding, especially in patients
with juvenile dermatomyositis.

157- A patient has multiple erythematous/violaceous psoriasiform dermatitis affecting the ears,
nose, hands and feet. The patient is diagnosed with Bazex syndrome. The most appropriate course
of action is to:

A. To do an EGD
B. Use topical corticosteroids
C. To do a CT scan of the abdomen
D. To order lipase and amylase levels
E. To do a CT of the chest

Correct choice: A. To do an EGD

Explanation: A patient with Bazex syndrome also known as Acrokeratosis Paraneoplastica should
have an EGD. Patients can also have nail dystrophy , acquired keratoderma, and the skin findings
usually precede the underlying malignancy. The other listed testing is not required for Bazex
syndrome.

509
158- This 17-year-old boy also reports dark urine, bilateral knee pain, and intermittent abdominal
pain. Which of the following is true regarding this condition?

A. He has a high likelihood of developing permanent renal impairment


B. Azathioprine is the treatment of choice
C. It is more common in adults than children
D. It usually occurs after a respiratory tract infectioon
E. A DIF biopsy showing intravascular IgA deposition is diagnostic

Correct choice: D. It usually occurs after a respiratory tract infectioon

Explanation: This patient has IgA vasculitis, also known as Henoch-Schonlein purpura (HSP). HSP
typically occurs after a respiratory tract infection in children.

HSP is much less common in adults. A DIF biopsy showing perivascular IgA deposition is
diagnostic (C3 and fibrin may also deposit perivascularly). Two percent of patients develop
permanent renal impairment. Treatment is mainly supportive as HSP is typically self-limited.
Corticosteroids may be effective in rapid resolution of renal manifestations and treating joint and
abdominal pain, but they are not proven effective for treating organ manifestations and
complications, such as glomerulonephritis, bowel infarction or intussusception. Mycophenolate
mofetil or cyclosporine A may be better treatment choices in case of renal involvement. Other
immunosuppressive and immunomodulating drugs, such as rituximab and dapsone, are promising,
but larger studies are needed to confirm these findings. Cancer screening should be considered in
older males diagnosed with Henoch-Schönlein purpura.

159- This 36-year-old woman presents to the ED for recent-onset progressive shortness of breath.
She is otherwise healthy and takes no medications. She denies fever, chills, dysphagia, chest pain,
nausea, vomiting, joint pain or muscle weakness. You are called to evaluate the pictured skin
findings. Which of the following autoantibodies is most likely positive in this woman?

510
A. MDA5
B. TIF1-gamma
C. Jo-1
D. SRP
E. Mi-2

Correct choice: A. MDA5

Explanation: This patient's findings of ragged cuticles (Samitz sign), periungual erythema, and
dorsal finger erosions are highly suggestive of the subtype of dermatomyositis (DM) with positive
MDA5 (melanoma differentiation-associated gene 5) autoantibodies. This subtype is usually
amyopathic or hypomyopathic, explaining this patient's lack of muscle weakness. It is also
associated with rapidly-progressive interstitial lung disease, which accounts for her pulmonary
symptoms.

Mi-2 autoantibodies are associated with classic manifestations of DM (heliotrope rash, Gottron
papules) and a positive prognosis. SRP autoantibodies are associated with DM with cardiac
involvement and poor prognosis. TIF1-gamma autoantibodies are associated with paraneoplastic
DM. Lastly, the DM anti-synthetase syndrome often has Jo-1 autoantibodies and is characterized by
pulmonary disease, arthritis, Raynaud's, and myositis.

160- Which of the following is FDA-approved to treat oral ulcers associated with Behcet's
syndrome?

A. Hydroxychloroquine
B. Mycophenolate mofetil
C. Apremilast
D. Azathioprine
E. Methotrexate

511
Correct choice: C. Apremilast

Explanation: The small-molecule phosphodiesterase 4 inhibitor apremilast modulates cytokines that


are up-regulated in Behçet's syndrome. In patients with oral ulcers associated with Behçet's
syndrome, apremilast resulted in a greater reduction in the number of oral ulcers than placebo but
was associated with adverse events, including diarrhea, nausea, and headache. Consequently, in July
2019, apremilast was FDA-approved for the treatment of oral ulcers associated with Behcet's
syndrome. The remaining answers are not FDA-approved to treat oral ulcers associated with
Behcet's syndrome.

161- What is the treatment of choice?

A. Isotretinoin
B. Insulin
C. Minocycline
D. Topical corticosteroids
E. Selenium sulfide

Correct choice: C. Minocycline

Explanation: The picture shows confluent and reticulated papillomatosis. There are coalescing and
hyperkeratotic papules located on the central trunk. Minocycline is the most effective treatment;
however, recurrence is common.Insulin may be needed for patients with acanthosis nigricans, a
cutaneous marker of insulin resistance and metabolic disorders. Selenium suflide is the treatment of
choice for tinea veriscolor. Tinea veriscolor is characterized by hypo or hyperpigmented macules
and papules with fine scale. The diagnosis can be confirmed with a potassium hydroxide prep and

512
visualization of clustered yeast and hyphae.Topical corticosteroids are ineffective for the treatment
of confluent and reticulated papillomatosis. Isotretinoin has been reported to be effective in the
treatment of confluent and reticulated papillomatosis; however, in light of the high likelihood of
recurrence and side effect profile, it is not a first line treatment.

162- This 34-year-old woman reports a long history of Raynaud's phenomenon. The only
medication she takes is albuterol as needed for exercise-induced asthma. On exam, you notice
splinter hemorrhages of the fingernails as well as the pictured findings. Which of the following
antibodies is LEAST likely to be positive in this woman?

A. Anti-nuclear
B. Anti-B2 glycoprotein
C. Anti-cardiolipin
D. Lupus anticoagulant
E. Anti-histone

Correct choice: E. Anti-histone

Explanation: This patient is presenting with signs and symptoms concerning for anti-phospholipid
antibody syndrome (APS). Anti-histone antibodies are typically only present in the setting of drug-
induced systemic lupus erythematosus (SLE), which is extremely unlikely given that this patient's
only medication is albuterol. Common culprit medications for drug-induced SLE are procainamide,
hydralazine, isoniazid, and minocycline. Patients with APS commonly have the remaining listed
positive antibodies. APS is an autoimmune disease characterized by antiphospholipid antibodies
(aPL) associated with thrombosis and/or pregnancy morbidity. Most APS events are directly related
to thrombotic events, which may affect small, medium or large vessels. Other clinical features like
thrombocytopenia, nephropathy, cardiac valve disease, cognitive dysfunction and skin ulcers (called
non-criteria manifestations) add significant morbidity to this syndrome and represent clinical
situations that are challenging. APS was initially described in patients with systemic lupus

513
erythematosus (SLE) but it can occur in patients without any other autoimmune disease. Despite the
autoimmune nature of this syndrome, APS treatment is still based on anticoagulation and
antiplatelet therapy.

163- A plain film showing “net-like calcification” may obviate the need for biopsy, potentially
significantly increasing mortality if secondary ulceration occurs, in which of the following
conditions?

A. Pancreatic panniculitis
B. Scleroderma
C. Wegener’s granulomatosis
D. Calciphylaxis
E. Pseudoxanthoma elasticum

Correct choice: D. Calciphylaxis

Explanation: The correct answer is E. Xrays of the affected limb may demonstrate vascular
calcification within the skin; however, this may also be seen in healthy patients with renal disease
that are not affected by calciphylaxis.
Bone scintigraphy using technetium Tc 99m bisphosphonates in patients with calciphylaxis shows
increased radiotracer uptake in soft tissues throughout the body and is specifically enhanced in
indurated plaques affected by calciphylaxis (but is absent in ulcers due to reduced blood flow at
sites of tissue necrosis). Calciphylaxis is a condition characterised by necrosis (cellular death) of the
skin and fatty tissue. It is seen mainly in patients with end-stage kidney disease. It is also sometimes
called calcific uraemic arteriolopathy or calcific vasculopathy.
In 1981, approximately 50 cases of calciphylaxis were reported in the world literature. Today,
the incidence is estimated at 1 per cent per year in patients undergoing renal dialysis. The mortality
is extremely high, up to 80%, often within several months of onset. The primary cause of death is
from secondary infection of the ulcers, and sepsis. Calciphylaxis is also called calcific uraemic
arteriolopathy or calcific vasculopathy. A deep wedge skin biopsy may be necessary to diagnose
calciphylaxis, as a similar appearance can be seen in other conditions such as necrotizing fasciitis,
Cryoglobulinemia, antiphospholipid syndrome, coumarin necrosis, and vasculitis. Multiple biopsies
may be necessary, with a risk of propagating calciphylaxis. The pathologist looks for calcium
deposited within scarred and blocked blood vessels in the subcutaneous tissue. Perieccrine calcium
deposition may be noted when vascular calcification is absent but may be subtle. There may also be
inflammation of the fat (panniculitis).

514
164- Which of the following is FALSE?

A. This typically occurs on the extremities, especially legs.


B. Histologically, a parakeratotic column with underlying dyskeratotic keratinocytes may
be seen.
C. The plaque (Mibelli) subtype of this diagnosis typically appears in childhood.
D. Treatment of these lesions is usually unsuccessful.
E. The palmoplantar subtype of this diagnosis has the greatest malignant potential of all
subtypes.

Correct choice: E. The palmoplantar subtype of this diagnosis has the greatest malignant potential
of all subtypes.

Explanation: The correct answer is A (A is false). Linear porokeratosis has the greatest risk of
malignant (SCC) transformation of all subtypes of porokeratosis. This photo depicted is
disseminated superficial actinic porokeratosis (DSAP) which is seen on the legs>arms and is very
difficult to treat. E describes the cornoid lamella characteristically seen in porokeratosis. The plaque
(Mibelli) subtype of this diagnosis typically appears in childhood. The palmoplantar subtype of this
diagnosis does not have the greatest malignant potential of all subtypes.

165- What is the treatment of choice?

A. Incision and drainage

515
B. Excision
C. Topical antibiotics
D. Tooth extraction
E. Oral antibiotics

Correct choice: D. Tooth extraction

Explanation: The image is of a dental sinus tract, an intraoral connection with a periapical abscess.
Extraction of the nonviable tooth is essential to treatment. Topical antibiotics are not an effective
treatment for dental abscesses. Oral antibitoics and incision and drainage may be needed in addition
to tooth extraction for infection eradication. An excision would not address the primary problem,
which is a chronic dental infection.

166- A patient presents with a rash on his back for a 4-week duration. History elicits a new couch
purchase prior to the onset of this rash. Which of the following allergens is the most likely culprit?

A. Colophony
B. Chromate
C. Dimethyl fumarate
D. Quaternium-15
E. Thiuram

Correct choice: C. Dimethyl fumarate

Explanation: Dimethyl fumarate, the contact allergen of 2011, is present in sachets to prevent the
mold formation on shipped furniture. Thiuram is typically present in rubber, latex, and adhesives.
Colophony or rosin is found in adhesive tape, cosmetics, insulating tape, glossy paper, polish,
paints, inks, epilation wax, varnishes, violin bows, and chewing gum. Chromate is found in leather,
cements, and some green felts (pool tables). Quaternium-15 is the most common cosmetic
preservative to cause ACD. It is a preservative that releases formaldehyde.

167- Which of the following is LEAST likely to produce the pictured skin finding?

A. Pityriasis rubra pilaris

516
B. Mycosis fungoides
C. Psoriasis
D. Seborrheic dermatitis
E. Atopic dermatitis

Correct choice: D. Seborrheic dermatitis

Explanation: This man has erythroderma. Seborrheic dermatitis is a rare cause of erythroderma in
young children, and would be an extremely unlikely cause in an adult. Pre-existing dermatoses
(primarily atopic dermatitis or psoriasis), medications, cutaneous T-cell lymphoma (especially the
leukemic form of mycosis fungoides called Sezary syndrome), and pityriasis rubra pilaris are more
common causes of erythroderma than seborrheic dermatitis in adults.

168- Which of the following is a feature of Schnitzler's Syndrome?

A. Monoclonal IgG gammopathy


B. Thromboembolic events
C. Bronchospasm
D. Hematuria
E. Sensorimotor neuropathy

►E

Schnitzler's syndrome presents as episodes of urticarial vasculitis that occur in association with a
monoclonal IgM M component. Fever, lymphadenopathy, hepatosplenomegaly, bone pain, and
sensorimotor neuropathy also occur.

517
169 -TH2 immune responses:
A. Are associated with cell-mediated immunity
B. Produce IL-6
C. Produce IFN-gamma
D. Produce TNF-beta
E. Produce IL-2

►B

TH1 cells produce IL-2, IFN-gamma, and TNF-beta, and are associated with cell-mediated
immunity. TH2 cells produce IL-4, IL-5, IL-6, IL-10, and IL-13, and are associated with antibody-
mediated immune responses.

170 -What is the treatment of choice for this condition?

A. Nystatin
B. Fluconazole
C. Better oral hygiene
D. Penicillin
E. Acyclovir

►C

Black tongue is associated with poor oral hygiene, the use of medications, and radiation to the
head and neck region. In many cases, simply brushing the tongue with a toothbrush or using a
commercially available tongue scraper is sufficient improve the condition.

171-The differential diagnosis for the attached image should include all of the followings except

518
A. Candidiasis
B. Lichen planus
C. Contact dermatitis
D. Inverse psoriasis
E. Erythrasma

►E

The image shown is inverse type of psoriasis. Although psoriatic lesions are classically distributed
on the extensor surfaces (the elbows, knees, and lumbosacral regions), lesions may also be found in
a flexural distribution with involvement of the axillae, groin, perineum, central chest, and umbilical
region. Differential diagnosis include all other choices in question. Although erythrasma might be
included in differentials in case of groin involvement only, it is unlikly for eryhrasma to presnt on
penis with well demarctaed erythamtos plaques. Erythrasma is a superficial bacterial infection of
the skin caused by C. minutissimum. It is characterized by asymptomatic, well-demarcated, reddish
brown, slightly scaly patches in the groin, axillae, gluteal crease, or inframammary regions, and less
often the interdigital spaces of the feet. Erythrasma is frequently confused with a dermatophyte
infection, it can be differentiated from tinea infection by the characteristic coral red fluorescence
seen when viewed under Wood's lamp illumination (due to the production of porphyrins by the
corynebacteria.

172-Which of the following is a progestin used in oral contraceptives with low intrinsic androgenic
properties?

A. Norgestrel
B. Novicane
C. Norgestimate
D. Levonorgestrel
E. Spironolactone

►C

Desogestril, norgestimate and gestodone are progestins with low intrinsic androgenic properties.

519
173 -Patients that have this form of acne is seen affecting young males with suppurating nodules
and plaques that ulcerate and form blackish eschar seen on the trunk most of the time. These
patients can have leukocytosis, fever, arthralgia and lytic changes in the bone most commonly
affecting the:

A. Sternoclavicular joint and chest wall


B. Shoulder blades and sternum
C. Elbows and knees
D. Neck and sternum
E. Jaw and frontal bone

►A

Patients with acne fulminans can have severe acne lesions with lytic lesions indicative of sterile
osteomyelitis seen on x-ray and bone scans. The sternoclavicular joint and the chest wall are most
frequently affected. Treatment includes oral prednisone, antibiotics and isotretinoin.

174- Efficacy of oral contraceptives is reduced by co-administration with which of the following
drugs?

A. Minocycline
B. Doxycycline
C. Rifampin
D. TMP-SMX
E. All of these answers are correct.

►C

Rifampin is a potent hepatic microenzyme inducer and has been shown to reduce efficacy of oral
contraceptives. Anecdotal reports of decreased contraceptive efficacy on oral antibiotics have not
been confirmed in the literature. The putative mechanism is decreased absorption of hormones
secondary to altered gut microflora.

175- Secondary systemic amyloidosis:

520
A. Classically involves the tongue and periorbital skin
B. Involves deposition of AL protein
C. Can involve deposition of beta 2-microglobulin in the setting of rheumatoid arthritis
D. Can be noted on biopsy of normal skin
E. Involves deposition of keratin-derived amyloid

►D

Secondary systemic amyloidosis presents with deposition of amyloid in the adrenals, liver, spleen,
and kidney as a result of some chronic disease, such as TB, leprosy, Hodgkin's, Behcet's,
rheumatoid arthritis, ulcerative colitis, schistosomiasis, or syphilis. The skin is not involved. AA
amyloid fibrils, derived from SAA protein (an acute phase reactant) are deposited. AA is also seen
in Muckle-Wells and familial mediterranean fever. Biopsy of normal skin may be positive for
perivascular amyloid. Dialysis-related amyloidosis occurs via deposition of beta 2microglobulin
component altered by uremia, and resuls in carpal tunnel syndrome, bone cysts, and
spondyloarthropathy.

176- A 35 year old man with a history of celiac disease presents with a beefy, red tongue,
hyperpigmented palmar creases, and premature grey hair. Which of the following statements are is
correct?
A. The best therapy is riboflavin 5mg/day
B. This condition mimics folate deficiency
C. It is often associated with carcinoid tumors which divert tryptophan to serotonin
D. This condition can be caused by azithioprine, 5-FU, and isoniazid
E. Eating raw egg whites is a risk factor

►B

This condition is vitamin B12 deficiency and is characterized by glossitis and hyperpigmentation in
sun exposed areas and creases. Neurologic abnormaolities and megaloblastic anemia can be seen.
The symptoms can mimic folic acid deficiency. Riboflavin (B2) is associated with oralocular-
genital syndrome. Carcinoid tumors as well as azathioprine, 5-FU, and isoniazid are associated with
niacin deficiency. Eating raw eggs is a risk factor for biotin deficiency.

521
177- Which of the following HLA types is associated with early onset psoriasis?

A. HLA-B13 or HLA-B17
B. HLA-B27
C. HLA-B57, HLA-Cw6, or HLA-DR7
D. HLA-Cw2
E. HLA-DR3

►C

HLA-B57, HLA-Cw6, or HLA-DR7 are most commonly associated with early onset type I
psoriasis. The presence of HLA-B13 or B17 is associated with a 5-fold risk of developing psoriasis
and are increase in guttate and erythrodermic psoriasis. HLA-B27 may be seen in pustular psoriasis.
HLA-Cw2 is seen with late onset psoriasis, or type II. HLA-DR3 is commonly found with subacute
cutaneous lupus

178- Monotherapy for acne with topical antibiotics is discouraged because of:

A. Slow onset of comedolytic action


B. Potential for irritation
C. Lack of anti-inflammatory action
D. Potential for bacterial resistance
E. Poor patient compliance

►D

Topical antibiotics reduce the population of P. acnes on the skin, and thus are indirectly anti
inflammatory. In contrast to topical retinoids, topical antibiotics are not comedolytic. They are
generally well tolerated by patients.

179 -What condition does this patient have given the extent of arthritis seen in this X-ray?

A. Antiphospholipid syndrome
B. Multicentric reticulohistiocytosis

522
C. Alpha-1-antitrypsin syndrome
D. Eosinophilic fasciitis
E. Dermatomyositis

►B

Multicentric reticulohistiocytosis is a non-Langerhans histiocytosis associated with mutilating


arthritis. Dermatologically, patient may present with coral beading around the fingers. There is a
30% incidence of malignancy in these patients.

180- A 16 year old developmentally normal male presents to his pediatrician intermittent vague
epidodes of hand and feet paresthesias and non specific episode of GI distress. He is referred to you
to evaluate numerous punctate to 5 slightly verrucous, deep-red to blue-black papules distributed
diffusely on his trunk in a bathing suit distribution. Polarization microscopy of the sediment of his
urine demonstrates birefringent lipid globules (ie, renal tubular epithelial cells or cell fragments
with lipid inclusions) with the characteristic Maltese cross configuration. How is this disorder
inherited?

A. Autosomal dominant
B. Autosomal recessive
C. X-linked dominant
D. X-linked recessive
E. Not an inherited disorder

►D

Fabry's disease is inherited is x-linked recessive. Female carriers often exhibit mild forms of this
disorder. It can be establish that a patient has FD by searching for low activity of alpha galactosyl A
in plasma, leukocytes, cultured skin fibroblasts, or dried blood spots on filter paper. Because of the
Lyon effect, enzymatic detection of carriers can be misleading; thus, specific genetic analysis can be
helpful in making the diagnosis.

523
181- Mutations in the MEFV gene product, pyrin, produce an autosomal recessive syndrome
characterized by recurrent fevers, peritonitis, pleuritis, arthritis and erysipelas-like erythema. Which
of the following syndromes is described above?

A. Familial Mediterranean Fever


B. PAPA syndrome
C. TNF receptor associated periodic syndrome
D. Hyper IgD syndrome
E. Familial cold autoinflammatory syndrome

►A

Familial Mediterranean Fever is described above. It is autosomal recessive with incomplete


penetrance. The main therapy is Colchicine. PAPA syndrome, TNF receptor associated periodic
syndrome, hyper-IgD syndrome and familial cold autoinflammatory syndrome are related
autoinflammatory syndromes. PAPA syndrome is Pyogenic Arthritis, Pyoderma gangrenosum and
Acne and is caused by proline serine threonine phosphatase-interacting protein (PSTPIP1) or CD2-
binding protein 1 (CD2BP1) which interact with pyrin. TNF receptor associated periodic syndrome
(TRAPS) has similar findings to FMF, but the attacks are of longer duration, is autosomal dominant
in transmission, and does not respond to colchicine. TRAPS is caused by a mutation in the
TNFRSF1A gene which results in a decrease in soluble TNF receptor. Hyper IgD syndrome is
associated with defects in the mevalonate kinase gene, which presents with a hereditary periodic
fever. Familial cold autoinflammatory syndrome is associated with cryopyrin defects and is
characterized by fever, rash, conjunctivitis and arthralgia upon exposure to cold.

182- Which of the following is important in the pathogenesis of acne vulgaris?

A. Activation of toll-like receptor-3 by P. acnes


B. Activation of toll-like receptor-2 by M. furfur
C. Activation of toll-like receptor-2 by P. acnes
D. P. acnes produces lipase which cleaves cholesterol into triglycerides
E. Demodex activates complement

►C

524
Acne vulgaris is a disease of follicular hyperkeratosis and the microcomedone is thought to be the
precursor lesion. P. acnes has lipase that cleaves triglycerides into free fatty acids. P. acnes can
activate complement and PMN chemotaxis. Toll-like receptors (TLR) recognize bacterial patterns
and P. acnes activates TLR-2.

183- A patient is diagnosed with glucagon secreting tumor of the pancreas with painful scaling
erythematous patches and plaques at pressure and friction sites. The cells that involved in t his
tumor are:

A. alpha islet cells


B. beta islet cells
C. glycon cells
D. insulin cells
E. gamma islet cells

►A

The cells secreting the glucagon tumor are the alpha islet cells of the pancreas. This patient has
necrolytic migratory erythema which is associated with pancreatic glucagonoma, malignancies,
liver disease, malabsorption states, and inflammatory bowel disease.

184- A patient with holocarboxylase synthetase deficiency is most at risk for developing a
deficiency in:
A. Niacin
B. Pyridoxine
C. Biotin
D. Zinc
E. Essential fatty acids

►C

There are three autosomal recessive syndromes that can lead to biotin deficiency: holocarboxylase
synthetase deficiency, biotinidase deficiency, and an inability to transport biotin into cells. All of

525
these syndromes present with a dermatitis similar to that of zinc deficiency as well as neurologic
abnormalities

185- The most appropriate treatment for the condition shown is:

A. Topical clindamycin 1% lotion


B. Topical metronidazole 0.75% gel
C. Azelaic acid 20% cream
D. Salicylic acid 2% wash
E. Adapalene 0.1% gel

►E

This slide shows comedonal acne. Topical retinoids are the treatment of choice. Salicylic acid
containing cleansers may be helpful adjuncts to treatment with mild comedolytic properties.

186 -A patient with this cutanous finding along with peri-ocular erythema, deltoid weakness and
pulmonary disease is most likely to have which laboratory finding?

A. Anti-Jo-1 antibodies against histidyl-tRNA synthetase


B. Anti-Jo-1 antibodies against nuclear helicase
C. c-ANCA against proteinase-3
D. Lupus anticoagulant

526
E. Anti-histone antibodies

►A

Anti-Jo-1 antibodies against histidyl-tRNA synthetase are described in patients with


dermatomyositis and pulmonary disease. Anti-Mi-2 antibodies against nuclear helicase are
described in classic DM with a good prognosis. Patients with Wegener's disease display cANCA
antibodies against proteinase-3 in a cytoplasmic pattern. Antiphospholipid antibodies are also
known as anticardiolipin antibodies and lupus anticoagulant. They can cause a false positive VDRL.
Clinical features include livedo reticularis, thrombotic events and spontaneous abortions. Anti-
histone antibodies are seen in drug-induced lupus.

187- Natural infection with which of the following infectious agents has been shown to mitigate
atopic dermatitis?

A. Measles
B. S. aureus
C. Varicella
D. Rubella
E. Coxsackie virus

►A

Natural measles infection has been shown to ameliorate atopic dermatitis.

188- A 32 year-old pregnant woman presents for treatment comedonal and moderately
inflammatory acne. You discuss treatment options with her, and she asks about the evidence for
safety in pregnancy of various treatments. According to the FDA classifications, which of the
following treatments either shows no risk to the fetus in controlled studies (but may show risk to
animals), or shows no risk in animal studies (but no human studies have been conducted)?

A. Benzoyl peroxide
B. Trimethoprim-sulfamethoxazole
C. Topical tretinoin

527
D. Azeleic acid
E. Tetracycline

►D

The FDA classifies medication safety for use in pregnancy as follows: Category A: Controlled
studies in humans show no risk to fetus. Category B: Controlled human studies show no risk (but
may show risk to animals), or no risk in animal studies (but no human studies have been
conducted). Category C: Risk to human fetus cannot be ruled out, studies are lacking; animal
studies are equivocal. Category D: Controlled studies show risk, but in some instances benefits may
outweigh risks. Category X: Contraindicated in pregnancy. The question describes category B.
Azeleic acid is a category B medication. Benzoyl peroxide, topical tretinoin, and TMP/SMX are
category C medications. Tetracycline is a category D medication.

189- Kveim-Siltzback test is useful in the diagnosis of:

A. Scarlet fever
B. Sarcoidosis
C. Leprosy
D. Leishmaniasis
E. Chancroid

►B

Kveim-Siltzback test is for sarcoidosis, Montenegro-leishmanin test is for Leishmaniasis, and


lepromin test or histamine/methacholine sweat test can be used for leprosy. Kveim -Siltzback test is
done by intradermal injection of a suspension from granuloma-containing spleen, lymph node, or
other tissue from a confirmed case of sarcoidosis. A positive test is characterized by the formation
of a papule at the site of injection within 4-6 weeks which, on microscopic examination, exhibits
non-necrotizing granulomas and the absence of foreign material. This teast is rarely done nowdays
because of the difficulties involved in preparation, standardization and validation of the test material
as well as significant variation in the sensitivity and specificity of test suspensions obtained from
different sources.

528
190- Patients that have a persistent mongolian spot with indistinct borders that are persistent and or
progressive in behavior may have a sign of:

A. GM1 gangliosidosis type I


B. Handlers Disease
C. Mosaic hypopigmentation
D. Neurofibromatosis type II
E. Nevus anemicus

►A

A patient with a persistent mongolian spot with indistinct borders, persistent or progressive
behavior may have an underlying lysosomal storage disease, most commonly GM1 gangliosidosis
type I and Hurler disease.

191- A patient has multiple erythematous/violaceous psoriasiform dermatitis affecting the ears,
nose, hands and feet. The patient is diagnosed with Bazex syndrome. The most appropriate course
of action is to:

A. Do an age appropriate cancer screening.


B. Use topical corticosteroids
C. To do a CT scan of the abdomen
D. To order lipase and amylase levels
E. To do a CT of the chest

►A

A patient with Bazex syndrome also known as Acrokeratosis Paraneoplastica should have an age
appropriate cancer screening. Patients can also have nail dystrophy , acquired keratoderma, and the
skin findings usually precede the underlying malignancy.

192- A patient with inflammatory bowel disease develops acute tender juicy plaques on the head
and neck, fever, and malaise. The skin lesions respond well to prednisone. Data shows increased

529
ESR and neutrophilic infiltrate with dermal edema on skin biopsy. Which one of the following is a
major criteria for this condition?

A. Fever and malaise


B. History of inflammatory bowel disease
C. Abrupt onset of plaques
D. Increased ESR
E. Good response to prednisone

►C

This condition is called Sweet's syndrome, or acute febrile neutrophilic dermatosis. Diagnosis relies
on two major and two minor criteria. Major ones include 1) an abrupt onset of juicy painful plaques
and bullae and 2) neutrophilic infiltration in the dermis on pathology. Minor criteria include 1)
presence of associated conditions ie inflammatory bowel disease, infections, pregnancy, leukemia,
etc 2) fever and malaise, 3) laboratory values ie high ESR and CRP, and 4) excellent response to
prednisone.

193- Which of the following is associated with Reiter‘s syndrome:

A. Ulcerative colitis
B. Multiple sclerosis
C. HLA-Cw6
D. Asymmetric arthritis
E. Perioral dermatitis

►D

Reiter‘s syndrome is a chronic inflammatory disease similar to psoriatic arthritis. Commonly,


patients present with either a peripheral, asymmetric arthritis, or a urethritis, or conjunctivitis. Not
all of these findings need be present. Patients are usually men with the HLA -B27 genotype. Other
clinical findings include fever, weight loss, keratitis, iritis, and cardiac disease. Skin lesions are
often found on the palms (keratoderma blenorrhagicum _ hyperkeratotic papules and plaques) or the
penis (balanitis circinata). Reiter‘s follows an infection of either the genitourinary tract or the
gastrointestinal tract.

530
194- Which form of sarcoidosis is associated with camptodactyly?

A. Lofgren's syndrome
B. Heerfordt's syndrome
C. Darier-Roussy
D. Mikulicz syndrome
E. Blau syndrome

►E

Camptodactyly is a flexion contracture of the 3rd through the 5th proximal interphalangeal joints
and elbows bilaterally. Blau syndrome is an autosomal dominant form of sarcodosis due to a defect
in the CARD15 gene manifesting also with arthritis, cutaneous sarcoid, uveitis, and synovial cysts.
It does not have lung or visceral involvement.

195 -Increased chylomicrons are a feature of which type of hyperlipoproteinemia?

A. Type I
B. Type IIa
C. Type IIb
D. Type III
E. Type IV

►A

Type I (familial lipoprotein lipase deficiency or apoprotein CII deficiency) hyperlipoproteinemia


has increased chylomicrons as a feature, and presents with eruptive xanthomas and lipemia retinalis.
For the other hyperlipoproteinemias, the associated lipid abnormalities are as follows: Type IIa --
increased LDL; Type IIb -- increased LDL and VLDL; Type III -- increased IDL; Type IV --
increased VLDL. Type V has increased chylomicrons and increased VLDL.

196- Which systemic anesthetic can be safely used in mastocytosis?

A. Lidocaine

531
B. Succinylcholine
C. D-tubocurarine
D. Thiopental
E. Propofol

►E

Local lidocaine can be safely used but systemic lidocaine can precipitate anaphylaxis in patients
with mastocytosis. Other systemic analgesics that are contraindicated in mastocytosis include
succinylcholine, d-tubocurarine, thiopental, metocurine, enflurane, isoflurane, and etomidate.
Propofol, vecuronium bromide, and fentanyl are safe options for systemic anesthesia in patients
with mastocytosis.

197- A patient with spontaneous occurrence of petechiae and purpura, particularly around the eyes,
will also most likely have which of the following?

A. Diffuse scaling of the scalp


B. Acne
C. Cheilits
D. Macroglossia
E. Uveitis

►D

This patient has primary systemic amyloidosis. Mucocutaneous findings can be seen in up to 40%
of patients. The surface of the tongue may be smooth and dry or covered with waxy papules and
nodules. Teeth indentations may be seen along the lateral borders. The most common cutaneous
signs are petechiae and purpura that occur spontaneous or after minor trauma ("pinch purpura"
about the eyelids). The most characteristic skin lesions are waxy, shiny, smooth papules and nodules
which are usually hemorrhagic or flesh-colored. Flexural areas are common sites. These patients
may also present with carpal tunnel syndrome, hepatomegaly, and edema.

198- A teenage girl presents with recurrent upper eyelid edema and upper vermillion and cutaneous
lip swelling, giving her the appearance of having a ―double upper lip.ǁ She reports that the

532
swelling eventually self-resolves over the course of several days. Additionally, she is noted to have
an enlarged, non-tender thyroid gland upon palpation. She is most likely to suffer from which of the
following conditions?

A. Ascher syndrome
B. Hereditary angioedema
C. Sarcoidosis
D. Airborne allergic contact dermatitis
E. Granulomatous cheilitis

►A

Ascher syndrome is a rare syndrome characterized by recurrent upper (and sometimes lower) eyelid
swelling as well as upper vermillion and cutaneous lip swelling. The latter often yields the
appearance of a ―double upper lip,ǁ while multiple episodes of the former results in stretching and
redundancy of the eyelid skin, causing blepharochalasis. Ascher syndrome is frequently seen in
association with a euthryoid (non-toxic) goiter, although up to 50% of patients may have no
associated thyroid abnormality. The pathogenesis of this condition remains unknown.

199- Follicular lichen planus of the skin, multifocal cicatricial alopecia of the scalp and nonscarring
alopecia of the axillary and pubic areas area characteristics of:

A. Graham-Little-Piccardi-Laassueur syndrome
B. Erosive lichen planus
C. Lichen planus pigmentosus
D. Vesiculobullous lichen planus
E. Atropic lichen planus

►A

Graham-Little-Piccardi-Lassuer syndrome has a triad of follicular LP of the skin and or scalp,


multifocal cicatricial alopecia of the scalp and non-scarring alopecia of axillary and pubic areas.

200 -What organism caused this disease?

533
A. Micrococcus sedenditaris
B. Candida albicans
C. Staphylococcus aureus
D. Corynebacterium tenuis
E. Corynebacterium diptheria

►A

Pitted keratolysis is a skin disorder characterized by crateriform pitting that primarily affects the
pressure-bearing aspects of the plantar surface of the feet. It is caused by a cutaneous infection with
micrococcus sedentarius which is included in the Corynebacteria genus. These are grampositive,
catalase-positive, aerobic or facultatively anaerobic, generally nonmotile rods. Corynebacterium
diphtheriae infection is classically characterized by a local inflammation, usually in the upper
respiratory tract, associated with toxin-mediated cardiac and neural disease. Corynebacterium tenuis
causes trichomycosis Patients typically present with yellow, black, or red pinpoint nodules on the
hair shafts in the inguinal region. Staphylococcus aureus is a common bacterium that can result in
formation of pustules, furuncles, cellulitis and abscesses. Candida species are responsible for
superficial infections such as oropharyngeal candidiasis (thrush) and vulvovaginal candidiasis
(vaginal Candidiasis) and is also occurs as an opportunistic infection

201- A 57yo male with hx of hypothyroidism and SCC of the head and neck s/p XRT several years
ago presented with a skin eruption present on his back and upper arms for the past several months.
What is the most likely diagnosis?

A. Tinea corporis
B. Granuloma annulare
C. Metastatic squamous cell carcinoma
D. Radiation dermatitis
E. Mycosis fungoides

534
►B

It is most likely a granulomatous inflammatory disease with infiltrates of macrophages arranged


around focus of degenerated collagen and mucin. The etiology is unknown: metabolic disturbances,
autoimmune, allergy, and infection.

202-A patient has many painful oral erosions/stomatitis with many lichenoid lesions,
erythemamultiforme-like lesions and flaccid bullae. The patient is diagnosed with paraneoplastic
pemphigus and is associated with all of the following malignancy except:

A. Non-Hogkins lymphoma
B. Chronic lymphocytic leukemia
C. Thymoma
D. Castleman's tumor
E. Gastric carcinoma

►E

Patients with paraneoplastic pemphigus has painful oral lesions and polymorphous skin eruption.
The patients die from complications of underlying malignancy. Bronchiolitis obliterans has also
been reported as a complication and cause of death.

203 -Which of the following is NOT associated with this disease of symmetric induration caused
mucin deposition?

A. Diabetes mellitus
B. Streptococcal infection
C. Monoclonal gammopathy

535
D. Hepatitis C
E. All of the answers are associated with this disease

►D

Scleredema is a type of dermal degenerating mucinosis characterized by diffuse symmetric


induration of the upper body. 3 types of scleredema have been described. The first type is seen in
children following a stretococcal infection. The second type is associated with a monoclonal
gammopathy. The third type is related to insulin dependent diabetes.

204- Reiter's syndrome is known as reactive arthritis and involves conjunctivitis, urethritis and
arthritis after a urogenital or gastrointestinal infection. The gene associated with Reiter's syndrome
is:

A. HLA-B27
B. HLA-B32
C. HLA-DQ23
D. HLA-B29
E. HLA-DQ34

►A

Reiter's syndrome is associated with HLA-B27 and can have a variable course often 3-12 months.
There is a chronic sequelae in 15-20% of patients.

205- Patients that are diagnosed with Grover's disease have exacerbation most commonly in the:
A. Winter
B. Fall
C. Summer
D. Spring
E. Humidity

►A

536
Patients diagnosed with Grover's disease have exacerbation commonly in the winter time. Authors
suggest that there may be due to xerotic epidermis and impaired epidermal integrity and decreased
sweat production. It is not caused by sweating and heat and is not increased in summer.

206- Which of the following is a feature of Sneddon's syndrome?

A. Venous thromboses
B. Hepatosplenomegaly
C. anti-Scl-70 autoantibodies
D. M. tuberculosis infection
E. C2 deficiency

►A

Sneddon's syndrome presents as livedo reticularis and livedoid vasculitis associated with ischemic
cerebrovascular lesions, hypertension, and extracerebral arterial and venous thromboses.

207 -The risk of developing systemic involvement in a patient with nodular amyloidosis is:

A. 1-4%
B. 5-10%
C. 25-30%
D. 50-60%
E. >95%

►B

Nodular amyloidosis is a rare type of primary cutaneous amyloidosis made up of AL protein. The
risk for systemic involvement is 7%.

208- Almost all patients with SLE have positive ANAs. A patient can have ANA-negative SLE if
they only make antibodies to:

537
A. ssDNA
B. Sm
C. U1RNP
D. Ro
E. dsDNA

►A

ANA-negative SLE is uncommon. The ANA has a high negative predictive value and low positive
predictive value. ANA-negative SLE can result if a patient only makes antibodies to ssDNA, which
is not detected by most tests.

209- Lichen planus pemphigoides has been associated with an antigen to which structure?

A. Type XVII collagen


B. Hemidesmosome plaque
C. Desmoglein 3
D. Desmoglein 1
E. Type VII collagen

►A

Type XVII collagen, or BPag2 (180 kD) has been associated with LP pemphigoides. The
hemisdesomosome plauqe is also known as BPag1 (230 - kD), desmoglein 1 and 3 are associated
with the Pemphigous family, and antibodies to Type VII collagen is found in Epidermolysis Bullosa
Aquisita

210- First line therapy for subcorneal pustular dermatosis is:

A. Dapsone
B. Tetracycline
C. Vitamin E
D. Topical retinoids
E. Topical steroids

538
►A

First line therapy for subcorneal pustular dermatosis or Sneddon-Wilkinson disease is dapsone
50-200mg/day. Sulfapyridine, acitretin, PUVA, NB UVB, topical and systemic steroids, vitamin E
and antibiotics have been reported to be helpful also.

211 -The most common association with the generalized lesions shown is:

A. Hodgkin's disease
B. Non-Hodgkin's lymphoma
C. Granulomatous mycosis fungoides
D. HIV infection
E. Diabetes mellitus
►E

In a large retrospective study of 1383 patients, diabetes mellitus was diagnosed in 21% of patients
with generalized GA, compared with 9.7% of patients with localized GA. Othere choices also have
been reported to be associated with granuloma annulare.

212- Angiolymphoid hyperplasia with eosinophilia most commonly occurs on the:

A. Ear
B. Lip
C. Dorsal hands
D. Trunk
E. Lower extremities

►A

539
Angiolymphoid hyperplasia with eosinophilia often presents with red to reddish-brown papules or
nodules on the head or neck. While lesions can occur on any body site, most common locations are
the ears and scalp.

213- Xanthoma striata palmaris are diagnostic of:

A. Familial hypertriglyceridemia (type IV)


B. Familial dysbetalipoproteinemia (type III)
C. Familial lipoprotein lipase deficiency (AR)
D. Familial lipoprotein lipase deficiency (AD)
E. Apoprotein CII deficiency

►B

Xanthoma striata palmaris are diagnostic of type III dysbetalipoproteinemia (AR; broad beta
disease). This condition also presents with palmar, planar, tendinous, tuberous, eruptive, and
intertriginous xanthomas, increased IDL, and atherosclerosis. It is associated with diabetes, gout,
and obesity.

214 -Relapsing polychondritis is characterized by circulating auto-antibodies to which of the


following?

A. Collagen type IV
B. Collagen type VII
C. Collagen type II
D. Elastin
E. Chondroitin sulfate

►C

Relapsing polychondritis is a chronic, recurrent rheumatic disease characterized by cartilaginous


inflammation involving the nose, ears, and trachea. A non-erosive inflammatory arthritis may also
be present, and affected patients may suffer from neurosensory hearing loss, tinnitus, and/or vertigo

540
secondary to vestibular or cochlear damage. Patients may have circulating autoantibodies to type II
collagen (found exclusively in cartilage) with circulating titers corresponding to disease activity.

215- Which of the following medications is NOT associated with exacerbating or causing this
eruption?

A. Lithium
B. Prednisone
C. Phenytoin
D. Isoniazid
E. Propranolol

►E

Acne and acneiform eruptions can be caused or exacerbated by cortiosteroids, oral contraceptives,
androgens, ACTH, lithium, phenytoin, halogens, INH, and haloperidol.

216 -What is the treatment of choice for neurotic excoriations?

A. Risperidone
B. Doxepin
C. Olanzapine
D. Gabapentin
E. Diphenhydramine

►B

Doxepin is the treatment of choice for patients with neurotic excoriations. It has both antidepressant
and antipruritic effects. Combined psychiatric and pharmacologic intervention is recommended.
Other agents such as risperidone, olanzapine, gabapentin, and diphenhydramine are not drugs of
choice for this disorder.

217 -Diarrhea, Dementia and a photosensitive dermatitis are associated with a deficiency of which

541
vitamin?

A. Niacin
B. Biotin
C. Thiamine
D. Riboflavin
E. Pyridoxine

►A

Niacin (Vitamin B3) deficiency is associated with a photosensitive dermatitis, diarrhea and
dementia. The photosensitive dermatitis classically involves the face, neck and upper chest
(Casal's necklace) and forearms (as pictured). Other findings may include angular cheilitis and
thickening and hyperpigmentation of skin overlying bony prominences.

218 -What complication can occur if this disease is left untreated?

A. Development of a squamous cell carcinoma


B. Testicular torsion
C. Erectile dysfunction
D. Impotence
E. Pseudo-ainhum

►A

Lichen sclerosis is most commonly located in the genitalia. Longstanding lichen sclerosis is
thought to be a risk factor for the development of a squamous cell carcinoma.

219 -Concomitant discoid lupus is found in what percentage of patients with subacute cutaneous
lupus?

A. 5%
B. 20%
C. 40%

542
D. 60%
E. 80%

►B

Sub acute cutaneous lupus presents with scaly papules that evolve into either psoriasiform or
polycyclic annular lesions (more commonly). Sun-exposed surfaces of the face and neck are
commonly involves. Photo sensitivity is seen in 40%, the hard palate is involved in 40%, and
concomitant DLE is seen in 20%.

220-What is the most common digital manifestation of patients with systemic lupus erythematosus?

A. Periungual telangiectasia
B. Raynaud phenomenon
C. Splinter hemorrhages
D. Sclerodactyly
E. Digital pitting scars

►B

This is the most common digital manifestation in patients with SLE and it could present in up to
30% of the patients. It is a paroxysmal vasospam of digits in response to cold exposure or
emotional stress.

221 -What is the condition which is a diagnostic cutaneous manifestation of sarcoid?


A. Discoid lupus
B. Lupus pernio
C. Lupus vulgaris
D. Granulomatous rosacea
E. Rhinoscleroma

►B

543
Sarcoidosis is a systemic disease charcterized by non-caseating granulomas. Organ involved
include the skin, lung, liver and eyes. Lupus pernio manifests as indurated, red-brown, swollen
plaques of the nose, lips, cheeks and ears.

222 -What is the most likely diagnosis?

A. Dermatofibromasarcoma protuberans
B. Dermatofibroma
C. Lobomycosis
D. Blastomycosis
E. Keloid

►E

Keloids are dense overgrowths of fibrous tissue that generally form a response to tissue injury.
Lobomycosis and dermatofibromasarcoma protuberans may mimic keloids.

223- A 30 year old pregnant female presents with a lesion that has rapidly appeared. She is in her
third trimester with normal pregnancy course to date. What is the most likely diagnosis?

A. Hemangioma
B. Glomus tumor
C. Bacillary angiomatosis
D. Pyogenic granuloma
E. Inflammed wart

►D

544
It is one of several vascular lesions that can present on the skin during pregnancy. Other common
vascular lesions include palmar erythema, spider angiomas, varicose veins, cavernous
hemangiomas, glomus tumors, and hemorrhoids. Etiologic factors in development of vascular
lesions during pregnancy thought to be a combination of increased hormones and increased
intravascular pressure. Pyogenic granulomas may develop as a reaction to trauma. They often
regress after delivery and treatment is not usually required unless they cause excessive bleeding.

224- Nekam's Disease:

A. Is generally responsive to topical and intralesional steroids


B. Characteristically lacks scale
C. Rarely involves the buttocks
D. Presents with a reticulate pattern on the dorsal hands and feet
E. Presents with hypopigmented, atrophic lesions on the extremities

►D

Nekam's Disease (keratosis lichenoides chronica) presents with violaceous papules and nodules,
hyperpigmented and hyperkeratotic, covered with gray scales. There is often a linear and reticulate
pattern on the dorsal hands and feet, extremities and buttocks. This condition is generally very
refractory to treatment.

225- Acquired angioedema is characterized by:

A. Inheritance
B. Normal levels of C1
C. Self-limited course
D. Association with underlying malignancy
E. Increased C1 esterase inhibitor

►D

Hereditary angioedema is an autosomal dominant condition associated with recurrent attacks of


angioedema of the skin, respiratory, and gastrointestinal tract, without urticaria and is caused by a

545
functional deficiency in C1 esterase inhibitor. In contrast, acquired angioedema is caused by a
depletion in C1 esterase inhibitor. There are two forms of acquired angioedema; one results from
the production of antibodies directed against C1 esterase inhibitor, and the other is seen in
association with underlying malignancies, especially B cell lymphomas, with antibody production
against overexpressed paraproteins. Acquired angioedema can be distinguished from hereditary
angioedema by differences in serum levels of C1. While the level is markedly decreased in acquired
angioedema, a normal C1 is seen in patients with hereditary angioedema.

226- This syndrome is a variant of Dowling-Degos disease with reticulated hyperpigmentation on


the flexures and scaly erythematous papules on flexures on the trunk, AD, keratin 5 mut ation. This
syndrome is called:

A. Galli-Galli Syndrome
B. Dowling Degos type I
C. Haim-Munk Syndrome
D. Dengue syndrome
E. Dowling Degos type II

►A

Gali-gali syndrome is a variant of Dowling Degos syndrome. It is characteristic of 1- to 2-mm


slightly keratotic red to dark brown papules which are focally confluent in a reticulate pattern. The
disease is also characterized by slowly progressive and disfiguring reticulate hyperpigmentation of
the flexures.

227- Acquired C1 esterase inhibitor deficiency results in:


A. A normal level of serum C1q
B. A decreased level of serum C1q
C. None of these answers are correct
D. A presentation of angioedema very early in life
E. A normal level of C4

►B

546
Acquired C1 esterase inhibitor deficiency occurs in the setting of lymphoproliferative disease,
monoclonal gammopathy of undetermined significance, or rheumatologic disease and results in a
decreased serum C1q level. Inherited C1 esterase inhibitor deficiency shows an autosomal dominant
inheritance, and earlier presentation, and a normal level of serum C1q. In the inherited disease, the
C1 esterase inhibitor may display normal levels but be functionally impaired.

228- The most common systemic manifestations of systemic sclerosis are:

A. Gastrointestinal
B. Cardiovascular
C. Pulmonary
D. Renal
E. Neurologic

►A

Esophageal dysfunction is the most systemic finding in systemic sclerosis (>90%). Dysphagia
results from decreased peristalsis and may occur before skin findings are seen. Small intestinal
involvement is also common. Pulmonary fibrosis, myocardial fibrosis (seen in 50-70%), cardiac
conduction defects, heart failure, pericarditis with effusion, and renal disease with slowly
progressive uremia may all be seen.

229- The joint most frequently affected in acne fulminans is the:

A. Elbow
B. Intervertebral
C. Distal interphalangeal joints of the hand
D. Sternoclavicular
E. Sacroiliac

►D

Acne fulminans may be associated with lytic bone changes which are indicative of a sterile
osteomyelitis. The sternoclavicular and chest wall joints are most frequently affected.

547
230 -A person that always burns and sometimes tans has the skin phototype:

A. II
B. I
C. III
D. IV
E. V

►A

A person that always burns and sometimes tans is type II skin.

231- What is the most common malignancy associated with this condition in this female?

A. Brain cancer
B. Thyroid cancer
C. Rectal cancer
D. Ovarian cancer
E. Liver cancer

►D

Dermatomyositis is an autoimmune polymyositis with characteristic cutaneous findings. Patients


with dermatomyositis should be screened for a underlying malignancy. Risk factors for having an
underlying malignancy include a negative ANA, adult age, and female gender. Ovarian cancer is
one of the most frequently associated with dermatomyositis.

232- A 40 year-old man presents with a complaint of nail changes for several years. He was treated
by an outside physician with terbinafine without improvement. On further questioning, he reports
morning shoulder stiffness and back pain that lasts 1-2 hours and improves with activity. Which of
the following is true regarding his condition?

A. 50% of patients present with joint disease prior to skin involvement


B. Bony erosions are not commonly seen on radiographs

548
C. Cyclosporine is effective in treating the arthritis in this condition
D. A positive rheumatoid factor may be seen
E. Joint disease correlates with severity of skin involvement

►D

This patient has psoriatic nail changes and a history suggestive of psoriatic arthritis. Psoriatic
arthritis is an inflammatory arthropathy associated with psoriasis. In 80% of patients the rheumatoid
factor is negative; however a positive rheumatoid factor may sometimes be seen. 80% of patients
present with skin disease first. Large eccentric erosions are classically present on
radiographs. Cyclosporine is not effective in treating psoriatic arthritis. Mild skin disease may be
associated with moderate-to-severe joint disease, and vice versa.

233- Which enzyme can be a useful adjunct test to diagnose zinc deficiency where the serum zinc
level is normal or near normal?

A. Zinc sulfatase
B. Alkaline phosphatase
C. AST
D. ALT
E. Creatinine kinase

►B

A low serum alkaline phosphatase, a zinc dependent enzyme, may be a valuable adjuctive test
where the serum zinc level is normal or near normal. The diagnosis of zinc deficiency should be
suspected in at-risk individuals with acral or periorificial dermatitis. It is usually confirmed by low
serum zinc levels.

549
234- Which antibody is 93% specific for Sjogren's syndrome?

A. anti-Ro
B. anti-La
C. anti-fodrin
D. anti-Schirmer
E. RF

►C

Sjogren's syndrome is a triad of keratoconjunctivitis sicca, xerostomia, and rheumatoid arthritis.


More than 90% of patients are women. Labial salivary gland biopsy is useful for diagnosis, and the
Schirmer test for xerostomia detects diminished glandular secretions. Laboratory findings often
include positive cryoglobulins, anti-Ro, anti-La, and RF positivity. Antibodies to fodrin are 93%
specific for this diagnosis.

235- Elastophagocytosis is characteristic of which condition?

A. Sarcoidosis
B. Granuloma annulare
C. Necrobiosis lipoidica
D. Annular elastolytic giant cell granuloma
E. Palisading neutrophilic and granulomatous dermatitis

►D

Annular elastolytic giant cell granuloma is characterized by asymptomatic annular plaques on sun
exposed surfaces. Elastic fibers are absent in the center of lesions. The disease is seen primarily in
middle aged women.

236 -Most common cause of these tender lesions on this 12-year old child is

A. Inflammatory bowel disease


B. Tuberculosis

550
C. beta-hemolytic streptococcal infection
D. Yersinia
E. Infectious mononucleosis

►C

The picture shows classical case of erythema nodosum (EN). Although all other choices in question
cause EN, the most common in the pediatric patient is beta-hemolytic streptococcal infection
(especially pharyngitis). The management of erythema nodosum is directed at identification and
treatment of the underlying cause. Minimal evaluation usually involves obtaining an ASO/DNase B
titer, chest radiograph, and tuberculin testing. Bed rest, with elevation of the patient's legs, helps
reduce pain and edema. When pain, inflammation, or arthralgia is prominent, NSAIDs can be
prescribed. Salicylates, colchicine, and potassium iodide are the most commonly used alternative
therapies.

237- Which of the following is true regarding relapsing polychondritis?

A. Involvement is often bilateral


B. Pathogenic antibodies have not yet been identified
C. The course is chronic, yet mortality is low
D. Both sexes are equally affected
E. Migratory arthralgias are uncommonly seen

►D

Relapsing polychondritis involves intermittent episodes of inflammation of the articular and


nonarticular cartilage, resulting in chondrolysis, dystrophy, and atrophy of the cartilage. Both sexes
are equally affected. IgG anti-type II collagen antibodies are pathogenic, with titers corresponding
to disease activity found in up to 50% of patients with relapsing polychondritis (and in only 15% of
those with RA). Involvement is often unilateral. Migratory arthralgias are seen in 50-80%. The
course is unpredictable, often chronic and variable with episodic flares.
Relapsing polychondritis causes death in 1/3 of patients secondary to airway collapse,
cardiovascular complications, and infection (secondary to systemic steroids).

551
238- The main cause of nutritional disease in developed nations is:

A. Unusual diets
B. Inflammatory bowel disease
C. Malabsorption syndromes
D. Alcoholism
E. Psychiatric illness

►D

Alcoholism is the main cause of nutritional disease in developed nations. Other conditions that
cause nutritional disease include: unusual diets, postoperative state, psychiatric illness,
inflammatory bowel disease, cystic fibrosis, surgical bowel dysfunction, and inborn errors of
metabolism.

239- This patient also has poliosis and deafness. What is the associated ophthalmologic finding?

A. Normal ophthalmologic exam


B. Retinal detachment
C. Lisch nodules
D. Corneal opacities
E. Granulomatous uveitis

►E

This patient has Vogt-Koyanagi-Haradi syndrome (VKHS). VKHS is marked by bilateral


granulomatous uveitis, vitiligo, deafness, poliosis, and alopecia. This syndrome begins with a
meningoencephalitic phase with fever, malaise, headaches, nausea, and vomiting. Varying degrees

552
of neurological impairment may be present. Recovery is usual complete. The second phase is the
ophthalmic-auditory phase during which uveitis appears rapidly and can results in blindness.
Treatment with systemic steroids may prevent blindness. Lisch nodules are iris hamartomas which
are seen in neurofibromatosis. Corneal opacities are seen in X-linked ichthyosis. Retinal detachment
can occur as a result of head trauma, as in shaken-baby syndrome.

240- This tumor has associated pain, tenderness, temperature sensitivity and a reddish spot in the
nail bed:

A. Glomus tumor
B. Pyogenic granuloma
C. Myxoid cyst
D. Osteochondroma
E. Onychomatricoma

►A

Glomus tumors appear as one of the painful tumors that are reddish spots on the nail.

241 -A patient has circular to oval erythematous plaques that are hyperkeratotic with adherent
cartpettack scale. This patient with discoid lupus erythematosus has a risk progression to develop
SLE that is approximately:

A. 5%
B. 15%
C. 20%

553
D. 50%
E. 60%

►A

Patients that have discoid lupus erythematosus have a 5% chance of developing systemic lupus
erythematosus. DLE is complicated by scarring and there is also a risk for development of
squamous cell carcinoma and basal cell carcinoma.

242 -Which contact allergen is found in Krazy glue?

A. Formaldehyde
B. Lanolin alchohol
C. Ethyl cyanoacrylate
D. Octyl-dimethyl-paba
E. Mercaptobenzothiazole

►C

Ethyl cyanoacrylate is a chemical found in Krazy Glue. It can also be found in nail adhesives
causing a dermatitis on the face and fingers, nail dystrophy, and a generalized rash. Formaldehyde is
a widely used chemical found in a variety of applications and is a common allergen. It can be found
in paper, skin and hair products, cosmetics, and permanent press textiles. Lanolin alcohol is found
in wool fat, wool wax, adhesives, cosmetics and pharmaceuticals. Octyl-dimethyl-paba is a
chemical found in sunscreens and is the most frequently used PABA group sunscreen.
Mercaptobenzothiazole is a rubber accelerant and is the most common allergen found in dermatitis
to shoes. It is also found in veterinarian products such as flea and tick sprays and powders.
Mercapotbenzothiazole can also be found in cutting oil, antifreeze, fungicides and photographic
film emulsions.

554
243- A 24 year old female suffers periodic attacks of urticaria and fever. Additionally, she has
suffered from worsening deafness and mild renal failure. Her father and grandfather had similar
symptoms. What abnormally deposited protein is driving her symptoms?

A. AA
B. AL
C. B amyloid protein
D. Keratin
E. Beta 2-microglobulin

►A

Muckle Wells is an autosomal dominant disorder with periodic attacks of urticaria, fever, deafness,
and renal amyloidosis. The implicated protein is AA, which is derived from SAA, an acute phase
reactant.

244-A morbidly obese 52-year-old woman presents with dark brown velvety plaques and
acrochordons on the neck and axillae. Which of the following is most likely to be associated with
her cutaneous findings?

A. Diabetes mellitus
B. Hypertension
C. Hypercholesterolemia
D. Hypertriglyceridemia
E. Peripheral vascular disease

►A

This obese patient has acanthosis nigricans and acrochordons, both of which may be associated with
insulin resistance and diabetes mellitus. While she may very well have hypertensio n,
hyperlipidemia, hypercholesterolemia, and peripheral vascular disease, these are not significantly
related to her cutaneous findings. Acanthosis nigricans may also be seen in patients with gastric
adenocarcinoma.

555
245- In a well-designed study, the impact of psoriasis on health-related quality of life was found to
be similar to which of the following conditions?

A. Diabetes
B. Acne vulgaris
C. Onychomycosis
D. Seasonal allergic rhinitis
E. Hypercholesterolemia

►A

The emotional and physical burden of psoriasis is easil y underestimated. Rapp et al found that
patients with psoriasis report reduced physical and mental functioning (―heath related quality of
lifeǁ) similar to that seen in patients with cancer, hypertension, arthritis, heart disease, diabetes, and
depression.

246- A patient is on hemodialysis for end stage renal disease. After a few years of therapy, he
develops carpal tunnel syndrome, bone cysts, and spondyloarthropathy. What deposited substance is
likely causing his symptoms?

A. Beta 2-microglobulin
B. Uric acid
C. AL
D. Triglyceride
E. AA

►A

The patient described likely has dialysis associated amyloidosis. Beta 2-microglobulin is the protein
component which is involved, and can become deposited in various tissues causing, among other
thing, carpal tunnel syndrome, bone cysts, and spondyloarthropathy.

247- Scalp biopsy of a 44-year-old female with suspected discoid lupus erythematosis would likely
reveal inflammation around which portion of the hair follicle:

556
A. Dermal papilla
B. Hair bulb
C. Matrix
D. Isthmus
E. Infundibulum

►D

On biopsy, alopecia areata exhibits a peribulbar lymphocytic ―swarm of bees.ǁ Discoid lupus
typically exhibits inflammation surrounding the isthmus, along with a perivascular dermatitis and
vacuolar interface changes. Lichen planopilaris exhibits inflammation most densely concentrated
about the infundibulum.

248- Human orf, also known as ecthyma contagiosum, was diagnosed in a 43 year old farmer by an
astute resident dermatologist. Patient presented with a dome shaped, firm bulla with an umbilicated
crust. Which of the following virus is responsible?

A. Pox virus
B. HHV-8
C. Parapox virus
D. HPV 5,8
E. Mycobacterium bovis

►C

Orf, caused by the parapox virus is usually contracted by direct exposure to infected sheep or goats.
Milker’s nodules are caused by a closely related virus found in cows. Both of these viruses can
be contracted by exposure to fomites (fence post, soil) containing the virus. One to several lesions
may develop usually on the hands or forearm and generally resolve without therapy in 4-6 weeks.

249- Which HLA type is more commonly associated with this clinical finding?

A. HLA-B7
B. HLA-B15

557
C. HLA-B27
D. HLA-B51
E. HLA-DR4

►C

The condition shown is balnitis circinata which is part of the constellation of findings in Reiter's
syndrome in addition to arthritis, urethritis, and conjunctivitis. There is a higher incidence of this
condition people with HLA-B27.

250- What bacteria may play a role in the pathogenesis of this


disease?

A. B. burgdorferi
B. H. pylori
C. E. coli
D. S. aureus
E. S. enteritidis

►A

Morphea is a inflammatory disease primarily of the dermis and subcutaneous fat that ultimately
leads to a scar like sclerosis. Clinically, morphea can be divided into plaque, linear, and generalized.
There is some thought that Borrelia infection may play a role in mor phea. mRNA specific for
Borrelia has been found in some lesions of morphea. This association however is controversial.

558
251- Which of the following is a cause of immunologic urticaria?

A. Polymyxin B
B. Amoxicillin
C. Ibuprofen
D. Opiates
E. Tartrazine

►B

Amoxicillin is in the penicillin family of antibiotics and is an immunologic cause of urticaria.


Immunologic urticaria is most commonly caused by exposure to this family and other related beta-
lactam antibiotics. Patients with reaction to penicillins have an increased risk of crossreacting to
cephalosporins, mostly the earlier generations. The third-generation cephalosporins are less likely to
cause reactions in a penicillin allergic patient. The other listed options are causes of non-
immunologic urticaria. They alter prostaglandin metabolism which increases degranulation of mast
cells.

252- What is the most common cause of erythroderma in non-HIV patients?

A. Drugs
B. Underlying malignancy
C. Pre-existing dermatoses
D. Infection
E. Idiopathic

►C

Pre-existing dermatoses is most common cause in non-HIV patients, including atopic dermatitis,
psoriasis, seborrheic dermatitis, chronic actinic dermatitis, mycosis fungoides, pityriasis rubra
pilaris, and allergic contact dermatitis. Drugs are the second most common overall, and the most
common in HIV patients

253- Attached image can be associated with all of the followings except:

559
A. Smooth muscle hamartoma
B. Unilateral breast hypoplasia
C. Acneform lesion
D. Skeletal defect
E. Cardiac defect

►E

Becker's nevus can be occasionally associated with smooth muscle hamartoma, hypoplasia of
underlying structures, such as unilateral breast hypoplasia, unilateral or ipsilateral pectoralis major
aplasia, ipsilateral limb shortening, ipsilateral foot enlargement, spina bifida, scoliosis, pectus
carinatum, localized lipoatrophy, congenital adrenal hyperplasia, polythelia, and accessory scrotum.
In addition to acneform lesions and eczematous dermatitis. There is no association with cardiac
defect.

254- The dermatosis pictured is most commonly associated with:

A. Insulin resistance
B. Cushing's syndrome
C. Acromegaly
D. Polycystic ovarian syndrome
E. Gastric carcinoma

►A

Insulin resistance is the most common cause of acanthosis nigricans. Insulin-like growth factors,
produced by the liver in response to high levels of circulating insulin, bind epidermal growth factor
receptors to produce thickening of the epidermis and hyperkeratosis. Cushing's syndrome,
acromegaly and polycystic ovarian syndrome are associated with acanthosis nigricans, although less

560
common than insulin resistance. Paraneoplastic acanthosis nigricans is associated with
gastrointestinal carcinoma, classically gastric carcinoma.

255- Which antibody is specific for CREST syndrome?

A. Anti-mitochondrial
B. Anti-histone
C. Anti-ds DNA
D. Anti-nucleolar
E. Anti-centromere

►E

The antinuclear antibody (ANA) pattern most specific for CREST is the anti-centromere pattern.
The specificity rate is approximately 50-90% and carries a more favorable prognosis than Scl-70.
The target protein for the anti-centromere pattern is the kinetochore.

256 -Patients with this HLA type, which has the most definitive association with psoriasis, have a
relative risk of having psoriasis that is 9-15 times normal.

A. HLA-B13
B. HLA-B17
C. HLA-Bw57
D. HLA-Cw6
E. HLA-B27

►D

HLA-Cw6 is the HLA type most definitively associated with psoriasis. It carries a relative risk 915
times normal.

257- The protein component of primary cutaneous amyloidosis is:

561
A. SAA protein
B. AL protein
C. Keratin
D. Collagen
E. Bp180

►C

Primary cutaneous amyloidosis presents as either macular or lichen amyloidosis. The protein
component is keratin. Macular amyloidosis often presents over the upper back, while lichen
amyloid presents over the shins.

258- A patient recently underwent parotid surgery and now reports unilateral flushing and sweating
around mealtime. The nerve injured in this syndrome is a branch of the

A. Facial nerve
B. Maxillary nerve
C. Mandibular nerve
D. Cervical nerve
E. Frontal nerve

►C

This patient has Frey syndrome or auriculotemporal nerve syndrome. This is characterized by facial
flushing, sweating, or both localized to the distribution of the auriculotemporal nerve that occurs in
response to gustatory stimuli. In adults, the syndrome usually results from surgical injury or trauma
to the parotid gland.

259- Which allergen is the most likely cause of this eyelid dermatitis?

A. Mercaptobenzothiazole
B. Ethyleneurea melamine formaldehyde
C. Ethylenediamine dihydrochloride
D. Tosylonamide formaldehyde resin

562
E. Benzalkonium chloride

►D

Tosylonomide formaldehyde resin is found nail polish and is a common cause of eyelid dermatitis
in women.

260- This structure can function in reproduction:

A. Conidia
B. Rackets
C. Pectinate bodies
D. Favic chandeliers
E. Mycelium

►A

Conidia function as asexual organs. Favic chandeliers, pectinate bodies, racket forms and mycelium
have no reproductive capabilities.

261- Acrokeratosis paraneoplastica is most commonly associated with which of the following
malignancies:

A. Squamous cell carcinoma


B. Adenocarcinoma

563
C. Transitional cell carcinoma
D. T-cell lymphoma
E. Breast cancer

►A

Acrokeratosis paraneoplastica, also known as Bazex’s syndrome, is a rare paraneoplastic


syndrome. Clinically, it appears as symmetric, hyperkeratotic lesions on red base, in an acral
distribution, nose and helices of the ears. Most commonly associated with upper aerodigestive tract
tumors. Treatment of the tumor leads to disappearance of lesions.

262- This type of psoriasis is associated with more severe body psoriasis. The location of this type
is:

A. Centrofacial psoriasis
B. Palmoplantar psoriasis
C. Scalp psoriasis
D. Nail psoriasis
E. Psoriatic arthritis

►A

Centrofacial psoriasis may be associated with more severe body psoriasis. Palmoplantar psoriasis
can be difficult to treat. Patients that have psoriatic arthritis can be candidates for biologic therapy.

263- A young man presents with explosive onset of severe cystic acne with acute, suppurative
nodules and plaques that ulcerate and form a blackish eschar on the trunk as well as the face. Which
of the following is true regarding this entity?

A. Women are more often affected than men


B. P. acnes osteomyelitis presents with lytic changes on x-rays and bone scans
C. The sternoclavicular joint is often involved in this entity
D. Systemic corticosteroids are contraindicated given risk of sepsis
E. High-dose isotretinoin monotherapy is the treatment of choice

564
►C

Acne fulminans is a rare, explosive form of severe cystic acne affecting young males. Patients may
be systemically ill, with leukocytosis, fever, arthralgias, and myalgias. Lytic changes, indicative of a
sterile osteomyelitis, can be seen on x-ray and bone scans. The sternoclavicular joint and chest wall
are most frequently affected. Treatment is with oral prednisone, intralesional steroids, antibiotics,
and isotretinoin.

264 -Malignancy of the aerodigestive tract is most closely associated with which paraneoplastic
dermatosis?

A. Bazex's syndrome
B. Paraneoplastic pemphigus
C. Sweet's syndrome
D. Acanthosis nigricans
E. Dermatomyositis

►A

Bazex's syndrome (acrokeratosis paraneoplastica) is a paraneoplastic dermatosis associated with


malignancy of the aerodigestive tract. Bazex's syndrome classically presents with violaceous
erythema involving the ears, nose, hands and feet. Lesions progress to become hyperkeratotic and
psoriasiform in appearance. The other answer choices all represent paraneoplastic dermatoses, but
none are as closely associated with aerodigestive tract malignancies as Bazex's syndrome.

265- Which of the following are risk factor(s) for post-transplant CTCL?

A. Renal transplant
B. Liver transplant
C. Cyclosporine therapy
D. Female Sex
E. Renal transplant and Cyclosporine therapy

565
►E

In a recent case series and review (Arch Dermatol. 2010; 146 (5): 513-516) it was shown that the
following are associated with post-transplant CTCL: renal transplant, cyclosporine, tacrolimus, and
male sex. There have been 29 cases of post-transplant CTCL documented in the literature to date.

266- In addition to the lesions seen on the feet of this young man he also had geographic tongue,
erythematous plaques on his penis and arthritis, what HLA type is associated with this syndrome?

A. HLA-B27
B. HLA-B51
C. HLA-DR1
D. HLA-DR3
E. HLA-DQW2

►A

Reiter syndrome, now referred to as reactive arthritis (ReA), is a condition that most often occurs
following enteric or urogenital infections. Reactive arthritis is associated with human leukocyte
antigen (HLA) B27, although HLA-B27 is not always present in individuals who are HIV+.
Bacteria associated with reactive arthritis are generally enteric or venereal and include the
following: Shigella flexneri, Salmonella typhimurium, Salmonella enteritidis, Streptococcus
viridans, Mycoplasma pneumonia, Cyclospora, Chlamydia trachomatis, Yersinia enterocolitica, and
Yersinia pseudotuberculosis.

267- What is the minimum amount of time that you would advise this woman to avoid childbearing
after completing a course of isotretinoin?
A. 1 week

566
B. 1 month
C. 6 months
D. 1 year
E. 3 years

►B

Isotretinoin is a synthetic retinoid that is used primarily in the treatment of acne. Isotretinoin has a
half-life of approximately 20 hours. Women should be advised to avoid becoming pregnant for at
least one month.

268- Which of the following is an innate antimicrobial peptide expressed by keratinocytes in


response to injury or inflammation?

A. Human Beta defensin 1


B. Human Beta defensin 2
C. TNF-A
D. IL-4
E. IL-13

►B

Human-Beta-Defensin (HBD) and cathelicidins are innate antimicrobial peptides in human skin.
HBD-1 is constitutively expressed. HBD-2 (and the cathelicidin LL 37) is expressed in response to
skin injury and inflammation.

567
269 -You suspect that a patient has acquired angioedema. Levels of which of the following would
you expect to be low?

A. C1q, C3
B. C1q, bradykinin
C. C4, C1q
D. bradykinin, C3
E. C4, C3

►C

Initial screening test in angioedema for both inherited and acquired should include C4, which is
low. C3 is normal in angioedema. C1q is low in acquired angioedema but normal in the
hereditary type. Bradykinin is elevated in both inherited and acquired angioedema.

270 -Best treatment option for this stable type of vitiligo is

A. Phototherapy with narrow band UVB


B. Excimer laser
C. Oral prednisone
D. 20% monobenzyl ether of hydroquinone
E. Nitrogen mustard

►D

The picture shows generalized type of vitiligo or vitiligo universalis. Patients who have widespread
disease with only a few areas of normally pigmented skin in exposed sites can be treated with
depigmenting agents. The patients must be carefully chosen, i.e. adults who recognize that their
appearance will be altered significantly and who understand that depigmentation requires lifelong
care of the skin (sunscreens, protective clothing, etc.). The most commonly used agent is
monobenzyl ether of hydroquinone (MBEH) 20% applied twice daily to the affected areas for 9-12
months or longer. Monobenzyl ether of hydroquinone is a potent irritant and/or allergen, and an
open use test should be performed before more widespread application. It normally takes 1-3
months to initiate a response, and a loss of pigment can occur at distant sites. Although
depigmentation from MBEH is considered permanent, repigmentation (especially perifollicular) can
be seen following a sunburn or even intense sun exposure. Monomethyl ether of hydroquinone in a

568
20% cream can be used as an alternative to MBEH. Side effects include contact dermatitis,
exogenous ochronosis and leukomelanoderma en confetti. Phototherapy and excimer laser are not
good or practical choices for this type of vitiligo. Nitrogen mustard is not used in vitiligo.

271 -Carcinoid tumors in which site do not lead to flushing or other manifestations of the
carcinoid syndrome?

A. Appendix
B. Ileum
C. Ascending colon
D. Rectum
E. Bronchus

►D

Carcinoid tumors are derived from enterochromaffin (Kulchitsky) cells. The appendix is the most
common site, followed by the ileum. Less than 4% of abdominal carcinoid tum ors have the
carcinoid syndrome; the presence of the syndrome implies hepatic metastases, extraabdominal
carcinoid tumor, or a large enough tumor burden such that the liver cannot degrade the increased
level of hormone. Foregut tumors (bronchus, stomach, pancreas) produce histamine, cause peptide
ulcers, and produce more persistent, intense flushing, and lacrimation, sweating, vomiting, and
asthma. Midgut tumors (small-intestine to mid-colon) cause cyanotic flush, hypotension, and
bronchoconstriction more commonly. Hindgut tumors (descending colon and rectal) do not lead to
flushing or other manifestations.

272 -The presentation of a foregut carcinoid tumor involves:

A. The production of serotonin, cyanotic flush, and bronchoconstriction


B. The production of histamine, intense flushing, peptic ulcer, and lacrimation
C. Cutaneous findings
D. The production of kallikrein with bronchial asthma and angioedema
E. Frequent episodes of tongue swelling and urticaria

►B

569
Carcinoid syndrome involves tumors derived from the enterochromaffin cells. The appendix is
the most common site. Tumors can be foregut (bronchus, stomach, or pancreas), midgut (small
intestine to mid-colon), or hindgut (descending colon and rectal). Tumors from the foregut area
produce histamine, cause peptic ulcers, and produce intense, persistent flushing and lacrimation.

273 -What is the diagnosis?

A. Psoriasis
B. Lichen planus
C. Balanitis circinata
D. Candida
E. Syphilis

►C

Balanitis circinata presents as sharply demarcated, serpiginous ulcers or plaques on the penile
head. Balanitis circinata is usually seen in Reactive arthritis which has a classic triad of arthritis,
urethritis, and conjunctivitis. The syndrome typically occurs post-infection of the GI or urinary
tract.

274- Which of the following is true regarding neonatal lupus erythematosus?

A. Most cases involve boys


B. Lesions generally resolve spontaneously by 6 months, healing with scarring
C. Photosensitivity is generally not a feature
D. 75% of mothers have symptomatic systemic lupus erythematous at the time of delivery
E. Congenital heart block may be the only manifestation of the disease

570
►E

Neonatal LE presents with annular scaling erythematous macules and plaques on the head and
extremities within the first few months of life in babies born to mothers with LE, rheumatic
diseases, or other connective tissue disorders. 50% of mothers are asymptomatic at delivery.
Lesions resolve spontaneously by 6 months, healing without scarring. Photosensitivity may be
prominent. 75% of cases involve girls. 50% have congenital heart block, which is permanent, and
may be the only manifestation of the disease. Thrombocytopenia and hepatic disease are as frequent
as cardiac disease.

275- Which of the following dermatoses occurs at the latest stage of pregnancy?

A. Darkening of nevi
B. Linea nigra
C. Melasma
D. Areolar hyperpigmentation
E. Psoriasis exacerbation

►B

Linear nigra is one the latest skin changes in pregnancy. It is a hyperpigmented, linear streak which
extends from the pubic symphysis to the xiphoid process. It usually appears from t he 20th week.

276- Which of the following is NOT true regarding primary cutaneous amyloidosis?

A. AA is not the protein component


B. Amyloid is present around blood vessels
C. AL is not the protein component
D. Notalgia paresthetica may be associated with the macular form
E. Amyloid found in benign and malignant neoplasms does not represent primary cutaneous
amyloidosis

►B

571
Keratin is the protein component in primary cutaneous amyloidosis. No amyloid is found around
blood vessels. Macular amyloidosis (which may have associated notalgia paresthetica) and lichen
amyloidosis are forms of primary cutaneous amyloidosis. Secondary cutaneous amyloidosis
presents with keratin-derived amyloid and is seen following PUVA therapy and in benign and
malignant neoplasms.

277 -A 3 year-old girl has a red-yellow papule on her cheek which her mother is extremely anxious
about. Skin biopsy reveals a juvenile xanthogranuloma. Which specialist should you refer the
patient to?

A. Plastic surgeon
B. Endocrinologist
C. Neurologist
D. Ophthalmologist
E. Oncologist

►D

Juvenile xanthogranulomas are the most common form of non-Langerhans cell histiocytoses. 80%
of these lesions appear within the first year of life and they present as firm, red-brown to yellowish
papule. The most common noncutaneous site is the iris, so referral to ophthalmologist is warranted.

278- A patient presents with recurrent genital and oral ulcerations and a diagnosis of posterior
uveitis. What HLA type is associated with the diagnosis you suspect?

A. HLA-B27
B. HLA-B51
C. HLA-DR3
D. HLA-Cw6
E. HLA-DR4

►B

572
The patient has Behcet's disease. Behcet's disease is diagnosed based on recurrent oral ulceration (at
least 3 times in a 12 month period) plus 2 of the following: recurrent genital ulceration, posterior
uveitis, skin lesions (erythema nodosum, pseudofolliculitis, papulopustular lesions, or acneiform
nodules), and a positive pathergy test. Behcet's disease is associated with HLA-B51.

279- What marker is specific for acral melanocytic lesions?

A. b-raf
B. c-myc
C. c-kit
D. Ras
E. p53

►C

C-kit staining is specific for acral melanocytic lesions. Ras is found in all melanocytic lesions. P53
mutations are found in actinic keratoses, SCCIS, and SCC.

280- One of your acne patients has been treated with doxycycline for several months and develops
culture positive gram negative folliculitis. What is the next appropriate therapy

A. Tetracycline
B. Bactrim
C. Isotretinoin
D. Ceftriaxone
E. Cefepime

►C

Gram negative folliculitis may occur after prolonged systemic antibiotic use for acne vulgaris. It
should be suspected in patients who develop a sudden acneiform eruption after having been stable
for some time. The treatment of choice is isotretinoin.

573
281 -The most common associated malignant neoplasm in a nevus sebaceous of Jadassohn is:

A. Basal cell carcinoma


B. Merkel cell carcinoma
C. Squamous cell carcinoma
D. Sebaceous carcinoma
E. Microcystic adnexal tumor

►A

The most common associated malignant neoplasm is basal cell carcinoma in a nevus sebaceous of
Jadassohn.

282- Twenty-nail dystrophy, nail plate splitting and pterygium formation are nail changes seen in:

A. Lichen Planus
B. Darier's disease
C. Psoriasis
D. Scleroderma
E. Dermatomyositis

►A

Twenty-nail dystrophy, nail plate splitting and pterygium formation are nail changes associated with
Lichen Planus. Darier's disease is associated with longitudinal red and white streaks of the nail plate
and V-shaped knicking distally. Psoriasis is associated with many nail abnormalities including
pitting, onycholysis and oil-spots. Scleroderma may cause nail fold capillary dilation and
destruction while patients with dermatomyositis may exhibit nail fold telangiectasias and frayed
cuticles.

283- Inherited Quincke's edema can be detected in the first or second decade and is inherited in an
autosomal dominant pattern. The type II has a:

A. Normal amounts of dysfunctional C1 esterase inhibitor

574
B. Low amounts of normal C1 sterase inhibitor
C. Low C1q
D. Decreased C2 kinin levels
E. Decreased C4 levels

►A

Inherited Quincke's edema have a normal C1q levels. Type I has low amounts of normal C1 sterase
inhibitors and type II has normal amounts of dysfunctional C1 esterase inhibitor.

284- An internal hordeolum can be a painful and erythematous. It is an infection to the meibomian
glands and is caused by:

A. Staphylococcus aureus
B. Actinomyces israelii
C. Borrelia burgdorferi
D. Mycobacterium tuberculosis
E. Streptococcus pyogenes

►A

Internal hordeolum are secondary infection of the Meibomian gland in the tarsal plate. The
infectious agent is Staphylococcus aureus in 90-95% of cases.

285- Pernio or Chilblains is an inflammatory skin condition which is triggered by cold, wet,
nonfreezing environmental conditions. Acral skin has violaceous discoloration accompanied by
burning or itching. While avoidance and prevention is best, the most effective pharmacologic
treatment is:

A. Nifedipine
B. Nicotinamide
C. Phenoxybenzamine
D. Psoralen+UVA
E. Aspirin

575
►A

Nifedipine is effective in about 70% of patients with pernio in prevention of the development of
new skin lesions. The other options listed, other than aspirin, are anecdotally suggested to be
helpful.

286- A 45-year old woman from the United States develops erythema and swelling around her
eyelids and symmetric weakness of her shoulders and hips. What malignancy is overrepresented in
patients with this condition compared to the general population?

A. Ovarian Cancer
B. Lung Cancer
C. Leukemia
D. Uterine Cancer
E. Breast Cancer

►A

This patient has a heliotrope rash and proximal muscle weakness, consistent with the diagnosis of
dermatomyositis. 18-32% of patients with dermatomyositis will develop malignancy. The risk of
malignancy is greatest within the first 3 years of diagnosis. Ovarian cancer is overrepresented. In
Southeast Asia, nasopharyngeal cancer is overrepresented.

287- A potentially dangerous side effect of spironolactone is:

A. Hypokalemia
B. Hyperkalemia
C. Hypercalcemia
D. Hyponatremia
E. Hypernatremia

►B

576
Spironolactone is a potassium-sparing aldactone antagonist. Renal failure predisposes patients to
potentially dangerous hyperkalemia.

288- Probiotics, which are cultures of potentially beneficial gut microflora bacteria, have been
studied in the primary prevention of which of the following diseases?

A. Celiac disease
B. Atopic dermatitis
C. Psoriasis
D. Cutaneous T-cell lymphoma
E. Asthma

►B

Probiotics have been studied in the primary prevention of atopic dermatitis by Kalliomaki et al.
Lactobacillus GG cultures were given to pregnant women with a history of atopy to assess the effect
of potentially beneficial gut flora on the prevention of atopic disease in their children. The
frequency of atopic dermatitis in the children in the probiotic group was half that in the placebo
group at two years of life.

289- The most common laboratory abnormality in patients treated with isotretinoin is:

A. Decreased white blood cell count


B. Increased cholesterol
C. Elevated liver enzymes
D. Hypertriglyceridemia
E. Elevated CPK

►D

The most common laboratory abnormality seen in patients taking isotretinoin is increased
triglycerides, followed by elevation of ALT and AST.

577
290- A 32 year old woman, now 12 weeks pregnant, presents to your office with pruritic scaly
papules and plaques. A biopsy reveals focal spongiosis and parakeratosis in mounds, a superficial
perivascular dermatitis, and extravasated red blood cells in the dermis. Which of the following is
true?

A. It has been associated with EBV.


B. There is often a flare post-partum and during subsequent pregnancies.
C. There is no increased incidence in immunocompromised patients.
D. There is an increased risk of miscarriage in mothers who developed pityriasis rosea
within the first 26 weeks of their pregnancy
E. Acyclovir may be effective in this condition

►E

The question describes a case of pityriasis rosea, which has been associated with HHV6,7. There is
an increased incidence in immunocompromised patients. There is an increased risk of miscarriage
in women who develop pityriasis rosea before 15 weeks of gestation. Acyclovir may be effective in
the treatment of pityriasis rosea, especially if treated in the first week of presentation (JAAD
2006;54(1):82-5).

291- What is the most common paraproteinemia in scleromyxedema :

A. IgA
B. IgG kappa
C. IgG lambda
D. IgM
E. It is uncommon to see paraproteinemia with scleromyxedema

►C

An abnormal paraproteinemia is found in approximately 90% of cases with scleromyxedema,


generally IgG lambda. IgA paraproteinemia can be seen in pyoderma gangrenosum and Sneddon-
Wilkinson disease. Amyloidosis and NXG can be associated with IgG kappa paraproteinemia, and
Schnitzler syndrome is associated with an IgM paraproteinemia.

578
292- Which of the following statements is true regarding this entity?

A. 80% of patients with early onset disease have a positive family history
B. Twin concordance (identical twins) is 55%
C. The inner root sheath and matrix of normal hair express MHC class I
D. The sisapho pattern involves the occipital scalp
E. Atopic dermatitis is a predictor of good prognosis

►B

Twin concordance (identical twins) is 55% in alopecia areata. There is a high frequency of positive
family history, especially in patients with early onset (37%). The inner root sheath and matrix are
immune privileged, not expressing MHC class I, and this privilege may collapse in alopecia areata.
The sisapho or ophiasis inversus pattern of alopecia areata is a bandlike pattern of the fronto
parietotemporal scalp. Atopic dermatitis is a predictor of poor prognosis in patients with alopecia
areata.

293- A patient has on exam multiple thick banded livedo patterns on the shins with painful, stellate,
necrotic and purpuric plaques on the shin. She is diagnosed with calciphylaxis. All the following are
risk for this disease except:

A. Dialysis
B. Warfarin therapy
C. Iron and vitamin D therapy
D. Liver disease
E. Smoking

►E

579
Most patients with calciphylaxis have chronic renal insufficiency and hyperparathyroidism. The
associated risk factors are diabetes mellitus, obestiy, dialysis, warfarin therapy, iron and vitamin D
replacement, weight loss, and liver disease.

294- What is deposited in the upper dermis in this condition?

A. AK
B. AL
C. Transthyretin
D. Beta2-microglobulin
E. AA

►A

Lichen Amyloidosis is a primary cutaneous amyloidosis. Deposits of amyloid of believed to be


derived from necrotic keratinocytes. AL deposits are seen in systemic amyloidosis which is usually
associated with a monoclonal gammopathy or myeloma and the amyloid is derived from
immunoglobulin light chains. Amyloid deposits composed of AA are seen in secondary systemic
amyloidosis such as familial Mediterranean fever and Muckle Wells syndrome, amyloid deposits
composed of transthyretin are seen in familial amyloidotic polyneuropathy and amyloid deposits
composed of Beta2-microglobulin are seen in hemodialysis patients

295 -Kidney disease in Henoch-Schonlein Purpura may be predicted by:

A. Limited skin involvement


B. Spread of purpura to the upper trunk

580
C. Synovial involvement
D. Bullous lesions
E. Facial edema

►B

Henoch-Schonlein Purpura occurs mostly in children. There is an antecedent URI in 75% of cases.
HSP involves the skin, synovia, GI tract, and kidneys. Long-term morbidity results from renal
disease, which is predicted by the spread of purpura to the upper trunk. Skin lesions of adults show
blisters and necrosis.

296- Immunologic abnormalities in atopic dermatitis include:

A. Increased CD8 T-cell number and function


B. Increased secretion of IFN-gamma
C. Decreased expression of CD23 on B cells and monocytes
D. Increased secretion of IL-4
E. Accentuated DTH response

►D

Immunologic abnormalities in atopic dermatitis include increased synthesis of IgE, increased


specific IgE to multiple allergens, increased expression of CD23 (low-affinity IgE receptor) on B
cells and monocytes, increased basophil histamine release, impaired DTH response, decreased CD8
suppressor/cytotoxic T-cell number and function, increased secretion of IL-4 and IL-5 by TH2 cells,
and decreased secretion of IFN-gamma by TH1 cells.

297- Reiter's syndrome is a chronic inflammatory disease similar to psoriasis with arthritis,
urethritis, and conjunctivitis. The most common cause of non-urethral form of Reiter's is:

A. Shigella flexneri
B. Salmonella spp
C. Yersinia spp
D. Ureaplasma urealyticum

581
E. Borrelia burgdorferi

►A

Reiter's syndrome is characteristics of urethritis, conjunctivitis, and arthritis. It occurs in young men
of HLA-B27 genotype that rarely occurs in women.

298- Sarcoidosis presenting with fever, cough, joint pains, hilar adenopathy and erythema nodosum
is known as:

A. Erythema contusiforme
B. Loeffler's syndrome
C. Lofgren's syndrome
D. Darier-Roussy sarcoid
E. Heerfordt's syndrome

►C

Sarcoidosis presenting with fever, cough, joint pains, hilar adenopathy and erythema nodosum is
known as Lofgren's syndrome. Erythema nodosum is the most common nonspecific cutaneous
finding in sarcoidosis. Lofgren's syndrome occurs frequently in Scandinavian whites and is
uncommon in American blacks.

299- Sweet's syndrome may be caused by:

A. Bleomycin
B. Cytoxan
C. Granulocyte colony stimulating factor
D. Intravenous immune globulin
E. Methotrexate

►C

582
Sweet's syndrome is a dramatic skin disease characterized by eruptions of tender, pseudovesicular
coalescing papules and plaques most commonly appearing on the face, neck and upper trunk. It is
seen in association with fever, leukocytosis and typically responds promptly to systemic steroid
therapy. Cutaneous pathergy has been described. The etiology of Sweet's syndrome is unclear. A
hypersensitivity reaction to a bacterial, viral or tumor antigen has been postulated. Nearly 20% of
cases are seen in association with an underlying malignancy, particularly hematologic malignancies,
with acute myelogenous leukemia being the most common. In addition, drug-related variants of
Sweet's syndrome have been described. The following drugs have been implicated: granulocyte
colony-stimulating factor (G-CSF), all-trans retinoic acid, hydralazine, carbamazepine,
levonorgestrel/ethinyl estradiol, trimethoprim/sulfamethoxazole, and minocycline.

300 -A patient has concentric erythematous rings with trailing scale on the trunk and proximal
extremities. It is associated with intense pruritus. The patient is diagnosed with erythema gyratum
repens. The most common cancer that is associated with erythema gyratum repens is:

A. Lung carcinoma
B. Pancreatic carcinoma
C. Stomach cancer
D. Colon cancer
E. Brain cancer

►A

Patients with erythema gyratum repens have a "wood grain appearance" and is most commonly
associated with lung carcinoma. Other cancers associated with erythema gyratum repens is breast,
cervical, bowel, and bladder cancer.

301- The diagnosis is

A. Psoriasis:
B. Alopecia areata
C. Onychomycosis
D. Lichen planus
E. Tetracycline-induced photo-onycholysis

583
►D

Lichen planus-related nail changes seen here include thinning of the nail plate with onychorrhexis,
ridging and pterygium formation.

302- Which of the following leukodermas has a normal number of epidermal melanocytes?

A. Albinism
B. Vitiligo
C. Piebaldism
D. Waardenburg's syndrome
E. Ziprowski-Margolis syndrome

►A

Patients with albinism have a normal number of epidermal melanocytes, however, these
melanocytes synthesize inadequate amounts of melanin. Vitiligo, piebaldism, Waardenburg's
syndrome and Ziprowski-Margolis syndrome all feature a decreased number or total absence of
epidermal melanocytes.

303- What is the most common variant of morphea in children?

A. Plaque
B. Generalized
C. Bullous
D. Deep (morphea profunda)
E. Linear

584
►E

Linear morphea is the most common presentation in children, comprising between 40% to 70% of
children with morphea. This subtype includes linear morphea of the extremity, en coup de sabre, or
progressive facial hemiatrophy, all of which may be accompanied by underlying tissue atrophy.

304- In lichen planus pemphigoides:

A. Bullae develop characteristically in lesions of longstanding lichen planus


B. Circulating IgG antibodies react to the 230 kDa antigen within the basement membrane
zone
C. There is granular deposition of IgG and C3 at the dermoepidermal junction
D. Bullae result from intense lichenoid inflammation and extensive liquefactive
degeneration of basal keratinocytes
E. Vesicles may develop de novo on previously uninvolved skin

►E

Lichen planus pemphigoides presents with tense blisters atop lesions of LP or de novo on
uninvolved skin. It can be differentiated from bullous LP, where blisters develop in lesions of
longstanding LP as a result of intense lichenoid inflammation and extensive liquefactive
degeneration of basal keratinocytes. Histologically it resembles LP, with linear deposition of IgG
and C3 at the DE junction. Circulating IgG autoantibodies react to a 180/200 kDa antigen within the
basement membrane zone.

305- What is the most common extracutaneous manifestation of Sweet's syndrome?

A. Arthralgias
B. Conjunctivitis
C. Renal involvement
D. Sterile osteomyelitis
E. Fever

►E

585
Extracutaneous manifestations of Sweet's syndrome occur in more than 75 percent of patients with
Sweetǁs syndrome. The most common is fever, followed by arthralgias, arthritis, or myalgias. Other
less frequent manifestations include conjunctivitis, episcleritis, oral aphthaelike lesions, cough,
dyspnea, pleuritis, and sterile multifocal osteomyelitis. Cardiac, renal, hepatic, intestinal, or
neurologic manifestations are rare.

306 - Phrynoderma is associated with deficiency of which vitamin?

A. Vitamin A
B. Vitamin B1
C. Vitamin B6
D. Vitamin D
E. Vitamin K

►A

Phrynoderma (toad skin) presents clinically as keratotic papules on the extremities and shoulders
and results from a deficiency of Vitamin A. Vitamin A is a fat soluble vitamin. Hypovitaminosis A is
often seen in diseases such as Crohn's disease, celiac disease and cystic fibrosis which manifest
with fat malabsorbtion. Hypovitaminosis A is additionally associated with night blindness,
xeropthalmia and keratomalacia.

307- The most important mediator of retinoid activity in the skin is:

A. RAR-alpha
B. RAR-beta
C. RAR-gamma
D. RXR-alpha
E. RXR-beta

►C

586
Retinoid activity in humans is mediated by retinoid receptors. Two groups exist: RA receptors
(RAR) and RX receptors (RXR). Each has three receptor subtypes: alpha, beta, and gamma. RAR-
gamma is the most important mediator of retinoid activity in the skin.

308- This patient was started on isotretinoin but failed to discontinue the tetracycline. He is at risk
for what complication?

A. Acne fulminans
B. Pseudotumor cerebri
C. Diarrhea
D. Depression
E. Myositis

►B

Both isotretinoin and tetracycline are known to cause pseudotumor cerebri. In combination, the risk
is significantly elevated. Symptoms of pseudotumor cerebri include headaches, pulsatile tinnitus,
diplopia, and blurred vision.

309 -Which of the following is true regarding syndromes associated with amyloidosis?

A. Familial mediterranean fever involves AL protein


B. MEN IIa involves AA protein
C. The syndrome that presents with renal amyloidosis, fevers, limb pains, and deafness
involves AA protein
D. Familial amyloidotic polyneuropathy type IV involves mutations in apolipoprotein A-1
E. Familial amyloidotic polyneuropathy type III involves mutations in gelsolin

►C

Familial mediterranean fever and Muckle-Wells (renal amyloidosis, urticaria, fevers, limb pains,
and deafness) involve AA. MENA IIa involves keratin-derived amyloid. Familial amyloidotic
polyneuropathy (FAP) type III involves mutations in apolipoprotein A-1; FAP type IV involves
mutations in gelsolin.

587
310- Which of the following is true about nevus anemicus:

A. Usually occurs in association with vitiligo


B. Occurs more frequently in men than in women
C. Most commonly involves the upper chest
D. Results from locally decreased vascular reactivity to catecholamines
E. Contains dilated blood vessels

►C

Nevus anemicus is caused by localized hypersensitivity to catecholamines and most commonly


found in the upper chest

311 -The most common location of dermatofibrosarcoma protuberans is:

A. Trunk
B. Head and neck
C. Extremities
D. None of these answers are correct
E. head and neck and extremities

►A

Dermatofibrosarcoma protuberans is typically located on the trunk. The extremities are the second
most common location for this type of neoplasm.

312 -Which of the following is true regarding topical therapies for psoriasis?

A. Vitamin D3 analogues deactivate salicylic acid


B. Retinoids are effective in decreasing lesional erythema
C. Anthralin can cause irreversible staining of peri-lesional skin
D. Calcipotriol is deactivated by UV light
E. Anthralin inhibits PMNs and monocytes

588
►E

Anthralin, in addition to possessing antiproliferative activity on human keratinocytes, has strong


anti-inflammatory effects by inhibiting PMNs and monocytes. Vitamin D3 analogues are
inactivated by salicylic acid and should be used after UV light (calcipotriol absorbs UV). Retinoids
reduce scaling and plaque thickness, but do not generally decrease lesional erythema. Anthralin can
stain hair purple and cause reversible brownish discoloration of surrounding skin.

313- This woman has restricted ability to open her mouth with tight bound down skin of her hands
and pulmonary fibrosis. In addition to a positive ANA with a nucleolar pattern, what is the most
common autoimmune antibody that is associated with pulmonary fibrosis?

A. Topoisomerase I antibodies (formerly Scl-70)


B. Fibrillarin antibodies
C. Anti-U3RNP antibodies
D. Anti-PM-Scl antibodies
E. Anti-thyroglobulin antibodies

►A

This woman has scleroderma which is a systemic disease characterized by skin induration and
thickening. The cutaneous findings are accompanied by various degrees of tissue fibrosis and
chronic inflammatory infiltration in numerous visceral organs, prominent fibroproliferative
vasculopathy, and humoral and cellular immune alterations. Antinuclear antibodies are present in
about 95% of the patients, usually with a speckled or homogenous pattern. A nucleolar pattern,
although less common, is more specific for systemic sclerosis. Topoisomerase I antibodies
(formerly Scl-70) are present in approximately 30% of patients with diffuse disease (absent in
limited disease) and are associated with pulmonary fibrosis. Anticentromere antibodies are present
in about 60-90% of patients with limited disease and are rare in patients with diffuse disease.

589
Fibrillarin antibodies and antibodies to U3 ribonucleoprotein (RNP) may also be present but are
more common in patients with skeletal muscle involvement. Anti-U3RNP is present mostly in
patients with diffuse disease with overlap syndromes. Anti-ThRNP is present mostly in limited
disease and is associated with more extensive visceral disease. Anti-PM-Scl is present in limited
and overlap states and is associated with myositis and renal involvement.

314- Which of the following is true regarding acquired C1 esterase inhibitor deficiency?

A. This condition generally occurs in the first or second decade of life


B. Serum C1q is normal
C. C2 and C4 are both decreased
D. C1 esterase inhibitor may be at normal levels with functional impairment
E. Positive family history is common

►C

C1 esterase inhibitor is a protease inhibitor that inhibits the catalytic subunits of the first
components of the classical pathway. In the absence of C1 esterase inhibitor, activated C1 and
plasmin generate activated C2 kinin, which mediates angioedema. Acquired C1 esterase inhibitor
deficiency generally affects adults or elderly individuals with no family history. Serum C1q is
decreased. It occurs in the setting of lymphoproliferative disease or rheumatologic illness, where
idiotype/anti-idiotype immune complexes consume available C1q and functionally and
quantitatively lower the amounts of C1 esterase inhibitor. It can also occur in the setting of
autoimmunity directed against the C1 esterase protein. Inherited C1 esterase inhibitor deficiency is
detected in the first or second decade of life and is autosomal dominantly inherited. Serum C1q is
normal in the inherited form, but there is a defect in the synthesis and/or function of C1 esterase
inhibitor. In both the inherited and acquired forms levels of C2 and C4 are decreased because of the
uncontrolled actions of C1s.

315 -A 32 year-old woman presents with moderate hirsutism. She has normal menses, normal-sized
ovaries, no evidence of tumors of the adrenal or ovary, and normal adrenal function, but does have
slight elevations of plasma androstenedione and testosterone. What is the most likely diagnosis?

A. Stein-Leventhal syndrome
B. Cushing syndrome

590
C. Idiopathic hirsutism
D. Occult virilizing tumor
E. Kruckenberg tumor

►C

Idiopathic hirsutism is diagnosed in women with evidence of androgen excess but with normal
menses, normal-sized ovaries, no evidence of tumors of the adrenal or ovary, and normal androgen
function. Slight elevations of plasma androstenedione and testosterone are common.

316- This 10 year-old girl presents to your office. These lesions have been present for months. She
denies any other systemic complaints. What will you tell her parents?

A. They should expect her to get more lesions in non sun-exposed areas
B. She is more likely to develop systemic lupus erythematosus than an adult with these
lesions
C. She is less likely than an adult to develop renal disease
D. She is more likely than an adult to develop cardiovascular disease
E. No hematologic labs are required for evaluation

►B

Discoid lupus erythematosus (DLE) presents with plaques characterized by scarring, atrophy,
follicular plugging, and scale and photosensitivity. Children presenting with DLE have a higher
incident of developing systemic lupus (SLE) than adults. Because of progression from DLE to SLE,
children should be screened and followed with antinuclear antibodies and anti - DNA antibodies.
Children and adolescents have a higher incidence of renal involvement. Treatment for DLE includes
topical steroids, oral steroids, and hydroxychloroquine.

591
317- A 17 year old high school cheerleader returns to clinic for follow up of her acne and
management of her isotretinoin. She denies depression or any GI complications. She is pleased with
her progress but does mention painful red bumps on her lower legs for 1 week. She attributes these
to her cheer practice. What is the most likely cause of her lower extremity lesions.

A. Training regimen
B. trip and fall
C. Oral contraceptives
D. Allergy medication
E. Recent strep infection

►C

The patient is likely experiencing erythema nodosum (EN), a delayed hypersensitivity response to a
wide variety of eliciting factors. They consist of an eruption of erythematous, tender nodules,
typically over the anterior tibial areas. Common causes include oral contraceptives, which is a
probable choice given that the patient is likely taking them as part of her isotretinoin treatment.
Strep infection is also a common cause of EN but with no mention of symptoms is less likely in this
case. Other causes include TB, fungal infections, sarcoidosis, ulcerative colitis, and regional
enteritis.

318- In this patient with an autoimmune disorder, which autoantibody would be indicative of
increased risk of pulmonary disease?

A. U1RNP
B. DsDNA
C. anti-Jo-1 antibody
D. Mi-2 antibody
E. Scl-70 antibody

592
►C

The patient depicted has dermatomyositis. Autoantibodies to anti-Jo-1 antibody targets histidyl
transfer RNA synthetase. In dermatomyositis correlates with the development of pulmonary disease.

319- The most common complication for sarcoidosis is:

A. Anterior uveitis
B. Glaucoma
C. Pars planitis
D. Diplopia
E. Paresis if ocular muscles

►A

The most common eye complication of sarcoidosis is anterior uveitis with mutton fat keratotic
precipitates. All the others can be a complication of sarcoidosis also.

320- Which of the following is a manifestation of psoriasis of the nail matrix?

A. Splinter hemorrhages
B. Oil spotsǁ
C. Subungual hyperkeratosis
D. Pits
E. Onycholysis

►D

Psoriatic nail changes may be of nail matrix or nail bed origin. Pits are the common finding; splinter
hemorrhages the least. Psoriatic nail changes of matrix origin include: pits (representing focal
psoriasis of the proximal matrix) and leukonychia. Psoriatic nails changes of nail bed origin
include: salmon spots, oil spots, onycholysis, subungual hyperkeratosis, and splinter hemorrhages.

593
321- This patient has similar lesions on his distal extremities. Which laboratory test can be done in
order to make a diagnosis?

A. Hemogram
B. Alkaline phosphatase
C. Creatinine
D. Potassium
E. TSH

►B

This patient has acrodermatitis enteropathica, a rare, inhertied disorder caused by an inability to
absorb zinc. This disease is characterized by a traid of acral dermatitis, diarrhea, and alopecia. AE is
rapidly reversed by zinc supplementation. Alkaline phosphatase is a zinc-dependent enzyme; it is a
moderately-sensitive marker for zinc deficiency (although not an early marker).

322- This variant of amyloidosis is derived from degenerated tonofilaments of keratinocytes:

A. Lichen amyloidosis
B. Nodular amyloidosis
C. Primary systemic amyloidosis
D. Dialysis-related amyloidosis
E. Secondary systemic amyloidosis

►A

Lichen amyloidosis and Macular amyloidosis are derived from degenerated tonofilaments of
keratinocytes. Primary systemic amyloidosis results from deposition of protein AL derived from Ig

594
Lambda light chains. Nodular amyloidosis is also associated with AL type protein. Secondary
systemic amyloidosis is associated with AA amyloid fibrils derived from SAA protein. Dialysis
related amyloidosis is associated with beta-2-microglobulin protein deposition.

323- Solid confluent palmoplantar keratosis, salmon-orange follicular papules and diffuse
symmetrical involvement with characteristic small islands of normal skin within affected areas are
all clinical characteristics of what skin disease?

A. Lichen Sclerosis
B. Dermatomyosistis
C. Phrynoderma
D. Reiterǁs syndrome
E. Pityriasis rubra pilaris

►E

PRP can affect any portion of the body, but commonly affect the extensor surfaces of the extremities
as well as the sides of the neck and trunk. It may progress with an almost erythrodermic type state
with characteristic small island of normal skin within affected areas. Phrynoderma refers to the
toadskin like skin seen in some vitamin deficiencies including vitamin A deficiency.

324 -The desmoplastic trichoepithelioma is most often found in this location:

A. Face
B. Arm
C. Hands
D. Neck
E. Scalp

►A

Desmoplastic trichoepithelioma is referred to as sclerosing epithelial hamartoma. The location is


usually on the face of women

595
325- The mechanism of action of azithromycin is:

A. Inhibition of bacterial cell wall synthesis


B. Inhibition of RNA-dependent protein synthesis by binding to the 30s ribosomal subunit
C. Inhibition of RNA-dependent protein synthesis by binding to the 50s ribosomal subunit
D. Inhibition of DNA-dependent RNA polymerase
E. Inhibition of bacterial topoisomerase

►C

Azithromycin is a macrolide antibiotic. It binds the bacterial 50s ribosomal subunit and inhibits
RNA-dependent protein synthesis.

326- After being treated for several months with doxycycline, this patient develops a gram negative
folliculitis. What is the next appropriate therapy?

A. Tetracycline
B. Bactrim
C. Isotretinoin
D. Ceftriaxone
E. Cefepime

►C

Gram negative folliculitis may occur after prolonged antibiotic therapy for acne vulgaris. It should
be suspected in patients who are well controlled and then suddenly flare. The treatment of choice is
isotretinoin.

327 -The arthritis of Behcet's disease is characteristically:

A. Symmetric, erosive polyarthritis


B. Asymmetric, erosive polyarthritis
C. Asymmetric, non-erosive polyarthritis
D. Asymmetric, erosive monoarthritis

596
E. Symmetric, non-erosive polyarthritis

►C

Behcet's disease is characterized by recurrent oral ulceration plus 2 of the following: recurrent
genital ulceration, eye lesions (posterior uveitis), skin lesions, positive pathergy test. Clinical
features include thrombosis of the superior vena cava, thrombophlebitis, CNS lesions that give a
picture of multiple sclerosis, and an asymmetric non-erosive polyarthritis.

328 -Which of the following is most characteristic of seborrheic dermatitis?

A. Greasy, yellow, scaly plaques on the central face and chest


B. Well defined, round, scaly, pink plaques on the knees and elbows
C. Stuck on waxy papules and plaques
D. Pink papules and telangiectasias on the cheeks
E. Heliotrope violaceous periocular patches

►A

Seborrheic dermatitis is characterized by greasy, yellow, scaly plaques involving the scalp, central
face, and chest. It can be treated with medications such as ketoconazole or selenium sulfide.
Psoriasis is defined by well defined round, scaly, pink plaques on the knees and elbows. Seborrheic
keratoses are defined by stuck on waxy papules and plaques. Rosacea may be characterized by pink
papules and telangiectasias on the cheeks. Dermatomyositis is characterized by heliotrope
violaceous periocular patches.

329- A 35 year old man has a history of intensely pruritic papules and vesicles on the extensor
surfaces of his lower extremities. Antibodies to epidermal transglutaminase are detected. What is
this patient at risk for developing?

A. Non-Hodgkin lymphoma
B. Colon cancer
C. Lung cancer
D. CLL

597
E. Esophageal cancer

►A

A number of studies have indicated an increased risk of non-Hodgkins lymphoma and enteropathy
T-cell lymphoma in patients with dermatitis herpetiformis. There is also increased prevalence of
thyroid disease, type I diabetes, and other autoimmune disorders, such as vitiligo, Addison's, and
alopecia areata.

330 -All of the following are true regarding Reiter's syndrome except:

A. The classic clinical triad is urethritis, conjunctivitis and arthritis


B. Usually occurs in young women of HLA-B27 genotype
C. May be associated with keratoderma belnnorrhagicum
D. May be associated with Chlamydia trachomatis
E. Nails may become thick and brittle with heavy subungual hyperkeratotic
deposits

►B

Reiter's syndrome is a chronic inflammatory disease similar to psoriasis with psoriatoc arthritis.
Reiter's syndrome usually occurs in men of HLA-B27 genotype and rarely occurs in women. Few
patients present with the classic triad, thus can be diagnosed with peripheral arthritis >1 month and
associated urethritis. Keratoderma blennorrhagicum is crusted, hyperkeratotic papules and plaques
on plantar surfaces. Painful and bloody urination and pyuria, cystitis, prostatitis and seminal
vesiculitis may occur secondary to Chlamydia trachomatis. Nail lesions as described may occur.

331 -Which of the following would be an important diagnostic sign of this non-infectious disorder?

A. Lacrimal gland enlargement


B. Periungual fibromas
C. Cafe-au-lait macules
D. Peg shaped teeth
E. Dystopia canthorum

598
►A

Sarcoidosis is a non-infectious granulomatous disorder of unclear etiology. Ocular manifestation are


relatively common in sarcoidosis and may include acute anterior uveitis (classic finding), posterior
uveitis, lacrimal gland enlargement, and conjuctival nodules.

332 -A 50-year-old female with sarcoid has enlargement of the parotid, submandibular, and lacrimal
glands that is also known as:

A. Mikulicz's syndrome
B. Heerfordt-Waldenstromn syndrome
C. Darier-Roussy sarcoid
D. Lofgren's syndrome
E. Blau's syndrome

►A

This patient has Mikulicz's syndrome which is sarcoid of the parotid, submandibular and lacrimal
glands. Heerfordt-Waldenstromn syndrome demonstrates a combination of fever, parotid
enlargement, anterior uveitis, and facial nerve palsy. Darier-Roussy sarcoid is characterized by
subcutaneous nodular sarcoid on the trunk and extremities. Lofgren's syndrome features acute
sarcoid, erythema nodosum and migratory polyarthritis, fever, iritis, and bilatera l hilar adenopathy.
Blau's syndrome is a rare autosomal dominant familial granulomatous syndrome that presents with
arthritis, uveitis, and skin lesions that appear as "red dots;" there is no pulmonary involvement.

333- Which antibody is associated with a positive prognosis in dermatomyositis?

599
A. Anti-Jo
B. Anti-Smith
C. SRP
D. Anti-Mi
E. Aldolase

►D

Several antibodies have prognostic importance in dermatomyositis. Anti-Mi is associated with an


improved prognosis. Anti-Jo is associated with a poor prognosis and lung involvement. SRP is also
associated with a poor prognosis and heart involvement.

334 -You are consulted to evaluate this patient with tender, warm plaques on his shins. What other
physical signs should you look for?

A. Exophthalmos
B. Uveitis
C. Tachycardia
D. Clubbing of the fingers
E. Cough

►B

Erythema Nodosum (EN) is represented by tender, warm,nodules and plaques, often but not
exclusivley located on the anterior shin. It represents a reactive panniculitis. Causes include,
infections (Streptococcal, tuberculosis, yersina, mycoplasma, campylobacter,salmonella,
histoplasmosis, blastomycosis, coccidiomycosis), drugs (sulfonamides, gold, and OCP's),
enteropathies, pregnancy, hodgkin's disease and lymphoma and sarcoidosis. Loefgren's disease is a

600
varient of sarcoidosis with EN, hilar adenopathy, fever, uveitis and arthritis. Differential diagnosis
includes: pretibial myxedema and erythema induratum.

335- A 16 year old developmentally normal male presents to his pediatrician intermittent vague
epidodes of hand and feet paresthesias and non specific episode of GI distress. He is referred to you
to evaluate numerous punctate to 5 slightly verrucous, deep-red to blue-black papules distributed
diffusely on his trunk in a bathing suit distribution. Polarization microscopy of the sediment of his
urine demonstrates birefringent lipid globules (ie, renal tubular epithelial cells or cell fragments
with lipid inclusions) with the characteristic Maltese cross configuration. What is the classic ocular
finding in this disorder?

A. Pseudo-herpetic corneal ulcerations


B. Brushfield spots
C. Hyperpigmentation of the retinal epithelium
D. Corneal opacities
E. Lester irides

►D

Whorled corneal opacities are classically seen in Fabry's disease. Pseudo-herpetic ulcerations are
seen in Richner-Hanhart. Brushfield spots can be seen in Down's syndrome. Congenital
hyperpigmentation of the retinal epithelium is seen in Gardner's syndrome. Lester irides are seen in
Nail-Patella syndrome.

336- A young girl presents with recurrent severe arthritis of the ankles. She also has a large ulcer on
her leg and severe acne. Which gene is mutated?

A. PSTPIP1
B. NOD2
C. CIAS1
D. AIRE
E. FOXP3

►A

601
This patient has PAPA (pyogenic arthritis-pyoderma gangrenosum-acne) syndrome. The gene
mutated is PSTPIP1, also known as CD2 binding protein 1 (CD2BP1) which encodes prolineserine-
threonine phosphatase-interacting protein 1. NOD2 is the gene involved in Blau syndrome; CIAS1
with CINCA/Muckle-Wells/Familial cold autoinflammatory syndromes; AIRE with APECED
(autoimmune polyendocrinopathy-candidiasis-ectodermal dystrophy); FOXP3 with IPEX (immune
dysregulation, polyendocrinopathy, enteropathy, X -linked) syndrome.

337- Which of the following is true regarding this entity?

A. The subcutaneous variant is most common in children


B. 75% of localized lesions are still present 2 years after diagnosis
C. Localized lesions often ulcerate and heal with scarring
D. The perforating variant classically involves the lower extremities
E. Diabetes is present in the majority of patients with the generalized variant

►A

Granuloma annulare is the entity pictured. Subtypes include localized GA, generalized GA, macular
GA, subcutaneous GA, and perforating GA. The subcutaneous variant is most common in children;
there is often a history of trauma to the area. Localized lesions never u lcerate and heal without
scarring; 75% of localized lesions clear within 2 years. The perforating variant classically involves
the dorsa of the hands. Diabetes is present in 20% of patients with the generalized variant.

602
338- This radiograph shows a characteristic finding of severe psoriatic arthritis: bone proliferation
at the base of the distal phalanx with resorption of the tufts. What is the most common form of
psoriatic arthritis?
A. Arthritis mutilans
B. Axial
C. Symmetric polyarthritis
D. Asymmetric oligoarthritis
E. Symmetric oligoarthritis

►D

Approximately 70 % of psoriatic arthritis is asymmetric oligoarthritis. The slide depicts arthritis


mutilans, which occurs in about 5% of patients with psoriatic arthritis.

339- Which of the following is most associated with pruritus?

A. Purpura annularis telangiectodes


B. Gougerot–Blum syndrome (pigmented purpuric lichenoid dermatitis)
C. Lichen aureus
D. Schamberg\'s disease
E. Ducas and Kapetanakis pigmented purpura

►E

Ducas and Kapetanakis pigmented purpura presents as eczematous patches with petechiae and
hemosiderin staining. Pruritus is common, and the lesions are often more extensive than the other
pigmented purpuras. Histologically, spongiosis is noted. The other listed pigmented purpuric

603
eruptions tend to be asymptomatic, although lesions of Schamberg\'s disease can rarely be
associated with itch.

340- A patient has rapidly increasing seborrheic keratosis, acanthosis nigricans and tripe palms. The
most common associated malignancy with Sign of Lesser-Trelat is:

A. Gastric carcinoma
B. Lymphoma
C. Lung cancer
D. Ovarian cancer
E. Prostate cancer

►A

The Sign of Lesser-Trelat is defined as the rapidly increasing in size or number of seborrheic
keratosis. The associated malignancy is gastric and colon cancer. It may improve with treatment of
underlying malignancy.

341- Which HLA is associated with psoriatic arthritis in disequilibrium, especially if spondylitis is
present?

A. HLA-B17
B. HLA-B13
C. HLA-Bw57
D. HLA-Cw6
E. HLA-B27

►E

HLA-B17 is associated with an earlier onset and more serious disease of psoriasis. HLA-Cw6 is the
most definitive associated HLA type of psoriasis, with a relative risk 9-15 times normal. HLA-B17
amd HLA-B13 are also associated with psoriasis but not psoriatic arthiritis

604
342- Acute hemorrhagic edema of childhood often presents initially with:

A. Facial edema
B. Laryngospasm
C. Acute abdomen
D. Hematuria
E. Hematochezia

►A

Acute hemorrhagic edema of childhood affects children and infants < 2 years of age. It presents
with painful, edematous petechiae and ecchymoses on the head and distal extremities. Facial edema
may be the initial sign. Triggering factors include infection, drugs, and immunization. It lacks
systemic features and resolves in 1-3 weeks without sequelae.

343- A 61 year old man presents with painful and pruritic unilateral vesicles on his lateral back.
Patient described pain for 1 week prior to development of the rash. Diagnosis was confirmed with a
Tzank smear. Which of the following is a frequently seen complication of this disease?

A. Blindness
B. Infertility
C. Postherpetic neuralgia
D. Renal failure
E. Hepatic insufficiency

►C

Postherpetic neuralgia (PHN) is a common complication seen in herpes zoster. Defined as the
presence of pain after skin lesions have healed. Occurs in 10-15% of patients but incidence may
increase to 50% in patients over the age of 60 who develop herpes zoster. In most cases PHN
resolves within the first 12 months but may persist for years. While generally self -limited,
antiretroviral therapy is recommended for patients over the age of 50, those who are
immunocompromised, or have ophthalmic zoster.

605
344 -The most effective treatment for symptomatic ocular rosacea is:

A. Lubricant eye drops


B. Combination topical antibiotic and corticosteroid eye drops
C. Dilute baby shampoo washes
D. Doxycycline
E. Artificial tears

►D

The most definitive treatment for symptomatic ocular rosacea is an oral tetracycline.

345- A 35 year-old otherwise healthy man presents with moderate-to-severe plaque psoriasis,
improved on cyclosporine. The patient weighs 70 kilograms and is taking a dose of 300 mg/day. His
baseline creatinine was 0.8; on follow-up testing it is 1.1. All other exam and laboratory parameters
are within normal limits. The patient is pleased with his treatment and asks to continue it. Which of
the following is correct?

A. The patient has exceeded the recommeded dosage of cyclosporine


B. You offer a decrease of the cyclosporine dose to 225 mg/day and close followup
C. The change in creatinine is not significant, no change is needed
D. Cyclosporine rarely has renal toxicity in young, healthy individuals; thus you must
work-up other causes of the increased creatinine
E. The cyclosporine should be stopped immediately; the patient must avoid cyclosporine in
the future

►B

Cyclosporine is highly effective in most patients with severe chronic plaque-type psoriasis. Doses
start at 2.5 to 4mg/kg/day and can go as high as 5.5mg/kg/day. Renal impairment may occur and is
often reversible. If the creatinine increases 30% or greater from baseline the dose should be reduced
by 25% and the patient followed closely.

346- Anti-Jo-1 antibodies in patients with Dermatomyositis are associated with:

606
A. Pulmonary fibrosis
B. Cardiac disease
C. Photosensitivity
D. Calcinosis
E. Heliotrope rash

►A

Anti-Jo-1 (histidyl TRNA syntetase) antibodies are seen in a minority of patients with
Dermatomyositis and are associated with pulmonary fibrosis. Anti-Jo-1 Abs are also associated with
Raynaud's and polyarthritis. Treatment of dermatomyositis may include systemic corticosteroids,
antimalarials, methotrexate, azatioprine and photoprotection.

347- Which immunoglobulins are seen in type III cryoglobulinemia?

A. Monoclonal IgG and monoclonal IgM


B. Monoclonal IgG or monoclonal IgM
C. Monoclonal IgM and polyclonal IgG
D. Polyclonal IgM and monoclonal IgG
E. Polyclonal IgM and polyclonal IgG

►E

Type III (mixed) cryoglobulinemia consists of rheumatoid factors that are polyclonal IgM and IgG,
complexed with each other or with protein. Type II (mixed) cryoglobulinemia consists of
monoclonal IgM rheumatoid factor complexed with polyclonal IgG. Type I cryoglobulinemia
consists of a single monoclonal immunoglobulin, usually IgG or IgM, usually due to an underlying
B-cell malignancy (myeloma or lymphoma).

348- Acneiform eruptions have been associated with which of the following vitamins?

A. Vitamin C
B. Vitamin E
C. Vitamin A

607
D. Biotin
E. Vitamin B12

►E

Vitamin B12 can cause acneiform eruptions.

349- A patient has tender erythematous plaques with fever, arthritis, conjunctivitis and oral ulcers.
On histopathology, there are prominent edema, dermal infiltrate rich with neutrophils with
leukocytoclasis. The most common cancer associated is:

A. Acute myelogenous leukemia


B. Chronic lymphocytic leukemia
C. IgM gammopathy
D. IgG gammopathy
E. Chronic myelogenous leukemia

►A

This patient has Sweet's syndrome or Febril neutrophilic dermatosis. The most common cancer
associated is acute myelogenous leukemia. it is also associated with lymphoma and polycythemia
vera. It is also associated with anemia, leukocytosis, neutrophilia, and elevated ESR.

350 -The best diagnosis for this congenital melanopenic lesion without extracutaneous associations
is

A. Hypomelanosis of Ito
B. Segmental vitiligo
C. Ash leaf spots
D. Nevus anemicus
E. Nevus achromicus

►E

608
Nevus achromicus is another name for nevus depigmentosus. It usually presents at birth or appear
during early infancy as normal pigmentation increases. Most individuals will have a solitary lesion
of nevus depigmentosus, but multiple lesions and segmental forms of nevus depigmentosus have
been described. Nevus depigmentosus tends to persist lifelong, but remains unchanged after onset.
The hypopigmented white spots of tuberous sclerosis are most difficult to distinguish from nevus
depigmentosus, but lack of other cutaneous or systemic manifestaions exclude tubrous sclerosis.
Lesions of vitiligo tend to be depigmented (melanocytopenic not melanopenic as in the question),
and show a bright white coloration with Wood's lamp examination. Nevus anemicus is a distinct
vascular birthmark characterized by blanching of cutaneous blood vessels, hence presenting as a
"white" patch of skin that becomes unnoticeable when the surrounding skin is blanched with a glass
slide ("diascopy").

351 -A patient presents with cutaneous lesions suggestive of sarcoidosis. You consider ordering a
serum ACE level to help with the diagnosis. What is the sensitivity and specificity of checking an
ace level in this pt?

A. Sensitivity: 60 Specificity: 80
B. Sensitivity: 90 Specificity: 60
C. Sensitivity: 95 Specificity: 80
D. Sensitivity: 60 Specificity: 60
E. Sensitivity: 90 Specificity: 25

►A

Serum ACE levels are positive in only approximately 60% of patients with sarcoidosis. However
the specificity is only 80%. Therefore, serum ace levels are less helpful in diagnosis (they can be
helpful in following treatment response.) For example, if your pretest clinical suspicion of
sarcoidosis is 25% (i.e. 1 of 4 likely possible etiologies based on differential diagnosis), then of a
1000 patients tested, 250 will have sarcoidosis. 160 of these will be detected by an abnormal serum
ace level, but 150 of the non-sarcoidosis patients will also have an abnormal ace level giving you a
positive predictive value of just over 50% making it a poor diagnositic test.

352- Which of the following hormones bind the androgen receptor?

A. Dehydroepiandrosterone

609
B. Androstenedione
C. Dihydrotestosterone
D. Dehydroepiandrosterone and dihydrotestosterone
E. Dehydroepiandrosterone, androstenedione, and dihydrotestosterone

►C

Only testosterone and dihydrotestosterone bind the androgen receptor, thus adrenal androgens
(androstenedione and dehydroepiandrosterone) virilize only in so far as they serve as precursors for
testosterone and dihydrotestosterone.

353- The changes that occur on the nose in patients with rosacea is due to:

A. Sebaceous hyperplasia
B. Sarcoid granulomas
C. Fibrous hyperplasia
D. Acanthosis
E. Tuberculoid granulomas

►A

In patients with rosacea, patients experience rhinophyma that is from the sebaceous hyperplasia.
Initially the nose becomes swollen with prominent pores followed by fibrosis in the later stages.

354 -RNP antibodies include:

A. anti-SSA, anti-centromere, anti-SSB


B. anti-SSA, anti-SSB, anti-Sm, anti-U1RNP
C. anti-SSA, anti-SSB, anti-Sm, anti-dsDNA
D. ANA, anti-dsDNA
E. anti-SSA, anti-U1RNP, anti-centromere

►B

610
RNP antibodies target small ribonucleoproteins. These include SS-A (Ro), SS-B (La), Sm, and
U1RNP. The total amount of antibody has more diagnostic value than the mere presence of
antibody.

355- Eosinophilic pustular folliculitis is a noninfectious eosinophilic infiltration of hair follicles. It


is mainly seen in this ethnic group:

A. Japanese men
B. European men
C. Mediterranean men
D. African American men
E. Asian women

►A

Eosinophilic pustular folliculitis is a noninfectious eosinophilic infiltration of hair follicles. The


classic case is mainly from Japanese men. It is also immunosuppression association.

356- Which of the following is characteristic of lichen planus pigmentosus?

A. Lesions are typically hypopigmented


B. Most cases present in Caucasians
C. Oral involvement is pathognomonic
D. Can occasionally see epidermotropic T-cells
E. Trunk is typically spared

►D

Occasionally epidermotropic T-cells are seen in the lichenoid reaction and thus may raise concern
for mycosis fungoides. Lichen planus pigmentosus inversus, in particular, presents in classic sites of
mycosis fungoides, including the axilla, inguinal, and inframammary areas. The individual lesions
of lichen planus pigmentosis are typically smaller, however, than those encountered in mycosis
fungoides, thus helping with the differential. Lesions of lichen planus pigmentosus are

611
hyperpigmented, with most cases prsenting in skin of color. Oral involvement is rare and the trunk
can be involved.

357- Which of the following is not a common food or environmental allergen implicated in atopic
dermatitis?

A. Dermatophagoides pteronyssimus
B. Wheat
C. Corn
D. Eggs
E. Fish

►C

While most AD patients do not have food allergy, food allergens exacerbate AD in at least a subset
of patients, particularly infants and young children. Eggs, milk, peanuts, soybeans, tree nuts, fish,
and wheat are the most common food allergens implicated. Dust mites (Dermatophagoides
pteronyssimus) are among the environmental allergens that may exacerbate AD.

358- Which of the following is an example of a large vessel vasculitis?

A. Takayasu arteritis
B. Wegener's granulomatosis
C. Chrug-Strauss syndrome
D. Polyarteritis nodosa
E. Henoch-Schonlein purpura

►A

Takayasu arteritis is a large vessel vasculitis that manifests as progressive granulomatous


inflammation of the aorta and its major branches. The systemic vasculitidies are classified into three
categories: large vessel, medium-sized vessel and small vessel vasculitis. Takayasu arteritis and
Giant cell (temporal) arteritis are the two large vessel vasculitidies. Polyart eritis nodosa and
Kawasaki disease are medium-sized vasculitidies. Wegener's granulomatosis, Churg-Strauss

612
syndrome, microscopic polyangiitis, Henoch-Schonlein purpura and cutaneous leukocytoclastic
vasculitis are examples of small vessel vasculitidies.

359- What mast cell mediators are synthesized upon exposure to a trigger, rather than preformed?

A. tryptase
B. histamine
C. serotonin
D. prostaglandin D2
E. heparin

►D

Mast cell mediators can be grouped into two classes: preformed and newly formed. Preformed
mediators include tryptase, histamine, serotonin, and heparin. Newly formed mediators include
prostaglandin D2, leukotriene C4, and platelet activating factor.

360- You are suspicious for a new diagnosis of lupus in a patient recently treated with
penicillamine. Which antibodies would you expect to have been induced by this medication?

A. anti-dsDNA
B. anti-histone
C. anti-SSA
D. anti-SSB
E. anti-Mi-2

►A

Penicillamine induces native systemic lupus erythematosus, associated with anti-dsDNA antibodies,
in contrast to drug-induced lupus erythematosus which has been associated with exposure to
hydralazine, procainamide, sulfonamides, penicillin, anticonvulsants, minocycline, and INH and is
associated with anti-histone antibodies.

613
361- A 40 year old woman presents to the clinic with multiple pustules in annular and serpiginous
patterns on the abdomen, axillae and groin. Histopathology reveals pustules below the stratum
corneum with many neutrophils and without any acantholysis. What is the diagnosis?

A. Sneddon-Wilkinson disease
B. Reiter's syndrome
C. SAPHO syndrome
D. Transient pustular melanosis
E. Pthirus pubis

►A

Sneddon- Wilkinson disease, also know as subcorneal pustular dermatosis classically presents with
what has been described in this question. SAPHO syndrome is a mnemonic which stands for
synovitis, acne conglobata or fulminans, pustular psoriasis, hyperostosis and osteitis. Transient
pustular melanosis is seen in neonates. Pthirus pubis is pubic lice.

362- Which technique used for hemostasis may cause iatrogenic tattooing?

A. Monsel’s
B. Aluminium Cloride
C. Gel Foam
D. Cautery
E. Epinephrine

►A

Monsel’s solution is also known as ferrous subsulfate and can be a cause of iatrogenic tattooing.

363- What is the most common infection that can cause guttate psoriasis?

A. Streptococcus pyogenes
B. Coccidiomycosis
C. Herpes

614
D. Mycoplasma pneumonia
E. Tuberculosis

►A

Guttate psoriasis refers to a distinctive, acute clinical presentation of an eruption characterized by


small, droplike, 1-10 mm in diameter, salmon-pink papules, usually with a fine scale. It is more
commonly seen in individuals younger than 30 years with a history of upper respiratory infection
that precedes the eruption by 2-3 weeks. The most common organism is group A beta-hemolytic
streptococci (eg, Streptococcus pyogenes). Although recurrent episodes may occur, especially those
due to pharyngeal carriage of streptococci, isolated bouts are known to occur. Guttae psoriasis may
also occur with other infections or in isolation without any identifiable infection.

364- The pigmentation shown here is most likely due to which of the following drugs?

A. Chloroquine
B. Amiodarone
C. Minocycline
D. Doxycycline
E. TMP-SMX

615
►C

This slide shows blue-gray discoloration at the anterior shins and is characteristic of minocycline
hyperpigmentation.

365 -A patient presents with hemorrhagic onycholysis. The drug class most commonly associated
with this finding is:

A. Quinolone antibiotics
B. Systemic retinoids
C. Calcineurin inhibitors
D. Taxanes
E. Tetracyclines

►D

Taxane probably cause nail changes more commonly than other drugs. Cutaneous toxicity has been
reported with taxanes and includes erythema and desquamation, involving primarily the hands.
Taxanes exert their cytotoxic effect by reversibly binding the α-subunit of tubulin, thereby inducing
tubulin polymerization and inhibiting microtubule depolymerization. A balance between
polymerization and depolymerization is needed for normal microtubule function.Taxanes disrupt
this balance, leading to arrest at the G2/M phase of the cell cycle.

366 -Dermatomyositis is associated with muscle weakness and arthritis. Drug induced
dermatomyositis is most commonly caused by:

A. Hydroxyurea
B. Statins
C. Cyclosporin
D. Cyclophosphamide
E. Tetracycline

►A

616
Drug induced dermatomyositis is the most common cause is hydroxyurea. Other common causes
are statins, cyclophosphamide, and terbinafine.

367- Which autoantibodies are associated with an increased risk of malignancy in dermatomyositis?

A. anti-SRP
B. anti-Mi2
C. Anti-Jo1
D. anti-155/140
E. anti-PL-7

►D

Anti-155/140 has been associated with an increased risk of malignancy in dermatomyositis patients.
Anti-SRP is associated with fulminant dermatomyositis/polymyositis and cardiac involvement; anti-
Mi2 is associated with the shawl sign, periungual telangiectasias, cuticular overgrowth, and
Gottrons papules; anti-Jo1 and anti-PL-7 are associated with antisynthetase syndrome and
interstitial lung disease.

368- The Dunnigan variant of partial lipodystrophy is caused by a mutation in which gene?

A. AGPAT2
B. Seipin
C. Zinc metalloproteinase
D. Neutrophil elastase
E. LMNA

►E

The Dunnigan variant of partial lipodystrophy is characterized by normal appearance at birth


followed by a gradual loss of subcutaneous tissue from the arms and legs around the time of
puberty. The genetic defect involves the gene encoding lamins A and C (LMNA) which are
intermediate filaments integral to the nuclear envelope. The AGPAT2 and seipin genes are mutated

617
in type I and type II congenital generalized lipodystrophies, respectively. The gene for zinc
metalloproteinase (ZMPSTE24) is mutated in mandibuloacral dysplasia.

369-The drug that is the most common cause for precipitating guttate psoriasis is:

A. Beta blockers
B. Hydrochlorothiazide
C. Tetracyclines
D. Penicillin
E. ACE inhibitors

►A

Patients that are on beta blockers can be at risk for precipitating gutate psoriasis. Streptococcus
can also precipitate gutatte psoriasis.

370 -Carcinoma of which of the following has been most associated with erythema gyratum repens

A. Lung cancer
B. Breast cancer
C. Colon cancer
D. Prostate cancer
E. Upper GI tract cancer

►A

80% of cases of erythema gyratum repens have been associated with underlying malignancy. Lung
cancer is the most common neoplasm. The skin eruption preceedes the detection of malignancy by
an average of 9 months

371 - Macroglossia can present in all of the following disorders except

618
A. Primary systemic amyloidosis
B. Down's syndrome
C. Mucopolysaccharidoses
D. Cretinism
E. Behchets disease

►E

Macroglossia is not a feature of Behchets disease. Macroglossia, a classic feature that occurs in
about 20% of patients with primary systemic amyloidosis due to deposition of amyloid in the
tongue leading to firm and enlarged tongue. These patients can also have hemorrhagic papules,
plaques and blisters on its surface. Macroglossia is seen in many diseases and syndromes including
Down's syndrome, mucopolysaccharoidoses, cretinism, hypothyroidism, lipoid proteinosis and
Beckwith-Wiederman Syndrome.

372 -A patient complained of sudden appearance of multiple papules on chest and upper arms.
Preliminary report by the dermpath fellow are cystic spaces lined by two layers of cuboidal cells
and epithelial strands of similar cells. What is the most likely diagnosis?

A. Syringomas
B. Hydrocystomas
C. Acrospiromas
D. Spiradenomas
E. Mixed tumors

►A

The eruptive type of syringomas. In addition to the description above, some of the cysts have small
comma-like tails, which is known as a tadpole pattern. Eruptive syringomas are histologically
identical to those on the eyelid, but appear suddenly on the neck, chest, axillae, upper arms, and
periumbilically. It usually occurs in young people. The other options do not have an eruptive form
and tend to occur in different locations of the body other than the arms and thorax.

619
373 -A 2 year-old boy with crusted skin papules is found to have osteolytic defects and diabetes
insipidus. These features are seen in:

A. Osteogenesis imperfecta
B. Conradi-Hunnerman disease
C. Metastatic small cell lung carcinoma
D. Hand-Schuller-Christian disease
E. Epidermal nevus syndrome

►D

Hand-Schuller-Christian disease is a chronic multifocal form of Langerhans cell histiocytosis.70%


of cases occur between the ages of 2 and 6. The four characteristic clinical findings are bone
lesions, diabetes insipidus, exophthalmus, mucocutaneous lesions. Bones lesions are osteolytic and
preferentially involve the calvarium.

374- A 20 year old woman presents with a sunken appearance to her face and trunk with normal
legs. The patient has attributed her appearance to stress from college, but is seeing you because of
her family's concern. What lab finding would you expect in this patient

A. Decreased C3 nephritic factor


B. Decreased C3
C. Decreased C1
D. Increased C1
E. Increased C4

►B

Acquired partial lipodystrophy (Barraquer-Simmons Syndrome) is characterized by an insidious


progressive loss of fat that usually begins in the face and scalp and progressed downward. Most
patients with this form of lipodystrophy have reduced levels of C3 resulting from circulation
polyclonal IgG called "C3 nephritic factor" which results in uncontrolled activation of C3 and
contributes to renal damage.

620
375- Eosinophilia, asthma, neuropathy and sinus abnormalities are associated with which of the
following systemic vasculitidies?

A. Churg-Strauss syndrome
B. Polyarteritis nodosa
C. Microscopic polyangiitis
D. Wegener's granulomatosis
E. Kawasaki disease

►A

Churg-Strauss syndrome has six diagnostic criteria as set by the American College of
Rheumatology: Blood Eosinophilia, Asthma, Neuropathy, Sinus abnormalities, Allergies and
Perivascular eosinophils. The presence of four of these six criteria yields a diagnostic sensitivity of
85% and specificity of 99.7%.

376- Cigarette smoking has been shown to:

A. Decrease the incidence of BCC


B. Not affect the survival of surgical repairs
C. Demonstrate less solar elastosis than photoaged skin
D. Have no correlation with facial wrinkling
E. Increase cutaneous vasoconstriction

►E

Cigarette smoking accelerated photoaging. Solar elastosis in patients with significant smoking
history is present not only in the papillary dermis, but also the reticular dermis. It has also been
shown to increase the incidence of skin cancer, decrease the survival of surgical grafts, increase the
severity of wrinkling.

377 -A common site for chloracne is the:

A. Occipital scalp

621
B. Forehead
C. Scrotum
D. Forearms
E. Shoulders

►C

Common sites for chloracne include the malar cheek, the post-auricular scalp and in men, the
scrotum.

378- Which of the following HLA types is associated with psoriasis and predicts earlier onset and
more severe disease?

A. HLA-B13
B. HLA-B17
C. HLA-Bw57
D. HLA-Cw6
E. HLA-B27

►B

Psoriasis has HLA associations with all of these HLA types. HLA-B17 is associated with earlier
onset and more serious disease.

379- Which of the following medications is most likely to induce a flare of this patient's disease?

622
A. ciprofloxacin
B. captopril
C. diphenhydramine
D. sertraline
E. acetominophen

►B

Medications that have been reported to cause flares of psoriasis include antimalarials, beta blockers,
NSAIDS, penicillin, tetracycline, ACE inhibitors, G-CSF, interferons and lithium. In addition,
withdrawal from systemic corticosteroids can induce a flare.

380- The most common internal manifestation of scleroderma is:

A. Esophageal dysmotility
B. Sepsis
C. Pulmonary hypertension
D. Acute renal failure
E. Raynaud's phenomenon

►A

Extracutaneous manifestations of scleroderma include esophageal dysmotility, pulmonary fibrosis


with resultant pulmonary hypertension, cardiac involvement with conduction defects, pericarditis or
heart failure, and renal involvement with acute renal failure. Esophageal dysmotility is the most
common systemic manifestation with up to 90% of systemic sclerosis patients affected. Dysphagia
can precede cutaneous involvement thereby giving rise to the term systemic sclerosis sine
scleroderma. Dysmotility is manifested by reduced peristalsis, especially in the lower two-thirds of
the esophagus. This can be demonstrated on a radiologic or manometric study.

381- The first site in body that shows yellowish pigmentation in carotenoderma is

A. Face
B. Trunk

623
C. Palms and soles
D. Nails
E. Sclera

►A

Carotenoderma is yellowish discoloration of the skin secondary to carotenemia. Carotene is


excreted by sebaceous glands and in sweat, so the yellow pigmentation appears first on the face
(especially nasolabial folds and forehead) and then becomes diffusely distributed with accentuation
in palms and soles. In contrast to jaundice, carotenoderma spares mucous membranes and sclera

382- All of the following drugs have been reported to cause rash similar to the attached image
except

A. Hydroxyurea
B. D-penicillamine
C. Statins
D. Phenytoin
E. Captopril

►E

The image shows symmetrical erthematous to violaceous plaque on dorsal hands with some
periungual erythema and telangiectasia. Skin biopsy was consistent with interface dermatitis and
mucin. Many drugs have been reported to cause dermatomyositis-like picture, which include
hydroxyurea, D-penicillamine, statins, phenytoin and alfuzosin (alpha antagonist for BPH).
Captopril is not associated with DM-like rash.

624
383 -Which auto-antibody correlates with this finding, shawl sign and an overall favorable
prognosis in Dermatomyositis?

A. Anti-Mi-2 antibody
B. Anti-Jo-1 antibody
C. Anti-SRB antibody
D. Anti-KU antibody
E. Anti-PL7 antibody

►A

Anti-Mi-2 antibodies correlate with shawl sign, cuticular changes (as seen in the image) and an
overall favorable prognosis. Anti-Jo-1 antibodies correlate with pulmonary fibrosis, Raynaud's and
polyarthritis. Anti-SRB antibodies correlate with cardiac disease and a poor prognosis. AntiKU
antibodies correlate with sclerodermatomyositis. Anti-PL7 and Anti-PL12 antibodies
(antisynthetase antibodies) also correlate with pulmonary disease.

384 - Patients of which HLA type are more prone to drug-induced lupus erythematosus when
exposed to hydralazine?

A. HLA-Cw6
B. HLA-B7
C. HLA-DR3
D. HLA-DR4
E. HLA-B6

►D

Drug-induced lupus erythematous has been associated with exposure to hydralazine, procainamide,
sulfonamides, penicillin, anticonvulsants, minocycline, and INH. It generally has a benign course.
The presence of anti-histone antibodies are closely associated with symptomatic disease.
Hydralazine and procainamide are common culprits, with slow acetylators of hydralazine (HLA-
DR4) more prone.

625
385- Mastocytosis is a systemic disease that causes wheal upon rubbing a lesion called a positive
Darier's sign. Mastocytosis can be measured by examination of serum:

A. Tryptase
B. Kinase
C. Bilirubin
D. Histamine
E. IgE

►A

The most common form of cutaneous mastocytosis is urticaria pigmentosa. It is diagnosed by


measuring serum tryptase levels. The new diagnostic method is measurement of tryptase in
bonemarrow which is a new sensitive marker of the mast cell.

386- A patient has palpable purpura, Raynaud and livedo reticularis has been diagnosed with
cryoglobulinemia type I. The immunoglobulins that is associated with cryoglobulinemia type I is:

A. Monoclonal IgM
B. Polyclonal IgM
C. Monoclonal IgG
D. Polyclonal IgG
E. Monoclonal IgA

►A

Type I cryoglobulinemia is associated with monoclonal IgM. Other associated malignancy are
multiple myeloma and Waldenstroms macroglobulinemia. The lab findings are elevated
cryoglobulins and on pathology there are eosinophilic intravascular deposits.

387- Which of the following is a function of Propionibacterium acnes (P. acnes) in contributin g to
the pathogenesis of acne?

A. Downregulation of TLR-2 expression

626
B. Inhibition of complement
C. Activation of TLR-2
D. Inhibition of Il-1
E. Reduction in free fatty acid levels

►C

P. acnes flourishes in the presence of sebum and makes an enzymatic lipase which cleaves
triglycerides into free fatty acids. Free fatty acids serve as an indirect measure of P. acnes
populations on the skin. P. acnes itself is also pro-inflammatory; it can activate complement, as well
as neutrophil chemotaxis and activity. Il-1, a pro-inflammatory cytokine, may promote follicular
plugging and microcomedo formation. TLR-2 activation by P. acnes leads to stimulation of pro-
inflammatory cytokines.

388- The treatment of choice for imaging a ateriovenous malformation is:

A. Arteriography
B. Doppler
C. Annual Xray
D. Multidetetor Xray
E. 3D MRI venography

►A

Arteriography is the technique of choice to evaluate arteriovenous malformations. Doppler should


be done at age 1 and annual xray after the age of 2. Multi-detector CT and fast 3D MRI venography
can also be done.

389- Which of the following statements comparing inflammatory lesions from patients with
psoriasis and patients with atopic dermatitis is true?

A. Human beta-defensin 2 (HBD-2) and LL 37 levels are significantly lower in psoriasis


lesions compared to atopic dermatitis lesions
B. Expression of HBD-2 and LL 37 is negligible in psoriatic lesions

627
C. HBD-2 and LL 37 are significantly elevated in atopic dermatitis lesions
D. Expression of HBD-2 and LL 37 in psoriasis is similar to normal skin, whereas it is
decreased in atopic dermatitis lesions
E. HBD-2 and LL 37 are significantly lower in atopic dermatitis lesions compared to
psoriasis lesions

►E

Ong et al found a deficiency of HBD-2 and LL 37, which are innate antimicrobial peptides, in
lesions from patients with atopic dermatitis compared to lesions from patients with psoriasis. IL4
and IL-13, which are Th2-type cytokines, were found to inhibit expression of HBD-2 and LL 37.
Decreased expression of HBD-2 and LL 37 may account for the increased susceptibility to S. aureus
infection in patients with atopic dermatitis.

390 -A patient presents to you with purpura after minor trauma. In addition, macroglossia with teeth
indentations. Which amyloid protein is associated with her condition?

A. AB2M
B. AA
C. AL
D. Altered keratinAB
E. AB

►C

AL protein is associated with primary systemic amyloid. AA is associated with secondary systemic
amyloid. AB2M is associated with hemodialysis. AB is associated with alzheimer‘s disease. Altered
keratin is associated with lichen and macular amyloid.

391 -You prescribe oral erythromycin to a 35 year-old woman. Co-administration of which of the
following medications could lead to potential adverse outcomes?

A. Oral contraceptives
B. Warfarin

628
C. Carbamazepine
D. Methylprednisolone
E. Warfarin, carbamazepine, or methylprednisolone

►E

Erythromycin inhibits the hepatic cytochrome P450 system and can increase serum levels and
potential toxicities of carbamazepine, theophylline, warfarin, digoxin, and methylprednisolone.

392- Which special stain is utilized to confirm the diagnosis of Langerhans cell histiocytosis?

A. H&E sufficient for diagnosis


B. C-kit
C. CD20
D. CD1a
E. Chloroacetate esterase

►D

Langerhan cell histiocytosis includes the diseases Letterer-Siwe, Hand-Schuller-Christian,


Eosinophilic Granuloma, and Hashimoto-Pritzker. Cells stain positively with CD1a, S100, langerin,
and peanut agglutinin. Cells may also be indentified by the presence of Birbeck granules.

393- Which of the following statements is true regarding Morbihan's Disease?

A. It is often misdiagnosed as cellulitis


B. It presents with blepharitis, conjunctivitis, iritis, and keratitis
C. Histopathology reveals perifollicular and perivascular noncaseating epithelioid
granulomas
D. It occurs around the mouth and/or nose and eyes and may be triggered by topical steroid
use
E. It presents with large coalescent nodules and confluent draining sinuses occupying most
of the face

629
►A

Morbihan's Disease, also known as rosacea lymphedema or persistent edema of rosacea, presents
with hard, nonpitting edema. It is often misdiagnosed as cellulitis.

394- Which of the following medications is most likely to induce or exacerbate psoriasis in your
patient.

A. Hydrochlorothiazide
B. Lisinopril
C. Amlodipine
D. Metoprolol
E. Diltiazem

►D

Several drugs have been incriminated as inducers of psoriasis, in particular lithium, B-Blockers,
antimalarials, and interferon. More recent additions include terbinafine, calcium channel blockers
(nicardipine, nifedipine, nisoldipine, verapamil, and diltiazem), captopril, glyburide, and lipi-
lowering drugs such as gemfibrozil.

395- Topical calcineurin inhibitors may offer several benefits over topical steroids in treating the
scaly, lichenified periorbital plaques in this patient with atopic dermatitis, including:

A. More cost effective


B. Quicker onset of action
C. Decreased irritant potential
D. Absence of reports of cutaneous atrophy
E. Once daily application

►D

630
Tacrolimus and pimecrolimus are non-steroid anti-inflammatory drugs and are not associated with
side effects reported with use of topical corticosteroids, including atrophy, striae formation,
telangiectasias, cataracts, and HPA axis suppression.

396- A patient has multiple skin colored papule that when lanced, express clear fluid. On histology,
the cyst is lined by eosinophilic cuticle with a thin squamous epithelium and mature sebaceous
glands attached to the wall. The most likely defect is:

A. Keratin 17
B. Keratin 1
C. Keratin 5
D. Keratin 7
E. Keratin 20

►A

The patient has steatocystoma multiplex that is linked to the defect in keratin 17, 10. The lesions
usually occur in the second and third decades of life. It also occurs sporadically and is inherited in
the benign form.

397- A pregnant woman presents with the following condition which is bothersome to her. Which of
the following is the most appropriate treatment option based on FDA classifications of medication
in pregnancy?

A. Topical erythromycin/benzoyl peroxide gel


B. Topical tretinoin
C. Tazarotene 0.1% cream
D. Azelaic acid 20% cream
E. Bactrim DS

►D

This patient has mild-moderate inflammatory acne. Azelaic acid is the only medication listed which
falls under category B. The others listed are category C, except for tazarotene, which is category X

631
398- Most often, xanthelasma is associated with:

A. An IgM paraprotein
B. An IgG paraprotein
C. No associated disease
D. Type I hyperlipoproteinemia
E. Type III hyperlipoproteinemia

►C

Xanthelasma are the most common type of xanthoma. They are usually present without any other
disease, but may occur concomitantly with other xanthomas, and can occur in types II and III
hyperlipoproteinemias (familial hypercholesterolemia, common hypercholesterolemia, and familial
dysbetalipoproteinemia). They are also common among women with biliary or hepatic disoders, and
are also seen in myxedema, diabetes, and phytosterolemia.

399- A 55 year-old male presents with a indurated yellow-brown plaque near the eye. Work-up
reveals:

A. IgG monoclonal gammopathy


B. IgG polyclonal gammopathy
C. IgA monoclonal gammopathy
D. IgA polyclonal gammopathy
E. no paraproteinemia seen in 50% of cases

►A

IgG monoclonal gammopathy is seen in over 80% of cases of necrobiotic xanthogranuloma.


Pathology reveals palisading granulomas with necrobiosis and cholesterol clefts. Recurrence rates
as high as 42% have been reported after surgical excision.

400 -Which of the following cytokines is most characteristic of atopic dermatitis in its chronic
phase?

632
A. Interferon-gamma
B. IL-4
C. IL-5
D. IL-10
E. IL-13

►A

While acute atopic dermatitis is a TH2 state, chronic eczematous AD is most often a TH1 cytokine
environment with interferon-gamma being the best choice. Initial lesions TH2 dominant, cytokines
IL-4,5,10,13. IL-4/5 produce elevated IgE and eosinophilia, IL-10 inhibits delayed-type
hypersensitivity. IL-4 downregulates IFN-gamma.

401- Lacrimal gland involvement is characteristically seen in patients with:

A. Lymphoma
B. Sarcoid
C. Rosacea
D. Syphilis
E. Pemphigus

►B

Sarcoid is granulomatous autoimmune condition characterized by the formation of non-caseating


granulomas which may affect any organ system. Lacrimal gland involvement occurs in 15 -28% of
patients. Manifestation of lacrimal gland involvement includes bilateral painless swelling.

402- This 35 year-old man presents with the lesions shown. He was treated elsewhere for a different
skin condition. Biopsy of these lesions is likely to show:

A. Increased staining on Fontana Masson but not Perls stain


B. Increased staining on Perls stain but not Fontana Masson
C. Increased staining on both Fontana Masson and Perls stain
D. Increased melanin at the basal layer and within macrophages only

633
E. Fibrosis and increased mucin deposition

►C

The patient has Type 2 minocycline-associated hyperpigmentation. Three types of


minocyclineassociated hyperpigmentation are generally described. The first is blue-black
discoloration appearing in areas of prior skin injury, such as acne scars. The second type is a blue-
gray discoloration, often on the lower anterior legs and forearms. The third type is the least
common, and is characterized by muddy brown discoloration of sun-exposed areas. The first two
types show staining for both iron and melanin (Fontana Masson stains melanin black; Perls stains
iron [hemosiderin] blue). The third type shows increased melanin at the basal layer and within
macrophages.

403- The National Psoriasis Foundation Consensus recommends that patients with latent TB should
be treated with TB within:

A. 1-2 months of starting anti-TNF therapy


B. 3-4 months of starting anti-TNF therapy
C. 5-6 months of starting anti-TNF therapy
D. 6-8 months of starting anti-TNF therapy
E. 8-12 months of starting anti-TNF therapy

►A

The National Psoriasis Foundation Consensus states that patients with a positive TB. Treat latent TB
prior to starting therapy. After 1-2 months of appropriate TB treatment, psoriasis therapy may begin
is necessary.

404 -This woman is being treated for corticosteroid-induced rosacea with topical metronidazole and
an oral tetracycline. She is very concerned about the redness of her face. Judicious use of what color
concealer can reduce the appearance of redness on the skin?

A. Yellow
B. Lavender

634
C. Pink
D. Bronze
E. Green

►E

This slide shows a woman with rosacea. Green and red are on opposite sides of the color wheel and
thus can ―cancelǁ each other out. Green concealer can neutralize redness on the skin.

405- Menopausal flushing can be effectively treated with:

A. Nadolol
B. Tryptophan
C. Clonidine
D. Danazol
E. Tamoxifen

►C

Menopausal flushing occurs at menopause or perimenstrual when estrogen levels are low.
Pharmacologic menopause may be caused by drugs, including danazol, tamoxifen, clomiphene
citrate, decapeptyl, leuprolide, and 4-hydroxyandrostenedione. Treatment is with oral estrogen
replacement or clonidine hydrochloride 0.05mg bid. Nadolol is an effective treatment of emotional
flushing.

635
406- Mycosis fungoides is characterized with atypical lymphocytes in the epidermis. The marker
for more advanced mycosis fungoides is:

A. CA27.29
B. CA30.32
C. CA40.42
D. CA50.52
E. CA55.57

►A

Mycosis fungoides is the most common form of cutaneous T cell lymphoma. It can look like
chronic eczema and can be histologically very difficult to distinguish from mycosis fungoides.
CA27.29 may be a marker for advanced mycosis fungoides.

407 -A child has a cutaneous ossification disorder characterized by endochondral bone formation,
deafness, baldness, and mental retardation. Which form of osteoma cutis does this patient have?

A. Fibrodysplasia ossificans progressiva


B. Progressive osseous heteroplasia
C. Plate-like osteoma cutis
D. Albright's hereditary osteodystrophy
E. Calcinosis cutis

►A

This patient has fibrodysplasia ossificans progressiva characterized by endochondral bone


formation (the other types have intramembranous bone formation), noggin gene defects and other
systemic symptoms as listed. This is a progressive and potentially fatal condition. Progressive
osseous heteroplasia is also progressive, seen in mostly females, and demonstrates increased
alkaline phosphatase, LDH and CPK with normal calcium, phosphate, and PTH. Platelike osteoma
cutis is limited and seen in children and newborns. Albright's hereditary osteodystrophy is due to a
mutation in GNAS-1. This disorder is characterized by a lack of responsiveness to parathyroid
hormone, resulting in low serum calcium, high serum phosphate, and appropriately high serum
parathyroid hormone. Individuals with Albright's hereditary osteodystrophy have short stature,

636
characteristically shortened fourth and fifth metacarpals, rounded facies, and often mild mental
retardation. Calcinosis cutis is not a form of osteoma cutis.

408- Which of the following drugs is bactericidal?

A. Clindamycin
B. Tetracycline
C. Trimethoprim
D. Penicillin
E. Chloramphenicol

►D

Other bactericidal drugs include bacitracin, monobactams, quinolones, vancomycin, and polymyxin
B. Clindamycin, tetracycline, trimethoprim, and chloramphenicol and bacteriostatic.

409 -A healthy 50 year old man presents to your office with 3 red-brown indurated plaques on his
forehead. Biopsy reveals a focal leukocytoclastic vasculitis, with a diffuse infiltration of
neutrophils, eosinophils, and lymphocytes. Which of the following is a potential treatment option
for this patient?

A. Q-switched alexandrite laser


B. Pulse dye laser
C. Q-switched ruby laser
D. Nd:YAG laser
E. Frequency-doubled Nd:YAG laser

►B

The question stem describes a case of granuloma faciale. This condition presents with red-brown
papules and plaques on the face, most commonly in middle-aged Caucasian males. While
traditional treatment options include intralesional steroids with or without cryotherapy, pulse dye
laser (595 nm) represents another treatment option for this condition, with successful clearance in a
number of case reports.

637
410- Patients with psoriasis treated with cyclosporine should be monitored for:

A. Sicca symptoms
B. Hypermagnesemia
C. Alopecia
D. Acute interstitial pneumonitis
E. Hyperkalemia

►E

Well-documented adverse effects and toxicities of cyclosporine include renal impairment ,


hypertension, elevated triglycerides, hyperkalemia, hypomagnesemia, hepatotoxicity,
hypertrichosis, and long-term increased risk of malignancy.

411- The constellation of erythema nodosum, bilateral hilar lymphadenopathy, uveitis, fever, and
arthritis in patients with sarcoidosis is known as:

A. Loeffler's syndrome
B. Schnitzler's syndrome
C. Lofgren's syndrome
D. Heerfordt's syndrome
E. Mikulicz's syndrome

►C

The presence of EN, bilateral hilar lymphadenopathy, uveitis, fever, and arthritis in patients with
sarcoidosis is known as Lofgren's syndrome. Heerfordt's syndrome, also known as uveoparotid
fever, frequently occurs in patients with sarcoidosis of the central nervous syndrome. It consists of
uveitis, facial nerve palsy, fever, and parotid gland involvement. Milulicz's syndrome is bilateral
sarcoidosis of the parotid, submandibular, sublingual, and lacrimal glands. Loeffler's syndrome is
characterized by a patchy infiltrate in the lungs and eosinophilia of the blood and sputum. Loeffler's
syndrome can be with creeping eruption (larva migrans). Schnitzler's syndrome is a rare disorder of
chronic urticaria, fever, disabling bone pain, hyperostosis, increased erythrocyte sedimentation rate,
and monoclonal IgM gammopathy. Pruritus is not a feature of Schnitzler syndrome.

638
412- A patient has gout found on the helices of the ear diagnosed by the dermatopathologist.
Fixation in this allows visualization of urate crystals:

A. Carnoy's fluid
B. Formaldehyde
C. Michele's media
D. Normal saline
E. Water

►A

Absolute ethanol or Carnoy's fluid allows visualization of urate crystals. The crystals are brown and
refractile with polarized light.

413- What characteristic body would be found on histopathology of this disease?

A. Kamino
B. Civatte
C. Henderson-Patterson
D. Donovan
E. Dutcher

►B

Hypertrophic lichen planus is a type of lichen planus characterized by hyperkeratosis. They are
usually pruritic and often found on the extremities. Civatte bodies are degenerated keratinocytes
often seen in lichen planus.

414- First line treatment of chromoblastomycosis includes surgery and a medication with which
mechanism of action?

A. Inhibits squalene epoxidase


B. Inhibits 14-alpha-demethylase
C. Disrupts microtubule mitotic spindle formation

639
D. Inhibits sythesis of beta-1,3-diglucan and disrupts cell walls
E. Inhibits fungal cytochrome P-450 mediated 14 alpha-lanosterol demethylation

►B

The correct answer is surgery and itraconazole, which inhibits 14-alpha-demethylase.


Terbinafine inhibits squalene epoxidase. Griseofulvin disrupts microtubule mitotic spindle
formation. Caspofungin inhibits synthesis of beta-1,3-diglucan. Foscarnet inhibits fungal
cytochrome P-450 mediated 14 alpha-lanosterol demethylation.

415 -A pregnant woman in her third trimester presents with nonpruritic erythematous plaques and
pustules in the intertriginous regions, the trunk, and the extremities as well as systemic symptoms of
malaise and fever. Which complication is most associated with this condition?

A. Premature labor
B. Placental insufficiency
C. No risk to fetus or mother
D. Small for gestational age
E. Microcephaly

►B

This patient has pustular psoriasis of pregnancy which is also called "impetigo herpetiformis." It
is characterized by red plaques with a peripheral ring of pustules that are distributed symmetrically
in flexural areas and trunk and extremities. Patients can have elevated ESR and leukocytosis as well
as hypocalcemia. There is a risk of fetal morbidity and mortality secondary to placental
insufficiency and maternal mortality secondary to cardiac or renal failure. Premature labor may be
associated with cholestasis or pregnancy. Patients with herpes gestationis may have a risk of
premature labor and small for gestational age. Microcephaly may occur with isotretinoin taken
during pregnancy.

416 -Patients that are on hemodialysis can develop amyloidosis due to altered uremia. Dialysis
related amyloidosis has this kind of fibril protein:

640
A. Beta 2-microglobulin
B. AL
C. AA
D. Altered keratin
E. B amyloid protein

►A

Dialysis related amyloidosis is due to beta 2-microglobulin. The amyloid fibril is altered by
uremia and causes carpal tunnel, bone cyst, and spondyloarthropathy.

417 -A 52 year-old man presents with large comedones as well as inflammatory papules, pustules,
and cysts on the malar cheeks, postauricular scalp, and scrotum. Which of the following could be a
cause of this presentation?

A. Erlotinib chemotherapy
B. A pituitary adenoma
C. 2,3,7,8 tetrachlorobenzodioxin exposure
D. PTEN mutations
E. Isotretinoin overdose

►C

This patient has a presentation consistent with industrial acne. Dioxin (2,3,7,8
tetrachlorobenzodioxin) is a well-known, potent trigger of this acneiform eruption. Overall,
insoluble cutting oils are the most frequent cause of industrial acne.

418- What is the major constituent of sebaceous gland lipid?

A. Triglyceride
B. Wax esters
C. Squalene
D. Free cholesterol
E. Cholesterol esters

641
►A

Triglyceride is the major constituent of sebaceous lipid, accounting for over 50% of the lipid. Wax
esters constitute about 25% of sebaceous lipid and Squalene accounts for about 15%. The remainder
is free cholesterol and cholesterol esters.

419- Patients that have vascular malformations can experience localized intravascular coagulation.
One of the risk factors are:

A. Muscle involvement
B. Gorham-Stout syndrome
C. Smaller surface area
D. Skeletal hypertrophy
E. Grouped superficial vesicles and hyperkeratotitc plaques

►A

Patients that have vascular malformations can experience localized intravascular coagulation. Less
then 85% will experience this with segmental pattern. The risk factors are large surface areas,
muscle involvement, palpable pheboliths. The treatment is low molecular weight heparin.

420- Which cellular organelle is affected in the partial lipodystrophies?

A. Nucleolus
B. Rough endoplasmic reticulum
C. Golgi apparatus
D. Cell membrane
E. Nuclear lamina

►E

Both the congenital (Dunnigan - Type 1) and acquired (Barraquer-Simons) lipodystrophies are
caused by mutations involving the nuclear lamina of the cell. Dunnigan lipodystrophy is
characterized by a mutation in LMNA and Barraquer-Simons is caused by a LMNB2 mutation.

642
421 -Which of the following is a manifestation of psoriasis of the nail matrix?

A. Splinter hemorrhages
B. Oil spots
C. Subungal hyperkeratosis
D. Onycholysis
E. Pits

►E

Psoriatic nail changes may be of nail matrix or bed in origen. Pits are the most common findings,
while splinter hemorrhages are the least. Psoriatic nail changes involving the matrix include pits
(representing focal psoriasis of the proximal nail matrix) and leukonychia.

422- Acute hemorrhagic edema of childhood is distinguished from Henoch-Schonlein Purpura


based on:

A. The presence of pupura on the upper trunk


B. The lack of an antecedent infection
C. The involvement of the synovia
D. The neurologic complications
E. The lack of systemic features

►E

Acute hemorrhagic edema of childhood affects children and infants < 2 years of age. It presents
with painful, edematous petechiae and ecchymoses on the head and distal extremities. Facial edema
may be the initial sign. Triggering factors include infection, drugs, and immunization. It lacks the
systemic features of HSP, and resolves in 1-3 weeks without sequelae. HSP occurs mostly in
children. There is an antecedent URI in 75% of cases. HSP involves the skin, synovia, GI tract, and
kidneys. Long-term morbidity results from renal disease, which is predicted by the spread of
purpura to the upper trunk.

643
423- A patient begins to lose the fat in her face and upper torso acutely after a viral illness. Which
internal organ may be affected by this disease?

A. Lungs
B. CNS
C. Bone marrow
D. Kidney
E. Heart

►D

Barraquer-Simons syndrome, acquired partial lipodystrophy, presents typically in females and is


secondary to a mutation in LMNB2. It is often preceded by a viral illness. More than one third of
patients will have glomerulonephritis, which may lead to chronic renal sequelae.

424- What is the recommended allowance of daily vitamin D intake for a 35-year-old female?

A. 200 IU
B. 400 IU
C. 600 IU
D. 800 IU
E. 1000 IU

►C

Recommended dietary allowance of vitamin D as established by the Institute of Medicine in 2010 is


400 international units for infants 0 to 12 months, 600 IU for ages 1 to 70 years, and 800 IU for
ages 71 and above. Additionally, recommended allowance for pregnant/lactating women is 600 IU.

425- Acute intermittent porphyria has an enzyme defect:

A. Porphobilinogen deaminase
B. Uroporphyrinogen III synthase
C. Coproporphyrinogen oxidase

644
D. Uroporphyrinogen decarboxylase
E. Protoporphyrinogen oxidase

►A

Acute intermittent porphyria has a defect in porphobilinogen deaminase. Acute intermittent


porphyria (AIP) is a rare autosomal dominant metabolic disorder affecting the production of heme,
the oxygen-binding prosthetic group of hemoglobin.

426- Which of the following viruses is associated the most with lichen planus?

A. Hepatitis C virus
B. Hepatitis B virus
C. Human immunodeficiency virus
D. Hepatitis A virus
E. Epstein-barr virus

►A

Lichen planus classically involves the wrists, ankles, and oral mucosa. It is characterized by
violaceous polygonal, purple, pruritic papules. It has a strong association with hepatitis C virus.
Many other associations exist, including other viruses and medications, but hepatitis C is the best
choice.

427- A 22 year-old woman notes irregular menses and acne that has been refractory to several
overthe-counter and prescription regimens. Appropriate initial work-up includes

A. Serum prolactin levels


B. 24-hour urine cortisol
C. Serum free and total testosterone, LH, FSH, androstenedione
D. Serum free and total testosterone, LH, FSH, DHEA-S
E. Serum free and total testosterone, LH, FSH, 17-OH, hydroxylase

645
►D

The history and clinical appearance of this patient suggest a possible hormonal basis to her acne.
The work-up in answer d assesses for adrenal hormonal production and screens for PCOS.
Androstenedione is of ovarian origin.

428- What is the treatment of choice for this patient who developed acne fulminans one month after
starting isotretinoin therapy?

A. Add oral prednisone


B. Increase isotretinoin dose
C. Add doxycycline
D. Acne surgery
E. Check lipid levels

►A

Acne fulminans is an explosive form of acne in which patients may develop systemic symptoms,
ulcerated nodules on the face and trunk. Acne fulminans may be triggered by initiation of
isotretinoin therapy. The treatment of choice is systemic steroids at a dosage of 1 mg/kg/day tapered
over the course of 6 weeks.

429- A patient with a recent diagnosis of tuberculosis on treatment presents with a new
photosensitive eruption on his face, neck and upper chest. In addition, he has progressive diarrhea
and depression. What is the etiology of his symptoms?

A. Vitamin B12 deficiency

646
B. Drug induced pellagra
C. Drug induced riboflavin deficiency
D. Carcinoid syndrome
E. Photoallergic contact dermatitis

►B

This patient has pellagra secondary to isoniazid treatment. He has the photosensitive eruption and
Casal's necklace •, in addition to diarrhea and depression. Other symptoms include the 3 D 's:
dermatitis, diarrhea, dementia. Other potential medications that may cause this constellations of
symptoms include azathioprine and 5-FU.

430- Pyoderma gangrenosum is most commonly reported with which malignancy?

A. Leukemia
B. Medullary thyroid carcinoma
C. Lung carcinoma
D. Breast carcinoma
E. Prostate carcinoma

►A

Pyoderma gangrenosum, (PG), is an uncommon, ulcerative inflammatory skin condition


characterized by boggy ulcerations with undermined borders preceded by the breakdown of a
painful nodule or pustule. These lesions characteristically enlarge progressively over time and
display marked tenderness. The diagnosis of PG is a diagnosis of exclusion and infectious etiologies
should be excluded. Pyoderma gangrenosum can arise in the absence of an underlying disorder or it
can be seen in association with underlying systemic conditions. These conditions include
inflammatory bowel disease such as ulcerative colitis and Crohn's disease, polyarthritis, chronic
active hepatitis and Behcet's disease. Pyoderma gangrenosum has also been associated with an
underlying paraproteinemia, mostly of the IgA type, although IgM and IgG types have also been
described. In addition, some patients have myeloma at presentation or develop it subsequent to their
diagnosis of PG. Pyoderma gangrenosum has also been described in association with
myelodysplasia, and in acute myeloblastic, myelomonocytic, and chronic myeloid leukemia.
Treatment of pyoderma gangrenosum should include therapy directed at the underlying systemic
disorder.

647
431 -A patient has a rash from one of her cosmetic products. The most common cosmetic
preservative to cause contact dermatitis is:

A. Quaternium 15
B. Thimerosal
C. Balsam of Peru
D. Diethyl methylene
E. Benzene

►A

The most common allergen in cosmetic products is quaternium 15. It is a preservative in personal
care products and cosmetics.

432- Treatment of Raynaud's phenomenon includes:

A. Nifedipine
B. Metoprolol
C. UVB
D. Tetracyline
E. Simvastatin

►A

Treatment of Raynaud's includes calcium channel blockers such as nifedipine 30-6-mg/d,


antiplatelet aggregation drugs (such as aspirin or dipyridamole), pentoxyphilline 400mg BID TID
and D-penicillamine. Losartan 50mg/d may reduce frequency and severity of Raynaud's.

433- A patient with renal amyloidosis, urticaria, fevers, limb pains, and deafness with familial
Mediterranean fever has:

A. Muckle-Wells
B. MEN IIa
C. MEN IIIa

648
D. FAP I
E. FAP III

►A

Patients with Muckle-Wells has amyloid AA fibrils. It is an autosomal dominant condition


characterized by deafness, hives, and mutation in the CIAS1 gene.

434 -A patient has non-tender reddish brown nodular lesions located on the dorsal hands and nail
folds. Patients also have symmetric arthritis of interphalangeal and temporomandibular joints and is
diagnosed with multicentric reticulohistiocytosis. The % of patient that develop malignancy is:

A. 10%
B. 20%
C. 30%
D. 40%
E. 50%

►B

20% of patients that are diagnosed with multicentric reticulohistiocytosis have a risk of developing
a malignancy but there is no predominant type. Pathology shows nodular infiltrate composed of
multinucleated oncocytic giant cells with eosinophilic cytoplasm.

435- A patient with eruptive vellus hairs seen in steatocystoma multiplex and pachyonychia
congenita type 2 has a defect in :

A. Keratin 17
B. Keratin 18
C. Keratin 19
D. Keratin 20
E. Keratin 21

649
►A

A patient with this condition has a defect in keratin 17. These conditions can be seen together since
they share the same defect in the genotype. Both pachyonychia congenital type 2 and steatocystoma
multiplex have a defect in keratin 17.

436- A patient has shiny, firm translucent papules around the mouth, nose and eyes. There is also
pinch purpura and glossitis. The patient has amyloidosis has an association with:

A. Multiple myeloma
B. T cell Lymphoma
C. B cell Lymphoma
D. Leukemia
E. Chronic lymphocytic leukemia

►A

Patients with amyloidosis has an association malignancy with multiple myeloma and plasma cell
dyscrasia. The pathology is eosinophilic, amorphic fissured globules in the dermis. The amyloid is
composed of immunoglobulin lambda light chains.

437- Which of the following is true regarding this diagnosis?

A. The protein deposited is derived from Ig light chains, kappa subtype


B. Skin is involved in <10% of cases
C. A different protein is found in skin lesions associated with a plasmacytoma
D. Bullae, when present, are subepidermal

650
E. Factor V deficiencies are commonly associated with this entity

►D

Primary sytemic amyloidosis involves the skin in 40% of cases. The tongue, heart, and GI tract are
commonly involved. The protein AL is derived from Ig light chains (lambda subtype); AL is also
found in nodular or tumefactive cutaneous amyloidosis produced by a plasmacytoma. Glossitis is
common, and may lead to dysphagia; the lateral aspects of the tongue often show indentations from
teeth. Purpuric lesions result from amyloid infiltration of blood vessels, and occur after trauma
(pinch purpura). Bullous amyloidosis presents with tense, hemorrhagic bullae at areas of trauma;
lesions are subepidermal. Arthropathies of small joints, enlarged deltoids (shoulder pad sign), factor
IX and X deficiencies, cardiac arrythmias, and CHF may all result.

438- The primary component of human sebum is:

A. Triglycerides
B. Free fatty acids
C. Ceramides
D. Cholesterol
E. Squalene

►A

Human sebum is rich in triglycerides. The lipase made by P. acnes cleaves triglycerides to free fatty
acids.

439- What is the most likely diagnosis?

A. Voight line
B. Linea nigra
C. Fuchter line
D. Phytophotodermatitis
E. Incontinentia pigmenti

651
►B

Linea nigra is a hyperpigmented, linear patch that often becomes apparent during pregnancy. It is
thought to be secondary to hormonal fluctuations and usually spontaneously resolves after
pregnancy. Voight and Fuchter lines delineate a vestigial hyperpigmentation of the dorsal aspect of
organisms.

440- Patients with Dermatitis Herpetiformis are most likely to have:

A. Antibodies to BPAg2
B. Antibodies to transglutaminase 3
C. Mutations in transglutaminase I
D. Mutations in laminin 5
E. Mutations in plectin

►B

In dermatitis herpetiformis, antibodies are found to transglutaminase 3, and the direct


immunofluorescent studies show granular IgA and C3 in the dermal papillae. Antibodies to BPag2
are found in bullous pemphigoid. Mutations in plectin are found in EBS with muscular dystrophy.
Mutations in laminin 5 are found in patients with JEB,Herlitz type. Mutations in transglutaminase I
are found in pateints with lamellar ichthyosis and non bullous congenital ichthyosiform
erythroderma.

441- Phrynoderma can be seen in all of the following nutritional deficiency except

A. Vitamin A
B. Vitamin B
C. Vitamin C
D. Vitamin D
E. Vitamin E

►D

652
Phrynoderma or ―toad skinǁ• is typically associated with vitamin A deficiency. These keratotic
follicular papules often first develop on anterolateral thighs and posterolateral upper arms then
spread to extremities, shoulders, abdomen and back. Although phrynoderma is originally reported in
association with vitamin A deficiency, it can also be observed with defeciencies in Bcomplex
vitamins and vitamins C and E, in addition to essential fatty acid deficiency.

442 -The first step in the formation of a closed comedone is:

A. Obstruction of the pilosebaceous duct


B. Increased sebum production
C. Colonization by P. acnes
D. Increased hormonal stimulation of the pilosebaceous unit
E. Clogging of the pore with make-up

►A

The first step in the formation of a microcomedone is the obstruction of the pilosebaceous unit by
keratinocytes. After that has occured, increased sebum and P. acnes can contribute to the further
formation of the comedone.

443- Sturge-Weber syndrome is a vascular malformation. It can occur in the first year of life and
more common in patients with bilateral port wine stains. The most common neurologic abnormality
is:
A. Seizures
B. Migraines
C. Glaucoma
D. Tram track calcification
E. Tunnel vision

►A

Patients with Sturge-Weber have seizures as a common neurologic abnormality. They can have
glaucoma in 2/3 of patients at birth. Tram track calcifications don't appear on XR before two years
old.

653
444- This syndrome is characterized with fever, erythema nodosum, bilateral hilar adenopathy, and
arthralgias:

A. Lofgren syndrom
B. Heerfordt syndrome
C. Richner Hanhart syndrome
D. Wilson's syndrme
E. Carney syndrome

►A

Lofgren syndrome is characterized by fever, erythema nodosum, bilateral hilar adenopathy, and
arthralgias. It is associated with anterior uveitis in 6% of patients.

445- A 25 year-old previously healthy man presents with the skin findings shown, urethritis, and
one month of peripheral arthritis. Which of the following is true regarding this condition?

A. Females and males are equally affected


B. A chronic deforming arthritis occurs in 20%
C. TNF-alpha inhibiting agents have no role in the treatment of this condition
D. Patients must have urethritis, conjunctivitis, and arthritis for diagnosis
E. Chlamydia cervicitis is not associated with this condition

►B

The patient has Reiter's syndrome. Reiter's syndrome is a chronic inflammatory disease similar to
psoriasis with psoriatic arthritis, and is thought to be a variant form. The classic triad consists of
urethritis, conjunctivitis, and arthritis. Few patients present with the classic triad, and thus the
syndrome can be diagnosed with peripheral arthritis >1 month duration and associated urethritis (or

654
cervicitis). It occurs in young men with the HLA-B27 genotype and rarely occurs in women. Skin
findings include keratoderma blennorrhagicum and circinate balanitis (in men), as well as oral
erosions, severe stomatitis, and nail changes. The course of disease is marked by exacerbation and
remission; a chronic deforming arthritis occurs in 20%. Treatment includes topical steroids,
NSAIDs, methotrexate, acitretin, cyclosporine, and TNF -inhibiting biologics, such as etanercept.

446 -What is the most photosensitive form of all cutaneous lupus subtypes?

A. Acute cutaneous lupus


B. Subacute cutaneous lupus
C. Classic discoid lupus
D. Lupus tumidus
E. Chiblain

►D

It is a rare entity. Patients complaint of edematous and erythematous plaques, usually on the trunk.
The lesions generally respond to antimalarials.

447- Sneddon-Wilkinson Disease:

A. Most often occurs in elderly women


B. Rarely involves intertriginous areas
C. May occur in association with an IgG monoclonal gammopathy
D. Can be treated with narrow band UVB
E. Is usually an acute, self-limited condition

►D

Sneddon-Wilkinson disease, or subcorneal pustular dermatosis, presents with superficial pustules in


annular and serpiginous patterns in the axillae, groin, and abdomen. Middle-aged women are most
often affected. This condition rarely occurs in association with an IgA monoclonal gammopathy. It
is a chronic condition, possibly related to psoriasis, with remissions of variable duration. Treatments
including dapsone, acitretin, and narrow band UVB.

655
448 -This 57-year old male complains of weakness when climbing the stairs and this clinical
presentation. Which cytokine has been implicated in the etiology of this condition?

A. IL-1
B. Il-10
C. IL-5
D. Interferon-gamma
E. Tumor necrosis factor alpha

►E

Dermatomyositis is an autoimmune condition which presents with typical skin findings and muscle
weakness. Age-appropriate screening should be done for internal malignancy as there is a higher
incidence of cancer in these patients. Polymorphisms of tumor necrosis factor-alpha have been
implicated in the etiology. Interferon alpha has also been implicated.

449 - Patients with Gottrons papules and periungual telangiectasias will likely have a positive
serologic test to which antibody?

A. Anti-SM
B. Scl-70
C. Anti-Ro
D. Anti-centromere
E. Anti- Jo-1

►E

Gottron's papules and periungual telangiectasias (Samitz's sign) are among the cutaneous features of
dermatomyositis, which also include heliotrope rash, photosensitive poikiloderma of the upper back
(shawl sign). Anti-Jo-1 antibody is present in 20-30% of these patients. The target antigen is anti-
histidyl-tRNA synthetase (Jo-1). The presence of this antibody corresponds to the development of
pulmonary disease.

656
450 -There are many HLA types that are associated with psoriasis. The HLA association that has a
relative risk that is 9-15 times the normal limit is:

A. HLA-Cw6
B. HLA-B13
C. HLA-Bw57
D. HLA-B17
E. HLA-B20

►A

HLA-Cw6 has a relative risk of 9-15 times the normal risk for psoriasis. HLA B-17 is associated
with early onset and more serious disease.

451- An elderly gentleman with rheumatoid arthritis has a 2-month history of recurrent painful, red,
swollen ears and hearing loss. Physical exam is notable for sparing of the earlobes. You suspect he
has:
A. Chondrodermatitis nodularis helices
B. Recurrent otitis externa
C. Relapsing polychondritis
D. Systemic lupus erthematosus
E. Severe seborrheic dermatitis

►C

Relapsing polychondritis is a rare disease manifested by recurring inflammation of cartilagi nous


tissue. Antibodies to type II collagen are thought to be pathogenic in this disease. Clinically, patients
have auricular chondritis and arthritis. The chondritis is limited to the cartilaginous portion of the
external ears. Involvement of the cartilaginous portions of the eye, respiratory tract, the inner ear,
and the cardiovascular system has been reported as well. A significant portion of patients with
relapsing polychondritis have an associated rheumatic or autoimmune disease.

657
452- Industrial acne can be caused by insoluble oils seen on the scalp, malar cheeks and scrotum.
All of the following have been known to cause acneiform eruptions except:

A. Dioxin
B. Halogen bromide
C. Iodide
D. Androgenic hormones
E. Tazarotene

►E

Dioxin, halogen bromide, iodide, androgenic hormones such as testosterone can cause acneiform
eruption. Tazarotene is a topical retinoid used to treat acne.

453- A farmer develops a rash in photodistributed areas due to a crop that he has been handling. The
most common cause is:

A. Celery
B. Apples
C. Lemons
D. Lime
E. Tomatoes

►A

The celery is a common photosensitizing agent. It can cause both a phototoxic and photoallergic
reactions. Celery and parsnip and weeds also can cause these reactions.

454- Which of following medications may increase the likelihood of pseudotumor cerebri in
patients taking oral isotretinoin?

A. Spironolactone
B. Dapsone
C. Amoxicillin

658
D. Rifampin
E. Tetracycline

►E

Pseudotumor cerebri, or benign intracranial hypertension is more common in adolescent and young
adult women, but can occur in children. Medicines reported to be associated with the condition
include vitamin A analogues, tetracyclines, steroids (especially in withdrawal), nalidixic acid,
sulphonamides, lithium, thyroxine, growth hormone, amiodarone and tamoxifen. It most often
presents with headache (90% of cases), pulsatile in quality. Less frequent symptoms are visual
disturbances and pulsatile tinnitus. Pseudotumor can also be completely asymptomatic. The
mechanism is not fully understood but current opinion favors impaired reabsorption of
cerebrospinal fluid. Of the medicines associated with the condition, minocycline is most frequently
reported in the literature followed by tetracycline and doxycycline. Isotretinion has been repored to
cause it and it is possible that the incidence of pseudotumor may increase if two or more drugs
which might cause it are used together. For this reason tetracyclines should not be prescribed
concomitantly with oral retinoids.

455 -A patient has a contact allergy from her contact lens and ophthalmic solution. The allergen that
is most likely the culprit is:

A. Thimerosal
B. PPD
C. Benzene
D. Tuliposidase A
E. Formaldehyde

►A

Contact lens and ophthalmic solutions have the allergen thimerosal.

456- The nail changes seen in the capillary nail folds from the autoimmune connective tissue
disorders such as dermatomyositis and scleroderma are:

659
A. roughness, hemorrhages, necrosis of the cuticles
B. vascular areas with condensed capillary areas
C. normal density
D. tortuous and dilated
E. increased capillary density

►A

Patients that have dermatomyositis and scleroderma have roughness, hemorrhages, and necrosis of
the cuticles. Patients that have scleroderma have reduced capillary density and avascular areas
which alternate with dilated capillary loupes. Systemic lupus erythematosus reveals a normal
density of capillaries which are tortuous and dilated.

457- The most common cause of mortality in patients with limited systemic sclerosis is:

A. Myocardial infarction
B. Pulmonary fibrosis
C. Pulmonary hypertension
D. Renal failure
E. Sepsis

►C

The most common cause of morbidity and mortality in patients with limited systemic sclerosis is
pulmonary hypertension. Pulmonary fibrosis is more often associated with diffuse systemic
sclerosis. Mortality from renal failure has been significantly reduced by the use of ACE inhibitors.

458 -A patient presents to the ER with skin pain and large sheets of epidermal detachment on
greater than 10% of his body surface area after starting bactrim. No purpuric macules or target
lesions are noted. Which of the following can help predict mortality in this patient?

A. glucose greater than 150 mg/dL


B. age over 40 years
C. heart rate greater than 100

660
D. history of diabetes
E. BUN greater than 15 mg/dL

►B

The patient described has TEN. A diagnosis of TEN can be made if the patient presents with
epidermal detachment of >30% of the body surface with widespread purpuric macules or flat
atypical targets, or if there are large sheets of epidermal detachment involving >10% of the body
surface without purpuric macules or target lesions. The SCORTEN scoring system was developed
to assess severity of illness and predict mortality in TEN, and is meant to be calculated within the
first 24 hours after admission and again on day three. The score is the sum of the following clinical
variables: (1) age over 40 years; (2) heart rate >120 beats per minute; (3) the presence of cancer or
hematologic malignancy; (4) epidermal detachment involving body surface area >10% on day one;
(5) blood urea nitrogen >28 mg/dL (10 mmol/L); (6) glucose >252 mg/dL (14 mmol/L); and (7)
bicarbonate >20 mEq/L. The mortality increases sharply with each additional point, with a score of
5 or greater having a 90% mortality.

459- Which organic system is least likely to be involved in the attached image?

A. Liver
B. Spleen
C. Genitourinary
D. Gastrointestinal tract
E. Bone marrow

►C

Attached image shows mastocytosis. The disease presents classically with tan-like or red brown
macules or papules. There are different forms of cutaneous mastocytosis, most common form is

661
urticaria pigmentosa which can appear in both children and adults and diffuse cutaneous
mastocytosis which occur almost exclusively in infants, solitary mastocytoma and telangiectasia
macularis eruptiva perstans. Systemic involvement occurs most commonly in adults and bone
marrow,lymph nodes, liver, spleen and GIT are among the most common involved systems whereas
genitourinary and neuroendocrine systems are least likely to be involved.

460 -Which lab abnormality is most associated with sarcoidosis?

A. Hypercalcemia
B. Hyperkalemia
C. Hypernatremia
D. Hyperchloremia
E. Hyperlipidemia

►A

Up to 10 % of patients with sarcoidosis may have hypercalcemia. Resultant hypercalciuria and


nephrolithiasis may lead to renal failure. The other listed lab abnormalities are not typically
associated with sarcoidosis.

461- Which of the following MOST favors a diagnosis of Sweet‘s over atypical (bullous) pyoderma
gangrenosum?

A. Recurrence of lesions
B. Presence of lesions on the H/N and hands
C. Association with infections
D. Presence of constitutional symptoms
E. Presence of hemorrhagic bulla and ulcerations

►D

Both Sweet‘s syndrome and atypical pyoderma gangrenosum fall within the category of
neutrophilic dermatoses with bullous and ulcerated lesions. Both favor the head and neck and
extremities. Both can recur, and both demonstrate a heavy neutrophilic infiltrate on histopathology.

662
Both are associated with infections and hematologic disorders, though atypical PG is more likely
associated with hematologic malignancy. The presence of constitutional symptoms such as fever,
arthralgia, and arthritis are strongly associated with Sweet‘s and can assist in differentiation
between these entities.

462- The most common associated disorder in a patient with elastosis perforans serpiginosa is:

A. Marfan syndrome
B. Down syndrome
C. Osteogenesis imperfecta
D. Ehlers-Danlos syndrome
E. Rothmund-Thomsom syndrome

►B

About 1/3 of case of elastosis perforans serpiginosa occur in patients with other concomitant
disorders. All of the above can occur in patients with EPS; however the most common is Down
syndrome. One mnemonic to remember the associated disorders is: PROMEDA. This stands for
PXE,Rothmund-Thomson, Osteogenesis imperfecta,Marfan syndrome, Ehlers-Danlos syndrome,
Down syndrome, acrogeria.

463 -What autoimmune disease is associated with a PRP-like eruption?

A. Systemic lupus
B. Rheumatoid arthritis
C. Dermatomyositis
D. Scleroderma
E. Pemphigus erythematosus

►C

The Wong type of dermatomyositis is characterized by erythematous, hyperkeratotic, follicular


papules . It can have a limited or generalized distribution. It can evolve into full -blown
dermatomyositis with classic cutaneous features. This rare variant bears a striking clinical

663
resemblance to PRP. It can be distinguished on skin biopsy. It has been reported in both adults
and children. Some reports suggest the follicular papules are confined to the skin overlying the
extensor knees and elbows and this variant may be more common in Asian patients.

464 -A patient with chronic lymphedema has swelling and verrucous changes with hypertrophic
fibrosis. Patient also has mossy leg and lymphangitis. This patient most likely has:

A. Elephantiasis nostras verrucosa


B. Acanthosis nigricans
C. Palmoplantar keratoderma
D. Psoriasis
E. Seborrheic keratosis

►A

This patient most likely has elephatiasis nostra, which is a complication where there is swelling and
verrucous changes to the skin. The other choices do not describe the changes that you would see.
Psoriasis are silvery scales and PPK are thickened heels. Acanthosis nigracans are velvety changes
that occur on the axillary region and the neck.

465 -Which of the following treatments for acne inhibits RNA-dependent protein synthesis by
binding to the 50s ribosomal subunit?

A. Erythromycin
B. Tetracycline
C. Trimethoprim-sulfamethoxazle
D. Benzoyl peroxide
E. Azeleic acid

►A

Macrolide antibiotics inhibit RNA-dependent protein synthesis by binding to the 50s ribosomal
subunit, and include erythromycin and azithromycin. Tetracyclines inhibit RNA-dependent protein
synthesis by binding to the bacterial 30s ribosomal subunit. TMP-SMX inhibits bacterial folic acid

664
synthesis. Benzoyl peroxide is a bactericidal agent with direct oxidizing effects. Azeleic acid is a
dicarboxylic acid that inhibits tyrosinase; the mechanism of action against P. acnes is not
completely understood.

466 -A patient is found to have an allergy to a substance in wrinkle resistant clothing. The allergen
is most likely:

A. Formadehyde
B. Thimerosal
C. Tuliposidase A
D. PPD
E. Benzene

►A

The most common allergen found in wrinkle free clothing is formaldehyde.

467 -In the setting of the lupus erythematosus-associated complement deficiency syndrome, which
of the following would most often be observed?

A. low C3 and C4
B. low C2 and C3
C. low C2 and C4
D. low C2 only
E. low C3 only

►C

In the complement deficiency syndrome, low C2 and C4 are most commonly seen.
Photosensitivity, annular SCLE lesions, and Ro antibody formation are commonly observed.

665
468- What is the most common paraproteinemia in scleromyxedema?

A. IgA
B. IgG kappa
C. IgG lambda
D. IgM
E. It is extremely uncommon to see a paraproteinemia with scleromyxedema

►C

An abnormal paraproteinemia is found in 90% of cases of scleromyxedema, usually IgG lamda.

469- Steven Johnson syndrome and TEN is a hypersensitivity syndrome most often caused by
drugs, infections and rarely cancers. The most common cause of SJS/TEN is:

A. Allopurinol
B. Bactrim
C. Prednisone
D. Hydrocholorothiazide
E. Carbamazepine

►A

SJS and TEN is a hypersensitivity syndrome characterized by epidermal necrosis. The most
common cause is allopurinol.

470 -A patient has a contact allergy to an ingredient found in permanent hair dyes. The most likely
allergen is:

A. PPD
B. Tuliposidase A
C. Thimerosal
D. Fomaldehyde
E. Parabens

666
►A

Patients that are allergic to permanent hair dyes are allergic to PPD. The contact dermatitis can
present as a erythematous rash on the hands or on the frontal scalp.

471- The protein component of nodular amyloidosis is:

A. SAA protein
B. AL protein
C. Keratin
D. Collagen
E. Bp180

►B

Nodular amyloidosis presents as single or multiple nodules, often on the extremities. The primary
protein component is of the AL type.

472- Neutrophilic dermatoses en plaque is often associated with which of the following?

A. IgA monoclonal gammopathy and a benign course


B. IgA monoclonal gammopathy and a malignant course
C. Myeloma and B lymphomas and a benign course
D. IgG monoclonal gammopathy and a benign course
E. IgG monoclonal gammopathy and a malignant course

►A

Neutrophilic dermatoses en plaque are well defined, sharply demarcated intensely red plaques.
Myeloma and B cell lymphomas are rarely associated. These resolve with treatment of the
gammopathy.

667
473- An overweight, post-menopausal woman presents with intense pain in the fatty deposits on her
knees, thighs and hips. She also describes swelling of her hands and feet, associated fatigue, and
history of depression. On clinical exam the pain in the adipose tissue appears out of proportion to
the clinical findings. The diagnosis is:

A. Angiolipomas
B. Adiposis dolorosa
C. Congenital lipodystrophy
D. Lipodermatosclerosis
E. Erythema nodosum

►B

Adiposis dolorosa (Dercum\'s disease) is a disorder that usually occurs in obese women 40 to 60
years of age, and it is characterized by pain in adipose tissue that appears to be out of proportion to
the physical findings. The pain can be localized to multiple, painful lipomas on lower extremities
and knees. This condition is accompanied by swelling of different areas of the body, such as hand
and feet, which may be transient. Patients typically have subjective fatigue or confusion, and may
have history of depression or emotional instability. The cause of adiposis dolorosa is not known.
The cause of the pain is speculated to be due to the pressure on nerves by the adipose tissue
deposits. Therapeutic treatments are not very effective, and can involved procedures such as
liposuction, systemic corticosteroids, pregabalin, lidocaine, and psychiatric care.

474- An 8 year old boy with seasonal allergies presents with red brown macules on the trunk
present for several months. Parent mentions that when these lesions are scratched they form an
irritated, red wheal. Which of the following sign is likely described?

A. Asboe-Hansen sign
B. Homan’s sign
C. Darier’s sign
D. Auspitz sign
E. Hutchinson sign

►C

668
Darier’s sign is seen in patients with urticarial pigmentosa, a common form of mastocytosis.
Erythema and wheals are commonly elicited with stroking or rubbing secondary to mast cell
degranulation. Symptoms can range from very mild (flushing, hives, no treatment needed) to life-
threatening (vascular collapse). Asboe Hansen, commonly seen in pemphigus vulgaris, refers to
extension of a bulla to adjacent unblistered skin when pressure is put on top of a bulla.

475 -The most common cause of allergic contact dermatitis overall in clinical studies is:

A. Nickel
B. Jewelry
C. Dimethylglyoxime
D. Orange Pekoe tea
E. Epoxy Resin

►A

The most common cause of allergic contact dermatitis is nickel and jewelry is also another common
allergen. Dimethylglyoxime test is used to detect nickel. If you rub on the item the solution turns
color from pink to reddish that can indicate a positive reaction.

476- Pyostomatitis vegetans is characteristically associated with which systemic disease?

A. Pemphigus vulgaris
B. Ulcerative colitis
C. Lichen planus
D. Lymphoma
E. Rheumatoid arthritis

►B

Pyostomatitis vegetans is a pustular, vegetative variant of pyoderma gangrenosum, found in the oral
mucous membranes. It is most frequently associated with inflammatory bowel disease.

669
477- Bullous lupus erythematosus is most commonly associated with antibodies to:

A. Type IV collagen
B. Laminin 5
C. Desmoglein I
D. Plectin
E. Type VII collagen

►E

Type VII collagen is found in the sublamina densa and patients with bullous lupus erythematosus
have been found to have antibodies to this protein. Bullous LE and EBA share antibasement
membrane zone antibodies of identical specificity (type VII collagen) as well as clinical and
histologic overlap. A differentiating feature between bullous LE and EBA is that bullous LE has a
dramatic response to dapsone.

478- There are four types of kaposi sarcoma and can be associated with HIV. The viral strain
associated with classic AIDS-associated kaposi sarcoma is:

A. HHV-8
B. HHV-6
C. HHV-5
D. HHV-4
E. HHV-3

►A

670
Patients with kaposi sarcoma associated with AIDs can have kaposi sarcoma associated with HHV8.
It can present as a erythematous purple plaque.

479- Eyebrow growth:

A. Is androgen-dependent in men but not in women


B. Is regulated by dehydroepiandrosterone but not androstenedione
C. Is not androgen-dependent
D. Requires conversion of testosterone to dihydrotestosterone in the hair follicle
E. Is regulated only by testosterone and dihydrotestosterone

►C

Eyebrows, eyelashes, and vellus hair are not androgen-dependent, thus there is no difference
between these areas of hair growth in men and women.

480- A 48 year old woman was recently diagnosed with dermatomyositis. Which examination
would be most important in a work up for malignancy?

A. Breast exam
B. Chest xray
C. Thyroid exam
D. Pelvic exam
E. Stool guiac

►D

Dermatomyositis is an idiopathic inflammatory disease with myositis and characteristic cutaneous


manifestations. There is an increased incidence of malignancy in these patients which may precede,
occur with or follow the diagnosis of dermatomyositis. The most common form of malignancy in
adult women is ovarian cancer. Other malignancies that have been associated include testicular
cancer, gastrointestinal, lung and nasopharyngeal carcinomas.

671
481 -Which of the following is the most common type of melanoma found in black patients?

A. Acral lentiginous melanoma


B. Superficial spreading melanoma
C. Nodular melanoma
D. Amelanotic melanoma
E. Lentigo maligna melanoma

►A

Overall, superficial spreading melanomas are most frequently seen. However, acral lentiginous
melanoma is most frequently seen in patients with darker skin types. It is seen on the palms, soles,
and nail units.

482- A patient presents with lesion that can be herniated into the skin on palpation. What is thought
to be important in the pathogenesis of this condition?

A. Calcification of the elastic fibers


B. Formation of excessive collagen
C. Focal loss of elastic tissue
D. Atrophy of the epidermis
E. Inflammation of the panniculus

►C

Anetoderma is a benign condition caused by focal loss of elastic tissue. Primary or idiopathic
anetoderma originates from previously healthy skin with unknown pathogenesis. Various ocular,
bony, cardiac, and other abnormalities have been reported with primary anetoderma. Secondary
anetoderma occurs after the resolution of an inflammatory disease of the skin.

483- This type of psoriasis is the acute variant with a fever that last for several days with eruption of
sterile pustules 2-3 mm in diameter paralleling the fever:

A. Von Zumbusch type

672
B. Psoriatic erythroderma
C. Eruptive psoriasis
D. Localized pustular psoriasis
E. Plaque type psoriasis

►A

Von Zumbusch type is also known as generalized pustular psoriasis. The sterile pustules is on trunk,
extremities including the nail beds, palms, and soles. Pustules arise in highly erythematous skin.

484 -P. acnes activation of which of the following may stimulate a pro-inflammatory cytokine
cascade?

A. TLR-2
B. TLR-5
C. IL-1
D. IL-8
E. TNF-000

►A

Toll-like receptors are a large group of receptors that recognize a variety of bacterial motifs. P.
acnes has been shown to activate TLR-2, leading to signal transduction and production of
proinflammatory cytokines.

485- Necrobiotic xanthogranuloma is most commonly associated with which of the following:

A. IgA monoclonal gammopathy


B. IgM monoclonal gammopathy
C. IgG lambda monoclonal gammopathy
D. IgG kappa monoclonal gammopathy
E. IgG/IgA polyclonal gammopathy

673
►D

Necrobiotic xanthogranuloma is associated with a monoclonal IgG kappa paraproteinemia in


approximately 80% of cases. It is less commonly associated with an IgA gammopathy. Myeloma or
myelodysplastic syndromes may resultantly occur.

486- What is the most common presentation of psoriatic arthritis?

A. Asymmetric oligo- or polyarthritis


B. Symmetric polyarthritis
C. Spondylitis (axial)
D. Distal interphalangeal joint (DIP) disease
E. Arthritis mutilans

►A

Psoriatic arthritis affects 20-40% of patients with psoriasis. Most (~80%) of psoriatic arthritis
patients are rheumatoid factor negative. All of the answer choices represent possible presentations
of psoriatic arthritis, but asymmetric oligo- or polyarthritis is the most common presentation.

487- What disease is found to be associated with retention of perifollicular pigmentation?

A. Dermatomyositis
B. Secondary syphilis
C. Scleroderma
D. Discoid lupus
E. Sarcoidosis

►C

Scleroderma is a systemic disease which may affect almost any organ in the body. Cutaneous
findings are characterized by symmetric, thickening of the skin. Dyspigmentation may occur with a
"salt and pepper" appearance due to perifollicular retention of pigmentation with depigmentation of
the interfollicular skin.

674
488- A positive ANA with a speckled staining pattern correlates with staining of what cellular
component?

A. Ribonucleoproteins
B. Nucleolar RNA
C. Native DNA
D. Kinetochore
E. Histones

►A

A speckled ANA pattern correlates with staining of ribonucleoproteins and is seen in Mixed
Connective Tissue Disease (MCTD), Systemic Lupus Erythematosus (SLE), systemic scle rosis and
Sjogren's syndrome. A nucleolar staining pattern correlates with staining of nucleolar RNA.
Centromere staining correlates with staining of kinetochore. A peripheral staining pattern correlates
with staining of native DNA and a homogeneous staining pattern correlates with staining of native
DNA and histones.

489- Which of the following is true regarding this entity?

A. Extragenital lesions commonly involve the torso and are usually pruritic
B. Childhood onset is reported in ~5% of cases
C. 20% of both men and women with this diagnosis have at least one autoimmune disease
D. Girls outnumber boys 2:1
E. Genital disease represents 50% of childhood cases

►C

675
The patient pictured has lichen sclerosus. This likely represents an autoimmune phenomenon, as
20% of both men and women have at least one autoimmune disease (vitiligo, alopecia areata, or
thyroid disease), and a larger proportion have circulating antibodies. Extragenital lesions most
frequently involve the torso and are usually asymptomatic. Childhood onset occurs in 10-15% of
cases, and girls outnumber boys 10:1. Genital disease represents 90% of childhood lichen sclerosus.

490- The substance that is responsible for the yellow tattoo color is:

A. Cadmium sulfide
B. Cobalt aluminate
C. Ferric oxide
D. Chromic oxide
E. Cinnabar

►A

Cadmium sulfide is responsible for the yellow color in tattoos.

491- Androstenedione is produced by:

A. The adrenals
B. The ovaries
C. The adrenals and ovaries
D. Extraglandular conversion
E. The adrenals, ovaries, and extraglandular conversion

►C

Androstenedione is produced by the adrenals and the ovaries. Testosterone is produced by the
adrenals, ovaries, and by extraglandular conversion of androstenedione and
dehydroepiandrosterone.

676
492- A patient's biopsy is found to have curled hair shafts with hyperkeratotic plugs. Which of the
following is not an associated feature of this condition

A. Hypochondriasis
B. Gingivitis
C. Delayed wound healing
D. Nail changes
E. Tender nodules on the lower extremities

►D

The patient's biopsy demonstrates the corkscrew hairs of scurvy, or vitamin C deficiency, which is
characterized by the 4 "H"s: hemorrhagic signs (tender nodules on the lower extremities),
hyperkeratosis of hair follicle, hypochondriasis, and hematology abnormalities

493- Which of the following is true regarding atopic dermatitis?

A. Over 80% of affected individuals present in the first year of life


B. Natural measles infection has been noted to improve atopic dermatitis
C. Most children develop worsening of atopic dermatitis if they develop asthma later in
childhood
D. Staphylococcus aureus is found in ~25% of atopic dermatitis skin lesions
E. Increased expression of cathelicidins such as LL 37 has been noted in atopic dermatitis

►B

Natural measles infection has been noted to improve atopic dermatitis. Atopic dermatitis typically
begins in infancy, with ~50% in the first year of life and an additional 30% between 1 and 5 years.
Most children with AD eventually develop allergic rhinitis or asthma later in childhood. Many
outgrow AD as respiratory allergy develops. S. aureus is found in over 90% of AD skin lesions.
Decreased expression of innate antimicrobial peptides, such as human beta defensin and
cathelicidins, such as LL 37, may explain the increased susceptibility to colonization and skin
infection with S. aureus in patients with atopic dermatitis.

677
494- Which of the following is NOT a feature associated with this condition?

A. Accelerated blanch response


B. White dermatographism
C. Anterior subcapsular cataracts
D. Keratoconus
E. Pityriasis alba

►A

The child pictured has atopic dermatitis. Features associated with atopic dermatitis include
pityriasis alba, white dermatographism and delayed blanch reponse, anterior subcapsular cataracts,
and keratoconus.

495- Ulcerative colitis and crohn'€™s disease are most commonly associated with which of the
following:

A. Pyoderma gangrenosum
B. Erythema elevatum diutinum
C. Sweet'€™s syndrome
D. IgA pemphigus
E. Sneddon-Wilkinson's disease

►C

678
Sweet's syndrome is also associated with acute myelogenous leukemia, colony-stimulating factor,
all-trans retinoic acid and tetracyclines. Flu-like symptoms, high-grade fever, malaise and
peripheral neutrophils accompany the cutaneous lesions. Steroids and dapsone are the treatments of
choice.

496- What is the most common site of extracutaneous involvement in this non-infectious disease?

A. Lungs
B. Eyes
C. Bone Marrow
D. Liver
E. Nasal mucosa

►A

Sarcoidosis is a non-infectious granulomatous disorder of unclear etiology. It has a predilection for


the lungs (90%), lymph nodes (75-90%), eyes (25%), nasal mucosa (20%), bone marrow (2540%),
and liver (15-40%).

497- Which of the following is associated with this scarring condition?

679
A. Thyroid disease
B. Acne conglobata
C. Sarcoidosis
D. Systemic lupus erythematosus
E. Lichen planus

►B

Dissecting cellulitis is part of the follicular occlusion tetrad which also includes acne conglobata,
hidradenitis suppurtiva, and pilonidal cysts.

498- Which of the following is a clinical manifestation of neonatal lupus erythematosus:

A. Renal disease
B. Cerebral vascular accident
C. Arthritis
D. Thrombocytopenia
E. Neutrophilia

►D

Neonatal lupus involves annular scaling macules and plaques appearing within the first few months
of life in babies born to mothers with LE or other autoimmune connective tissue diseases. In
addition to the rash, babies display congenital heart block, hepatic disease, and thrombocytopenia.

499- A 42 year-old woman has the findings pictured, as well as erythematous patches on the upper
chest and shoulders. No Raynaud's, polyarthritis, pulmonary, or cardiac involvement is noted.
Which antibody might you expect to be present in this patient?

A. Anti-SRB
B. Anti-Jo-1
C. Anti-Mi-2
D. Anti-Ku
E. Anti-La

680
►C

This patient has dermatomyositis. Anti-Mi-2 antibodies in DM correlate with the presence of a
shawl sign, cuticular changes, and good prognosis. Anti-Jo-1 antibodies correlate with pulmonary
fibrosis, Raynaud's, and polyarthritis. Anti-SRP antibodies correlate with cardiac disease and poor
prognosis. Anti-Ku antibodies correlate with sclerodermatomyositis. Anti-La antibodies correlate
with Sjogren's syndrome.

500- A patient develops pulmonary fibrosis, Raynaud's phenomenon and skin changes similar to
systemic sclerosis. These changes resolve following discontinuation of which of the following
medications?

A. Bleomycin
B. Penicillamine
C. Hydralazine
D. Minocycline
E. Isoniazide

►A

The changes described above are similar to findings of scleroderma. Patients receiving bleomycin
can develop a reversible syndrome similar to scleroderma. The other four listed options are related
to drug-induced lupus erythematosus and are not linked to scleroderma-like syndromes.

501- Periorbital purpura is a characteristic cutaneous manifestation of:

A. Sarcoidosis
B. Amyloidosis
C. Dermatomyositis
D. Lymphoma
E. Thrombocytopenia

►B

681
Primary systemic amyloidosis, often myeloma associated, is characterized by the deposition of
amyloid fibrils derived from immunoglobulin light chains. 40% of patients manifest with cutaneous
findings including petechiae, purpura, waxy translucent papules on the eyelids, diffuse alopecia,
scleroderma and nail dystrophy.

502- In cryosurgery, the temperature needed for destruction of keratinocytes is?

A. 0-10 degrees C
B. -20- -30 degrees C
C. -5 degrees C
D. -50 degrees C
E. -196 degrees C

►B

Cryosurgery is the application of low temperature to produce local tissue destruction. -20- -30
degrees C is needed to destroy keratinocytes. -4- -7 degrees are needed to destroy melanocytes.
Temperatures of -50 are recommended to treat malignant lesions.

503- The condition shown here may be exacerbated by the inappropriate use of which of the
following?

A. Comedogenic make-up foundation


B. Sunscreens
C. Erythromycin 2% ointment
D. Topical corticosteroids
E. Mineral oil-containing moisturizers

682
►D

This slide shows perioral dermatitis, characterized by small, erythematous papules in a perioral,
often periorificial, distribution. Topical corticosteroids can exacerbate or trigger perioral dermatitis.
Note the absence of comedones, which distinguishes rosacea and its subsets from acne.

504- A 48 year-old man with a long history of alcohol abuse and dependence presents with angular
cheilitis, atrophic glossitis, a scrotal dermatitis that spares the midline and extends to t he thighs,
photophobia, and blepharitis. Which nutritional deficiency do you suspect?

A. Vitamin B12
B. Vitamin C
C. Pyridoxine
D. Vitamin B2
E. Iron

►D

The patient presents with classic findings of the oral-ocular-genital syndrome seen in the setting of
vitamin B2 (riboflavin) deficiency. This deficiency occurs most often in alcoholics. It
characteristically presents with angular cheilitis, atrophic glossitis (magenta), a seborrheic-like
dermatitis around the nose, genital dermatitis (scrotal dermatitis sparing the mi dline and extending
to the thighs), photophobia, and blepharitis.

505- The hyperproliferative epithelium of this mature psoriasis plaque is associated with increased
expression of which keratin(s)?

A. K1, K10
B. K5, K14
C. K6, K16
D. K17
E. K2e

►C

683
Expression of keratins K6 and K16 is upregulated in hyperprolferative psoriasis plaques.

506- Folliculotropic CTCL presents most commonly on the head and neck. The treatment of choice
is :

A. PUVA
B. Narrow band UVB
C. Methotrexate
D. Mycophenolate mofetil
E. Cyclosporin

►A

The most common location for folliculotropic CTCL is head/neck. It can present as a infiltrative
plaque, acneiform lesions or keratosis pilaris like lesions and is usually very pruritic. The treatment
of choice is PUVA.

507- A 3 month old presents with a diffuse vesiculobullous rash with copper colored macules on the
palms and soles. You ascertain from the history that his mother had a nonpainful erosion on her
labia during pregnancy which spontaneously resolved. What other symptoms would you expect this
infant to have?

A. Pseudoparalysis of Parrot
B. Higoumenakis sign
C. Clutton joints
D. Mulberry molars
E. Saddle nose deformity

►A

Pseudoparalysis of Parrot is a sign of early congenital syphilis. Early congenital syphilis occurs
before 2 years of age, whereas late congenital syphilis generally occurs after 2 years of age.
Higoumenakis sign, clutton joints, mulberry molars, and saddle nose deformity are all signs of late
congenital syphilis.

684
508 -The "follicular occlusion "triad" consists of:

A. Acne conglobata, hidradenitis suppurativa, kerion


B. Acne conglobata, chloracne, pyoderma faciale
C. Acne fulminans, hidradenitis suppurativa, dissecting cellulites of the scalp
D. Acne comedonicus, hidradenitis suppurativa, dissecting cellulites of the scalp
E. Acne conglobata, hidradenitis suppurativa, dissecting cellulites of the scalp

►E

Acne conglobata is a sever variant of acne vulgaris characterized by large, often multiple
comedones, abscesses with sinus formation and inflammatory nodules. Hidradenitis suppurativa is a
chronic condition characterized by swollen, painful, inflamed lesions in the axillae, groin, and other
parts of the body that contain apocrine glands. Dissecting cellulites of the scalp, also known as
perifolliculitis capitis abscedens et Suffodiens of Hoffman, consists of deep inflammatory boggy
nodules +/- sinus tracts on the occipital scalp, most commonly in African American males.

509- ACE inhibitors cause angioedema via stimulation of?

A. Bradykinins
B. Histamine
C. Prostaglandins
D. Leukotrienes
E. Complement

►A

Bradykinins are responsible for angiotensin converting enzyme inhibitor induced angioedema.

510 -Teenage boys with severe, eruptive cystic acne and fever, leukocytosis, myalgias and other
constitutional symptoms can have which of the following bony change?

A. Dimpling above the 5th MCP


B. Osteolytic lesions of the clavicle

685
C. Osteopoikilosis
D. Stippled epiphyses
E. Jaw cysts

►B

The syndrome described is that of acne fulminans. Osteolytic bone lesions may accompany this
syndrome, most commonly of the clavicle. Dimpling above the 5th MCP is a feature of Albright‘s
Hereditary Osteodystrophy. Osteopoikilosis is a feature of Buschke-Ollendorf syndrome. Stippled
epiphyses occurs in chondrodysplasia punctata. Jaw cysts are a feature of Gorlin‘s syndrome.

511 -MAGIC syndrome involves:

A. Relapsing polychondritis
B. Acne conglobata
C. Livedo reticularis
D. Lipodystrophy
E. Psoriatic arthritis

►A
MAGIC syndrome is a combination of Behçets disease and relapsing polychondritis. Patients
show mouth and genital ulcers with inflamed cartilage.

512 -The most common benign neoplasm in a nevus sebaceous of Jadassohn is:

A. Syringocystadenoma papilliferum
B. Trichoblastoma
C. Fibroepithelioma
D. Sebaceous adenoma
E. Sebaceoma

►B

686
The most common benign neoplasm is Trichoblastoma.

513 -What is the diagnosis of this lesion on the penile shaft?

A. Sclerosing lymphangitis
B. Loaloa
C. Varicose vein
D. Syphilis
E. Lymphogranuloma venereum

►A

Sclerosing lymphangitis is a self-limited, cord-like structure on the penile shaft, which typically
occurs 24-48 hours after vigorous sexual intercourse.

514 -Sarcoidosis presenting as uveitis, facial nerve palsy, fever and parotid gland swelling is
referred to as:

A. Heerfordt's syndrome
B. Lofgren's syndrome
C. Lupus pernio
D. Darier-Roussy disease
E. Schaumann syndrome

►A

Heerfordt's syndrome is the name given to sarcoidosis presenting with uveitis, facial nerve palsy,

687
fever and parotid gland swelling. Lofgren's syndrome is an acute presentation of sarcoidosis that
presents wth fever, arthritis, erythema nodosum and hilar adenopathy. Darier-Roussy disease is
sarcoidosis presenting as painless firm subcutaneous nodules. Lupus pernio refers to sarcoidosis
presenting as papulonodules and plaques involving the nose (especially the alar rim), ears and
cheeks. Schaumann syndrome was a distractor as Shaumann bodies are seen in sarcoidosis on
histopathology.

515 -A 35-year-old man presents with scattered infiltrative cutaneous plaques, chronic fever, parotid
gland enlargement, and a facial nerve palsy. What other finding is most likely on physical
exam?

A. Anterior uveitis
B. Periungual telangiectasia
C. Nail pits
D. Non-scarring alopecia
E. Condyloma lata

►A

Heerfordt-Waldenstrom Syndrome is a form of sarcoidosis consisting of fever, parotid enlargement,


facial nerve palsy, and anterior uveitis. Periungual telangiectasias are commonly seen in collage
vascular diseases. Nail pits are commonly seen in psoriasis and alopecia areata. Sarcoidosis
involvement of the scalp often results in a scarring alopecia. Condyloma lata is seen in secondary
syphilis.

516- A young man treated with minocycline developed blue-black discoloration with acne scars at
his cheeks. A Perls stain would show:

A. Increased melanin at the basal layer of the epidermis


B. Black staining granules within macrophages
C. Blue staining granules within macrophages
D. Muddy brown pigment granules
E. Giant melanosomes

688
►C

Perls stain stains hemosiderin (iron) blue. Fontana Masson stains melanin black. Minocycline
hyperpigmentation often shows positive staining for both iron and melanin.

517- Leukokeratosis nicotina palati:

A. Is not attributed to smoking


B. Represents a premalignant condition
C. Responds to intralesional steroids
D. Represents inflamed minor salivary glands
E. Resolves with tetracycline therapy

►D

Leukokeratosis nicotina palati, also called smoker�s palate, is a complication of tobacco use,
especially pipe smokers. Lesions develop as a result of inflamed minor salivary glands and manifest
as uniform keratosis of the hard palate with multiple red, umbilicated papules.

518- Mutations in which of the following genes has been implicated in the pathogenesis of
disseminated superficial actinic porokeratosis?

A. BRAF
B. K-RAS
C. PTCH
D. PTEN
E. SART3

►E

Disseminated superficial actinic porokeratosis (DSAP) is a disorder characterized by numerous


keratotic macules and papules often localized in a photodistribution. Clinically, they are
distinguished by a rim of keratotic scale and often may have an atropic appearance centrally.
Histologically, they often display a lichenoid infiltrate flanked by characteristic cornoid lamella

689
(inward-bending tiers of parakeratosis with underlying hypogranulosis and dyskeratosis). Recently,
mutations in SART3 (squamous cell carcinoma antigen recognized by T -cells 3) have been
implicated in a Taiwainese patient cohort affected by DSAP. Of note, while all varieties of
porokeratoses have the potential for malignant degeneration and development of squamous cell
carcinoma, lesions of DSAP have the lowest risk.

519- Patients with xanthoma disseminaturn may demonstrate:

A. Elevated serum beta lipoproteins


B. Sparing of the oral mucosa
C. Involvement of the pituitary gland
D. Sparing of the flexural areas
E. Increased risk of malignant degeneration

►C

Xanthoma disseminatum is a rare mucocutaneous disease of discrete yellowish-red to brown


papules in the axillary and inguinal folds. Patient are generally normolipidemic. Involvement of the
pituitary gland may occur, leading to diabetes insipidus.

520- The "flag sign" is seen in which of the following conditions?

A. psoriasis
B. sarcoidosis
C. hidradenitis suppurativa
D. kwashiorkor
E. telogen effluvium

►D

The "flag sign" is a striking physical finding most readily seen in long and dark hair. Hair that has
grown during periods of inadequate nutrition is pale so alternating bands of light and dark can be
seen along a single strand (reflecting periods of adequate and inadequate nutrition). The "flag sign"
can be seen with other nutritional disorders as well.

690
521 - Pilomatricomas has been associated with which of the following conditions?

A. Acute myelogenous leukemia


B. Multiple sclerosis
C. Crohns Disease
D. Myotonic dystrophy
E. Pneumothorax

►D

Answer D. Pilomatricomas are benign tumors that are derived from hair matrix cells. This tumor
presents more frequently in children as a solitary asymptomatic purple red papule or nodule usually
on the head and neck. Mutations in beta catenin is associated with the development of these tumors.
Multiple pilomatricomas may be a marker for the development of myotonic dystrophy. This rare
disorder is AD, and is associated with difficulty relaxing muscles after contraction. In a review of
patients with this disorder, patients tended to have mulitple pilomatricomas. Pilomatricomas can
also be found with Turner syndrome, Rubinstein-Taybi, and Churg Strauss syndromes.

522 -Spindle cell lipoma is commonly found on the:


A. Head
B. Lower extremities
C. Buttocks
D. Breast
E. Posterior shoulder

►E

Spindle cell lipoma is a solitary benign tumor seen in adult male patients, and is most often located
on the posterior shoulder and neck regions.The tumor histologically consists of mature collagen,
adpose tissue, spindle cells, and mast cells. Treatment is with local excision.

523- What name is commonly used to describe this condition of nodular elastosis with cysts and
comedones?

691
A. Actinic granuloma
B. Chlorance
C. Cutis rhomboidalis nuchae
D. Colloid milium
E. Favre-Racouchot

►E

Favre-Racouchot syndrome is a condition that generally affects elderly white males with history of
chronic sun exposure and smoking. It is characterized by multiple open comedones on the temples
and forehead with a background of actinincally damaged skin.

524- Patients with chronic idiopathic urticaria should avoid:

A. Latex products
B. Alcohol
C. Nickel sulfate
D. �-blockers
E. aspirin

►E

Chronic idiopathic urticaria is defined by the presence of urticaria of unknown etiology lasting
greater than 6 weeks. Patient with chronic idiopathic urticaria should avoid aspirin as it aggravates
urticaria in about 30% of patients.

692
525- Which statement best describes the mechanism of action of spironolactone?
A. Non-steroid inhibitor of DHT binding to the androgen receptor
B. Progestin
C. GnRH agonist
D. Mineralocorticoid
E. Steroid inhibitor of androgen receptor and androgen biosynthesis

►E

Spironolactone is a steroid molecule. It acts as an anti-androgen by blocking the androgen receptor


and inhibiting androgen synthesis.

526- A 35 year old man with plaque psoriasis well-controlled with twice daily application of
calcipotriene 0.005% cream noticed a flare of his psoriasis shortly after he started using a
prescription moisturizer lotion for psoriasis. Which is the most likely explanation for the apparent
decreased efficacy of calcipotriene?

A. The moisturizer impaired penetration of the calcipotriene


B. The moisturizer diluted the calcipotriene
C. The two events are unrelated
D. The prescription moisturizer likely contains lactic acid.
E. The decreased efficacy is due to tachyphylaxis

►D

The active ingredient in Dovonex is easily inactivated, particularly by acidic compounds like
salicylic acid and lactic acid.

527- All of the following are consider intermediate filament except:

A. Microtubules
B. Keratins
C. Vimentin
D. Desmin

693
E. Peripherin

►A

Intermediate filaments are composed of keratins, vimentin, desmin, peripherin, neurofilaments,


nuclear laminins, and nestin. These are part of cytoskeletal elements.

528- An atypical finding in patients with Lofgren syndrome is?

A. Fever
B. Arthritis
C. Subcutaneous Tender Nodules
D. Acute Iritis
E. Beaded papules on nasal rim

►E

Lofgren syndrome is a form of acute, self resolving, sarcoidosis manifested with hilar adenopathy,
fever, migrating polyarthritis, acute iritis, and erythema nodosum. In general these patients do not
have other manifestations of cutaneous sarcoidosis. Papules along the nasal rim are suggestive of
Lupus Pernio sarcoidosis and suggest a worse prognosis with bronchial involvement.

529- A patient has erythema, vesicles, and erosions in periorificial, and acral flexural distribution.
Adult onset diabetes, glucose intolerance, and weight loss is associated with these symptoms. The
% of patients that have metastatic disease by diagnosis is:

A. 10%
B. 25%
C. 50%
D. 75%
E. 90%

►D

694
Patients that are diagnosed with necrolytic migratory erythema have a 75% chance of having
metastatic disease by the time of diagnosis. Lesions have a circinate pattern due to peripheral
spread. Patient experience adult onset diabetes, glucose intolerance and weight loss.

530- A 9-year-old boy presents with calcinosis cutis, periorbital violaceous erythema, and
periungual telangiectasias. Which substance is most likely to be elevated in this patient?

A. Fodrin
B. Fibrillarin
C. Thrombospondin-1
D. Complement
E. Triglycerides

►C

This patient has juvenile dermatomyositis which can feature a heliotrope rash, periungual
telangiectasias, psoriasiform dermatitis, calcinosis and lipodystrophy. Thrombospondin-1 is a
mediator of angiogeneis that is increased in patients with juvenile dermatomyositis. Systemic
sclerosis can demonstrate antibodies to fibrillarin.

532- This is used to describe the blanching seen around psoriatic plaques secondary to decrease in
prostagladin, PGE2:

A. Woronoff Ring
B. Auspitz sign
C. Koebnerization
D. Silver sign
E. Ashoe sign

►A

Woronoff ring is an area of blanching around psoriatic plaques secondary to decrease in


prostagladin, PGE2. Koebnerization is seen in 20% of patients. Auspitz sign is bleeding upon
removal of scale.

695
533- What type of amyloid is deposited into the skin of this pruritic disorder?

A. Amyloid AA
B. Amyloid AL
C. Keratin derived
D. Beta-2 microglobulin
E. Transthyretin

►C

Lichen amyloidosis is a pruritic eruption that often occurs in areas of chronic rubbing. Clinically, it
has a rippled appearance. Keratin derived amyloid is the type deposited in the skin.

534- The most common associated malignancy with dermatomyositis is:

A. Colon carcinoma
B. Hepatocellular carcinoma
C. Renal cell carcinoma
D. Esophageal carcinoma
E. Ovarian carcinoma

►E

Patients with dermatomyositis (DM) may have cancer that precedes, occurs simultaneously as, or
follows the diagnosis of DM. Ovarian cancer is overrepresented in this population.

535- What is the most common cause of death in malignant atrophic papulosis Degos disease?

A. Cutaneous infection
B. Cerebrovascular accident
C. Myocardial ischemia
D. Bowel infarction
E. Pulmonary hemorrhage

696
►D

Degos disease is an obliterative arteritis clinically presenting with pink-red papules that later
develop atrophic, porcelain-white centers. Death is usually secondary to Infarction of the bowel and
perforation. Less commonly, death results from cerebral infarction.

536- A patient presents requesting removal of a tattoo. Examination reveals a tattoo with red,
orange, yellow, and purple pigment. Which laser would be appropriate to treat this tattoo?

A. Q switched Nd:YAG (532nm)


B. Q switched alexandrite (755nm)
C. Q switched ruby (694nm)
D. Nd:Yag (1064nm)
E. Alexandrite (755nm)

►A

The Q switched Nd:YAG can also be used to treat tan pigment. QS alexandrite and QS ruby can
remove green pigment. QS Nd:YAG is a good choice for patients with darker skin color.

537- A patient has a rapid onset of hyperpigmented velvety plaques involving the intertriginous
areas. The most likely associated cancer with this disease is:

A. Gastric carcinoma
B. Lung carcinoma
C. Breast carcinoma
D. Lymphoma
E. Mycosis fungoides

►A

A patient with acanthosis nigricans can also have florid oral papillomatosis. The most commonly
seen cancer is gastric carcinoma. Other cancers associated are lung, br east, uterine, ovarian,
lymphoma and mycosis fungoides. It is associated with an increase in growth hormone.

697
538- Which of the following is associated with dermatitis herpetiformis?

A. Gluten-sensitive enteropathy
B. Inflammatory bowel disease
C. Autoimmune hepatitis
D. Herpes labialis
E. Rheumatoid arthritis

►A

Gluten-sensitive enteropathy or Celiac disease is demonstrated on small bowel biopsy of all patients
with dermatitis herpetiformis. However, most of these patients do not demonstrate symptoms of
gastrointestinal disease. Dermatitis herpetiformis (DH) is not associated with inflammatory bowel
disease, autoimmune hepatitis, herpes labialis, or rheumatoid arthritis.

539- Which of the following disorders is primarily associated with a monoclonal gammopathy of
the IgA type?

A. Scleromyxedema
B. Scleredema
C. Necrobiotic xanthogranuloma
D. Schnitzler's syndrome
E. Pyoderma gangrenosusm

►E

Pyoderma gangrenosum is an destructive, inflammatory disease of the skin characterized by a


painful nodule or pustule that later forms a progressively enlarging ulcer. On pathology, there is a
neutrophilic infiltrate with leukocytoclasia. Diseases that have been associated with pyoderma
gangrenosum include inflammatory bowel disease, arthritis, an IgA monoclonal gammopathy
(primarily), and myelodysplasia.

540- Melanocytes can be found in all of the following except:

698
A. Nevus depigmentosa
B. Tyrosinase positive albinism
C. Nevus anemicus
D. Vitiligo
E. Postinflammatory hypopigmentation

►D

Vitiligo is an acquired disease in which there is total loss of pigment. The central process in vitiligo
is the destruction of melanocytes. With silver stains or dopa reaction, well established lesions of
vitiligo are completely devoid of melanocytes.

541- A 28 year old previously healthy man presents with thick crusting/hyperkeratosis of the palms
and soles, urethritis, and one month of peripheral arthritis. What of the following is true regarding
his condition

A. Females and males are equally affected


B. A chronic deforming arthritis occurs in 20%
C. TNF-alpha inhibiting agents have no role in the treatment of this condition
D. Patients must have urethritis, conjunctivitis, and arthritis for diagnosis
E. Clamydia cervicitis is not associated with this condition

►B

The patient has Reiter's syndrome. Reiter's syndrome is a chronic in flammatory disease similar to
psoriasis with psoriatic arthritis, and is though to be a variant form. The classic triad includes
urethritis, conjunctivitis, and arthritis. Few patients present with the classic triad, and thus the
syndrome can be diagnosed with peripheral arthritis for greater then one month duration and rarely
occurs in women. Skin findings include keratoderma blennorrhagicum and circinate balanitis (in
men), as well as oral erosions, sever stomatitis, and nail changes. Chronic deforming
arthritis occurs in 20% of patients. Treatments include topical steroids, NSAIDs, methotrexate,
acitretin, cyclosporin, and TNF-inhibiting biologics such as etanercept.

699
542- A 48 year old African American woman presents with 1-3 cm red-brown plaques on the
extremities, fevers, bilateral knee pain, and tender nodules on bilateral shins. What is her
presentation most consistent with?

A. Lofgren‘s syndrome
B. Rheumatoid arthritis
C. Grave‘s disease
D. Psoriatic arthritis
E. Gout

►A

This patient has Lofgren‘s syndrome, a variant of sarcoidosis, with erythema nodosum, hilar
adenopathy, fever, and arthritis. Rheumatoid arthritis, psoriatic arthritis, and gout result in joint
pain, but are not consistent with this patient‘s presentation.

543- This syndrome has characteristics of fever, uveitis, parotid enlargement, and facial nerve palsy:

A. Heerfordt syndrome
B. Lofgren syndrome
C. Sjogrens syndrome
D. Richer Hanjart syndrome
E. Wilson's syndrome

►A

Heerfordt syndrome is also known as uveoparotid fever. It is characterized by fever, uveitis, which
may precede parotid enlargement and facial nerve palsy.

544- Juvenile pityriasis rubra pilaris accounts for 10% of all cases occurs between 5 and 10 years
old. It often follows an acute infection. This is the cardinal feature of juvenile pityriasis rubra
pilaris:

A. Palmoplantar hyperkeratosis

700
B. Erythroderma
C. Hyperkeratotic flexures
D. Ichthyosis
E. Plantar dermatosis

►A

Palmoplantar hyperkeratosis is a cardinal feature of juvenile pityriasis rubra pilaris. It occurs in the
age of 5-10 years of age and can follow after an infection.

545- A patient is diagnosed with carcinoid syndrome with facial flushing, diarrhea and intermittent
bronchospasm. The urine test is:

A. 5-hydroxyindole-acetic acid
B. Gastric acid
C. Amylase
D. Lipase
E. Serotonin

►A

Patients with carcinoid syndrome will develop telangiectasia, pellagra like or sclerodermoid like
eruption. The neoplasm originated in the GI tract and 5-hydroxyindole-acetic acid (5-HIAA) levels
would be elevated. Surgical treatment is the therapy.

546- Which of the following forms of angioedema is mediated by kinin release?

A. Hereditary angioedema
B. Acute allergic angioedema
C. Angioedema associated with urticarial vasculitis
D. NSAID angioedema
E. Infectious angioedema

►A

701
Types of angioedema mediated through kinin release include hereditary angioedema, acquired C1
inhibitor deficiency, and ACE inhibitor associated angioedema. These conditions are not associated
with concurrent urticaria.

547- Which of the following is correct about eosinophilic folliculitis?

A. painful
B. only seen in adults
C. classified as an AIDS-defining illness
D. more common in females
E. associated with P. acnes infection of hair follicles

►C

Eosinophilic pustular folliculitis is a non-infectious eosinophilic infiltrate of the hair follicle. It is


characterized by intense pruritus. The 3 variants of eosinophilic folliculitis include classic
eosinophilic pustular folliculitis, immunosuppression-associated (mostly HIV-related), and infancy-
associated eosinophilic folliculitis. The classic type (Ofuji′s disease) is more common in Japanese
men. The male-to-female ratio of eosinophilic folliculitis is 5:1. It is associated with
immunosuppresion and has been classified as an AIDS defining illness. Eosinophilic pustular
folliculitis should be viewed as a possible cutaneous sign of immunosuppression.

548- Which of the following medications would likely exacerbate your patient's psoriasis?

A. Hydrochlorothiazide
B. Lisinopril
C. Amlodipine
D. Metroprolol
E. Diltiazem

►D

702
Several drugs have been incriminated as inducers of psoriasis, in particular and most notably
lithium, beta-blockers, antimalarials, and interferon. More recent additions include terbinafine,
calcium channel blockers, captopril, glyburide, and lipid-lowering drugs such as gemfibrozil

549- A newborn is exposed to the cold and a lacy reticulated pattern is revealed. The most common
abnormality with cutis marmorata telangiectatica congenita is:

A. Limb atrophy
B. Limb hypertrophy
C. Bone resorption
D. AV malformation
E. Leg shortening

►A

Cutis marmorata telangietatica is seen in newborn infants when exposed to cold weather with a
reticulated pattern. It represents a vasomotor response and decreases with age.

550- A 45-year-old woman develops bilateral poikilodermatous patches without atrophy on her
lateral thighs. This is most likely a manifestation of what disease?

A. Lupus Erythematosus
B. Dermatomyositis
C. Mycosis Fungoides
D. Hepatitis C
E. Diabetes Mellitus

►B

Poikilodermatous changes on the lateral thigh represent the "holster sign"ۥ seen in
dermatomyositis. The poikilodermatous patches seen in dermatomyositis are often symmetric
macular violaceous erythemas with dyspigmentation. Other characteristic poikilodermatous
manifestations of dermatomyositis include the "V-sign" on the anterior neck and upper chest and the

703
"shawl sign"€™ on the posterior neck, upper back, and the posterior shoulders. Lesions of
Mycosis Fungoides typically have some component of epidermal atrophy.

551- You prescribe doxycyline to a 22 year-old woman with acne. Your patient takes an oral
contraceptive to prevent pregnancy. Your patient read in a magazine that the doxycycline may
decrease the efficacy of her contraceptive, and asks you about this. Which of the following
antibiotics has been definitively shown to reduce contraceptive efficacy?

A. Tetracycline
B. Minocycline
C. Azithromycin
D. TMP-SMX
E. Rifampin

►E

Theoretically, decreased enterohepatic absorption of hormones due to altered gut flora could
decrease contraceptive efficacy. However, this theory has not been borne out in studies. Only
rifampin, which is a potent hepatic microenzyme inducer, has been definitively shown to reduce
contraceptive efficacy.

552- What mast cell mediators are preformed, rather than synthesized upon exposure to a trigger?

A. tryptase
B. leukotriene C4
C. thromboxane
D. prostaglandin D2
E. platelet activating factor

►A

Mast cell mediators can be grouped into two classes: preformed and newly formed. Preformed
mediators include tryptase, histamine, serotonin, and heparin. Newly formed mediators include
prostaglandin D2, leukotriene C4, and platelet activating factor.

704
553- Behcet's disease is associated with which HLA type?

A. HLA-B51
B. HLA-B17
C. HLA-B13
D. HLA-DR4
E. HLA-Cw6

►A

Behcet's disease is associated with HLA-B51. HLA-Cw6 is the HLA type most closely associated
with psoriasis. HLA-B13 and HLA-B17 are also both associated with psoriasis; HLAB17 is
assiated with earlier onset and more serious psoriasis.

554- A patient presents with recurrent crops of papules that ulcerate and then spontaneously heal.
What immunohistochemical stain would be helpful in making the diagnosis?

A. CD 4
B. CD 20
C. CD 30
D. CD 56
E. CD 68

►C

Lymphomatoid papulosis Lymphomatoid papulosis (LyP; Macaulay disease) is a chronic


lymphoproliferative disease of the skin characterized by recurrent crops of papules that may
ulcerate. The papules heal spontaneously over a period of 1-2 months and may leave a depressed
scars. The T-cells in this disorder typically stain positively for CD 30.

555- The causative organism for mucocutaneous leishmaniasis is:

A. Leishmania aethiopica
B. Leishmania infantum

705
C. Leishmania brasiliensis
D. Leishmania tropica
E. Leishmania major

►C

Mucocutaneous disease is most commonly caused by New World species, although Old World L
aethiopica has been reported to cause this syndrome. Infection by Leishmania viannia
braziliensis may lead to mucosal involvement in up to 10% of infections depending on the region
in which it was acquired. Initial infection is characterized by a persistent cutaneous lesion that
eventually heals, although as many as 30% of patients report no prior evidence of leishmaniasis.
Several years later, oral and respiratory mucosal involvement occurs, causing inflammation and
mutilation of the nose, mouth, oropharynx, and trachea.

556- Histologic examination of this chronic pruritic plaque on the back, which of the following s
tains would NOT be positive?

A. Von Kossa
B. Cotton dyes
C. Crystal violet
D. PAS
E. Thioflavin T

►A

706
Macular amyloid is a form of keratin derived amyloid, which is typically located between the
shoulder blades. Many stains are used to identify amyloid in the skin including Congo red, cotton
dyes, crystal violet, PAS and thioflavin T.

557- What is the best muscle to biopsy in dermatomyositis?

A. triceps
B. biceps
C. quadriceps
D. gluteus maximus
E. deltoid

►A

The triceps muscle is involved early in the disease; therefore, it has the highest yield in a muscle
biopsy looking for the changes seen in dermatomyositis. Surgeons have traditionally biopsied the
deltoid mulscle, but it is not involved until late in the disease.

558- What systemic condition is often associated with this disease?

A. Diabetes mellitus
B. Tobacco use
C. Alopecia areata
D. Coronary artery disease
E. Hepatitis C

►E

Oral lichen planus is often found in patients with hepatitis C. The association between cutaneous
lichen planus and hepatitis C is not clear

707
559- A 62 year-old man with chronic renal failure on hemodialysis presents with carpal tunnel
syndrome, bone cysts, and spondyloarthropathy. A diagnosis of amyloidosis is suspected. Which of
the following is true regarding his most likely diagnosis?

A. AA amyloid is likely causative


B. AL amyloid is likely causative
C. beta 2-microglobulin is likely causative
D. keratin-derived amyloid is likely causative
E. Amyloid P component will not be found in affected tissues

►C

The patient likely has dialysis-related amyloidosis. In this condition, beta 2-microglobulin is the
protein component altered by uremia. Carpal tunnel syndrome, bone cysts, and spondyloarthropathy
commonly result. Amyloid P component and ground substance are found in all forms of
amyloidosis; the protein-derived amyloid fibers differ among the various forms.

560- A teenager comes into your office requesting laser hair removal of her significant facial hair.
Before you agree to treat her, you order which of the following laboratory analyses?

A. FSH and LH
B. Glucose and hemoglobin A1C
C. Ferritin and TIBC
D. Testosterone and DHEA-S
E. TSH and T3

►D

Hisrutism describes excessive terminal hair growth in areas on women that are normally found only
in post-pubescent males (such as beard, chest, inner thigh). Hypertrichosis refers to excessive hair
density or length. Hirsutism is under the influence of androgen stimulation. Testosterone and
DHEA-S can be used to detect excessive adrenal or ovarian androgen production. Other clinical
manifestations of androgen excess in women are acne and virilization.

708
561- Presence of which of the following autoantibodies is diagnostic of SLE and not reported in
patients with other connective tissue diseases?

A. anti-U1RNP
B. anti-dsDNA
C. anti-Ro
D. anti-La
E. anti-Sm

►E

Anti-Sm is diagnostic of SLE and not reported in patients with other connective tissue diseases. It is
found in 15-40% of patients with SLE. Most patients with anti-Sm also have antibodies to U1RNP,
but the converse is not true. anti-U1RNP is found in 100% of patients with MCTD and in 30% of
patients with SLE (the majority of patients with positive U1RNP have SLE rather than MCTD).
Anti-dsDNA correlates with renal involvement in SLE. Anti-Ro and anti-La antibodies are found in
LE and Sjogren's, and strongly associated with photosensitivity.

562- Which of the following disorders is most strongly associated with a monoclonal gammopathy
of the IgG-lambda type?

A. Scleredema
B. POEMS syndrome
C. Necrobiotic xanthogranuloma
D. EED
E. Scleromyxedema

►E

All of the above conditions are assocaited with some type of monoclonal gammopathy.
Scleromyxedema is associated with an IgG-lambda monoclonal gammopathy. Scleredema and
NXG are most often associated with IgG-kappa, and both POEMS syndrome and EED are usually
associated with IgA monoclonal gammopathies.

709
563- A young woman presents with tender, erythematous nodules an the anterior lower extremities.
Which of the following would not be an appropriate test to consider?

A. TSH
B. ESR
C. ASO
D. Fungal cultures
E. PPD

►A

Erythema nodosum can be triggered b y several medical conditions, including drugs, IBD,
infections (strep, hepatitis, TB, fungus), sarcoidosis, and malginancy. Thyroid disease is not a
known trigger for erythema nodosum

564- A concerned 23 year old male presents to clinic with multiple verrucoid lesions on his penis.
He is anxious to have them removed. You explain that these are commonly treated cryosurgically.
What is the boiling point of liquid nitrogen in Celsius?

A. -89.5
B. -40.8
C. 3.8
D. -195.8
E. -78.5

►D

The boiling point of liquid nitrogen is -195.8. Nitrous oxide is -89.5. Carbon dioxide is -78.5.
Dichlorotrifluoroethane (Freon 114) is 3.8. Chlorodifluoromethane (Freon 22) is 40.8. Colder
cryogen is needed for adequate deep destruction, making liquid nitrogen the only reliable agent of
the five listed. The Freons, solid CO2 and nitrous oxide are used only for topical anesthesia and
superficial destruction.

710
565- Syndrome that is associated with disease shown in image is known as

A. SAPHO syndrome
B. Follicular occlusion tetrad syndrome
C. LEOPARD syndrome
D. NAME syndrome
E. PAPA syndrome

►E

PAPA syndrome ( Pyogenic Arthritis, Pyoderma gangerosum and Acne) The image shows classic
ulcerative pyoderma gangrenosum with undermined violaceous gray border. Reepithelialization
occurs from the margins and the ulcer heal usually with atrophic cribriform pigmented scar.
Although the classic morphologic clinical presentation of pyoderma gangrenosum is an ulceration,
there are several variants (bullous, pustular, and superficial granulomatous) which differ by their
clinical presentation, location, and associated diseases.

566- Increased LDL and VLDL is seen in which type of hyperlipoproteinemia?

A. Type IIb
B. Type I
C. Type III
D. Type IV
E. Type V

►A

Type IIb hyperlipoproteinemia is autosomal dominant and features increased LDL and VLDL.
Associated clinical findings include xanthelasma, planar, tendinous and tuberous xanthomas. Type I
hyperlipoproteinemia is autosomal recessive and manifests with increased chylomicrons. Clinically,
patients develop eruptive xanthomas and lipemia retinalis. Type IIa hyperlipoproteinemia is
autosomal dominant. Patients have increased LDL and may develop tendinous, tuberous, planar or
eruptive xanthomas in addition to xanthelasma and arcus juvenilis. Type III hyperlipoproteinemia is
autosomal recessive and features increased IDL. Clinically, these patients are prone to develop
palmar, planar, tendinous, tuberous and eruptive xanthomas. Type IV hyperlipoproteinemia is
autosomal dominant and manifests with increased VLDL. Patients develop eruptive, tendinous and

711
tuberous xanthomas. Type V hyperlipoproteinemia is autosomal dominant. Patients have increased
chylomicrons and VLDL and may develop eruptive xanthomas and lipemia retinalis.

567 -The most common finding in patients with systemic sclerosis is:

A. Migratory polyarthritis
B. Esophageal dysfunction
C. Pulmonary fibrosis
D. Renal disease
E. Cardiac conduction defects

►B

Esophageal dysfunction is the most common systemic finding in systemic sclerosis. A migratory
polyarthritis is the first manifestation of disease in many. Pulmonary fibrosis, renal disease and
cardiac conduction defects are all manifestations but not the most common.

568- A pregnant woman presents with mild inflammatory acne which is very bothersome to her.
Which of the following is the most appropriate treatment option based on FDA classifications of
medication in pregnancy?

A. Topical erythromycin/benzoyl peroxide gel


B. Topical tretinoin
C. Tazarotene 0.1% cream
D. Azelaic acid 20% cream
E. Bactrim

►D

Azeleic acid is category B in pregnancy. The others are category C (benzoyl peroxide, topical
tretinoin, and bactrim) or category X (tazarotene). The categories for safety of drugs in pregnancies
are as follows: Category A: controlled studies in humans show no risk to fetus. Category B:
controlled human studies show no risk to fetus but may show risk to animals, or no risk in animal
studies but no human studies conducted. Category C: risk to human fetus cannot be ruled out,

712
studies are lacking; animal studies equivocal. Category D: controlled studies show risk to human
fetus, benefits may sometimes outweigh risk. Cateogory X: contraindicated in pregnancy.

569- Child abuse is often incorrectly suspected when a young girl presents with which cutaneous
finding?

A. Molluscum contagiosum
B. Lichen sclerosus et atrophicus
C. Condyloma accuminata
D. Ecchymoses on face and hands
E. Genital herpes simplex

►B

Lichen sclerosus et atrophicus is an inflammatory disease of the skin that most often occurs oin the
genital region. Clinically, lichen sclerosus may appear as a hypopigmented, well demarcated patch
with a crinkled appearance. It is often pruritic and if left untreated may result in progressive
scarring. The treatment of choice is ultra potent topical steroids.

570- The majority of patients with positive U1RNP autoantibodies have which diagnosis?

A. Mixed connective tissue disorder (MCTD)


B. Rheumatoid arthritis
C. Systemic lupus erythematosus
D. Sjogren's syndrome
E. Dermatomyositis

►C

Anti-U1RNP is found in 100% of patients with MCTD and in 30% of patients with SLE; patients
with SLE and anti-U1RNP also have other positive serologies. The majority of patients with
positive U1RNP have SLE rather than MCTD. The presence of U1RNP autoantibodies is associated
with sclerodactyly, Raynaud's, and esophageal dysmotility.

713
571- 412 A 69 year old man with a history of multiple myeloma develops a deposition disorder of
his heart, tongue, GI tract, and skin. What form of amyloid is most likely present?

A. AL
B. AA
C. Beta 2-microglobulin
D. Keratin
E. Colloid

►A

Primary systemic amyloidosis involves the deposition of protein AL, which is derived from Ig light
chains. AL is also found in nodular amyloidosis produced by a plasmacytoma. 40% of patients with
primary systemic amyloidosis have skin involvement.

572- A premature infant who is being weaned off breast milk develops vesicobullous and
eczematous skin lesions and diarrhea. Which of the following is not another classic precipitant for
this condition

A. Parenteral nutrition
B. Stress (i.e. infection)
C. Diets with mainly cereal grains
D. Liver disease
E. Alcoholism

►D

Zinc deficiency can be seen in premature or term infants being weaned off breast milk, which is
usually high in zinc content, as well as in parenteral nutrition use, alcoholism because of poor
nutritional intake, malabsorption, IBD, diets high in grains containing phytate which binds zinc, and
metabolic stress

714
573- Necrobiosis is characteristic of which two entities below?

A. Sarcoid and granuloma annulare


B. Sarcoid and necrobiosis lipoidica
C. Sarcoid and annular elastolytic giant cell granuloma
D. Granuloma annulare and necrobiosis lipoidica
E. Granuloma annulare and cutaneous Crohn's disease

►D

Necrobiosis is seen in GA and NLD as well as rheumatoid nodules, interstitial granulomatous


dermatitis and palisading neutrophilic and granulomatous dermatitis. Mucin is also present in GA.
NLD shows giant cells, extracellular lipid, and vascular changes.

574- Which of the following medications is not associated with exacerbating this condition?

A. Lithium
B. Prednisone
C. Phenytoin
D. Isoniazid
E. Propranolol

►E

Acne or acneiform erruptions can be caused or exacerbated by corticosteroids, oral contraceptives,


androgens, ACTH, lithium, phenytoin, INH, and haloperidol

715
575- Which medication used to treat pruritus works by depleting substance P from nerve fibers?

A. Capsaicin
B. Doxepin
C. Diphenhydramine
D. Pramoxine
E. Gabapentin

►A

Capsaicin is an antipruritic and analgesic that works by depleting substance P. Repeated use leads to
decreased heat, pain, and itch sensations. Doxepin is a topical tricyclic antidepressant.
Diphenhydramine is an antihistamine. Pramoxine is a topical anesthetic. Gabapentin, commonly
used to treat notalgia paresthetica, has an unknown mechanism of action.

576- Drug induced psoriasis can be from steroid withdrawal. Other medications known to cause
drug induced psoriasis are all of the following except:

A. Lithium
B. Beta-blockers
C. Interferons
D. ACE inhibitors
E. Hydrocholorthiazide

►E

Lithium, beta-blockers, interferons, ACE inhibitors, and granulocyte-colony stimulating factors


have all been known to cause exacerbation of drug induced psoriasis.

577- Which of the following is true regarding the components of amyloid?

A. Ground substance is present only in secondary cutaneous or tumor-associated


amyloidosis

716
B. The amyloid P component is present in all forms
C. Protein AA is present in primary systemic amyloidosis
D. The amyloid present in secondary systemic amyloid does not lose its birefringence after
treatment with potassium permanganate
E. Protein AL loses its biregringence after treatment with potassium permanganate

►B

All forms of amyloidosis contain amyloid P component and ground substance. The proteinderived
amyloid fibers differ among various forms of amyloid. Protein AL is found in primary systemic
amyloidosis. Protein AA is found in secondary systemic amyloidosis. Secondary systemic amyloid
(AA) loses its birefringence after treatment with potassium permanganate, but amyloid in primary
and localized forms do not.

578- Which of the following receptor is the most important mediator of retinoid activity in the skin?

A. RAR-gamma
B. RXR-alpha
C. RXR-gamma
D. RAR-beta
E. RXR-beta

►A

Retinoid activity is mediated by retinoid receptors. Two groups exist: RA receptors (RAR) and RX
receptors (RXR). Each has three receptor subtypes: alpha, beta, gamma. RAR-gamma is the most
important mediator of retinoid activity in the skin.

579 -Cutaneous manifestations of vitamin D deficiency include:

A. Alopecia
B. Follicular hyperkeratosis
C. Edema
D. Angular cheilitis

717
E. Atrophic glossitis

►A

Alopecia is the only cutaneous manifestation of vitamin D deficiency. Edema is the only cutaneous
manifestation of vitamine B1 (thiamine) deficiency (Beriberi).Follicular hyperkeratosis is seen in
hypervitaminosis A. Angular cheilitis is seen in vitamin B2 (riboflavin), B6 (pyridoxine), folic acid,
niacin (vitamin B3 or nicotinic acid), biotin, zinc, and iron deficiency. Atrophic glossitis is observed
in vitamin B2 (riboflavin), vitamin B6 (pyridoxine), vitamin B12, folic acid, and iron deficiency
(among others).

580 -A 35 year-old woman presents with predominantly lower facial inflammatory acne, hirsutism,
and irregular menses. She has failed multiple conventional treatments. Laboratory work-up
reveals a very high level of DHEA-S. You are most concerned about:

A. Congenital adrenal hyperplasia


B. Adrenal tumor
C. Polycystic ovary syndrome
D. Toxicity from prolonged use of spironolactone
E. Progestin excess from oral contraceptives

►B

A patient whose acne fails to respond to conventional therapy, whose acne flares cyclically, with
hirsutism, alopecia, or irregular menses warrants an endocrine work-up, including free and total
testosterone, LH, FSH, and DHEA-S. Although rare, very high levels of DHEA-S may suggest
an adrenal adrogen-secreting tumor.

581 -A patient has allergic contact dermatitis to paraphenylenediamine. Which of the following
allergens may show a potential cross- reaction?

A. Adhesive
B. Sulfa drugs
C. Lidocaine

718
D. Fragrance
E. Nickel

►B

Paraphenylenediamine (PPD) may cross react with a number of substances, including azo dyes,
benzocaine, procaine, sulfa drugs, and para-aminobenzoic acid (PABA). PPD is often found in
dark hair dyes and in modified henna tattoos. Fragrance, nickel, lidocaine, and adhesive do not
show cross-reaction with PPD.

582- A 25 y/o male is currently being treated with Dupilumab. Which of the following is likely
elevated at baseline in this patient?

A. IL-2
B. IL-12
C. TNF-b
D. IL-10
E. IFN-y

Correct choice: D. IL-10

Explanation: This patient is presenting with atopic dermatitis which is a Th2 mediated disease.
IL-10, along with IL-4, 5, and 13, are TH-2 cytokines. Dupilumab is a monoclonal antibody that
blocks the IL-4R alpha subunit, which lowers IL-4 and IL-13, and is approved for moderate to
severe atopic dermatitis.
IL-2, IL-12, TNF-b, and IFN-y are all associated with a TH1 response.
Reference: Odom RB. Andrews’ Diseases of the Skin, 9th ed. Philadelphia: WB Saunders; 2000.

583- Reactivation of which of the following virus(es) can lead to drug hypersensitivity?

A. HHV-6
B. HHV-7
C. EBV
D. CMV

719
E. All of the above

Correct choice: E. All of the above

Explanation: Reactivation of latent viruses such as HHV-6, HHV-7, EBV, and CMV can increase
the risk of drug hypersensitivity. Drug hypersensitivity syndrome is characterized by fever, skin
eruption and internal organ involvement.

584- Which of the following molecules have been shown to play a role in the development of TEN/
SJS severe drug reactions?

A. FAS
B. Granulysin
C. Perforin
D. Granzyme B
E. All of the above

Correct choice: E. All of the above

Explanation: All of the molecules have been implicated in playing an important pathogenic role in
development of SJS/TEN.

720
721
Derma pa ology

1. This neoplasm derives from which structure?

A-Apocrine gland
B-Hair follicle
C-Eccrine duct
D-Subcutaneous fat
E-Muscle

Correct choice: A. Apocrine gland

Explanation: Adenoid cystic carcinoma is considered a malignant apocrine tumor. It appears most
commonly in the oral cavity and has a very unpredictable course.

2. If positive, which immunohistochemical stain supports further evaluation with colonoscopy and
cystoscopy?

722
to
th
A. CK20

B. Congo Red

C. S100

D. CD34

E. Neuron specific enolase

Correct choice: A. CK20

Explanation: Extramammary Paget’s Disease is an intraepithelial adenocarcinoma of apocrine


glands that presents with a well-demarcated erythematous plaque in the genital area. Histologically,
pagetoid cells stain positive for CK7, unlike pagetoid melanoma cells which stain positive for S100.
CK20 staining has been associated with secondary EMP in the setting of underlying gastrointestinal
or genitourinary carcinoma.

3. A 60-year-old man presents with a poorly demarcated, symmetric, very painful patches of
erythema and retiform purpura favoring the buttocks. Bullae and a dusky gray discoloration
developed and ulcerations appeared with a black, leathery eschar. Upon palpation there is a
surrounding subcutaneous induration that extends beyond the margins of the visibly active lesions.
You perform a skin biopsy (see image). Which of the following is the most likely diagnosis?

A. Calciphylaxis

B. Cryoglobulinemia

C. Protein C deficiency

D. Warfarin necrosis

E. Subcutaneous fat necrosis

723
Correct choice: A. Calciphylaxis

Explanation: calciphylaxis is characterized by intimal fibrosis and medial vascular calcification


(that can become transmural) as well as transdifferentiation of vascular smooth muscle cells into
osteoblast-like cells; these changes plus thrombosis lead to ischemic necrosis of the skin and soft
tissues. Early lesions usually present as poorly demarcated, often symmetric, very painful patches of
erythema or retiform purpura. They favor areas with abundant adipose tissue or sites of trauma.
Bullae or a dusky gray color may then develop, signifying imminent tissue necrosis and the
appearance of ulcerations with black, leathery eschars. Often there is surrounding subcutaneous
induration that extends beyond the margins of the visibly active lesions and this can be helpful in
distinguishing calciphylaxis from other forms of retiform purpura.

2-Cryoglobulins cause disease via two mechanisms: occlusion (type I) or immune complex-
mediated vasculitis (types II, III). Simple occlusion with minimal early inflammation and often
retiform purpura/necrosis develops when cryoproteins precipitate upon cold exposure, and this is
primarily a reflection of monoclonal immunoglobulins (IgG, IgM ≫ IgA, light chains), i.e. type I
cryoglobulinemia that is due to an underlying plasma cell dyscrasia or lymphoproliferative disorder.
Immune complex disease leads to inflammatory purpura that is palpable, and this is related to mixed
cryoglobulinemia (types II or III), often due to hepatitis C viral infection.

3-In contrast, homozygous (or compound heterozygous) deficiency or severe dysfunction of either
protein C or protein S leads to neonatal purpura fulminans within a few hours to 5 days after birth,
and it is fatal unless treated.

4-In warfarin necrosis, the procoagulant system takes significantly longer to reach its low-point
equilibrium than does the protein C-dependent anticoagulant activity. Warfarin necrosis usually
develops within 2–5 days of starting warfarin in the absence of heparin, and it is much more likely
to occur if loading doses of warfarin are used. A history of warfarin therapy is not mentioned in the
clinical stem.

5-Subcutaneous fat necrosis is the development of one or more mobile, firm, subcutaneous nodules
or plaques during the newborn period. It is sometimes associated with hypercalcemia or
thrombocytopenia. Histology is characterized by granulomatous lobular panniculitis with needle-
shaped clefts within lipocytes and giant cells.

4. What medication causes the following lesion to erupt?

724
A. Metformin

B. coumadin

C. ace inhibitors

D. Indinavir

E. azathioprine

Correct choice: D. Indinavir

Explanation: Indinavir, capecitabine, systemic retinoids, hydroxyurea and GMCSF cause pyogenic
granulomas to erupt.

5. This rash is characterized histologically by what finding?

A. Wedge-shaped infiltrate with large cells within the infiltrate

B. Lichenoid infiltrate

C. Acanthosis and neutrophilic aggregates in the epidermis

D. Mounds of parakeratosis

E. Irregular acanthosis

725
Correct choice:D. Mounds of parakeratosis

Explanation: This is pitryriasis rosea which is characterized by mounds of parakeratosis


histologically.

6. The following clinical manifestation is most commonly seen in the setting of:

A- liver failure
B- renal failure
C- thyroid disease
D- a genetic disorder
E- obesity

Correct choice: B. renal failure

Explanation: Calciphylaxis (calcific uremic arteriolopathy): ulcers with black eschars, violaceous
reticulated plaques on proximal limbs of patients with end-stage renal disease, often with
hyperparathyroidism, sometimes resulting in death, calcification in small- to medium-sized blood
vessels in dermis and fat, with secondary thrombosis, panniculitis, and fat necrosis.

7. Immunohistochemical staining with neuron-specific enolase is positive in:

A. Anaplastic large cell lymphoma

B. Malignant firbroushistiocytoma

C. Cutaneous T cell lymphoma

D. Merkel cell carcinoma

E. Sebaceous carcinoma

726
Correct choice: D. Merkel cell carcinoma

Explanation: Neuron-specific enolase (NSE) is a cytoplasmic product produced by Schwann cells


and neurons. This enzyme is present in neuroendocrine cells, neurons and tumors derived from
them. Positive staining for NSE is found in Merkel cell carcinomas, carcinoid tumors, and
malignant melanoma (among others). The remaining answer choices show negative staining with
NSE.

8. This lesion is characterized by dermal deposition of which material?

A. Amyloid

B. Mucin

C. Fibrin

D. Uric acid

E. Lipid

Correct choice: D. Uric acid

Explanation: Gouty tophi are formed by the deposition of monosodium urate crystals. Fibrin is
found in rheumatoid nodules, and mucin is deposited in lesions of granuloma annulare.

9. On histology of a eccrine ductal carcinoma there are ducts and nest of epithelial cells associated
with a dense fibrous stroma. There is variable nuclear pleomorphism, mitotic activity, and are
positive for CK7 and also:

A. Estrogen and progesterone receptor positive

727
B. CK20 positive

C. Oil red O

D. p53 positive

E. Keratin positive

Correct choice: A. Estrogen and progesterone receptor positive

Explanation: In an eccrine ductal carcinoma, the estrogen and progesterone receptor can be positive.
They are often positive for CK7, CEA, negative CK20, S100 and GCDFP-15 positive.

10. What is this condition?

A. Gouty Tophus

B. Sarcoid

C. Keratin granuloma

D. Rheumatoid Nodule

E. Granuloma annulare

Correct choice: A. Gouty Tophus

728
Explanation: Gouty Tophi occur due to an accumulation of monosodium urate in the tissues.
Deposits in the dermis act as a foreign body and granulomas form around these deposits. Unless the
tissue is fixed with alcohol (Carnoy's fixative), the yellow brown crystals of gout are not well
visualized on H and E. When the tissue is submitted in formalin the tophus appears pink and
amorphous.Gout is diagnosed in the initial attack by using a needle and syringe to extract a small
sample of fluid from around the affected joint space. MSU crystals can be identified under the
microscope. This test is also important to rule out other causes of an inflamed joint, such as
infection. If a biopsy sample is taken, the specimen should be placed in alcohol, as formalin
dissolves urate crystals. Joint x-rays may show findings consistent with gout, but these findings are
not diagnostic on their own. Furthermore, early on in the disease, x-rays may be normal or show
soft tissue swelling only. Urate levels may be elevated in the blood, but this finding alone is not
diagnostic. In some cases, the level may even be below urate saturation.

11. Positive staining with Gross cystic disease fluid protein 15 (GCDFP-15) suggests:

A. An apocrine origin

B. An eccrine origin

C. A sebaceous origin

D. A follicular origin

E. A keratinocyte origin

Correct choice: A. An apocrine origin

Explanation: Gross cystic disease fluid protein-15 (GCDFP-15) is a commonly used apocrine
marker. Gross cystic disease fluid protein-15 (GCDFP-15) is not a marker of the remaining answer
choices.

12. The best diagnosis is:

729
A. Schwannoma

B. Neurofibroma

C. Pallisaded and encapsulated neuroma

D. Perineurioma

Correct choice:C. Pallisaded and encapsulated neuroma

Explanation: Pallisaded and encapsulated neuroma

13. What is your best diagnosis?

A. Sebaceous Glands

B. Sebaceum

C. Lichen Planus

D. Atopic Eczema

E. Angiofibroma

Correct choice:E. Angiofibroma

Explanation: Angiofibroma is a small reddish spot or bump that consists of fibrous tissue and blood
vessels. They are most commonly found around the nose, cheeks, and chin, often combining to
form a distinctive butterfly-shaped pattern. Previously known as adenoma sebaceum.

730
14. What immunohistochemical stain is most likely to be positive in this condition?

A. HPV

B. HHV-8

C. PAS

D. S100

E. Tryptase

Correct choice: E. Tryptase

Explanation: The figure illustrates lesions of urticaria pigmentosa, which stain positively for
tryptase or c-kit as well as Giemsa, Leder, or toluidine blue. Patients typically present with red-
brown macules or slightly raised papules which may demonstrate a positive Darier’s sign
(urticaration with rubbing). HPV stain is positive in verrucae, HHV-8 is positive in Kaposi
Sarcoma, PAS is positive in a variety of conditions and is helpful in visualizing dermatophytes in
the epidermis, and S100 is found in a variety of conditions including melanoma.

15. Which of the following is the most likely diagnosis of this image?

731
A. Lipoma

B. Merkel cell carcinoma

C. Metastatic adenocarcinoma

D. Melanoma

E. Myxoid neurothekeoma

Correct choice:C. Metastatic adenocarcinoma

Explanation: In this image, the lymphatics are filled with large atypical cells trying to form glands
making metastatic adenocarcionma the most likely diagnosis. Merkel cell carcinoma is a sheet of
blue cells with salt and pepper nuclei. A lipoma is a collection of normal fat. Melanoma would have
nests of atypical melanocytes. A myxoid neurothekeoma would be a loose collection of cells in
thekes with a cellular stroma and the cells do not attempt to form glands.

16. What is this neoplasm?

A. Mastocytoma

B. Poroma

C. Glomus tumor

D. Langerhans cell histocytosis

E. Nevus

Correct choice:E. Mastocytoma

732
Explanation: Approximately 10% cases of mastocytosis present as a solitary lesion, otherwise
known as a mastocytoma. Typically they present in childhood on trunk or wrist and usually are self
limited and spontaneously resolve. Histologically the epidermis overlying the mast cell proliferation
is often hyperpigmented. Within the dermis there are increased numbers of “fried egg mast cells.
Often the mast cells are primarily located in the upper dermis where they fill and expand the dermal
papillae. Frequently admixed among the mast cells there are increased numbers of eosinophils.

17. A male patient presents with hypopigmented macules on his upper back that are asymptomatic
and more noticeable in the summer. What is the most likely diagnosis?

A. pityriasis rosea

B. vitiligo

C. tinea versicolor

D. eczema

E. waardenberg syndrome

Correct choice:C. tinea versicolor

Explanation: Tinea versicolor often is asymptomatic and more noticeable in the summer as patients
often tan and the areas with the tinea versicolor do not tan as azeleic acid is secreted by the
malassezia and melanin production is suppressed.

733

18. On histology, this type of dermatofibrosarcoma protuberans is associated with recurrence and is
regarded as a poor prognosis. It is characterized by a typical herringbone morphology of more
closely packed spindled cells with increased mitosis. This best describes:

A. Bednar tumor

B. Fibrosarcomatous tumor

C. Giant cell fibroblastoma

D. Fibroblastic tumor

E. Angiodermatofibrosarcoma tumor

Correct choice: B. Fibrosarcomatous tumor

Explanation: This best describes the fibrosarcomatous type of dermatofibrosarcoma protuberans. It


has a high rate of recurrence and is associated with a poor prognosis. They are also CD34+ and
wide excision is recommended.

19. Cellular neurothekeoma stains with:

A. Stromelysin-3

B. Desmin

C. S-100

D. PGP-9.5

E. Low molecular weight keratin

Correct choice:D. PGP-9.5

Explanation: PGP-9.5 and S100-a6 stains cellular neurothekeoma.

Stromelysin-3 is positive in dermatofibromas and negative in dermatofibrosarcoma protuberans.


Desmin stains rhabdomyosarcoma. S-100 stains neural tumors and melanocytic tumors among other
things, but cellular neurothekeomas are generally S100-negative.

734
20. What is the best diagnosis for this vesiculobullous disorder?

A. Bullous Pemphigoid

B. Porphyria Cutanea Tarda

C. Pemphigus vulgaris

D. Dermatitis Herpetiformis

E. Arthropod Bite

Correct choice:A. Bullous Pemphigoid

Explanation: Bullous Pemphigoid: Most common subepidermal blistering disease that frequently
occurs on abdomen, groin, flexor surface of arms and legs of the elderly. In men an association with
HLA-DQ7 has been seen. Often eosinophilia and elevated IgE is seen in the serum. The
pathogenesis of this blistering disorder is due to binding of antibodies, IgG1 and 4 or IgE and rarely
IgA, to BPAg I (230kd) and/or BP Ag II (180kd) in the hemidesmosome. Histologically there is a
unilocular subepidermal blister with eosinophils. Early lesions may show eosinophilic spongiosis
with papillary dermal edema. Within the dermis there is a lymphocytic eosinophilic infiltrate that is
both perivascular and interstitial. DIF shows linear deposits of C3> IgG along the DEJ. Salt split
skin show localization of immunoreactants in the vesicle roof. Circulating anti-BMZ antibodies are
seen in 60-80% of patients but do not correlate to disease activity.

21. The PDGF gene has been shown to be translocated into the collagen I gene in:

A. Angiofibroma

B. Dermatofibroma

C. Dermatofibrosarcoma protuberans

D. Fibrous hamartoma of infancy

735
E. Solitary fibrous tumor

Correct choice:C. Dermatofibrosarcoma protuberans

Explanation: A translocation involving the PDGF gene and collagen I gene is found in
dermatofibrosarcoma protuberans.

22. Which disease process best describes Texier's disease?

A. Neutrophilic dermatosis

B. Deposition disorder

C. Infectious process

D. Panniculitis

E. Granulomatous disease

Correct choice:D. Panniculitis

Explanation: Texier's disease is a panniculitis secondary to vitamin K injections causing sclerotic


lesions with lilac borders on the buttocks and thighs resembling a cowboy belt and holster. Inflamed
nodules with varying degrees of pain and fibrosis have been observed in other forms of panniculitis
due to injection, with the distribution of lesions sometimes providing a clue to their cause. A
dramatic example of this is Texier disease, a panniculitis due to phytonadione (vitamin K)
injections. In this disorder, sclerotic lesions with lilac-colored borders form on the buttocks and
thighs, in aconfiguration resembling a “cowboy gunbelt and holster”. Lesions due to blunt trauma
often have an ecchymotic character andinvolve locations such as the shin, arm or hand.
Hypertrichosis may also be present, possibly the consequence of local hyperemia or angiogenesis.

23. What is the most likely diagnosis?

736
A. Wood splinter

B. suture

C. silicone

D. gel foam

E. Hyaluronic acid

Correct choice:A. Wood splinter

Explanation: Wood splinter is brown with rectangular cells. Suture is typcially a circle filled with
many smaller circles. Silicone is a collection of vacoules. Gel foam is an irregularly shaped purple
mass. Hyaluronic acids are just like mucin wispy and purple collections.

24. This disorder is sometimes asscociated with what other symptoms?

A. Fissured tongue and facial nerve palsy

B. Shortness of breath

C. Angiomas

D. Atopic dermatitis

E. Neuroma

Correct choice:A. Fissured tongue and facial nerve palsy

Explanation: Melkersson-Rosenthal syndrome is traditionally characterized by the triad of


granulomatous cheilitis, facial nerve palsy, and fissured tongue.

737
25. Diffuse staining with this marker is suggestive of BCC rather than trichoepithelioma:

A. CAM 5.2

B. Bcl-2

C. PTAH

D. CEA

E. Cytokeratin 8

Correct choice:B. Bcl-2

Explanation: Trichoepitheliomas stains with bcl-2 on the periphery of individual tumor islands
while BCC stains diffusely.

26. This biopsy was obtained from an immunosuppressed patient with multiple bruise like lesions
on arms and legs. What is this neoplasm?

A. Kaposi's sarcoma

B. Lobular hemangioma

C. Targetoid hemosiderotic hemangioma

D. Tufted hemangioma

E. Angiosarcoma

Correct choice A. Kaposi's sarcoma

738
Explanation: Kaposi's Sarcoma: The vascular proliferation in all subtypes of Kaposi's sarcoma are
associated with an infection with HHV-8. Histologically early on there is a subtle proliferation of
irregular angulated slit like blood vessels in dermis that dissects the collagen. When these new
blood vessels encase preexisting blood vessels it's called a promontory sign. In the stroma there is a
lymphoplasmacytic infiltrate, extravasated red blood cells and hemosiderin.

Blood tests may show no abnormality, depending on whether there are associated disorders such as
AIDS. Anaemia may arise if there is bleeding. KSHS assays or antibody titers to KSHS are
challenging to interpret. CD4 lymphocyte counts and plasma HIV load studies are performed in
patients with HIV infection. The appearance of Kaposi sarcoma lesions is often typical, but a skin
biopsy of a lesion allows a definite diagnosis, as various lesions such as melanoma, fungal
infections, and mycetoma mimic Kaposi sarcoma in appearance and location. The histopathology of
Kaposi sarcoma shows red cells in slit-like spaces formed by atypical spindle cell proliferation of
endothelial cells and associated with inflammatory cells.

27. What is the diagnosis?

A. Sarcoidosis

B. Tuberculosis

C. Rosacea

D. Leprosy

E. Granulomatous leishmaniasis

Correct choice:E. Granulomatous leishmaniasis

739
Explanation: Long-standing lesions of chronic cutaneous leishmaniasis will often show
granulomatous inflammation with scarce organisms.

28. Mutations in this gene are associated with this dermatopathologic presentation:

A. PTHRP

B. c-Kit (CD117)

C. FGFR3

D. NRAS

E. TERT

Correct choice:B. c-Kit (CD117)

Explanation: Picture is of mastocyotosis pathology with positive Giemsa staining.

29. A 44-year old man presents with a beefy red plaque involving the right penile shaft. Biopsy
would reveal which of the following histologic findings:

A. Asteroid Bodies

B. Caterpillar Bodies

740
C. Cowdry Type A Bodies

D. Donovan Bodies

E. Dutcher Bodies

Correct choice D. Donovan Bodies

Explanation: Granuloma Inguinale is caused by Klebsiella granulomatis. This infection presents as


a chronic suppurative infection of the genital region. Biopsy reveals Donovan bodies, which are
intrahistiocyte inclusions comprised of the implicated organisms. These stain positively with both
Warthin-Starry stain and Giemsa.

30. Treatment of postmenopausal women with systemic conjugated estrogens has demonstrated:

A. Increased cellular atypia

B. Decreased mitotic activity of keratinocytes

C. Increased dermal collagen content

D. Decreased sebaceous gland activity

E. Increased skin laxity

Correct choice:C. Increased dermal collagen content

Explanation: Systemic use of conjugated estrogens in postmenopausal women increases total skin
thickness, dermal collagen content and mitotic activity of keratinocytes. In addition, there is a
reduction of dry skin, slackness and increased hydration.

31. Leukocytoclastic vasculitis is typically a part of the histology of which condition?

A. Rheumatoid neutrophilic dermatosis

B. Erythema elevatum diutinum

C. Urticaria

741
D. Granuloma inguinale

E. Sweet's Syndrome

Correct choice:. B . Erythema elevatum diutinum

Explanation: Erythema elevatum diutinum histologically shows a leukocytoclastic vaculitis, but


with prominent interstitial neutrophils. Other conditions with evidence of LCV include IgA
vasculitis, granuloma faciale, urticarial vasculitis, and occasionally serum sickness. Rheumatoid
neutrophilic dermatitis is seen in the setting of severe rheumatoid arthritis. The pathogenesis is not
understood, but in the few reports of this rare dermatosis, a true vasculitis has not been seen.
Classically, Sweet's syndrome also does not show evidence of a true vasculitis. LCV is not a typical
histologic finding in urticaria or granuloma inguinale.

32. What structure does this tumor most commonly invade?

A. blood vessels

B. hair follicle

C. nerves

D. sebaceous gland

E. arrector pili muscle

Correct choice:C. nerves

Explanation: Microcystic adnexal carcinomas typically have perineural involvement. They must be
distinguished from desmoplastic trichoepitheliomas, syringomas and morpheaform BCCs.

742
33. The pathology seen here is taken from a 68-year-old male with a diffuse skin eruption, with few
blisters and crusting on the oral mucosa. Which of the followin is the most likely diagnosis?

A. Lupus erythematosus (chronic or discoid)

B. Pemphigus vulgaris

C. Tumid lupus

D. Porphyria cutanea tarda

E. Bullous pemphigoid

Correct choice:B. Pemphigus vulgaris

Explanation: The suprabasal acantholysis that extends down adnexal structures is most consistent
with pemphigus vulgaris. Chronic discoid lupus and tumid lupus will have superficial and deep
perivascular lymphoid infiltrates and dermal mucin.PCT will have a cell-poor subepidermal vesicle
and thickening of superficial dermal blood vessels. Bullous pemphigoid will have a cell-rich
subepidermal vesicle with eosinophils.

34. What is the diagnosis?

743
A. Calciphylaxis

B. Polyarteritis nodosa

C. Leukocytoclastic vasculitis

D. Erythema nodosum

E. Cryoglobulinemia

Correct choice:A. Calciphylaxis

Explanation: Calciphylaxis: seen in patients with elevated Ca-PO4 product and secondary
hyperparathyroidism most commonly patients in renal failure. Calcification occurs within the media
of vessels in the lower dermis resulting in intimal hyperplasia and secondary intraluminal thrombi.
Vascular occlusion then results in epidermal/dermal infarction.

A deep wedge skin biopsy may be necessary to diagnose calciphylaxis, as a similar appearance can
be seen in other conditions such as necrotizing fasciitis, cryoglobulinemia, antiphospholipid
syndrome, coumarin necrosis, and vasculitis. Multiple biopsies may be necessary, with a risk of
propagating calciphylaxis. The pathologist looks for calcium deposited within scarred and blocked
blood vessels in the subcutaneous tissue. Perieccrine calcium deposition may be noted when
vascular calcification is absent but may be subtle. There may also be inflammation of the fat
(panniculitis). X-rays of the affected limb may demonstrate vascular calcification within the skin;
however, this may also be seen in healthy patients with renal disease that are not affected by
calciphylaxis. Bone scintigraphy using technetium Tc 99m bisphosphonates in patients with
calciphylaxis shows increased radiotracer uptake in soft tissues throughout the body and is
specifically enhanced in indurated plaques affected by calciphylaxis (but is absent in ulcers due to
reduced blood flow at sites of tissue necrosis).

35. This lesion is part of the constellation of findings in which genodermatosis?

744
A. Neurofibromatosis I

B. Darier's disease

C. Brooke-Spiegler syndrome

D. Cowden's disease

E. Birt-Hogg-Dube syndrome

Correct choice:E. Birt-Hogg-Dube syndrome

Explanation: Birt-Hogg-Dube syndrome is caused by a mutation of the FLCN gene which encodes a
protein called folliculin. The disorder is characterized by the presence of fibrofolliculomas,
trichodiscomas, and acrochordons.

36. What type of nevus is represented in this image?

A. Blue

B. Spitz

C. Dysplastic

D. Combined

E. Balloon cell

Correct choice:D. Combined

Explanation: This nevus is composed of two distinct populations of melanocytes; thus, it would be
considered a combined nevus.

37. This presented as a papule with a tuft of hair on the face, what is this neoplasm?

745
A. Trichofolliculoma

B. Trichoepithelioma

C. Fibrous Papule

D. Acrochordon

E. Fibrofolliculoma

Correct choice:A. Trichofolliculoma

Explanation: Trichofolliculoma: Most often occurs on the face as a papule with a tuft of hair.
Histologically composed of a central dilated large follicle (Mama hair), from which many
secondary smaller hair follicles radiate (Baby hairs), with the entire unit often enveloped in a
vascularized fibrotic stroma. If sebaceous glands are associated with the hair follicles then the term
used is a sebaceous trichofolliculoma or a Folliculosebaceous cystic hamartoma. Within the
mesenchymal stroma there are increased CD34 and factor XIIIA fibroblasts and Merkel cells as is
seen in the ORS of the follicles. Prototypically, a trichofolliculoma consists of a central cystic space
with infundibular cornification and central orthokeratin. Sometimes, cross-sections of hair shafts are
identifiable within the cyst. Relatively well-developed and occasionally oddly formed vellus
follicles protrude in radial fashion from the central structure. The follicles usually display a bulb
and papilla and exhibit inner and outer sheath and isthmic differentiation. The entire structure,
including the central cyst and its associated radiating follicles, is enveloped by a vascularized
fibrous (angiofibroma-like) stroma.

38. Which of the following are true of reticulohistiocytoma?

A. Common in children

B. Giant cells with “ground-glass” cytoplasm

C. Association with arthritis

746
D. Immunostaining is positive for PTEN

E. Trauma is not a precipitating factor

Correct choice: B . Giant cells with “ground-glass” cytoplasm

Explanation: Reticulohistiocytomas, also called giant cell reticulohistiocytomas, occur almost


exclusively in adults. They are generally solitary, have a ground-glass like cytoplasm and unlike the
multicentric type, are not associated with mutilating arthritis or predisposition for malignancy.
Trauma may be a predisposing factor, and they do not stain positively for PTEN mutations.

39. A 62-year-old man presents a 1.8cm pink, friable mass on his right upper back. A full skin exam
shows no other suspicious lesions and no palpable lymph nodes. A shave biopsy is done and the
H+E and CK20 immunohistochemistry are shown in the images. Which of the following is the best
treatment to proceed with next?

A. Mohs' chemosurgery

B. Mohs' chemosurgery and radiation therapy

C. Wide local excision

D. Wide local excision with sentinel lymph node biopsy

E. Wide local excision and radiation therapy

Correct choice:D. Wide local excision with sentinel lymph node biopsy

Explanation: The clinical and pathological findings are consistent with a Merkel cell carcinoma.
The preferred treatment for a tumor on the trunk without clinically apparent lymphadenopathy is
wide local excision (1-3 cm margins) and sentinel lymph node biopsy. If the lymph node biopsy is

747
positive, then the patient will likely require radiation therapy. For lesions >2cm, some clinicians
will not perform a sentinel lymph node biopsy since radiation therapy is indicated regardless, given
the high risk of lymph node metastasis.

40. Supporting evidence for the diagnosis of mycosis fungoides is CD4+ lymphocytes with loss of
CD7 as well as loss of

A. CD27

B. CD68

C. CD5

D. CD20

E. CD30

Correct choice:C. CD5

Explanation: CD5 as well as CD7 are sometimes lost on the surface of epidermotropic T cells in
mycosis fungoides. CD2, CD3, and CD5 are T cell markers. CD20 is a B cell marker. CD30 is
positive in anaplastic large cell lymphoma cells, Hodgkins lymphoma, and lymphomatoid
papulosis. Reactive infiltrates can also have some CD30-positive cells. CD68 marks histocytes.

41. Which of the following immunostains would be positive in primary cutaneous amyloid?

A. Cytokeratin 5

B. Cytokeratin 15

C. Melanoma Antigen Recognized by T cells 1

D. CD-31

E. CD-57

Correct choice: A. Cytokeratin 5

748
Explanation: The amyloid deposits in primary cutnaeous amyloid are derived from keratin
intermediate filament proteins. Degeneration of basal keratinocytes in the overlying epidermis
likely play a strong role. In one theory, cytokeratin released from apoptotic basal keratinocytes form
the amyloid fibrils. Pathology specimens from primary cutaneous amyloid stain positive for
cytokeratin 5. The other listed immunostains are not positive in primary cutnaeous amyloid.

42. The organism on this slide stained with Warthin-Starry is responsible for what disease?

A. Bacillary Angiomatosis

B. Cellulitis

C. Madura foot

D. Pyoderma gangrenosum

E. Pyogenic granuloma

Correct choice:A. Bacillary Angiomatosis

Explanation: Bacillary (epithelioid) angiomatosis is caused by Bartonella Henselae or Bartonella


quintana (trench fever- from body louse). Clinically resembles PG but multiple lesions and in a
HIV+ patient, usually with history of exposure to cats. Histologically there is dome shaped lesion
with an epidermal collarette with a proliferation of reactive plump blood vessels similar to a PG, but
the lesion is less lobular and there is PMNs and dust throughout the lesion along with extracellular
clumps of purplish gram ‚¬â€œ bacilli. The organism is + for Warthin-Starry, which highlights the
organisms in a characteristic Chinese letter configuration.

749
43. This patient has a black toenail. What is the diagnosis?

A. Medication deposition

B. Tattoo

C. Ungual melanoma

D. Tinea unguium

E. Intraungual hematoma

Correct choice: E. Intraungual hematoma

Explanation: Intraungual hematoma can develop as a result of trauma and can appear black on
clinical exam. A histologic examination will show blood within the nail plate.

44. What is this lesion?

A. Hidradenoma

750
B. Spiradenoma

C. Dermal duct tumor

D. Trichoepithelioma

E. Chondroid syringoma

Correct choice:E. Chondroid syringoma

Explanation: Chondroid syringomas (mixed tumors) are well-circumscribed dermal nodules


demonstrating epithelial cords and ductal structures in a myxoid, chondroid and fibrous stroma.

45. These lesions typically occur on which part of the body?

A. Face

B. Hands

C. Mucosa

D. Legs

E. Scalp

Correct choice: A. Face

Explanation: Hidrocystomas are cysts lined by a double layer of epithelial cells. They most
commonly occur on the face but can occur in other areas as well.

751
46. What does this show on histopathology?

A. Massive dermal edema with telangiectasias and a lichenoid infiltrate below the dermal
edema

B. Interface dermatitis

C. Lichenoid dermatitis

D. Loose myxoid collagen

E. Squared off biopsy with thick collagen, decreased adnexal structures and entrapped
eccrine glands higher up in the dermis

Correct choice: E. Squared off biopsy with thick collagen, decreased adnexal structures and
entrapped eccrine glands higher up in the dermis

Explanation: This is morphea which is characterized by a squared off biopsy with dense fibrotic
collagen, decreased adnexal structures and entrapped eccrine glands higher up in the dermis.

47. All of the following are true regarding smooth muscle hamartomas except:

A. Hyperpigmentation

B. Transient piloerection

C. Hypertrichosis

D. Association with “Michelin tire baby” in females

E. Becker’s nevus syndrome is an example

Correct choice: D. Association with “Michelin tire baby” in females

752
Explanation: Smooth muscle hamartomas of the skin may develop from three locations: the arrector
pili muscles, blood vessel walls, and genital/areolar skin. The have been associated with
hyperpigmentation, tranisent piloerection ("pseudo-Darier's sign") and hypertrichosis. It is thought
to be the underlying lesion in "Michelin tire baby" syndrome in boys, not girls.

48. Which of the following entities can prove difficult to diagnose on H+E histology, as it can
appear with similar features to erythema multiforme?

A. Erythema elevatum diutinum

B. Lichen planus pigmentosus

C. Paraneoplastic pemphigus

D. Pemphigus erythematosus

E. Zoon's balanitis

Correct choice: C. Paraneoplastic pemphigus

Explanation: In paraneoplastic pemphigus, there is usually an overt interface or lichenoid infiltrate


associated with necrotic keratinocytes at all levels of the epidermis which can closely resemble
erythema multiforme. In addition, foci of suprabasal acantholysis are seen. Direct
immunofluorescence usually shows intercellular and basement membrane deposition of IgG and
C3. The other listed entities do not appear as similar to EM on H+E.

49. The predominant location of the cleft in acropustulosis of infancy is:

A. Dermal

B. Basment mebrane zone

C. Basal keratinocytes

D. Suprabasal

E. Subcorneal/granular

753
Correct choice: E. Subcorneal/granular

Explanation: Acropustulosis of infancy presents as idiopathic pustules on acral skin. Diagnosis is


made only after other causes of pustules have been ruled out, and it usually resolves in a few years.
The cleft in acropustulosis of infancy is subcorneal/granular with neutrophils.The other listed levels
of split are not accurate.

50. A patient presents to you for an initial consultation of hair loss. You explain that hair disorders
may affect various levels of the hair follicle. What are the layers of the lower portion of the hair
follicle from outer to inner?

A. Fibrous root sheath, vitreous layer, outer root sheath, inner root sheath (Henle, Huxley,
cuticle of inner root sheath), hair cuticle, hair cortex, hair medulla

B. Fibrous root sheath, vitreous layer, outer root sheath, inner root sheath (Huxley, Henle,
cuticle of inner root sheath), hair cuticle, hair cortex, hair medulla

C. Fibrous root sheath, vitreous layer, outer root sheath, inner root sheath (Henle, Huxley,
cuticle of inner root sheath), hair cortex, hair medulla, hair cuticle

D. Fibrous root sheath, outer root sheath, inner root sheath (Henle, Huxley, cuticle of
inner root sheath), vitreous layer, hair cuticle, hair cortex, hair medulla

E. Vitreous layer, outer root sheath, inner root sheath (Henle, Huxley, cuticle of inner
root sheath), hair cuticle, hair cortex, hair medulla, fibrous root sheath

Correct choice: A. Fibrous root sheath, vitreous layer, outer root sheath, inner root sheath (Henle,
Huxley, cuticle of inner root sheath), hair cuticle, hair cortex, hair medulla

Explanation: The layers of the lower portion of the hair follicle from outer to inner are: fibrous root
sheath, glassy/vitreous layer, outer root sheath, inner root sheath (Henle, Huxley, cuticle of inner
root sheath), hair cuticle, hair cortex, hair medulla

51. From what cell is this deposition derived?

A. Lymphocyte

B. Melanocyte

754
C. Sebocyte

D. Keratinocyte

E. Macrophage

Correct choice:D. Keratinocyte

Explanation: This is macular amyloid which is keratinocyte derived and due to chronic rubbing. The
other listed cells do not contribute to macular amyloid

52. These biopsy findings are most likely related to


which compound?

A. Silver

B. Gold

C. Amiodarone

D. Minocycline

E. Topical hydroquinone

Correct choice:E. Topical hydroquinone

Explanation: Exogenous ochronosis is most frequently seen after long-term application of topical
hydroquinone compounds. On pathology, there are large yellow brown 'banana-shaped' deposits in

755

the dermis. Identical findings can be seen in alkaptonuria, which can cause an endogenous
ochronosis. Alkaptonuria is caused by an autosomal recessive deficiency of homogentisic acid
oxidase.

53. Which form of calcinosis cutis would describe calcinosis scroti?

A. Idiopathic

B. Metastatic

C. Dystrophic

D. Iatrogenic

E. Autoimmune

Correct choice:A. Idiopathic

Explanation: Localized calcium deposition frequently occurs idiopathically on the scrotum, penis,
or vulva. Metastatic calcification occurs in the setting of abnormal calcium or phosphorus
metabolism. Dystrophic calcification occurs in areas of trauma or other damage to tissue. Iatrogenic
calcification occurs secondary to a treatment or procedure.

54. What is the diagnosis?

A. Dermatofibrosarcoma

B. Nodular fasciitis

C. Angiolipoma

756
D. Epithelioid sarcoma

E. Liposarcoma

Correct choice:C. Angiolipoma

Explanation: Angiolipomas are tumors of fat that are characteristically painful. Histopathologically,
mature adipocytes are seen with numerous vessels. Microthrombi are often present. The other
answer choices do not have the pictured histopathological findings.

55. Histologic examination of cutaneous metastases from breast cancer is likely to demonstrate:

A. Dense lymphocytic infiltration

B. Tumors cells perivascularly

C. Eosinophilia

D. Tumor cells in the lymphatics

E. Band-like dermal infiltrate

Correct choice:D. Tumor cells in the lymphatics

Explanation: Histologic appearance of metastatic breast carcinoma to the skin typically shows
tumor cells in linear arrangement in so-called "Indian-filing" pattern. These tend to occur in the
dermis and subcutaneous lymphatics. Extensive lymphatic dissemination may be caused by
retrograde lymphatic spread. The tumor cells are large, pleomorphic with hyperchromatic nuclei.

757
56. Mulberry cells contain increased:

A. Phagolysosomes

B. Mitochondria

C. Golgi

D. Ribosomes

E. Phagolysosomes and mitochondria

Correct choice: B. Mitochondria

Explanation: Hibernomas commonly arise in the neck, axillae, and posterior shoulder. The cells are
multivacuolated and resemble mulberries; the cells are filled with mitochondria, as are the cells in
normal brown fat. The mulberry cells in hibernomas do not contain the other listed organelles.

57. A focal non-Langerhans cell histiocytosis with Touton giant cells is seen in a child with
Neurofibromatosis Type I. The patient has an increased risk of which of the following conditions?

A. Glaucoma

B. Acute myeloid leukemia

C. Juvenile myelomonocytic leukemia

D. Atrial myxoma

E. Ocular hypertelorism

Correct choice: C. Juvenile myelomonocytic leukemia

Explanation: The described lesion is a Juvenile Xanthogranuloma, which is the most common non-
langerhans cell histiocytic neoplasm. These usually affect children and are found in the head and
neck area. Touton giant cells are a characteristic histologic finding. There is a vastly increased risk
of developing Juvenile myelomonocytic leukemia (choice 3) in patients with JXG and NF1, thought
to be due to the RAS oncogene being affected. Ocular JXG may be associated with glaucoma or
blindness and thus remains an important entity, but glaucoma risk is not increased with cutaneous

758
JXG. There is no increased risk of AML. Atrial myxoma are seen in NAME/LAMB syndrome.
Ocular hypertelorism is seen in LEOPARD syndrome.

58. Pustulo-ovoid bodies of Milian can be found associated with which of the following conditions?

A. Granular cell tumor

B. Leishmaniasis

C. Schwannoma

D. Chromomycosis

E. Rhinoscleroma

Correct choice: A. Granular cell tumor

Explanation: Pustulo-ovoid bodies of Milian can be seen in association with granular cell tumors.
They appear as relatively large and eosinophilic intracytoplasmic inclusions with a surrounding
clear halo. These inclusions characteristically stain positive with PAS and are diastase resistant.
Leishmaniasis is associated with leishman-Donovan bodies, which are intracytoplasmic basophilic
organismal (amastigotes) inclusions found in parasitized histiocytes. Verocay bodies may be seen in
schwannomas, which appear as pallisaded nuclei in rows. Medlar bodies (“copper pennies”) can be
seen in cutaneous infectious of chromomycosis and appear as grouped ovoid spores measuring 6 to
12 micrometers both within histiocytes as well as interstitially. Russell bodies are associated with
rhinoscleroma and are elliptical basophilic inclusions found in plasma cells and are comprised of
immunoglobulins.

59. What HPV strain causes this lesion?

A. HPV 2

B. HPV 6

C. HPV 5

D. HPV 7

E. HPV 1

759
Correct choice: E. HPV 1

Explanation: Myrimecial warts are endophytic growths on the soles of the feet. They are caused by
HPV 1. There are numerous eosinophilic inclusions within the keratinocytes.

60. Biopsy of this lesion reveals a poorly-defined dermal mass of small blue monomorphic round
cells with scant cytoplasm and nuclear molding. Which histologic stain is most helpful in
distinguishing this lesion from small cell lung carcinoma.

A. CK20

B. TTF-1

C. Chromogranin

D. Synaptophysin

E. Somatostatin

Correct choice:B. TTF-1

Explanation: The pictured image is that of merkel cell carcinoma (MCC), a rare highly aggressive
malignant neuroendocrine carcinoma. In order to differentiate MCC from small cell lung carcinoma

760
(SCLC; aka Oat cell carcinoma), another malignant neuroendocrine carcinoma, histologic staining
with TTF-1 is most helpful. This is because MCC (nearly always) stains negative for TTF-1,
whereas SCLC stains positive for TTF-1. The remaining answer choices will be positive in both
MCC and SCLC, so they cannot be used to differentiate between the two malignancies.

61. A 40-year-old man presents with many skin-colored papules of the face. He thinks that his
grandfather may have had the same condition. A representative skin lesion is biopsied for further
evaluation. Which pilar neoplasm is histologically characterized as a fibrous pink or amphophilic
fibromucinous orb, no hair fibers, and epithelial strands radiating outward from a central follicle-
like structure?

A. Fibrofolliculoma

B. Trichoepithelioma

C. Pilomatricoma

D. Trichofolliculoma

E. Trichoadenoma

Correct choice:A. Fibrofolliculoma

Explanation: In fibrofolliculomas, the strands of epithelium are not well enough differentiated to
form hair fibers. No bulb, inner or outer root sheath is present. The strands of epithelium may have
an anastomosing pattern. Histolocially there are epithelial strands radiating outward from a central
follicle-like structure, no hair fibers, and a fibrous pink or amphophilic fibromucinous orb.

Trichoepithelioma: A trichoepithelioma is a blue tumor composed of basaloid cells and is a


distinctive type of trichoblastoma. At scan, it has finger-like projections and cribriform nodules. The
stroma resembles the normal fibrous sheath of the hair follicle, with concentric collagen and many
fibroblasts. Papillary mesenchymal bodies are prominent. Mucin may be present within cribriform
tumor islands, but never within the stroma.

Pilomatricoma: A pilomatricoma appears as a large ball with internal trabeculae and is characterized
by basophilic cells that resemble those of the hair matrix which keratinize to form shadow cells.
There is often calcification and bone formation.

Trichofolliculoma: A trichofolliculoma demonstrates miniature follicles converging on a central


infundibulum embedded in an eosinophilic fibrous orb or stroma.

761
Trichoadenoma: A trichoadenoma is a pilar neoplasm histologically characterized by multiple red
doughnuts in the dermis, each resembling a follicular infundibulum, often in pairs resembling
eyeglasses or toasted oat cereal.

62. The mutation in the syndrome that is most associated with this lesion is which of the following?

A. PTCH

B. APC

C. MSH2

D. PTEN

E. STK11

Correct choice D. PTEN

Explanation: Cowden's syndrome is a mutation in PTEN and is associated with tricholemmomas


and breast, thyroid and GI carcinoma. They also get acral keratoses and palmar pits and lipomas.

63. What is the most likely diagnosis for this single lesion?

A. wart

B. hailey hailey

C. dariers

D. pemphigus vegetans

E. Warty dyskeratoma

762
Correct choice: E. Warty dyskeratoma

Explanation: Warty dyskeratoma is a cup-shaped invagination with acanthlytic and dyskeratosis.


Both hailey hailey and darier's do not have this architecture. Hailey hailey has more acantholysis,
dariers has more dyskeratosis, and both are larger plaques. It would be unusal to have one single
lesion of hailey hailey, dairers or pemphigus vegetans.

64. A 6-month old infant develops tense blisters on the distal extremities which develop on normal-
appearing skin. He has a neurologic disease and has been immobilized for a long period of time.
The blisters spontaneously heal within 1-2 weeks. You suspect a diagnosis of coma bullae. Which of
the following is the key characteristic histologic feature that would assist you in making this
diagnosis?

A. Necrosis of sweat glands

B. Intraepidermal bullae

C. Subepidermal bullae

D. Pronounced inflammatory cell infiltrate

E. Positive direct immunofluorescence for IgG and C3

763
Correct choice: A. Necrosis of sweat glands

Explanation: Coma bullae are predominantly subepidermal. However, intraepidermal blisters are
also observed as the result of re-epithelialization of older lesions or epidermal spongiosis. There is a
variable degree of epidermal necrosis. Sweat gland necrosis is a characteristic finding, and focal
necrosis of the epithelium of the pilosebaceous follicles can also be seen. There is only a sparse
inflammatory cell infiltrate. DIF is usually negative, but deposits of IgG, IgM and/or C3 have been
found within the walls of dermal blood vessels.

65. Biopsy of a mucosal neuroma from a patient with MEN IIb looks histologically like a:

A. Neurilemmoma

B. Neurofibroma

C. Palisaded encapsulated neuroma

D. Traumatic neuroma

E. Neurothekeoma

Correct choice: C. Palisaded encapsulated neuroma

Explanation: Mucosal neuromas in MEN IIb often look histologically like PENs. Occasionally
mucosal neuromas display thickened nerves/perineurium.

The other listed entities do not typically share the same features as mucosal neuromas in MEN.

66. What immunohistochemical stain is most likely to be positive in this condition?

A. HPV

B. HHV-8

C. PAS

D. S100

E. Tryptase

764
Correct choice: E. Tryptase

Explanation: The figure illustrates lesions of urticaria pigmentosa, which stain positively for
tryptase or c-kit as well as Giemsa, Leder (chloroacetate esterase), or toluidine blue. Patients
typically present with red-brown macules or slightly raised papules which may demonstrate a
positive Darier’s sign (urticaration with rubbing). HPV stain is positive in verrucae; HHV-8 is
positive in Kaposi Sarcoma. PAS is positive in a variety of conditions and is helpful in visualizing
dermatophytes in the epidermis, and S100 is found in a variety of conditions including melanoma.

67. A deposition of what substances leads to these deposits?

A. Homogentisic acid

B. Amyloid

C. Fungal organisms

D. Colloid

E. Tattoo

Correct choice: A. Homogentisic acid

Explanation: Ochronosis: can be either endogenous or exogenous. The endogenous form is due to
an AR defect in homogentisic acid oxidase, which prevents tyrosine and phenylalanine from being

765
degraded beyond homogentisic acid. The exogenous form of ochronosis, on the other hand, occurs
from topical hydroquinone, mercury resorcinol, phenol, picric acid or benzene, or systemic quinine
or chloroquine which all inhibits homogentisic oxidase and leads to a focal accumulation of
homogentisic acid. Within the superficial dermis there are irregular shaped elongated yellow
deposits which have been described as as bananas in the dermis that occur due to deposition of
homogentisic acid on the collagen.

68. In children with pernio, what may be associated?

A. cryoglobulins

B. diabetes

C. lupus

D. calcinosis cutis

E. osteoma cutis

Correct choice: A. cryoglobulins

Explanation: In kids, pernio can be associated with cryoglobulins and cold agglutinins. It is
triggered by cold and wet which results in acral violaceous color with burning and itching.

69. In which condition are Gamma-Favre bodies seen?

A. Granuloma inguinale

766
B. Lymphogranuloma venereum (LGV)

C. Syphilis

D. Chancroid

E. Herpes virus

Correct choice: B. Lymphogranuloma venereum (LGV)

Explanation: LGV is a sexually transmitted disease that is characterized by suppurative inguinal


adenitis with matted lymph nodes, inguinal bubo with secondary ulceration, and constitutional
symptoms. It is caused by Chlamydia trachomatis serotypes L1, L2, and L3. First line treatment is
doxycycline 100mg bid for 3 weeks as well as treating the sexual partners. Gamma-Favre bodies are
found in histiocytes in LGV.

70. This tumor is vimentin+ and cytokeratin+:

A. Angiosarcoma

B. Neurothekeoma

C. Nodular fasciitis

D. Dermatofibroma

E. Epithelioid sarcoma

Correct choice: E. Epithelioid sarcoma

Explanation: Characteristic immunohistochemistry of epithelioid sarcoma is vimentin and


cytokeratin positivity. The other listed tumors do not stain both vimentin+ and cytokeratin+.

71. This disorder is characterized by what process?

A. Vascular damage caused by infiltration of neutrophils

B. Vascular fragility

767
C. Large vessel involvement

D. Delayed-type hypersensitivity reaction

E. The presence of amyloid

Correct choice: A. Vascular damage caused by infiltration of neutrophils

Explanation: Leukocytoclastic vasculitis is characterized by vascular damage caused by the


infiltration of neutrophils

72-Biopsy of this lesion shows palisading of histiocytes around a focus of necrobiosis and increased
mucin deposition in the upper dermis. Lymphocytes and fibroblasts are present around the palisades
as well. What is the most likely diagnosis?

1. Cutaneous sarcoidosis

2. Rheumatoid nodule

3. Cutaneous tuberculosis

4. Necrobiosis lipoidica

5. Granuloma annulare

Correct choice: E. Granuloma annulare

768
Explanation: Granuloma annulare (GA) typically presents as an annular, pink/red to violaceous,
non-scaly papule or plaque on the dorsal hands or feet. The classic biopsy findings for GA include:
mid dermal necrobiotic collagen center surrounded by palisading histiocytes, as well as fibroblasts
and lymphocytes; occasional foreign body giant cells, vasculitis and mucin. It is important to note
that other variants of GA (subcutaneous, interstitial, perforating) show a different histopathologic
picture.

Cutaneous sarcoidosis may present as a clinical mimic of GA, but biopsy typically shows "naked"
histiocytic collections (i.e. no surrounding lymphocytes) in the dermis without increased mucin
deposition. Biopsy of a rheumatoid nodule usually reveals fibrin deposition and necrobiosis often
seen in center of nodule surrounded by well developed palisading of histiocytes, vascular
granulation tissue, lymphocytes, plasma cells, eosinophils, variable giant cells, fibrosis in older
lesions, occasional giant cells. It is important to note that these findings are seen typically in
subcutaneous fat or soft tissue, although may extend to deep reticular dermis, whereas granuloma
annulare has more superficial findings. Cutaneous tuberculosis is distinguished from GA by the
presence of histiocytes surrounding areas of caseation necrosis, as well as the presence of acid-fast
bacilli. Lastly, necrobiosis lipoidica most often presents as atrophic, yellow depressed plaques with
telangiectasias and active inflammatory edges on the bilateral pretibial areas. A biopsy will show
palisading, necrobiotic granulomas consisting of a large confluent area of necrobiosis centered in
the superficial dermis and subcutaneous fat (this arrangement has been likened to the appearance of
a "layered cake").

73. Where is the classic location of this lesion?

A. Ear

B. Back

C. Leg

D. Along the milk line

E. Dorsal hand

769
Correct choice: D. Along the milk line

Explanation: This is an accessory nipple. You can see the ducts and the smooth muscle as well as
the papillomatous epidermis. It is most commonly located on the trunk along the milk line.

74. Which blistering disorder clinically and histologically appears as non-inflammatory bullae?

A. Porphyria cutanea tarda

B. Bullous pemphigoid

C. Pemphigus vulgaris

D. Linear IgA bullous dermatosis

E. Bullous lupus erythematosus

Correct choice: A. Porphyria cutanea tarda

Explanation: PCT clinically and histologically presents as non-inflammatory bullae.

75. This neoplasm occurs most frequently in what anatomic location?

A. Leg

B. Mucosa

C. Acral

D. Head and neck

E. Trunk

770
Correct choice: D. Head and neck

Explanation: Desmoplastic melanoma is spindle cell variant of malignant melanoma. It typically


occurs in the head and neck region in older individuals. It demonstrates a high recurrence rate but
lower rates of metastasis.

76. A four-year-old boy presents with a subcutaneous nodule on his cheek which often becomes
inflamed and red. This neoplasm (shown in photo) has demonstrated mutations in which gene
product, which is a component of a key signaling pathway that influences cell differentiation and
proliferation?

A. B-catenin

B. P53

C. Smoothened

D. Rb

E. PTCH

Correct choice: A. B-catenin

Explanation: Pilomatricoma is a benign neoplasm that demonstrates mutations in CTNNB1, the


gene that encodes B-catenin, a component of a key signaling pathway that influences cell
differentiation and proliferation and is generally present in matrical neoplasms. Pilomatricomas
present as a cyst with basaloid matrical cells that show an abrupt transition to central eosinophilic,
cornified matrical cells in which barely discernible nuclear outlines remain. Sometimes pink
trichohyaline granules are identified at transition points. Central anucleate cornified cells are
referred to as "ghost" or "shadow" cells.

2 – P53 is a tumor suppressor gene and functions primarily as a transcription factor. It is not
involved in pilomatricomas. 3 – Smoothened is a protein that is encoded by the SMO gene. It is a

771
class Frizzled G protein-coupled receptor that is a component of the hedgehog signaling pathway.
Mutations in SMO are detected in sporadic basal cell carcinomas in which PTCH1 is intact. 4 – Rb
gene encodes the retinoblastoma protein, a tumor suppressor protein that is dysfunctional in several
major cancers. One function of Rb is to prevent excessive cell growth by inhibiting cell cycle
progression until a cell is ready to divide. 5 – PTCH is a tumor suppressor gene that is involved in
basal cell carcinoma. Inactivating mutations in PTCH lead to dysfunctional Patched and this
stimulates Hedgehog binding and results in constitutive activation in downstream target genes.

77. The organisms highlighted by this GMS stain are?

A. Blastomycosis

B. Coccidioidomycosis

C. Mucormycosis

D. Candida albicans

E. Histoplasmosis capsulatum

Correct choice: A. Blastomycosis

Explanation: Blastomycosis: Gilchrist disease, Bird droppings or spores on wood along Mississippi
basin, incubation 33-44 days. Most infections develop in previously healthy people. In patients with
HIV, 46% will get CNS symptoms, which is 5-10x greater than non-HIV pts. 4 forms, pulmonary,
disseminated (CNS, bone, skin and GU) with secondary cutaneous lesions and primary inoculation
(rare-1-2 weeks after exposure-sporotrichoid pattern- mixed infiltrate numerous budding
organisms). Lesions more typically are verrucous or ulcerated plaques in males on exposed skin-
face. Histology: Pseudoepitheliomatosis hyperplasia overlying acute dermal inflammation usually
only a few organisms in center of abscess with thick walled 7-15 microns, single broad based bud,
has a WI-1 adhesion protein and are best seen with PAS or GMS.

772
Histologic examination of skin lesions demonstrates pseudoepitheliomatous hyperplasia,
suppurative and granulomatous inflammation, and round yeast forms with characteristic broad-
based budding and thick, double-contoured walls. This budding pattern helps to differentiate
blastomycosis from other fungal infections. Methenamine silver and PAS staining allow better
visualization of the fungi within giant cells and neutrophilic abscesses. Similar budding yeast forms
can be seen in sputum samples.

78. What is the diagnosis of this neoplasm in infants?

A. Infantile hemangioperictyoma

B. Tufted angioma

C. Kaposiform hemangioendothelioma

D. Dermatomyofibroma

E. Giant cell fibroblastoma

Correct choice: A. Infantile hemangioperictyoma

Explanation: Infantile hemangiopericytomas are benign lesions. They are multilobulate, with
perivascular and intravascular tumors outside the main tumor mass.

79. On which anatomic location does this neoplasm typically occur?

A. Scalp

B. Lower legs

C. Arms

773
D. Oral mucosa

E. Trunk

Correct choice: B. Lower legs

Explanation: Clear cell acanthomas tend to occur on the lower legs of middle-aged or and elderly
individuals.

80. What is the most likely diagnosis?

A. Steatocystoma

B. Epidermoid inclusion cyst

C. Eccrine Hidrocystoma

D. Bronchogenic cyst

E. Thyroglossal cyst

Correct choice: C. Eccrine Hidrocystoma

774
Explanation: This is an eccrine hidrocystoma (choice 3) as evidenced by the two layers of cuboidal
epithelium with eosinophilic cytoplasm. A steatocystoma typically has a "shark fin" cuticle. An
epidermoid cyst approximates the layers of the normal epidermis. Bronchogenic cysts have a
respiratory, ciliated lining. A thyroglossal cyst typically has pseudostratified columnar epithelium
and lymphoid follicles.

81. Treatment of a patient with chronic venous ulceration is best achieved by which modality?

A. endovenous laser ablation

B. ambulatory phlebectomy

C. sclerotherapy

D. observation

E. surgical excision and debridement

Correct choice: A. endovenous laser ablation

Explanation: Venous stasis ulcers often occur due to an incompetent superficial venous system
which creates pooling of blood in the ankles. Treatment of the atypical greater saphenous vein with
endovenous laser ablation, removes the atypical veins allowing the blood to return to the heart in
the deep system, relieveing the pressure in the venous system and thus allows the venous ulceration
to heal.

82. A lesion with the same pathology as in the photograph is located on the patient's penis. What is
the name for these penile lesions?

775
A. Pearly penile papules

B. Warts

C. Montgomery's tubercles

D. Glands of Zeis

E. Moll's glands

Correct choice: A. Pearly penile papules

Explanation: The pathology shows an angiofibroma. Angiofibromas on the penis are specifically
called pearly penile papules and are benign. The patient should be reassured.

83. Which of the following is commonly seen histopathologically in pityriasis rosea?

A. Absent spongiosis

B. Full thickness necrosis

C. Exravasated erythrocytes in the subcutaneous fat

D. Lichenoid lymphocytic infiltrate

E. Focal parakeratosis

Correct choice: E. Focal parakeratosis

Explanation: Histopathological features of Pityriasis Rosea include: mild subacute spongiotic


dermatitis, focal parakeratosis (corresponds to cigarette paper-like scale), perivascular lymphocytic
infiltrate, hemorrhage in papillary dermis, and sometimes a few necrotic keratinocytes. The other
answer choices are not commonly seen histopathologically in pityriasis rosea.

776
84. What is the most common symptom of this tumor?

A. Tingling

B. No symptoms

C. Itching

D. Burning

E. Pain

Correct choice: E. Pain

Explanation: This is an angiolipoma, one of the painful tumors. It is most commonly located on the
forearm and occurs in men more then women. Histologically you see mature lipocytes with
numerous thrombosed vessels.

85. In this disorder, colloid bodies are composed of which material?

A. Fibrin

B. T-cells

C. Apoptotic keratinocytes, IgM, and complement

D. Langerhans cells

E. Basement membrane

777
Correct choice: C. Apoptotic keratinocytes, IgM, and complement

Explanation: Colloid bodies are composed of apoptotic keratinocytes, IgM, and complement.

86. Psoriasis has been shown to be genetically linked to:

A. Pityriasis rosea

B. Clear cell acanthomas

C. Epidermodysplasia verruciformis

D. Verruciform xanthomas

E. Poromas

Correct choice: C. Epidermodysplasia verruciformis

Explanation: Mutations have been found in the EVER1 and EVER2 genes in epidermodysplasia
verruciformis; these genes are on 17q25, the location of the PSORS2 gene (one of the genes
implicated in the inheritance of psoriasis).

87. Psoriasis vulgaris may demonstrate all except one of the following histopathologic changes.

A. Munro microabscesses

B. Clubbing of the rete ridges

C. Spongiform pustules of Kogoj

D. Hypergranulosis

E. Suprapapillary thinning

Correct choice: D. Hypergranulosis

Explanation: Psoriasis is characterized by confluent parakeratosis (not focal); hyperkeratosis;


neutrophils in stratum corneum (Munro microabscesses) and in spinous layer (spongiform pustules
of Kogoj); hypogranulosis; suprapapillary thinning of epidermis (epidermis is very thin over dermal
papillar; regular acanthosis (rete ridges about same length), often with clubbed rete ridges; dilated

778
capillaries in dermal papillae (causes Auspitz sign of pinpoint bleeding if scale picked off);
perivascular lymphocytes.

88. Which of the following stains with Ulex europeus agglutinin I?

A. Smooth muscle

B. Eccrine glands

C. Endothelial cells

D. Macrophages

E. Melanocytes

Correct choice: C. Endothelial cells

Explanation: Ulex europeus agglutinin I is a stain which identifies endothelial cells, keratinocytes,
angiosarcomes and Kaposi's sarcoma.

89. What is the best description of the histopathology expected for the border of the lesion seen
here?

A. Regular elongated rete ridges with superficial small vessels

B. Epidermal spongiosis with atypical lymphocytes spreading into the epidermis

C. Perivascular lymphocytic infiltrate with prominent eosinophils

D. Hyperplasia of the superficial epidermis

E. Parakeratosis in a column over focal dyskeratotic or vacuolated keratinocytes

779
Correct choice: E. Parakeratosis in a column over focal dyskeratotic or vacuolated keratinocytes

Explanation: This is an image of porokeratosis. While the center of the lesion can have variable
histopathology (mild spongiosis or psoriasiform dermatitis), the raised border is represented
histologically with a column of parakeratosis over often dyskeratotic or vacuolated keratinocytes.

Regular elongated rete ridges is seen in psoriasis. Epidermal spongiosis with atypical lymphocytes
spreading into the epidermis can be seen in mycosis fungoides. Perivascular lymphocytic infiltrate
with prominent eosinophils can be seen with DRESS syndrome or other drug eruptions.
Hyperplasia of the superficial epidermis can be seen in other benign epidermal tumors.

90. What is the diagnosis?

A. Pigmented purpuric dermatosis

B. Livedoid vasculopathy

C. Hematoma

D. Leukocytoclastic vasculitis

E. Thrombotic vasculopathy

Correct choice: C. Hematoma

Explanation: A hematoma is defined by the presence of extravasated erythrocytes.

91. This type of lupus completely spares the epidermis.

780
A. acute cutaneous lupus

B. discoid lupus

C. chronic cutaneous lupus

D. chilblains lupus

E. tumid lupus

Correct choice E. tumid lupus

Explanation: Tumid lupus completely lacks any interface dermatitis and the epidermis is completely
spared. It is the most photosensitive of all the lupus types. There is lots of mucin on biopsy.

92. Which of the following types of acrospiroma consists of a variable composition of poroid,
squamoid, clear cells with ductal areas, may be solid/cystic, and is based in the deeper dermis?

A. Hidroacanthoma simplex

B. Poroma

C. Dermal duct tumor

D. Hidradenoma

E. None of the above

Correct choice: D. Hidradenoma

Explanation: Acrospiromas include hidroacanthoma simplex (located intraepidermally), poroma,


dermal duct tumor, hidradenoma (deeper dermis). A hidradenoma has a variable composition of
poroid, squamoid, clear cells with ductal areas; may be solid/cystic; and is based deeper in the
dermis than the other lesions listed.

781
93. A grenz zone is seen in which of the following conditions?

A. Granuloma faciale

B. Polyarteritis nodosa

C. Telogen effluvium

D. Sweet's syndrome

E. Erythema ab igne

Correct choice A. Granuloma faciale

Explanation: Granuloma faciale overlies a mix of neutrophils, eosinophils, lymphocytes,


histiocytes. GF often presents clinically as a reddish brown macule or plaque. It is characterized by
a Grenz zone with leukocytoclastic vasculitis with eosinophils benath this. Onion skin fibrosis can
be seen in chronic cases. Grenz zone differential: Lepromatous leprosy, cutaneous B cell
lymphoma, Chronic lymphocytic leukemia/small lymphocytic lymphoma, acute myelomonocytic
Leukemia, leiomyosarcoma, and Botryoid rhabdomyosarcoma.

94. A 55 year old male presents for evaluation of a slowly growing subcutaneous nodule on his
posterior neck. What is the most likely diagnosis?

A. Spindle cell lipoma

B. Angiolipoma

C. Piloleiomyoma

D. Metastatic adenocarcinoma

E. Cytophagic histiocytic panniculitis

782
Correct choice: A. Spindle cell lipoma

Explanation: This is a spindle cell lipoma (choice 1). These demonstrate “rope-like” collagen,
myxoid stroma, spindle-cells, mast cells, and mature fat. Angiolipoma show many blood vessels.
Piloleiomyoma is a tumor of muscle, which is not pictured. Adenocarcinoma would show "indian-
filing" of cells. Cytophagic histiocytic panniculitis shows macrophages phagocytizing other cells,
and characteristic "bean bag cells."

95. A middle-aged man presents with many 2-3 mm, firm, waxy closely aligned papules in a
widespread symmetrical pattern throughout his head and neck, upper trunk, hands, forearms and
thighs. The papules are arranged in a strikingly linear array. The surrounding skin is shiny and
indurated and the glabella has deep longitudinal furrowing. Deep furrowing also is present on the
trunk and extremities. A biopsy showed: mucin in the upper and mid-reticular dermis, increase
collagen deposition, and a marked proliferation of irregularly arranged fibroblasts. Which of the
following is the most likely diagnosis?

A. Scleroderma

B. Scleredema

C. Pachydermoperiostosis

D. Scleromyxedema

E. Nephrogenic systemic fibrosis

Correct choice:D. Scleromyxedema

Explanation: In scleromyxedema, numerous 2–3 mm firm waxy closely aligned papules develop in
a relatively widespread symmetrical pattern. The most common sites of involvement are the head
and neck region, upper trunk, hands, forearms and thighs. Papules are often arranged in a strikingly
linear array. The surrounding skin is shiny and indurated, i.e. sclerodermoid in appearance, and the
glabella is typically involved with deep longitudinal furrowing. Severe involvement of the face can
result in a leonine facies. Deep furrowing can also occur on the trunk and extremities and is referred
as the “Shar-Pei sign.”

Scleromyxedema is characterized by a triad of microscopic features:

- a diffuse deposit of mucin in the upper and mid reticular dermis

- an increase in collagen deposition

783
- a marked proliferation of irregularly arranged fibroblasts

1 -The primary differential diagnosis for scleromyxedema is systemic sclerosis (scleroderma) and
scleredema. The presence of papules, especially in linear arrays, is a very helpful clinical sign in
distinguishing scleromyxedema

2- Scleredema is a symmetrical diffuse induration of the upper part of the body due to a thickened
dermis and deposition of mucin.

3 - Pachydermoperiostosis (primary hypertrophic osteoarthropathy) is associated with facial


involvement, thickening of the skin on the hands and feet, and clubbing of the digits.

5 - Nephrogenic systemic fibrosis, which develops in individuals with renal impairment exposed to
gadolinium-containing contrast media, may show mucin in biopsy specimens, but patients lack both
facial involvement (commonly seen in scleromyxedema) and paraproteinemia.

96. A too superficial biopsy of this neoplasm could be misdiagnosed as which lesion?

A. Desmplastic trichoepithelioma

B. Cylindroma

C. Spiradenoma

D. Syringoma

E. Chondroid syringoma

Correct choice: D. Syringoma

784
Explanation: Superficial biopsies of microcystic adnexal carcinoma are sometimes misdiagnosed as
syringomas. Microcystic adnexal carcinomas demonstrate ductal structures and cords in a fibrous
stroma. They extend deep into the dermis and even subcutis.

97. Weibel-Palade bodies are seen in:

A. Endothelial cells

B. Spitz Nevi

C. Cells infected with MCV

D. Plasmacytoid Cells

E. Malakoplakia

Correct choice: A. Endothelial cells

Explanation: Weibel-Palade bodies are seen in endothelial cells and are therefore found in vascular
lesions. Kamino bodies are found in Spitz nevi. Henderson Patterson bodies are seen in molluscum.
Dutcher bodies are intranuclear inclusions seen in plasmacytoid cells. Michaelis Gutmann bodies
are partially digested bacteria seen in malakoplakia.

At the ultrastructural level, vascular endothelial cells are characterized by tight junctions between
neighboring cells and by specialized organelles, the rod-shaped Weibel–Palade bodies, that serve as
storage organelles for the coagulation factor von Willebrand factor, also known as factor VIII-
related antigen. Weibel–Palade bodies also contain P-selectin, angiopoietin-2, and endothelin-1.
Activation of endothelial cells by proinflammatory cytokines or pro-angiogenic factors readily leads
to translocation of these storage organelles to the cell membrane, with consecutive enhanced
membrane expression of P-selectin and release of von Willebrand factor. Other characteristic
ultrastructural features include caveolae (minute invaginations of the membrane surface),
pinocytotic vesicles, and the formation of vesiculo-vacuolar organelles. Circulating macromolecules
cross the endothelium through interendothelial cell gaps and transendothelial cell pores, some of
which arise from VVOs. Endothelial cell fenestrations, areas with direct apposition of the
endothelial cell membranes without intervening cytoplasm, are rarely seen in normal skin except in
angiogenic perifollicular blood vessels during the growth phase of the hair follicle. However,
fenestrated endothelial cells are frequently seen in skin diseases with pronounced angiogenesis and
vascular hyperpermeability, including psoriasis.

785
98. In this disorder which typically occurs on the head and neck, the cells would demonstrate which
staining pattern?

A. CD1a + and S100 +

B. CD1a- and S100 -

C. CD1a+ and S100-

D. CD1a- and S100+

E. CD68-

Correct choice: B. CD1a- and S100 -

Explanation: Benign cephalic histiocytosis is a benign eruption that typically occurs on the face of
children. It is a non-Langerhans cell histiocytosis; hence, it is CD1a and S100 negative but is CD68
positive.

99. A pregnant 33-year-old female presents with the shown painful lesions that appeared abruptly
one week ago. She additionally has been febrile to 38.3C with malaise and headaches. What do you
expect to find on biopsy of a representative lesion?

A. Perivascular neutrophils with fibrinoid necrosis and RBC extravasation

B. Apoptotic individual keratinocytes with intracellular spongiosis

C. Dense peri-eccrine gland neutrophilic infiltrate

D. Diffuse infiltration of malignant leukocytes in the dermis

786
E. Superficial dermal edema with dense neutrophilic infiltrate

Correct choice: E. Superficial dermal edema with dense neutrophilic infiltrate

Explanation: The clinical presentation most likely represents Sweet syndrome (acute febrile
neutrophilic dermatosis). Classical Sweet syndrome is frequently associated with recent infections,
IBD and pregnancy. Diagnosis is made with 2 major and 2 minor criteria.

Major criteria: 1. Abrupt onset of painful erythematous plaques 2. Histopathological evidence of a


dense neutrophilic infiltrate without evidence of leukocytoclastic vasculitis

Minor criteria: 1. Fever 2. Association with malignancy, inflammatory disease, pregnancy or recent
URI, GI infection, vaccination 3. Excellent response to systemic steroids 4. Abnormal lab values
(ESR, CRP, leukocytosis, >70% neutrophils)

Option A represents leukocytoclastic vasculitis. Option B represents erythema multiforme. Option C


is the pathology for neutrophilic eccrine hidradenitis. Option D represents leukemia cutis.

100. Histopathologically, a swiss cheese type appearance is characteristic to which of the


following?

A. Silica

B. Tattoos

C. Liquid silicone

D. Sea urchin spines

E. Zirconium

787
Correct choice: C. Liquid silicone

Explanation: Paraffinoma and silicone granuloma both have a swiss cheese type appearance in the
dermis to subcutaneous fat. The remaining options typically appear sarcoidal.

101. What is the most likely diagnosis?

A. Verrucous carcinoma

B. Keratoacanthoma

C. Basal cell carcinoma

D. bowen's disease

E. pilomatrichoma

Correct choice: A. Verrucous carcinoma

Explanation: Verrucous carcinoma is a large nedophytic condylomatous lesion. It is giant, well


differentiated and has a pushing margin.

102. Multiple pilomatricomas are seen in all of the following syndromes EXCEPT:

A. Gardner syndrome

B. Cowden syndrome

C. Turner syndrome

788
D. Rubinstein-Taybi syndrome

E. Myotonic dystrophy

Correct choice: B. Cowden syndrome

Explanation: Multiple pilomatricomas are seen in Rubinstein-Taybi, Gardner's (cyst-like


pilomatricomas), myotonic dystrophy, Turner's, sarcoidosis, sternal cleft and coagulation defects.
Cowden syndrome does not commonly demonstrate pilomatricomas.

103. The mutation that most likely resulted in this lesion is which of the following?

A. PTCH

B. CDKN2A

C. VEGF

D. P53

E. RET

Correct choice: D. P53

Explanation: P53 is the most common mutation in squamous cell carcinoma. This SCC is located on
the lip. PTCH is mutated in BCCs and CDKN2A is mutated in familial melanomas.

789
104. A healthy, full-term, 24-hour-old newborn girl develops a rash consisting of blotchy,
erythematous papules on the face, trunk and extremities. Which of the following would most likely
be seen on biopsy?

A. Perifollicular subcorneal pustules with eosinophils

B. Subcorneal pustules with neutrophils

C. Spongiosis and hyphae in the stratum corneum

D. Acantholysis and multinucleated cells

E. Abrupt ballooning of the upper one-third of the epidermis

Correct choice: A. Perifollicular subcorneal pustules with eosinophils

Explanation: This scenario most likely describeds erythema toxicum neonatorum, which would
show perrifollicular subcorneal pustules with eosinophils on biopsy. Subcorneal pustules with
neutrophils is consistent with transient neonatal pustular melanosis, which typically presents at
birth. Spongiosis and hyphae in the stratum corneum can be seen with dermatophyte infection.
Acantholysis and multinucleated cells can be seen with viral infection such as HSV. Abrupt
ballooning of the upper one-third of the epidermis is consistent with nutritional deficiency.

105. All are sebaceous glands except:

A. Tyson

B. Moll

C. Montgomery tubercle

D. Fordyce

E. Meibomian

Correct choice: B. Moll

Explanation: Moll’s glands are modified apocrine glands located on the eyelid. Sebaceous glands in
certain locations have an associated name (Tyson’s on the penis, Montgomery tubercle on the
areola, Meibomian on the eyelid, and Fordyce on the vermilion/oral mucosa).

790
106. What stain will more clearly reveal the organisms in the biopsy shown?

A. PAS

B. S100

C. CD68

D. CD3

E. Oil red O

Correct choice: A. PAS

Explanation: The histology shows an infiltrate of fungal hyphae. They will highlight with PAS and a
silver stain. This is aspergillus.

107. Eosinophilia-Myalgia syndrome is caused by:

A. Norwegian salt-petter

B. Unadultered Spanish grapeseed oil

C. Pb intoxication

D. L-Tryptophan

E. Excessive anaerobic exercise

Correct choice: D. L-Tryptophan

791
Explanation: The eosinophilia myalgia syndrome is characterized by marked peripheral eosinophilia
with a clinical spectrum of signs and symptoms, including generalized myalgias, pneumonitis,
myocarditis, neuropathy, encephalopathy and fibrosis. Many patients progress to a clinical picture
clinically indistinguishable from eosinophilic fasciitis. The disease is caused by the ingestion of
certain lots of L-tryptophan.

Eosinophilia-myalgia syndrome (EMS) was first identified in 1989, when approximately 1500
patients developed subacute onset of myalgias and peripheral eosinophilia.They then went on to
have chronic muscle, fascia, nerve, and skin involvement. The Centers for Disease Control and
Prevention (CDC) proposed a surveillance case definition that included the following:

• Incapacitating myalgias

• Peripheral eosinophil count greater than 1000/µL

• No evidence of infection (ie, trichinosis) or neoplasm

Studies quickly linked EMS to dietary supplements which contained L-tryptophan, a supplement
commonly used by patients with fibromyalgia syndrome (FMS), which was made using genetically
engineered bacteria. Specifically, the analysis found an impurity identified as 1’1’-
ethylidenebis[tryptophan] (EBT). This supplement was quickly removed from the market resulting
in swift resolution of the EMS epidemic, but unfortunately there were over 30 deaths in just 6
months. Since then, other causes of EMS have been implicated. Six other impurities in L-
tryptophan, including 3-(phenylamino) alanine (PAA), shares similar properties with 3-(N-
phenylamino)-1,2-propanediol (the chemical found in rapeseed oil and implicated in the 1981
Spanish toxic oil syndrome epidemic, have been associated with EMS.

108. Which class of medication is associated with the development of this neoplasm?

A. ACE inhibitors

B. Beta blockers

792
C. Protease inhibitors

D. TNF alpha inhibitors

E. Antifungals

Correct choice: C. Protease inhibitors

Explanation: Protease inhibitors have been associated with a lipodystrophy syndrome in some
patients with HIV, including the development of angiolipomas. Histologic features of an
angiolipoma include normal subcutaneous fat with the presence of thrombosed blood vessels.

109. This is a DIF image of a lesion of lichen planus (Anti-IgM ab). What does the image show?

A. Granular deposits at the dermal-epidermal junction

B. Shaggy deposits along the dermal-epidermal junction

C. Colloid/cytoid bodies

D. Caterpillar bodies

Correct choice: C. Colloid/cytoid bodies

Explanation: Colloid/cytoid bodies

110. Which of the following is true of Rosai-Dorfman disease?

793
A. S-100 negative, CD1a positive, not characterized by emperipolesis

B. S-100 positive, CD1a positive, not characterized by emperipolesis

C. S-100 positive, CD1a negative, characterized by emperipolesis

D. S-100 positive, CD1a positive, characterized by emperipolesis

E. S-100 negative, CD1a negative, characterized by emperipolesis

Correct choice: C. S-100 positive, CD1a negative, characterized by emperipolesis

Explanation: Rosai-Dofrman disease (also known as sinus histiocytosis with massive


lymphadenopathy) is characterized by large "fluffy" histiocytes that are S-100 positive but CD1a
negative, and is characterized by multinucleated giant cells, plasma cells, aggregates of
lymphocytes, and emperipolesis. There is debate as to whether the condition is related to herpes
type 6.

111. This lesion is often found in which anatomic location?

A. Skin folds

B. Acral sites

C. Scalp

D. Mucosa

E. Trunk

Correct choice: A. Skin folds

Explanation: This image demonstrates an infarcted skin tag. Skin tags are common in skin folds.

794
112. Which of the following pathologic markers is most likely to be negative in a case of
melanoma?

A. HMB45

B. MelanA

C. Keratin

D. S100

E. Vimentin

Correct choice: C. Keratin

Explanation: Melanomas uniformly stain negative for keratin, which helps differentiate between
Paget's/Extramammary Paget's Disease (keratin stain is positive) and pagetoid melanoma.

The remaining listed answer choices will be positive in melanoma. The only exception is cases of
desmoplastic/spindle cell melanoma, wherein HMB45 is negative.

113. The predominant location of the cleft in transient neonatal pustular melanosis is:

A. Dermal

B. Basement membrane zone

C. Basal keratinocytes

D. Suprabasal

E. Subcorneal/granular

Correct choice: E. Subcorneal/granular

Explanation: Transient neonatal pustular melanosis is an idiopathic pustular eruption of newborns,


mostly on the chest, that heals with hyperpigmentation. It is most common on pigmented
individuals. Histologically it presents as subcorneal pustules with eosinophils and neutrophils.

114. Which of the following have been found to develop within a nevus sebaceous?

795
A. Angiosarcoma

B. Merkel cell carcinoma

C. Syringocystadenoma Papilliferum

D. Syringoma

E. Hidradenoma Papilliferum

Correct choice: C. Syringocystadenoma Papilliferum

Explanation: Historically, the most common benign tumor to develop within a nevus sebaceous was
syringocystadenoma papilliferum, although the most recent literature indicates that trichoblastoma
arises more commonly. The most common malignant tumor to arise is a basal cell carcinoma.
Cutaneous leiomyosarcoma, verruca, sebaceoma, and squamous cell carcinoma have also been
reported to be found within a nevus sebaceous. There are no reports of angiosarcoma, merkel cell
carcinoma, syringoma, or hidradenoma papilliferum arising within a nevus sebaceous.

115. What is the best diagnosis?

A. Cholesterol emboli

B. Masson's Tumor

C. Calciphylaxis

D. Leukocytoclastic vasculitis

E. Processing Artifact

796
Correct choice: A. Cholesterol emboli

Explanation: Cholesterol Emboli: Typically occurs after a vascular procedure but can occur
spontaneously. Usually presents as livedo reticularis of distal lower extremities, associated with
eosinophilia and acute renal failure. Often need multiple deeper levels on sectioning a punch biopsy
to see the characteristic cholesterol clefts and fibrin thrombi in the lower dermis or subcutis. A high
index of suspicion is required especially in a patient with known atherosclerosis who has developed
the typical skin changes, kidney failure, abdominal pain or diarrhea, following a vascular procedure.

Definitive diagnosis is made from a skin biopsy or a biopsy of other involved tissue. The histology
of cholesterol emboli should show diagnostic cholesterol crystals or clefts within the blood vessel
wall, along with thrombi (blood clots). The clefts are spaces left by the crystals that have been
washed out by the tissue fixation.

Blood tests show an eosinophilia in up to 80% of affected patients. Other changes that may be seen
include: Raised white cell count and/or platelet count, Microscopic blood in the urine or stool,
Elevated erythrocyte sedimentation rate (ESR), Deranged kidney function tests, Raised amylase
levels, Decreased serum complement.

116. Which syndrome is associated with this lesion?

A. Proteus

B. Sturge- Weber

C. Parkes Weber

D. Klippel-Trenaunay

E. Cutis marmorata telangiectatica congenital

797
Correct choice: C. Parkes Weber

Explanation: Parkes Weber syndrome is associated with arteriovenous malformations. All other
listed syndromes are associated with capillary malformations.

117. On histologic examination of this adenocarcinoma of the perineal area, which of the following
stains would NOT be positive?

A. PAS

B. Mucicarmine

C. CEA

D. EMA

E. HMB45

Correct choice: E. HMB45

Explanation: Extramammary Paget's disease (EMPD) is a cutaneous adenocarcinoma of glandular


differentiation. Approximately 25% of EMPD cases are associated with underlying neoplasms.
They stain negative for HMB45, which is a melanocytic marker. Histologically, the Paget cells stain
positively with PAS, mucicarmine, CEA, EMA, LMW keratin.

118. Which of the following is correct regarding using histologic stains to differentiate between
primary extramammary Paget's disease and pagetoid metastasis to the skin from underlying colon
cancer?

A. Extramammary Paget's disease is carcinoembryonic antigen (CEA) negative.

B. Extramammary Paget's disease is HMB45-positive

798
C. Extramammary Paget's disease is gross cystic disease fluid protein-15 positive.

D. Pagetoid metastasis to the skin from underlying colon cancer is CK7-positive

E. Pagetoid metastasis to the skin from underlying colon cancer is CK20-negative

Correct choice: C. Extramammary Paget's disease is gross cystic disease fluid protein-15 positive.

Explanation: Primary extramammary Paget's disease is gross cystic disease fluid protein-15
positive, whereas pagetoid metastasis to the skin from underlying colon cancer is typically gross
cystic disease fluid protein-15 negative. Primary extramammary Paget's disease also is CK7-
positive and CK20-negative, whereas pagetoid metastasis to the skin from underlying colon cancer
is CK7-negative and CK20-positive. Both conditions typically show positive staining with CEA and
EMA, and negative staining with HMB-45 (which would be positive in Pagetoid melanoma).

119. What is the foreign material seen in this biopsy?

A. Gel foam

B. Splinter

C. Sculptra

D. Hyaluronic acid

E. Silicone

Correct choice: A. Gel foam

Explanation: This is gelfoam on histology, it typically has the deep purple color, is amorphous and
has jagged edges.

799
120. Which of the following is an immunhistochemical marker for Merkel Cell Carcinoma?:

A. S-100

B. Vimentin

C. HMB-45

D. Neuron specific enolase

E. TTF-1

Correct choice: D. Neuron specific enolase

Explanation: Neuron specific enolase stains merkel cells. Vimentin stains melanocytic lesions,
sarcomas and lymphomas. S-100 and HMB-45 stains melanocytic lesions, such as melanoma.
TTF-1 is positive in small cell carcinoma of the lung.

121. Goblet cells are seen in:

A. Cutaneous ciliated cyst

B. Endometriosis

C. Dermoid cyst

D. Bronchogenic cyst

E. Steatocystoma

Correct choice: D. Bronchogenic cyst

Explanation: Bronchogenic cysts have a pseudostratified cuboidal or columnar lining that is


ciliated; goblet cells are found in the lining as well.

122. The type of necrosis shown in the center of this lesion is which of the following?

A. Fat necrosis

B. Liquefactive necrosis

C. Fibrinoid necrosis

800
D. Caseous necrosis

E. Coagulative necrosis

Correct choice: D. Caseous necrosis

Explanation: Granulomas with central caseous necrosis is a classic histopathologic finding for
LMDF.

123. These lesions are common in the genodermatosis caused by which gene mutation?

A. PTEN

B. BRAF

C. NFKB

D. CYLD

E. ABCC6

801
Correct choice: D. CYLD

Explanation: Brooke-Spiegler syndrome is caused by a mutation in the CYLD gene. It is


characterized by the presence of spiradenomas, cylindromas, and trichoepitheliomas.

124. Which of the following statements about the direct immunofluorescence pattern in lichen
planus is correct?

A. The DIF is negative in the majority of cases

B. Deposition of IgG is within cytoid bodies in the superficial dermis

C. Shaggy deposition of fibrin occurs at the DEJ

D. Deposition of fibrinogen is within cytoid bodies in the deep dermis

E. There is prominent deposition of IgM within the spinous layer of the epidermis

Correct choice: C. Shaggy deposition of fibrin occurs at the DEJ

Explanation: Shaggy fibrin deposition at the DEJ, which is the single best indicator in the diagnosis
of LP, was found in 56% of cases according to one study evaluating the DIF fndings in patients with
LP. The DIF is positive in the majority of cases (75% according to one study). Deposition of IgM
and fibrinogen is within cytoid bodies in the superficial dermis. There is no deposition of IgM
within the spinous layer of the epidermis.

125. You examine a biopsy that is square on low power. The clinical history reports an IgG
paraproteinemia and the presence of a "doughnut sign". Which of the following conditions would
you think of without looking on higher power?

A. Scleromyxedema

B. Scleroderma

C. Scleredema

D. Localized Morphea

E. Pretibial myxedema

802
Correct choice: A. Scleromyxedema

Explanation: The doughnut sign is seen on proximal interphalangeal joints where a central
depression surrounded by an elevated rim can be seen. This is a feature of the skin thickening seen
in Scleromyxedema. The other options will have a square appearance on low power microscopic
examination and scleredema can have an IgG paraprotein. Other microscopic findings in
scleromyxedema include incrased fibroblasts with fibrosis/increased collagen and thickening of the
dermis. Mucin may be scant or absent. This biopsy is more cellular than pretibial myxedema. The
other options will have a square appearance on low power microscopic examination and scleredema
can have an IgG paraprotein. Other microscopic findings in scleromyxedema include incrased
fibroblasts with fibrosis/increased collagen and thickening of the dermis. Mucin may be scant or
absent. This biopsy is more cellular than pretibial myxedema.

126. When attempting to identify Langerhans cells in a specimen, which of the following markers is
most helpful?

A. CD1a

B. CD4

C. CD7

D. CD8

E. CD20

Correct choice: A. CD1a

Explanation: CD1a is a surface antigen specific for epidermal Langerhans cells. It is not expressed
on other epidermal structures. Another specific marker of Langerhans cells is CD207 (aka langerin).
The other characteristic ultrastructural feature of Langerhans cells is the Birbeck granule, which is
shaped like a tennis racket. CD4 is found on T-helper cells and occasionally on Langerhans cells.
CD7 is a T-cell marker that is often lost in cutaneous T-cell lymphoma. CD8 is found on cytotoxic
T-cells. CD20 is a B-cell marker.

127. Scalp biopsy of a 14-year-old female with suspected alopecia areata would likely reveal
inflammation around which portion of the hair follicle:

A. Dermal papilla

803
B. Hair bulb

C. Inner root sheath

D. Isthmus

E. Infundibulum

Correct choice: B. Hair bulb

Explanation: On biopsy, alopecia areata exhibits a peribulbar lymphocytic “swarm of bees.”

Discoid lupus typically exhibits inflammation surrounding the isthmus, along with a perivascular
dermatitis and vacuolar interface changes. Lichen planopilaris exhibits inflammation most densely
concentrated about the infindibulum.

128. The promontory sign is seen in:

A. Tufted angioma

B. Glomeruloid hemangioma

C. Spindle cell hemangioendothelioma

D. Acroangiodermatitis of Mali

E. Kaposi’s sarcoma

Correct choice: E. Kaposi’s sarcoma

Explanation: The promontory sign refers to the formation of new vessels around existing vessels
and adnexal structures. This is typically seen in Kaposi’s sarcoma. However, it is not
pathognomonic, because it has also been described in angiosarcoma and in benign vascular tumors.

129. What is this lesion called when it is located in the salivary gland?

A. Chondroid syringoma

B. Pleomorphic adenoma

C. Pororma

804
D. Basal cell carcinoma

E. Spiradenoma

Correct choice: B. Pleomorphic adenoma

Explanation: This is a chondroid syringoma or mixed tumor of the skin. When a chondroid
syringoma is in the salivary gland it is called a pleomorphic adenoma. They look almost identical
histologically, and are named differently due to their location.

130. A 14-year-old boy presents with brownish, verrucous, scaly skin with prominent flexural
involvement. A biopsy shows the findings above (pictured). Which of the following is the most
likely diagnosis?

A. X-linked ichthyosis

B. Bullous congenital ichthyosiform erythroderma

C. Lamellar ichthyosis

D. Erythrokeratoderma variabilis

E. Sjogren-Larsson syndrome

805
Correct choice: B. Bullous congenital ichthyosiform erythroderma

Explanation: Epidermolytic hyperkeratosis is also known as bullous congenital ichthyosiform


erythroderma and is autosomal dominant. It is characterized by brownish, verrucous, scaly skin with
a "corrugated cardboard" appearance to the skin, prominent flexural involvement, and vesicles and
bullae that may occur early in life. It is caused by a defect in keratin 1 and keratin 10 (suprabasal
keratins). Histologically, it demonstrates compact orthokeratosis and acanthosis, hypergranulosis,
and a distinct type of degeneration of keratinocytes known as “epidermolytic hyperkeratosis” which
appears as though the superficial epidermis is degenerated or “blown out by a shotgun.” Sometimes
there are intraepidermal blisters in the degenerated areas early in infancy.

1 – X-linked ichthyosis is a brownish scaly eruption that begins in early childhood in males. It is
associated with steroid sulfatase deficiency (STS gene). It does not have the same characteristic
histologic features as epidermolytic hyperkeratosis. 3 – Lamellar ichthyosis is an autosomal
recessive disorder characterized by severe thick plates of scale that resemble reptile scales with an
onset usually at birth. It may present as collodion baby and can involve the flexures, palms and
soles. It is associated with a mutation in keratinocyte transglutaminase. It does not have the same
characteristic histologic features as epidermolytic hyperkeratosis. 4 – Erythrokeratoderma variabilis
is a rare, autosomal dominant mutation in the genes for gap junction proteins GJB3 and GJB4, and
is characterized by onset within 1 year of age, polycyclic to annular migrating red scaly plaques and
often palmoplantar keratoderma. It does not have the same characteristic histologic features as
epidermolytic hyperkeratosis. 5 – Sjogren-Larsson syndrome is a rare autosomal recessive condition
caused by a mutation in ALDH3A2 gene for fatty aldehyde dehydrogenase. It is characterized by
seizures, retinitis pigmentosa, lamellar ichthyosis, mental retardation and spastic paresis. It does not
have the same characteristic histologic features as epidermolytic hyperkeratosis.

131. What is the diagnosis?

A. silicone

B. monsels

C. aluminum chloride

806
D. gel foam

E. tattoo

Correct choice: E. tattoo

Explanation: Tattoo is a pigment deposited deep in the dermis, and is whatever color the tatto itself
is. Gel foam is purple; silicone is a group of small bubbles; and monsels is more of a brown color
and is localized.

132. A 35-year-old man develops indurated, mildly erythematous plaques involving the face and
upper back. Biopsy reveals a perivascular and periadnexal lymphocytic infiltrate with no overlying
epidermal changes. Which histopathologic stain could be used to support a diagnosis of tumid
lupus?

A. Fontana-Masson

B. Giemsa

C. von Kossa

D. Colloidal iron

E. Osmium tetroxide

Correct choice: D. Colloidal iron

Explanation: Tumid lupus is characterized by increased mucin, and thus a mucin stain such as
colloidal iron may be used to accentuate these findings. Other disorders that may present with a
superficial and deep perivascular lymphocytic infiltrate without associated increased dermal mucin
include polymorphous light eruption, pernio, syphilis, and reactive lymphocytic infiltrate of Jessner,
among others. Giemsa stains the granules in mast cells.

133. A patient presents with a brown patch that has darker macules within it and this pathology.
What is the best diagnosis?

807
A. Nevus spilus

B. linchen nitidus

C. lichen striatus

D. nevus comedonicus

E. agminated nevus

Correct choice: A. Nevus spilus

Explanation: Nevus spilus is a tan macule/patch with darker smaller maclues scattered within it. On
pathology, you see an increased number of melanocytes in a lentigenous pattern along the DEJ with
increased nests in focal areas. In an agminanated nevus there are groups of brown macules on
normal skin. There is no background brown macule differentiating it from nevus spilus.

134. Which of the following stains would you expect to be positive in a normal eccrine unit?

A. S-100

B. Prussian blue

C. Giemsa

D. Verhoeff von Gieson

E. Steiner

Correct choice A. S-100

Explanation: Eccrine glands stain S-100 and CEA positive. The remaining stains would not be
expected to stain normal eccrine sweat glands. Prussian blue (Perl’s) is an iron stain which stains
iron or hemosiderin bright blue. Giemsa stains mast cell granules purple (heparin in the granules)
and can also be useful in staining in Leishmaniasis. Verhoeff von Gieson is a stain for elastic tissue

808
that stains blue-black. Steiner stain is a silver stin for spirochetes similar to a Warthin Starry or
Dieterle stain.

135. Which of the following is characteristic of lichen planus histopathologically?

A. Wedge-shaped parakeratosis

B. Irregular acanthosis

C. Absent interface changes

D. Parakeratosis

E. Absent granular layer

Correct choice:. Irregular acanthosis

Explanation: Histopathology of lichen planus: Acanthosis with wedge-shaped hypergranulosis,


irregular (sawtooth acanthosis), hyperkeratosis without significant parakeratosis, and basal
vacuolization (interface changes). In most cases, lichen planus is diagnosed by observing its clinical
features. A biopsy is often recommended to confirm or make the diagnosis and to look for cancer.
The histopathological signs are of a lichenoid tissue reaction affecting the epidermis.

Typical features include:

1. Irregularly thickened epidermis

2. Degenerative skin cells

3. Liquefaction degeneration of the basal layer of the epidermis

4. Band of inflammatory cells just beneath the epidermis

5. Melanin (pigment) beneath the epidermis

6. Direct immunofluorescent staining may reveal deposits of immunoglobulins at the base


of the epidermis.

136. This patient presented with a papulosquamous eruption involving his palms, soles and trunk.
He notes that three weeks ago he had a small lesion on his penis that healed within a week. What is
your most likely diagnosis?

809
a. Syphilis

b. haemophilus ducryi

c. LGV

d. HSV

e. psoriasis

Correct choice: A. Syphilis

Explanation: Secondary syphilis presents with a papulosquamous eruption that includes the palms
and soles. They may or may not remember the penile chancre as it is painless. Both haemophilus
ducryi and HSV are painful.

137. A biopsy was performed from the scalp of an elderly man what is the most likely diagnosis?

A. Metastatic renal cell carcinoma

B. Angiosarcoma

C. Carcinoid

D. Sebaceous Carcinoma

810
E. Clear cell hidradenoma

Correct choice A. Metastatic renal cell carcinoma

Explanation: Clear Cell Renal Carcinoma: Metastatic lesions are commonly located on the scalp.
The tumor itself is composed of cells with clear to slightly granular cytoplasm secondary to
increased glycogen and lipid. The tumors typically forms abortive tubes/ducts, cords or sheets of
cells. Immunohistochemical stains are + for EMA and CD10. Typically the tumor is very
vascular with scant stroma associated with extravasated RBC and hemosiderin. The differential
diagnosis includes clear cell hidroadenoma. This latter tumor is usually composed of a mixture
of components; solid areas composed of small poroid cells often with duct formation admixed
with clear cells and squamoid cells. The tumor can be solid or cystic or a combination of the two.
The large cystic spaces typically contain sialomucin. The stroma is delicate fibrovascular. The
tumor is + CAM 5.2, CEA, EMA, with glycogen and no lipid in the clear cells.

Skin metastases of renal cell carcinoma present as nodular, rapidly growing, round or oval-
shaped lesions, which can be of various colors ranging from normal skin color to a red-purple
color. Clinical presentation may be confused with hemangioma, basal cell carcinoma or pyogenic
granuloma. There was a similar appearance of hemangioma in our case. In histopathological
examination, atypical nucleated cells are expected to be seen in clear cell type. The nodular mass
is surrounded by the atrophic epidermis, and moderate lymphocytic infiltration can be observed.
Lesions should be considered xanthoma, xanthelasma, hidradenoma in the differential diagnosis.
The immunohistochemical examination provides a microscopic differential diagnosis.
Epithelioid membrane antigen, carcinoembryonic antigen, CD-10, renal cell carcinoma marker
are markers used to identify skin metastases of renal cell carcinoma.

138. Lipomembranous change is seen in:

A. Hibernoma

B. Cystic sebaceous adenoma

C. Sebaceous carcinoma

D. Mucocele

E. Sclerosing panniculitis

Correct choice: E. Sclerosing panniculitis

811
Explanation: Lipomembranous change is a non-specific histologic pattern that is most commonly
seen in lipodermatosclerosis, which is also known as sclerosing panniculitis; this condition may
be secondary to venous stasis.

139. What material is present in the dermis in this image?

A. Hyaluronic acid filler

B. Cartilage

C. Bone

D. Calcium

E. Urate

Correct choice: D. Calcium

Explanation: This image demonstrates calcinosis in the dermis.

140. Which of the following does NOT stain for mucopolysaccharides (mucin)?

A. Periodic-acid Schiff (PAS)

B. Trichome

C. Alcian blue

D. Colloidal iron

E. Mucicarmine

812
Correct choice: B. Trichome

Explanation: Trichome does not stain acid mucopolysaccharides. Trichome stains collagen blue or
green and muscle red depending on reagents used. PAS stains mucopolysaccharides red, alcian blue
stains mucopolysaccharides blue, colloidal iron stains mucin blue and mucicarmine stains mucin
red. PAS stains mucopolysaccharides red, alcian blue stains mucopolysaccharides blue, colloidal
iron stains mucin blue and mucicarmine stains mucin red.

141. The globi in this Fite-Faraco stain represent:

A. clumping of mycobacteria

B. degradation of mycobacteria

C. isolated bacilli

D. fragmentation of organisms

E. granular mycobacteria

Correct choice: A. clumping of mycobacteria

Explanation: In lepromatous leprosy, acid-fast bacilli are seen with the Fite stain, sometimes in
clumps called globi.

142. Merkel cell carcinomas are seen in older patients and present with a skin-colored to
erythematous or violaceous papule in a sun exposed distribution. The cytokeratin 20 is positive in:

A. 90% of cases

B. 70% of cases

813
C. 50% of cases

D. 30% of cases

E. 20% of cases

Correct choice: A. 90% of cases

Explanation: Patient that are diagnosed with Merkel cell carcinoma will test positive for cytokeratin
20 in 90% of the time with perinuclear dot pattern. It also tests positive in CAM5.2, NSE +,
chromogranin, and synaptophysin+. It is also thyroidd transcription factor-1 negative and s-100
negative.

143. The cytoplasmic granules seen in granular cell tumor are:

A. Phagolysosomes

B. Ribosomes

C. Mitochondria

D. Intermediate filaments

E. Vacuoles

Correct choice: A. Phagolysosomes

Explanation: The granules in granular cell tumor are phagolysosomes.

The granularity of the granular cell layer in epidermodysplasia verruciformis may be secondary to
increased ribosomes. Mitochondria fill the cells in hibernoma.

144. What would this lesion stain positively with?

814
A. Von Kossa

B. VVG

C. Oil red O

D. Thioflavin T

E. Desmin

Correct choice: D. Thioflavin T

Explanation: This is nodular amyloid which stains with thioflavin T and crystal violet. Oil red O
stains fat, elastin is stained by VVG, Von Kossa stains calcium and desmin stains muscle.

145. Which of the following stains would be helpful in the diagnosis of cryptococcosis?

A. Colloidal iron

B. Oil red O

C. Mucicarmine

D. Giemsa

E. Verhoeff von Gieson

Correct choice: C. Mucicarmine

Explanation: The yeast cytoplasm of Cryptococcus neoformans stains with PAS and methenamine
silver, while the the capsule stains with Alcian blue and mucicarmine.Colloidal iron stains mucin;
Oil red O stains fat; Giemsa stains mast cells and leishmaniasis; Verhoeff von Gieson stains elastic
tissue.

146. One month after starting carbamazepine for new onset epilepsy, your patient develops an
extensive eruption with peripheral eosinophilia and associated liver dysfunction. Which histologic
finding would be most suggestive of the rash based on the clinical history?

A. Perivascular lymphocytic infiltrate with mild interface changes

B. Atypical basaloid cells in the papillary dermis

815
C. Irregular acanthosis with saw-toothed rete ridges

D. Neutrophils in the stratum corneum

E. Multinucleated giant cells

Correct choice: A. Perivascular lymphocytic infiltrate with mild interface changes

Explanation: Drug Reaction with Eosinophilia and Systemic Symptoms (DRESS) - also termed
Drug-Induced Hypersensivity Syndrome (DIHS) is a syndrome of systemic illness related to a
medication. Anti-epileptic medications and a longer duration of use prior to reaction (here 4 weeks)
is typical. The histopathology is variable but most often shows a superficial perivascular
lymphocytic infiltrate with mild interface changes.

The other options are not seen in this kind of drug reaction;Atypical basaloid cells in the papillary
dermis are more concerning for a malignancy. Irregular acanthosis and saw-toothed rete ridges is
seen in lichen planus. Neutrophils in the stratum corneum may be seen in AGEP. Multinucleated
giant cells are seen in other scenarios such as histocytic reactions.

147. This lesion (choose CORRECT answer)

A. Is a hypertrophic scar

B. Is a leiomyoma

C. Is a Becker's nevus

D. Is a dermatomyofibroma

Correct choice: D. Is a dermatomyofibroma

816
148. Which type is most common in children?

A. Type 1

B. Type 4

C. Type 3

D. Type 2

E. Type 5

Correct choice: B. Type 4

Explanation: In PRP, type 4, circumscribed juvenile is the most common form in kids. Type 1 is
classic adult. Type 2 is atyical, chronic adult. Type 3 is classic juvenile, type 5 is atypical juvenile.

149. Psoriasiform dermatitis with irregular hyperkeratosis and alternating vertical and horizontal
ortho- and parakeratosis is distinctive of this disorder:

A. psoriasis vulgaris

B. guttate psoriasis

817
C. parapsoriasis

D. pityriasis rubra pilaris

E. cutaneous T-cell lymphoma

Correct choice: D. pityriasis rubra pilaris

Explanation: Psoriasiform dermatitis with irregular hyperkeratosis and alternating vertical &
horizontal ortho- and parakeratosis ("checkerboard pattern") is distinctive of pityriasis rubra pilaris.
The hair follicles are dilated and filled with a keratinous plug, while the "shoulder" of stratum
corneum surrounding the follicular opening frequently shows parakeratosis.

150. The pathology of this lesion demonstrates reticulate strands of the epidermis with clefting
between the strands and the stroma. What is the diagnosis?

A. Eccrine syringofibroadenoma

B. Fibroepithelioma of Pinkus

C. Squamous cell carcinoma

D. Reticulate seborrheic keratosis

E. Tumor of the follicular infundibulum

Correct choice: B. Fibroepithelioma of Pinkus

Explanation: This is a fibroepithelioma of Pinkus, which is a BCC. Therefore, it has clefting


between the reticulate strands and the stroma.

818
151. Which disease frequently demonstrates this finding?

A. Morphea

B. Atopic dermatitis

C. Langerhans cell histiocytosis

D. Rosacea

E. Dermatomyositis

Correct choice: E. Dermatomyositis

Explanation: Calcinosis cutis is a common occurrence in dermatomyositis, especially the juvenile


form of the disease.

152. Which of the following is characteristic of PLEVA?

A. Alternating orthokeratosis and parakeratosis

B. Red cell extravasation

C. Dermal mucin deposition

D. Fibrinoid necrosis of medium sized vessels

E. Numerous eosinophils and neutrophils

Correct choice: B. Red cell extravasation

819
Explanation: Pityriasis lichenoides et varioliformis acuta (PLEVA) has characteristic findings.
Parakeratosis, spongiosis with vacuolar alteration, papillary dermal edema, wedge shaped infiltrate,
and extravasated RBC's can all be seen. Occasionally, the infiltrate can include atypical lymphoid
cells. Eosinophils and neutrophils are more frequently seen in LyP. The other answer choices are not
characteristic of PLEVA.

153. Histologically, adenoma sebaceum represent which of the following lesions?

A. Neurofibromas

B. Angiofibromas

C. Collagenomas

D. Angiokeratomas

E. Smooth muscle hamartomas

Correct choice: B. Angiofibromas

Explanation: Adenoma sebaceum, fibrous papules and pearly penile papules all have similar
features histologically, presenting as angiofibromas. Features include atrophic epidermis with
patchy melanocytic hyperplasia and hyperkeratosis, vertically oriented collagen, increased
fibroblasts and blood vessels.

154. This condition frequently occurs on which part of the body?

A. Abdomen

B. Forearms

C. Scalp

D. Neck

E. Finger

820
Correct choice: B. Forearms

Explanation: Actinic purpura typically occurs in older individuals on sun-damaged, traumatized


skin such as the forearms.

155. What infectious agent is most likely responsible for a reaction of fibrin and antibodies which
help to prevent phagocytosis?

A. Nocardia

B. Ricketsii species

C. Actinomycosis

D. Candida albicans

E. Anthrax

Correct choice: C. Actinomycosis

Explanation: Hoeppli-Splendore reaction is characterized histologically by intensely eosinophilic


material consisting of fibrin and antibodies. Causes of the phenomenon include Actinomycosis
israelii, Staph aureus, Proteus, Pseudomonas and E. coli.

Nocardia, Ricketsii, Candida albicans, and anthrax are not responsible for a reaction of fibrin and
antibodies which help to prevent phagocytosis.

Actinomycosis is a chronic or slowly progressive infection caused by various bacterial species of


the Actinomyces genus, most commonly Actinomyces israelii. Actinomyces are normal inhabitants
of the mouth, gastrointestinal tract, and female genital tract, and do not cause an infection unless
there is a break in the skin or mucosa. Actinomyces also appear to require the presence of other
accompanying bacteria in order to cause disease.

821
The disease is characterised by the formation of an abscess and draining sinus tracts (small tunnels
which open onto the surface of the skin or mucous membranes and drain pus). The draining pus
contains yellow granules called sulphur granules. These are named from the colour of the granules,
not their content.

Actinomycosis is to be differentiated from actinomycetoma, which is a chronic infection of the skin


and subcutaneous tissue, usually involving the foot (see mycetoma). Actinomycetoma is caused by
different species of Actinomyces that are found in soil and plant material in tropical regions.

156. Which of the following is more commonly seen in lichen sclerosus et atrophicus compared to
morphea?

A. Inflammation and fibrosis in the subcutis

B. Marked edema in the papillary dermis

C. Minimal hydropic degeneration noted at the dermaldermal-epidermal junction

D. Epidermis relatively normal with no thinning of the rete ridges

E. No follicular plugging

Correct choice: B. Marked edema in the papillary dermis

Explanation: Lichen schlerosus et atrophicus displays edema of the papillary dermis more often
than morphea.

Morphea vs. Lichen schlerosus et atrophicus: Morphea has a relatively normal epidermis without
follicular plugging. Interface changes are rare. In the dermis, the collagen fibers appear
homogenized. Elastic fibers are preserved. The inflammation and sclerosis can extend into the
subcutaneous fat. LS&A has an atrophic epidermis with follicular plugging. There is often
prominent hydropic changes with clefting. There is often edema of the papillary dermis and elastic
fibers are often absent. The subcutis is uninvolved.

157. This is a culture of a fungus. What is it?

A. Rhizopus

B. Phaeohyphomycosis

822
C. Aspergillosis

D. Mucor

E. Absidia

Correct choice: A. Rhizopus

Explanation: Mucormycosis: Rhizopus the roots are directly under sporangia. This is # 1 cause of
localized and systemic disease due to mucormycosis. Absidia the roots are on either side or the
sporangia. Causes localized infections such as in burns, under dressings and also seen in IV drug
users. Mucor the roots are absent. Mucor more frequently causes disseminated lesions such as
rhinocerebral infections in diabetics with ketoacidosis or in patients with leukemia or neutropenia.
These organisms result in a necrotizing cellulitis or a plaque or nodule with a necrotic eschar. The
organisms invade the wall of blood vessels resulting in thrombosis and infarction. The organism in
tissue displays irregular branching at 90 degree (right angle) and the hyphae are broad ribbon-like
with no septae. The organism can be seen on H & E, but better with GMS.

158. The atypical T cells of lymphomatoid papulosis are most likely to stain positive for which
marker?

A. CD30

B. CD31

C. CD34

D. CD56

E. CD68

Correct choice: A. CD30

823
Explanation: Lymphomatoid papulosis is a T-cell lymphoproliferative disorder characterized by
recurrent crops of erythematous necrotic papulo-nodules. Histology reveals a wedge-shaped dense
mixed dermal infiltrate, including atypical T lymphocytes that stain positive for CD30 (Ki-1).

CD31 is typically positive in vascular neoplasms. CD34 positivity usually occurs in vascular or
fibrous neoplasms. CD56 is the archetypal phenotypic marker of natural killer cells but can actually
be expressed by many more immune cells, including alpha beta T cells, gamma delta T cells,
dendritic cells, and monocytes. CD68 is highly expressed by monocytes and tissue macrophages,
including Kupffer cells, microglia, histiocytes and osteoclasts, and to a lesser extent on dendritic
cells and peripheral blood granulocytes.

159. Which histologic subtype is most aggressive?

A. Solid

B. Nodular

C. Superficial

D. Morpheaform

E. All are equal

Correct choice: D. Morpheaform

Explanation: Morpheaform, micronodular and sclerosing are the most aggressive histologic
subtypes of basal cell carcinoma. Superficial is the least aggressive.

160. The patient just returned from a sunny vacation with this eruption. What is your diagnosis

824
A. rosacea

B. acute cutaneous lupus

C. phototoxic eruption

D. Porphyria cutanea tarda

E. seborrheic dermatitis

Correct choice: C. phototoxic eruption

Explanation: The epidermis is still a basket weave so this process happened acutely. There is no
lymphocytic infiltrate that would be expected in lupus. We are not on sebaceous skin as would be
expected for seborrheic dermatitis and rosacea. Porphyria cutanea tarta results in a subepidermal
split with festooning of the dermal papillae.

161. This tumor was excised from a verrucous yellow plaque on the scalp what is it?

A. Syringocystadenoma papilliferum

B. Eccrine acrospiroma

C. Hidradenoma Papilliferum

825
D. Tubular apocrine adenoma

E. Syringofibroadenoma

Correct choice: A. Syringocystadenoma papilliferum

Explanation: Syringocystadenoma Papilliferum: 33% arise in association with a nevus sebaceous,


10% may coexist with a BCC or trichoblastoma. A subset of cases has a deletion of 9q22 PTCH
gene and 9q21 (p16). Histologically they present as surface invagination of glandular cords
composed of one to two layers of cuboidal cells associated with a fibrovascular stroma with
numerous plasma cells. Cystic spaces may form within which are free floating islands which are
peripherally lined by cuboidal cells with an inner core composed of loose connective tissue, blood
vessels and numerous plasma cells.

Papillary syringocystadenoma (syringocystadenoma papilliferum) is distinctive for its wide


papillary fronds that are in continuity with the surface squamous epithelium. Syringocystadenomas
are only rarely cystic. The fronds are lined by a bilayer, with basal cuboidal cells and apical
columnar apocrine cells, and the cores of fronds virtually always contain a dense infiltrate of
lymphocytes and plasma cells. A plasmacytic infiltrate commonly accompanies neoplasms of
apocrine lineage, and thus the mere presence of plasma cells in a given lesion is not sufficient to
confirm a diagnosis of syringocystadenoma.

At low magnification, papillary hidradenoma (hidradenoma papilliferum) consists of a well-


circumscribed nodule within the dermis and usually lacks both a connection to the surface
epithelium and the prominent plasmacytic infiltrate of syringocystadenoma papilliferum. Both
tubules and papillary fronds are usually present, and broad elongated fronds with delicate fibrous
cores are usually identifiable. Both tubules and fronds are usually lined by a bilayer, with basal
cuboidal myoepithelial cells and apical apocrine cells, and the apical cells usually display a
conspicuous holocrine secretion. At least some of the basal layer cells represent authentic
myoepithelial cells with contractile function, and labeling of the myoepithelial layer through
immunohistochemical staining for actin filaments can sometimes be of diagnostic value in the
distinction from adenocarcinoma, which often lacks a complete myoepithelial layer.

162. When this lesion occurs in multiples, which disorder should be suspected?

A. Rombo syndrome

B. Multiple Familial Trichoepithelioma

C. Goltz

826
D. Brooke-Spiegler syndrome

E. Carney complex

Correct choice: D. Brooke-Spiegler syndrome

Explanation: Patients with Brooke-Spiegler syndrome often have multiple spiradenomas,


cylindromas, and trichoepitheliomas.

163. This lesion most commonly metastasizes to which location?

A. Scalp

B. Arm

C. Leg

D. Back

E. Nose

Correct choice: A. Scalp

827
Explanation: Renal cell carcioma most commonly metastasizes to the scalp.

164. What is your best diagnosis?

A. Lichen Trichophyticus

B. Secondary Lues

C. Lichen Nitidus

D. Scabies

E. Acral Hyperkeratosis

Correct choice: C. Lichen Nitidus

Explanation: A chronic inflammatory disease characterized by shiny, flat-topped, usually flesh-


coloured micropapules no larger than the head of a pin. Lesions are localized in the early stages,
found chiefly on the lower abdomen, penis, and inner surface of the thighs. Distribution may
become generalized as the disease progresses.

Although the clinical presentation will suggest the diagnosis, lichen nitidus usually requires a skin
biopsy to be confirmed. The histology of lichen nitidus is very typical and described as the ‘claw
and ball’ appearance:

• Focal dense lymphohistiocytic infiltrate in the upper dermis very close to the epidermis.

• Langhans giant cells are often present in the infiltrate.

• Rete ridges of the epidermis are elongated and ‘clutch’ the infiltrate.

828
• Red blood cells are seen just under the epidermis in the hemorrhagic/purpuric form.

• Eosinophilic dermal material with some cell nuclei may be seen within the epidermis in the
perforating variant.

165. Calciphylaxis is characterized by intimal fibrosis and medial vascular calcification (that can
become transmural) as well as transdifferentiation of vascular smooth muscle cells into osteoblast-
like cells; these changes plus thrombosis lead to ischemic necrosis of the skin and soft tissues.
Which of the following histological stains (shown below) would best highlight the medial vascular
calcification and assist with making the diagnosis?

A. Sudan black

B. Verhoff Van Geisen

C. Von Kossa

D. Trichrome

E. Alcian blue

Correct choice: C. Von Kossa

Explanation: Calciphylaxis is characterized by intimal fibrosis and medial vascular calcification


(that can become transmural) as well as transdifferentiation of vascular smooth muscle cells into
osteoblast-like cells; these changes plus thrombosis lead to ischemic necrosis of the skin and soft
tissues. Because calcium deposits may be subtle in H&E-stained sections, von Kossa and Alizarin
red stains are used to increase detection.

1- Sudan black stains lipids in granulocytes.

829
2- Verfoeff-van Gieson stains elastic fibers black and outlines elastic lamina of muscular arteries
and media of aorta. The background is trichrome.

4- Trichrome stains collagen blue.

5- Alcian blue is a common "routine" stain (not an immunohistochemical stain) to detect mucins.

166. Which of the following is true regarding pilomatricomas?

A. Calcification is rarely seen

B. It is a poorly demarcated tumor

C. It is comprised of shadow cells and basophilic cells

D. Mutations are found in the APC gene

E. Frequent malignant degeneration occurs

Correct choice: C. It is comprised of shadow cells and basophilic cells

Explanation: Pilomatricomas are comprised of germinative-matrical cells that are basaloid and
transition to "ghost" or "shadow" cells. Pilomatricomas are well-defined lobular tumors in the
dermis or subcutis. Calcification or ossification is frequently seen. Mutations are found in beta
catenin. Malignant degeneration into a pilomatrical carcinoma is rare.

167. Which gene mutation and/or amplifications are more commonly found in this type of
melanoma and mucosal sites than in melanomas on intermittently sun-exposed sites

A. NRAS

B. KIT

C. GNAQ

D. HRAS

E. PTEN

830
Correct choice: B. KIT

Explanation: KIT mutations and/or amplifications are more commonly found in melanomas located
on acral and mucosal sites than in melanomas on intermittently sun-exposed sites.

168. What is the neoplasm in this biopsy?

A. Chondroid syringoma

B. Papillary eccrine adenoma

C. Tubular apocrine adenoma

D. Eccrine acrospiroma

E. Trichoblastoma

Correct choice: A. Chondroid syringoma

Explanation: Chondroid Syringoma: Most commonly occurs in middle age males on the head and
neck. Histologically this is a well circumscribed dermal tumor composed of dilated ducts, hyaline or

831
plasmacytoid cells (myoepithelial cells), keratinous cysts, +/- cells that show hair matricial
differentiation occasionally with some areas of mature fat. All elements arise within a homogenous
bluish chondromyxoid matrix (+ Type II collagen, + acid MPS stains with Alcian blue/green (stains
at high and low PH) consistent with chondroitin sulfate). Focal calcification may occur.

Apocrine mixed tumors form a well circumscribed dermal mass. These distinctive tumors are
composed of both a mesenchymal component which is usually chondroid, and an epithelial
component. The cartilage is mature, with bland chondrocytes and a silver/grey matrix. The
epithelium is bland, and consists in most areas of two cell layers: a myoepithelial layer and a lining
luminal layer.

169. Which of the following histologic features is seen in aging skin?

A. Thickened dermal-epidermal junction

B. Increased mast cells

C. Increased number of terminal hairs

D. Fewer Langerhans cells

E. Increased sebum production

Correct choice: D. Fewer Langerhans cells

Explanation: Histologic features of aging epidermis include flattened dermo-epidermal junction,


occasional nuclear atypia, decrease in the number of melanocytes and Langerhans cells. Changes
that are present in the dermis include atrophy, decrease in fibroblasts, mast cells and blood vessels.

170. An infant presents with yellow-brown crusted papules associated with purpura in a seborrheic
distribution and with diaper involvement. On pathology, what stains would be positive?

A. CD207

B. CD117

C. CD11a

D. CD56

E. CD20

832
Correct choice: A. CD207

Explanation: The vignette describes Langerhans cell histiocytosis, which stains with langerin, or
CD207.

171. Which immunohistochemical stain would be positive in eosinophilic granuloma?

A. HMB-45

B. Cytokeratin 20

C. Congo red

D. Mucin

E. CD1a

Correct choice: E. CD1a

Explanation: Eosinophilic granuloma is a form of Langerhans Cell Histocytosis (LCH), previously


called Histiocytosis X. Eosinophilic granuloma is a localized, benign form which is more common
in males and generally affects the bones. All forms of LCH are characterized by the infiltration of
Langerhans cells on pathology, which stains for S-100, CD1a and contain cytoplasmic birbeck
granules.

172. A 35 year old man presents with this lesion shown in the photo on his hand. The patient
undergoes a biopsy for confirmation of the diagnosis. What finding is specific for this condition?

A. Wedge-shaped perivascular lymphocytic infiltrate with eosinophils

B. Fibrinoid necrosis and leukocytoclasia

C. Collapsed keratin bundles

D. Peforating elastin, parakeratosis

E. Epithelial cells with cytoplasmic vacuoles

833
Correct choice:E. Epithelial cells with cytoplasmic vacuoles

Explanation: This is a wart, which on histology has epithelial cells with cytoplasmic vacuoles
refered to as koilocytes. A. is seen with arthropod bites. B. is seen in cutaneous vasculitis. C. is in
Darier. D. Elastosis perforans serpiginosa.

173. What is the classic histologic finding?

A. Lichenoid infiltrate

B. Alternating ortho and parakeratosis

C. Mounds of parakeratosis

D. Coronoid lamella

E. Basaloid proliferations

Correct choice: D. Coronoid lamella

Explanation: This is disseminated superficial actinic porokeratosis. The classic histologic finding is
a coronoid lamella which corresponds to the raised edge of the lesion.

834
174. Histologically, this lesion shows plump, polygonal cells arranged in nests and fascicles with
granular cytoplasm. Which immunohistochemical stain would be positive?

A. CD31

B. Synaptophysin

C. Factor XIIIa

D. S-100

E. CD34

Correct choice: D. S-100

Explanation: Granular cell tumors are benign growths which typically occur on the tongue. They
are typically well-circumscribed, raised, firm nodules. Histologically, the cells have uniform nuclear
characteristics and granular cytoplasm due to presence of lysozyme. These tumors are PAS positive
and S-100 positive.CD31 stains positive in vascular neoplasms (e.g. angiosarcoma). Synaptophysin
stains positive in neuroendocrine neoplasms (e.g. Merkel cell carcinoma). Factor XIIIa stains
positive in dermatofibroma, xanthoma disseminatum, fibrous papule, atypical fibroxanthoma, and
xanthogranuloma. CD34 stains positive in vascular tumors (benign/malignant),
dermatofibrosarcoma protuberans, neurofibroma, epithelioid sarcoma, spindle cell lipoma, and
fibrous papule.

175. CK20 is useful in the diagnosis of:

A. Dermatofibrosarcoma

B. Dermatofibroma

C. Epitheloid sarcoma

D. Merkel cell carcinoma

835
E. Plasmacytoma

Correct choice: D. Merkel cell carcinoma

Explanation: CK20 is useful in diagnosis of Merkel Cell Ca. Dermatofibrosarcoma would stain
CD34+. Dermatofibroma would stain CD68 +. Plasmacytoma would stain CD138 + and CEDa.
Epithelioid sarcoma would stain CD34+.

176. What is this neoplasm?

A. Hibernoma

B. Malakoplakia

C. Myospherulosis

D. Liposarcoma

E. Pleomorphic lipoma

Correct choice: A. Hibernoma

Explanation: Hibernoma: This is a rare neoplasm that typically occurs in 30-40 year old males as a
slowly enlarging warm mass on scapular region, trunk, axilla or thigh. There are also myxoid &
spindle cell variants that occur on the posterior neck/shoulder. The lesion enhances with contrast on
CT and MRI fails to reveal fat septations which differs for that seen with other lipomas.
Abnormalities in 11q13 and loss of MEN1 gene on 10q22 have been noted. Histologically: The
lesion is composed of large polygonal adipocytes that have with multiple vacuoles and an
eosinophilic granular cytoplasm with a central nucleus and prominent nucleolus. These cells are

836
referred to as “Mulberry cells." Admixed among these larger adipocytes are smaller cells with a
granular cytoplasm, mature white fat and some times spindle cells.

177. A lichenoid infiltrate that surrounds eccrine glands is seen in:

A. Lichenoid drug rection

B. Lichen striatus

C. Lichen planus

D. Lichenoid purpura

E. Lichen planopilaris

Correct choice: B. Lichen striatus

Explanation: Lichen striatus is an uncommon inflammatory dermatitis seen most commonly in


children aged 5 to 15. It presents unilaterally along Blaschko's lines as raised, slightly scaly,
erythematous papules, which are often pruritic. These lesions typically regress spontaneously within
a year. The histopathologic features of lichen striatus include a superficial perivascular
inflammatory lymphohistiocytic infiltrate with rare plasma cells and eosinophils. There is a focal
lichenoid infiltrate in the papillary dermis with basilar vacuolar alteration and necrotic
keratinocytes. Spongiosis with exocytosis of lymphocytes can be seen in the epidermis. A specific
and distinctive feature of lichen striatus is the presence of an inflammatory infiltrate that surrounds
hair follicles and eccrine glands.

178. A 56-year-old Mexico-born man, currently residing in New Mexico, presents with a one-year
history of the rash shown. He also complains of distal extremity numbness. A punch biopsy is
performed and a special stain is performed and shown in the second image. Which of the following
stains were performed to best visualize the organisms found in this image?

837
A. Brown and Brenn

B. Giemsa

C. Periodic Acid-Schiff

D. Wade-Fite

E. Warthin-Starry

Correct choice: D- Wade-Fite

Explanation: The clinical scenario and image are most consistent with lepromatous leprosy. The
Wade-Fite stain is used to stain the acid-fast bacilli. The Fite stain highlights mycobacteria in
general but is specifically used to identify mycobacterium leprosum. The stronger acid used in the
Ziehl-Neelsen stain is deemed too harsh for M. leprae and the lipid in the cell’s membrane is
washed away, making visualization of the organism difficult. Fite uses a weaker acid in the
decolorization phase of the procedure, preserving the more delicate cell walls of the organism. The
Fite and the Ziehl-Neelsen methods share their positive acid-fast profiles; red is read as a positive
stain.

Brown and Brenn is a tissue Gram stain that is a practical method of identifying and differentially
staining bacteria. Giemsa stain is primarily used to stain mast cells, but can also stain certain
organisms-- spirochetes, protozoans, and cutaneous Leishmania in particular. Periodic Acid-Schiff
accentuates fungal cell walls. Warthin-Starry is used chiefly in the identification of spirochetes in
diseases such as syphilis, Lyme disease, and acrodermatitis chronica atrophicans. A positive stain
will reveal black spirochetes.

179. A 12-month old infant presents with bilateral plaques in the inguinal folds and erythematous
geometric plaques underlying pressure points from the diaper. A biopsy demonstrates retention of
basophilic keratohyaline granules within areas of parakeratosis in the stratum corneum. The stratum
corner is thickened and compacted with increased eosinophilic staining. Retained nuclei are present
throughout the keratin layer. What is the most likely diagnosis?

A. Inverse psoriasis

B. Hailey-Hailey disease

C. Darier disease

D. Pemphigus vegetans

E. Granular parakeratosis

838
Correct choice: E. Granular parakeratosis

Explanation: The characteristic feature of granular parakeratosis is an unusual form of


parakeratosis. The stratum corneum is thickened and compacted with increased eosinophilic
staining. Retained nuclei are present throughout this keratin layer, creating the parakeratosis. The
most unusual feature is the visible retention of basophilic keratohyalin granules within these areas
of parakeratosis. In the infantile form, bilateral plaques in the inguinal folds or erythematous
geometric plaques underlying pressure points from the diaper can be seen.

1 - Inverse psoriasis: The differential diagnosis of granular parakeratosis includes the most common
causes of intertrigo (e.g. seborrheic dermatitis, candidiasis, inverse psoriasis, erythrasma) as well as
Hailey–Hailey disease, Darier disease, and pemphigus vegetans. A biopsy will confirm the
diagnosis of granular parakeratosis and exclude inverse psoriasis.

2 - Hailey-Hailey disease: The differential diagnosis of granular parakeratosis includes the most
common causes of intertrigo (e.g. seborrheic dermatitis, candidiasis, inverse psoriasis, erythrasma)
as well as Hailey–Hailey disease, Darier disease, and pemphigus vegetans. A biopsy will confirm
the diagnosis of granular parakeratosis and exclude Hailey-Hailey disease.

3 - Darier disease: The differential diagnosis of granular parakeratosis includes the most common
causes of intertrigo (e.g. seborrheic dermatitis, candidiasis, inverse psoriasis, erythrasma) as well as
Hailey–Hailey disease, Darier disease, and pemphigus vegetans. A biopsy will confirm the
diagnosis of granular parakeratosis and exclude Darier disease.

4 - Pemphigus vegetans: The differential diagnosis of granular parakeratosis includes the most
common causes of intertrigo (e.g. seborrheic dermatitis, candidiasis, inverse psoriasis, erythrasma)
as well as Hailey–Hailey disease, Darier disease, and pemphigus vegetans. A biopsy will confirm
the diagnosis of granular parakeratosis and exclude Pemphigus vegetans.

180. A 15-year-old girl presents with recurrent crops of spontaneously regressing erythematous to
purpuric papules. The lesions are crusted and occasionally vesiculopustular. A biopsy demonstrates
a superficial perivascular interface dermatitis, and a denser infiltrate that is top-heavy and wedge-
shaped. Lymphocytes predominate in the infiltrate, although neutrophils are admixed. The
epidermis shows focal parakeratosis and evidence of damage ranging from edema to extensive
epidermal necrosis. There is extravasation of erythrocytes. What is the most likely diagnosis?

A. Pityriasis lichenoides chronica

B. Pityriasis lichenoides et varioliformis acuta

C. Arthropod bites

839
D. Pityriasis rosea

E. Lymphomatoid papulosis

Correct choice: B. Pityriasis lichenoides et varioliformis acuta

Explanation: Pityriasis lichenoides et varioliformis acuta is an uncommon idiopathic eruption


mostly of the trunk of older children and young adults. Usually there are many small lesions less
than 5 mm in diameter with variable morphology from scaly papules to crusted papulovesicles. It is
self-limited in months to years. Histologically it is characterized by focal parakeratosis, often with
scale crust, and a dense wedge-shaped infiltrate centered upon basal layer zone of the papule with
prominent lymphocytic exocytosis into the epidermis. Necrotic keratinocytes are often seen.
Spongiosis with intraepidermal vesicles is sometimes present along with liquefaction degeneration
of the basal layer. Extravasation of erythrocytes are often in the epidermis.

1 - Pityriasis lichenoides chronica: is a less acute, chronic form which is histologically similar,
except that there is less scale crust, fewer neutrophils, less spongiosis, fewer vesicles, and fewer
necrotic keratinocytes.

3 - Arthropod bites: These lesions are typically more common on the face and extremities. Usually
more eosinophils are present.

4 - Pityriasis rosea: is characterized by focal parakeratosis, spongiosis, and perivascular


lymphocytes. Occasionally there is dyskeratosis, mild acanthosis, and focal extravasated red blood
cells.

5 - Lymphomatoid papulosis: Histologically, atypical lymphocytes are present in lymphomatoid


papulosis.

181. The most likely diagnosis of this image is which of the following?

A. lipodermatosclerosis

840
B. sclerema neonatorum

C. erythema nodosum

D. lupus profundus

E. subcutaneous fat necrosis of the newborn

Correct choice E. subcutaneous fat necrosis of the newborn

Explanation: Subcutaneous fat necrosis of the newborn is easily recognized by the intense
inflamation, radially arranged eosinophilic crystals in the fat cells, and the foreign body giant cells,
in comparison to sclerema neonatorum, where there is no inflammation as the patient is too sick to
mount a response.

182. This tumor was excised from the scalp of an older woman, what is this neoplasm?

A. Cylindroma

B. Sebaceous carcinoma

C. Pilomatricoma

D. Basal cell carcinoma

E. Spiradenoma

Correct choice:A. Cylindroma

Explanation: Cylindroma: Most often seen in middle age females on the scalp. Multiple lesions can
be associated with trichoepitheliomas, spiradenoma and adenomas of parotid gland in Brooke
Spieler syndrome. Histologically this is a poorly circumscribed dermal tumor with multiple small
irregularly shaped basophilic islands often likened to puzzle pieces composed of a peripheral layer

841
of smaller darker cells, with cells that have increased eosinophilic cytoplasm centrally. Each island
is surrounded by a thick eosinophilic cuticle (type IV and VII collagen), with similar eosinophilic
hyaline droplets seen within the island.

Low power view of cylindroma shows a non-encapsulated tumor nodule arising from the dermis.
This is formed by multiple irregular tumor islands, distributed in an aptly named ‘jigsaw’ pattern.
Surrounding the tumor islands, and in discrete droplets within the nodules is a thick hyaline deposit.
Two populations of cells are noted to make up the tumor nodules. A smaller cell with a
hyperchromatic nucleus tending to the periphery, and larger cells with open nuclei throughout the
centre of the nodules.

183. A nodule is biopsied from a 70 year old woman's arm. The histology is shown here. What is the
diagnosis best stained with?

A. CD56

B. CEA

C. S-100

D. CD31

E. CK5-6

Correct choice: A. CD56

Explanation: Merkel cell carcinoma stains positive with neuron-specific enolase, epithelial
membrane antigen, CD56, neurofilament, synaptophysin, chromogranin, argyrophil. The most
widely and accepted marker is CK20 (perinuclear dot staining), and CK8/18/19 (CAM 5.2) is

842
another uniformly good choice. However, CD56 is more intensely positive (cell membrane
staining).

184. Which of the following body contains calcium:

A. Negri body

B. Lipschutz body

C. Michaelis-Gutman body

D. Guarnieri body

E. Negri body and Michaelis-Gutman body

Correct choice: C. Michaelis-Gutman body

Explanation: The Michaelis-Gutman body is a concentrically laminated spherical inclusion that


contains calcium that is seen within macrophages in malakoplakia. Other inclusion bodies that
contain calcium are Schaumann bodies and psammoma bodies. The Negri body is seen in rabies.
The Lipschutz body is an intranuclear inclusion seen in herpes. It is considered synonymous with
the Cowdry A body. The Guarnieri body is seen in smallpox.

Malakoplakia diagnosis is made on the characteristic histology of a biopsy or excision specimen of


the tumor. The diagnostic feature is the presence of von Hansemann cells containing Michaelis-
Gutmann bodies. These intracellular bodies stain positively for calcium and iron and, when fully-
developed, they resemble an owl's eye. Bacterial culture will determine which organism is involved.

185. This neoplasm tends to occur most frequently on which anatomic region?

843
A. Sun-exposed areas of the head and neck

B. Genital region

C. Acral sites

D. Oral mucosa

E. Extremities

Correct choice: A. Sun-exposed areas of the head and neck

Explanation: Lentigo maligna tends to occur most frequently in the head and neck region in sun-
exposed areas.

186. A patient presents with this path and erythema nodosum and iritis. What is their likely
diagnosis?

A. Lofgren Syndrome

B. Heerfordt Waldenstrom syndrome

C. darrier-roussy syndrome

D. blau syndrome

E. mikulicz syndrome

844
Correct choice: A. Lofgren Syndrome

Explanation: Lofgren syndrome makes you SMILE: Sarcoid, migratory polyarthritis, iritis, LAD
(hilar), EN, because you laugh and grin.

187. An increased number of miniaturized hairs are seen in:

A. Lichen planopilaris

B. Alopecia areata

C. Trichotillomania

D. Telogen effluvium

E. Lichen planopilaris and alopecia areata

Correct choice: B. Alopecia areata

Explanation: In alopecia areata, especially early stages, increased miniaturized catagen hairs can be
seen in addition to the finding of peribulbar lymphocytes resembling a swarm of bees. Lichen
planopilaris is a scarring alopecia in which vertical tracts of fibrosis are seen in place of follicles.
Trichotillomania displays follicular plugging, trichomalacia, pigmented casts, hemorrhage, and
increased catagen hairs on biopsy. Telogen effluvium is characterized by an increased number of
telogen hairs.

188. The predominant location of the cleft in cicatricial pemphigoid is:

A. Dermal

B. Basment membrane zone

C. Basal keratinocytes

D. Supra basal

E. Subcorneal/granular

Correct choice: B. Basment membrane zone

845
Explanation: Cicatricial pemphigoid is an autoimmune blistering disease that presents with ulcers,
blisters and erosions of mucosal surfaces, especially the eyes and mouth. The cleft in cicatricial
pemphigoid is found in the basement membrane zone/subepidermal as the antigens are usually
BPAg2, laminin 5 and alpha-6-beta-4 integrin. Direct immunoflourescence is identical to that of
bullous pemphigoid showing linear IgG and complement deposits in the basement membrane zone.

189. Caterpillar bodies are seen in:

A. Lipoid proteinosis

B. Amyloidosis

C. Porphyria cutanea tarda

D. Mucocele

E. Dyskeratosis congenital

Correct choice: C. Porphyria cutanea tarda

Explanation: Caterpillar bodies are thought to be type IV collagen. They are found along the BMZ
in PCT.

190. On histology, there is a pseudocyst within the dermis surrounded by fibrous pseudocapsule
with no epithelial lining. The center is composed of bluish myxoid material with overlying
hyperkeratosis. This describes a:

A. Mucous cyst

B. Focal mucinosis

C. Dermoid cyst

D. Granuloma annulare

E. Mucinous carcinoma

Correct choice: A. Mucous cyst

846
Explanation: This describes a digial myxoid cyst or mucous cyst. It is located overlying
osteoarthritis. It is usually benign with potential for recurrence. Mucin stains with colloidal iron or
alcian blue.

191. What exogenous material is shown here?

A. Calcium hydroxylapatite

B. Hyaluronic acid filler

C. Suture

D. Gelfoam

E. Poly-L-lactic acid

Correct choice:D .gelfoam

Explanation: Gelfoam is present in this tissue specimen. Gelfoam has a characteristic appearance
with slightly basophilic honeycomb or collapsed net-like deposits.

192. A 58-year-old man presents with grouped erythematous nodules located around his ears that
are occasionally painful and pruritic. A biopsy showed proliferations of capillary-sized vessels with
epithelioid endothelial cells surrounding larger, thick-walled vessels accompanied by eosinophils
and lymphocytes. Which is the most likely diagnosis?

A. Angiolymphoid hyperplasia with eosinophilia

B. Kimura disease

847
C. Sarcoidosis

D. Lymphoma cutis

E. Epithelioid hemangioendothelioma

Correct choice A. Angiolymphoid hyperplasia with eosinophilia

Explanation: Angiolymphoid hyperplasia with eosinophilia is characterized by benign angiomatous


nodules or plaques, often multiple and grouped. They are usually located in the head and neck,
especially around the ears. The lesions may be painful, pruritic or pulsatile. Histologically they are
characterized by proliferations of capillary-sized vessels with epithelioid endothelial cells
surrounding larger, thick-walled vessels, accompanied by eosinophils and lymphocytes. Although
rare instances of spontaneous regression have been reported, surgical excision is generally required.

2 – ALHE (angiolymphoid hyperplasia with eosinophilia) may resemble Kimura disease, which is
now considered to be a separate entity, is usually located on the posterior neck and histologically
has much larger lymphoid follicles. 3 – ALHE can resemble sarcoidosis clinically but has different
histological features. Sarcoidosis is characterized by “naked” non-caseating granulomas in the
dermis and subcutaneous tissues. 4 – ALHE can resemble lymphoma cutis clinically but has
different histological features. Lymphoma cutis shows a mixture of B and T lymphocytes with
benign immunohistochemistry. 5 – ALHE can resemble epithelioid hemangioendothelioma but is
distinguished histologically. Epithelioid hemangioendothelioma is characterized by poorly
canalized cords and nests of endothelial cells forming small intracytoplasmic vacuoles as a sign of
primitive vascular differentiation.

193. What is the most likely diagnosis?

A. tinea versicolor

848
B. tinea nigra

C. candida

D. sporothrixosis

E. chromomycosis

Correct choice: A. tinea versicolor

Explanation: Tinea veriscolor has the typical spaghetti and meatballs as seen here, the hypae and
spores. Tinea nigra is pigmented.

194. Which of the following is true regarding S-100 Protein?

A. S-100 protein is a basic protein that binds Ca2+ and Zn2+

B. It is not soluble in 100% ammonium sulfate at neutral pH

C. Can be detected in melanocytes and in Schwann cells

D. It is not useful in diagnosing of spindle cell melanoma & desmoplastic melanoma

E. It is not useful in diagnosing poorly differentiated cutaneous metastases

Correct choice: C. Can be detected in melanocytes and in Schwann cells

Explanation: S-100 protein is an acidic protein that binds Ca2+ and Zn2+. At a neutral pH, it is
soluble in 100% ammonium sulfate. It is useful in diagnosing both spindle cell melanomas,
desmoplastic melanomas, as well as poorly differentiated cutaneous metastases. It also stains
positively in neurofibromas and schwannomas.

849
195. You would expect this neoplasm to:

A. Be painful on palpation

B. Have a high metastatic rate and poor prognosis

C. Disseminate to the skin from an internal focus

D. Dimple on lateral palpation

E. Have mutations in the SHH/PTCH signaling pathway

Correct choice: A. Be painful on palpation

Explanation: Eccrine Spiradenomas are classically painful to palpation and are in the BLUE
ANGEL mnemonic for painful dermal nodules: Blue Rubber Bleb Nevus Syndrome Angiolipomas
Neuromas Glomus tumors Eccrine Spiradenomas Leiomyomas

196. This material is formed during which stage of wound healing?

A. Inflammatory

B. Proliferative

C. Remodeling

850
D. Contraction

E. Vasoconstrictive

Correct choice: B. Proliferative

Explanation: Granulation tissue forms during the proliferative phase of wound healing.

197. Stromelysin 3 is a negative marker for which of the following?

A. Dermatofibroma

B. Basal cell carcinoma

C. Dermatofibromasarcoma protuberans

D. Squamous cell carcinoma

E. Breast carcinoma

Correct choice: C. Dermatofibromasarcoma protuberans

Explanation: Stromelysin 3 is a negative marker which helps to distinguish dermatofibrosarcoma


protuberans from dermatofibromas. Stromelysin 3 is a metalloproteinase which is expressed tissue
remodeling. In a study performed by Cribier et.al. 100% of dermatofibromas stained positive
Stromelysin 3 (ST3) is a member of the metalloproteinase family, which is expressed in tissue
remodeling processes such as scarring, embryogenesis, or tumoral invasion.

198. What type of reaction is demonstrated here?

A. Connective tissue

B. Psoriasiform

C. Lichenoid

D. Foreign body

E. Neoplastic

851
Correct choice: D. Foreign body

Explanation: A foreign body reaction develops in response to the deposition of foreign material or
altered endogenous material in the dermis. Granulomatous inflammation with histiocytes and
multinucleate giant cells forms around the material. Tattoo pigment is present in this particular case.

199. On histology, there are large amounts of mucin deposited within reticular dermis, resulting in
separation of collagen bundles. Clinically is can be seen in the lower legs associated with Grave's
disease. This best describes:

A. Pretibial myxedema

B. Scleredema

C. Follicular mucinosis

D. Lichen myxedematosus

E. Scleromyxedema

Correct choice: A. Pretibial myxedema

Explanation: This best describes pretibial myxedema and can be associated with Grave's disease.
The skin is diffusely doughy with skin thickening with nonpitting indurated plaques and nodules
seen bilaterally. The skin surface is shiny and sometimes seen a peau d'orange appearance.

200. Sebaceous carcinomas commonly arise on the eyelid. A biopsy of an ocular sebaceous
carcinoma typically reveals:

852
A. uniform, eosinophilic cells with rare atypia

B. pagetoid cells in the epidermis

C. negative staining with Androgen Receptor (AR)

D. positive staining with Ber-Ep4

E. may be associated with overexpression of MLH-1 and MSH-2

Correct choice: B. pagetoid cells in the epidermis

Explanation: Periocular sebaceous carcinomas are histologically composed of lobules of cell that
extend deep in the dermis and subcutaneous tissue. The cells are pleomorphic, with vacuolated
cytoplasm with moderate atypia. A characteristic feature in ocular tumors is the pagetoid spread of
the tumor in the overlying epidermis. Immunostaining for the androgen receptor has been reported
as a method to determine sebaceous differentiation, and it is helpful in diagnosing poorly
differentiated sebaceous carcinomas, which may lack staining for EMA and other markers. Basal
cell carcinomas, nodular-type are typically positive for Ber-Ep4 while sebaceous tumors are almost
always negative for this marker. Sebaceous carcinomas may be associated with Muir-Torre
syndrome, which is characterized with loss of expression of MLH-1 and MSH-2.

201. A 42-year-old multiparous female develops hyperkeratotic papules involving the lower
abdomen. As expected, biopsy is consistent with a perforating disorder. Histopathologic findings
could be accentuated with what stain?

A. Fontana-Masson

B. Fite

C. von Kossa

D. Colloidal iron

E. Osmium tetroxide

Correct choice: C. von Kossa

Explanation: Perforating periumbilical calcific elastosis is most commonly seen in multiparous


African American females. Biopsy findings are similar to those of pseudoxanthoma elasticum, with
calcified elastic fibers extruding through the epidermis. Staining with von Kossa may be used to
accentuate these findings.

853
202. Mantle cell lymphoma is characteristically positive for which of the following?

A. CD10

B. CD23

C. CD138

D. Bcl-1

E. bcl-6

Correct choice: D. Bcl-1

Explanation: Bcl-1 (Cyclin D1) is a marker for mantle cell lymphoma. CD10, bcl-6, and bcl-2 are
markers for follicular cell lymphoma. Bcl-2 also stains normal T cells. CD23 is a marker for CLL/
SLL and is negative in mantle cell lymphoma. CD138 is a marker for plasma cells.

Centrocytic lymphoma, or mantle cell lymphoma (MCL), is characterized by a chromosomal


translocation t(11;14) (q13;q32) involving the bcl-1 locus on chromosome 11. Cyclin D1 is a cell-
cycle regulatory protein essential for G1-S transition and has been identified as a potential
transforming gene affected by the translocation. Cyclin D1 is involved in the pathogenesis of MCL
and can be exploited as a diagnostic marker in the differential diagnosis of B-cell lymphomas and in
the identification of MCL.

203. Granular cell tumors will stain positive for all of the following markers EXCEPT:

A. S100

B. PAS

C. PTAH

D. HMB-45

E. Neuron specific enolase

Correct choice: D. HMB-45

Explanation: Granular cell tumors are of neural origin, with the granules representing an
accumulation of lysosomes in the cytoplasm. Lesions present on the tongue in 25% of cases. The
granules stain positive with periodic acid-Schiff (PAS) staining and are resistant to diastase. Being

854
of neural origin, lesions also stain with S100 and neuron specific enolase. The cells would not react
with melanocyte markers, such as HMB-45.

204. What is the best diagnosis?

A. Psoriasiform

B. Dermatitis Herpetiformis

C. Angiolymphoid Hyperplasia with Eosinophilia

D. Lichen Nitidus

E. Ochronosis

Correct choice: C. Angiolymphoid Hyperplasia with Eosinophilia

Explanation: Angiolymphoid Hyperplasia with Eosinophilia(EPITHELIOID HEMANGIOMA)


consists of solitary or multiple benign cutaneous nodules comprised of immature and mature
vascular structures intermingled with endothelial cells and a varied infiltrate of eosinophils,
histiocytes, lymphocytes, and mast cells.

Angiolymphoid hyperplasia with eosinophilia is an apparently non-malignant, locally proliferating


lesion composed of channels of small blood vessels surrounded by lymphocytes and eosinophils
(these are two types of white blood cell). Angiolymphoid hyperplasia with eosinophilia is also
known as epithelioid or histiocytoid haemangioma.

Kimura disease is probably distinct from angiolymphoid hyperplasia with eosinophilia. In Kimura
disease the lesions are deeper-seated, with no initial overlying skin lesions. In angiolymphoid
hyperplasia with eosinophilia the lesions are smaller and characterised by thick-walled so-called
histiocytoid or epithelioid blood vessels.

855
Angiolymphoid hyperplasia with eosinophilia has been reported from many parts of the world but
appears to be particularly common in Japan.

205. What is this neoplasm?

A. Sclerotic fibroma

B. Angiofibroma

C. Keloid

D. Neurofibroma

E. Dermatofibroma

Correct choice A. Sclerotic fibroma

Explanation: Sclerotic Fibroma (circumscribed storiform collagenoma): This can occur as an


isolated lesion n the head or neck or upper extremities, but multiple lesions are associated with
Cowden's disease which is an AD disorder linked to a mutation of PTEN on chromosome 10p.
Histology: Dome shaped papule with an overlying attenuated epidermis. A well-circumscribed
nodule composed of thickened homogenous collagen that is arranged in whorls with a thumbprint or
grains of wood appearance is seen in the dermis. Overall the lesion exhibits low cellularity and the
elastic fibers are absent. The tumor stains positively with vimentin, Factor XIIIA and focally for
CD34.

206. Michaelis Gutmann bodies are seen histopathologically in which condition?

856
A. Granuloma inguinale

B. Lymphogranuloma venereum

C. Herpes genitalis

D. Chancroid

E. Malakoplakia

Correct choice: E. Malakoplakia

Explanation: Malakoplakia is a rare and chronic inflammatory condition typically associated with
bacterial infections of the urinary tract, although a wide variety of organs have been reported.
Cutaneous malakoplakia is a rarer entity that has predilection for the perianal and genital regions.
Malakoplakia has been associated with immunosuppression such as HIV, connective tissue diseases,
tuberculosis, sarcoidosis, and malignancy as well as developing in sites of surgical wounds and
irradiated cancerous tissue.

The clinical presentation of cutaneous disease is highly variable. Among the documented
presentations are papules, plaques, nodules, ulcerative lesions, or fistulas. These presentations can
mimic inflammatory, infectious, or infiltrative cutaneous disease, making the diagnosis on clinical
grounds challenging. Owing to the nonspecific presentation, histopathologic evaluation is vital to
establish this diagnosis. The presence of sheets of foamy macrophages (von Hansemann) containing
granular and concentrically lamellar intracytoplasmic inclusions (the Michaelis-Gutmann bodies)
can establish the diagnosis. These are best seen by von Kossa stain because of a peripheral calcified
zone surrounding a central sphere of calcium apatite. However, those bodies can also stain positive
with periodic acid–Schiff and Perl's (iron) stains.

207. You diagnose a patient with multiple epidermal tumors that on histopathology shows a fibrous
pink orb with epithelial strands coming through the follicular center. The patient most likely has a
mutation in which gene?

A. CYLD

B. p53

C. FLCN

D. RAS

E. ECM1

857
Correct choice: C. FLCN

Explanation: This describes a fibrofolliculoma, a tumor seen in Birt-Hogg-Dubé syndrome. This is


an autosomally dominant syndrome due to a mutation in FLCN - folliculin (Choice 3), a tumor
suppressor gene. Patients show these skin lesions and have an increased risk for renal cell
carcinoma, spontaneous pneumothoraces, pulmonary cysts, and medullary carcinoma of the thyroid.

CYLD is mutated in Brooks-Spiegler syndrome, characterized by multiple adnexal tumors often


located on the scalp. p53 is an important tumor suppressor gene, mutations of which lead to tumor
syndromes such as Li-Fraumeni syndrome. The p53 gene product is inhibited by the E6 gene
product of HPV, which is a major virulence factor of HPV. RAS is an oncogene which is
inappropriately activated in diseases such as Neurofibromatosis, leading to multiple tumors. ECM1
is a gene which is mutated in Lipoid proteinosis. Autoantibodies to ECM-1 are seen in Lichen
Sclerosus.

208. Parasited macrophages may be seen histopathologically in all of the following EXCEPT:

A. Histoplasmosis

B. Donovanosis

C. Rhinosporidiosis

D. Leishmaniasis

E. Penicilliosis

Correct choice: C. Rhinosporidiosis

Explanation: Rhinosporidiosis is due to infection with the aquatic protozoan Rhinosporidium


seeberi, which is found mainly in India, Sri Lanks, and Africa. It often presents after local traumatic
inoculation as painless papules involving mainly nasal mucosa with subsequent development of

858
hyperplastic red friable (raspberry-like) polyps. Histology shows characteristic giant sporangia with
thousands of endospores.

The differential for parasitized macrophages can be remembered using the mnemonic His GIRL
Penelope:

• His-Histoplasmosis

• GI-Granuloma inguinale (aka Donovanosis)

• R- Rhinoscleroma (not to be confused with Rhinosporidiosis)

• L- Leishmaniasis

• Penelope- Penicilliosis

209. An aquarium owner presents with an erythematous, indurated nodule on the right hand that is
tender and warm. The patient has a history of drug induced lupus when he used minocycline for
acne. What is the most serious adverse effect associated with the first line alternative treatment?

A. Sudden cardiac death

B. Idiopathic pulmonary fibrosis

C. Systemic lupus erythematosus

D. Acute renal failure

E. Pulmonary embolism

Correct choice: A. Sudden cardiac death

Explanation: In a patient with exposure to marine environments, M. marinum is a common cause of


an infection that presents with an erythematous, indurated nodule on the extremities. First list
treatment is minocycline. Second line treatment is clarithromycin. A serious adverse effect of
clarithromycin is sudden cardiac death, as shown in the CLARICOR trial.

210. Which of the following is sometimes used to stain BCC during Mohs micrographic surgery?

A. Carcinoembryonic Antigen

B. Factor 13A

859
C. Glial Fibrillary Acidic Protein

D. Desmin

E. Toluidine Blue

Correct choice: E. Toluidine Blue

Explanation: The correct answer is toluidine blue. Toluidine blue staining shows dark blue BCC
tumor cells and a magenta-colored stroma around the tumor cells. CEA is used to stain
adenocarcinoma, extramammary Paget\'s disease, and eccrine neoplasms. Desmin stains muscle.
GFAP stains neurologic cells including astrocytes and Schwann cells. Factor 13A is used to stain
dermatofibroma.

Toluidine blue stain can also be used because it is rapid and the purplish metachromatic stromal
staining helps to distinguish BCCs from adnexal structures.

Carcinoembryonic antigen:

Glial Fibrillary Acidic Protein: In histology, the GFAP stain is done to determine whether cells
contain glial fibrillary acidic protein, a protein found in glial cells. It is useful for determining
whether a tumour is of glial origin.

Factor 13A: Fibrohistiocytic marker; Marker of fibrohistiocytic proliferations; Marker of dermal


dencrocytes; Differentiate dermatofibrosarcoma protuberans (CD34+, D2-40-, Factor XIIIa-) from
dermatofibroma (CD34-, Factor XIIIa+, D2-40+)

Desmin: Common stain used in panels for mesenchymal markers. Confirms myogenic origin of
tissue / tumors. Helps differentiate smooth muscle tumors (desmin+) from GIST (c-kit+, almost
always desmin-)

211- This tumor is best visualized using PAS stains with and without diastase what is it?

A. Clear cell acanthoma

B. Seborrheic keratosis

C. Bowen's disease

D. Tricholemmoma

E. Inverted follicular keratosis

860
►A

Clear cell acanthoma is composed of pale staining keratinocytes that have increased glycogen
content. The increased glycogen in this tumor is due to a defect in phosphorylase. Histologically the
keratinocytes are mildly enlarged and pale and distinctly separated from the surrounding epidermis.
The epidermis appears focally expanded by an acanthotic plate like growth that spares the follicular
epithelium. Also associated with this tumor are PMNs that extend into the epidermis. There may be
edema of the papillary dermal and some telangiectasis. PAS stain with and without diastase
highlights the abundant glycogen.

212- A 30-year-old man presents with multiple cysts, and requests excision of a cyst on his back.
After the excision, the specimen is sent for histopathologic evaluation. The report indicates a cystic
structure lined by stratified squamous epithelium showing foci of pilomatrical differentiation. What
is the next best step in management of this patient?

A. No further treatment or work-up is needed.

B. Re-excise the lesion to ensure it has been completely removed.

C. Closely monitor the lesion as these are high risk for recurrence.

D. Refer the patient for colonoscopy.

E. Perform a full body skin exam to exclude any cutaneous malignancies.

Correct choice: D. Refer the patient for colonoscopy.

Explanation: The pathology describes an epidermal inclusion cyst with pilomatrical differentiation,
which can be seen in Garder sydrome. Therefore the next best step would be to refer the patient for
colonoscopy to rule out malignancy because these patients develop premalignant polyps that
inevitably become malignant.

Patients often develop skull osteomas, which can be seen by Xray but is not an urgent matter as the
patient should have a colonoscopy first. The epidermal inclusion cyst itself is not at risk of
becoming malignant or recurring. The patient is not at high risk for cutaneous malignancy therefore
a full skin exam is not urgently needed. Although a CT of the abdomen may show a malignancy in
the colon, a colonoscopy would be a superior test and allow for biopsy and removal of any lesions
necessary. Multiple pilomatricomas have been associated with myotonic dystrophy (muscle
wasting/weakness/contraction), however this patient does not have a pilomatricoma he has an EIC
with pilomatrical differentiation.

861
213- The large cells are most likely to stain positively for which marker?

A. CD30

B. S100

C. Neuron specific enolase

D. CD20

E. Amyloid

Correct choice: A. CD30

Explanation: The image is of lymphomatoid papulosis with the "lumps of coal" cells in the dermis.
The histologic architecture can be similar to PLEVA but with wedge-shaped infiltrate and atypical
lymphocytes. Eosinophils and neutrophils are also intermixed. These cells are CD30+.

S100 can be positive in melanoma and Langerhans Cell Histiocytoses. Neuron specific enolase is
positive in merkel cell, carcinoid tumors, and melanoma. CD20 is positive in Primary Cutaneous
Follicle Center Lymphoma. Amyloid is positive in Amyloidosis

214- A patient presents with a few of these lesions and some hyperkeratotic papules on his anterior
shins. What diagnositic test should the patient undergo?

A. Colonoscopy

B. Cystoscopy

C. Bronchoscopy

862
D. Cortisone stimulation test

E. Otoscopy

Correct choice: A. Colonoscopy

Explanation: The image is a sebaceous adenoma, and the question describes keratoacanthomas on
the anterior shin, thus this patient should undergo colonoscopy to evaluate for colon cancer. Given
this constellation of symptoms one must evaluate for Muir-Torre syndrome.

215- This is a DIF image of a case of lichen planus (anti-fibrin ab)


? What does the image show?

A. Shaggy deposition of fibrin

B. Granular deposits in dermal papillae

C. Linear deposits of IgG at DE junction

D. Vascular deposition

E. Dermal mucin

863
Correct choice: A. Shaggy deposition of fibrin

Direct immunofluorescence of lichen planus reveals immunoglobulin (mainly IgM), complement,


and fibrin staining of colloid bodies in the deeper epidermis and superficial dermis. Although the
findings are characteristic of LP, direct immunofluorescence of classic LP is not necessary

Lichen planus - histologic features

Compact hyperkeratosis, usually no parakeratosis except when rubbed or oral

Hypergranulosis, often wedge-shaped

Irregular acanthosis with “saw-toothed” rete ridges rarely epidermal atrophy

Colloid bodies often

Liquefaction degeneration of the basal layer

Lichenoid lymphocytes in the papillary dermis

Melanin incontinence often

216-Which of the following lesions demonstrates a pseudo-Darier‘s sign?

A. Mastocytoma

B. Spitz nevus

C. Smooth muscle hamartoma

D. Pilomatricoma

E. Bullous pemphigoid

►C

Smooth muscle hamartomas are benign tumors which arise from smooth muscle of the dermis.
Pseudo-Darier's sign may be elicited due to transient piloerection after rubbing. Histologically, red-
orange bundles and fascicles are present with blunt-ended nuclei.

217-Multiple dermatofibromas are seen in:

A. Cowden's

864
B. Lobomycosis

C. Incontinentia pigmenti

D. Lupus erythematosus

E. Reticulohistiocytosis

►D

Multiple dermatofibromas are seen in lupus erythematosus and immunosuppression/HIV.

218-What diagnosis is shown here?

A. Lipodermatosclerosis

B. Erythema nodosum

C. Pancreatic panniculitis

D. Lupus profundus

E. Erythema induratum

►B

On histology, erythema nodosum is characterized by a septal panniculitis with septal thickening and
granulomatous inflammation.

219-Granular cell tumors are derived from:

865
A. Connective tissue

B. Smooth muscle

C. Vascular tissue

D. Neural tissue

E. Adipose tissue

►D

40% of granular cell tumors occur on the tongue. They appear well-circumscribed, raised, firm
nodules. Histologically, the cells are plump and polygonal arranged in nests and cords. Cells are
filled with fine granules representing lysozymes. The tumors are neurally derived and stain with
S-100 and PAS.

220-Langerhans cells express or are characterized by all of the following except:

A. Chromagranin

B. HLA-DR

C. CD1a

D. Birbeck granules

E. S-100

►A

Chromagranin stain neuroendocrine cells, Merkel cellcarcinomas and eccrine glands. They do not
stain Langerhans cells.

221-Which test is the first to become reactive?

A. RPR

B. FTA-ABS

C. VDRL

866
D. ELISA

E. Lumbar puncture

►D

In syphilis, the first test to become reactive is the ELISA, therefore it is the test of choice for early
primary syphilis and congenital disease. It is less useful in late disease due to decreased IgM. FTA-
ABS is the most sensitive with late primary syphilis.

222-The green color in is secondary to:

A. Stromelysin

B. Chloracetate

C. Fumarase

D. Myeloperoxidase

E. Alkaline phosphatase

►D

Chloromas are greenish tumor grossly secondary to involvement of the skin in acute granulocytic
leukemia. The green color is secondary to myeloperoxidase.

223-Which gene mutation and/or amplifications are more commonly found in this type of
melanoma and mucosal sites than in melanomas on intermittently sun-exposed sites

A. NRAS

B. KIT

C. GNAQ

D. HRAS

E. PTEN

867
►B

KIT mutations and/or amplifications are more commonly found in melanomas located on acral and
mucosal sites than in melanomas on intermittently sun-exposed sites.

224-The predominant location of the cleft in linear IgA is:

A. Dermal

B. Basement membrane zone

C. Basal keratinocytes

D. Suprabasal

E. Subcorneal/granular

►B

Linear IgA bullous dermatosis is an autoimmune disorder that presents with tense blisters along red
annular rings (like a string of pearls). Antibodies are found in the lamina lucida against LAD-1
antigen in anchoring filaments. The cleft in linear IgA would be found in the basement membrane
zone/subepidermal with direct immunostaining IgA in a linear pattern at the dermal-epidermal
junction.

225- B Cell lymphoma, leg type demonstrates which of the following characteristics?

A. EBV+

B. CD20+, often MUM1/bcl-2+

C. HHV8+

D. CD20-

E. CD4+

►B

B Cell lymphoma, leg type is characterized by a diffuse dermal infiltration of CD20+ cells which
are also usually positive for MUM1/IRF4 and bcl-2.

868
226-The special stain that can show maturation of dermal component and decreases expression with
descent into the dermis in a normal melanocytic lesion is:

A. HMB45

B. Ki67

C. Melanin A

D. MART-1

E. S100

►A

In a normal proliferating melanocytic nevus, HMB45 shows maturation of the dermal component
with decrease expression with descent into the dermis. Ki67 shows low proliferation in the dermis.

227-Which of the following stains with Ulex europeus agglutinin I?

A. Smooth muscle

B. Eccrine glands

C. Endothelial cells

D. Macrophages

E. Melanocytes

►C

Ulex europeus agglutinin I is a stain which identifies endothelial cells, keratinocytes, angiosarcomes
and Kaposi's sarcoma.

228-A skin biopsy shows numerous fibroblasts with fibrosis and thickening of the dermis. There is
sparse mucin deposition and on low power the biopsy appears square. Which of the following
paraproteins would you expect to find in this patient?

A. IgG lambda

869
B. IgA

C. IgM

D. IgG kappa

E. IgA gamma

►A

The description of the biopsy above is that of scleromyxedema. These patients have an associated
IgG lambda paraprotein. Other IgG subtypes may occur but lambda is most common.
Scleromyxedema is a subset of lichen myxedematosus (papular mucinosis). It will present with
coalescent erythematous to yellow papules and plaques. The most common location is the face, but
will occur in many other locations. The surrounding skin is usually sclerodermoid in appearance.
(Bolognia, p648-9)

229- This is a plakin:

A. BPAg1

B. Plakoglobin

C. Plakophilin

D. Desmocollin

E. Beta-catenin

►A

Desmoplakins include desmoplakin 1, BPAg1, envoplakin, and periplakin.

230-Hypopigmented Mycosis Fungoide:

A. Occurs in older patient populations rather than Classical MF

B. Occurs in women more commonly than men

C. Is more likely to occur on the face

870
D. Does not differ from classic MF in terms of those affected

E. Is more likely to occur in patients with skin types V and VI

►E

Hypopigmented MF occurs more commonly in younger patients and patients with darker skin
types.

231-The inclusions in infantile digital fibromatosis stain for trichrome and:

A. Phosphotungstic acid hematoxylin

B. Osmium tetroxide

C. Thioflavin T

D. Bodian

E. Pentahydroxy flavanol

►A

Osmium tetroxide stains fat. Thioflavin T stains amyloid. The Bodian stain is for nerves.
Pentahydroxy flavanol is a fluorescent stain for calcium.

232-A 22-year-old man with a diagnosis of Noonan‘s syndrome presents with a slowly enlarging
nodularity on the tongue. Biopsy is most likely to reveal what finding:

A. Psammoma bodies

B. Pustulo-ovoid bodies

C. Russel bodies

D. Verocay bodies

E. Weibel-Palade bodies

►B

871
The most likely diagnosis is a granular cell tumor, which most commonly arises on the tongue and
is associated with Noonan‘s syndrome. Biopsy would reveal intracellular granules along with
pustule-ovoid bodies representing phagolysosomes. Psammoma bodies are concentrically laminated
calcified bodies seen in meningioma, ovarian, and thyroid neoplasms. Russel bodies are
immunoglobulin inclusions in plasma cells often seen in Rhinoscleroma. Verocay bodies are
palisading nuclei arranged in rows with peripheral eosinophilic cytoplasm characteristic of
Schwannomas. Weibel-Palade bodies are organelles that are seen on macroscopy of endothelial
cells.

233-Which of the following would suggest a diagnosis of malignant lymphoma, B-cell type over
pseudolymphoma, B-cell type?

A. Mixed-cell infiltrate

B. Mitotic figures only in germinal centers

C. Histoicytic giant cells episodically

D. Larger lymphocytes predominate

E. Prominant vasculature with thick-walled blood vessels lined by plump endothelial cells

►D

Histological findings in pseudolymphoma, B-cell type include a mixed-cell infiltrate (important


diagnostic criterion), small lymphocytes (that predominate), the presence of histiocytes and giant
cells, mitotic figures limited to the germinal centers and prominant vasculature.

234-Degeneration of cartilage in chondrodermatitis nodularis helices affects which type of


collagen?

A. Type I collagen

B. Type II collagen

C. Type III collagen

D. Type IV collagen

E. Type VII collagen

872
►B

Chondrodermatitis nodularis helices presents as a tender nodule usually on the helix or antihelix of
the ear. It is thought to be a result of vascular insufficiency. Degeneration of the cartilage is seen on
histopathology which is primarily composed of type II collagen.

235-Paraproteinemia is associated with all except:

A. Scleromyxedema

B. Necrobiotic xanthogranuloma

C. Scleredema

D. Sclerosing panniculitis

E. Plane xanthoma

►D

Sclerosing panniculitis (lipodermatosclerosis) displays characteristic changes in the fat


(lipomembranous change); it is not associated with paraproteinemia. Generalized plane xanthomas,
scleromyxedema, necrobiotic xanthogranuloma, scleredema, erythema elevatum diutinum,
xanthoma disseminatum, and pyoderma gangrenosum have all been associated with a
paraproteinemia.

236-This is considered to be the juvenile counterpart of DFSP:

A. Juvenile hyaline fibromatosis

B. Giant cell fibroblastoma

C. Myxofibrosarcoma

D. Myofibromatosis

E. Plexiform fibrous histiocytoma

►B

873
An entity called giant cell fibroblastoma is CD34-positive, is mostly seen in male children on the
neck/trunk, and is thought to be a juvenile counterpart of DFSP.

237-What type of lesion is shown here?

A. Chondroid syringoma

B. Hidradenoma

C. Trichoblastoma

D. Cylindroma

E. Spiradenoma

►E

On histology, a spiradenoma is a dermal nodule composed of basaloid cells in a trabecular pattern.


Two cell types are usually present: small basaloid cells and larger cells with pale nuclei.

238-A patient has a yellow to pink papule with epitheloid melanocytic cells with nests that are large
and associated with retraction artifact. The nest and cell sizes diminish with depth. Also,
eosinophilic cytoplasmic extracellular globules are seen in the junctional region and are called:

A. Kamino bodies

B. Cytoid bodies

C. Nuclear bodies

D. Spitz bodies

874
E. Von Kossa bodies

►A

The above lesions describes a spitz nevus. They occur most commonly in children and seen on the
face, trunk, or extremities. An atypical spitz nevus is regarded as having an uncertain malignant
potential. Kamino bodies are seen in Spitz nevus that are eosinophilic cytoplasmic extracellular
globules in the junctional region.

239-A patient with a blue-red discoloration of the nail plate reports that the same finger becomes
very tender when exposed to the cold. You suspect a:

A. Periungual verruca

B. Glomus tumor

C. Pterygium

D. Mucous cyst

E. Pyogenic granuloma

►B

Glomus tumors are tumors of the arterio-venous anastamosis of the digital dermis. They occur most
frequently in the nail bed. The commonly have a bluish-red discoloration and may be tender or
painful with exposure to heat or cold.

240-Eccrine glands are found on the:

A. Labia majora

B. Labia minora

C. Glans

D. Prepuce

E. Vermilion

875
►A

Eccrine glands are present all over the body except on the vermilion of lips, glans, labia minora, nail
beds, and inner prepuce.

241-There is a red-yellow, smooth firm lobulated papule that is associated with Muir-Torre
syndrome on the face of a patient. On histology, there are lobulated sebaceous tumors often with a
connection to the epidermis. There are mature sebocytes (>50%) within a predominantly basaloid
cell population. This describes a:

A. Sebaceoma

B. Sebaceous adenoma

C. Sebaceous carcinoma

D. Sebaceous hyperplasia

E. Basal cell carcinoma with sebaceous differentiation

►A

This describes a sebaceoma and is located on the head and neck on the face and scalp. It is
associated with Muir-Torre syndrome with a mutation in the MSH2 and MLH1. It is a benign tumor
and surgical excision is curative with little chance of recurrence.

242-Comma-shaped bodies are seen in:

A. Benign cephalic histiocytosis

B. Malakoplakia

C. Sarcoidosis

D. Lipoid proteinosis

E. Gaucherǁs

►A

876
Comma-shaped bodies and worm-shaped bodies can be seen in a variety of histiocytoses and are
non-specific. The most common association is with benign cephalic histiocytosis.

243-The most common cause of a longitudinal groove of the nail plate is:

A. Myxoid cyst

B. Verruca vulgaris

C. Psoriasis

D. Onychomycosis

E. Paronychia

►A

A myxoid cyst is not a true cyst as it is devoid of cyst lining. They usually occur in an acral location
and are the most common cause of a longitudinal groove of a nail plate.

244-What neoplasm is pictured here?

A. Trichoadenoma

B. Tumor of the follicular infundibulum

877
C. Cutaneous lymphadenoma

D. Desmoplastic trichoepithelioma

E. Syringoma

►D

Desmoplastic trichoepitheliomas are composed of small cords and islands of basaloid cells within a
fibrous stroma. Keratinous cysts are often present.

245-Calcium can be seen this stain:

A. Von Kossa

B. Mucicarmine

C. Grocott

D. Warthin Starry

E. Melanin A

►A

Calcium is stained with Von Kossa and it stains it black. Mucicarmine stains mucin red. Grocott
stains fungus cell walls black and Warthin-Starry stains spirochetes black.

246-On histology this disease has mucin deposition within the external root sheath and sebaceous
gland. The mucin distends into the intracellular spaces within follicular epithelium forming cystic
cavities. The following stains will be positive except for:

A. CD3

B. CD4

C. CD7

D. CD8

E. CD9

878
►E

This patient has follicular mucinosis. This is benign and seen in children and is a nonscarring
alopecia and may be pruritic or tender seen on the face and scalp. All the following stains will be
positive CD3, CD4, CD7, CD8.

247-Which of the following stains Natural Killer Cells?

A. CD1a

B. CD11

C. CD30

D. CD56

E. CD68

►D

CD56 stain Natural Killer Cells and angiocentric lymphomas.

248-What substance is found histologically in talon noir to be the source of the pigment seen
clinically?

A. Hemoglobin

B. Hemosiderin

C. Melanin

D. Carbon

E. Keratin

►A

Talon noir is caused by trauma and hemorrhage trapped in the stratum corneum. It can clinically
appear pigmented and is often biopsied to rule out melanoma. Because phagocytosis of
extravascular RBCs and subsequent degradation of hemoglobin to hemosiderin does not occur in

879
the stratum corneum, traditional iron stains do not work and histochemical stains must be directed
toward hemoglobin.

249-A male patient presents with scattered pink-purple papules on the lower legs. He is otherwise
healthy. What is your diangosis based on this path?

A. pigmented purpuric lichenoid dermatosis of gougeroit and blum

B. mastocytosis

C. kaposi sarcoma

D. lichen nitidus

E. mycosis fungoides

►A

Pigmented purpuric lichenoid dermatosis of gougeroid and blum is easily identified on pathology
by a lichenoid infiltrate with deep melanophages and a few necrotic keratinoctytes. It can look
similar to EM but is more localized and the melanophages are much deeper in the pigmented
purpuric dermatosis. Kaposi sarcoma is on the differential clinically, however on path it is not a
lichenoid infiltrate and therefore is easily ruled out.

250-This was excised from the face, what is this neoplasm ?

A. Syringoma

880
B. Trichoadenoma

C. Basal cell carcinoma

D. Desmoplastic trichoepithelioma

E. Microcystic adenocarcinoma

►A

Syringomas on the eyelids, cheeks, chests and can be eruptive and increased numbers in Down's
syndrome and a clear cell change has been associated with diabetes. Histologically this i s a dermal
tumor consisting of eccrine ducts, lined by two cell layers sometimes creating a tad pole
appearance, there can also be small basaloid islands or strands of cells. Within the lumens of the
ducts is eosinophilic material, the lumens are CEA+. The tumor is located superficially and fails to
display perineural extension as is seen in MAC. The glands are associated with a fibrous sclerotic
stroma, usually no (or minimal) keratin cysts or foreign body granulomas as seen in desmoplastic
trichoepithelioma. Enlargement of the glandular cells more of a clear or vacuolated cytoplasm can
be seen in patients with diabetes.

251-CK20 stains this tumor in what type of pattern?

A. Perinuclear dot

B. homogeneous

C. speckeled

D. double dot

E. centromere

►A

Merkel cell carcionma is stained by CK20 in a perinuclear dot pattern.

252-In a halo nevus, there is diffuse lymphohistiocytic infiltrate with melanocytes. There are dermal
melanophages and mild dermal fibrosis. The marker used to label the lymphocytes but not
melanocytes is:

881
A. Ki-67

B. Melanin A

C. S100

D. CD1A

E. MART-1

►A

Ki-67 labels the lymphocytes but not the melanocytes in a halo nevus. It usually presents with a
brown papule with a white depigmented rim and the back is the most common site.

253-What is the diagnosis?

A. Lichen planus-like keratosis

B. Malignant melanoma

C. Syphilis

D. Spitz nevus

E. Halo nevus

►E

A halo nevus is characterized by a symmetric proliferation of nevus cells in the dermis, surrounded
by an intense lymphocytic infiltrate.

882
254-Which genetic abnormality has been found in the cells of this neoplasm?

A. t(17;22)

B. BRAF V600E mutation

C. NRAS mutation

D. t(9;22)

E. KIT mutation

►A

Dermatofibrosarcoma protuberans has been associated with reciprocal translocations of


chromosomes 17 and 22, t(17;22). This rearrangement fuses the collagen type I alpha 1 (COL1A1)
and the PDGF-beta chain (PDGFB) genes.

255-Which of the following sets of special immunohistochemical stains would help differentiate an
atypical fibroxanthoma (AFX) from a malignant fibrous histiocytoma (MFH)?

A. CD74 and CD99

B. CD34 and Stromelysin-3

C. HMB45 and p75NPR

D. CK20 and TTF-1

883
E. CK20 and GCDFP-15

►A

CD74 (LN2) and CD99 help differentiate an AFX from an MFH where an AFX is CD74 -,CD99+
and an MFH stains weakly from CD99 and is CD74 negative. CD34 and Stromelysin-3 differentiate
dermatofibromas and DFSPs. HMB45 and p75NPR are stains helping to differentiate melanomas
from desmoplastic melanomas. CK20 and TTF-1 staining differentiates a merkel cell carcinoma and
metastatic small cell lung carcinoma. CK20 and GCDFP-15 staining helps distinguish primary and
secondary (assoc with underlying neoplasm) Paget's disease.

256-Subcutaneous panniculitis-like T-cell lymphoma with an indolent course is positive for which
of the following?

A. CD4

B. CD8

C. CD10

D. CD41

E. CD57

►B

There seems to be two subsets of subcutaneous panniculitis-like T-cell lymphoma. One has an
indolent course, and is often CD8-positive and is positive for the alpha-beta T-cell receptor. The
other has an aggressive course, sometimes with evidence of systemic hemophagocytosis and high
mortality. This subset is CD56-positive and positive for the gamma-delta T-cell receptor.

257-All have been associated with increased risk of breast cancer except:

A. Cowden‘s

B. Multicentric reticulohistiocytosis

C. Peutz-Jeghers

D. Birt-Hogg-Dube

884
E. ataxia telangiectasia

►D

Birt-Hogg-Dube is associated with renal cancer and thyroid cancer. Female carriers of a mutated
ATM (homozygous mutations ATM cause ataxia telangiectasia) have an increased risk of breast
cancer.

258-This eruption is mediated by antibodies to which structure?

A. Desmoglein 1

B. Desmoplakin

C. Collagen VII

D. BP180

E. Desmoglein 3

►D

Bullous pemphigoid is most often associated with antibodies to BP180.

259-Which marker helps to differentiate extranodal NK-T cell lymphoma from cutaneous gamma
delta lymphoma?

885
A. CD56

B. MUM-1

C. EBV status

D. CD43

E. bcl-2

►C

Both cutaneous gamma delta lymphoma and extranodal NK-T cell lymphoma are CD56+.
However, only extranodal NK-T cell lymphoma is EBV + while cutaneous gamma delta lymphoma
is EBV negative. Both these lymphomas have a 5-year survival rate that approaches 0%.

260-Birt-Hogg-Dube has been associated with increased risk of renal cancer and:

A. Spontaneous pneumothorax

B. Hepatic cysts

C. Adrenal hemorrhage

D. Cirrhosis

E. Gastrointestinal polyps

►A

The gene defect in Birt-Hogg-Dube is folliculin.

261-This lesion typically occurs in which population?

A. Children on the legs

B. Middle-aged on the face

C. All ages on the trunk

D. Elderly on the head and neck

886
E. Infants on the trunk

►D

Atypical fibroxanthomas develop in elderly patients primarily on the highly sun-damaged skin of
the head and neck. The prognosis is usually very good despite the pleomorphic histo logy which is
characterized by very bizarre, atypical spindle-shaped and anaplastic cells with numerous mitoses.

262-Which of the following is characteristic of pleomorphic lipoma?

A. Foreign body giant cells

B. Floret giant cells

C. Frequent mitoses

D. Exocytosis of lymphocytes

E. Virchow bodies

►B

Pleomorphic lipomas characteristically have a mixture of variably sized fat cells with a varying
number of pleomorphic enlarged cells. These cells contain nuclei arranged in a circumferential
pattern that has been termed floret cells. Rare lipoblasts are found. Focal collections of lymphocytes
and plasma cells within the tumor are seen.

887
263-What material is found inside multinucleated giant cells in this condition?

A. Calcium

B. Red blood cells

C. White blood cells

D. Collagen fibers

E. Elastic fibers

►E

Annular elastolytic giant cell granuloma presents as multiple papules and annular plaques. On
histology, there is granulomatous inflammation with multinucleated giant cells demonstrating
elastophagocytosis

264-What infection does this patient have?

888
A. Cryptococcosis

B. Blastomycosis

C. Coccidioidomycosis

D. Histoplasmosis

E. Leprosy

►C

Lesions of cutaneous coccidioidomycosis demonstrate non-caseating granulomas in the dermis,


often with overlying pseudoepitheliomatous hyperplasia. Thick walled spherules of C. immitis are
found within multinucleated giant cells and granulomas.

265-What diagnosis is pictured here?

A. Lichen planus

B. Syphilis

C. Discoid lupus

D. Erythema multiforme

E. Fixed drug

►C

889
Discoid lupus is characterized on histology by vacuolar degeneration of the basal layer of the
epidermis, necrotic keratinocytes, basement membrane thickening, hyperkeratosis, follicular
plugging,dermal mucin, and a lichenoid perivascular and periadnexal infiltrate.

266-This patient presents with scatterd wheals. What is the most likely diagnosis?

A. Urticaria

B. Psoriasis

C. PRP

D. Sweets syndrome

E. contact dermatitis

►A

A patient with wheals most likely has urticaria. When neutrophils are seen in the vessels with
significant dermal edema, urticaria must be ruled out. Psoriasis, contact deramtitis and PRP are
papulosquamous diesease. Sweets would have a dense neutrophilic infiltrate .

267-Both mesenchymal and epithelial elements may be found in which of the following tumors?

A. Cylindroma

B. Microcystic adnexal carcinoma

C. Chondroid syringoma

D. Bednar tumor

E. Folliculosebaceous cystic hamartoma

►C

Cutaneous mixed tumor, also known as Chondroid syringoma, represents an acquired hamartoma
with folliculosebaceous-apocrine differentiation that has been generally interpreted as a form of
adnexal adenoma (neoplasm). It has both a mesenchymal and epithelial component.

890
268-A 9 year-old girl presents for evaluation of alopecia. Physical examination reveals a bizarrely
shaped patch of hair loss in the vertex area, with broken hairs of varying lengths. Biopsy is most
likely to demonstrate:

A. Lymphoid inflammation at the level of the hair bulb

B. Mucin within follicular epithelium

C. Fungal spores within hair shafts

D. Traumatized hair follicles with perifollicular hemorrhage, empty follicles, and

deformed hair shafts

E. Suppurative folliculitis

►D

Biopsy of trichotillomania, characterized by an abnormal urge to pull out the hair, reveals
traumatized hair follicles with perifollicular hemorrhage, empty anagen follicles, many catagen
hairs, deformed hair shafts (known as trichomalacia), and melanin casts within the follicular canal.
Lymphoid inflammation at the level of the hair bulb is a feature of alopecia areata. Mucin within
follicular epithelium is characteristic of follicular mucinosis. Tinea capitis demonstrates fungal
spores within hair shafts. Suppurative folliculitis may be seen in folliculitis decalvans.

269- Which of the following immunohistochemical stains can help distinguish basal cell carcinomas
and trichoepitheliomas from microcystic adnexal carcinomas?

A. Pro-collagen 1

B. CD34

C. Peanut agglutinin

D. TTF-1

E. Ber-Ep4

►E

Ber-Ep4 helps distinguish between BCCs/Trichoepitheliomas and microcystic adnexal carcinomas.


Morpheaform BCC and desmoplastic trichoepithelioma are Ber-Ep4 positive where as microcystic
adnexal carcinomas (MAC) are Ber-Ep4 negative. Other markers that help distinguish between

891
BCCs and trichoepitheliomas are peanut agglutinin and CD34. Peanut agglutinin is positive in BCC
and negative in trichoepithelioma vs CD34 which is negative in BCC and positive in the
peritumoral fibroblasts of trichoepitheliomas.

270-Multiple pilomatricomas are seen in:

A. Myotonic dystrophy

B. Cowden's

C. Turner's

D. Gorlin's

E. Myotonic dystrophy and Turner's

►E

Multiple pilomatricomas are seen in Rubinstein-Taybi, Gardner's (cyst-like pilomatricomas),


myotonic dystrophy, Turner's, sarcoidosis, sternal cleft and coagulation defects.

271-This form of subcutaneous sarcoid is known as which of the following?

A. Heerfordt Waldenstrom syndrome

B. lofgren syndrome

C. Darier-Roussy

D. blau syndrome

E. mikulicz syndrome

►C

Darier-Roussy is the subcutaneos form of sarcoid.

892
272-A healthy 6 month old girl has a subcutaneous nodule above her right eyebrow. A skin biopsy
demonstrates a cystic lesion with adnexal structures in the wall. Your diagnosis is:

A. Steatocystoma

B. Pilar cyst

C. Nevus sebaceous

D. Epidermal inclusion cyst

E. Dermoid cyst

►E

Dermoid cysts present along lines of embryonic closure. The are most commonly found on the head
(around the eyes) and the neck. They are lined by an epidermis that contains various epidermal
appendages that are usually fully matured.

273-Clear cell syringomas are associated with:

A. Malignancy

B. Sarcoidosis

C. Argyria

D. Diabetes

E. Lichen myxedematosis

►D

Clear cell syringomas are associated with diabetes. Syringomas are associated with Downǁs
syndrome.

274-Silver preferentially deposits in:

A. Eccrine glands

B. Apocrine glands

893
C. Hair follicles

D. Fat

E. Eccrine glands and apocrine glands

►A

Argyria can look like normal skin if you do not notice the silver (black) deposits in the eccrine
glands in the deep dermis.

275-On histology there is acanthosis, hyperkeratosis, vaculoar interface change and apoptotic
keratinocytes. There is also papillary dermal edema with endothelial swelling and mixed
inflammatory infiltrate. These features are seen in:

A. Erythema multiforme

B. Toxic epidermal necrolysis

C. Lupus erythematosus

D. Discoid lupus

E. Graft versus host disease

►A

This is the description of erythema multiforme. It is a hypersensitivity reaction with characteristic


targetoid skin lesions and mucosal involvement. Etiologies include herpes simplex, Mycoplasma
infection and drugs.

276-Methyl-green pyronin stains RNA what color?

A. Pink

B. Green

C. Blue

D. Purple

894
E. Black

►A

Methyl-green pyronin stains RNA pink and stain DNA green.

277-What is the best diagnosis?

A. Keloid

B. Ochronosis

C. Lichen Nitidus

D. Compound Spitz Nevus

E. Dermatitis Herpetiformis

►A

Distinguishing hypertrophic scar (HS) from keloid histopathologically is sometimes difficult


because thickened hyalinized collagen (keloidal collagen), the hallmark of keloid, is not always
detectable and [alpha]-smooth muscle actin ([alpha]-SMA), a differentiating marker of HS, is
variably expressed in both forms of scar.

278-This is associated with MEN IIa:

A. Neurothekeoma

895
B. Macular amyloidosis

C. Malignant peripheral nerve sheath tumor

D. Chondroid syringoma

E. Mucocele

►B

Macular amyloidosis is associated with MEN IIa.

279-The predominant cleft in dermatitis herpetiformis is:

A. Dermal

B. Basement membrane zone

C. Basal keratinocytes

D. Suprabasal

E. Subcorneal/granular

►B

Dermatitis herpetiformis or Duhring's disease, presents with very pruritic vesicles symmetrically on
extensor surfaces. On histology it presents as suprapapillary vesicles with mostly neutrophils and
inflammatory destruction of the basement membrane zone. Direct immunoflourescence shows
granular deposition of IgA in the dermal papillae and along the basement membrane zone. The cleft
in dermatitis herpetiformis is most commonly found in the basement membrane zone/subepidermal.
The antigen is transglutaminase.

280-Which of the following shows granular deposition of IgA in the dermal papillae and along the
basement membrane zone on direct immunoflourescence:

A. IgA pemphigus

B. Bullous pemphigoid

C. Linear IgA dermatosis

896
D. Dematitis herpetiformis

E. Herpes gestationalis

►D

Dermatitis herpetiformis or Duhring's disease, presents with very pruritic vesicles symmetrically on
extensor surfaces. On histology it presents as suprapapillary vesicles with mostly neutrophils and
inflammatory destruction of the basement membrane zone. Direct immunoflourescence shows
granular deposition of IgA in the dermal papillae and along the basement membrane zone.

281-A 26-year-old man with Basal Cell Nevus syndrome is diagnosed with a meningioma.
Histologic findings are most likely to reveal:

A. Psammoma bodies

B. Pustulo-ovoid bodies

C. Russel bodies

D. Verocay bodies

E. Weibel-Palade bodies

►A

Patients with Basal Cell Nevus syndrome are predisposed to developing meduloblastomas and
meningiomas, both of which can rarely present with cutaneous involvement. Psammoma bodies are
concentrically laminated calcified bodies seen in meningioma, along with ovarian and thyroid
neoplasms. Pustolo-ovoid bodies are seen in granular cell tumors. Russel bodies are
immunoglobulin inclusions in plasma cells often seen in Rhinoscleroma. Verocay bodies are
palisading nuclei arranged in rows with peripheral eosinophilic cytoplasm characteristic of
Schwannomas. Weibel-Palade bodies are organelles that are seen on macroscopy of endothelial
cells.

282-Which of the following is characteristic of Birt-Hogg-Dube Syndrome?

A. Autosomal recessive mode of inheritance

897
B. Multiple trichoepitheliomas

C. Caused by mutation in hamartin

D. Multiple trichodiscomas

E. Colon cancer common

►D

Birt-Hogg-Dube Syndrome (BHD) characterized by multiple small harartomas of mesodermal


component of hair discs, which were indentified as trichodiscromas. It is an autosomal dominant
disease caused by mutations in folliculin. Patients with multiple fibrofolliculomas may also have
acrochordons, collagenomas, lipomas, and/or oral fibromas. BHDS recently has been reported in
association with various types of renal tumors, such as oncocytoma and a variant of papillary renal
cell carcinoma. There are no trichoepitheliomas in BHD syndrome.

283-Atypical keratinocytes are noted on this biopsy. What HPV type is likely associated?

A. HPV 16

B. HPV 2

C. HPV 1

D. HVP 4

E. HPV 7

►A

HPV 16 is an oncogenic strain of HPV which can trigger development of SCC within a wart.

898
284-Which marker stains melanosomes?

A. S-100

B. Mart-1/Melan-A

C. MITF

D. HMB-45

E. Fontana-Masson

►D

HMB-45 is a melanosome immunostain targeting glycoprotein.

S-100 is a melanocyte immunostain, but it has low specificity also staining Langerhans cells,
smooth and skeletal muscle, adipocytes, and eccrine coils. MITF is a melanocyte nuclear stain.
Fontana-Masson is a melanin stain.

285-Which of the following stains is specific for melanin?

A. S-100

B. HMB-45

C. Fontana-Masson

D. MART-1

E. Melan-A

►C

Fontana-Masson is a silver stain and stains melanin black. S-100, HMB 45, MART-1 (Melan-A) are
melanocyte stains. S-100 also decorates Langerhans cells, acrosyringium, and neural crest-derived
cells.

286-Which of the following characteristics describes Atypical fibrous histiocytomas ?

A. Metastatic upon presentation

899
B. Benign

C. Low recurrence rate, high risk of metastasis

D. High recurrence rate, low risk of metastasis

E. Low recurrence rate, low risk of metastasis

►E

Atypical fibrous histiocytomas are also known as dermatofibromas with monster cells. These
tumors are uncommon and are characterized by a low recurrence rate and a rare risk of distant
metastases.

287-Leukocytoclastic vasculitis can be seen in which condition?

A. Rheumatoid neutrophilic dermatosis

B. Erythema elevatum diutinum

C. Urticaria

D. Granuloma inguinale

E. Sweet's Syndrome

►B

Rheumatoid neutrophilic dermatitis is seen in the setting of severe rheumatoid arthritis. The
pathogenesis is not understood, but in the few reports of this rare dermatosis, a true vasculitis has
not been seen. Sweet's syndrome also does not show evidence of a true vasculitis. Erythema
elevatum diutinum histologically shows a leukocytoclastic vaculitis, but with prominent interstitial
neutrophils. Other conditions with evidence of LCV include Henoch -Schonlein purpura, granuloma
faciale, urticarial vasculitis, and occasionally serum sickness.

288-On histology, there is a follicular based epithelial cystic invagination with differentiation
towards the outer root sheath forming eosinophilic cellar bodies. Central invagination contains
lamellated keratinous material. This best describes:

A. Pilar shealth acanthoma

900
B. Tumor of follicular infundibulum

C. Desmoplastic trichilemmoma

D. Basaloid follicular hamartoma

E. Trichoblastoma

►A

This describes a pilar sheath acanthoma and is a benign follicular based, comedo-like epithelial
proliferation with differentiation toward the outer root sheath. It presents as a skin colored papule
with central depressed pore.

289-What is this neoplasm?

A. Poroma

B. Eccrine acrospiroma

C. Sebaceous adenoma

D. Basal Cell Carcinoma

E. Trichoblastoma

►A

Poroma: Palmar/plantar skin, trunk and lower extremities. Named based upon location, If purely
intraepidermal called a hidroacanthoma simplex, if only in the dermis it is a dermal duct tumor, but
more commonly seen both in the epidermis and dermis and then it is given general classification as

901
poroma. Associated with Schopf-Schultz-Passarge syndrome and Clouston's syndrome.
Histologically composed of a proliferation of small uniform cuboidal basaloid cells that are
PAS+DS (i.e. contains glycogen). The cells are smaller than neighboring keratinocytes. The tumors
can be pigmented and there can be necrosis en mass. The latter finding is an exception to the
general rule that benign tumors do not show necrosis en mass.

290-This lesion is defined by the presence of what material in the dermis?

A. Amiodarone

B. Ink

C. Iron

D. Melanin

E. Carbon

►D

A blue nevus is characterized by the presence of dendritic melanocytes and melanophages.

291-The predominant location of the cleft in cicatricial pemphigoid is:

A. Dermal

B. Basment membrane zone

C. Basal keratinocytes

902
D. Supra basal

E. Subcorneal/granular

►B

Cicatricial pemphigoid is an autoimmune blistering disease that presents with ulcers, blisters and
erosions of mucosal surfaces, especially the eyes and mouth. The cleft in cicatricial pemphigoid is
found in the basement membrane zone/subepidermal as the antigens are usually BPAg2,laminin 5
and alpha-6-beta-4 integrin. Direct immunoflourescence is identical to that of bullous pemphigoid
showing linear IgG and complement deposits in the basement membrane zone.

292-Mantle cell lymphoma is characteristically positive for which of the following?

A. CD10

B. CD23

C. CD138

D. Bcl-1

E. bcl-6

►D

Bcl-1 (Cyclin D1) is a marker for mantle cell lymphoma. CD10, bcl-6, and bcl-2 are markers for
follicular cell lymphoma. Bcl-2 also stains normal T cells. CD23 is a marker for CLL/SLL and is
negative in mantle cell lymphoma. CD138 is a marker for plasma cells.

293-What is this neoplasm?

A. Pilomatricoma

B. Proliferating pilar tumor

C. Trichoepithelioma

D. Basal cell carcinoma

E. Eccrine acrospiroma

903
►A

Pilomatrixoma (Calcifying epithelioma of Malherbe): Found on the head, neck and upper
extremities in the first 2 decades of life. Typically solitary but multiple pilomatricomas arise in
several syndromes. Even in isolation these tumors may arise due to activating mu tations in the
beta-catenin gene. Histologically: This tumor consists of two major cell types plus an intermediate
or transitional cell type. Initially the tumor is more cystic with the cells at the periphery of the tumor
that are more basophilic with indistinct cell borders and little cytoplasm. The cells have
hyperchromatic nuclei and often normal mitoses can be appreciated. Centrally there are eosinophilic
―ghost or shadowǁ• cells which are cells that have undergone terminal differentiation. These cells
have more distinct borders, increased cytoplasm but only a ghost of a nucleus. Then there are cells
that reside somewhere in between these two cell types. The proportion of these cells varies
depending on the stage of the lesion; i.e younger lesions have more basophilic cells and appear
more cystic, older lesions have a greater component of ghost cells and up to 20% of lesions on
removal are completely composed of ghost cells.

294-The presence of which cell type confirms this diagnosis?

904
A. Lipidized histiocyte

B. Mast cell

C. Multinucleate keratinocyte

D. CD30+

E. Langhans giant cell

►C

Herpes infection is confirmed by the presence of multinucleate keratinocytes which often have
intranuclear inclusion bodies.

295-On histology there is acanthosis, papillomatosis with occasional neutrophils in the stratum
corneum. There is alternation of hyperkeratosis alternates with parakeratosis. There is a lack of
granular layer in the areas of parakeratosis and is most likely:

A. Inflammatory linear verrucous epidermal nevus

B. Psoriasis

C. Pityriasis rubra pilaris

D. Lichen simplex chronicus

E. Ritter's syndrome

►A

This is ILVEN seen unilateral with inflamed papules and plaques in a linear distribution. It presents
during infancy and childhood but can arise in adults. It follows the line of Blaschko and is located
on the extremities and is usually pruritic.

296-Diffuse staining with this marker is suggestive of BCC rather than trichoepithelioma:

A. CAM 5.2

B. Bcl-2

905
C. PTAH

D. CEA

E. Cytokeratin 8

►B

Trichoepitheliomas stain with bcl-2 on the periphery of individual tumor islands.

297-Which of the following can be used to stain amyloid?

A. PAS

B. Eosin

C. Giemsa

D. Aldehyde fuchsin

E. Crystal violet

►E

Stains for amyloidosis: Congo Red, Thioflavin T, Crystal Violet, Methyl violet, Pagoda red no.9,
PAS + diastase, amyloid P-component antibody

298-Acantholysis is not a prominent histopathologic feature of which disease?

A. Dermatitis herpetiformis

B. Darier's disease

C. Grover's disease

D. Pemphigus vulgaris

E. Hailey-Hailey diesease

►A

906
Dermatitis herpetiformis is a bullous disease that on histology shows neutrophils in the dermal
papillae. On direct immunoflourescence IgA is seen in a granular depostion pattern. Acantholysis is
seen histologicaly in Darier's disease, Grover's disease, Pemphigus vulgaris, Hailey-Hailey disease
and warty dyskeratoma.

299-What underlying disease does this patient have?

A. Wiskott-Aldrich syndrome

B. PHACES

C. Osteogenesis imperfecta

D. Darier's disease

E. Vitiligo

►C

Elastosis perforans serpiginosa is associated with an underlying condition in up to a third of cases.


Some of these conditions include osteogenesis imperfecta, Ehlers-Danlos syndrome, Marfan's
syndrome, acrogeria, scleroderma, and Down syndrome.

300-What is the diagnosis?

A. Marjolin's ulcer

B. Lichen planus

C. Basal cell carcinoma

907
D. Lichen amyloid

E. Dermatofibroma

►A

Marjolin's ulcer is an aggressive ulcerated squamous cell carcionoma that arises due to chronic
inflammation. The image shows the invasive atypical keratinocytes. In lichen amyloid there are
pink amorphous deposits that do not have nuclei.

301-Subcutaneous fat necrosis of the newborn has been associated with:

A. Hypocalcemia

B. Hypercalcemia

C. Hypokalemia

D. Hyperkalemia

E. Hyponatremia

►B

Hypercalcemia has been noted in some cases of subcutaneous fat necrosis of the newborn.

302-This eruption is mediated by autoantibodies to which antigen?

908
A. Desmoplakin

B. Desmoglein 1

C. BP230

D. Collagen VII

E. BP180

►D

Bullous systemic lupus erythematosus is mediate by antibodies to collagen VII. Histology is similar
to that of linear IgA bullous dermatosis and dermatitis herpetiformis.

303-Using the salt-split skin technique with direct immunofluorescence, epidermolysis bullosa
acquisita will show linear deposition of complement in what position?

A. Roof of the split

B. Roof and floor of the split

C. Floor of the split

D. Neither the roof or floor of the split since IgA is the most common reactant

E. None of the above since the pattern is not linear

►C

Epidermolysis bullosa acquisita (EBA) is a bullous disease of adults in which minor trauma
(usually on the hands and feet) leads to blisters that heal with scaring. On histology there is
classically a noninflammatory subepidermal split. The blister will immunostain with IgG on the
floor of salt-split skin, as the antigen is type VII collagen.

909
304-A 64-year old woman presents with scalloped erosions in the dermatome of the first branch of
the trigeminal nerve. Biopsy would reveal which of the following histologic findings:

A. Asteroid Bodies

B. Caterpillar Bodies

C. Cowdry Type A Bodies

D. Donovan Bodies

E. Dutcher Bodies

►C

Varicella zoster virus is the causative agent in chicken pox and shingles, where recrudescence from
latency in the dorsal root ganglion causes a dermatomal distribution of painful vesicles and
erosions. Biopsy of herpes viruses reveals cells with multinucleation, margination of the chromatin,
and Cowdry Type A bodies, which are intranuclear eosinophilic amorphous bodies surrounded by a
clear halo.

305-Clinically, a nondescript hyperkeratotic papule on the ulnar side of the base of the fifth finger is
most likely:

A. Acquired digital fibrokeratoma

B. Accessory digit

C. Cutaneous horn

D. Digital fibromatosis

E. Glomus tumor

►B

Accessory digits (supernumerary digits) are usually found at the base of the fifth finger, often
bilaterally.

910
306 -A 44-year old woman presents to clinic two weeks after injuring her finger while pruning her
rose garden. Exam reveals a suppurative nodule on her right index finger with lymphangitic spread
involving her forearm. Biopsy exhibits a dense granulomatous infiltrate and which of the following
histologic findings:

A. Asteroid Bodies

B. Caterpillar Bodies

C. Cowdry Type A Bodies

D. Donovan Bodies

E. Dutcher Bodies

►A

This is a classic presentation of sporotrichosis, which presents as lymphangitic spread after


penetrating trauma. Other organisms with true lymphangitic spread include leishmania, nocardia,
and atypical mycobacteria. Biopsy of sporotrichosis reveals granulomatous inflammation and
asteroid bodies, along with cigar bodies of budding yeast cells.

307-The predominant location of the cleft in porphyria cutanea tarda is:

A. Dermal

B. Basement membrane zone

C. Basal keratinocytes

D. Subrabasal

E. Subcorneal/granular

►B

Porphyria cutanea tarda (PCT) is the most common porhyria. It is due to a deficiency of
uroprophyrin decarboxylase. Adults present with sun induced vesicles, papules, crusts and milia in
areas of scaring. On histology the cleft in porphyria cutanea tarda is subepidermal or in the
basement membrane zone with festooning of dermal papillae into the blister. Hyalinized material
around blood vessels in the papillary dermis may be seen, as well as caterpillar bodies (eosinophilic

911
segemented basement membrane) in the roof of the blister. PCT is associated with hepatitis C,
alcohol abuse and liver disease.

308-What is the most common location of a verruciform xanthoma?

A. Mouth

B. Lower leg

C. Face

D. Hand

E. Thigh

►A

The oral mucosa is the most common location of a verruciform xanthoma. It is also seen in CHILD
syndrome, EBA, GVHD, and lymphedema syndromes.

309-What is this neoplasm?

A. Trichoadenoma

B. Trichoblastoma

C. Trichoepithelioma

D. Basal cell carcinoma

E. Nevus comedonicus

912
►A

Trichoadenoma: A nodule on the face or buttock that is slightly depressed. Typically confined to the
upper dermis and composed of multiple milia or infundibular-like cysts that have a squamous
epithelial lining associated with a granular layer and central flakey keratin in the lumen . The
stroma can be sclerotic. The lesion is composed primary of cysts with a few thin strands of basaloid
cells; if basaloid strands predominate with only a few cysts then lesion is a trichoepithelioma.

310-In addition to psoriasis, all of the following dermatoses demonstrate regular psoriasiform
hyperplasia except:

A. Lichen simplex chronicus

B. Pityriasis rubra pilaris

C. Acanthosis nigricans

D. Reiter‘s syndrome

E. Inflammatory linear verrucous epidermal nevus

►A

Lichen simplex chronicus develops in areas of chronically rubbed skin. On histopathology, there is
hyperkeratosis with areas of parakeratosis, hypergranulosis, slight spongiosis, and a sparse
superficial perivascular infiltrate. The acanthosis in LSC is irregular. The other choices all have
regular psoriasiform hyperplasia.

311-A pregnant patient presents complaining of a worsening skin eruption for the duration of the
past three years. What is the most likely diagnosis?

A. Seborrheic dermatitis

B. Tinea versicolor

C. Tinea corporis

D. Prurigo of pregnancy

E. Pityriasis rosea

913
►B

Tinea versicolor presents as hyper or hypopigmented coalescing scaly macules on the trunk and
upper extremities. It is more common during the summer and favors oily areas of skin. Mild
pruritus may be present. It is caused by Malassezia furfur. Pityrosporum orbiculare is the yeast
phase of the organism. This patient had tinea versicolor for several years and apparently got worse
during her pregnancy. A study in Italy, revealed the frequency of tinea versicolor during pregnancy
(5.7%) does not seem different from that reported in general population living in temperate climates
(2-5%). However, higher degree of colonization by Malassezia resulted at the end of pregnancy and
postpartum.

312-Which of the following can be seen in dermatofibromas?

A. Hypopigmentation of the basal layer

B. Infiltration into the fat with a honeycomb pattern

C. Hypoplasia of the epidermis

D. S-100 positivity

E. Vimentin positivity

►E

Dermatofibromas have characteristic features that can include collagen trapping, hyperplasia of
epidermis, hyperpigmentation of basal layer, dermal spindle cells, and whorls of fibrous tissue with
keloidal collagen. It stains with factor XIIIa(+) but not MAC 387 (-), S-100(-), or CD34 (-). DFSP
infiltrates the fat with a honeycomb pattern.

313-Verruciform xanthoma is seen most commonly on:

A. Head and neck

B. Distal extremities

C. Oral mucosae and genital areas

D. Mucosal surfaces and trunk/proximal extremities

E. Nail bed and periungual areas

914
►C

Verruciform xanthoma is an uncommon lesion that usually occurs on the oral mucosa of middle-
aged persons or on the scrotum of middle-aged to elderly Japanese men. The most common site for
verruciform xanthoma is the oral mucosa.

314-Which type of lymphoma with an excellent prognosis is shown here?

A. Follicuar

B. Marginal zone

C. Diffuse large B-cell

D. Cutaneous T-cell

E. NK/T-cell

►A

Follicular lymphoma demonstrates a nodular or diffuse infiltrate involving the dermis and
sometimes the subcutis. There may be a follicular, follicular and diffuse, or a diffuse pattern of
atypical B-cells.

315-This was a deep seated large tumor in the thigh, what is it?

915
A. Liposarcoma

B. Nodular fascitis

C. Hibernoma

D. Spindle cell lipoma

E. Pleomorphic lipoma

►A

A liposarcoma is one of the most common sarcomas to occur in adults, 50-70yos. Those that occur
in resectable regions (like the thigh) have a good prognosis; those that occur in retroperitoneum,
mediastinum or spermatic cord recur repeatedly and have a risk to dedifferentiate with a mortality
approaching 80% over the following 10-20 years. It has a characteristic supernumerary ring with
amplification of 12q14 MDM2 region. Histologically The adipocytes vary in size throughout the
tumor. Pleomorphism is variable depending upon the degree of differentiation that the tumor
displays. The nuclei of the more well-differentiated adipocytes, as well as, the spindled cells are
hyperchromatic with nuclear atypia. The presence of lipoblasts, i.e. either mono-vacuolated signet
ring type of lipoblast as is more commonly seen in myxoid liposarcoma and/or multi-vacuolated
lipoblasts with central nuclei scalloped by vacuoles, are characteristic of a liposarcoma, but are not
necessary to make the diagnosis. There is often a plexiform proliferation of blood vessels, with an
appearance that has been likened to crow's feet or chicken wire. The stroma can be loose and
delicate, fibrous or myxoid. Thick ropey fibrous septae are common in liposarcoma and in
lipoblastoma.

316 -An obese 15-year-old boy is diagnosed with acanthosis nigricans. The presence of this disorder
correlates best with elevated plasma:

916
A. Glucagon

B. Calcitonin

C. Insulin

D. Glucose

E. Calcium

►C

Acanthosis nigricans correlates most closely with elevated insulin resistance. A canthosis nigricans
may also present in association with internal malignancies, most commonly gastric cancer.
Additionally, certain drugs, classically niacinamie, may induce this disorder.

317-A 54-year-old woman presents to clinic with a tender lesion involving the index finger that is
worsened by cold. Biopsy findings reveal a glomus tumor. Electron microscopy of these tumors
characteristically exhibits:

A. Comma-shaped bodies

B. Pustulo-ovoid bodies

C. Russel bodies

D. Verocay bodies

E. Weibel-Palade bodies

►E

Weibel-Palade bodies are organelles that are seen on macroscopy of endothelial cells. These are
increased in glomus tumors. Pustolo-ovoid bodies are seen in granular cell tumors. Verocay bodies
are palisading nuclei arranged in rows with peripheral eosinophilic cytoplasm characteristic of
Schwannomas. Russel bodies are immunoglobulin inclusions in plasma cells. Comma-shaped
bodies may be seen on electron microscopic analysis of benign cephalic histiocytosis.

318-All of the following are true of reticulohistiocytoma except:

917
A. Rare occurrence in children

B. Giant cells with ―ground-glassǁ cytoplasm

C. Association with arthritis

D. Immunostaining is positive for OKM1

E. Trauma is precipitating factor

►C

Reticulohistiocytomas, also called giant cell reticulohistiocytomas, occur almost exclusively in


adults. They are generally solitary, and unlike the multicentric type, are not associated with
mutilating arthritis or predisposition for malignancy.

319-Apocrine hidrocystoma may be associated with which of the following syndromes?

A. Downs syndrome

B. Schopf-Schulz-Passarge

C. Buschke-Ollendorf syndrome

D. Cowdenǁs Disease

E. Osteogensis Imperfecta

►B

Schopf-Schulz-Passarge is a syndrome of ectodermal dysplasia which presents with keratosis


palmoplantaris with hypodontia, hypotrichosis, and cysts of the eyelids. The other answer choices
have not been associated with apocrine hidrocystomas.

320-This characteristic change of the fat is characteristic of which panniculitis?

A. Subcutaneous panniculitis like T cell lymphoma

B. Lupus panniculitis

C. Sclerosing panniculitis

918
D. Pancreratic panniculitis

E. Calciphylaxis

►B

Lupus panniculitis is characterized by hyalinization of the fat with a lymphoplasmocytic infiltrate,


as depicted here. It is a lobular panniculitis without vasculitis. The other answer choices are other
lobular panniculitidies. Subcutaneous panniculitis like t cell lymphoma has atypical cells rimming
the fat lobules. Sclerosing panniculitis is lipodermatosclerosis which has lipomembranous change.
Pancreatic panniculitis has liquefactive fat necrosis and saponificatoin of the fat. Calciphylaxis has
calium deposites in the walls of tbe blood vessels.

321-A 30-year old male presents with clustered, painful plaques on his shoulder. Histopathology
showed interlacing bundles of cells with eosinophilic cytoplasm and no mitoses. Desmin and SMA
stains were positive. What gene defect is associated with the development of these lesions?

919
A. Fumarate Hydratase

B. Endoglin

C. ERCC8

D. RECQL2

E. GJB3

►A

The patient has multiple cutaneous leiomyomas. Hereditary leiomyomatosis is an inherited defect of
fumarate hydratase. These patients develop multiple cutaneous leiomyomas and have an increased
incidence of renal cell carcinoma. Female patients will usually also develop significant uterine
leiomyomatosis resulting in hysterectomy.

322-Scalp biopsy of a 54-year-old female with suspected lichen planopilaris would likely reveal
inflammation around which portion of the hair follicle:

A. Dermal papilla

B. Hair bulb

C. Matrix

D. Isthmus

E. Infundibulum

920
►E

On biopsy, alopecia areata exhibits a peribulbar lymphocytic ―swarm of bees.ǁ Discoid lupus
typically exhibits inflammation surrounding the isthmus, along with a perivascular dermatitis and
vacuolar interface changes. Lichen planopilaris exhibits inflammation most densely concentrated
about the infundibulum.

323-Elastosis perforans serpiginosa is associated with all except:

A. Rothmund-Thompson

B. Scleredema

C. Ehlers-Danlos

D. Acrogeria

E. Down‘s

►B

Elastosis perforans serpiginosa is associated with Down‘s syndrome, Ehlers-Danlos type IV,
Osteogenesis imperfecta, Rothmund Thompson, Marfan‘s, Werner‘s, acrogeria, and penicillamine
therapy.

324-Which of the following regarding stains is true?

A. A Verhoeff-van Gieson stain is used to stain elastic fibers red.

B. A methanamine silver stain is used to identify bacteria.

C. A fite stain is used to identify spirochetes.

D. A Von Kossa stain is used to identify calcium.

E. A giesma stain is used to identify eosinophil granules.

►D

Stains/Application/Result:

921
-Van Gieson - Elastic fibers - Black

-Methanamine-silver - Fungi, parasites - Black

-AFB/Fite - Acid-fast bacilli - Red

-Von Kossa - Calcium - Black

-Giesma - Mast cells - Metachromatically purple

325-This is an H and E stained slide, what is the organism?

A. Exophiala jeanselmei

B. Chromomycosis

C. Nocardia

D. Zygomycosis

E. Mucormycosis

►A

Phaeohyphomycotic cyst: Typically the result of an opportunist infection caused by being impaled
by a splinter contaminated with a dematiaceous or pigmented fungus. Most commonly seen on
distal extremities and results in a dermal abscess with fibrous tissue and granulomas within which
are pigmented hyphae and yeast. The most common organism is Exophiala jeanselmei (yellow-
brown septae hypha) and Wangiella dermatitidis. Histology: Circumscribed cyst with a fibrous wall
within which is a chronic granulomatous reaction or dermal abscess, a splinter is sometimes seen
along with the brown filamentous hyphae and yeast.

922
326-Osteoclast-like giant cells are characteristic of:

A. Giant cell fibroblastoma

B. Reticulohistiocytoma

C. Necrobiosis lipoidica diabeticorum

D. Giant cell tumor of the tendon sheath

E. Necrobiotic xanthogranuloma

►D

Osteoclast-like giant cells are seen in giant cell tumor of the tendon sheath. Osteoclast -like giant
cells have eosinophilic cytoplasm with haphazardly arranged nuclei. Reticulohistiocytoma is
characterized by giant cells with an oncocytic glassy pink cytoplasm.

327-Which of the following histological features would be most helpful in differentiating lichenoid
drug eruption from lichen planus?

A. Civatte bodies

B. Parakeratosis and eosinophils

C. Squamatization of the basal layer

D. Presence of pruritus

E. Band-like infiltrate with ―Saw-toothǁ rete ridges

Lichenoid drug reaction Lichen planus

923
►B

Lichenoid drug eruptions share clinical and histopathologic features with lichen planus. Sometimes
differentiation is not possible; however, eosinophil, parakeratosis, and a deeper perivascular
infiltrate is more suggestive of lichenoid drug. Implicated medications include captopril,
penicillamine, and chloroquine.

328-On histology there is a proliferation of basaloid cells with larger pale cells in a jigsaw pattern.
There is an eosinophilic cuticle surrounding the lobules. This describes a:

A. Cylindroma

B. Eccrine spiradenoma

C. Mixed tumor

D. Nodular hidradenoma

E. Eccrine syringofibroadenoma

►A

This describes a cylindroma. This is a benign sweat tumor linked with CYLD gene. It presents as a
skin colored to erythematous bluish rubbery nodule. Located on the head and neck and on the scalp.
It is also known as a turban tumor.

329-All of the following markers can be helpful in differentiating basal cell carcinoma from
trichoepithelioma except

A. CD10

B. CD31

C. CD34

D. Bcl-2

E. Stromelysin-3

►B

924
The histopathologic discrimination of trichoepithelioma and BCC often presents a diagnostic
challenge. The distinction is of clinical significance due to the differences in prognosis and
treatment of these tumors. Thus, a large number of ancillary laboratory techniques have been
investigated as an aid in this differential, include CD10, CD34, BCL-2, Stromelysin-3 and Ki-67.
CD31 is a vascular marker that stains positive in angiosarcoma and Kaposi sarcoma.

330-Multiple clear cell acanthomas are associated with:

A. Ichthyosis

B. Cowden's

C. Immunosuppression

D. Gastrointestinal polyps

E. Breast cancer

►A

Clear cell acanthoma is associated with ichthyosis.

331-Which gene mutation and/or amplifications are more commonly found in this type of
melanoma and mucosal sites than in melanomas on intermittently sun-exposed sites?

A. NRAS

925
B. KIT

C. GNAQ

D. HRAS

E. PTEN

►B

KIT mutations and/or amplifications are more commonly found in melanomas located on acral and
mucosal sites than in melanomas on intermittently sun-exposed sites.

332-A 32-year-old woman presents to clinic with hypertrophic plaques on the external nares.
Biopsy revealed findings consistent with Rhinoscleroma. Findings are most likely to include:

A. Psammoma bodies

B. Pustulo-ovoid bodies

C. Russel bodies

D. Verocay bodies

E. Weibel-Palade bodies

►C

Rhinoscleroma is caused by Klebsiella rhinoscleromatis. Biopsy findings include Mikulicz cells,


which are foamy macrophages containing bacteria, and Russel bodies. Russel bodies are
immunoglobulin inclusions in plasma cells. Pustolo-ovoid bodies are seen in granular cell tumors.
Verocay bodies are palisading nuclei arranged in rows with peripheral eosinophilic cytoplasm
characteristic of Schwannomas. Weibel-Palade bodies are organelles that are seen on macroscopy of
endothelial cells.

333-What type of lymphoma is shown here?

A. Follicle center cell

B. Intravascular large B-cell

926
C. Marginal zone

D. Cutaneous T-cell

E. NK/T-cell

►B

Intravascular B-cell lymphoma is characterized by the presence of large, atypical cells partially or
fully occluding blood vessels in the dermis and subcutis. The prognosis is very poor.

334-What is the diagnosis?

A. Squamous cell carcinoma

B. Trichilemmoma

927
C. Poroma

D. Pilar sheath acanthoma

E. Inverted follicular keratosis

►E

An inverted follicular keratosis is a benign tumor of the follicular infundibulum. On histology, they
demonstrate tumor lobules composed of central squamous cells and peripheral basaloid cells.
Squamous eddies are commonly noted.

335-Of the patients that present with discoid lupus, how many will progress to systemic lupus
erythematosus?

A. 5%

B. 25%

C. 50%

D. 75%

E. 100%

►A

Of the patients that present with DLE, 5% of them will prgress to SLE. However, of the patients
that present with SLE, 25% will get DLE.

336-A patient presents with a few of these lesions and some hyperkeratotic papuleson his anterior
shins. What diagnositic test should the patient undergo?

A. Colonoscopy

B. cystoscopy

C. bronchoscopy

D. cortisone stimulation

928
E. otoscopy

►A

The image is a sebaceous adenoma, and the question describes keratoacanthomas on the anterior
shin, thus this patient should undergo colonoscopy to evaluate for colon cancer. Given this
constellation of symptoms one must evaluate for muir torre syndrome.

337-This clinically raspberry-like papilloma occurs in which syndrome?

A. Goltz

B. Rothmund-Thomson

C. Incontinentia pigmenti

D. Hermansky Pudlak

E. CHILD

►A

929
Goltz syndrome is characterized by distinct raspberry-like papillomas at junctions of mucosa and
skin. Other findings include the following: skin atrophy, ulcerations, hernialike outpouchings of
fatty tissue, syndromic facies, and lobster claw deformity.

338-Which of the following regarding necrobiotic xanthogranuloma and normolipemic plane


xanthomas is FALSE:

A. Both can have an associate paraproteinemia

B. Both commonly have normal serum lipids

C. Both are most commonly distributed on the upper body

D. Normolipemic plane xanthomas lack the induration and ulceration commonly seen in
necrobiotic xanthogranuloma

E. Multiple myeloma is seen in 80% of necrobiotic xanthogranuloma but is rarely seen in


normolipemic plane xanthomas

►E

Necrobiotic xanthogranulomas are closely related to normolipemic plane xanthomas: both have a
yellow hue, associated paraproteinemia, normal serum lipids, upper body distribution. However,
normolipemic plane xanthomas have a much stronger association with multiple myeloma, their
plaques lack induration and they rarely ulcerate. Treatment is usually directed at the
paraproteinemia.

339-The deficiency in familial multiple cutaneous leiomyomatosis is:

A. Arginase

B. Adenosine deaminase

C. Endoglin

D. MC1R

E. Fumarate hydratase

►E

930
Fumarate hydratase is deficient in familial multiple cutaneous leiomyomatosis. This same enzyme
is deficient in familial uterine leiomyomatosis associated with renal cell cancer. Arginase is
deficient in arginemia. Adenosine deaminase is deficient in autosomal recessive severe combined
immunodeficiency disease (SCID). Endoglin is deficient in some patients with Osler-Weber-Rendu.
MC1R (melanocortin 1 receptor) shows sequence variation in people with red hair.

340-A male patient presents with hypopigmented macules on his upper back that are asymptomatic
and more noticeable in the summer. What is the most likely diagnosis?

A. pityriasis rosea

B. vitiligo

C. tinea versicolor

D. eczema

E. waardenberg syndrome

►C

Tinea versicolor often is asymptomatic and more noticeable in the summer as patients often tan and
the areas with the tinea versicolor do not tan as azeleic acid is secreted by the malassezia and
melanin production is suppressed.

341-This biopsy was obtained from an annular pigmented plaque on the trunk, what is this
neoplasm?

931
A. Targetoid hemosiderotic hemangioma

B. Glomeruloid hemangioma

C. Angiosarcoma

D. Kaposi's sarcoma

E. Tufted hemangioma

►A

Targetoid Hemosiderotic Hemangioma: Occurs most commonly on young adult males on the trunk
and named for its clinical appearance. The lesion typically appears as a plaque with concentric
rings, there is violaceous central region surrounded by a ring of pallor which is then surrounded by
an erythematous or brown rim. Histologically in the superficial dermis there are dilated blood
vessels, some of which have plump hobnailed endothelial cells. In the deeper dermis the vessels
have a narrower lumen and dissect between the collagen bundles. Often within the lesion there are
extravasated red blood cells and hemosiderin.

342-The pigment deposits in ochronosis are accentuated with:

A. Cresyl violet

B. Methyl-green pyronin

C. Silver nitrate

D. Bodian

E. Cresyl violet and silver nitrate

►A

Cresyl violet stains the pigment deposits in ochronosis black. Methylene blue will also stain the
pigment black. This pigment does not stain with silver nitrate. The Bodian stain is for nerves.
Methyl-green pyronin stains RNA and DNA.

343-62-year old female with history of acute myeloid leukemia presents with multiple edematous,
erythematous papules after starting G-CSF.

932
A. Sweet's syndrome

B. Leukocytoclastic vasculitis

C. Bowel bypass dermatosis

D. Polymorphous light eruption

E. Erythema multiforme

►A

Sweet's syndrome, or acute febrile neutrophilic dermatoses, is often associated with AML and
GCSF. Histologically, there is marked dermal edema with a prominent infiltrate composed of
neutrophils with leukocytoclasia. There is an absence of extensive vascular damage.

344-Guarnieri bodies can be found associated with which of the following viral infections?

A. Variola

B. HHV6

C. HHV8

D. Rubeola

E. Rubella

►A

Guarnieri bodies are eosinophilic intracytoplasmic inclusions associated with variola infection
(smallpox), however they can also be seen in infections with other pox viridae, including vaccinia,
cowpox, and parapox. These inclusions represent aggregates of viral particles.

345- a pregnant woman <15 weeks presents with pink scaly patches all over her body and has this
pathology, what is she at risk for?

A. LGA

933
B. death

C. SGA

D. Premature delivery

E. SIDS

►D

If pityriasis rosea occurs in women <15 weeks pregnant, they are at risk for premature delivery and
neonatal hypotonia.

346 -Steatocystoma multiplex is associated with:

A. Jadassohn-Lewandowsky

B. Jackson-Lawler

C. Schaufer-Brunauer

D. Zinsser-Engman-Cole

E. Touraine-Solente-Gole

►B

Jackson-Lawler (Jackson-Sertoli) is known as pachyonychia congenital type 2. Multiple steatocysts


can be seen in this condition.

934
347-This presented as a papule with a tuft of hair on the face, what is this neoplasm?

A. Trichofolliculoma

B. Trichoepithelioma

C. Fibrous Papule

D. Acrochordon

E. Fibrofolliculoma

►A

Trichofolliculoma: Most often occurs on the face as a papule with a tuft of hair. Histologically
composed of a central dilated large follicle (Mama hair), from which many secondary smaller hair
follicles radiate (Baby hairs), with the entire unit often enveloped in a vascularized fibrotic stroma.
If sebaceous glands are associated with the hair follicles then the term used is a sebaceous
trichofolliculoma or a Folliculosebaceous cystic hamartoma. Within the mesenchymal stroma there
are increased CD34 and factor XIIIA fibroblasts and Merkel cells as is seen in the ORS of the
follicles.

348-Which of the following immunohistochemical staining profiles of a non-breast skin lesion best
characterizes extra-mammary Paget‘s disease secondary to an underlying visceral malignancy?

935
A. MART-1 positive/cytokeratin 20 negative/cytokeratin 7 negative/gross cystic fluid
disease protein-15 negative

B. MART-1 negative/pankeratin positive/cytokeratin 7 positive/cytokeratin 20 positive/


gross cystic fluid disease protein-15 negative

C. MART-1 negative/pankerative positive/cytokeratin 7 negative

D. MART-1 negative/pankeratin positive/cytokeratin 7 positive/cytokeratin 20 negative/


gross cystic fluid disease protein-15 positive

E. CD34 positive, Factor XIIIa negative

►B

The differential for Pagetoid cells in the epidermis includes melanoma in situ, Paget‘s disease of the
breast, extra-mammary Paget‘s disease, squamous cell carcinoma in situ, sebaceous carcinoma, and
others. MART-1 positivity in the setting of pagetoid spread suggests melanoma in situ. The
combination of MART-1 negativity, pankeratin positivity, and cytokeratin 7 negativity suggests
squamous cell carcinoma in situ. Both Paget‘s disease of the breast and extra-mammary Paget‘s
disease are characterized by pankeratin positivity and cytokeratin 7 positivity. Extra-mammary
Paget‘s is defined clinically by its location outside of the breast. Extra- mammary Paget‘s disease
arising secondary to a visceral malignancy is further characterized by cytokeratin 20 positivity and
gross cystic fluid disease protein -15 negativity, whereas primary extra-mammary Paget‘s disease is
cytokeratin 20 negative and gross cystic fluid disease protein- 15 positive.

349-These skin changes are caused by which type of trauma?

A. Cryotherapy

B. Thermal burn

C. Electrocautery

936
D. Chemotherapy

E. Ultraviolet radiation

►B

The histology of thermal burns include epidermal and dermal necrosis, thrombosis of blood vessels,
destruction of pilosebaceous units, fusion of the collagen bundles and a sparse inflammatory cell
infiltrate. Electrocautery effect often shows vertical elongation of keratinocytes and homogenization
of dermal collagen. Topical chemotherapy will result in bizarre keratinocyte mitoses. Ultraviolet
radiation will result in necrotic keratinocytes. Cryotherapy will cause the epidermis to become
homogenous and a subepidermal bulla to form.

350-A 74-year-old man presents with red-brown papules with horny scales involving the dorsal feet
and lower legs. Biopsy reveals a discrete region of hyperkeratosis overlying thinned granular and
spinous layers with irregular acanthosis and a bandlike lymphoplasmacytic infiltrate in the papillary
dermis, thus confirming the suspected diagnosis of Flegel Disease. What abnormality of the
epidermis is most likely causative?

A. Absent keratohyaline granules

B. Diminished loricrin

C. Diminished involucrin

D. Diminished lamellar bodies

E. Decreased Transglutaminase I activity

►D

Diminished lamellar, or Odland, bodies is characteristic of Flegel‘s disease, otherwise known as


hyperkeratosis lenticularis perstans. Complete absence of lamellar bodies is observed in Harlequin
Fetus. Consistent abnormalities of the other epidermal components listed are not seen in Flegel‘s
disease.

351 -Clinically, this lesion often has a blue hue and usually occurs on the face:

937
A. Apocrine hidrocystoma

B. Epidermoid inclusion cyst

C. Bronchogenic cyst

D. Eccrine hidradenoma

E. Myxoid cyst

►A

apocrine hidrocystoma are usually solitary, tranlucent nodules which may have a bluish hue due to
the Tyndall effect. Histologically, they have one or several large cystic spaces with decapitation
secretion.

352 -The histologic finding of "shoulder parakaratosis", parakeratosis with prediliection for the
follicular ostia, is characteristic of pityriasis rubra pilaris as well as:

A. Stasis dermatitis

B. Atopic dermatitis

C. Seborrheic dermatitis

D. Nummular dermatitis

E. Allergic contact dermatitis

►C

Parakeratosis refers to pyknotic keratinocyte nuclei in the stratum corneum, where nuclei are not
usually present. It is common in diseases with changes in the epidermis. Histologically seborrheic
dermatitis can shows "shoulder parakeratosis" with epidermal spongiosis. Histologically atopic,
nummular and contact dermatitis present with spongiosis with or without vesicles. Stasis dermatitis
presents with more dilated papillary dermal small blood vessels and hemosiderin.

353-This lesion was excised from the face, what is this neoplasm?

A. Clear cell syringoma

938
B. Sebaceous carcinoma

C. Microcystic adenocarcinoma

D. Renal cell carcinoma

E. Clear cell hidradenoma

►A

Syringoma: Multiple eyelids, cheeks, chests, can be eruptive and increased numbers in Down's
syndrome and a clear cell change has been associated with Diabetes. Histologically this is a dermal
tumor consisting of eccrine ducts, lined by two cell layers sometimes creating a tad pole
appearance, there can also be small basaloid islands or strands of cells. Within the lumens of the
ducts is eosinophilic material, the lumens are CEA+. The tumor is located superficially and fails to
display perineural extension as is seen in MAC. The glands are associated with a fibrous sclerotic
stroma, usually no (or minimal) keratin cysts or foreign body granulomas as seen in desmoplastic
trichoepithelioma. Enlargement of the glandular cells more of a clear or vacuolated cytoplasm can
be seen in patients with diabetes.

354-A biopsy was obtained from the nasal mucosa, what are the organisms in this biopsy?

939
A. Rhinosporidium

B. Coccidioidomycosis

C. Prototheca wickerhamii

D. Histoplasmosis capsulatum

E. Cryptococcus neoformans

►A

Rhinosporidiosis originally thought to be due to a fungus Rhinosporidium seeberi, now believed to


be caused by an aquatic protistan parasite or Cyanobacteria, Microcystics aeruginosa. Sri Lanka &
South America polypoid nasal/mucosal lesions in males and conjunctival lesions in females.
Obtained from water or soil. Histology: granulomatous dermatitis with mixed inflammatory cell
infiltrate with large thick walled birefringent sporangia which mature toward the center of a cyst,
the organism measures 10-200 microns contain 7-8 micron endospores which contain eosinophilic
globules. Watery environments causes cyst to rupture.

355-This keratin is associated with hair and nail:

A. Keratin 6

B. Keratin 8

C. Keratin 10

D. Keratin 16

E. Keratin 17

►E

Keratins 6 and 16 are found in the nail. Keratin 17 is seen in the nail as well as in the hair follicle,
and patients with pachyonychia congenital type 2, in which there is a mutation of keratins 6b and
17, have nail findings as well as steatocysts.

356-Dorf balls are seen in which tumor?

940
A. Kaposi's sarcoma

B. Angiosarcoma

C. Tufted angioma

D. Kaposiform hemangioendothelioma

E. Dermatofibroma sarcoma protuberans

►A

Dorf balls are pink amorphous globules seen in vessels in Kaposi's sarcoma. Typical histologic
findings include proliferation of spindle cells, prominent slitlike vascular spaces, and extravasated
red blood cells.

357-What lesion is shown here?

A. Chondroid syringoma

B. Hyaluronic acid filler nodule

C. Glomus tumor

D. Myofibroma

E. Scleromyxedema

►C

941
Glomus tumors show aggregates of glomus cells surrounding numerous blood vessels within a
fibrous and sometimes myxoid stroma.

358-Which fixative would best preserve the histologic features of a gout tophus?

A. Saline

B. Absolute ethanol

C. Formaldehyde

D. Michel's fixative

E. Tissue culture media

►B

Gout tophi are caused by deposition of monosodium urate monohydrate crystals. Using absolute
ethanol to fix the biopsy specimen will preserve the crystalline architechture. In formaldehyde fixed
tissure, the crystals are less obvious.

359-What is the diagnosis of this neoplasm?

A. Fibroma of tendon sheath

B. Dermatofibroma

C. Dermatofibrosarcoma protuberans

D. Spindle cell lipoma

942
E. Leiomyoma

►A

Fibroma of tendon sheath is a solitary, slow-growing tumor that is usually located on the hands,
wrists or fingers of middle-aged individuals. Spindle or stellate cells are embedded in a dense
fibrocollagenous stroma. This lesion demonstrates characteristic dilated or slit-like vascular
channels.

360-A 54-year old woman who is healthy except for chronic osteoarthritis, for which she takes daily
naproxen, with bullae and milia on the dorsal hands. Biopsy is most likely to exhibit which of the
following histologic findings:

A. Asteroid Bodies

B. Caterpillar Bodies

C. Cowdry Type A Bodies

D. Donovan Bodies

E. Dutcher Bodies

►B

Caterpillar bodies represent degenerated type IV collagen and are classically associated with
porphyria cutanea tarda. In this case, it is most likely that the patient is experiencing
pseudoporphyria, which is often caused by NSAIDs such as naproxen. The other histologic bodies
listed are not associated with porphyria.

361-The mutated product in Darier's disease is:

A. SPINK5

B. SERCA2

C. ATP2A2

D. ATP2C1

943
E. SPINK5 and ATP2A2

►B

In Darier's disease, the ATP2A2 gene encoding the SERCA2 Ca(2+)-ATPase is mutated in some
patients. SPINK5 is the gene that is mutated in some patients with Netherton's syndrome, and this
gene the serine protease inhibitor LEKTI. ATP2C1 is the gene mutated in some patients with
Hailey-Hailey disease.

362-On histology there is fibrosis around nerves and blood vessels with nerve degenerative changes
diagnosed as Morton's neuroma. It is a benign neural degenerative reaction seen on the:

A. Sole

B. Trunk

C. Fingers

D. Head

E. Neck

►A

In a Morton's neuroma it is found on the sole of the foot and is considered to be a benign neural
degenerative reaction. It occurs in adults more commonly than females. Excision is curative.

363-Blue-gray pigmentation on the legs secondary to minocycline on biopsy stains with:

A. Fontana Masson

B. Perls

C. Sudan black

D. Fontana Masson and Perls

E. All of these answers are correct

944
►D

There are three types of pigmentary change that are caused by minocycline. The blue -gray
pigmentation on the legs and the blue pigment in scars is thought to be secondary to a drugprotein
complex deposited in the dermis. The blue-gray pigment on the legs stains with Perls and Fontana-
Masson. The blue in scars (often on the face) stains with Perls. The muddy-brown discoloration on
sun-exposed areas shows increased basilar pigment and melanin incontinence on biopsy. It is likely
secondary to phototoxicity.

364-Cicatricial pemphigoid antibodies directed against this are associated with high frequency of
malignancy:

A. Laminin 5

B. Laminin 6

C. Beta4-integrin

D. BPAg2

E. All of these answers are correct

►A

Anti-laminin 5 cicatricial pemphigoid (CP) is also known as anti-epiligrin CP. Anti-epiligrin CP is


associated with an increased frequency of internal adenocarcinomas. Laminin 5 is composed of
three chains (heterotrimer), alpha3, beta3, gamma2. Antibodies are frequently directed against the
alpha3 chain, and so cross-reactivity can be observed with laminin 6, as laminin 6

945
(alpha3beta1gamma1) has the alpha3 chain as well. Beta4-integrin antibodies have been associated
with ocular CP. BPAg2 antibodies are seen in CP patients that have mucosal as well as skin disease.

365-A question for a nerve sheath myxoma is what stain is positive in this tumor?

A. EMA

B. S100

C. PAS

D. CD68

E. SMA

►B

Nerve sheath myxoma or myxoid neurothekeoma is S100 positive and EMA negative. It is a
collection of stellate and pale spindle cells in a myxoid matrix.

366-Multiple trichoepitheliomas are seen in all except:

A. Bazex

B. Brooke-Fordyce syndrome

C. Brooke-Spiegler syndrome

D. Gorlin's syndrome

946
E. Rombo syndrome

►D

Gorlin's syndrome is nevoid basal cell carcinoma syndrome; multiple trichoepitheliomas are not
seen. Several syndromes have been associated with multiple trichoepitheliomas: Bazex,
BrookeFordyce, Brooke-Spiegler, Rombo, and possibly Rasmussen. (Rasmussen described one
family in 1975 with autosomal dominant inheritance of multiple trichoepitheliomas, milia, and
cylindromas.) Bazex (follicular atrophoderma, hypotrichosis, occasional trichoepitheliomas, basal
cell carcinomas, and localized or generalized hypohidrosis) is inherited in an X -linked dominant
manner. Dont confuse with Bazex syndrome or acrokeratosis neoplastica. Brooke and Fordyce both
described multiple trichoepitheliomas concurrently in 1892, and therefore multiple familial
trichoepitheliomas are sometimes called ―Brooke-Fordyceǁ sydrome. Spiegler described patients
with multiple cylindromas in 1899 and also noted that many of these patients had mutiple
trichoepitheliomas; more recently it has been noted that multiple spiradenomas may be seen in
patients with multiple trichoepitheliomas and cylinidromas; this co-occurrence of tumors has been
referred to as ―Brooke-Spieglerǁ syndrome. (Brooke-Fordyce and Brooke-Spiegler are likely the
same syndrome.) Rombo syndrome is characterized by vermiculate atrophoderma, multiple BCCs,
multiple trichoepitheliomas, cyanosis and peripheral vasodilation.

367- A 40-year-old female presents for a scalp biopsy. On histology, you note premature
desquamation of the inner root sheath, eccetricial epithelial atrophy with hair shafts in close
proximity to the dermis, and concentric lamella fibroplasia of some follicles. There is variably
dense lymphocytic perifollicular inflammation, primarily at the level of the upper isthmus and lower
infundibulum. There is occasional fusion of the infundibula. Based on the findings of the scalp
biopsy, what is the most likely diagnosis?

A. androgenetic alopecia

947
B. lichen planopilaris
C. telogen effluvium
D. central centrifugal cicatricial alopecia
E. trichotillomania

Correct choice: D. central centrifugal cicatricial alopecia

Explanation: On histology, central centrifugal cicatricial alopecia demonstrates premature


desquamation of the inner root sheath, eccentric epithelial atrophy (thining) with hair shafts in close
proximity to the dermis, and concentric lamellar fibroplasia of affected follicles. There is variably
dense lymphocytic perifollicular inflammation, primarily at the level of the upper isthmus and lower
infundibulum. There is occasional fusion of infundibula (polytrichia). In advanced lesions, total
destruction of the follicular epithelium with retained hair shaft fragments and granulomatous
inflammation are present.

- In trichotillomania, follicles are normal size, there is trichomalacia and pigment casts without
significant inflammation. There is an increased number of terminal catagen or telogen hairs. There
is incomplete disrupted follicular anatomy. The total number of hairs both terminal and vellus is
normal.
- In androgenetic alopecia there is a normal total numbr of follicles and no significant inlfmamation,
increased number and percentage of vellus hairs, numerous fibrous streamers, and slightly increased
telogen count. Uninvolved scalp appears normal.
- Telogen effluvium demosntrates a normal total number of hairs, normal number of terminal (large)
hairs, absence of inflammation and scarring. There is an increase in telogen count to > 20%.
- Lichen planopilaris demonstrates a band-like mononuclear cell infiltrate obscuring the interface
between follicular epithelium and dermis; vacuolar alteration at the interface and hypergranulosis
within affected infundibula is typical. Colloid or Civatte bodies are occasionally found as part of the
itnerface alteration. Inflammation affects the upper portion of the follicle (infundibulum and
isthmus) most severely. Occasionally, epidermal changes of lichen planus are found.

368- A patient with a CD4 count of 38 presents to your clinic with several red-purple smooth
nodules on the legs. A biopsy of one of these nodules is likely to stain positive for all of the
following EXCEPT:

A. CD31
B. Factor VIII related antigen
C. CEA
D. CD34

948
E. Ulex aeropaeus agglutinin 1

Correct choice: C. CEA

Explanation: The stem describes a case of AIDS-associated Kaposi sarcoma, a malignant vascular
neoplasm due to infection with HHV-8 (aka KSHV). CEA is a marker of eccrine glands and will
thus will not stain negative. The remaining answer choices are markers of endothelial cells and will
thus stain positive in Kaposi sarcoma.

369- What is the most likely causative organism?

A. Blastomyces dermatitidis
B. Coccidioides immitis
C. Lacazia loboi
D. Fonsecaea pedrosoi
E. Sporothrix schenckii

Correct choice: D. Fonsecaea pedrosoi

Explanation: These biopsy findings (Medlar bodies/copper pennies/sclerotic bodies) are diagnostic
of chromoblastomycosis (chromomycosis), which is most often due to infection with Fonsecaea
pedrosoi.

370- What would this lesion show histologically?

949
A. Well formed tuberculoid granulomas in a linear pattern
B. Scattered comedo-like cysts
C. A proliferation of basaloid cells with small ducts
D. A grenz zone with a lot of inflammatory cells and globi
E. Acanthosis and neutrophilic infiltrate

Correct choice: A. Well formed tuberculoid granulomas in a linear pattern

Explanation: This is tuberculoid leprosy which shows well formed tuberculoid granulomas in a
linear pattern on histology. They are linear because they follow nerves and destroy them.
Lepromatous leprosy is characterized by a dense infiltrate of macrophages with few lymphocytes in
the dermis and subcutaneous tissue with a grenz zone, Virchow cells and globi.

371- This lesion will stain positive for all of the following markers EXCEPT:

A. Synaptophysin
B. TTF-1
C. Chromogranin A
D. CK20
E. EMA

950
Correct choice: B. TTF-1

Explanation: This is a biopsy of merkel cell carcinoma (MCC). MCC can be difficult to distinguish
from small cell lung cancer, but MCC characteristically stains negative for TTF-1 (thyroid
transcription factor-1) whereas small cell lung cancer stains positive for TTF-1.

372- A 47-year-old female presents with a diffuse dermatosis with tiny pustules most prominent in
the flexural surfaces. The pustules then resolve with superficial, serpiginous, fine desquamation.
Biopsy is performed and results are seen here. What cell type is depicted here, in the superficial
pustule?

A. Melanocyte
B. Neutrophil
C. Histiocyte
D. Eosinophil
E. Mast cell

Correct choice: B. Neutrophil

Explanation: The patient most likely has Sneddon-Wilkinson syndrome or subcorneal pustular
dermatosis. The histopathology shows a superficial neutrophilic pustule.

373- Which of the following histologic bodies is typically found in this condition?
A. Russell bodies
B. Cowdry Type A bodies

951
C. Henderson-Patterson bodies
D. Cowdry Type B bodies
E. Guarnieri bodies

Correct choice: B. Cowdry Type A bodies

Explanation: The concurrent presence of vesicles, pustules, erosions, and crusts is indicative of
primary varicella infection. Cowdry Type A inclusion bodies are seen in HSV and VZV infection,
and are characterized as large eosinophilic intranuclear inclusions with surrounding clear halos in
infected cells.
Guarnieri bodies are seen in smallpox, which clinically displays lesions at the same stage of
development (contrast this with the various stages of lesion development seen in varicella). Cowdry
Type B inclusion bodies are seen in polio. Henderson-Patterson bodies are seen in molluscum
contagiosum, and Russell bodies are found in rhinoscleroma and granuloma inguinale.

374- The patient just returned from a sunny vacation with this eruption. What is your diagnosis

A. Acute cutaneous lupus


B. Porphyria cutanea tarda
C. Seborrheic dermatitis
D. Rosacea

952
E. Phototoxic eruption

Correct choice: E. Phototoxic eruption

Explanation: The epidermis is still a basket weave so this process happened acutely.
There is no lymphocytic infiltrate that would be expected in lupus. We are not on sebaceous skin as
would be expected for seborrheic dermatitis and rosacea. Porphyria cutanea tarta results in a
subepidermal split with festooning of the dermal papillae.

375- This is a genodermatosis. Which statement is INCORRECT?

A. Nikolsky's sign can be positive.


B. Has a predilection for neck/ axillae, intertriginous areas
C. Superinfection with herpes simplex virus can occur in the genital region
D. The responsible gene is ATP2C1
E. The responsible gene encodes a Ca2+ pump located in the sarcoplasmic reticulum

Correct choice: E. The responsible gene encodes a Ca2+ pump located in the sarcoplasmic
reticulum

Explanation: This biopsy reveals diffuse suprabasilar acantholysis with a "dilapidated brick wall"
appearance, which is indicative of Hailey-Hailey disease (familial benign chronic pemphigus). The
responsible gene encodes a Ca2+ pump located in the Golgi apparatus (as opposed to Darier's
disease, which is due to a mutation in the ATP2A2 gene, which encodes a Ca2+ pump in the
endoplasmic reticulum). Hailey-Hailey disease is due to an autosomal dominant mutation in
ATP2C1. It presents initially with flaccid vesicles on an erythematous base, with a predilection for
the neck/ axillae & intertriginous areas. The vesicles rupture easily and this gives rise to macerated

953
or crusted erosions. Nikolsky's sign can be positive, and superinfection with herpes simplex virus
(usually HSV2) can occur in the genital region.

376- A 10-year-old girl undergoes a surgical excision of a circumscribed nodule on the face (shown
in image). In some areas there are shards of keratinous debris and calcification. The origin of this
following neoplasm is typified by which of the following?

A. Follicular neural network


B. Cornification of the inner root sheath
C. Follicular dermal papillae
D. Vellus hair follicles
E. Follicular matrical cornification

Correct choice: E. Follicular matrical cornification

Explanation: Pilomatricomas are benign neoplasms or cysts typified by follicular matrical


cornification. They presents as a solitary skin-colored or bluish nodule and are more common in
childhood and adolescence. Calcification and ossification can develop in late lesions. Histologically,
an early pilomatricoma frequently presents as a cyst with central matrical cornification. Pink
trichohyaline granules illustrate matrical cornification. The central anucleate cornified cells are
called ghost or shadow cells. Pilomatricoma is typified by follicular matrical cornification. Its pink
trichohyaline granules illustrate matrical cornification and are identifiable at the transition point.

954
955
Benign and Malignant Neoplasms

1- An elderly patient with no prior history of skin cancer points out a hyperpigmented skin lesion on
the trunk that clinically looks like the most common, non-melanocytic growth found mostly in older
individuals. For further evaluation and confirmation, you pull out your dermatoscope as a
diagnostic tool. Which of the following features would point away from a seborrheic keratosis?

A- Fissures
B- Fingerprinting
C- Telangiectasias arranged in regular loops
D- Pigment Network
E- Milia-like cysts

Correct choice: D. Pigment Network

Explanation: Seborrheic keratosis are the most common benign epidermal tumor which frequently
occur in individuals older than 30. Dermoscopic features are comedo-like openings, milia-like
cysts, fissures (brain-like cerebriform appearance), fingerprinting, lack of true pigment network,
and telangiectasias arranged in regular loops (hair-pin like vessels). The absence of pigment
network, branched streaks, and pigment globules are the key features that differentiate SKs from
melanocytic neoplasms.

2- Clinically, this lesion often has a blue hue and usually occurs on the face:

A- Apocrine hidrocystoma
B-Epidermoid inclusion cyst
C- Bronchogenic cyst
D- Eccrine hidradenoma
E- Myxoid cyst

Correct choice: A. Apocrine hidrocystoma

956
Explanation: Apocrine hidrocystoma are usually solitary, tranlucent nodules which may have a
bluish hue due to the Tyndall effect. Histologically, they have one or several large cystic spaces with
decapitation secretion. The other listed tumors are less likely to fit this clinical description.

3- The most likely diagnosis is?

A- CTCL
B - B-cell lymphoma
C- Angiosarcoma
D- Hemangioma
E- Merkel cell carcinoma

Correct choice: B. B-cell lymphoma

Explanation: These red juicy papules are characteristic for B-cell lymphoma. CTCL presents as
erythematous scaly patches and plaques. Angiosarcoma is typically a violaceous nodule on the
scalp. Merkel cell carcinoma is often a pink-skin colored solitary nodule on the head and neck.

4- An elderly native Floridian with a fair complexion and evident photodamage due to a history of
chronic sun exposure presents for a skin check. He has numerous erythematous, flat, rough macules
that are easily felt thorough his head, neck, forearms and dorsal hands. He usually presents to your
office every 6 months for treatment of these lesions with cryotherapy. The etiologies responsible for
the development of these skin lesions include which of the following?

A- UVA from sunlight


B- Formation of pyrimidine dimers in DNA
C- Mutations in patched within keratinocytes
D- Increasing apoptosis

957
E- Increased host immune response

Correct choice: B. Formation of pyrimidine dimers in DNA

Explanation: The etiology of actinic keratoses involves UVB from sunlight which is responsible for
the development of AKs. This triggers the formation of thymidine dimers in DNA & RNA, resulting
in mutated keratinocytes. The mutations occur on the tumor suppressor gene p53 within
keratinocytes resulting in impaired apoptosis. Clonal expansion of mutated keratinocytes occurs,
which may lead to formation of AKs.

5- All of the following accurately describe properties and mechanism of action of Denileukin
diftitox (ONTAK®) EXCEPT:

A- Is a systemic treatment option for CTCL


B- Is a diphtheria fusion toxin
C - Targets the interleukin-2 receptor
D- May cause capillary leak syndrome as an adverse reaction
E- Targets the CD28 protein on T cells

Correct choice: E. targets the CD28 protein on T cells

Explanation: Denileukin diftitox is a diphtheria fusion protein chemotherapeutic agent used for the
treatment of persistent or recurrent CTCL. It binds selectively to the high- and intermediateaffinity
IL-2 receptor (CD25+) on lymphocytes and is internalized by these cells. Inside the cells, the toxin
portion of the fusion protein is cleaved by proteolytic enzymes, causing cell death. It produces
durable responses and may forestall disease progression. The most frequent and clinically
significant adverse events include infusion reactions, capillary leak syndrome, hypoalbuminemia,
visual changes, consititutional symptoms, rash, and hepatobiliary disorders - many of these can be
managed without dose reduction. Deniluekin diftitox does not target the CD28 expression protein
on T cells.

958
6- A patient presents for treatment of a melanoma on the cheek with Breslow depth of 0.7 mm and
ulceration on pathology. There is no evidence of nodal metastasis. Based on the 8th edition AJCC,
what is the correct staging for this patient?

A- Stage 0
B Stage IA
C -Stage IB
D- Stage IIA
E- Stage IIB

Correct choice: C. Stage IB

Explanation: Tumors less than 0.8 mm WITH ulceration OR tumors 0.8 - 1mm (regardless of
ulceration) are a T stage of T1b. If there is no nodal or distant metstasis then the patient is a cancer
StageIB. Patients with tumors 1.01 - 2mm without ulceration are T2a, and if there is no nodal
metastasis they are also considered StageIB.
Stage 0 is melanoma in situ (T stage is referred to as Tis).
Stage 1A includes melanomas that are Breslow depth 0.01-0.7 WITHOUT ulceration. Note that
tumors should be reported with a single digit after the decimal, and rounded up or down
accordingly. Therefore a Breslow of 0.74 mm would be rounded down and reported as 0.7 mm, and
a Breslow of 0.75 would be rounded up and reported as 0.8 mm).
Stage IIA would be T2b or T3a and no nodal metastasis.
Stage IIB T3b or T4a and no nodal metastasis.

7- What is true about this tumor?

A- It is the second most common type of skin cancer


B- Telangiectasias are not commonly seen

959
C- Rombo syndrome has these tumors
D- Seen in Cowden syndrome
E- Mohs micrographic surgery is not an effective treatment

Correct choice: C. Rombo syndrome has these tumors

Explanation: This is a classic nodular basal cell. Mohs is often the most effective treatment. Rombo
syndrome presents with atrophoderma vermiculatum, basal cell carcinoma, milia, telangiectasias,
and acral erythema. The following syndromes can have multiple BCCs: Gorlin, Bazex-Dupre-
Christol, Rombo, Brooke-Spiegler, xeroderma pigmentosum, and Schöpf-Schulz-Passarge.

8- What will the histology of this lesion look like?

A- Small round blue cells with salt and pepper chromatin


B- Basaloid proliferations budding off the epidermis and in the
dermis
C- Atypical keratinocytes invading the dermis
D- Lentiginous proliferation of atypical melanocytes
E- Clonal keratinocytes within the epidermis

Correct choice: D. Lentiginous proliferation of atypical melanocytes.

Explanation: This is a lentigo maligna melanoma on dermoscopy which will show a lentiginous
proliferation of atypical melanocytes along the DEJ and invading into the dermis. It is not a basal
cell carcinoma which would be basaloid islands within the dermis, clonal keratinocytes within the
epidermis would be a seborrheic keratosis. Merkel cell carcinoma is characterized by salt and
pepper chromatin. SCC would be atypical keratinocytes invading the dermis.

960
9- Epithelioma cuniculatum is an HPV-associated verrucous carcinoma involving the:

A- Oral mucosa
B- Scalp
C- Sole
D- Penis
E- Trunk

Correct choice: C. Sole

Explanation: Verrucous carcinoma is a subtype of SCC that presents with well-defined exophytic
growths that can resemble large warts. Lesions are subclassified according to site. Epithelioma
cuniculatum is a form of verrucous carcinoma involving the sole. Verrucous carcinoma of the oral
mucosa is called oral florid papillomatosis. Verrucous carcinoma of the anogenital area is called
giant condyloma acumintum of Buschke-Lowenstein. Verrucous carcinoma of the scalp does not
have a unique name.

10- This is considered to be the juvenile counterpart of DFSP:

A- Juvenile hyaline fibromatosis


B- Giant cell fibroblastoma
C- Myxofibrosarcoma
D- Myofibromatosis
E- Plexiform fibrous histiocytoma

Correct choice: B. Giant cell fibroblastoma

Explanation: Giant cell fibroblastoma is CD34-positive, is mostly seen in male children on the
neck/trunk, and is thought to be a juvenile counterpart of DFSP. The other listed tumors are not as
closely related to DFSP.

11- This has what finding on dermoscopy?

A- Brown globules

961
B- Arborizing vessels
C- Blue-grey veil
D- Starburst pattern
E- Red lagoons

Correct choice: D. Starburst pattern

Explanation: This is a Spitz nevus. A starburst pattern suggests a Spitz nevus. A basal cell
carcinoma which would show arborizing vessels on dermoscopy. A blue-grey veil is associated with
melanoma. Brown globules is indicative of pigmented nests in the papillary dermis. Red lagoons
indicate a hemangioma.

12- A 65 year-old woman is referred to your office for evaluation of a skin lesion that has been
increasing in size over the past 5 years. You perform a skin biopsy that results as poorly
differentiated squamous cell carcinoma (SCC). Risk factors for cutaneous squamous cell carcinoma
(cSCC) include all of the following EXCEPT:

A- Kidney disease
B- Fitzpatrick skin types I and II
C- Chronic ulcers
D- Immunosuppression
E- Chronic sun exposure

Correct choice: A. Kidney disease

962
Explanation: Kidney disease in and of itself is not a risk factor for cSCC, however a patient who has
had a kidney transplant is at higher risk for the development of cSCC, 65x increased risk over the
general population due to the use of immunosuppressive agents to prevent organ rejection. Other
risk factors for SCC include: chronic sun exposure, skin types I and II, chemical carcinogens
(arsenic, tobacco, coal, tar), immunosuppression, chronic ulcers, burn scars, genetic syndrome (e.g.
xeroderma pigmentosa), male gender, and older age. The remaining answer choices are risk factors
for SCC.

13- Dermoscopic features suggestive of malignant melanoma include:

A- Lacunar pattern
B- Arborizing blood vessels
C- Maple leaf structures
D- D Glomeruloid vessels
E- Blue-white veil

Correct choice: E. Blue-white veil

Explanation: Asymmetry, multicomponent pattern, blue-whitish veil, parallel-ridge pattern, atypical


pigment network, uneven radial streaming, localized irregular and diffuse pigmentation, irregularly
distributed globules, and regression structures are all dermoscopic features suggestive of malignant
melanoma. Lacunar pattern is seen with cherry angiomas. Arborizing blood vessesl are seen with
basal cell carcinomas. Glomeruloid vessels are seen with squamous cell carcinoma in situ. Maple
leaf structures are seen with basal cell carcinoma.

14- Excisional biopsy was performed on this lesion and revealed melanoma in situ. In the photo
shown, what is the recommended margin for surgical excision?

A- 0.5-1.0cm
B- 1.0-1.5cm
C- 1.5-2cm
D- 2-2.5cm
E- 3cm

963
Correct choice: A. 0.5-1.0cm

Explanation: Lentigo maligna or melanoma in-situ requires a surgical margin of 0.5-1cm.

15- Which of the following markers do not stain melanocytic lesions:

A. CK7
B. Vimentin
C. S-100
D. HMB-45
E. All of these answers are correct

Correct choice: A. CK7

Explanation: CK7 will stain keratinocytes. Vimentin, S-100 and HMB-45 stains melanocytic
lesions.

16- The treatment for Merkel cell carcinoma is all of the following except:

A. Wide local excision 2-3 cm


B. Mohs
C. Radiation sensitive therapy
D. Chemotherapy
E. Local excision of 1-2 cm

964
Correct choice: A. Wide local excision 2-3 cm

Explanation: Merkel cell carcinoma is a type of aggressive cancer with a high rate of recurrence. It
is a neuroendocine carcinoma that is most commonly on sun exposed areas like the head and neck
in the elderly. Treatment includes wide local excision of 1-2 cm (per the NCCN guidelines), Mohs,
radiation, and chemotherapy.

17- When performing a biopsy of a suspected keratoacanthoma, which of the following is incorrect?

A- Fusiform incision through the entire KA may be performed


B- It is necessary to biopsy down to subcutaneous fat
C- A full-thickness shave biopsy is acceptable
D- A complete excisional biopsy may be performed
E- It is important to biopsy the lesion, even if it is less than 1cm

Correct choice: B. It is necessary to biopsy down to subcutaneous fat

Explanation: When considering a keratoacanthoma, it is not required to obtain a biopsy of the


specimen down to the subcutaneous fat. It is important to biopsy all lesions concerning for
keratoacanthoma, even those sized less than 1cm. Biopsy can be achieved either by complete
excisional biopsy, full-thickness shave biopsy, or fusiform incision through the entire KA including
its center and sides.

19- Your patient with extensive actinic damage and many of the lesions seen here is interested in
field therapy with a lightbased modality. On a cellular level, how does photodynamic therapy kill
cells that make up this lesion?

A- Heat shock proteins


B- Inhibition of pyrimidine nucleotide synthesis

965
C- Neutrophil-mediated, antibody-dependent cellular cytotoxicity
D- Reactive singlet oxygen
E- Proteosome inhibition

Correct choice: D. Reactive singlet oxygen

Explanation: The lesions seen here is an example of Bowen's disease (superficial squamous cell
carcinoma). Photodynamic therapy is a photochemical reaction in which a photosensitizing
molecule, under an activating wavelength, reacts with oxygen in the tissue to create reactive singlet
oxygen species. This leads to local necrosis. The other options do not explain how photodynamic
therapy works.

20- What pathway is involved with this tumors growth?

A- Hedgehog pathway
B.RAS pathway
C- WNTpathway
D- ERK pathway
E- MAPK pathway

Correct choice: A. Hedgehog pathway

Explanation: Basal cell carcinomas are associated with PTCH mutations which are in the sonic
hedgehog pathway.

966
21- A patient requests removal of a hyperpigmented plaque with a stuck-on appearance that is
eczematous and erythematous in appearance due to trauma. Under the microscope, the hallmark
histopathologic findings that you are most likely to see that help distinguish this lesion from other
neoplasms include:

A- Papillomatosis, acanthosis, hyperkeratosis


B- Hypogranulosis, hyperkeratosis, pseudoepitheliomatous hyperplasia
C- Hypergranulosis, parakeratosis, acanthosis
D- Parakeratosis, hyperkeratosis, vacuolated keratinocytes
E- Vacuolated keratinocytes, acanthosis, hyperkeratosis

Correct choice: A. Papillomatosis, acanthosis, hyperkeratosis

Explanation: Hallmark histopathologic findings of seborrheic keratoses include: acanthosis,


papillomatosis, hyperkeratosis, horn cysts and pseudohorn cysts. In contrast to a common wart,
vacuolated keratinocytes are not present in stucco keratoses, a clinicopathologic variant of
seborrheic keratoses.

22- Which of the following is true regarding digital HPVassociated squamous cell cancers?

A- The rate of metastasis approaches 15%.


B- HPV18 is the most common associated sybtype.
C- Mohs micrographic surgery yields a 20% recurrence rate.
D- Women outnumber men 2:1.
E- These lesions only occur in association with immunosuppression.

Correct choice: C. Mohs micrographic surgery yields a 20% recurrence rate.

Explanation: According to Riddel et al (JAAD 2011;64(6):1147- 1153), Mohs micrographic surgery,


although the treatment of choice, results in a 20% recurrence rate, which is significantly higher than
cutaneous SCC. HPV16 is most often implicated. Men outnumber women 2:1. The rate of
metastasis averages between 2-3%. Although common in transplant patients, HPV associated digital
SCCs can occur secondary to trauma and in immunocompetent patients.

967
23- What is the original function of the gene mutated in this tumor?

A- Inhibit p53
B- Inhibit c-KIT
C- Inhibit smoothened
D- Activate smoothened
E- Inhibit hedgehog

Correct choice: D. Inhibit smoothened

Explanation: PTCH is the most common gene mutated in basal cell carcinomas. The normal
function of PTCH is to inhibit smoothened. When PTCH is inactivated smoothened is free to
activate the sonic hedgehog pathway and stimulate the cell-cycle resulting in tumor cell
proliferation.

24- UVB induced mutations on the PTCH gene is associated with the development of:

A- BCC
B- Merkel cell carcinoma
C- Angiosarcoma
D- BCC and Merkel cell carcinoma
E- Merkel cell carcinoma and Angiosarcoma

Correct choice: A. BCC

Explanation: The p53 and PTCH genes are the major targets of UVB for the development of BCC.
Other genes involved include the Smoothened-activating mutations, and PTCH2 mutations. The
other listed tumors are not due to mutations in the PTCH gene.

968
25- Which form of BCC is the most common form in patients with HIV infection?

A- Superficial BCC
B- Nodular BCC
C- Morpheaform BCC
D- Infiltrative BCC
E- Micronodular BCC

Correct choice: A. Superficial BCC

Explanation: Superficial BCC is the most common form of BCC seen in patients with HIV.

26- A 56-year-old marathon runner presents with a superficial BCC On the left lower leg. He
prefers to avoid excision or dessication and curettage and presents 4 weeks later with the findings
seen here. Which of the following is the best statement regarding counseling for the patient?

A- Treatment with topical creams for superficial cancers has a lower cure rate than surgery
but has an excellent cosmetic outcome. The findings here are consistent with a robust
response and suggests a better clearance of the tumor at completion of therapy

B- The tumor here has grown significantly and the patient must now undergo Mohs surgery
for therapy given the size and location of the lesion

C- Desiccation and curettage has a similar cosmetic outcome to topical therapies (5FU or
imiquimod) as well as similar cure rate to excision

D- Mohs surgery is never appropriate for superficial basal cell or squamous cell carcinoma
in situ malignancies as they are minimally invasive

E- It is best to monitor the superficial basal cell cancer annually because it is not an
aggressive malignancy

969
Correct choice: A. Treatment with topical creams for superficial cancers has a lower cure rate than
surgery but has an excellent cosmetic outcome. The findings here are consistent with a robust
response and suggests a better clearance of the tumor at completion of therapy.

Explanation: Imiquimod can be a reasonable treatment for superficial basal cell cancers. It has an
excellent cosmetic outcome but can be very inflammatory, as seen in the image. The tumor has not
grown significantly, this is a reaction to the imiquimod. Dessication and curettage often has a worse
cosmetic outcome than topical chemotherapeutics and has a lower cure rate than excision. Based on
location and size, Mohs surgery is sometimes an appropriate option for superficial keratinocyte
cancers. It is not unreasonable to monitor a superficial BCC as they are slow growing tumors,
however it is not the best answer and annual follow-up may not be enough treatment.

27- Where is the classic location of this lesion?

A- Scalp
B- Chest
C- Back
D- Nose
E- Ear

970
Correct choice: A. Scalp

Explanation: This is an angiosarcoma as evident by the numerous atypical vessels, hemorrhage and
atypical endothelial cells within the atypical vessels. These tumors most commonly occur on the
scalp as violaceous nodules in elderly men.

28- The application of what topical medication has been shown to decrease the risk of developing
squamous cell carcinomas in the first year after treatment?

A- Ingenol mebutate
B- Imiquimod
C- Tacrolimus
D- Diclofenac
E- Fluorouracil 5%

Correct choice: E. fluorouracil 5%

Explanation: Fluorouracil 5%. A standard course of fluorouracil treatment (application twice daily
for 2 to 4 weeks) to the face and ears were shown to decrease the risk of SCC by 75% in the first
year after treatment, although it was not shown to decrease the risk of keratinocyte carcinomas over
a 4-year period. The remaining medications are FDA-approved for the treatment of actinic
keratosis, but have not yet been studied for chemoprevention of basal or squamous cell carcinomas.

29- Where is the classic location of this lesion?

A- Forehead
B- Dorsal nose

971
C- Upper cutaneous lip at the base of the nasal ala
D- Ear
E- Chin

Correct choice: C. Upper cutaneous lip at the base of the nasal ala

Explanation: This is a microcystic adnexal carcinoma. These typically occur on the upper cutaneous
lip at the base of the nasal ala in young adult females. It is imperative the base of the tumor be seen
to ensure it is completely removed as the tumor is aggressive.

30- What will this show on dermoscopy?

A- Arborizing vessels
B- Red lagoons
C- Blue-grey veil
D- Brown globules
E- Starburst pattern

Correct choice: A. Arborizing vessels

Explanation: This is a basal cell carcinoma which will show arborizing vessels. A hemangioma will
show red lagoons on dermoscopy. A spitz nevus has a starburst pattern. A blue-grey veil is
associated with melanoma. Brown globules is indicative of pigmented nests in the papillary dermis.

972
31- All of the following statements are true regarding the ‘MCW Melanoma Cocktail’ except:

A- it is a mixture of monoclonal antibodies that include MART-1


B- it is performed intraoperatively
C- it is a mixture of monoclonal antibodies that include Melan-A
D- it demonstrates micrometastases in sentinel lymph nodes
E- it is a mixture of polyclonal antibodies to tyrosinase

Correct choice: E. it is a mixture of polyclonal antibodies to tyrosinase

Explanation: The ‘MCW Melanoma Cocktail’ is an immunostain made up of monoclonal antibodies


to MART-1, Melan-A and tryrosine. It is used intraoperatively during sentinel lymph node biopsy
and allows for rapid and accurate determination of micrometastases. The mixture of stains are
monoclonal antibodies; not polyclonal.

32- What is the mechanism of action of vorinostat, a therapy for refractory cutaneous T cell
lymphoma?

A- Histone deacetylase inhibitor


B- IL-2 receptor inhibitor
C- Antibody against CTLA-4
D- JAK 1/3 inhibitor
E- Alkylating agent

Correct choice: A. Histone deacetylase inhibitor

Explanation: Vorinostat and romidepsin are histone deacetylase (HDAC) inhibitors. The most
common side effects are fatigue, gastrointestinal symptoms and reversible thrombocytopenia.
Denileukin difitox is an IL-2 receptor inhibitor, and is associated with capillary leak syndrome.
Ipilimumab is an antibody to CTLA-4. Tofacitinib is a JAK 1/3 inhibitor.

33- Spindle cell lipoma is most commonly found on the:

A- Head

973
B- Lower extremities
C- Buttocks
D- Breast
E- Posterior shoulder

Correct choice: E. Posterior shoulder

Explanation: Spindle cell lipoma is a solitary benign tumor seen in adult male patients, and is most
often located on the posterior shoulder or neck regions. The tumor histologically consists of mature
collagen, adipose tissue, spindle cells, and mast cells. Treatment is with local excision. None of the
remaining answer choices represent the most common location for spindle cell lipoma.

34- This tumor commonly results from mutations in p53. Which of the following is correct?

A- It is the most common type of skin cancer in Caucasians


B- Oral variants most commonly present on the dorsal tongue
C- It is typically less aggressive in patients with CLL
D- Organ transplant patients have a 10-fold increased risk of developing this tumor
E- Tumors arising within scars/chronic ulcers have the highest risk of metastasis

Correct choice: E. Tumors arising within scars/chronic ulcers have the highest risk of metastasis

Explanation: This is SCC of the ear. SCCs arising within scars/chronic ulcers metastasize in
~30-40% of cases, whereas those arising on the lip or ear have a 10-20% risk of metastasis.
Recurrent SCCs have a metastasis risk of up to 30%. SCC is the 2nd most common skin cancer in
Caucasians (BCC is most common). Oral SCC most commonly presents on the lateral tongue. SCC

974
is typically more aggressive in patient with CLL. Organ transplant patients have a 65-fold increased
risk for developing cutaneous SCC.

35- Which of the following is true about this neoplasm?

A- A sentinel lymph node biopsy is standard of care for tumors thicker than 1 mm
B- Locally advanced or metastatic variants may respond to sonic hedgehog inhibitors
C- Grows very rapidly over several weeks/months before stabilizing
D- Associated with a polyomavirus infection
E- Occurs at sites of chronic burns or fistulae

Correct choice: B. Locally advanced or metastatic variants may respond to sonic hedgehog
inhibitors

Explanation: The dermoscopic picture is of a pigmented BCC.


Answer choice 1 refers to melanomas; choice 3 refers to keratoacanthoma; choice 4 refers to merkel
cell carcinoma; choice 5 refers to squamous cell carcinoma.

36- The green color in chloroma is secondary to:

A- Stromelysin
B- Chloracetate
C- Fumarase
D- Myeloperoxidase
E- Alkaline phosphatase

975
Correct choice: D. Myeloperoxidase

Explanation: Chloromas are greenish tumor grossly secondary to involvement of the skin in acute
granulocytic leukemia. The green color is secondary to myeloperoxidase. The other listed answers
do not contribute to the green color of chloroma.

37- Which of the following is FALSE?

A- These lesions are often mistaken for basal cell carcinomas.


B- In the absence of ulceration, sentinel lymph node biopsy should be considered if tumor
thickness is 0.8mm or greater.
C- These lesions are not treated differently than their pigmented variants.

D- These lesions often have a better prognosis than their pigmented variants.
E- Increased tumor thickness is associated with a worse prognosis.

Correct choice: D. These lesions often have a better prognosis than their pigmented variants.

Explanation: The correct answer is D (D is false). Amelanotic melanomas are often diagnosed later
(i.e. thicker Breshlow depth) than their pigmented counterparts, which is the reason for their poorer
prognosis. Amelanotic melanomas are often mistaken for basal cell carcinomas as both can appear
clinically similar. The recently published AJCC 8th Edition Melanoma Staging System revised the
definitions of T1a and T1b so that T1a melanomas include those <0.8 mm without ulceration while
T1b melanomas include those 0.8-1 mm with or without ulceration and those <0.8 mm with
ulceration. Mitotic rate is no longer a T1 category criterion but should be documented for all
invasive primary melanomas. Thus, in the absence of ulceration, sentinel lymph node biopsy should
be considered if tumor thickness is 0.8mm or greater. Amelanotic melanomas are treated identically

976
to their pigmented counterparts, and increased tumor thickness is associated with a worse prognosis
(irrespective of the clinical subtype of melanoma).

38- This lesion may be associated with which of the following?

A. Cowden disease
B. Rombo syndrome
C. Reed's syndrome
D. Brooke-Spiegler syndrome
E. Gorlin syndrome

Correct choice: D. Brooke-Spiegler syndrome

Explanation: This is a biopsy of a cylindroma. Multiple cylindromas seen with CYLD mutation
either in cylindromatosis or in conjunction with other adnexal neoplasms in Brooke-Spiegler
syndrome.

39- A 40 year-old man presents with a slow-growing, large, firm, nodular tumor on the back.
Immunohistochemical staining of the biopsy specimen from this tumor is positive for CD34 and
negative for Factor XIIIa. The patient subsequently undergoes wide excision with adjuvant
chemotherapy. Which of the following chemotherapeutic agents was most likely used?

A. Imatinib
B. Vismodegib
C. Avelumab
D. Ipilimumab

977
E. Nivolumab

Correct choice: A. Imatinib

Explanation: Imatinib has been used in the treatment of primary or locally recurrent
dermatofibrosarcoma protuberans. Even with wide excision, the 5 year recurrence rate for this
tumor can be 20-25%. This neoplasm commonly possesses a reciprocal translocation t(17;22)
resulting in fusion of collagen 1alpha1 and platelet derived growth factor B (a fusion oncogene).
Activation of the platelet derived growth factor receptor associated with overexpression of platelet
derived growth factor is central to the development of DFSP. Imatinib is a protein tyrosine kinase
inhibitor used primarily to treat chronic myelogenous leukemia with the Philadelphia chromosome
defect. Imatinib also inhibits the tyrosine kinases associated with platelet derived growth factor and
stem cell factor. Therefore, it directly inhibits the platelet derived growth factor receptor signaling
cascade, which plays a critical role in the pathogenesis and growth of DFSPs. While Imatinib has
been successful in clinical trials, it is not yet FDA-approved for the treatment of DFSP. DFSP stains
positive for CD34 and negative for Factor XIIIa, which differentiates it from a dermatofibroma
(stains positive for Factor XIIIa and negative for CD34).

Vismodegib is a smoothened-inhibitor that is FDA-approved for the treatment of basal cell


carcinoma. Avelumab is an anti-PD-L1 monoclonal antibody that was recently FDA-approved for
the treatment of merkel cell carcinoma. Ipilimumab is an anti-CTLA-4 monoclonal antibody that is
FDA-approved for the treatment of melanoma. Nivolumab is an anti-PD-1 monoclonal antibody
that is FDA-approved for the treatment of melanoma.

40- Typical dermoscopic features of this common benign lesion that typically begins to appear
during the fourth decade of life include all of the following except:

A. Fat fingers
B. Reticulated pigment network

978
C. Milia-like cysts
D. Cerebriform surface
E. Light brown parallel structures

Correct choice: B. Reticulated pigment network

Explanation: Typical dermoscopic features of seborrheic keratoses include: milia-like cysts,


irregular crypts, fissures/ridges, blue-gray lobules, light brown fingerprint-like parallel structures,
fat fingers (the gyri of a cerebriform surface). A reticulated pigment network is not a common
feature of seborrheic keratoses.

41- Which of the following would you expect to find on dermoscopy of this lesion?

A. Blue-grey ovoid nests

B. Milky red globules


C. Orange crust
D. Maple leaf structures
E. Spoke wheel vasculature

Correct choice: B. Milky red globules

Explanation: This is an image of melanoma, which is associated with milky red globules on
dermoscopy.

979
42- Which of the following is TRUE regarding this lesion?

A. Ulceration is the most important prognostic factor


B. Immunosuppression is not a risk factor
C. Female gender is a poor prognostic factor
D. It is the most common type seen in darker-skinned patients
E. Anti-PD-1 antibodies are not helpful in treatment

Correct choice: D. It is the most common type seen in darker-skinned patients

Explanation: The associated image depicts an acral lentiginous melanoma (ALM), which is the
most common type of melanoma in darker-skinned individuals. Notably, and very sadly, the famous
musician Bob Marley was diagnosed with ALM in 1977 and succumbed to this malignancy in 1981
as it had metastasized to his lungs and brain.

Breslow depth, not ulceration, is the most important prognostic factor for melanoma.
Immunosuppression increases one's risk of melanoma by a factor of 3-4. Male gender is a poor
prognostic factor. Anti-PD-1 antibodies (e.g. nivolumab, pembrolizumab) are proven to prolong
survival in patients with metastatic melanoma.

43- This lesion is most likely to possess a mutation in which of the following?

980
A. NRAS
B. BAP-1
C. CDKN2A
D. BRAF
E. C-kit

Correct choice: D. BRAF

Explanation: The pictured lesion is a melanoma. The most common mutation in a melanoma is in
the BRAF gene (~50% of advanced or unresectable melanomas). NRAS mutations are the next
most commonly found mutation (~30% of melanomas). BAP-1 (BAP-1 tumor predisposition
syndrome), CDKN2A (melanoma pancreatic cancer syndrome), and C-kit may also be mutated in
cases of melanoma.

44- This syndrome presents with multiple keratoacanthomas appearing suddenly during childhood
or adolescence and inherited in an autosomal dominant pattern:

A. Ferguson-Smith
B. Grzybowski
C. Gorlin
D. KA centrifugum
E. Buschke-Lowenstein

Correct choice: A. Ferguson-Smith

Explanation: Patients that have a sudden appearance during childhood of multiple


keratoacanthomas have Ferguson-Smith type of KA. This is autosomal dominant and are the KAs
are typically self-healing.

981
Grzybowski is typically diagnosed in adulthood with the sudden appearance of hundreds to
thousands of lesions in a disseminated fashion.
Gorlin syndrome is characterized by the appearance of multiple BCCs (not KAs) during childhood.
It is also characterized by odontogenic keratocysts of the jaw and skeletal defects (e.g.,
macrocephaly, hypertelorism, frontoparietal bossing, spina bifida, or rib abnormality, among
others). Tumors associated with this disease include medulloblastoma, meningioma, ovarian
fibromas (bilateral), and cardiac fibromas. It is inherited in an autosomal dominant pattern, and due
to a mutation in PTCH gene.

KA centrifugum is a subtype of solitary KA that may reach a size of up to 20 cm in diameter.


Buschke-Lowenstein tumor is a verrucous carcinoma caused by HPV 6 and 11 that is locally
invasive and destructive but rarely metastatic.

45- Biopsy of one of the lesions pictured reveals spindle cells forming slit-like vascular channels.
Which of the following is also associated with the cause of these lesions?

A. Bacillary angiomatosis
B. Merkel cell carcinoma
C. Angiosarcoma
D. Multicentric Castleman disease
E. Nasopharyngeal carcinoma

Correct choice: D. Multicentric Castleman disease

Explanation: The clinico-pathologic correlation is best for Kaposi sarcoma, which is caused by
HHV-8 (KSHV). Multicentric Castleman disease and primary effusion lymphoma are also
associated with HHV-8. The other remaining answer choices are not associated with HHV-8.

982
46- What is the likely etiology of the pictured lesion?

A. PTCH mutation
B. CDKN2A mutation
C. Lichenoid infiltrate
D. HPV 5
E. p53 mutation

Correct choice: E. p53 mutation

Explanation: This is a SCC of the tongue. SCCs most commonly have mutations in p53.
PTCH mutations are in basal cell carcinomas. CDKN2A mutations are in familial melanomas. A
lichenoid infiltrate would be seen with lichen planus which is usually on the buccal mucosa. Heck’s
disease is due to HPV 13 and 32.

47- You perform a biopsy of a pink, soft, dome-shaped papule and the histology reveals a
predominantly intradermal melanocytic proliferation composed of nests and sheets of epithelioid
melanocytes with large pleomorphic nuclei and light eosinophilic cytoplasm. Immunohistochemical
staining is positive for BRCA1 associated protein-1. Which of the following malignancies is this
patient most at risk of developing?

A. Breast cancer
B. Ovarian cancer
C. Hodgkin's lymphoma
D. Mesothelioma
E. Pancreatic cancer

Correct choice: D. Mesothelioma

983
Explanation: This clinico-pathologic picture is most consistent with the BRCA1 associated
protein-1 (BAP1) cancer syndrome. BAP1 is a nuclear deubiquitinating enzyme that functions as a
tumor suppressor via its role in DNA damage repair. The BAP1 cancer syndrome is a rare
autosomal dominant genetic syndrome typified by the development of mesotheliomas and uveal
melanomas. Patients with BAP1 cancer syndrome develop several, distinct, melanocytic neoplasms.
These are raised, pink or tan, dome-shaped, benign lesions that have been referred to in the
literature as “melanocytic BAP1-mutated atypical intradermal tumors/MBAITs” or “BAPomas.”

Less commonly, cutaneous melanomas, various types of carcinomas (mostly from the kidney and
gallbladder), sarcomas, and brain tumors can arise in the BAP1 cancer syndrome. Thus, this patient
is less likely to develop one of the other listed answer choices compared to mesothelioma.

48- The clear-cell variant of this neoplasm is associated with which of the following?

A. Brooke-Spiegler syndrome
B. Cowden disease
C. Rombo syndrome
D. Diabetes mellitus
E. Hypothyroidism

Correct choice: D. Diabetes mellitus

Explanation: This is a syringoma. The clear-cell variant (a rare histological variant of syringoma
that is clinically indistinguishable from an ordinary syringoma) has been associated with diabetes
mellitus.

984
49- What is the most common gene mutated in this type of lesion?

A. PMS
B. c-KIT
C. p53
D. MSH
E. PTCH

Correct choice: E. PTCH

Explanation: This is a basal cell carcinoma. The most common gene mutation is PTCH.
p53 is mutated in SCCs, PMS2 and MSH6 are lost in sebaceous neoplasms of Muir-Torre. c-KIT is
mutated in acral melanoma, mucosal melanoma, and mast cell disorders.

50- Which of the following treatments carries the lowest risk of recurrence in the condition
pictured?

A. Pembrolizumab
B. Imatinib mesylate
C. Radiation therapy
D. Mohs micrographic surgery
E. Wide local excision

985
Correct choice: D. Mohs micrographic surgery

Explanation: Standard treatment of dermatofibrosarcoma protuberans (DFSP) consists of complete


surgical extirpation. The rate of recurrence is lower following Mohs micrographic surgery
compared to wide local excision.

This question asks the examinee to identify DFSP presenting as a characteristic firm red-brown
tumor on the shoulder, and know the treatment that will impart lowest recurrence risk. Initial
treatment of this tumor is complete surgical removal, with studies demonstrating lower risk of
recurrence with Mohs micrographic surgery (choice 4) versus wide local excision (choice 5). The
programmed cell death-1 inhibitor pembrolizumab (choice 1) has not been used to treat DFSP.
Imatinib mesylate (choice 2), which interferes with the activity of the COL1A1-PDGFB fusion
protein found in DFSP, is FDA-approved for unresectable, recurrent, and metastatic cases.
Radiation (choice 3) has been utilized for unresectable or recurrent DFSP, and would not be an
appropriate choice for initial therapy.

51- Merkel cell carcinoma should be treated with what size surgical margins?

A. 2mm
B. 5mm
C. 1-2cm to subcutaneous
D. 1-2cm to fascia
E. 5cm

Correct choice: D. 1-2cm to fascia

Explanation: Merkel cell carcinoma is an aggressive rare tumor of the skin accounting for less than
1% of cutaneous malignancies. Also known as neuroendocrine cancer of the skin, this tumor
presents as a painless red to violaceous, firm, solitary, nodule that usually presents on sun exposed
areas such as the head, neck and upper extremities. These tumors present usually during the 6th and
7th decades, and have a 2 year survival rate of 50-70%. Because of this tumor's high potential for
regional and distal metastasis, this tumor should be excised with wide local excision with 1-2cm
surgical margins to the depth of fascia or pericranium, or treated with Mohs Surgery.

52- On dermoscopy, which of the following findings most strongly supports the correct diagnosis?

986
A. Starburst pattern
B. Blue-whitish veil
C. Maple leaf structures
D. Regression structures
E. Irregular streaks

Correct choice: C. Maple leaf structures

Explanation: Maple leaf structures on dermoscopy support the diagnosis of basal cell carcinoma.

This question asks the examinee to identify a pigmented BCC and select the most closely associated
dermoscopic finding. In the case of this dark blue-black irregular papule at the alar crease, initial
concern for melanoma is quite reasonable. However, on close inspection, there is a subtly rolled
border, suggesting the correct diagnosis, pigmented BCC. Dermoscopy can assist in distinguishing
between pigmented BCCs and other pigmented lesions. Among the answer choices, maple leaf
structures (choice 3) are most closely associated with BCC. The starburst pattern (choice 1) is
classically seen in pigmented spindle cell nevus of Reed. Blue-whitish veil (choice 2), regression
structures (choice 4), and irregular streaks (choice 5) are features of melanoma, not routinely
observed in pigmented BCC.

53- The keratoacanthoma variant characterized by the sudden appearance during childhood or
adolescence of multiple KAs is called:

A. Gorlin syndrome
B. Xeroderma pigmentosa
C. Ferguson-Smith
D. Grzybowski
E. None of these answers are correct

987
►C

The Ferguson-Smith is a keratoacanthoma variant characterized by the sudden appearance during


childhood or adolescence of multiple KAs that may resolve and later-on reappear. This condition is
inherited in an autosomal dominant pattern

54- The treatment of choice for this lesion shown is:

A. Radiation therapy
B. Imiquimod
C. 5 Flourouracil
D. Wide excision with 2cm margins
E. Mohs surgery

►E

The treatment of choice for Dermaotfibrosarcoma protuberans is Mohs surgery. Radiation therapy
has been used, however has limited value as solitary therapy for thsi tumor. Radiation therapy can
be used as an adjunct to wide surgical excision. Classically, these tumors should be excised with
3cm margins. The recurrence rate associated with these tumors can be 10 -20 percent with wide
excision with 3 cm margins. With Mohs surgery, the recurrence rate ranges from 0% to 6%. 5FU
and Imiquimod are not effective modalitites in treating DFSPs, as it infiltrates deep into the
subcutaneous tissue.

55- Mutations in which gene would likely be found in the neoplastic cells of this lesion?
A. PATCH

988
B. p53
C. Fumarate hydratase
D. CREBBP
E. p63

►B

Squamous cell carcinoma is the second most common cancer of the skin. Mutations in the tumor
suppressor p16 and p53 are commonly found in SCC's. Normally, UV damage upregulates p53
thereby delaying cell cycle progression. DNA damage can then be repaired or the cell could
undergo apoptosis. In squamous cell carcinoma, p53 exhibits loss of heterozygosity due to C to T or
CC to TT mutations.

56 -A patient with a innumerable disseminated keratoacanthomas, including lesions on the larynx


and oral mucosa:

A. Is unlikely to have palmoplantar involvement


B. Likely has an underlying immune deficiency
C. Is at high risk for myelodysplasia
D. Likely inherited their condition in an autosomal dominant manner
E. Likely developed them during adulthood

►E

This patient has the Grzybowski type of keratoacanthomas. Typically diagnosed in adulthood,
these patients have the sudden appearance of hundreds of small lesions in a disseminated

989
fashion. The lesions can be found anywhere on the body including palms, soles, larynx, and oral
mucosa.

57 -The patient is a 45 year old male complaining of red, chapped lower lip. Which of the following
lasers is the most appropriate to treat this condition?

A. Pulsed Dye Laser


B. Nd:YAG laser
C. CO2 laser
D. Diode laser
E. Laser treatment is not an option

►C

The patient has actinic cheilitis. Notice the red, scaly lower lip, with erosions and fissures. The
CO2 laser is currently a common treatment alternative for this condition.

58 -Which of the following ethnic groups are commonly diagnosed with dermatosis papulosa nigra:

A. Asians
B. Hispanics
C. African-Americans & Hispanic patients
D. Caucasians
E. No difference between ethnic groups

990
►C

Characterized by the presence of multiple, small, hyperpigmented, sessile SKs on the face, DPN is
typically diagnosed on the African-American and Hispanic population.

59- Intermittent sun exposure with painful sunburns is a predisposing factor for the development of:

A. Atypical nevi
B. Seborrheic keratosis
C. Malignant melanoma
D. Atypical nevi and Malignant melanoma
E. All of these answers are correct

►D

It has been reported that the risk for the development of atypical nevi and melanoma is higher than
twofold with a history of five or more episodes of painful sunburn during adolescence.

60- The following lesion is the classic presentation of:

A. BCC
B. Merkel cell carcinoma
C. Melanoma
D. CTCL
E. Keratoacanthoma

991
►E

Keratoacanthomas present as a solitary, firm, dome-shaped papule with a cratiform center.

61 -A patient having Mohs surgery for a squamous cell carcinoma on the ear has tumor invading the
cartilage and perineural invasion. What stage disease does the patient have?

A. T0
B. T1
C. T2
D. T3
E. T4

►C

TX Primary tumor cannot be assessed T0 No evidence of primary tumor Tis Carcinoma in s itu T1
Tumor ≤2 cm in greatest dimension with <2 high-risk features∗ T2 Tumor >2 cm in greatest
dimension with or without one additional high-risk feature, or tumor any size with ≥2 high-risk
features∗ T3 Tumor with invasion of maxilla, mandible, orbit, or temporal bone T4 Tumor with
invasion of skeleton (axial or appendicular) or perineural invasion of skull base. *High risk
features: Depth/invasion >2 mm thickness or Clark level ≥IV,Perineural invasion, Primary site ear,
Primary site hair-bearing lip, Poorly differentiated or undifferentiated.

62- What would you expect to see under dermoscopy of this vascular neoplasm?

A. Red sacculae
B. Arborizing blood vessels
C. Hair pin telangectasia
D. Milky red globules
E. Blue-grey ovoid nests

►A

992
Hemangioma have a characteristic appearance under dermoscopy. Typically, they have a maroon
lagoon or red sacculae appearance.

63- The most important mutated gene associated with a predisposition to develop malignant
melanoma is:

A. PTCH
B. CDKN2A
C. PTCH2
D. None of these answers are correct
E. All of these answers are correct

►B

The CDKN2A gene located on chromosome 9p21 is the most important mutated gene associated
with MM of the listed choices. BRAF is another important mutation.

64 -A patient is diagnosed with squamous cell carcinoma. As a doctor you explain to the patient that
the metastatic rate is:

A. 0.3-16%
B. 15%-20%
C. 18%-25%
D. 25%-30%
E. 30%-45%

►A

Patients that have SCC have a 0.3-16% of chance of metastasis. The location, size and type of SCC
can determine the possibility of metastasis.

65- Which of the following is the most common initial site of metastasis from a primary BCC?

993
A. Lungs
B. Regional lymph nodes
C. Bone
D. Liver
E. Pleura

►B

The metastatic potential of BCC is very low with rates ranging from 0.0028 to 0.1%. The head and
neck region is the most frequent location of the primary tumor with regional lymph nodes being the
most common site of metastasis. The lungs, bone, liver, and pleural are also potential sites of
metastasis.

66 -Chloroma is a characteristic cutaneous manifestation of:

A. Tuberous sclerosis
B. Sweetǁs syndrome
C. Neurofibromatosis
D. Leukemia
E. Pseudomonas sepsis

►D

Chloromas, also termed granulocytic sarcomas, are a localized tumor composed of immature
granulocytic cells. They frequently have a greenish coloration due to the presence of
myeloperoxidase and most commonly affect the bone. The condition most often occurs in patients
with acute leukemia of the myeloid type.

67- Which phase of the cell cycle does p53 regulate?

A. G1
B. G2
C. S phase
D. Mitosis

994
E. Meiosis

►A

p53 is a tumor suppressor gene which arrests cell cycle in G1 as it controls the transition from G1 to
S. It also downregulates BCL-2. Mutations in p53 are associated with Li-Fraumeni syndrome as
well as the development of squamous cell carcinomas.

68- A 65 year-old female with multiple actinic keratosis on the face under treatment with 5-FU.
According to the image and aforementioned information, you may conclude that:

A. The patient must immediately stop treatment since unexpected side effects have
developed
B. The patient has been compliant with 5-FU treatment and the appearance of
inflammation, erythema and erosions are expected
C. The image is not relevant to 5-FU treatment
D. None of these answers are correct
E. All of these answers are correct

►B

Compliance is a key feature in treatment with 5-FU. Erythema, inflammation and erosion must
develop and is considered a sign of successful treatment.

69- Which of the following would you not expect to see under dermoscopy?

995
A. Maple leaf pattern
B. Arborizing blood vessels
C. Blue-grey ovoid nests
D. Orange crust
E. Milky red globules

►E

Dermoscopy is a useful tool in differentiating a pigmented basal cell carcinoma from melanoma.
Basal cell carcinomas may have arborizing blood vessels, maple leaf pattern, blue-grey ovoid nests,
and orange crust or ulcer. Milky red globules are sometimes seen in melanoma.

70- Which of the following melanoma subtypes is more consistently seen in dark-skinned
individuals?

A. Acral lentiginous melanoma


B. Superficial spreading melanoma
C. Lentigo maligna melanoma
D. Nodular melanoma
E. Superficial spreading melanoma and Lentigo maligna melanoma

►A

Acral lentiginous melanoma is the predominant type of melanoma in dark-skinned individuals. It is


usually located on the soles, palms, and subungeal region of patients in their fifth to sixth decade of
life.

71- Which of the following is an immunhistochemical marker for Merkel Cell Carcinoma?:

A. S-100
B. Vimentin
C. HMB-45
D. Neuron specific enolase
E. All of these answers are correct

996
►D
Neuron specific enolase stains merkel cells. Vimentin stains melanocytic lesions, sarcomas and
lymphomas. S-100 and HMB-45 stains melanocytic lesions, such as melanoma.

72- Which one of the following malignancies is associated with HPV infection?

A. Verrucous carcinoma
B. Metastatic melanoma
C. Basal cell carcinoma
D. Sebaceous carcinoma
E. Atypical fibroxanthoma

►A

Verrucous carcinomas are low-grade carcinomas which are slow-growing and metastasize very late
in the course. The presence of HPV has been demonstrated in cases both by electron microscopy
and DNA hybridization.

73- Dermatofibrosarcoma protuberans is:

A. Cytogenetically characterized by reciprocal translocation t(17;22)(q22;q13)


B. Factor XIIIa positive
C. CEA positive
D. CD 34 positive
E. Cytogenetically characterized by reciprocal translocation t(17;22)(q22;q13) and CD 34
positive

►E

A chromosomal reciprocal translocation t(17;22)(q22;q13), and supernumerary ring chromosome


have been reported as cytogenetic characteristics of DFSP. Typically DFSP is CD34 positive and
factor XIIIa negative, allowing its differentiation from dermatofibroma.

997
74- Amplification of which of the following genes is associated with Merkel cell carcinoma?

A. L-Myc
B. C-Myc
C. GLI1
D. CDKN2A
E. PTEN

►A

The L-Myc gene has been found to be amplified in Merkel cell carcinoma but not in normal skin.
C-Myc has been found to be amplified in neuroblastoma. GLI1 is a transcription factor involved in
hedgehog signaling and a potential target in basal cell carcinomas. CDKN2A is implicated in
familial forms of melanoma that are associated with pancreatic cancer. PTEN is a tumor suppressor
gene that is mutated in Cowden Disease, Bannayan-Riley-Ruvalcaba Syndrome, and Proteus
Syndrome.

75- The most common location for a basal cell carcinoma is:

A. Lower eyelid
B. Forehead
C. Ears
D. Back
E. Shoulders

►A

The most common location for a BCC is the lower eyelid. It is the most common epithelial tumor of
the eyelid.

76- If left untreated, which of the following is not at risk for malignant transformation?

A. Bowenoid papulosis
B. Cutaneous horn

998
C. Actinic cheilitis
D. Leukoplakia
E. Stucco keratosis

►E

Cutaneous horn can overlie an AK or SCC or a benign lesion and it presents as a conical
protuberance arising from an erythematous base. Actinic cheilitis results from the confluence of
multiple AKs on the lips. Leukoplakia is a clinical diagnosis and is defined as a white patch in the
oral cavity. It is the most common premalignant condition of the oral cavity. Bowenoid papulosis
manifests clinially as multiple red-brown warty papules that histologically represent high grade
squamous intraepithelial lesions.

77- Currently, the surgical margin for melanomas that measure less than 2 mm in thickness is:

A. 1 mm
B. 0.5 cm
C. 1 cm
D. 2 cms
E. 3 cms

►C

The current surgical margins are 0.5 cms for melanoma in situ, 1 cm for melanomas that measure
less than 2mm in thickness and 2cm for melanomas >2mm.

78- Denileukin diftitox (ONTAK®):

A. is a systemic treatment option for CTCL


B. is a diphtheria fusion toxin
C. targets the interleukin-2 receptor
D. None of these answers are correct
E. All of these answers are correct

999
►E

Denileukin diftitox is as diphtheria fusion toxin that targets the IL-2 receptor. It is a systemic
treatment alternative for recalcitrant or advance CTCL.

79- Maple leaf-like structures seen on dermoscopy are characteristic of which lesion?

A. Seborrheic keratoses
B. Pigmented basal cell carcinoma
C. Dermal nevi
D. Melanoma
E. Hemangioma

►B

Maple leaf-like structures seen on dermoscopy are an important diagnostic criterion for pigmented
basal cell carcinomas. They correspond to the heavily pigmented cells in the bas aloid cells.

80- Which one of the following agents has demonstrated potential benefit as a chemopreventive to
UV-induced skin cancer?

A. Prostaglandin E2
B. Vitamin D
C. Arachidonic acid
D. Celecoxib
E. Vitamin E

►D

Cyclooxygenase-1 and -2 and enzymes that catalyze the conversion of arachidonic acid to
prostaglandins. It is believed the prostaglandin E2 (PGE2), whose levels are increased by ultraviolet
irradiation, is pro-inflammatory and may contribute to skin carcinogenesis. In a study by Orengo
et.al., hairless mice who were given celecoxib were found to have a significantly longer latency
period between exposure to ultraviolet light and the development of skin carcinomas.

1000
81- UVB induced mutations on the PTCH gene is associated with the development of:

A. BCC
B. Merkel cell carcinoma
C. Angiosarcoma
D. BCC and Merkel cell carcinoma
E. Merkel cell carcinoma and Angiosarcoma

►A

The p53 and PTCH genes are the major targets of UVB for the development of BCC. Other genes
involved include the Smoothened-activating mutations, and PTCH2 mutations.

82- Periungual Squamous cell carcinoma has been linked to which HPV type(s)?

A. 6, 11
B. 2, 4
C. 16
D. 13
E. 8

►C

Infections associated with Human Papilloma Virus can produce growths on the epithelial or
mucosal surfaces. There are over 100 strains of these viruses, and some of these strains can
predispose to intraepithelial carcinomas, particularly when involving the anal or g enital mucosa. In
general, HPV strains 16 and 18 are classified as his risk virus types and can be associated with
cervical cancer , oral cancer, anal cancer and periungual cancers. HPV 6 and 11 are associated with
condyloma acuminata; HPV types 2, 4 are associated with common warts; HPV type 13 has been
associated with Heck's disease and HPV type 8 has been associated with epidermal dysplasia
verruciformis.

1001
83- Which of the following features of thin melanomas (<1 mm thick) has not been associated with
an increased risk of metastasis?

A. Regression
B. Location
C. Age
D. Gender
E. P53 expression

►E

Regression in malignant melanoma appears histologically as a focal area of fibrosis with


lymphocytes and melanophages in the papillary dermis. The presence of regression precludes
accurate measurement of true thickness of the melanoma. Extensive regression in thin melanomas
has a significant association for the risk of metastases.

84- What is the most common site of metastasis for this dermal tumor?

A. Lung
B. Brain
C. Kidney
D. Liver
E. Bone

►A

Dermatofibrosarcoma protuberans is a rare, low-grade dermal sarcoma. Typically, the lesion occurs
as a painless subcutaneous mass that grows slowly. This malignancy typically has lateral spread but
invade deep. Metastasis is rare but has been reported to the lung.

1002
85- Which substance does p53 normally activate to promote apoptosis via inhibition of bcl-2?

A. p21
B. p16
C. Puma
D. Mdm2
E. Akt

►C

p53 is the most commonly mutated tumor suppressor gene involved in human cancer and is often
mutated in SCC and BCC. p53 acts via two main pathways, 1) activation of p21 (Cdk inhibitor)
which leads to cell cycle arrest, and 2) activation of Puma which inhibits Bcl-2 (apoptosis inhibitor)
thereby leading to cell death. CKDN2A, a gene that when mutated leads to a risk of melanoma, acts
via 1) activation of p16 (another Cdk inhibitor) and 2) activation of p14ARF which inhibits Mdm2
(which normally degrades p53). Akt is involved in the PI3K-Akt signaling pathway and inhibits cell
cycle arrest and apoptosis.

86- The most common location for primary mucinous carcinoma is:

A. neck
B. eyelid
C. areola
D. scrotum
E. nose

►B

Mucinous carcinoma presents as a slowly growing, asymptomatic, round, erythematous nodule on


the head and neck. Forty percent of cases occur on the eyelid. Histologically it is characterized by
the presence of large areas of mucin (―sea of mucousǁ) with small islands of basophilic epithelial
cells. Primary mucinous carcinoma of the skin has an indolent course. Local recurrence occurs in
1/3 of patients following excision. The rate of metastasis is low (9.6%).

1003
87- All of the following disorders may manifest as exfoliative dermatitis except

A. Behcet's disease
B. Psoriasis
C. Pemphigus foliaceus
D. Drug reaction
E. Sezary syndrome

►A

The image shows erythrodermic patient with generalized desquamation of skin. Skin biopsy and
blood tests the diagnosis of Sezary syndrome. Sezary syndrome is a leukemic variant of mycosis
fungoides, is characterized by the triad of pruritic erythroderma, g eneralized lymphadenopathy, and
the presence of Sezary cells more than 1,000 cell/mm3 (abnormal, large hyperconvoluted
lymphocytes) in peripheral blood . Other manifestations include scaling and fissuring of palms and
soles, alopecia, pruritus, peripheral edema, and nail dystrophy. Exfoliative dermatits can be
manifested in many conditions other than Sezary syndrome inculude pemphigus foliaceus, psoriasis
and drug reaction.

88- The human papilloma virus type associated with red brown smooth and warty papules is

A. HPV 1
B. HPV 5
C. HPV 7
D. HPV 13
E. HPV 16

►E

1004
Bowenoid papulosis manifests clinically as multiple red-brown warty papules or confluent planques
on the external genitalia. These lesions may resemble genital warts but histologically
represent high-grade squamous intraepithelial lesions. Bowenoid papulosis is caused by infection by
HPV and linked to HPV 16, 18, 31, 35, and 39.

89- A patient has a malignant melanoma 1.6mm thick with ulceration and a micrometastasis in 1
node. The patient's staging according to the American Joint Committee on Cancer Staging System
is:
A. IIC
B. IIIA
C. IIIB
D. IIIC
E. IV

►C

An ulcerated tumor of any size with micrometastasis in 1 node is T1-4b N1a M0. This corresponds
to stage IIIB.

90- The type #1 site of metastasis for dermatofibrosarcoma protuberans is:

A. Lungs
B. Kidney
C. Stomach
D. Bladder
E. Colon

►A

The #1 site of metastasis for DFSP are lungs. It has a low grade of metastasis <5% and treatment is
wide local excision with 3 cm margins.

92- Immunohistochemistry of dermatofibrosarcoma protuberans typically reveals:

1005
A. CD34 negative and factor XIIIa positive
B. CD34 negative and factor XIIIa negative
C. CD34 positive and factor XIIIa positive
D. CD34 positive and factor XIIIa negative
E. None of these answers are correct

►D

Typically DFSP is CD34 positive and factor XIIIa negative, allowing its differentiation from
dermatofibroma.

93- All of the following statements are true regarding angiosarcomas EXCEPT:

A. They occur more commonly in Caucasians than in non-Caucasians


B. Men are more often affected than women
C. They are rarely symptomatic
D. Radiation is usually employed after surgical excision
E. Cervical lymph nodes are a common site of metastases

►C

Angiosarcomas are very rare, aggressive vascular tumors. They occur most commonly in the head
and neck region of white, elderly individuals. Men are more commonly diagnosed with this
neoplasm. The lesion initially arises as a painless, purple macule or plaque on the scalp or face.
Later on it becomes an elevated bluish or purple nodule that may ulcerate. Common symptoms
include bleeding, edema, and ultimately pain. Cervical lymph node and hematogenous metastases
commonly occur. Wide surgical excision is the treatment of choice, with radiation therapy usually
employed after surgical excision.

94- Which of the following are true for the sign of Leser-Trelat?

A. Lesions are commonly located on the chest and back


B. Classically described to appear in a ―Christmas treeǁ pattern
C. Usually associated with internal malignancies

1006
D. Eruptive nature
E. All of these answers are correct

►E

Characterized by the sudden appearance of multiple SKs, the sign of Leser-Trelat may have lesions
located anywhere in the body. However, the eruptionǁ is classically described to appear in a
Christmas treeǁ pattern commonly located on the chest and back. It may be associated with internal
malignancies such as adenocarcinomas.

95- The risk of metastasis from SCC increases with:

A. Tumor size
B. Depth of invasion
C. Degree of differentiation
D. Immunosupression
E. All of these answers are correct

►E

Tumor size (greater than 2 cms), location (lips, ears, eyelids), depth of invasion, degree of
differentiation, perineural invasion, immunosupression, as well as recurrent tumors and those that
arise in areas of chronic inflammation are all factors that increase the risk of metastasis.

96- All of the following statements regarding Bowen's disease are true EXCEPT:

A. Lesions arising on the lower limbs are more common in men than in women
B. The basement membrane remains intact on histopathology
C. 5% of patients with Bowen's disease develop invasive squamous cell carcinoma
D. Chronic sun exposure is a risk factor for Bowen's disease
E. Mucosal Bowen's disease may appear as a verrucous plaque

►A

1007
The most common locations for BD include the head and neck regions and the extremities. BD
arising on the lower limbs is frequently found in women, whereas lesions located on the ears and
scalp are more common in men. The basement membrane remains intact on histopathology. 5% of
patients develop invasive SCC. Risk factors include chronic sun exposure, immunosuppression,
HPV, arsenic exposure, and ionizing radiation. Mucosal BD may appear as a verrucous or polypoid
plaque, as an erythroplakic patch, or as a velvety red plaque.

97 -The incidence of cutaneous squamous cell carcinoma in organ transplant recipients is increased
by how much compared with the general population?

A. 2 fold
B. 5 fold
C. 10 fold
D. 20 fold
E. 65 fold

►E

The risk of skin cancer in organ transplant patients is dramatically increase and may be more
aggressive. In one study, SCCǁs had a 65x increased incidence, BCC 10x, and melanoma 3.4x.

98- What is the most common location for pagetoid reticulosis?

A. Trunk
B. Head and neck
C. Hands and feet
D. Flexural sites of upper and lower extremities
E. Genitals

►C

Pagetoid reticulosis is an indolent cutaneous T-cell lymphoma. Pagetoid reticulosis favors an acral
distribution and typically presents as scaly oval plaques. Epidermotropism is present, with tumor

1008
cells being CD4 positive or CD8 positive. Another interesting immunohistochemical finding is the
absence of CD45 expression.

99- A patient presents with tender papules with a pseudo-Darier's sign. She has other family
members with the same condition. Screening should be performed to rule out which malignancy?

A. Gastric carcinoma
B. Ovarian carcinoma
C. Renal cancer
D. Testicular cancer
E. Lung cancer

►C

Reed's syndrome is an autosomal dominant disease with incomplete penetrance characterized by


uterine and cutaneous leiomyomas. Clinically, leiomyomas appear as flesh colored or pinkish brown
dermal papules or nodules that range from 0.2 to 2.0 cm in diameter. Their presentation may
otherwise be quite variable. They may be isolated or many in number, may be variably distributed
or dermatomal, and may be asymptomatic or painful in response to pressure or cold. The
predisposition gene for Reedǁs syndrome has been localized to chromosome 1q42.3-43 and the gene
encoding fumarate hydratase. Currently, the United States National cancer Institute is
recommending screening for all patients with leiomyomatosis to evaluate for an occult renal
malignancy.

100- All of the following cytokines have demonstrated therapeutic benefit in the treatment of
melanoma EXCEPT:

A. IFN-alpha
B. IL-2
C. TNF-alpha
D. IL-10
E. GM-CSF

►D

1009
IFN-alpha, IL-2, TNF-alpha, have all been demonstrated to have some therapeutic benefit in t he
treatment of melanoma. GM-CSF is the immune stimulator in the vaccine Talimogene
laherparepvec. IL-10 has been used to treat inflammatory disorders such as atopic dermatitis and
psoriasis.

101- A 60 year-old female presents with a well-demarcated, scaly, erythematous plaque on her right
shin. The biopsy shows full thickness epidermal atypia with scattered mitotic figures and overlying
parakeratosis. Howerver, the basement membrane remains intact. According to the aforementioned
information, the diagnosis is:

A. BCC
B. SCC
C. Bowen's disease
D. Angiosarcoma
E. CTCL

►C

Bowenǁs disease arising on the lower limbs is frequently found in women; whereas lesions located
on the ears and scalp are more common in men. The epidermal dysplasia does not interrupt the
basement membrane confirming the diagnosis of SCC in situ.

102- What would be the characteristic histopathologic findings of this lesion?

A. Full thickness keratinocytic atypia


B. Cornoid lamella
C. Pale staining cells
D. Horn pseudocysts

1010
E. Wedge shaped granular layer with lichenoid infiltrate

►B

Five clinical variants of porokeratosis are recognized. They are the classic porokeratosis of Mibelli,
disseminated superficial actinic porokeratosis, prokeratosis palmaris et plantaris disseminata, linear
porokeratosis, and punctate porokeratosis. On histopathology they have varying degrees of a
cornoid lamella. This appears as a column of parakeratosis.

103- Topical 5-Fluorouracil:

A. Interferes with the synthesis of DNA and RNA


B. Is an alternative for the treatment of actinic keratosis
C. May cause pruritus and burning at the site of application
D. All of these answers are correct
E. None of these answers are correct

►D

Topical 5-FU blocks the methylation reaction of deoxyuridylic acid to thymidylic acid, thus
interfering with the synthesis of DNA and RNA. Normal side effects during treatment include
pruritus and burning at the site of application. It is a treatment option for patients with multiple AKs
within an area.

104- Knowing that the patient in this picture has a biopsy confirmed BCC, which of the following
would be the treatment of choice:

1011
A. Conventional excision
B. Imiquimod
C. Radiation
D. Mohs micrographic surgery
E. Cryosurgery

►D

Providing the highest cure rates, Mohs micrographic surgery is the treatment of choice for large
BCCs (1cm on the face or 2 cms on the trunk), located on high-risk anatomic areas (ear, eyelids,
lips, genitals, nose, temples). , Morpheaform BCCs, tumors with positive margins after
conventional excision, and recurrent BCCs, are also an indication for Mohs surgery.

105- Mutations of the p53 gene has been associated with the development of:

A. Melanoma
B. Kaposi's sarcoma
C. Actinic keratosis
D. Molluscum contagiosum
E. All of these answers are correct

►C

UVB radiation triggers the formation of thymidine dimers both in DNA and RNA, resulting in
mutated keratinocytes. The mutations occur on the tumor suppressor gene p53 within the
keratinocytes resulting in impairment of the mechanism of apoptosis. Therefore, clonal expansion
of mutated keratinocytes may occur leading to the formation of AKs.

106 -Spitz nevi can appear during childhood. The risk factors for metastatic Spitz include all of the
following except:

A. Ulceration
B. Increased Breslow thickness
C. Increased mitoses

1012
D. More H-RAS mutations
E. Location on the back

►E

Features of atypical spitz include location in the back over one cm and can have irregular color and
border. The risk factors for metastatic Spitz is ulceration, increased Breslow thickness, atypical
mitotic features and more H-RAS mutations as opposed to BRAF and NRAS.

107- Histologically, this lesion is shows plump, polygonal cells arranged in nests and fascicles with
granular cytoplasm. Which immunohistochemical stain would be positive?

A. Colloidal iron
B. Von kossa
C. Warthin-starry
D. S-100
E. CD68

►D

Granular cell tumors are benign growths which typically occur on the tongue. They are typically
well-circumscribed, raised, firm nodules. Histologically, the cells have uniform nuclear
characteristics and granular cytoplasm due to presence of lysozyme. The lesions are PAS positive
and S-100 positive.

108- The most common site for intra oral melanoma is?

A. buccal
B. soft palate
C. hard palate
D. gingiva
E. uvula

1013
►C

Multiple case series have shown that the hard palate (and specifically the anterior hard palate/
alveolar arch) is the highest risk location for intra-oral melanoma. Prognosis is generally
worse for oral melanoma, which is usually due to delay in diagnosis and presentation with more
invasive disease.

109- Which of the following variants of mycosis fungoides is best diagnosed using a punch biopsy
instead of a broad superficial shave biopsy?

A. Woringer-Kolopp pagetoid reticulosis


B. Syringotropic mycosis fungoides
C. Ketron-Goodman pagetoid reticulosis
D. Poikilodermatous mycosis fungoides
E. Sezary syndrome

►B

Syringotropic and folliculotropic mycosis fungoides (MF) are the variants of MF that should be
diagnosed by punch biopsy. Both variants of pagetoid reticulosis, Woringer-Kolopp disease and the
disseminated Ketron-Goodman disease, are best diagnosed with a broad shave biopsy. Classic and
poikilodermatous MF should also be diagnosed with a shave biopsy. To rule out Sezary syndrome,
flow cytometry should be performed.

110- Sentinel lymph node biopsy in malignant melanoma:

A. Has gained acceptance for the treatment of MM of intermediate thickness (14mm)


B. Is mainly being used for a more accurate staging
C. Is not recommended
D. None of these answers are correct
E. Has gained acceptance for the work up of MM of intermediate thickness (14mm) and is
mainly being used for a more accurate staging

►E

1014
Sentinel lymph node biopsy has gained acceptance as part of the work up for melanomas 1-4 mm in
thickness. Even though it has been widely accepted due to its low morbidity and high feasibility, the
role of SLN biopsy in survival has not been established. It is mainly being used for a more accurate
staging, prognosis and to determine if further adjuvant therapy is necessary.

111- Which of the following immunohistochemistry marker is negative in angiosarcomas?

A. CD31
B. CEA
C. Cytokeratin
D. CD34
E. Factor VIII

►B

Angiosarcomas are CD31, CD34, factor VIII, and cytokeratin positive. The carcinoembryonic
antigen (CEA) stains positive in Paget‘s disease, metastatic adenocarcinoma, and tumors with
eccrine differentiation.

112- Which of the following melanoma scenarios have the best prognosis?

A. Twenty-one year old female with primary lesion located on the right lower leg
B. Twenty-one year old male with primary lesion located on the chest
C. Thirty-six year old male with primary lesion located on the back
D. Twenty-one year old female with primary, ulcerated lesion located on the right lower leg
E. Thirty-six year old male with primary lesion located on his left lower leg with palpable
inguinal lymph nodes

►A

Increasing age and male gender have a negative effect on survival. Primary lesions located on the
extremities have a better prognosis than those located on the trunk, head or neck region. Ulceration
is also considered a poor prognostic factor. Furthermore, there is a significantly lower survival for

1015
those patients with palpable metastatic nodes (macrometastasis) when compared to those with
micrometastatic nodes (nonpalpable).

113- What is the most common location of metastasis of basal carcinoma?

A. liver
B. lung
C. bone
D. lymph node
E. skin

►D

Size of a BCC is related to its risk for metastasis. 3cm BCC has a 1.9% risk of metastasis. If a BCC
is >10cm it has approximately a 50% risk of metastasis. If metastasis does occur it most commonly
metastasizes to lymph nodes>lung>bone>skin>liver.

114 -An elderly white man presents with a slowly enlarging, well-demarcated pink, scaly plaque on
the neck. Biopsy of the lesion reveals epidermal dysplasia and keratinocytic disorganization with
preservation of the basement membrane. Hyperkeratosis and parakeratosis are also present, and
numerous atypical keratinocytes are seen throughout the epidermis, with loss of polarity, atypia, and
mitoses. A chronic inflammatory infiltrate is present in the upper dermis. Which of the following
treatment(s) should be considered for this patient?

A. Conventional excision
B. Mohs micrographic surgery
C. Imiquimod
D. Conventional excision and Mohs micrographic surgery
E. All of these answers are correct

►D

The patient has Bowen's Disease, or squamous cell carcinoma in situ. Multiple treatment options
are available, including conventional excision, Mohs micrographic surgery, imiquimod,

1016
cryosurgery, and 5-FU, among others. Conventional excision is the most commonly used method.
With this method, not only is the lesion removed, but it also provides the specimen for histologic
verification to rule out invasive SCC. Recurrence rates of 5% have been reported with conventional
excision. Mohs micrographic surgery is an option when the lesion is located in areas where there is
an increased risk of sub-clinical spreading, or when tissue sparing is a priority. Imiquimod is not
approved to treat SCCIS.

115- Compared with the general population, what is the overall risk of developing cutaneous and
systemic malignancies in organ transplant recipient?

A. 4 fold
B. 10 fold
C. 25 fold
D. 50 fold
E. 100 fold

►A

Organ transplant recipients are at increased risk of having both systemic and cutaneous develop.
The estimated increased risk is 3-4 times that of the general population. However, the risk of skin
cancer alone is much higher (e.g. SCC ― 65x increase, BCC 10-fold increase, Melanoma 3.4x
increase). This increased risk is thought to be partially due to the immunosuppressant agents used to
prevent graft rejection.

116- When performing a biopsy of a suspected keratoacanthoma:

A. Fusiform incision through the entire KA may be performed


B. It is necessary to biopsy down to subcutaneous fat
C. A full-thickness shave biopsy is acceptable
D. A complete excisional biopsy may be performed
E. All of these answers are correct

►E

1017
When considering a keratoacanthoma, it is important to obtain a biopsy of the specimen down to the
subcutaneous fat. This can be achieved either by complete excisional biopsy, full-thickness shave
biopsy, or fusiform incision through the entire KA including its center and sides.

117- Merkel cell carcinoma should be treated with what size surgical margins?

A. 2mm
B. 5mm
C. 1cm
D. 3cm
E. 5cm

►D

Merkel cell carcinoma is an aggressive rare tumor of the skin accounting for less than 1% of
cutaneous malignancies. Also known as neuroendocrine cancer of the skin, this tumor presents as a
painless red to violaceous, firm, solitary, nodule that usually presents on sun exposed areas such as
the head, neck and upper extremities. These tumors present usually during the 6th and 7th decades,
and have a 2 year survival rate of 50-70%. Because of this tumor's high potential for regional and
distal metastasis, this tumor should be excised with wide local excision with 3cm surgical margins.

118- Which of the following is a clinical stage of keratoacanthomas?

A. Plaque
B. Mature
C. Proliferative
D. Resolving
E. All of these answers are correct except plaque

►E

Keratoacanthomas have three, consecutive, clinical stages: proliferative, mature and resolving. The
proliferative stage is characterized by the appearance of a rapid growing papule. This phase is
followed by the mature stage when the lesion acquires its ch aracterisitic dome-shaped appearance

1018
with a central, keratinous core. Tumor resorption occurs during the involution stage resulting in a
slightly depressed, hypopigmented scar.

119- All of the following are risk factors for metastasis from a primary squamous cell carcinoma
EXCEPT:

A. Increasing tumor size


B. Location on the ear
C. Recurrent tumor
D. Tumor within ulcer
E. All of these answers are correct

►E

The risk of metastasis from a primary SCC increases with tumor size (>2cm), location (lips, ear, and
eyelid, among others), depth of invasion, immunosuppression, degree of differentiation, and
perineural invasion. Recurrent tumors and SCCs arising in areas of chronic inflammation (such as
osteomyelitis, burn scars, or ulcers) are also considered high risk for metastases.

120 -Which of the following lesions are ǁbetter felt than seenǁ:

A. Seborrheic keratosis
B. SCC in situ
C. Actinic keratosis
D. Sebaceous hyperplasia
E. All of these answers are correct

►C

Typical AKs appear as flat, erythematous, rough papules that are better felt than seen.

121- What is the eponym for this metastasis to the umbilicus?

1019
A. Tripe palm
B. Sign of Leser-Trelat
C. Pityriasis rotunda
D. Trousseau syndrome
E. Sister Mary Joseph Nodule

►E

Sister Mary Joseph nodule is a cutaneous metastasis that is most commonly associat ed with
stomach, large bowel, ovary, and pancreatic cancer. Tripe palms are most commonly associated with
lung, Leser-Trelat with adenocarcinomas of the stomach, colon and breast, Trousseau Sign with
carcinoma of the pancreas and pityriasis rotunda with hepatocellular carcinoma and gastric cancer.

122 -On histologic examination of the adenocarcinoma of the perineal area, which of the following
stains would NOT be positive?

A. PAS
B. Mucicarmine
C. CEA
D. EMA
E. HMB45

►E

Extramammary Paget's cutaneous adenocarcinoma of glandular differentiation. Approximately 25%


of these tumors are associated underlying neoplasms. Histologically, the Paget cells stain positively
with PAS, mucicarmine, CEA, EMA, LMW keratin. They are HMB45 negative which is a
melanocytic marker.

1020
123 -The cure rate of cryotherapy as a treatment for actinic keratoses is:

A. 80%
B. 85%
C. 90%
D. 95%
E. 99%

►E

Cryotherapy is the most common treatment for AKs, with a cure rate of 98.8%.

124- Which of the following is true regarding poikilodermatous mycosis fungoides?

A. Majority of cases are predominantly CD8(+)


B. Patients typically have a later age of onset compared to classic mycosis fungiodes
C. Can be associated with LyP
D. More women than men affected
E. Patients typically do not respond well to phototherapy

►C

While there is an overrepresentation of CD8+ cases compared to classic mycosis fungoides, more
cases are still predominantly CD4+. Patients typically present at a younger age (median age 44
years), with a slight male predominance. There is an excellent response to photo therapy with
clinical improvement in ~90% of patients treated. There is an increased association with LyP
compared to other types of mycosis fungoides. (Abbott et al, JAAD 2011; in press)

125- The most common locations of microcystic adnexal carcinoma include all of the following,
except:

A. Perioral
B. Nasolabial
C. Trunk

1021
D. Periorbital
E. Perioral,nasolabial, and periorbital

►C

Microcystic adnexal carcinomas are more commonly located in the perioral, nasolabial, or
periorbital areas.

126- Merkel cell carcinoma can be summarizes with certain clinical features. All of the following
are features except:

A. Asymptomatic
B. Expanding rapidly
C. Older than 50
D. Older than 60
E. Ultraviolet-exposed area in person with fair skin

►D

Merkel cell carcinoma can be summarized with "AEIOU" that stands for expanding rapidly,
immune suppression, older than 50 years, ultraviolet-exposed area in person with fair skin.

127- In which of the following ethnic groups is squamous cell carcinoma the most common type of
skin cancer?

A. Asian Indians
B. Caucasians
C. Hispanics
D. Japanese
E. Chinese

►A

1022
SCC is the most common type of skin cancer in Asian Indians and Blacks. BCC is the most
common type of skin cancer in Caucasians, Japanese, Chinese, and Hispanic people.

128- All of the following are true of melanoma-associated leukoderma except:

A. Lesions resemble vitiligo


B. Portends a worse prognosis
C. Seen in patients with metastatic disease but no primary lesion
D. Histology resembles that of a halo nevus
E. Lesions develop distant to melanoma

►B

Hypomelanosis associated with melanoma most commonly may be seen in three ways. The first
type is analogous to a halo nevus. The second is a remote leukoderma distant from the primary
lesion. The third is a vitiligo like leukoderma. It may portend a better prognosis in compa rison to
others with the same stage of disease.

129- Which of the following is commony seen in seborrheic keratoses when examined with
dermoscopy?

A. Maroon lagoons
B. Network
C. Pigment globules
D. Milia like cysts
E. Blue-gray veil

►D

The main dermoscopic features of seborrheic keratoses include comedolike openings, fissures,
milialike cysts, fingerprinting and lack of true pigment network. In fact, it is the lack of true
pigment network, branched streaks, and pigment globules that differentiate SKs from melanocytic
lesions.

1023
130- A 60 y/o male smoker with multiple sclerosis presents for evaluation of a painful 5cm x 4cm
plaque on right posterior shoulder, which appeared over 3 months. What is the most likely
diagnosis?

A. Irritated Seborrheic Keratosis


B. Squamous Cell Carcinoma
C. Basal Cell Carcinoma
D. Metastatic Carcinoma
E. Malignant Melanoma

►C

Basal cell carcinoma. It has a male to female ratio 2:1. UVB damages DNA and produces a C -T
transition mutation (65% of BCC). P53 and PTCH genes are commonly affected. It has a slow,
indolent growth potential: metastatic potential 0.0028% to 0.1%.; known to metastasize to lymph
nodes and lungs.

131- An elderly man presents to your office with a telangiectatic, violaceous 1cm dome-shaped
nodule on the neck. Biopsy reveals large, solid nests of cells of intermediate size, with a trabecular
pattern at the periphery. These cells involve the dermis and spread into the subcutaneous fat, but
spare the overlying epidermis. The cells are round and uniform in size, with a round to oval nucleus,
small nucleoli, and evenly dispersed chromatin. Numerous mitotic figures and necrotic areas are
seen. Neuron specific enolase is positive. Which of the following is true regarding this patient's
diagnosis?

A. Mohs micrographic surgery is contraindicated in treatment of this lesion


B. Vimentin and desmin stains may be positive
C. S-100 stains should be positive
D. This lesion may contain ACTH
E. The neoplasm should not contain gastrin

►D

This patient has a Merkel cell carcinoma. Vimentin, desmin, and S-100 are consistently absent in
MCC. Mohs micrographic surgery has been used successfully for the treatment of MCC, with or

1024
without adjuvant therapy. This neoplasm may sometimes contain several neuropeptides including
vasoactive intestinal peptide, calcitonin, ACTH, gastrin, and somatostatin.

132- The translocation for dermatofibrosarcoma protuberans is:

A. Chr 17 and 22
B. Chr 7 and 10
C. Chr 9 and 13
D. Chr 20 and 2
E. Chr 29 and 30

►A

The translocation for DFSP is Chr 17 and 22. These chromosomes involve the fusion of PDGF beta
chain and collagen type I alpha I genes. DFSP are generally CD34+ and factor XIIIa negative.

133- What virus is most closely associated with these lesions in this HIV infected patient?

A. Human herpes virus 2


B. Human herpes virus 6
C. Human herpes virus 8
D. Cytomegalovirus
E. Ebstein Barr virus

►C

Kaposi's sarcoma is an AIDS defining illness. It if found at a much higher incidence in homosexual
men. Human herpes virus 8 is thought to be pathogenic in this tumor.

134- What is the most likely diagnosis?


A. Acquired digital fibrokeratoma
B. Supernumery digit

1025
C. Verruca
D. Infantile digital fibroma
E. Acrochordon

►A

The condition shown is an acquired digital fibrokeratoma. It is a pedunculated, acral lesion with
collarette which is defined by the lack of nerve twigs and bone on pathologic exam.

135- Dermoscopy would be the least helpful in evaluating which lesion?

A. Pigmented basal cell carcinoma


B. Congenital melanocytic nevus
C. Seborrheic keratosis
D. Amelanotic melanoma
E. Hemangioma

►D

Amelanotic melanoma are difficult to diagnosis with the naked eye and even with the dermoscope.
Pigmented basal cell carcinomas may have a characteristic maple leaf pattern,
arborizing blood vessels, and /or blue-grey ovoid nests. Seborrheic keratoses often have comedolike
openings and milia cysts. Hemangiomas typically have red lagoons.

1026
136- The most common location of Merkel cell carcinoma is:

A. Digits
B. Trunk
C. Genitalia
D. Extremities
E. Head and neck

►E

Merkel cell carcinoma usually presents as a violaceous, solitary, dome-shaped nodule, most
commonly located on the head and neck region.

137- A 40 year-old female patient presents with the following lesion (see image). The biopsy report
shows duct like structures, tadpole structures within a fibrotic stroma. According to the
aforementioned information, the most likely diagnosis is:

A. Microcystic adnexal carcinoma


B. Seborrheic keratosis
C. Dermatofibrosarcoma protuberans
D. Merkel cell carcinoma
E. None of these answers are correct

►A

One of the most common locations for MAC includes the perioral area. Histologically it presents
with poorly demarcated tumor cells invading the dermal and subcutaneous tissue. Islands of
basaloid keratinocytes, horn cysts and duct structures are also seen within a desmoplastic stroma.

1027
138- All of the following may increase the incidence of SCC, EXCEPT:

A. Sun exposure
B. Immunosuppression
C. Increasing age
D. Proximity to the equator
E. All of the following may increase the incidence of SCC

►E

The incidence of SCC increases with age, and it is 35 times higher in individuals older than 75
years of age when compared to ages 50-55. The incidence doubles for each 8 to 10 degree decline
in latitude, therefore, individuals living closer to the equator have a greater risk.

139- Which of the following statements regarding prognosis for malignant melanoma is true?

A. Increasing age has a positive effect on survival.


B. Patients with primary lesions located on the extremities (except acral lesions) have a
worse prognosis than those with tumors located on the trunk.
C. There is a higher survival rate for patients with palpable metastatic nodes compared to
those with micrometastatic nodal disease.
D. For stage IV disease, patients with non-visceral metastases (eg skin, subcutis, distant
lymph nodes) have a better prognosis compared with those with visceral metastases.
E. Male gender has a positive effect on survival.

►D

Increasing age and male gender have a negative effect on survival in melanoma. Patients with
primary lesions located on the extremities (except acral lesions) have a better prognosis than those
with tumors located on the head, neck, or trunk. There is a significantly lower survival for patients
with palpable metastatic nodes (macrometastasis) when compared to those with micrometastatic
nodes. For stage IV disease, patients with non-visceral metastases (eg skin, subcutis, distant lymph
nodes) have a better prognosis compared with those with visceral metastases.

1028
140- A child presents with a giant congenital nevus overlying the back of the skull, extending onto
the shoulders. It is ~15% body surface area, sparing the face and anterior neck. Which test should
be ordered?

A. A head CT
B. A head MRI
C. A skull plain film X-ray
D. A skin biopsy
E. A PET scan

►B

Giant congenital nevi overlying the spinal columna nd skull can be associated with neurocutaneous
melanosis. Presenting symptoms include increased cranial pressure, spinal cord compression or
leptomeningeal melanoma. A MRI should be performed in these children to ruleout CNS
involvement. The other tests would not be as useful in this situation.

141 -What is the most common location for this rapidly growing tumor?

A. Head and neck


B. Chest
C. Back
D. Arms
E. Legs

►A

1029
Merkel cell carcinoma a cutaneous neuroendocrine carcinoma that usually occurs on the head and
neck. It is a high grade malignant tumor with a 5-year mortality rate of 30-64%. On histopathology,
there are trabecular strands of basophilic cells that stain with a characteristic paranuclear dot pattern
with cytokeratin 20.

142 -This lesion is associated with which of the following:

A. HPV 5
B. HPV 11
C. HPV 4
D. HPV 7
E. HPV 1

►B

HPV 6 and 11 are most often associated with anogenital warts or condyloma acuminata. They can
present as sessile lesions on the skin or peducnulated cauliflower lesions. HPV type 5 is associated
with epidermodysplasia verruciformis, HPV 4 is assoicatd with common warts, HPV type 7 is
associated with Butcher warts, and HPV type 1 is associated with common warts.

143- A 50 year-old female patient presents with a reddish indurated plaque located on her right
clavicular area (see image). The biopsy shows the presence of monomorphic spindle cells in a
storiform pattern, and deep subcutaneous infiltration. According to the aforementioned information,
the most likely diagnosis is:

A. Microcystic adnexal carcinoma


B. Seborrheic keratosis
C. Dermatofibrosarcoma protuberans
D. Merkel cell carcinoma
E. None of these answers are correct

►C

1030
DFSP intially arises as an asymptomatic, reddish or skin colored indurated plaque, which may later
enlarge, and become raised, firm and multinodular. DFSP is histologically characterized by the
presence of monomorphic spindle cells arranged in a "storiform" or "cartwheel" pattern.

144- Dermoscopic features suggestive of malignant melanoma include:

A. Presence of 2 or more colors within the lesion


B. Blue-whitish veil
C. Asymmetric radial streaming
D. Abrupt interruption of pigment network in the periphery
E. All of these answers are correct

►E

Asymmetry, multicomponent pattern, blue-whitish veil, parallel-ridge pattern, atypical pigment


network, uneven radial streaming, localized irregular and diffuse pigmentation, irregularly
distributed globules, and regression structures are all dermoscopic features suggestive of malignant
melanoma.

145- Which of the following is the most important prognostic indicator in a patient with cutaneous
lymphoma?

A. Age
B. Primary vs secondary cutaneous
C. Gender
D. Extent of cutaneous involvement
E. Subtype of lymphoma

►B

When staging lymphoma, it is critical to determine whether the lymphoma is primary cutaneous
arising in the skin or secondary cutaneous arising in association with nodal or extranodal tumor. The
prognosis is worse in secondary when compared to primary lymphomas, irrespective of histologic
diagnosis.

1031
146- Which of the following immunologic drugs has been shown to increase survival of patients
with stage III and stage IV melanoma?

A. Etanercept
B. Adalimumab
C. Ustekinumab
D. Ipilimumab
E. Infliximab

►D

A recent study in the New England Journal of Medicine reported an increase in survival of patients
with stage III and stage IV melanoma with a new immunologic therapy agent named ipilimumab.
The survival was increased from 6 months to 10 months (P < 0.001). All the other medicines are not
used to treat melanoma; they are used to treat psoriasis, among other disease.

147- The risk of a melanoma developing in a giant congenital melanocytic nevus is approximately:

A. 3%
B. 6%
C. 12%
D. 50%
E. Virtually all of these patients will develop melanoma

►B

Large/giant congenital nevi are greater than 20 cm or greater than 10% of the body surface area.
The risk of melanoma in this type of lesion is approximately 6% by the age of 60 (Rhodes, 1981;
Bett, 2005). 50% of these melanomas occur by 5 years of age.

148- Which of the following is not considered a high-risk location of SCC:

A. Eyelids
B. Nose

1032
C. Ear
D. Lips
E. Chest

►E

High risk locations for recurrence and metastases include the ―H-zoneǁ of the face, skin overlying
cartilage and bony structures (e.g. preauricular area, retroauricular sulcus, nasolabial fold, inner
canthus, philtrum, temple, upper lip, columella, nose, lower eyelid). Lesions located on the trunk
and extremities are usually considered low risk unless aggressive histologic pattern is present.

149- What type of skin cancer is associated with recessive dystrophic epidermolysis bullosa?

A. Squamous cell carcinoma


B. Basal cell carcinoma
C. Merkel cell carcinoma
D. Melanoma
E. Actinic keratosis

►A

Patients with recessive dystrophic epidermolysis bullosa havea high risk of aggressive SCC in the
areas of chronic erosion and must be screened thoroughly and frequently. Actinic keratosis is not
skin cancer.

150- The mucin found in this lesion is produced by:

1033
A. Fibroblasts
B. Keratinocyte
C. Merkel cells
D. Nail matrix
E. Osteoblast

►A

Digital mucous cysts are pseudocysts found on the dorsal digits between the distal interphalangeal
joint and the nail fold. The fibroblasts in digital mucous cysts produce large quantities of hyaluronic
acid.

151- Which of the following are features of dermatofibromas, which help differentiate it from
dermatofibrosarcoma protuberans?

A. Commonly located on the extremities


B. Factor XIIIa negative
C. Factor XIIIa positive
D. Commonly located on the trunk
E. Commonly located on the extremities and factor XIIIa positive

►E

Typically DFSP is CD34 positive and factor XIIIa negative, allowing its differentiation from
dermatofibroma.

152- What is the most common location for this lesion which lacks phosphorylase in the epidermal
cells?

1034
A. Back
B. Buttock
C. Leg
D. Neck
E. Sacrum

►C

Clear cell acanthomas are slow growing, benign growths which typically occur on the leg. They
have a striking histologic appearance in the epidermis with sharp demaracation and enlarged, pale
cells. Excess glycogen in the cells accounts for their clear appearance and is due to a defect in
phosphorylase.

153- The Gorlin syndrome is characterized by:

A. Multiple BCCs during childhood


B. Macrocephaly
C. Odontogenic keratocysts of the jaw
D. Autosomal-dominant inheritance pattern
E. All of these answers are correct

►E

Gorlins syndrome is characterized by the appearance of multiple BCCs during childhood,


odontogenic keratocysts of the jaw, and skeletal defects. It is inherited in an autosomal dominant
pattern.

154- Seventy-five year old, male patient, with a 3cms pink, pearly nodule on his chest. You notice
that the patient has a scar on the same area. The treatment of choice is:

A. Radiation
B. Conventional excision
C. Cryotherapy
D. Mohs surgery

1035
E. Imiquimod

►D

The most likely diagnosis in this patient is recurrent BCC, therefore Mohs surgery is the treatment
of choice. Furthermore, any NMSC measuring 3 cms located on any anatomical site is an indication
for Mohs surgery.

155- This type of keratoacanthoma is eruptive and disseminated in adulthood:

A. Grzybowski
B. Ferguson-Smith
C. KA centrifugum
D. Giant KA
E. Subungual
►A

Patients with Grzybowski keratoacanthoma have eruptive and disseminated KA during adulthood.
KA centrifugum are large and solitary and Giant KA can invade underlying.

156 -Which HPV serotype has been associated with periungual SCC?

A. 6
B. 16
C. 11
D. All of these answers are correct
E. HPV is not associated with SCC

►B

HPV infection has been associated with the development of cutaneous SCC. HPV 6 and 11 have
been linked to genital lesions, whereas HPV 16 has been found in periungeal lesions.

1036
157- Keratoacanthomas have been linked etiologically to:

A. Ultraviolet exposure
B. Human papilloma virus
C. Chemical carcinogens such as tar and pitch
D. Smoking
E. All of these answers are correct

►E
The origin of KAs has not been established. Ultraviolet exposure, exposure to chemical carcinogens
such as tar and pitch, as well as smoking, and a viral etiology, specifically the human papilloma
virus, have all been proposed as etiologic factors.

158- The treatment for Merkel cell carcinoma is all of the following except:

A. Wide local excision 2-3 cm


B. Mohs
C. Radiation sensitive therapy
D. Wide local excision 5-10 cm
E. Local excision of 1-2 cm

►E

Merkel cell carcinoma is a type of aggressive cancer with a high rate of recurrence. It is a
neuroendocine carcinoma that is most commonly on sun exposed areas like the head and neck in the
elderly. Treatment includes wide local excision of 2-3 cm, Mohs, radiation, and chemotherapy.

159- Sezary syndrome:

A. Has characteristic Sezary cells in peripheral blood


B. The five year survival is estimated to be between 10-20%.
C. Is characterized by the triad of pruritic erythroderma, generalized lymphadenopathy, and
presence of Sezary cells in peripheral blood
D. Is associated with a poor prognosis

1037
E. All of these answers are correct

►E

Sezary syndrome is characterized by the triad of pruritic erythroderma, generalized


lymphadenopathy, and presence of Sezary cells in peripheral blood. Sezary cells are abnormal, large
hyperconvoluted lymphocytes. The five year survival is estimated to be between 10-20%.

160- Which of the following best describes the incidence of skin cancer in transplant recipients
from greatest to least?

A. BCC>melanoma>SCC>Merkel cell
B. SCC>BCC>melanoma>Merkel cell
C. BCC>SCC>melanoma>Merkel cell
D. SCC>BCC>Merkel cell>melanoma
E. Merkel cell>SCC>melanoma>BCC

►B

The incidence of skin cancer in transplant recipient (from greatest to least) is as follows:
SCC>BCC>melanoma>Merkel cell.

161- Histopathological evidence of epidermotropism may be seen in which of the following


conditions:

A. Merkel cell carcinoma


B. SCC
C. CTCL
D. None of these answers are correct
E. All of these answers are correct

►C

1038
Epidermotropism, defined as the presence of lymphocytes in the epidermis is characteristic of
CTCL.

162- 40 year-old male presented with a slow growing large tumor on the back. The patient
undergoes wide excision with adjunctive chemotherapy. Which of the chemotherapeutic agents was
used?

A. Imatinib
B. Cytarabine
C. Doxorubicin
D. Vincristine
E. Cisplatin

►A

Imatinib or Gleevec is used in the treatment of primary or locally recurrent dermatofib rosarcoma
protuberans. Even with wide excision, the 5 year recurrence rate for this tumor can be 20-25%.
Activation of the Platelet derived growth factor receptor associated with overexpression of Platelet
Derived growth factor is central to the development of DFSP Imatinib is a protein tyosine kinase
inhibitor for Bcr-Abl kinase and is used primary in chronic myelogenous leukemia with the
Philadelphia chromosome defect. Imatinib also inhibits the typosine kinases associated with platelet
derived growth factor and stem cell factor. Therefore, it directly inhibits the Platelet Derived
Growth factor receptor signaling cascade,that plays a critical role in the pathogenesis and growth of
DFSPs. While Imatinib has been successful in clinical trials, it is not yet FDA approved for the
treatment of DFSPs. The other listed agents are not adjunctive or primary therapies for DFSP.

163- Which site of squamous cell carcinoma has the greatest risk of metastasis?

A. lip
B. ear
C. eyelid
D. trunk
E. nose

1039
►A

The lip has an approximate rate of metastasis of 13%, which is slightly higher than the metastasis
rate of 11% from the ear. Risks for metastasis include size greater than 2cm, perineural invasion,
immunosuppression, treatment history, degree of tumor differentiation, or location on the ear or lip.

164 - All of the following are true regarding curettage of giant congenital melanocytic nevi except:

A. Treatment remains a controversial topic


B. Best performed during the first 2 weeks of life
C. Decreases the risk of melanoma
D. Offers an adequate alternative to surgical excision
E. Mandates careful long-term follow-up

►C

Curettage of giant congenital nevis is a highly controversial treatment option. De Raeve and
Roseeuw reported on 16 neonates who underwent curettage for treatment of their giant congenital
nevi. They noted that it was best performed within the first two weeks of life. They claim that it
offers an adequate cosmetic alternative to surgical excision. The risk of melanoma developing
within the giant congenital nevus is not decreased and long-term follow-up is essential.

165- What is the mean time-frame for development of Stewart-Treves Syndrome?

A. Less than 1 year


B. 1 year
C. 5 years
D. 10 years
E. >20 years

►E

Stewart-Treves syndrome refers to an angiosarcoma arising from chronic lymphedema. It typically


occurs as a complication of long-lasting lymphedema of the arm after mastectomy and/or

1040
radiotherapy for breast cancer. It is a very late complication of disease, usually developing several
years later. The mean time-frame for development of Stewart-Treves Syndrome in the literature is
23 years (4-40 years).

166- What kind of T-lymphocyte is the most common neoplastic cell in Cutaneous T-Cell
Lymphoma:

A. CD4
B. CD8
C. Natural killer
D. None of these answers are correct
E. All of these answers are correct

►A

CTCL is a neoplasm of helper T cells (CD4) that originates in the skin.

167- Commonly used immunohistochemical markers that are positive in Merkel cell carcinomas
include all of the following EXCEPT:

A. Chromogranin A/B
B. Synaptophysin
C. Glial fibrillary acidic protein
D. Cytokeratin 20
E. Cytokeratin 8

►C

The most commonly used markers for MCC are monoclonal antibodies to cytokeratins 8, 18, and
20; neuron-specific enolase (the most constant marker); chromogranin A/B, and synaptophysin.
Leukocyte common antigen, vimentin, desmin, glial fibrillary acidic protein, and S-100 are
consistently absent in MCC.

1041
168- Which area of the face receives the most cumulative exposure to UV radiation?

A. Dorsum of nose
B. Orbital region
C. Base of the nose
D. Chin
E. Central cheek

►C

The most frequent sites of basal cell carcinomas in one study was the base of the nose (bordering
the nasolabial fold and extending midway up the side of the nose), orbital region, apex of the nose,
ear, forehead, temporal region, nasolabial area, and the buccal region (in order). The nose was by
far the most frequent site of BCCǁs.

169- Desmoplastic trichoepithelioma:

A. Is most common in middle-aged patients


B. More commonly appears in males than in females
C. Does not display foci of calcification or ossification
D. Presents as a well-circumscribed lesions located in the upper dermis
E. None of these answers are correct

►D

Desmoplastic trichoepithelioma is a variant of trichoepithelioma, an uncommon adnexal tumor with


differentiation toward hair structures. It is three times more common in females and usually occurs
in young adults. The tumor is a well-circumscribed nodule lesions located in the upper dermis.
Strands or columns of basaloid cells are seen surrounded by fibrotic or desmoplastic stroma. Horn
cysts may also be seen, as well as foci of sebaceous cells, calcification and ossification.

170- Which of the following is true regarding digital HPV-associated squamous cell cancers?

A. The rate of metastasis approaches 15%.

1042
B. HPV18 is the most common associated sybtype.
C. Mohs micrographic surgery yields a 20% recurrence rate.
D. Women outnumber men 2:1.
E. These lesions only occur in association with immunosuppression.

►C

According to Riddel et al (JAAD 2011;64(6):1147-1153), Mohs micrographic surgery, although the


treatment of choice, results in a 20% recurrence rate, which is significantly higher than cutaneous
SCC. HPV16 is most often implicated. Men outnumber women 2:1. The rate of metastasis averages
between 2-3%. Although common in transplant patients, HPV associated digital SCCs can occur
secondary to trauma and in immunocompetent patients.

171- All of the following are true of thick melanomas (>3 mm) except:

A. Predominantly nodular type


B. Women affected more than men
C. Predilection for the head and neck
D. Mainly in older patients (>50 years)
E. Associated with fewer nevi

►B

According to a study performed by Chamberlain, et.al., thick melanomas (> 3 mm) were
predominantly nodular in type. They occurred in older men, mostly on the head and neck and were
associated with fewer nevi.

172- Merkel cell carcinoma stains positively for:

A. Leukocyte common antigen


B. Neuron specific enolase
C. Vimentin
D. S-100
E. None of these answers are correct

1043
►B

Merkel cell carcinoma is frequently diagnosed in individuals older than 50 years of age. However,
there have been reports of MCC in children and young adults.

173- Pseudorosettes in Merkel cell carcinoma:

A. Are seen in the trabecular variant


B. Are seen in the intermediate-cell type
C. Are seen in the small-cell type
D. All of these answers are correct
E. None of these answers are correct

►A

There are three histologic patterns of MCC: trabecular, intermediate-cell type, and small-cell type.
The trabecular variant consists of interconnecting trabeculae separated by strands of connective
tissue. Pseudorosettes may be seen in this type.

174 -A patient was referred from the oncology service because of tumoral lesions of MF. There is
no bone marrow involvement. What is the stage on this patient?

A. IIA
B. IIB
C. IIIA
D. IIIB
E. IV

►B

This patient has the tumor stage of mycosis fungoides(MF). Patients with tumoral lesions are
automatically classified as IIB. IA is the limited patch or plaque disease with less than 10% BSA
involved. IB is the generalized patch/plaque stage with more than 10%BSA but less than 80% BSA
involved. IIA implies lymph node involvement. IIIA is erythroderma without blood

1044
involvement. IIIB is erythroderma with low blood tumor burden. The stage IV of MF is divided in
three substages: IVA1(high blood tumor burden-Sezary syndrome), IVA2(very abnormal nodes),
IVB(visceral involvement).

175 - Which of the following is true regarding actinic keratoses?

A. Salicylic acid, tretinoin, and alpha-hydroxy acids are not useful in treatment
B. Low fat diets may decrease the incidence of AKs
C. Cure rates for treatment with topical 5-fluorouracil are higher than for treatment with
cryotherapy
D. UVA from sunlight is most responsible for AK development
E. Male gender is not a risk factor for AKs.

►B

Salicylic acid, tretinoin, and alpha-hydroxy acids have been reported to be useful in treatment of
AKs. Low fat diets have been reported to be useful in decreasing the incidence of AKs. Cryotherapy
has a reported cure rate of 98.8% for AKs, whereas topical 5-FU has a reported cure rate of 93%.
UVB, not UVA, is most responsible for AK development. Male gender is a risk factor for the
development of AKs.

176 -Which neoplasm is associated with the Stewart-Treves syndrome?

A. Renal leiomyomas

1045
B. Basal cell carcinoma
C. Angiosarcoma
D. Keratoacanthoma
E. T cell lymphoma

►C

Stewart-Treves syndrome is the development of angiosarcoma in the setting of chronic


lymphedema. Originally, named after radical mastectomy for the treatment of breast cancer. The
term applies to the development of angiosarcoma in any chronic lymphedematous condition.

177- The most common location for angiosarcoma is:

A. Legs
B. Arm
C. Head and neck
D. Trunk
E. Digits

►C

Angiosarcomas present as painless, purple nodules in the head and neck region of elderly
individuals. Men are more frequently diagnosed with this aggressive neoplasm.

178- A patient with multiple skin nodules has a biopsy suggestive of mycosis fungoides. The next
appropriate step in making the diagnosis is:

A. Polymerase chain reaction


B. Complete blood count
C. Flow cytometry
D. Immunohistochemical stains
E. Chest xray

1046
►A

In a patient with skin lesions and pathology suggestive of cutaneous lymphoma, gene rearrangement
studies should be done to detect a clonal population of lymphocytes. Polymerase chain reaction
(PCR) is approximately 1000 times more sensitive than Southern blot in the detection of a clonal
population of cells. In addition, PCR may be used on formalin fixed tissue as well as fresh tissue,
whereas Southern blot testing must be done of fresh tissue only.

179- All of the following statements regarding Basal Cell Nevus Syndrome are true EXCEPT:

A. Associated tumors include medulloblastoma


B. It is inherited in an autosomal recessive manner
C. Hypertelorism is a feature
D. The affected gene is mutated in 30-40% of sporadic basal cell carcinomas
E. All of these answers are correct

►B

Basal cell nevus syndrome (Gorlin Syndrome) is caused by a mutation in the PTCH gene, located
on chromosome 9q22. This gene is found to be mutated in 30-40% of sporadic basal cell
carcinomas. Gorlin Syndrome is inherited in an autosomal dominant pattern. Tumors associated
with this sydrome include medulloblastoma and meningioma. It is characterized by the appearance
of multiple BCCs during childhood, odontogenic keratocysts of the jaw, and skeletal defects
(including frontoparietal bossing and hypertelorism, among others).

180 -What temperature must be achieved for adequate treatment of a small superficial squamous
cell carcinoma with cryotherapy?

A. -10 degrees Celcius


B. -20 degrees Celcius
C. -30 degrees Celcius
D. -40 degrees Celcius
E. -50 degrees Celcius

1047
►E

Cryosurgery destroys tumor if it is frozen to -40 to -70 degrees Celcius, for at least 2 cycles with a
60 second thaw. Benign lesions can be destroyed at temperatures around -25 degrees Celcius.

181- This tumor, also called a Shagreen patch, is characteristic of which of the following
genodermatoses?

A. Neurofibromatosis-1
B. Neurofibromatosis-2
C. Tuberous sclerosis
D. NAME syndrome
E. Buschke-Ollendorf

►C

Tuberous sclerosis a rare genodermatosis characterized by adenoma sebaceum, seizures and mental
retardation. Other cutaneous features include hypopigmented macules, periungual fibromas, fibrous
plaque of the face, cafe-au-lait macules and connective tissue nevus.

182- A 30 year-old gentleman presents with multiple self healing lesions similar to the one pictured.
Other family members also have the same disease. What is the most likely diagnosis?

A. Epidermodysplasia verruciformis
B. Ferguson-Smith syndrome
C. Nevoid basal cell carcinoma syndrome

1048
D. Dyskeratosis congenita
E. Basex syndrome

►B

Ferguson-Smith syndrome is a rare autosomal dominant condition that is characerized by multiple


keratoacanthomas found in sun-exposed areas. Lesions typically regress over weeks to months.

183- At what location is this tumor LEAST likely to recur after surgical excision?

A. Nasolabial fold
B. Lateral canthus
C. Chin
D. Preauricular
E. Scalp

►E

Basal cell carcinomas are slow-growing, epithelial tumors. Highest area of recurrences after
surgical excision is in the "H-zone" of the face which include nose, nasolabial folds, periocular,
periauricular. The scalp is a M-zone.

184- Which test should be used to detect monoclonal gene rearrangements in cutaneous T-cell
lymphoma?

A. Northern blot

1049
B. Southern blot
C. Western blot
D. ELISA
E. Electrophoresis

►B

Monoclonality and gene rearrangements can be detected with a Southern blot. Southern blots can be
used to detect specific DNA fragments by gel-transfer hybridization.

185- All of the following are true regarding lentigo maligna except:

A. High rates of recurrence


B. Occurs mostly on head and neck
C. Mostly in sun-exposed areas
D. Margins difficult to evaluate
E. Spares oral mucosa

►E

Lentigo maligna is a type of melanoma in situ that usually presents as a poorly circumscribed,
variably colored patch. It occurs on sun-exposed areas on the head and neck. Margins may be
difficult to evaluate and recurrences are relatively common. There are rare reports of spread onto
the oral mucosa as well as conjunctiva.

186- Characteristics indicative of a high risk of metastasis with SCC include:

A. Mitotic rate
B. Extremity location
C. Well differentiated
D. Deep invasion
E. Papillomavirus infection

1050
►D

The rate of SCC metastasis from all skin sites ranges from 0.5% to 5.2%. Local recurrences and
metastasis are related to: 1.treatment modality, 2. prior histological, 3. location, 4. size, 5. depth , 6.
histological differentiation, 7. histological evidence of perineural involvement, 8. histological
evidence of desmoplastic features, 9. precipitating factors other than UV light, and 10. host
immunosuppression. In reference to metastatic disease, the highest rates occur from scars, the lip,
and the external ear. Patients with perineural spread have a local recurrence rate of 47.2% and a
metastatic rate of 34.8%. Demosplastic SCC's are 6 times more likely to metastasize than other
histological patterns.

187- What cancer in women most commonly metastasizes to the skin?

A. Breast cancer
B. Medullary thyroid carcinoma
C. Glioblastoma multiforme
D. Colon adenocarcinoma
E. Cervical cancer

►A

Breast cancer is the most common cancer to metastasize to the skin in women.

188- A young woman presents with a single small, firm, umbilicated papule on the face. Biopsy
reveals a well-circumscribed lesion located in the upper dermis. Strands of basaloid cells are seen
surrounded by fibrotic or desmoplastic stroma. Horn cysts and foci of sebaceous cells and
calcification are also noted. How should this patient and her lesion be treated?

A. Reassurance and no further treatment


B. Close clinical follow-up
C. Topical 5-fluorouracil
D. Cryotherapy
E. Local surgical excision

1051
►E

This patient has a desmoplastic trichoepithelioma. Desmoplastic trichoepithelioma is a variant of


trichoepithelioma, an uncommon adnexal tumor with differentiation toward hair structures. It
presents commonly in young women as a small, firm, umbilicated papule on the face. Local surgical
excision is the treatment of choice.

189- A 55 year-old female presents with an ulcerated malignant melanoma with Breslow dept of
1.5mm. Sentinel lymph node biopsy is negative. Which of the following is the correct staging
classification?

A. Stage IB
B. Stage IIA
C. Stage IIB
D. Stage IIIA
E. Stage IIIB

►B

The current melanoma staging is developed by the American Joint Comittee on Cancer. This system
is based on four characterisitics: 1. Tumor thickness 2.Presence of ulceration within the primary
tumor, 3. Involvement of lymph nodes and 4.presence of distal metastases. Stage 1A characterizes a
melanoma that has less than 1mm in tumor thickness and no ulceration, no nodes, and no distant
mets. Stage IB characterizes a melanoma with tumor thickness less than 1mm however with
ulceration and no nodes or distal metastases. Stage IB also can characterize melanomas with tumor
thickness between 1.01-2.00 mm without ulceration. Stage IIA classifies tumor with thickness
between 1.01-2.00mm, with ulceration, and no nodes and no distal metastases. Stage IIA also
characterizes those melanomas with thickness between 2.01-4.0mm without ulceration, nodes, or
distal mets. Stage III and IV melanomas have nodal and distal met involvement respectively.

190- Which of the following is most likely to present with cutaneous metastases in men?

A. Lung cancer
B. Colon cancer
C. Prostate cancer

1052
D. Melanoma
E. Esophageal Cancer

►D

The most common malignancy to present with cutaneous metastases in men is from melanoma,
followed by squamous cell cancer of the head and neck. In women, breast cancer is most likely to
present with cutaneous metastases. Special presentations of cutaneous metastases include alopecia
neoplastica (scarring alopecia) and zosteriform metastases.

191- A 56-year old woman with a history significant for chronic lymphedema after radical
mastectomy twelve years ago presents with this growth on her arm. What is the diagnosis?

A. Angiosarcoma
B. Bacillary angiomatosis
C. Castleman's syndrome
D. Kaposi's sarcoma
E. Metastatic breast carcinoma

►A

Angiosarcoma may occur in association with chronic lymphedematous states. Stuart-Treves


syndrome applies to angiosarcoma arising in an area of chronic lymphedema, like upper arm
lymphedema after mastectomy.

1053
192- Imiquimod is an immune response modifier that stimulates innate and cell mediated immune
pathways. It induces all of the following cytokines EXCEPT:

A. IL-1
B. IL-4
C. IL-5
D. IL-6
E. IL-8

►B

Imiquimod induces the synthesis and release of cytokines such as interleukins 1, 5, 6, 8, 10, and 12,
among others.

193- Which syndrome is characterized by multiple keratoacanthomas beginning in childhood?

A. Muir-Torre
B. Ferguson Smith
C. Rombo
D. Grybowski
E. Keratoacanthoma centrifugum marginatum

►B

Ferguson Smith is familial (AD), and is characterized by regressing keratoacanthomas beginning in


childhood. Grybowski is a non-familial disorder in which patients develop generalized eruptive

1054
keratoacanthomas beginning between age 40 to 60. Keratoacanthoma centrifugum marginatum is
most commonly a solitary lesion occurring later in life. Muir Torre is a syndrome characterized by
gastrointestinal or genitourinary neoplasms which precede development of sebaceous neoplasms
and keratoacanthomas by one to two decades. Rombo is not associated with keratoacanthomas, but
is associated with multiple basal cell carcinomas, atrophoderma vermiculatum, hypotrichosis, and
milia.

194- A 3-year-old girl presents with multiple small, angulated, firm nodules. There is a positive
"tent sign" and biopsy reveals ghost cells and germinative cells. Of the following conditions
associated with multiple pilomatricomas, which is thought to be most closely linked?

A. Turner's
B. Myotonic dystrophy
C. Rubenstein-Taybi
D. Sarcoidosis
E. Gardner's syndrome

►B

Pilomatricomas are the most common superficial pediatric tumor. The occurrence of multiple
pilomatricomas has been most closely associated with the development of myotonic
dystrophy(Steinert Disease). The onset of myotonic dystrophy, an autosomal dominant disorder
with variable penetrance, can occur before or after the onset of lesions. Cigliano et al. reported
myotonic dystrophy in 1 out of 2 patients with multiple pilomatricomas; Julian et al. reported
myotonic dystophy in 1 out of 4 patients. The other syndromes associated with multiple
pilomatricomas include Rubenstein-Taybi, Turner syndrome, Gardner syndrome, and sarcoidosis.

195 -What is the most common location for an epitheloid sarcoma?

A. head and neck


B. proximal extremities
C. hands and forearms
D. lower legs
E. groin and buttocks

1055
►C

Epithelioid sarcoma is a rare soft tissue sarcoma that most commonly develops in young adults,
males greater than females, with a predilection for the distal upper extremities, namely hands and
forearms. Most tumors present as firm-to-hard palpable masses, either in the deep soft tissue or in
the dermis. The superficial lesions can present with ulceration. Five year survival and ten year
survival rate for patients with epithelioid sarcoma are approximately 50-70% and 42-55%
respectively (Journal of bone and Joint Surgery (Am), 70-A: 862-870, 1988). Female patients have
a more favorable outcome. Proximal lesions have been shown to have worse outcomes compared to
distal lesions.

196- What is the most common location of oral SCC?

A. Soft palate
B. Buccal mucosa
C. Gingiva
D. Dorsal tongue
E. Lateral tongue

►E

Squamous cell carcinomas are the most common carcinoma of the oral cavity. The most common
locations for this tumor are the lateral and ventral surfaces of the tongue and the floor of the mouth.

197- Desmoplastic trichoepitheliomas are commonly located on:

A. Legs
B. Chest
C. Back
D. Face
E. Scalp

►D

1056
Desmoplastic trichoepithelioma presents as small, firm, umbilicated papule on the face of young
adults.

198- The type of keratoacanthoma that is seen in children with a autosomal dominant pattern and
multiple in numbers is called:

A. Ferguson-Smith
B. Grzybowski
C. Verrucous carcioma
D. KA centrifugum
E. Buschke-Lowenstein

►A

Patients that have multiple keratoacanthoma in childhood have Ferguson-Smith type of KA. This is
autosomal dominant and are self-healing.

199- All of the followings can be used for treatemt of this condition except

A. Cryotherapy
B. Topical Imiquimod 5% cream
C. Topical 5-flurouracil
D. Topical retinoids
E. Surgical excision

►E

1057
Attached picture is disseminated superficial actinic porokeratosis (DSAP) which is the most
common type of all porokeratosis, with multiple thin papules appearing most commonly on the legs
of adult women. Many treatments have been used for this condition which include: cryotherapy,
topical 5-fluorouracil (5-FU), topical retinoids, CO2 laser, and dermabrasion. Although surgical
excision might be used for treatment of other forms of porokeratosis, it is not advised in this case
because of number of lesions and increase risk of scarring. Other forms of porokeratosis are:
porokeratosis of Mibelli, punctate porokeratosis, linear, and Porokeratosis palmaris et plantaris
disseminata.

200- What is the most common location for a fibroepithelioma of pinkus variant of BCC is

A. head and neck


B. extremities
C. hands and feet
D. upper trunk
E. lumbosacral region

►E

Fibroepithelioma of pinkus variant of BCC is often a fleshy or pink colored nodule. The most
common location of a fibroepithelioma of pinkus is the lumbosacral region. Superficial variants are
more common on the trunk and extremities. BCC makes up the most common non melanotic skin
cancer at 75%.

201- Merkel cell carcinoma has been found to be associated with which of the following viruses?

A. Herpes virus
B. Polyomavirus
C. Paramyxovirus
D. Flavivirus
E. Enterovirus

►B

1058
Merkel cell carcinoma has been found to be associated with the merkel cell polyomavirus
(MCPyV). Herpes virus causes diseases such as HSV, VZV, EBV, CMV, roseola, and Kaposi's
sarcoma. Paramyxovirus causes measles. Flavivirus causes diseases such as west nile virus, dengue,
and yellow fever. Enteroviruses are not associated with merkel cell carcinoma.

202 - The Grzybowski type of keratoacanthoma:

A. Is characterized by rapid growth of a single lesion reaching a diameter of 9 cm or more


B. Typically invades underlying cartilage
C. Demonstrates simultaneous central healing
D. Presents in childhood on sun-exposed surfaces
E. Presents with hundreds of disseminated lesions

►E

The Grzybowski type of keratoacanthoma is typically diagnosed in adulthood, with the sudden
appearance of hundreds of lesions in a disseminated fashion. The lesions are generally 2 -3mm in
diameter and can be found anywhere on the body including the palms, soles, larynx, and oral
mucosa.

202- Which of the following are the most reliable prognostic factors in malignant melanoma?

A. Breslow‘s depth and ulceration


B. Breslow‘s depth and Clark level
C. Clark level and ulceration
D. All of these answers are correct
E. None of these answers are correct

►A

The most reliable prognostic factors in MM are Breslow‘s depth and ulceration of the primary
tumor. Breslow depth is the thickness of the melanoma measured from the granular layer to the
deepest point of tumor invasion.

1059
203 -Patients that are organ transplant patients have a higher risk of developing skin cancer such as
squamous cell carcinoma. The risk can be as much as:

A. 65x
B. 50x
C. 40x
D. 25x
E. 10x

►A

The risk of SCC in an organ transplant population is elevated by 65x. These patients should have a
year skin examination.

204- The Pinkus BCC is frequently located on the:

A. Digits
B. Eyelid
C. Lips
D. Ear
E. Lumbosacral area

►E

Pinkus BCC (also known as fibroepithelioma) appears as a pedunculated, dome-shaped papule


frequently located on the back (lumbosacral area).

205- What is the average time between a BCC primary tumor and its metastasis?

A. 6 months
B. 1 year
C. 2-4 years
D. 9 years
E. greater than 15 years

1060
►D

The size of a BCC is related to its metastasis risk. Invasion of bone, cartilage, and muscle is not
common, however, spread does occur along perichondrium, periosteum, fascia, or tarsal plate.

206- All of the following statements regarding the patient pictured are true EXCEPT:

A. Spina bifida may be an associated finding


B. The patient likely has a mutation in the PTCH gene
C. Pheochromocytoma is a tumor associated with this disease
D. This patient likely had a similarly affected parent
E. These lesions appeared in childhood

►C

The patient has basal cell nevus syndrome, or Gorlin syndrome, which is characterized by the
appearance of multiple basal cell carcinomas in childhood, odontogenic keratocysts of the jaw, and
skeletal defects (i.e macrocephaly, hypertelorism, frontoparietal bossing, spina bifida, or rib
abnormalities). Tumors associated with this disease include medulloblastoma and meningioma (not
pheochromocytoma). It is caused by a mutation in the PTCH gene located on 9q22 and is inherited
in an autosomal dominant manner.

207- Which of the following neoplasms has demonstrated an association with HTLV-1 infection?

A. Mycosis fungoides
B. Adult T cell lymphoma
C. Follicular lymphoma
D. Multiple myeloma
E. Hodgkin‘s disease

►B

Mycosis fungoides is a rare form of cutaneous T-cell lymphoma, the etiology of which is not
completely known. Adult T Cell Lymphoma is also a type of T cell neoplasm that has been linked to

1061
HTLV-1. Adult T-cell lymphoma may have an acute and chronic, smoldering form. The chronic,
smoldering form can be difficult to distinguish from mycosis fungoides.

208- Mohs micrographic surgery the treatment of choice for all of the following, except:

A. 1 cm SCC located on the chest


B. 2 cms BCC on lower extremities
C. 1 cm BCC on the eyelid
D. Morpheaform BCC on the cheek
E. Recurrent BCC on the chest

►A

SCCs that measure 1cm on the chest are not an indication for Mohs micrographic surgery. Lesions
must measure 2cms on the chest and extremities, or 1cm for those located on the face to be
considered for Mohs surgery. Due to the aggressive growth patter and subclinical spread of
morpheaform and recurrent BCCs are always and indication for Mohs surgery.

1062
209- This type of verrucous carcinoma is locally aggressive but rarely metastasizes and is on the
plantar area is:

A. Epithelioma cuniculatum
B. Buschke-Lowenstein
C. Oral florid papillomatosis
D. Subungual
E. Giant SCC

►A

Epithelioma cuniculatum is a plantar SCC that is locally aggressive but rarely metastasizes, Oral
florid papillomatosis is the a type that is found in the mouth. Buschke-Lowenstein is the type that
you find in the genital area.

210- Which is the most common neoplasm in patients who have had long-term PUVA therapy?

A. Basal cell carcinoma


B. Atypical fibroxanthoma
C. Squamous cell carcinoma
D. Cutaneous T cell lymphoma
E. Melanoma

1063
►C

The most common neoplasm to arise in patients who have been treated with long -term PUVA
therapy is squamous cell carcinomas. In a study by Stern, et.al. a 12.8 fold risk was found for the
development of squamous cell cancers in patients who received high doses than those that received
low doses.

211- Which of the following markers do not stain melanocytic lesions:

A. CK7
B. Vimentin
C. S-100
D. HMB-45
E. All of these answers are correct

►A

CK7 will stain keratinocytes. Vimentin, S-100 and HMB-45 stains melanocytic lesions.

212 -Which of the following are features of microcystic adnexal carcinoma that help distinguish it
from desmoplastic trichoepitheliomas?

A. Deep subcutaneous infiltration


B. Perineural invasion
C. CEA positive staining
D. Commonly located on the face
E. Deep subcutaneous infiltration,perineural invasion, and CEA positive staining

►E

Desmoplastic trichoepitheliomas is one of the differential diagnosis for MAC. MAC show deep
subcutaneous and perineural invasion, as well as CEA positive staining, all features that may
help differentiate it from desmoplastic trichoepitheliomas. Both, MAC and desmoplastic
trichoepitheliomas are commonly located on the face.

1064
213 -Which of the following is a common location of melanoma in women?

A. Chest
B. Lower legs
C. Genitals
D. Digits
E. Scalp

►B

The most common locations of MM in women are the back, lower legs and upper extremities.

214 -The most common location of superficial spreading melanoma in men is:

A. Lower legs
B. Back
C. Upper extremities
D. Head and neck
E. Digits

►B

Superficial spreading melanoma may be located anywhere, but the back is the most common site in
male patients.

215- Bazex syndrome can be differentiated clinically from Rombo syndrome by presence of:

A. Multiple basal cell carcinomas


B. Trichiepitheliomas
C. Milia
D. Follicular atrophoderma
E. Hypohidrosis

1065
►E

Localized hypohidrosis is a feature found in Bazex syndrome but not Rombo syndrome. Another
differentiating feature is that Rombo syndrome classically has vermiculate atrophoderma, while
Bazex has follicular atrophoderma.

1066
1067
Medical Mycology

1- A soldier from Sudan experienced a penetrating wound of his foot 10 years ago which progresses
slowly, leading to draining sinuses, scarring, swelling, and deformity of his lower extremity. He
presents to your office and you collect samples for KOH, histology and culture. He relapses after a
debridement and a trial of oral antifungal therapy. Which of the following is NOT true of this
condition?

A. Caused by true fungi


B. Caused by filamentous bacteria
C. Characterized by triad of tumefaction, draining sinuses, grains or granules
D. Bony involvement may occur
E. Cutaneous disease results from hematogenous dissemination of the responsible pathogen

Correct choice: E. Cutaneous disease results from hematogenous dissemination of the responsible
pathogen

Explanation: Mycetoma (maduromycosis, Madura foot, fungus tumor) is caused by true fungi
(eumycotic) or filamentous bacteria (actinomycotic). Disease progresses slowly. It is characterized
by the triad of tumefaction, draining sinuses, grains or granules (aggregates of organism) causing
scarring, swelling, deformity. Bony involvement may occur. For evaluation, collect grains for KOH,
histology and culture.

2-A patient with a CD4 count of 48 presents with several dozen umbilicated papulonodules and
complains of fatigue and fever. Which of the following is the least likely underlying infectious
process?

A. Histoplasmosis
B. Coccidioidomycosis
C. Paracoccidioidomycosis
D. Penicilliosis
E. Cryptococcosis

1068
Correct choice: C. Paracoccidioidomycosis

Explanation: C is the correct answer. The remaining choices are in the differential diagnosis of
molluscum-like lesions, especially in AIDS or immunocompromised patients.
Paracoccidioidomycosis does not present with molluscum-type lesions.

3- What is the causative organism?

A. Fonsecaea pedrosoi
B. Actinomadura pelletieri
C. Lacazia loboi
D. Prototheca wickerhamii
E. Rhinosporidium seeberi

Correct choice: A. Fonsecaea pedrosoi

Explanation: Here you can see the classic copper pennies of chromoblastomycosis. Fonsecaea
pedrosi causes chromoblastomycosis. Actinomycotic red granules are due to actinomadura
pelletieri. Lacazia loboi causes lobomycosis, prototheca wickerhamii causes protothecosis and
rhinosporidum seeberi causes rhinosporidiosis.

4- A patient has a positive Wood's lamp exam that is caused by pteridine. What is the causative
organism?

A. Microsporum ferrugineum
B. Corynebacterium minutissimum
C. Trichophyton violaceum

1069
D. Pseudomonas aeruginosa
E. Trichophyton tonsurans

Correct choice: A. Microsporum ferrugineum

Explanation: Wood's light positive dermatophytes (M. canis, M. audouinii, M. distortum, M.


ferrugineum, T. schoenleinii, and sometimes M. gypseum) fluoresce secondary to pteridine.
Corynebacterium can fluoresce as well due to production of coproporphyrin III. Pseudomonas
fluoresces from production of pyocyanin. The other organisms do not react with a Wood's light.

5- A liver transplant patient on prednisone, tacrolimus and mycophenolate mofetil developed an


enlarging necrotic ulceration of the face. Bedside gram stain of purulent contents was obtained and
supports the diagnosis of which infectious organism?

A. Chromoblastomycoses
B. Cryptococcus
C. Mucor spp.
D. Blastomyces dermatitidis
E. Trichophyton mentagrophyes

Correct choice: C. Mucor spp.

Explanation: Large ribbon-like hyphae with 90-degree branching is consistent with Zygomycosis
(Mucormycosis). The most common species include Mucor spp, Rhizopus spp and Absidia spp.
Risk factors for mucormycosis include immunosuppression (neutropenia), diabetes and severe
burns.

1070
6- What is the best test to confirm the most likely diagnosis?

A. DsDNA antibody
B. Anti-Ro antibody
C. Biopsy
D. KOH
E. Gram stain

Correct choice: D. KOH

Explanation: The picture depicts tinea faceii. A KOH exam should be performed to look for hyphae.
The other options can present with erythema on the face, but the scaling and higher concentration of
papules at the periphery suggest a fungal origin. Tinea faceii is also more common.

7- A patient presents to your clinic after returning from vacation with painless smooth-surfaced
nodules on the arm. Biopsy of one of the nodules reveals round, brownish cells attached to one
another with narrow connections. What is the most likely diagnosis?

A. North American Blastomycosis


B. South American Blastomycosis
C. Chromoblastomycosis
D. Keloidal Blastomycosis
E. Coccidiomycosis

Correct choice: D. Keloidal Blastomycosis

Explanation: The question stem describes the typical clinico-pathologic findings of keloidal
blastomycosis (Lobomycosis), which is caused by Lacazia loboi. This deep fungal organism is

1071
found in the Amazon basin and Gulf of Mexico, and is associated with dolphins. The biopsy
findings are also described as "brass knuckles" or a "chain of coins." The treatment of choice is
surgical excision as antifungals are ineffective. The remaining answer choices do not present with
the clinico-pathologic findings described in the
stem.

8- A Caribbean agricultural worker sustained a prior traumatic injury to her leg decades ago. She
now has cauliflower-like tumors that coalesce into irregular verrucous plaques. A biopsy shows
brown, round, thick walled cells 5-12 microns thick with septations. What is the causative organism
causing this patient’s condition?

A. Sporothrix schenckii
B. Pseudallescheria boydii
C. Lacazia loboi
D. Blastomyces dermatitidis
E. Fonsecaea pedrosoi

Correct choice: E. Fonsecaea pedrosoi

Explanation: Chromoblastomycosis (chromomycosis, verrucous dermatitis) is seen mainly in the


tropics and subtropics especially the Caribbean islands. It is caused by saprophytic organisms found
in soil, decaying vegetation and wood and the most common causative organism is Fonsecaea
pedrosoi. It is usually instigated by a traumatic injury in agricultural workers to the foot, leg,
occasionally chest and shoulders. It is characterized by cauliflower-like tumors which may coalesce,
irregular verrucous plaques, nodules, annular plaques with a central clearing and transepidermal
elimination. A biopsy shows pseudoepitheliomatous hyperplasia, epidermal microabscesses, and a
granulomatous tissue response. “Copper pennies” (sclerotic bodies, medlar bodies, chromobodies)
are brown, round, thick walled cells 5-12 microns with septations characteristic for this condition on
histology. Lacazia loboi causes lobomycosis and Blastomyces dermatitidis causes
blastomycosis. Sporothrix schenckii causes sporotrichosis. Pseudallescheria boydii causes a
mycetoma.

9- You assess a patient in clinic with thick nails with debris that begins distally and involves the nail
bed, nail plate, and lateral nailfold. A nail culture will most likely grow which organism?

1072
A. T. megnininii
B. T. schoenleinii
C. T. tonsurans
D. T. mentagrophytes
E. T. rubrum

Correct choice: E. T. rubrum

Explanation: Distal lateral subungual onychomycosis is a fungal nail infection that begins distally
and involves the nail bed, nail plate and lateral nail fold. It appears as a thick nail with debris, loose
or cracked nail plate. The most likely causative organism is T. rubrum.

10- The most likely organism to cause this infection is:

A. T. tonsuran
B. C. immitis
C. C. albicans
D. P. aeruginoas
E. T. verrucosum

Correct choice: C. C. albicans

Explanation: The organism that causes erosio interdigitalis blastomycetica is candida albicans.

While other options may be causes of cutaneous infection, interdigital involvement is seen most
often with candidal infection.

1073
11- This patient had been treating a scaly rash with clobetasol for 4 weeks. It has significantly
increased in size despite treatment and is very itchy. Which of the following treatments is most
appropriate?
A. Oral prednisone
B. Oral terbinafine
C. Topical fluocinonide
D. Topical econazole
E. Topical pimecrolimus

Correct choice: B. Oral terbinafine

Explanation: The correct answer is B. The picture and history are suggestive of a fungal process; it
likely started out as tinea corporis and, with topical application of clobetasol, progressed to the
diagnosis shown- Majocchi’s granuloma. This often requires oral (not topical) antifungal treatment.
Terbinafine is typically first line.

12- Which of the following is NOT caused by a dematiaceous fungus?

A. Chromoblastomycosis
B. Tinea nigra
C. Black piedra
D. Eumycotic mycetoma
E. Hyalohyphomycosis

Correct choice: E. Hyalohyphomycosis

1074
Explanation: The correct answer is E. Hyalohyphomycosis may be caused by Penicillium marneffei,
Paecilomyces, and Fusarium. Tinea nigra and black piedra are types of phaeohyphomycoses
(pigmented hyphae) and chromoblastomycosis has pigmented yeast.

13- What is the most common cause, in the United States, of the disease seen in this kodachrome?

A. Trichophyton tonsurans
B. Trichophyton rubrum
C. Candida
D. Staph aureus
E. Trichophyton mentagrophytes

Correct choice: B. Trichophyton rubrum

Explanation: T. rubrum is the most common cause of onychomycosis in the USA. Trichophyton
mentagrophytes is the most frequent cause of White Superficial Onychomycosis. T. tonsurans is the
most common cause of tinea capitis in the US. Candida can cause chronic paronychia. Staph aureus
is the most common cause of acute paronychia.

14- What is the best treatment for this infection?

1075
A. Fluconazole
B. Ciprofloxacin
C. Argatroban
D. Itraconazole
E. Penicillin

Correct choice: D. Itraconazole

Explanation: The image shown is blastomycosis, indicated by the broad-based buds and the
refractile wall. This is a GMS stain which highlights the walls of the blastomycosis. This is treated
with itraconazole, or amphotericin B if the infection is severe.

15-Which two characteristics combined form a significant risk for acquiring sporotrichosis? 1.
Animal handler 2. Male gender 3. Alcoholism 4. Filipino or African decent 5. Genetic
predisposition

A. 1, 2
B. 1, 3
C. 2, 5
D. 4, 5
E. 3, 4

►B

A variety of domestic animals can carry Sporotrichosis. Alcoholism increases the risk for this
cutaneous infection.

16 -What is the most frequently reported cause of primary cutaneous aspergillosis?

A. Aspergillus flavus
B. Aspergillus fumigatus
C. Aspergillus niger
D. Aspergillus solani
E. Aspergillus marneffei

1076
►A

Primary cutaneous aspergillosis is a rare disease reported mostly frequently in children with
hematologic malignancies who developed skin lesions at the site of IV canulas.

17- Which of the following DOES NOT typically cause white superficial onychomycosis?

A. Aspergillus species
B. Fusarium species
C. Trichophyton mentagrophytes
D. Scopulariopsis species
E. Trichophyton rubrum

►E

In white superficial onychomycosis, the organism only invades the superficial nail plate, and it
clinically appears as chalky white patches on the nails. The most common organism is T.
mentagrophytes, but Aspergillus, Cephalosporium, Fusarium, Acreconium and Scopulariopsis are
implicated as well.

18- A blood culture from a neutropenic patient with onychomycosis grew which of the following
organism:

A. Scopulariopsis sp
B. Aspergillus sp
C. Fusarium sp
D. Acremonium sp
E. T. rubrum

►C

Fusarium is one of the few moulds, which yield positive blood cultures, neutropenia is one of the
risk factors for Fusariosis.

1077
19- Although this organism is not a fungus, it stains with PAS and GMS and produces 8-20 micron
spherules in tissue. This organism can be identified as:

A. Rhinosporidium seeberi
B. Coccidioides immitis
C. Penicillium marneffei
D. Prototheca wickerhami
E. Leishmania mexicana

►D

This achloic algae produces spherules or sporangia 8-20um in tissue. The mature form is called a
morula. Rhinosporidium is also not a fungus however, it produces sporangium 250-350 microns.

20- Which of the following is a fluorescent ectothrix dermatophyte?

A. Trichophyton rubrum
B. Trichophyton mentagrophytes
C. Microsporum ferrugineum
D. Trichophyton violaceum
E. Microsporum nanum

►C

Fluorescent ectothrix dermatophytes include M. canis, M. audouinii, M. distortum, M. ferrugineum,


and sometimes M. gypseum and T. schoenleinii ("Cats And Dogs Fight and Growl Sometimes.")
Nonfluorescent ectothrix dermatophytes include T. mentagrophytes, T. rubrum, T. verrucosum, T.
megninii, and M. nanum. Endothrix dermatophytes include T. rubrum, T. gourvilli, T. yaounde, T.
tonsurans, T. schoenleinii, T. soudanense, and T. violaceum.

21- Which of the following statements regarding candidal infection is FALSE?

A. Candida species displays true hyphae on potassium hydroxide examination

1078
B. Predisposing factors for candidal infection include diabetes mellitus, hyperhidrosis and
broad spectrum antibiotics
C. Candida albicans is the number one cause of mucocutaneous infections
D. Candidal infections typically do not spare the scrotum
E. Candida species may be associated with granuloma gluteale infantum

►A

Candida species are part of the resident flora but they are also the most common cause of
opportunistic mycotic infection. Cutaneous clinical manifestations include thrush, perleche,
paronychia, onychomycosis, intertrigo, and folliculitis. Candida albicans is the most common
organism; however, C. dubliniensis is often implicated in mucosal disease in AIDS patients. C.
parapsilosis causes chronic paronychia and C. glabrata is fluconazole resistant. Infection is most
common in the extreme of age, and the mucosal disease is prevalent in HIV patients. Factors
predisposing patients to infection include impaired mucocutaneous barrier function,
immunodeficiencies, broad spectrum antibiotic use, malignancies, heat, humidity, friction, diabetes,
and indwelling catheters. Groin infections often involve the scrotum unlike tinea cruris caused by
dermatophytes. On KOH examination, budding yeast and pseudohyphae (not true hyphae) are
diagnostic. Predisposing factors for granuloma gluteale infantum include occlusion, topical
corticosteroids, and possibly Candida diaper dermatitis.

22- Erythematous to violaceous papules that may progress to nodular and necrotizing skin lesions in
neutropenic patients may be caused by which of the following organisms?

A. Trichosporon asahii
B. Trichosporon beigelii
C. Trichosporon ovoides
D. Trichosporon inkin
E. Trichophyton rubrum

►A

Trichosporon asahii can cause trichosporanosis, which presents with disseminated disease in
neutropenic patients. Patients may present with erythematous to violaceous papules that may
progress to nodular and necrotizing skin lesions. Trichosporon beigelii (new nomenclature:
Trichosporon ovoides and Trichosporon inkin) causes white piedra and other superficial infections.

1079
23- A biopsy shows broad-based budding thick walled yeast cells, 10-15 um with a double
contoured appearance.

A. This yeast has a yeast phase at room temperature


B. Does not grow at 37º C
C. Usually produces a severe characteristic pulmonary disease
D. May be found in dogs
E. Is transmitted by mosquitoes

►D

This biopsy describes Blastomyces dermatitidis which generally can be found in decaying
vegetation but can be carried by dogs.

24- Which of the following is true regarding Dermatophyte Test Media (DTM)?

A. DTM contains chlortetracycline and minocycline


B. Alizarin red is the indicator present in DTM
C. Non-dermatophytes cause the media to turn yellow due to acid byproducts
D. DTM is useful for culturing dermatophytes from skin and nails, but not hair.
E. Dermatophytes utilize glucose as a carbon source, producing alkaline byproducts.

►C

Dermatophyte Test Media (DTM) contains peptones, dextrose, gentamicin, chlortetracycline,


cycloheximide, and phenol red. Dermatophytes utilize protein as a carbon source producing alkaline
byproducts causing the media to turn from amber to red. Nondermatophytes cause the media to turn
yellow due to acid byproducts.

25- The most common eumycotic organism in the US produces which color grains in its
microcolony?

A. White
B. Black

1080
C. Red
D. Yellow
E. Green

►A

The most common cause of fungal (eumycotic) eumycetoma in the US is Pseudallescheria boydii.
This and Acremonium produce white colonies. Black colonies are caused by Exophilia, Madurella,
and Curvalaria. Red colonies are created by Actinomadura pelletieri. Yellow colonies are made by
Streptomyces and Nocardia, both causes of bacterial mycetoma.

26- All of the following statements about paracoccidioidomycosis are true EXCEPT?

A. It is most common in male agricultural workers


B. It has a characteristic ―mariner‘s wheelǁ appearance on histopathology
C. Disease is almost always confined to the skin
D. It is endemic to Brazil
E. It may cause mucocutaneous lesions

►C

Paracoccidioidomycosis is caused by Paracoccidioidomycosis brasiliensis, and is endemic to


Central and South America, especially Brazil, Argentina, Venezuela, Ecuador and Colombia. Male
agricultural workers are at greatest risk. Infection is most commonly caused by direct inoculation
which leads to pulmonary disease. Lung disease may be followed by dissemination to
mucocutaneous surfaces, gastrointestinal tract, spleen, adrenal glands, and lymph nodes. Cutaneous
lesions are verrucous and/or ulcerative, and usually found on the face and in the nasal and oral
mucosa. Primary mucocutaneous disease exists as well and is caused by direct inoculation. Biopsy
specimens display multiple narrow-based budding yeast cells, described as a ―mariner‘s wheel.ǁ
The treatment of choice is itraconazole.

27- An elderly lady with moccasin-type tinea pedis has a fungal culture which demonstrates
smooth, teardrop-shaped microconidia which produce a ―port-wineǁ pigment. The organism is:

1081
A. Trichophyton verrucosum
B. Microsporum canus
C. Trichophyton rubrum
D. Microsporum gypseum
E. Trichophyton mentagrophytes

►C

Trichophyton rubrum is an anthropophilic dermatophyte that is a cause of T. pedis, T. manum, T.


corporis, T. cruris, onychomycosis, Majocchiǁs granuloma, and rarely T. capitis. Colonies appear
as fluffy to granular white to cream colored with reverse non-diffusible port-wine or red pigment.
Macroconidia are rare, thin-walled pencil shaped. Microconidia are delicate teardrop shaped.

28- A pet store owner comes into your office with pruritic scaly plaques on his arms. A fungal
culture demonstrates a yellow colony with spindle-shaped macroconidia which grows on polished
rice grains. The organism is:

A. Microsporum canis
B. Epidermophyton floccosum
C. Trichophyton rubrum
D. Microsporum gypseum
E. Trichophyton tonsurans
►A

Microsporum canis os a zoophilic dermatophyte, which causes ectothrix invasion when infecting
the hair. Colonies of M. canis are flat, spreading, white to cream-colored with a dense cottony
surface, The reverse of the plate is a canary yellow. Wet mount of the fungus shows spindleshaped
macroconidia with 5-15 cells and often have a terminal knob.

29- The most reliable method for distinguishing between Trichophyton rubrum and T.
mentagrophytes is:

A. Morphology of microconidia
B. Morphology of macroconidia
C. Pigmentation studies

1082
D. Hair perforation test
E. Colony morphology

►D

The diagnostic morphology of the Trichophytons overlap and may be difficult to differentiate. T.
mentagrophytes produces a positive hair perforation test (wedges in the test hair).

30- If the diagnosis is phaeohyphomycosis, what is the organism 1. Alternaria sp. 2. Bipolaris sp.
3. Fusarium sp. 4. Exophiala sp.

A. 1,2,3
B. 2,3,4
C. 1,2,4
D. 1,3,4
E. All of these answers are correct

►C

All these organisms are dematiaceous, Fusarium is a hyalohyphomycete.

31- Which of the following is TRUE regarding coccidiomycosis?

A. First line treatment in pregnancy is itraconazole


B. Droplet transmission is the most common method of acquisition of disease
C. Southeast Asians are at a higher risk of disseminated disease
D. Erythema nodosum is associated with a poor prognosis
E. Approximately 75% of those contracting the disease will be symptomatic

►C

Southeast Asians and African Americans are at a higher risk for disseminated cocci, as are pregnant
women and the immunocompromised. Approximately 60% of those contracting coccidiomycosis

1083
are asymptomatic, and the disease is infectious not contagious. The most common method of
disease acquisition is inhalation from the soil during natural events (dust storms etc). Erythema
nodosum is associated with a good prognosis and the first-line treatment during pregnancy is
amphotericin B. (JAAD 2006 CME)

32- A fungal culture demonstrates a suedelike cream-colored colony of teardrop and balloon-shaped
microconidia which produce a red-brown pigment. Which of the following is true of this organism?

A. Causes ectothrix infection


B. Requires partial thiamine for growth
C. Does not have arthroconidia
D. Causes fluorescent hair infection
E. Is not a cause of tinea unguim

►B

Trichophyton tonsurans is an anthropophilic dermatophyte that causes non-fluorescent endothrix


hair invasion. It can cause black dot tinea capitis, tinea corporis, tinea pedis, and tinea unguium. An
important identifying feature of T. tonsurans is its red-brown pigment that diffuses into the medium.
Also, abundant tear-drop or club shaped microconidia can be found. It grows best in the presence of
thiamine.

33- This organism is often considered a contaminant but has been reported to cause onychomycosis:

A. Sepedonium
B. Curvularia
C. Scopulariopsis
D. Penicillium
E. Phialophora

►C

Causes white superficial onychomycosis.

1084
34- Which of the following statements regarding histoplasmosis is TRUE?

A. Terbinafine is the treatment of choice


B. Histoplasma capsulatum is a yeast at 25°C
C. The organisms are intracellular on histological examination
D. The disease is most common in the San Joaquin Valley
E. The disease is primarily a mucocutaneous infection

►C

Histoplasmosis is primarily a pulmonary infection that can disseminate to other organs. It is


endemic to the Ohio, Missouri and Mississippi River Valleys where the bird and bat droppings in
the soil contain the fungus. Primary cutaneous disease is extremely rare. At 25°C it displays septate
hyphae and at 37°C H. capsulatum is a yeast. Itraconazole is the treatment of choice. On
histopathological examination, intracellular organisms that display a halo are seen. The halo is not
the result of a capsule but rather shrinkage artifact.

35- All of the following are common causes of chromoblastomycosis EXCEPT:

A. Cladosporium carrionii
B. Rhinocladiella aquaspera
C. Phialophora verrucosa
D. Pseudallescheria boydii
E. Fonsecaea pedrosi

►D

Chromoblastomycosis is a chronic fungal infection of the skin and the subcutaneous tissue caused
by traumatic inoculation of a specific group of dematiaceous (pigmented) fungi. Fonsecaea pedrosoi
is the most common causative organism, but Fonecaea compacta, Rhinocladiella aquaspersa,
Phialophora verrucosa, Exophilia jeanselmei and Cladosporium carrionii are pathogenic as well. It
is found most commonly in agricultural workers in the tropics and subtropics, and it is notoriously
resistant to therapy. It presents as verrucous papules and plaques that may coalesce.
Histopathological findings include brown, thick-walled cells known described as ―copper
pennies.ǁ Early in its course, limited disease may respond to surgical excision, electrodessication or

1085
cryosurgery. More extensive lesions may require systemic antifungal agents including itraconazole
or terbinafine, which are the treatments of choice.

36- A 37 year old woman from New Mexico, now 30 weeks pregnant, presents with flu-like
symptoms. Chest x-ray revealed diffuse miliary infiltrates, with blood cultures growing
Coccidioidis immitis. What is the treatment of choice?

A. Amphotericin B
B. Terbinafine
C. Itraconazole
D. Griseofulvin
E. No therapy

►A

There is an increased risk of dissemination of Coccidiomycosis in pregnant women, especially


during the third trimester and the post-partum period. In a review by Crum et al, maternal demise
correlated with disease diagnosed later in pregnancy, with only 45% of patients diagnosed in the
third trimester surviving (Am J Medicine 2006;119(11):Pages 993.e11-993.e17). Given that azoles
have been shown to cause teratogenicity, amphotericin B is recommended as the therapy in
pregnant women.

37- Which of the following organisms causes favus?

A. Trichophyton schoenleinii
B. Trichophyton mentagrophytes
C. Microsporum canis
D. Trichophyton rubrum
E. Microsporum distortum

►A

1086
Favus is a chronic dermatophyte infection defined by the presence of yellowish crusts in the hair
follicles called scutula. Prolonged infections lead to cicatricial alopecias of the scalp and glabrous
skin. The infection is most commonly caused by Trichophyton schoenleinii.

38 -Which of the following causes "black dot ringworm"?

A. M. canis
B. T. violaceum
C. T. verrucosum
D. M. gypseum
E. M. auddouinii

►B

"Black dot ringworm" is endothrix tinea capitis. Black dots are remnants of brittle hair broken at the
surface of the scalp (cuticle intact). On KOH prep, spores are seen within the hair shaft. Causes are
T. rubrum, T. gourvilli, T. yaounde, T. tonsurans, T. schoeleinii, T. soudanense, and T. violaceum
("Ringo Gave Yoko Two Squeaky Violins."). T. verrucosum causes nonfluorescent ectothrix tinea
capitis. M canis, gypseum, and auddouinii also cause ectothrix tinea capitis.

39- Which one of the following is not caused by a Candida infection?

A. Angular cheilitis
B. Balanitis
C. Median rhomboid glossitis
D. Leukoplakia
E. Mycetoma

►E

Candidal infections may clinically present as thrush/leukoplakia, perleche/angular cheilitis,


vulvovaginitis, balanitis, paronychia, onychomcosis, intertrigo, and folliculititis. Median rhomboid
glossitis is now also associated with candidal infections. Mycetomas are caused by true fungi
(eumycetoma) or filamentous bacteria (actinmycetoma).

1087
40- A patient with a pulmonary infection and cervical adenopathy has vesicles, papules and
ulcerations on the oral and nasal mucosa. Tissue culture reveals yeast with multiple buds giving a
marinerǁs wheelǁ appearance. The organism is:

A. Penicillium marneffei
B. Coccidioides immitis
C. Paracoccidioides brasiliensis
D. Blastomyces dermatitidis
E. Leishmania mexicana

►C

Paracoccidioides brasiliensis causes a chronic progressive infection that is most commonly seen in
rural areas of South America. Patients may have constitutional symptoms, lung involvement, ulcers
of the upper respiratory and digestive tract, and verrucous/ulcerated cutaneous lesions. Classically,
Paracoccidioides brasiliensis has a marinerǁs wheelǁ appearance.

41- This 16 year-old patient was recently diagnosed with HIV, the diagnosis is:

A. Distal Onychomycosis
B. Proximal Subungual Onychomycosis
C. Proximal White Subungual Onychomycosis
D. White Superficial Onychomycosis
E. Paronychia with Candida Onychomycosis

►C

1088
Proximal White Subungual onychomycosis is an AIDS marker, nails have a characteristic white
opaque appearance beginning in the region of the lunula and extending distally under the nail plate.

42- The etiologic agent responsible for White Piedra is:

A. Candida albicans
B. Pityrosporum obiculare
C. Corynebacterium tenuis
D. Piedra hortai
E. Trichosporon ovoides

►E

Formerly known as T. beigelii.

43- Which of the following statments regarding pseudohyphae is incorrect?

A. Pseudohyphae are seen in yeasts


B. Are constricted at septations
C. Branching occurs at septations
D. The terminal cell is smaller than the others
E. Are not septated

►E

Pseudohyphae are seen in yeast and resemble true hyphae except that they are constricted at
septations, branching occurs at septations, and the terminal cell is smaller than the others. They are
septated.

44 -Culturing T. rubrum from a white opacity on the fingernail plate should prompt testing for
what?

1089
A. Diabetes mellitus
B. Hypothyroidism
C. Hyperthyroidism
D. HIV
E. Cirrhosis

►D

White superficial onychomycosis of the fingernails is a marker for immunosuppresion and should
prompt testing for HIV. Generally, T. mentag is the most common cause of white superfical
onychomycosis. However, in immunosuppressed patients, T. rubrum is the more common
dermatophyte.

45- A creamy white colony might be any of these organisms except:

A. Candida albicans
B. Prototheca wickerhamii
C. Curvularia
D. Sporothrix schenckii at 37º C
E. Cryptococcus neoformans

►C

Is a dematiaceous organism and is the only organism listed that does not produce creamy white
colonies.

46- White piedra is caused by:

A. Trichophyton cutaneum
B. Trichophyton inkin
C. Trichophyton asahii
D. Trichophyton ovoides
E. Trichophyton mucoides

1090
►A

Patients with white piedra are infected with Trichophyton cutaneum also known as Trichophyton
beigelii. The treatment should be oral azole antifungals and shampoos without shaving the scalp.

47 -This organism does not produce microconidia. The macroconidia are club shaped, and smooth
walled, they grow singly or in clusters.

A. Trichophyton rubrum
B. Epidermophyton floccosum
C. Microsporum gypseum
D. Microsporum canis
E. Trichophyton mentagrophytes

►B

This is the only organism in this list that fits this description.

48- Septate true hyphae are characterized by:

A. Transverse cross walls forming within the hyphae


B. Discrete unicellular bodies
C. Constrictions at septations
D. Branching occuring at septations
E. A terminal cell that is smaller than the others

►A

Hyphae are vegetative tube-like structures. In septate hyphae, transverse cross walls form within the
hyphae. Pseudohyphae, seen in yeast, resemble true hyphae except that they are constricted at
septations, branching occurs at septations, and the terminal cell is smaller than the others.

1091
49- Which of the following is most likely to be a primary cutaneous infection?

A. North American Blastomycosis


B. Histoplasmosis
C. Paracoccidioidomycosis
D. Cryptococcosis
E. None

►A

All other mycoses are primary pulmonary infections, which may disseminate to the skin.

50- A 45 year-old agricultural worker from Brazil presented with ulcers of the buccal mucosa and
tongue. Cervical lymph nodes were tender and enlarged. The biopsy would most likely reveal:

A. Yeast cells in chains and a large thick walled round central yeast cell surrounded by
several thinly attached budding smaller yeast cells
B. Yeast cells with large capsules
C. Small budding yeast cells
D. Yeast cells with pseudohyphae
E. Copper pennies

►A

Yeast cells in chains and a large thick walled round central yeast cell surrounded by several thinly
attached budding smaller yeast cells p.18. This biopsy is describing Paracoccidioides brasiliensis,
an organism endemic in Brazil. Its yeast form seen in tissue produces a Mariners wheel-like
configuration. Generally this organism is inhaled and disseminates causing mucocutaneous lesions
with lymphadenopathy.

51- A patient with white nodules on the hair shaft has a KOH which shows hyphae and
arthroconidia. The etiology is:

A. Corynebacterium minutissimum

1092
B. Exophiala wernickii
C. Piedraia hortae
D. Trichosporon beigelii
E. Corynebacterium tenuis

►D

White piedra is an infection of the hair shaft caused by Trichosporon beigelii. Unlike black piedra
which is firmly adherent to the shaft, white piedra presents as light brown nodules composed of
hyphae and arthroconidia, which move easily along the hair shaft. The most common sites of
infection occur on the mustache, beard and pubic area.

52 -The most sensitive microscopic test for fungal infection is:

A. Potassium Hydroxide
B. Potassium Hydroxide with DMSO
C. Chlorazol Black E
D. Calcofluor white
E. Swartz Lamkins stain

►D

Calcofluor white is the most sensitive microscopic test for fungal infection. It is a glucan specific
immunofluorescent stain. The remaining options are useful in direct microscopic examination, but
not the most sensitive.

53 -A patient with scaly feet has a positive KOH. Fungal culture reveals smooth, club-shaped
macroconidia attached to hyphae in groups. No microconidida are seen. The organism is:

A. Microsporum canis
B. Epidermophyton floccosum
C. Trichophyton rubrum
D. Microsporum gypseum
E. Trichophyton tonsurans

1093
►B

The wet mount Epidermophyton floccosum shows smooth, thin-walled macroconidia without
microconidia. The appearance is occasionally referred to as ―snow shoesǁ. Epidermophyton
floccosum is incapable of hair invasion, and therefore, does not cause tinea capitis.

54- A whitish, heaped and convoluted colony with growth submerged into the agar and a colorless
reverse was isolated from the scalp of a 35-year-old male. The organism did not produce any
conidia. The diagnosis most likely is:

A. Microsporum canis
B. Microsporum ferrugineum
C. Trichophyton rubrum
D. Trichophyton tonsurans
E. Trichophyton schoenleinii

►E

Produces a white cerebriform colony lacking any conidia. M. ferrugineum produces a rust colored
colony with bamboo-like hyphae without conidia. The other organisms produce micro and
macroconidia.

55- A horticulturist of sphagnum moss topiaries comes in with a nodular eruption with lymphangitic
spread and treatment with oral potassium iodide is initiated. What is the most well recognized side
effect of this treatment.

A. Gastrointestinal distress
B. Shortness of breath
C. Flushing
D. Angioedema
E. Pruritus

►A

1094
This patient has sporotrichosis. Sporotrichosis is mainly an occupational disease of farmers,
gardeners, and horticulturists. Persons who handle thorny plants, sphagnum moss, or baled hay are
at increased risk. Outbreaks have occurred in nursery workers who handled sphagnum moss, rose
gardeners, children playing on baled hay, and greenhouse workers who handled bayberry thorns
contaminated by the fungus. Classic treatment is with oral potassium iodide for 3-4 weeks. The
most recognised side effect of treatment is gastrointestinal distress. Thyroid function tests should be
performed during treatment as suppression can occur.

56- A slimy, mucoid colony growing on Sabouraud's agar at 37 degrees Celsius is observed to
produce urease. The organism is:

A. Candida albicans
B. Cryptococcus neoformans
C. Candida glabrata
D. Candida tropicalis
E. Aspergilus flavus

►B

Cryptococcus neoformans is an encapsulated yeast with a worldwide distribution. Cryptococcosis is


usually acquired by inhalation and subsequent dissemination to various organs including the
meninges and the skin can occur. Immunosuppressed patients are particularly susceptible to
infection. In approximately ten percent of disseminated cases, cutaneous lesions may develop.
These present as acneiform papules or pustules and may progress to infiltrated plaques, nodules, or
ulcers. Less commonly, cutaneous lesions can represent primary cutaneous cryptococcosis via direct
inoculation of organisms into the skin. Diagnosis is made by direct microscopy and India ink stains
can aid visualization. The organisms are large, encapsulated budding yeasts. In histopathologic
sections, capsules stain with alcian blue and mucicarmine stains. In culture, C. neoformans is
distinguished by production of urease as well as the ability to pigment on Guizotia seed medium.
Colonies are described as slimy and mucoid. Serologic antigen-detection assays are also available.
Treatment of disseminated cryptococcosis includes amphotericin combined with flucytosine.

57- Which of the following statements regarding lobomycosis is FALSE?

A. Itraconazole is the treatment of choice

1095
B. It resembles a ―chain of coinsǁ on histopathology
C. It is also known as ―keloidal blastomycosisǁ
D. Lacazia (formerly Loboa) loboi is the number one cause
E. The infection also occurs in dolphins

►A

Lobomycosis, also know as keloidal blastomycosis is caused by Lacazia loboi (formerly Loboa
loboi). It is endemic to Brazil and the Caribbean and is associated with dolphins. Clinically, the
disease presents with painless keloidal papules and plaques, ulcerative lesions and/or verrucous
lesions. Biopsy specimens display multiple budding thick-walled cells attached with a bridge, often
referred to as a ―chain of coins.ǁ Surgical treatment is necessary as antifungal medications are
ineffective.

58- Medlar bodies are diagnostic of infection with which organism?

A. Blastomycosis
B. Chromomycosis
C. Coccidiomycosis
D. Histoplasmosis
E. Sporotrichosis

►B

Chromoblastomycosis, or chromomycosis, is a cutaneous mycosis caused by dematiaceous, or


pigmented, fungi. Several fungal species are associated with this infection including Phialophora
verrucosa, Fonsecaea pedrosoi, F compactum, Exophiala (Fonsecaea, Wangiella) dermatitidis and
Cladosporium carrionii. Infection is typically trauma-induced and involves the lower extremities.
Lesions appear as verrucous papules, nodules and plaques, with occasional elephantiasis resulting
from lymphatic blockage. Histopathologic findings include pseudoepitheliomatous epidermal
hyperplasia and a dermal infiltrate composed of epithelioid histiocytes, multinucleated giant cells,
and small clusters of inflammatory cells including plasma cells, neutrophils, eosinophils and
lymphocytes. The characteristic histopathologic feature is the presence of dark brown, thick-walled,
ovoid spheres in clusters or chains referred to as "copper pennies" or Medlar bodies. They are
visible without the use of special stains.

1096
59- Which of the following is FALSE with regards to asexual reproduction of fungi?

A. Arthroconidia are formed by the fragmentation of hyphae


B. Sporangia are spores that are produced in a sac
C. Chlamydoconidia are thin-walled and are susceptible to environmental injury
D. Conidia are cells produced on the sides or ends of hyphae
E. Dematophytes produce conidia

►C

The structures formed during asexual propagation of fungi are termed either spores or conidia
depending on their mode of production and they arise following mitosis of a parent nucleus.
Conidia arise either by budding off conida-producing hyphae (on the sides or ends) or by
differentiation of preformed hyphae. The size and shape are generally characteristic of the
organism. Dermatophytes produce conidia. Arthroconidia are produced when hyphae fragment or
lyse. They may appear as thick- or thin-walled cells and are typically larger than the hyphae from
which they came. The separation occurs at the septae. The mold form of Coccidioides immitis
reproduce in such a fashion. Sporangia are sacs or cases that are multicellular structures in which
spores are produced. Zygomycetes reproduce in this way. Chlamydoconidia are thick walled, round
conidia that are formed during unpleasant environmental conditions. Trichophyton tonsurans
produces these structures.

60- This dermatophyte has a growth requirement for inositol and thiamine

A. Trichophyton equinum
B. Trichophyton violaceum
C. Trichophyton concentricum
D. Trichophyton tonsurans
E. Trichophyton verrucosum

►E

Trichophyton verrucosum is a zoophilic fungus that requires thiamine and sometimes inositol for
growth. T. violaceum and tonsurans only have a partial requirement for thiamine. Trichophyton
equinum requires niacin. (Horses are nice).

1097
61- Which of the following is not true regarding fungal culture media containing cycloheximide?

A. Cycloheximide is not found in Sabouraud Dextrose Agar (SDA) Emmons Modification


B. Cycloheximide is found in Mycosel
C. Cycloheximide is found in Dermatophyte Test Media
D. Cycloheximide is found in Mycobiotic
E. Cycloheximide inhibits bacterial flora

►E

Sabouraud Dextrose Agar (SDA) Emmons Modification does not contain cycloheximide. Mycosel/
Mycobiotic (SDA with cycloheximide and chloramphenicol) and Dermatophyte Test Media contain
cycloheximide. Cycloheximide inhibits rapidly growing nonpathogenic molds and some pathogens
(Cryptococcus neoformans, some Candida species, Prototheca, Scytalidium species, yeast forms of
Histoplasma and Blastomyces). Chloramphenicol inhibits bacterial flora.

62 -Which structure is found in a biopsy of Candida tropicalis, it is branching and pinching in at the
points of septations:

A. Septate hyphae
B. Arthroconidia
C. Mosaic fungus
D. Pseudohyphae
E. Pectinate bodies

►D

Pseudohyphae are products of yeast, which produce elongated yeast cells that do not pinch off,
branching and pinching in at the points of septations.

63- A 30 year-old male living in the Chicago suburbs complained of a slowly growing verrucous
plaque with sharp borders on his left wrist. A biopsy revealed yeast cells 10-14 um. Mucicarmine
was negative. A fluffy white colony grew at room temperature having small round conidia on thin
conidiophores. The diagnosis is:

1098
A. Blastomycosis
B. Cryptococcosis
C. Candidiasis
D. South American Blastomycosis
E. Histoplasmosis

►A

The biopsy describes Blastomyces dermatitidis, the negative mucicarmine rules out Cryptococcus.
Chicago is an endemic area for Blastomyces. The colony morphology also describes Blastomyces.

64- All of the following statements regarding superficial mycotic infections are true EXCEPT:

A. Trichosporon species can cause systemic disease in immunocompromised patients


B. Malassezia furfur is the most common cause of tinea versicolor
C. Trichosporon beigelii causes white piedra
D. Malassezia species may be associated with neonatal cephalic pustulosis
E. Piedraia hortae causes black piedra

►B

Trichosporon asahii can cause disseminated disease in immunocompromised patients, especially


those with neutropenia. Patients develop nodular and then necrotic lesions and require systemic
antifungal agents. Malassezia globosa is the most common cause of tinea versicolor, not M. furfur.
M. pachydermatis, M. dermatis, M. obtuse, M. restrica, M. sympodialis and M. slooffiae are all
thought to be medically important as well. M.sympodialis is part of the skin‘s normal flora.
Trichosporon beigelii and Piedraia hortae are the etiological agents of white and black piedra
respectively. Several Malassezia species have been implicated in neonatal cephalic pustulosis
(‗neonatal acne‘).

65- A 6 year-old Hispanic girl came into the clinic with a history of alopecia and scaly scalp for 3
weeks. The Wood's lamp examination was negative and her cervical lymph nodes were not

1099
enlarged. It was noted that the hairs were broken off at the surface of the scalp. The most likely
organism to be isolated would be:

A. M. audouinii
B. M. canis
C. M. gypseum
D. T. tonsurans
E. T. verrucosum

►D

T. tonsurans and T. violaceum cause the endothrix infection "Black Dot Ringworm". All other
choices cause Grey Patch Ringworm.

66- A solitary chancre-like lesion appeared on the arm of a florist who has a pet dog. No organisms
were seen on biopsy, however Sporothrix schenckii was cultured from the tissue. This infection is
known as ―fixed cutaneous sporotrichosisǁ. It remains fixed because:

A. There is a resistance due to a prior exposure


B. There is a decreased resistance due to AIDS
C. The infection was acquired from the dog rather than from a plant
D. The infection occurred in the absence of tissue injury
E. This species of Sporothrix has limited infectivity

►A

There is a resistance due to a prior exposure.

67 -Which of the following is the most useful morphologic feature in identifying the mycelial phase
of Histoplasma capsulatum?

A. Arthroconidia in every other cell


B. Encapsulated spores 2-5 um

1100
C. Tuberculate macroconidia 8-14 um
D. Small oval conidia on long thin conidiophores
E. Microconidia laterally along the hyphae strand

►C

On artificial media the mould form of Histoplasmosis produces hyphae with tear drop microconidia
and round thick wall tuberculate macroconidia.

68- Nondermatophytes growing on Dermatophyte Test Media cause the media to turn what color?

A. Amber
B. Red
C. Yellow
D. Black
E. Green

►C

Dermatophytes utilize protein as a carbon source producing alkaline by-products causing


Dermatophyte Test Media to turn from amber to red. Nondermatophytes cause the media to turn
yellow due to acid by-products.

69-A patient has a positive Wood's light exam that is caused by pteridine. What is the causative
condition?

A. Microsporum ferrugineum
B. Corynebacterium
C. Trichophyton violaceum
D. Pseudomonas
E. Trichophyton tonsurans

►A

1101
Wood's light positive dermatophytes (M. canis, M. audouinii, M. distortum, M. ferrugineum, M
gypseum, and T. schoenleinii) fluoresce secondary to pteridine. Corynebacterium can fluoresce as
well due to coproporphyrin III. Pseudomonas fluoresces from production of pycyanin. The other
organisms do not react with a Wood's light.

70- A young girl presented with a scaly annular facial rash and alopecia of her lower eyelashes.
Which of the following statements is/are true? 1. A KOH prep and fungal culture might confirm the
diagnosis. 2. The etiologic agent might be Microsporum canis. 3. The family puppy might be
infected. 4. Griseofulvin would be the drug of choice. 5. A topical azole cream would be the drug of
choice.

A. 1,2,4
B. 1,2,3,4
C. 1,3,4
D. 1,2,5
E. 1,2,3,5

►B

KOH and culture are the best and most reliable lab tests to diagnose a fungal infection. M. canis is
frequently found in kittens and puppies and is a common cause of tinea faciei in children. An oral
antifungal should be prescribed when a fungal infection involves hair.

71- Growth of which of the following is not inhibited on Mycosel media?

A. Cryptococcus neoformans
B. Yeast forms of Histoplasma
C. Yeast forms of Blastomyces
D. Microsporum gypseum
E. Scytalidium species

►D

1102
Cycloheximide in Mycosel or Mycobiotic media (SDA with cycloheximide and chloramphenicol)
inhibits rapidly growing nonpathogenic molds and some pathogens (Cryptococcus neoformans,
some Candida species, Prototheca, Scytalidium species, yeast forms of Histoplasma and
Blastomyces).

72 -Which of the following most accurately describe conidia?

A. Cells produced on the end or sides of hypha or conidiophore


B. Spores that are produced in a sac
C. Thick-walled round cell
D. Formed by budding
E. Formed by fragmentation of hyphae

►A

Conidia are cells produced on the end or sides of hypha or conidiophore, the size and shape
arrangement are generally characteristic of the organism. The remaining descriptions are of specific
types of conidia. Spores produced in a sac are sporangia, thick-walled round cells that
are resistant to the environment are chlamydoconidia, blastoconidia are formed by budding and
arthroconidia are formed by fragmentation of hyphae.

73 - An organism producing an apricot colored colony and reflexive branching with few conidia
was cultured from a patient from Africa. Which of the following is/are true? 1. This mostly likely is
T. verrucosum 2. This most likely is T. soudanense 3. This most likely is T. violaceum 4. This
organism causes endothrix tinea capitis 5. This organism is zoophilic

A. 1,5
B. 2,4
C. 2,4,5
D. 3, 4
E. 3, 4, 5

►B

1103
T. soudanense produces an apricot colored colony with reflexive branching. It is an anthropophilic
organism causing an endothrix tinea capitis. T. verrucosum produces a white colony and requires
thiamine and inositol for sporulation, T. violaceum produces a purple colony and has a partial
requirement for thiamine

74- Yeast:

A. Are filamentous fungi


B. Are characterized by tubular branching cells
C. Form fuzzy colonies
D. Form smooth colonies
E. Are unicellular oval to round cells that reproduce by budding or fission

►E

All of the listed features are those of molds. Yeast are unicellular, oval to round cells that reproduce
by budding or fission. They form moist colonies, not smooth or fuzzy colonies.

75- Clinically, actinomycotic mycetoma and eumycotic mycetoma appear identical. The importance
of identifying the etiologic agent is

A. Academic
B. Selecting the appropriate therapy
C. Ordering appropriate stains
D. Determine if the infection is contagious
E. Determine if amputation is necessary

►B

The etiologic agents of actinomycotic mycetoma are filamentous bacteria that require antibiotics,
whereas fungal agents causing eumycotic mycetoma require antifungal therapy.

1104
76- Which of the following organisms will cause infections of skin, nails and endothrix hair?

A. Epidermophyton floccosum
B. Microsporum audouinii
C. Trichophyton schoenlinii
D. Microsporum canis
E. Trichophyton mentagrophytes

►C

E. floccosum does not infect hair; M. audouinii and M. canis cause fluorescent ectothrix T. capitis,
T. mentagrophytes ectothrix T. capitis. T.Schoenlinii can cause endothrix or ectothrix, fluoresces
blue white. It causese favus with hyphae with air spaces in them.

77- Which of the following statements about arthroconidia is correct?

A. Arthroconidia are formed by budding


B. Arthroconidia are formed by fragmentation of hyphae
C. Arthroconidia are thick-walled round cells
D. Arthroconidia are spores that are produced in a sac
E. Arthroconidia are yeast forms of dimorphic fungi

►B

Arthroconidia are formed by fragmentation of hyphae and may appear as thick or thin walled
rectangular cells. An example would be the mould form of Coccidioides immitis. The remaining
options are incorrect.

78- Numerous umbilicated nodules resembling molluscum contagiosum developed in a patient with
recently diagnosed HIV infection. Which of the following fungi might be the causative agent(s)? 1.
Candida tropicalis 2. Cryptococcus neoformans 3. Aspergillus fumigatus 4. Histoplasma capsulatum
5. Penicillium marneffei

A. 2

1105
B. 3
C. 2, 4
D. 4, 5
E. 2, 4, 5

►E

All three organisms are seen in AIDS patients and can produce molluscum-like lesions.

79- Which actinomycotic organism has red grains?

A. Streptomyces somaliensis
B. Nocardia asteroides
C. Actinomadura madurae
D. Actinomadura pelletieri
E. Nocardia brasiliensis

►D

Mycetoma is a granulomatous infection of dermal and subcutaneous tissues usually occurring on


the foot. Draining sinuses containing grains characterize such infections. Three types of mycetoma
exist: eumycotic (true fungal), actinomycotic (filamentous organisms) and botryomycotic (bacterial
infections). Madurella and Leptosphaeria species produce black grains. Pseudallescheria and
Acremonium species, along with dermatophytes, make white grains. Nocardia brasiliensis and N.
asteroides produce white grains, while Nocardia caviae and Actinomyces israelii tend to have
yellow-white grains. Actinomadura madurae produce pink or white grains and Actinomadura
pelleteri make red grains. Streptomyces somaliensis produce brown or yellow grains.

80- Which of the following statements regarding dermatophyte infection FALSE?

A. Trichophyton schoenleinii is a common cause of favus


B. Microsporum canis is associated with ectothrix tinea capitis
C. Trichophyton violaceum is normally associated with endothrix tinea capitis
D. Microsporum audouinii displays yellow fluorescence with Wood‘s lamp examination

1106
E. Trichophyton rubrum is always an ectothrix infection

►E

Organisms that cause an ectothrix pattern of tinea capitis include M. canis, M. audouinii, M.
ferruginosum, M. distortium, M. gypseum and occasionally T. rubrum. T. tonsurans, T. violaceum,
T. soudanense, T. gourvilli, T. yaoundei, and occasionally T. rubrum cause an endothrix pattern. T.
schoenleinii causes favus in which hyphae and air spaces are seen in t he hair shaft. A bluish-white
fluorescence pattern is seen with Wood‘s lamp. Clinically, patients have thick, yellow, cup-shaped
crusts (scutula); scarring and secondary infection may result. M.
canis, M. audouinii, M. ferruginosum, M. distortium display a yellow fluorescence on Wood‘s
lamp examination.

81- Penicillium marneffei is an infection endemic to which part of the world?

A. Mexico
B. South America
C. Africa
D. Southeast Asia
E. The Caribbean

►D

Penicillosis is a recently recognized disease caused by infection with Penicillium marneffei, a


dimorphous fungus that is contracted through inhalation. Cases have been described in Vietnam,
China, and Thailand as well as imported cases in the U.S. in veterans returning from Vietnam.
Bamboo rats in these areas have been shown to be infected with P. marneffei and may represent a
reservoir. Although penicillosis has a tendency to occur in immunocompromised hosts
opportunistically, it has also been described in normal hosts. Clinical presentation is typically with
pulmonary or disseminated disease which can affect the skin, gastrointestinal tract, spleen, lymph
nodes, skin, and bone marrow. Cutaneous features include multiple umbilicated papules which can
enlarge and ulcerate occurring commonly on the face and trunk. Diagnosis is by culture or
histopathology. Treatment is with itraconazole. Severe cases may require amphotericin B.

1107
82- Trichosporon ovoides is a cause of:

A. Black piedra
B. White piedra
C. Tinea nigra palmaris
D. Ectothrix tinea capitis
E. Endothrix tinea capitis

►B

White piedra is cause by Trichosporon beigelii, or Trichosporon ovoides and T. inkin (new
nomenclature). It presents with tan to white soft, nonadherent small concretions ~1mm, seen on the
scalp, beard, moustache, and pubic areas. The hairs may fluoresce.

83- Which one of the following agents accounts for the depigmentation seen in pityriasis
versicolor?

A. Thymidine kinase
B. Ketoconazole
C. Acetone
D. Dicarboxylic acid
E. Postinflammatory effect

►D

Pityriasis versicolor is primarily caused by the yeast M. furfur. M. furfur is a dimorphic, lipophilic
organism that can produce metabolites such as azaleic acid (a dicarboxylic acid) that can inhibit
tyrosinase and injure melanocytes.

84- What is the most likely cause of this infection?

A. T. tonsurans
B. T. schoenleinii
C. M. canis

1108
D. T. mentag
E. T. rubrum

►C

M. canis and T. tonsurans both cause tinea capitis. M. canis is more commonly associated with
inflammatory tinea capitis and kerion formation.

85- Tinea imbricata is most frequently caused by:

A. T. mentagrophytes
B. M. Audouinii
C. T. concentricum
D. E. floccosum
E. M. furfur

►C

Tinea imbricata or Tokelau, an unusual type of tinea corporis causing polycyclic scaly lesions, is
endemic in the South Pacific Islands, Far East and C. & S. America.

86- The major endemic area for Histoplasmosis

A. Africa
B. Central America
C. Southwest United States
D. South America
E. Eastern United States

►E

1109
The endemic area for Histoplasmosis includes: the Ohio, Mississippi and Missouri River Valleys
Syracuse NY area and the Caribbean. This organism prefers soil with a high nitrogen content such
as that enriched with bird and bat guano.

87 -Black granules are found in mycetoma caused all the following organisms except:

A. Madurella grisea
B. M. mycetomatis
C. Exophiala jeanselmei
D. Curvularia
E. Scedosporium apiospermum

►E

All organisms listed other than S. apiospermum cause black granules. S. Apiospermum which is the
filamentous mold form of pseudallescheria boydii has white granules.

88- Which of the following methods of direct microscopic examination is chitin specific?

A. Chlorazol black E
B. Calcofluor white
C. Gomori Methenamine Silver
D. Fontana-Masson
E. KOH

►A

Chlorazol black E is chitin specific. Calcofluor white is glucan specific. Gomori Methenamine
Silver (GMS) and Fontana-Masson are histology stains. KOH is a rapid, easy, reliable method for
diagnosing fungal infections, but is not chitin-specific.

89- Which of the following statements regarding mycelium is NOT true?

1110
A. A mass of hyphae is mycelium
B. Mycelium has reproductive capability
C. Hyphae forming corkscrew-like turns are spiral hyphae
D. Pectinate bodies are hyphae resembling a comb
E. Racket forms are club-shaped cells

►B

Mycelium does not have reproductive capabilities. The remaining statements are correct.

90 -All of the following are features of mycetoma except:

A. Swelling
B. Self-limited
C. Granules
D. Fascia and bone involvement
E. Draining sinuses

►B

Mycetomas may be subdivided into eumycetoma and actinomycetoma . Mycetomas generally begin
as subcutaneous swellings occurring on the feet. Later, sinuses, granules, and nodules may be seen.
The skin and subcutaneous tissue may also become involved. Treatment is difficult in later stages.

91- A landscape worker complained of several tender nodules on the right dorsal hand and forearm.
The biopsy of the lesion showed asteroid bodies but no organisms. The likely diagnosis is:

A. Nocardiosis
B. Blastomycosis
C. Sporotrichosis
D. Candidiasis
E. Coccidioidomycosis

1111
►C

Sporothrix schenckii produces tiny yeast cells 4-6 um difficult to demonstrate on biopsy unless
there is an overwhelming infection. Asteroid bodies are seen on H & E and PAS as radiating fingers
of eosinophilic material.

92- A 49-year-old man has painless subcutaneous nodules on his feet with sinus tracts and
abscesses. Pathology shows "grain" in sinus tract drainage. A diagnosis of eumycotic mycetoma is
made. What is the most common cause in the United States?

A. Acremonium
B. Curvalaria
C. Exophilia jeanselmei
D. Pseudallescheria boydii
E. Nocardia

►D

Eumycotic mycetoma is fungal and can be caused by Pseudallescheria boydii (most common in
US), Madurella, Acremonium, curvalaria, and Exophilia jeanselmei. Actinomycotic mycetoma is
bacterial and maybe caused by Nocardia asteroids or brasiliensis, Streptomyces, and Actinomyces.

93- Which of the following organisms would you expect to recover from a fluorescent tinea capitis?

A. T. tonsurans
B. M. gypseum
C. M. canis and M. gypseum
D. T. violaceum
E. T. rubrum

►C

M. gypseum may occasionally produce a dull fluorescence. M. canis is fluorescent.

1112
94- A child presents with several yellowish, cup-shaped crusts on the scalp, some with single hairs
piercing through the center. KOH prep reveals arthroconidia and airspaces within the hair shaft. A
likely cause is:

A. M. gypseum
B. M. canis
C. T. tonsurans
D. T. verrucosum
E. T. mentagrophytes

►A

This child is presenting with the characteristic scutula of favus. Favus is most often caused by T.
schoenleinii, T. violaceum, and M. gypseum.

95- Which of the following is true regarding culture of the organisms that cause Pityriasis
versicolor?

A. Difficult to grow, requires olive oil overlay


B. Grow easily on Mycosel media
C. Cannot be cultured
D. Grow easily on Dermatophyte Test Media
E. Grow easily on Sabouraud Dextrose Agar

►A

Pityriasis versicolor (Tinea versicolor) is caused by Malassezia furfur, M. globosa (most common
cause), M. pachydermatis, M. dermatis, M. obtuse, M. restrica, M. sympodialis, and M. sloofiae.
The organisms that cause Pityriasis versicolor are difficult to grow. The culture requires olive oil
overlay.

96- Choose the correct statement regarding Coccidioidomycosis:

A. The most common form of primary inoculation is cutaneous

1113
B. Dissemination may involve the bones, joints, viscera, brain and skin
C. Causative organism, C. immitits, is a thick-walled spherule with
a polysaccharide capsule demonstrated with Alcain blue
D. Erythema nodosum is a poor prognotic sign
E. Outbreaks occur in the Mississippi and Ohio River Valley

►B

Coccidioidomycosis: Primary inoculation is pulmonary. Erythema nodosum is a favorable


prognostic sighn. Dissemination from the localized pulomary lesions may occur in less than 1% of
infections. Target ogans of dissemination include the bones, joints, viscera, brain, meinnges, and
skin. The causative organisms is Coccidioides immitis, is a nonbudding, thick-walled spherule, with
a polysaccharide capsule demonstrated with Gridley or Gomori methanamine silver stain. It is
endemic in northern Mexico, Venezuela, and in southwest US, especially California

97 -This organism produces an endothrix tinea capitis:

A. Trichophyton mentagrophytes
B. Microsporum gypseum
C. Microsporum nanum
D. Trichophyton verrucosum
E. Trichophyton soudanense

►E

Only T. soudanense in this group of choices produces an endothrix type of tinea capitis. Endothrix
do not fluoresce. Trichophyton mentagrophytes, Microsporum nanum and Trichophyton
verrucosum produce nonfluorescent ectothrix tinea capitis. Microsporum gypseum may produce a
fluorescent or nonfluorescent ectothrix tinea capitis.

98 -Mycelia can form structures with a comb-lke appearance called:

A. Racket forms
B. Favic chandeliers

1114
C. Pectinate bodies
D. Spiral hyphae
E. Nodular bodies

►C

Racket forms (club-shaped cells), favic chandeliers (terminal hyphal branches having an antlerlike
appearance), pectinate bodies (hyphae resembling a comb), spiral hyphae (hyphae forming
corkscrew-like turns), and nodular bodies (knot-like structure of hyphae) are among the types of
mycelia.

99- Which of the following stains is specific for chitin?

A. KOH
B. KOH with DMSO
C. Swartz Lamkins
D. Chlorazol Black E
E. Calcofluor White

►D

Chlorazol Black E is a chitin specific stain. The remaining options are useful in direct microscopic
examination for fungal elements, but are not chitin specific.

100- Sabouraud Dextrose Agar (SDA) Emmons Modification contains which of the following?

A. Peptones
B. Phenol red
C. Chloramphenicol
D. Gentamicin
E. Cycloheximide

►A

1115
SDA Emmons Modification is the gold standard medium. It is nutritionally poor, containing
dextrose, peptones, water, and agar, and encourages sporulation. Phenol red, gentamicin, and
cycloheximide are found in dermatophyte test media (DTM). Chloramphenicol and cycloheximide
are found in Mycosel/Mycobiotic.

101 -A 12 year old boy has a pruritic bullous eruption on his feet. A KOH is positive and a fungal
culture shows microconidia in grape-like clusters. The etiology is:

A. Trichophyton verrucosum
B. Microsporum canus
C. Trichophyton mentagrophytes
D. Microsporum gypseum
E. Trichophyton tonsurans

►C

Trichophyton mentagrophytes is the dermatophyte primarily responsible for bullous tinea pedis. It
typically has septate, spiral hypahe with cigar or grape like thin-walled microconidia.

102- Cutaneous lesions of Cryptococcosis may be 1. Nodular 2. Papular 3. Granulomatous


-ulcerative 4. Herpetiform 5. Cellulitis-like:

A. 1, 3, 4
B. 3, 4, 5
C. 2, 3, 4
D. 1, 3, 4, 5
E. All of these answers are correct

►E

Polymorphous lesions have been reported in Cryptococcosis

1116
103- A neutropenic patient has erythematous nodules for several days. A biopsy shows branching
septate hyaline hyphae. Which of the following conditions should be included in the differential?
1. Coccidioidomycosis 2. Fusariosis 3. Aspergillosis 4. Zygomycosis 5. Phaeohyphomycosis

A. 1
B. 1, 2
C. 2, 3
D. 3, 4
E. 2, 3, 5

►C

Aspergillus and Fusarium produce similar appearing morphology in biopsy. Coccidioidomycosis


produces spherules, very few if any septations are seen in Zygomycosis, dark hyphae, yeast cells are
seen in Phaeohyphomycosis.

104 - Trichophyton tonsurans sporulates via thick-walled round cells resistant to the environment
known as:

A. Arthroconidia
B. Blastoconidia
C. Chlamydoconidia
D. Sporangia
E. Mycelia

►C

Chlamydoconidia are thick-walled round cells, resistant to the environment. Arthroconidia are
formed by fragmentation of hyphae, and may appear as thick or thin-walled rectangular cells.
Blastoconidia are formed by budding. Sporangia are spores that are produced in a sac.

105- Which of the following methods of direct microscopic examination for fungi is glucan
specific?

A. KOH
B. Swartz Lamkins

1117
C. Chlorazol black E
D. Calcofluor white
E. Mayer's mucicarmine

►D

Calcofluor white is a glucan specific stain. Chlorazol black E is chitin specific. KOH is a rapid,
easy, reliable method of diagnosing fungal infections but is not glucan specific; Swartz Lamkins
contains a counterstain but is not glucan specific. Mayer's mucicarmine is a histology stain.

106- Which of the following causes tinea imbricata?

A. Epidermophyton floccosum
B. Trichophyton concentricum
C. Trichophyton rubrum
D. Trichophyton mentagrophytes
E. Trichophyton tonsurans

►B

Tinea imbricata is characterized by concentric, annular, scaly rings resembling erythema annulare
centrifugum. The causative agent is T. concentricum, which is endemic to the South Pacific, South
and Central America and Asia. The infection is usually chronic.

107 -Which of the following statements regarding superficial mycotic infections is TRUE?

A. Malassezia species do not fluoresce under a Wood‘s lamp


B. Phaeoannellomyces wernecki causes tinea nigra
C. Trichophyton beigelii causes white piedra
D. Trichosporon hortae causes black piedra
E. Micrococcus sedentarius causes tinea nigra

►B

1118
Malassezia species fluoresce pale yellow with Wood‘s lamp examination. Trichosporon beigelii (not
trichophyton) causes white piedra, an infection that causes tan to white, soft non-adherent
concretions on head and pubic hairs. Piedraia hortae causes black piedra, which forms dark
concretions that are adherent to scalp, beard, and pubic hairs. Micrococcus sedentarius is a
bacterium that is responsible for pitted keratolysis, which is characterized by crateriform pitting that
primarily affects the pressure-bearing aspects of the plantar surface of the feet and, occasionally, the
palms of the hands. Tinea nigra (superficial phaeohyphomycosis) presents with asymptomatic
brown to black, macular, non-scaly macules and patches on the palms or soles. Phaeoannellomyces
werneckii is causative.

108 -The outstanding characteristics of Aspergillus species on biopsy include:

A. Blastoconidia
B. Hyaline, septate dichotomously branching hyphae
C. Copper pennies
D. Ribbon-like filaments that may be twisted and distorted branching at right angles
E. Stain with Fontana-Mason stain

►B

Aspergillus is seen as wide (3 um) septate dichotomously branching (45ǁ angles) hyphae.

109- Of the following Candida species, which is likely to cause disseminated Candidiasis with
cutaneous lesions?

A. Candida albicans
B. C. parapsilosis
C. C. tropicalis
D. C. krusei
E. C. neoformans

►C

1119
C. albicans is responsible for approximately 50% of all Candidemia, but C. tropicalis is seen with
increasing frequency among patients with dissemination and cutaneous manifestations.

110- This dermatophyte is not an anthropophilic organism:

A. Trichophyton rubrum
B. Epidermophyton floccosum
C. Microsporum gypseum
D. Microsporum audouinii
E. Trichophyton soudanense

►C

M. gypseum is a geophilic organism, all others listed are anthropophilic.

111- Two weeks after a bicycle accident a 25-year-old female diabetic patient complained of a
swollen tender right cheek The PAS biopsy demonstrated ring forms and distorted wide hyphae
with few septations and right angle branching. The diagnosis is:

A. Aspergillosis
B. Actinomycosis
C. Mucormycosis
D. Dermatophytosis
E. Fusariosis

►C

Having diabetes is a risk factor for Mucormycosis, ring forms in the biopsy are the x-section of the
wide sparsely septate hyphae.

112- Which of the following usually does not fluoresce bright green upon Woods lamp
examination?

1120
A. M. audouinii
B. M. canis
C. T. violaceum
D. T. schoenleinii
E. M. distortum

►C

Fluorescent-positive infections are caused b y:"Dogs And Cats Fight Some Gypsies" M. Distortum
M. audouinii M canis M. ferrugineum T. schoenleinii M. gypsium (occasionally)

113- Which of the following is NOT used for histologic examination for fungal infection?

A. Chlorazol Black-E
B. Gormori Methanamine Silver
C. Periodic Acid Schiff
D. Fontana-Masson
E. Mayer's mucicarmine

►A

Chlorazol Black-E is a stain specific for chitin and is used in direct examination, not histologic
preparations. The remaining options are histology stains useful in fungal identification in tissue.

114- This organism causes a resistant tinea pedis indistinguishable from dermatophytosis:

A. Aspergillus flavus
B. Scytalidium dimidiatum
C. Curvularia sp.
D. Scopulariopsis
E. Fonsecaea sp

►B

1121
This fungus is very difficult to treat due to its resistance to most antifungals. It is sensitive to
cylcoheximide so it should be cultured on media free of this antifungal.

115- Which of the following organisms is most likely to cause a sporotrichoid nodule on the arm:

A. Nocardia brasiliensis
B. Phialophora verrucosa
C. Rhinospiridium seeberii
D. Fusarium
E. Fonsecaea pedrosoi

►A

Nocardia brasiliensis is a common cause of actinomycotic mycetoma and the ulcerative, draining
lesions can be misdiagnosed as sporotrichosis.

116- The organism that causes white piedra:

A. Does not cause onychomycosis


B. Is known as Piedraia hortae
C. May also cause post-operative wound infections
D. Grows as small, compact, black-greenish velvety colonies
E. Can only be treated by cutting off affected hairs

►C

Trichosporon beigelii (new nomenclature: Trichosporon ovoides and Trichosporon inkin) causes
white piedra as well as other superficial infections, including post-operative wound infections,
paronychia, and onychomycosis. It grows as cream to yellow-colored pasty colonies when cultured.
Treatment is with topical amphotericin B lotion, benzoic acid, salicylic acid, or cutting off affected
hair. Black piedra is caused by Piedraia hortae, which grows as small, compact, black-greenish
velvety colonies when cultured, and can be treated only by cutting off affected hairs.

1122
117- A 56-year-old male with HIV presents with multiple umbilicated brown papules on the face.
Which of the following is the least likely cause of his lesions?

A. Cryptococcus
B. Histoplasmosis
C. Coccidiomycosis
D. Penicilliosis
E. Blastomycosis

►E

This patient has molluscum-like lesions on the face in the setting of immunosuppression. The
differential for such papules includes cryptococcus, histoplasmosis, coccidiomycosis, and
penicilliosis. Blastomycosis can cause sporotrichoid lesions, warty vegetations, papillomatous
proliferations, and osteomyelitis; it usually does not produce molluscum-type papules.

118- Which of the following statements about dimorphic fungi is INCORRECT?

A. Are in the mold form in the environment


B. Are in yeast/spherule form in tissue
C. Are in mold form at 25 degrees C
D. Are in yeast/spherule form at 37 degrees C
E. Are in mold form in tissue

►E

Yeasts and molds are not mutually exclusive. Dimorphism can occur - a dimorphic fungus will be in
a mold form in the environment at 25 degrees and in a yeast/spherule form at 37 degrees C.

119- A patient returns from a vacation in Brazil with keloidal-like nodules on the face and arms. The
patient denies any sick contacts or exposures and reports only lying on the beach and swimming
with dolphins. She likely has:

A. Actinomycosis

1123
B. Lobomycosis
C. Chromoblastomycosis
D. Mucormycosis
E. Sporotrichosis

►B

Lobomycosis, also called Keloidal blastomycosis, is caused by Loboa loboi. Lesions


characteristically appear keloidal with or without fistulas. A common location for the infection is on
the ears and has been associated with dolphins. Histologically, double contoured refractile spherules
with budding may be seen.

120- What organism produces round thick walled spiny macroconidia and pear shaped
microconidia?

A. Sporothrix schenckii
B. Blastomyces dermatitidis
C. Paracoccidioides brasiliensis
D. Cryptococcus neoformans
E. Histoplasma capsulatum

►E

At room temperature Histoplasma capsulatum produces round thick walled spiny macroconidia and
pear shaped microconidia.

121- Which of the following stains is NOT used to stain fungal elements?

A. Mucicarmine
B. Periodic Acid Schiff
C. Fontana-Masson
D. Von Kossa
E. Gormori methenamine silver

1124
►D

Mucicarmine stain is used to stain adenocarcinoma in addition to infectious organisms including


Cryptococcus and Rhinosporidium. The Cryptococcus capsule stains red. Periodic Acid Schiff
(PAS) is useful in identifying fungi, parasites, glycogen and the basement membrane. The wall
of the organism stains red. Fontana-Masson stains melanin black and therefore helps identify
dematiaceous (melanin-producing) fungi. Gormori methenamine silver stains fungi, parasites,
Donovan bodies and Rhinoscleroma black. Von Kossa stain is used to identify calcium by staining it
black.

122- Cryptococcus neoformans resides in:

A. The Mississippi Valley region


B. Chicken roosts
C. Pigeon droppings
D. Sandy soil
E. None of these answers are correct

►C

This organism is ubiquitous and is abundant in soil enriched with pigeon droppings.

123- All of the following parameters are used to distinguish dermatophytes except:

A. Nutritional requirements
B. Colony morphology
C. Sucrose hydrolysis
D. Hair fluorescence
E. Growth temperature

►C

1125
Sucrose hydrolysis is not a useful parameter to distinguish dermatophytes from one another. Growth
temperature, colony morphology, hair fluorescence and nutritional requirements can be used to
identify dermatophytes.

124- Septate hyphae with 90ǁ branching and thick walled barrel shaped arthroconidia alternating
with empty cells best describes the microscopic morphology of:

A. Histoplasma capsulatum
B. Sporothrix schenckii
C. Microsporum gypseum
D. Trichophyton tonsurans
E. Coccidioides immitis

►E

This is the only organism in this list that fits this description.

125- A mass of hyphae is known as a:

A. Sporangia
B. Conidia
C. Mycelium
D. Chlamydoconidia
E. Pseudohyphae

►C

A mass of hyphae (vegetative tube-like structures) is known as a mycelium. Sporangia, Conidia,


and Chlamydoconidia are structures formed through different types of asexual reproduction.
Pseudohyphae are seen in yeast.

126- Which pathogen is not inhibited by cycloheximide?

1126
A. Cryptococcus
B. Coccidioides immitis
C. Prototheca
D. Scopuloniopsos brevicaulis
E. Scytalidium species

►B

Cycloheximide is used in fungal culture media to reduce contaminants.

127- A 34-year-old male patient presents with a verrucous lesion of the nasal mucosa; the biopsy
showed spherules ranging in size from 200 to 325 um. The mature endospores have a rough
appearance. The diagnosis is:

A. Coccidioidomycosis
B. Protothecosis
C. Rhinosporidiosis
D. Pseudallescheriosis
E. Cryptococcosis

►C

The large size of the spherule and the rough endospores describe this organism along with the
clinical symptoms. The other organisms that produce spherules are Prototheca (8-20 um) and
Coccidioides immitis (10-80 um)

128- The most common cause of tinea capitis in the United States today is:

A. Trichophyton schoenleinii
B. T. tonsurans
C. T. mentagrophytes
D. Microsporum audouinii
E. M. canis

1127
►B

M. audouinii was previously the primary cause of T. capitis in the US.

129- Which of the following dermatophytes causes endothrix tinea capitis?

A. Microsporum gypseum
B. Microsporum audouinii
C. Trichophyton tonsurans
D. Microsporum canis
E. Microsporum ferrugineum

►C

Organisms that cause an ectothrix pattern of tinea capitis include M. canis, M. audouinii, M.
ferruginosum, M. distortium, M. gypseum and occasionally T. rubrum. T. tonsurans, T. violaceum,
T. soudanense, T. gourvilli, T. yaoundei, and occasionally T. rubrum cause an endothrix pattern. T.
schoenleinii causes favus in which hyphae and air spaces are seen in the hair shaft. A bluish-white
fluorescence pattern is seen with Wood‘s lamp. Clinically, patients have thick, yellow, cup-shaped
crusts (scutula); scarring and secondary infection may result. M. canis, M. audouinii, M.
ferruginosum, M. distortium display a yellow fluorescence on Wood‘s lamp examination.

130- This agent causes a superficial, asymptomatic infection usually on the palms of the hands. The
lesions are flat, nonscaly and appear as irregularly shaped brown macules:

A. Piedra hortae
B. Cladosporium carrionii
C. Aureobasidium pullulans
D. Phaeoannellomyces werneckii
E. Exophiala jeanselmei

►D

1128
These lesions often have a deeper pigment at the advancing border. The other organisms are
dematiaceous but are not the etiologic agent of tinea nigra.

131 -This organism is the most common cause of eumycotic mycetoma in the US:

A. Madurella mycetomatis
B. Nocardia asteriodes
C. Scedosporium apiospermum
D. Phialophora verrucosa
E. Wangiella dermatitis

►C

Although fairly uncommon, S. apiospermum is the most frequently recovered organism causing
eumycotic mycetoma in the US. This name represents the asexual stage of the organism which
grows as a filamentous mould, the sexual state, Pseudallescheria boydii, grows as a cleistothecia
(round sac containing ascospores).

132 -Which dermatophyte is most commonly responsible for tinea corporis gladiotorum?

A. T. mentag
B. T. tonsurans
C. T. schonlenleinii
D. T. rubrum
E. E. floccosum

►B

Tinea corporis gladiotorum is caused by Trychophyton tonsurans. Clinically, it appears as well-


defined, scaly plaques usually located on the arms, neck, and head.

1129
Coccidioidomycosis

Angioinvasive-fungus-fusarium

protothecosis

tinea-nigra

1130
onychomycosis

phaeohyphomycosis

eumycetoma

tinea-corporis

1131
tinea-capitis

candidiasis

tinea-versicolor

1132
histoplasmosis

blastomycosis

cryptococcosis

1133
https://dermatologyinreview.com

You might also like